실전수학올림피아드 1400제 해답

305
hKMO 2차시험 대비i 엠제곱(M 2 ) 실전 수학올림피아드 1400 중학생 중 / 고급 풀이편 고봉균 http://cafe.naver.com/xmo Rev: 200942KAIST 수학문제연구회

Upload: janghoon-lee

Post on 24-Oct-2015

845 views

Category:

Documents


44 download

DESCRIPTION

실전수학올림피아드 1400제 (해답)

TRANSCRIPT

Page 1: 실전수학올림피아드 1400제 해답

hKMO 2차시험 대비i

엠제곱(M2)

실전 수학올림피아드 1400

중학생 중/고급 풀이편

고봉 균

http://cafe.naver.com/xmo

Rev: 2009년 4월 2일

KAIST 수학문제연구회

Page 2: 실전수학올림피아드 1400제 해답

2

머 리 말

KAIST 수학문제연구회는 한국 수학올림피아드가 처음 탄생할 무렵인 1988년부터 지금

까지 다양한 영재교육 프로그램을 운영하거나 적극적으로 기여해왔습니다. 한국 수학올

림피아드 통신강좌와 계절학교 등의 교육프로그램, 전¢충남 지역 중학생 영재수학교실,

KAIST Cyber영재교육 등이 그 예들입니다.

이런 활동의 결과들을 수학 잡지 MathLetter를 통해 담아내어 왔으며, 수학경시 준비를

위한 교재를 따로 만들어 달라는 요청에 부응하여 2003년에 엠제곱(M2) 수학 올림피아드

셈본 중학생 초급/중급/고급을 펴내었습니다. 그후 셈이의 문제해결기법, 셈본 고등학생

초급, Baltic Way 팀 경시 회 등 출판활동이 꾸준히 이어졌고, 이번에는 셈본으로 기본학

습을 마쳤으나 보다 다양한 실전문제들로 KMO 2차시험을 준비하는 중학생들을 위해 또

한 권의 책을 펴내게 되었습니다.

이 책에는 수론/ 수/기하/조합의 네 분야별로,또 중급/고급의 난이도로 나뉘어,총 1400문

항에 이르는 매우 방 한 분량의 문제가 마련되어 있습니다. 각 분야의 고급은 KMO 중등

부 2차시험 정도의 난이도로 볼 수 있습니다. 각 분야의 중급은 1차시험 정도의 난이도로

볼 수는 있으나, 이 책의 실전 연습은 2차시험을 향해 맞추고 있으므로 문제의 유형은 단답

식인 1차시험과는 다를 것입니다. 많은 문제를 스스로 풀어보며 접하여본 학생일수록 안정

적인 실력을 갖추게 되고 또한 응용력도 성장하게 됩니다. 끈기있게 도전하여 힘든 여정을

겪으면서 자신의 능력에 차차 자신감을 갖게 될 것입니다. 그러나, 모든 문제를 다 풀고 넘

어가겠다고 하는 것은 시간이 아까운 면이 있으니, 어떤 수준의 문제가 70% 정도가 풀리고

있다면 그 수준은 그것으로 끝내고 다음 단계의 수준으로 넘어갈 것을 권합니다.

모든 문제에는 최 한 원래의 출전을 밝히려고 노력하였습니다. 원래의 출전 신에 그것

을 인용한 중간 출전을 적어둔 경우도 있습니다. 이 책에는 문제의 풀이와 해답이 실려있

지 않습니다. 책이 지나치게 두꺼워질 것을 우려하여 과감히 생략하였고, 신에 xMO 까

페(http://cafe.naver.com/xmo)의 `실전MO 풀이' 게시판에 문제의 풀이를 게시하고 있으

니, 거기서 찾아보시기 바랍니다. 기본 학습 단계가 아닌 실전 단계에서는, 몇몇 문제가 잘

풀리지 않더라도, 가능하면 풀이를 참고하지 않고 스스로 풀어내겠다고 마음 먹고 승부욕

을 불태우는 것이 꼭 필요하다고 생각합니다. 게시판에 올려진 풀이보다 더 멋진 여러분만

의 풀이를 찾아내어 까페에서 함께 논의하기를 기 해봅니다.

2008년 3월, 책임편집자 고 봉 균

Page 3: 실전수학올림피아드 1400제 해답

차례

1 수론 5

1.1 수론 중급문제 . . . . . . . . . . . . . . . . . . . . . . . . . . . . . . . . . . . 5

1.2 수론 고급문제 . . . . . . . . . . . . . . . . . . . . . . . . . . . . . . . . . . . 31

2 수 61

2.1 수 중급문제 . . . . . . . . . . . . . . . . . . . . . . . . . . . . . . . . . . . 61

2.2 수 고급문제 . . . . . . . . . . . . . . . . . . . . . . . . . . . . . . . . . . . 88

3 기하 123

3.1 기하 중급문제 . . . . . . . . . . . . . . . . . . . . . . . . . . . . . . . . . . . 123

3.2 기하 고급문제 . . . . . . . . . . . . . . . . . . . . . . . . . . . . . . . . . . . 155

4 조합 197

4.1 조합 중급문제 . . . . . . . . . . . . . . . . . . . . . . . . . . . . . . . . . . . 197

4.2 조합 고급문제 . . . . . . . . . . . . . . . . . . . . . . . . . . . . . . . . . . . 225

5 종합문제 265

5.1 종합 중급문제 . . . . . . . . . . . . . . . . . . . . . . . . . . . . . . . . . . . 265

5.2 종합 고급문제 . . . . . . . . . . . . . . . . . . . . . . . . . . . . . . . . . . . 283

Page 4: 실전수학올림피아드 1400제 해답

4 차례

| 문제의 풀이는 xMO 까페(http://cafe.naver.com/xmo)의 `실전MO 풀이' 게시판에 게시하고 있습니다. xMO

까페로 오시면 그밖에도 많은 학습자료를 찾을 수 있고, 여러 회원들과 온라인에서 만나 수학문제에 한 의견

교환 및 질의응답도 할 수 있습니다.

Page 5: 실전수학올림피아드 1400제 해답

제 1 장

수론

1.1 수론 중급문제

1. 정수계수의 이차방정식이 정수해를 가지면 판별식은 제곱수가 됨을 보여라.

증명 ax2 + bx + c = 0 에서 정수해 x는 근의 공식으로 x = ¡b§pD2a 를 만족하므로, 이것을 풀면

2ax = ¡b§pD, 2ax+ b = §pD, D = (2ax+ b)2 으로 제곱수가 된다. ¤

2. n진법의 수 100(n)과 21(n)은 n의 값과 관계없이 항상 서로소임을 보여라.

(KAIST Cyber영재교육 2003 겨울캠프 평가시험)

증명 다음과 같이 여러 방법으로 증명이 가능하다.

(1) 100(n) = n2, 21(n) = 2n+1 이다. 만일 두 수가 서로소가 아니라면 공통소인수 p가 존재한다. p는

n2의 소인수이므로 n의 소인수이기도 하다. 그럼 p는 (2n+ 1)¡ 2(n) = 1 의 소인수이기도 하다.

이것은 모순. 따라서 두 수는 서로소이다.

(2) 4(n2)¡ (2n¡ 1)(2n+ 1) = 1 이므로 n2과 2n+ 1은 항상 서로소이다.

(3) 유클리드 호제법의 원리에 의해 gcd(a; b) = gcd(a+ b; b) 이므로(gcd는 최 공약수를 나타냄),

gcd(n2; 2n+ 1) = gcd(n2; n2 + 2n+ 1)

= gcd(n2; (n+ 1)2)

= (gcd(n; n+ 1))2 = 1

이 된다.

¤

3. a와 b를 최 공약수(gcd)가 d인 두 자연수라 하자. 다음 수들

a; 2a; 3a; : : : ; (b¡ 1)a; ba

중에서 정확히 d개가 b로 나누어짐을 증명하여라. (헝가리 1901-3)

힌트 d j A, d j B이면 A j BC , A=d j (B=d)C, 또 1 = (A;B)이면 A j BC , A j C이다. }

4. 정수나 유한소수가 아닌 유리수는 순환소수임을 보여라.

Page 6: 실전수학올림피아드 1400제 해답

6 수론

증명 정수나 유한소수도 끝에 0이 계속 이어지는 것으로 생각하면 순환소수에 포함되는 것으로 생각

할 수 있다. 즉, 유리수는 항상 순환마디가 생긴다는 것을 보이면 된다. 임의의 유리수를qp라 하고, 나눗

셈을 통해 소수전개를 구하는 과정을 생각해보자. 소수점 이하의 자리를 계산할 때는 항상 p보다 작은수가 나머지로 남게 된다. p보다 작은 나머지는 음이 아닌 정수이므로 유한 개(p개)뿐이고, 따라서 나눗셈을 계속 하다 보면 같은 나머지가 반드시 생긴다. 즉, 소수점 이하 i번째와 j번째를 계산할 때 나머지가 같아지는 i < j 가 반드시 있다. 그럼 i번째 자리를 계산할 때부터와 j번째 자리를 계산할 때부터의과정이 완전히 일치하게 되므로, i번째부터의 소수전개와 j번째부터의 소수전개가 같다. 따라서, (길이가 j ¡ i 혹은 그 약수인) 순환마디가 생긴다. ¤

별증 유리수를 sr이라 할 때, s; 10s; 102s; : : : 중에 r로 나눈 나머지가 같은 것이 있다(비둘기집). 그럼

그 두 수는 r로 나누었을 때의 소수부가 같고, 하나가 다른 것의 10k배이므로 하나가 다른 수의 소수전개를 k자리만큼 올린 것이 되어 k자리의 순환마디가 생긴다. ¤

5. 다음 중 어떤 수도 완전제곱수가 될 수 없음을 보여라. (플란더즈 2002-J2)

11, 111, 1111, : : :22, 222, 2222, : : :33, 333, 3333, : : :

.

.....

88, 888, 8888, : : :99, 999, 9999, : : :

증명 완전제곱수는 1의 자리수로 0, 1, 4, 5, 6, 9만을 가질 수 있으므로 2, 3, 7, 8로 끝나는 것은 완전

제곱수가 될 수 없다. 또, 완전제곱수는 4로 나눈 나머지가 0 또는 1이므로 11, 22, 55, 66, 99로 끝나는것은 완전제곱수가 될 수 없다. 이제 4 ¢ ¢ ¢ 4 만 확인하면 되는데, 4 ¢ ¢ ¢ 4 = 4£ (1 ¢ ¢ ¢ 1) 이고 1 ¢ ¢ ¢ 1은 완전제곱수가 아니므로 이것도 역시 완전제곱수가 아니다. ¤

6. 자연수 A, B, C, D는 A5 = B4, C3 = D2, C = A+ 19 를 만족한다. D ¡ B 는 얼마인가?(AIME 1985-7)

풀이 We must have A = m^4, B = m^5 for some m, and C = n^2, D = n^3

for some n. Hence n^2 = m^4 + 19. The difference between n^2 and the

next smallest square (n-1)^2 is 2n-1, so n is at most 10. Checking

n = 1, 2, ... , 10, we find that only n = 10, m = 3 works.

¢ ¢ ¢ 답 757 }

7. 다음의 두 조건을 만족하는 정수 n이 존재함을 보여라.

(i) n을 이진법으로 전개하면 정확히 2004개의 0과 2004개의 1이 나타난다.

(ii) n은 2004의 배수이다. (영국 2004 2차-2)

증명 이 문제는 2004를 N번 곱하여 2진수가 1이 2004개, 0이 2004개인 수를 만들 수 있음을 밝히면

된다. 그것을 풀기 위하여 2004를 2진법으로 고치면 11111010100이다. 이 수에서 1은 7개, 0은 4개임을알 수 있다. 0은 마지막에 얼마든지 추가해줘도 되기 때문에(2를 계속 곱하는 작용) 1의 개수만 생각하겠다. 11111010100의 패턴이 여러 번 반복해서 이어붙여도 2004의 배수임을 알 수 있다(2004의 배수 2개를 더한 것이므로). 그럼 1을 7개씩 늘릴 수 있고, 이렇게 이 패턴을 285개 이어붙이면 모두 1995개의1이 생긴다(0은 현재 1140개). 한편 11111010100에 3을 곱하면 1011101111100이 되고, 여기에는 1이9개(0은 4개) 있다. 이 패턴을 앞에서 만든 것에 이어붙이면 1의 개수는 2004개로 맞춰지고, 여기에 2를계속 곱하면 0의 개수도 맞게 된다. ¤

8. 1987은 b진법으로 세 자리 수 xyz로 쓸 수 있다. x+ y + z = 1 + 9 + 8 + 7 일 때, x, y, z, b의 모든 가능한 값을 구하여라. (캐나다 1987-2)

풀이 1000(12) = 123 < 1987 < 452 = 100(45) 이므로 13 · b · 44 이다.

xb2 + yb+ z = 1987; x+ y + z = 25

에서 두 식을 빼면 x(b2 ¡ 1) + y(b ¡ 1) = 1962, 따라서 (b ¡ 1) j 1962 = 2 ¢ 32 ¢ 109. 여기서 b의 범

위를 만족하는 약수는 b ¡ 1 = 18 뿐이다. 즉, b = 19. 그리고, 1987 = 5 ¢ 192 + 9 ¢ 19 + 11. ¢ ¢ ¢ 답(x; y; z; b) = (5; 9; 11; 19) }

Page 7: 실전수학올림피아드 1400제 해답

1.1 수론 중급문제 7

9. 어떤 자연수를 받으면 그 수를 2진법으로 쓴 후 이것을 3진법으로 쓴 것으로 간주하여 계산하는 조작을A라 하자. 예를 들어 11은 2진법으로 1011이고 이것을 3진법으로 읽으면 31이 되므로 A(11) = 31 이다.A(A(A(n))) > 2006 이 되는 가장 작은 n은 얼마인가? (IT꿈나무 올림피아드 2006 1차)

풀이 x < y 이면 x를 이진법으로 쓴 것보다 y를 이진법으로 쓴 것이 더 크고, 그것을 삼진법으로 읽은

것도 y쪽이 더 크다. 즉, A(x) < A(y) 이므로, n이 증가함에 따라 A(n)도 증가함을 알 수 있다.

A(A(A(12))) = A(A(A(1100(2)))) = A(A(1100(3))) = A(A(36))

> A(A(32)) = A(A(100000(2))) = A(100000(3)) = A(243)

> A(128) = A(10000000(2)) = 10000000(3) = 2187

> 2006

A(A(A(11))) = A(A(A(1011(2)))) = A(A(1011(3))) = A(A(31))

= A(A(11111(2))) = A(11111(3)) = A(121)

< A(127) = A(1111111(2)) = 1111111(3) < 2 ¢ 729

< 2006

이므로 A(A(A(n))) > 2006 인 가장 작은 자연수 n은 12이다. }

10. 음이 아닌 어떠한 정수 n에 하여도, 22n+1은 네 개의 양의 정수의 제곱의 합으로 표현될 수 없음을 보여라. (한국 2003-J1)

증명 (1) n = 0: 2는 네 양의 정수의 제곱의 합이 될 수 없음(성립)

(2) n = k: 22k+1이 네 양의 정수의 제곱의 합이 될 수 없다고 가정(3) n = k + 1: 22(k+1)+1이 네 양의 정수의 제곱의 합이 된다면,

A = 22(k+1)+1 = a2 + b2 + c2 + d2 (a; b; c; d 2 N)

그런데, 8 j A이고, n 2 N에 해 n2 ´ 0; 1 or 4 (mod 8) 이므로, a2 + b2 + c2 + d2 = A가 되려면,a; b; c; d 모두 짝수이어야 한다.

) a = 2a0; b = 2b0; c = 2c0; d = 2d0

) A = 22(k+1)+1 = 4(a20 + b20 + c20 + d20)

) 22k+1 = a20 + b20 + c20 + d20 ) 가정에 모순

) 원 명제는 참임

(1){(3)에 의해 문제는 성립한다. ¤

11. 5보다 큰 임의의 소수 p에 하여, p2 ¡ 1 은 24로 나누어떨어짐을 증명하여라. (1997 교육청경시)

증명 먼저, p는 3의 배수가 아니므로, p = 3k§ 1꼴이고, 그럼 p+ 1 또는 p¡ 1 이 3의 배수이다. 따라

서, p2 ¡ 1 = (p+ 1)(p¡ 1) 은 3의 배수이다. 또, p는 홀수이므로 p+ 1과 p¡ 1 은 모두 짝수이다. 그리고, p+ 1 과 p¡ 1 은 차이가 2이므로 둘 중에 하나는 4의 배수이다. 따라서, p2 ¡ 1 = (p+ 1)(p¡ 1)은4 ¢ 2 = 8 의 배수이다. 즉, 3의 배수이고 8의 배수이므로 24의 배수이다. ¤

12. 10000을 연속되는 두 개 이상의 자연수의 합으로 나타낼 수 있는 경우의 수는 얼마인가? (단, 더하는 순서는 무시한다.) (한국 2004 1차-J2)

풀이

10000 = m+ (m+ 1) + ¢ ¢ ¢+ (m+ n) (m;n ¸ 1)으로 두자. 즉 양변에 2를 곱하면

(n+ 2m)(n+ 1) = 20000

Page 8: 실전수학올림피아드 1400제 해답

8 수론

이다. n+ 2m, n+ 1 은 홀짝이 다르고 각각 3 이상, 2 이상이다. 20000 = 2554 이므로 위와 같은 두 수의 곱으로 나타내는 방법은

(25; 54); (2551; 53); (2552; 52); (2553; 51)

의 네 가지뿐이다(순서쌍의 둘 중 큰 쪽이 n+ 2m). }

13. 다음과 같이 주어진 수열 fang, fbng이 있다

an = 3an¡1 + 2bn¡1; a0 = 1

bn = an¡1 + 2bn¡1; b0 = 2

이 두 수열에 공통적으로 나타나는 항을 모두 찾아라. (KAIST Cyber영재교육 2002 겨울캠프 평가시험)

풀이1 두 수열 모두 자연수로만 나온다는 것은 자명하다. 두 점화식을 직접 비교하면 an > bn (n ¸ 1),또 bn의 점화식에서 bn > an¡1 (n ¸ 1) 임을 알 수 있다. 즉,

(a0 < b0 < )b1 < a1 < b2 < a2 < b3 < a3 < ¢ ¢ ¢

와 같이 번갈아가며 커지는 수열이 된다. 따라서, 두 수열에 나타나는 항은 모두 다르다. }

풀이2 (an¡1; bn¡1) ´ (1; 2) (mod 3) 이면

an = 3an¡1 + 2bn¡1 ´ 0 + 2 ¢ 2 ´ 1 (mod 3)

bn = an¡1 + 2bn¡1 ´ 1 + 2 ¢ 2 ´ 2 (mod 3)

으로 (an; bn) ´ (1; 2) (mod 3) 도 된다. 즉, n = 0 일 때에도 참이므로, 수학적 귀납법으로 항상 이렇다. 따라서, 수열 (an)에 나타나는 항들과 수열 (bn)에 나타나는 항들은 3으로 나눈 나머지가 서로 다르므로 원래도 서로 다르다. }

풀이3 an + pbn = q(an¡1 + pbn¡1) 에 문제의 점화식의 an, bn을 입하여 p, q의 값을 구하면

(p; q) = (1; 4) or (¡2; 1)이 된다. 즉,

an + bn = 4(an¡1 + bn¡1) = ¢ ¢ ¢ = 4n(a0 + b0) = 3 ¢ 4n

an ¡ 2bn = an¡1 ¡ 2bn¡1 = ¢ ¢ ¢ = a0 ¡ 2b0 = ¡3

이 두 식을 연립하여an = 2 ¢ 4n ¡ 1; bn = 4

n + 1

와 같이 an, bn의 일반항을 구할 수 있다. 이런 두 수열에 공통항이 없음은 mod 4에 의한 비교나 크기비교, 혹은 ai = bj 라 두고 풀어보기 등 여러 가지 방법으로 알 수 있다. }

14. 가장 높은 자리의 숫자를 제거했을 때 원래 수의 1/35가 되는 정수는 존재하지 않음을 보여라.(캐나다 1970-4b)

풀이 가장 높은 10n자리수를 t라 하고 이것을 제외한 나머지 수를 a라 하자. 그럼

t ¢ 10n + a = 35a

t ¢ 10n = 34a

우변은 17의 배수인데, 1 · t · 9 이므로 t는 17을 소인수로 가질 수 없고 10n도 17을 소인수로 갖지 않는다. 이것은 모순이고, 따라서 문제의 성질을 만족하는 정수는 없다. }

Page 9: 실전수학올림피아드 1400제 해답

1.1 수론 중급문제 9

15. 분수n3 + 2n

n4 + 3n2 + 1은 모든 n에 하여 기약분수임을 보여라. (소련)

증명 gcd(n3 + 2n; n4 + 3n2 + 1) 즉 분자분모의 최 공약수가 1임을 보인다. a = bk + r (0 · r < b)

일 때 gcd(a; b) = gcd(b; r)임을 증명없이 이용한다(유클리드호제법).

gcd(n3 + 2n; n4 + 3n2 + 1) = gcd(n3 + 2n; n2 + 1) = gcd(n; n2 + 1) = gcd(n; 1) = 1

모든 n에 하여 주어진 분수의 분자 분모의 최 공약수는 1이므로 항상 기약분수이다. ¤

16. n2을 십진법으로 썼더니 0이 아닌 어떤 숫자가 자릿수에 m번 연속되어 나타나는 것으로 끝났다. m의 최값은 얼마인가? (아일랜드 1989-4)

풀이 0을 제외하고 제곱수의 일의 자리는 1, 4, 5, 6, 9만 될 수 있다. 이제 제곱수의 마지막 두 자

리가 11, 44, 55, 66, 99가 가능한지 살펴보자. 제곱수는 4로 나눈 나머지가 0 또는 1인 성질이 있으므로, 11, 55, 66, 99는 제곱수의 마지막 두 자리가 될 수 없다(이것을 마지막 두 자리로 갖는 수는4로 나눈 나머지가 각각 3, 3, 2, 3이 된다). 44는 122 = 144 에서 보듯이 가능하다. 444도 예를 들어4622 = 213444 에서 보듯이 가능하다. 4444는 불가능함을 보이자. 모든 정수는 8로 나눈 나머지에 따라8k + r (r = 0;§1;§2;§3; 4) 꼴로 분류할 수 있다. 그럼 (8k + r)2은 mod 16으로

(8k + r)2 = 64k2 + 16kr + r2 ´ r2 ´ 0; 1; 4; 9 (mod 16)

만이 가능하다. 그런데 마지막 네 자리가 4444로 끝나는 수는 10000n+ 4444 ´ 0 + 12 = 12 (mod 16)이다. 따라서, 제곱수가 될 수 없다. 이상으로부터, 0이 아닌 같은 수가 반복되는 제곱수의 꼬리의 길이m은 최 3임을 알 수 있다. ¢ ¢ ¢ 답 3 }

17. 자신을 제외한 양의 약수들의 곱이 자기 자신과 같은 수를 `멋진 수'라고 말하기로 하자. 가장 작은 10개의 멋진 수의 합을 구하여라. (AIME 1987-3)

풀이 p^n is not unless n = 3, pq is, p^2q is not, p3q is not, pqr is

not proper. So 6, 8, 10, 14, 15, 21, 22, 26, 27, 33.

¢ ¢ ¢ 답 182 }

18. 어떤 수도 다른 수로 나누어지지 않는데 각각의 제곱수는 다른 모든 수로 나누어지는 10개의 자연수가 존재하는가? (Towns 1998봄 JA1)

풀이 존재함. n = p1 ¢ ¢ ¢ p10 일 때, ni = pin 으로 두면 됨(i = 1; 2; : : : ; 10). }

19. 여섯 자리 수 abcabc는 24개의 (양의) 약수를 갖고 a, b, c는 소수이다. abc의 최 값을 구하여라.(1998 교육청경시)

풀이 abcabc = 1001abc = 7 ¢ 11 ¢ 13 ¢ abc 인데, 소인수가 3개 이상이므로 (양의) 약수의 개수는 3개 이

상의 (1보다 큰) 자연수의 곱이다. 그럼 24은 (1) 4 ¢ 2 ¢ 2 또는 (2) 2 ¢ 2 ¢ 2 ¢ 2 로 분해되어야 한다. (1)의경우는 더 이상의 소인수가 없고 p3qr꼴이어야 하므로 abc가 72, 112, 132 중 하나여야 하는데, 그럼 세자리 수가 될 수 없어 곤란하다. (2)의 경우는 abc가 소수라야 한다. a, b, c도 모두 소수임으로부터 가능한 수를 큰 수부터 차례로 써보면 777, 775, 773, : : : 이다. 이 중 777, 775는 소수가 아니고, 773은 소수이다. ¢ ¢ ¢ 답 773 }

20. 수열 fang을 a0 = a1 = 1, an+1 = an + an¡1 (n ¸ 1) 이라 하자. 이 때에 연속한 두 항이 서로소임을 증명하여라. (통신강좌 1991-3-4)

21. 임의의 정수 n > 1에 해, 3n + 1은 2n으로 나누어지지 않음을 증명하여라. (헝가리 1911-3)

증명 32 ´ 1 (mod 8) 이므로, n이 홀짝임에 따라 3n + 1 ´ 4; 2 (mod 8) 이 된다. 23으로 나누어지지

않으므로 n ¸ 3에 해서는 모두 되었고, n = 2일 때는 8로 나눈 나머지가 2이므로 22으로 나눈 나머지도 2가 되어 역시 나누어지지 않는다. ¤

22. 자신의 양의 약수의 개수의 제곱과 같은 자연수를 모두 구하여라. (셈본중등초급 도전문제 2.1.1)

Page 10: 실전수학올림피아드 1400제 해답

10 수론

풀이 우선 1은 문제의 조건을 만족하고, 이제부터는 소인수를 하나 이상 갖는 수만을 생각하자. 그런

자연수를 n이라 하고, n의 소인수분해를 n = pe11 ¢ ¢ ¢ perr 이라 하자. n이 제곱수이므로, e1; : : : ; er은 모

두 짝수이다. 그럼 n의 약수의 개수 (e1 + 1) ¢ ¢ ¢ (er + 1)은 홀수가 되고, 그 제곱수인 n도 홀수. 따라서,p1; : : : ; pr ¸ 3 이다.

pe11 ¢ ¢ ¢ perr = (e1 + 1)2 ¢ ¢ ¢ (er + 1)2 (¤)

pi ¸ 3 이므로 peii ¸ 3ei (등호는 pi = 3 일 때에만 성립) 이고, ei ¸ 2 (ei는 짝수이므로) 에서

3ei ¸ (ei + 1)2 (등호는 ei = 2 일 때에만 성립) 가 됨을 알 수 있다. 따라서,

peii ¸ (ei + 1)2 (등호는 pi = 3, ei = 2 일 때만 성립)

이다. 즉, (¤)의 등호가 성립하려면 이 식에서 등호가 아닌 경우가 있어서는 안 되고, 따라서, 만족하는

n은 32뿐이다. ¢ ¢ ¢ 답 1, 9 }

23. 분모가 짝수인 기약분수는 분모가 홀수인 기약분수들의 합으로 나타낼 수 없음을 보여라.

증명 홀수인 기약분수들의 합은 분모를 통분했을 때 여전히 분모가 홀수이다. 따라서, 분자를 계산하

고 나서 약분을 해도 여전히 분모는 홀수이다. 즉, 분모가 짝수인 기약분수는 될 수 없다. ¤

24. 임의의 정수 n에 해, n2 + 2n+ 12 는 121의 배수가 아님을 보여라. (캐나다 1971-6)

증명 121의 배수이면 11의 배수이기도 하다.

n2 + 2n+ 12 = (n+ 1)2 + 11

이 11의 배수이려면 (n+ 1)2이 11의 배수이고, 즉 n+ 1도 11의 배수. 그럼 (n+ 1)2은 121의 배수이므로 이 식은 121로 나누어 11이 남는다. 따라서, 121의 배수일 수 없다. ¤

25. k개의 연속한 자연수의 합이 311이 될 수 있을 때, k의 최 값은 얼마인가? (AIME 1987-11)

풀이 Take numbers as N+1, N+2, ... , N+k. So k(2N+k+1) = 2·3^11.

Since N >= 0, we have k < 2N+k+1, and hence k < (2·3^11)^1/2. Also k

divides 2·3^11. Largest such k is obviously 2·3^5.

¢ ¢ ¢ 답 486 }

26. 어떤 자연수의 제곱근이 유리수이면 이 유리수는 정수임을 증명하여라.

증명pn = q

p 라 하자(p와 q는 서로소). 이 때, p = 1 이 됨을 보이면 된다. p를 이항하고 양변을 제곱

하면 p2n = q2. 그럼 q2 j p2n 인데, p와 q는 서로소이므로 q2 j n 이어야 한다. n = q2r 이라 두고 입하면 p2r = 1. 따라서, p = r = 1 이어야 한다. ¤

27. 11p+ 9 가 완전제곱수가 되는 소수 p를 모두 구하여라. (2000 교육청경시-대전)

풀이 11p + 9 = n2 으로 놓으면 11p = n2 ¡ 9 = (n+ 3)(n¡ 3). 11과 p는 모두 소수이므로, 가능성

은 (1) 11 = n+ 3, p = n¡ 3, (2) 11 = n¡ 3, p = n+ 3, (3) 11p = n+ 3, 1 = n¡ 3 의 세 가지뿐이다. 각각 풀어보면 (1)은 n = 8, p = 5, (2)는 n = 14, p = 17, (3)은 n = 4, p = 7=11 이 되므로 가능한

p의 값은 ¢ ¢ ¢ 답 5, 17 }

28. 임의의 양의 정수 n에 해, 다음 수가 8의 배수임을 증명하여라. (헝가리 1912-2)

An = 5n + 2 ¢ 3n¡1 + 1

풀이 52 ´ 32 ´ 1 (mod 8) 이므로 An을 8로 나눈 나머지는 2를 주기로 반복하게 되며, 따라서

n = 1; 2일 때만 확인하면 된다. n = 1, 즉 홀수일 때, An ´ 5 + 2 + 1 ´ 0 으로 잘 되고, n = 2, 즉 짝수일 때, An ´ 1 + 2 ¢ 3 + 1 ´ 0 으로 역시 잘 된다. }

Page 11: 실전수학올림피아드 1400제 해답

1.1 수론 중급문제 11

29. 다음 방정식을 만족시키는 자연수 m, n이 무수히 많을 때, 정수 a의 값을 구하여라.

m2 ¡ n2 ¡ 6n¡ 14m+ n+ 2

= a (한국 2004 1차-J13)

풀이 분모를 이항하여 정리하면

m2 ¡ am¡ n2 ¡ (a+ 6)n¡ 2a¡ 14 = 0

()³m¡ a

2

´2 ¡ µn+ a+ 6

2

¶2= 5¡ a

A = 2(m¡ a2), B = 2(n+ 3 + a

2) 로 치환하면, 위 식은

(A+ B)(A¡ B) = 4(5¡ a)

가 된다. A, B, 그리고 우변이 모두 정수이므로, 우변이 0이 아니면 우변을 두 인수의 곱으로 나타내는경우를 따지는 방법으로 이 부정방정식을 풀 수 있고, 그럼 이 식을 만족하는 해가 유한 개 밖에 없게 된다. 따라서, 해가 무한히 많기 위해서는 a = 5 이다. (그리고 이 때 A = B 이면 되므로 이를 만족하는해는 실제로 무한히 많다.) }

30. 각 자릿수의 합이 원래의 수와 같은 자연수는 1, 2, 3, ..., 9 뿐임을 증명하여라.(셈본중등초급 도전문제 2.1.4a)

증명 그런 수 n이 k자리의 수라고 하자. n은 10k¡1 이상의 수이고, 그 자릿수의 합은 10k보다 작으므

로, k가 3 이상이면 n ¸ 10k¡1 > 10k > n 이 되어 해가 없다. k = 2, 즉 두 자리의 수는 그 자릿수의 합이 20보다 작으므로 10, ..., 19 만 살피면 되고, 이 중에는 해가 없다. 이제 k = 1 일 때만 남았는데, 한자리 수인 이들은 모두 해가 된다. ¤

별증 그런 자연수를 an ¢ ¢ ¢ a1a0 이라 하자. 그럼

10nan + ¢ ¢ ¢+ 10a1 + a0 = an + ¢ ¢ ¢+ a1 + a0

() (10n ¡ 1)an + ¢ ¢ ¢+ (10¡ 1)a1 = 0

좌변의 각 항이 0 이상이므로 이 식이 0이 되려면 좌변의 모든 항이 0이어야 한다. 따라서, an = ¢ ¢ ¢ =a1 = 0 이다. 즉, 한 자리 수들만 답이 될 수 있다. ¤

31. 10201은 3 이상의 몇 진법으로 읽어도 항상 합성수임을 증명하여라. (캐나다 1972-3a)

증명 다음과 같이 인수분해되므로 합성수이다.

10201(n) = n4 + 2n2 + 1 = (n2 + 1)2

¤

32. n = 1; 2; 3; : : : 에 해, 100 + n2 과 100 + (n+ 1)2 의 최 공약수를 f(n)으로 정의하자. f(n)의 최 값은 얼마인가? (AIME 1985-13)

풀이 Let gcd be d. d must divide difference 2n+1. Hence also 2n^2+n

and 2(n^2+100), hence n-200. Hence also (2n+1) - 2(n-200) = 401. So

certainly d ≤ 401. But taking n = 601 gives 401 divides 100+n2 and

2n+1 and hence d ≥ 401.

¢ ¢ ¢ 답 401 }

33. a, b, c는 서로 다른 세 자연수이고, abc+ ab+ bc+ ca+ a+ b+ c = 1000 이다. a+ b+ c 의 값을 구하여라. (아벨콘테스트 1993 예선)

Page 12: 실전수학올림피아드 1400제 해답

12 수론

풀이 양변에 1을 더하면 (a+ 1)(b+ 1)(c+ 1) = 1001 로 인수분해된다. 1001 = 7 ¢ 11 ¢ 13 이고 a+ 1,

b + 1, c + 1 은 모두 2 이상의 자연수이므로 fa + 1; b + 1; c + 1g = f7; 11; 13g 일 수밖에 없다. 즉,

(a+ 1) + (b+ 1) + (c+ 1) = 7 + 11 + 13 = 31 이고 a+ b+ c = 28 ¢ ¢ ¢ 답 }

34.1

a+1

b=

1

2004를 만족하는 양의 정수쌍 (a; b)는 모두 몇 개인가? (플란더즈 2004-J4)

풀이 a, b가 양수이므로 문제의 식에서 a; b > 2004 임을 쉽게 알 수 있다. 양변에 2004ab를 곱하여 정

리하면(a¡ 2004)(b¡ 2004) = 20042

따라서, (a ¡ 2004)(b ¡ 2004)는 20042 = 24 ¢ 32 ¢ 1672 을 두 자연수의 곱으로 나타내는 것과 같고, 그경우의 수는 양의 약수의 개수와 같으므로 (4 + 1)(2 + 1)(2 + 1) = 45개이다. }

35. 연속하는 두 자연수 n, n+ 1 의 합이 다른 자연수 m의 제곱과 같을 때, 다음을 증명하여라.

(1) n은 4의 배수이다.

(2) n, n+ 1, m을 세 변으로 하는 삼각형은 직각삼각형이다. (1991 도대회)

증명 2n + 1 = m2 임에서 m은 홀수이므로 m = 2r + 1 로 두면 2n + 1 = 4r2 + 4r + 1, 즉 n =

2r2+2r = 2r(r+1)이다. r(r+1)은 짝수이므로 (1)이 확인된다.또, n2+m2 = n2+(2n+1) = (n+1)2

이므로 피타고라스의 정리가 성립하여, (2)도 확인된다. ¤

36. p4 ¡ 5p2 + 9 가 소수가 되는 소수 p의 개수는 얼마인가? (한국 2004 1차-S2)

풀이 p > 5 인 경우 소수들은 5k § 1, 5k § 2 꼴이고, 그럼 언제나

p4 ¡ 5p2 + 9 = (p2 ¡ 1)(p2 ¡ 4) + 5 = (p+ 2)(p+ 1)(p¡ 1)(p¡ 2) + 5

로 5의 배수가 된다. 이것은 5보다 크므로 합성수이다. 따라서, p · 5 인 2, 3, 5만 생각하면 되고, 이 중준식을 소수가 되게 하는 것은 2, 5의 두 개뿐이다. }

주 페르마의 작은 정리를 안다면 5 - p 일 때 p4 ´ 1 (mod 5) 를 이용해도 되겠다.

37. m, n이 3으로도 5로도 나누어떨어지지 않을 때, m4 ¡ n4은 15로 나누어떨어진다는 것을 증명하여라.(셈본중등초급 도전문제 2.3.1 변형)

증명 15 = 3 ¢ 5 이므로 m4 ¡ n4이 3으로도 5로도 나누어떨어짐을 보이면 된다. 먼저 m4을 3으로 나

눈 나머지는 일정하게 1이 됨을 보이자.

m4 ¡ 1 = (m2 + 1)(m2 ¡ 1) = (m2 + 1)(m+ 1)(m¡ 1)

인데, m은 3의 배수가 아니므로 m + 1, m ¡ 1 둘 중에 하나가 3의 배수가 되고, 따라서 위의 식은 3의배수이다. 따라서, m4을 3으로 나눈 나머지는 항상 1이다.

또 m4을 5로 나눈 나머지도 일정하게 1이 됨을 보이자.

m4 ¡ 1 = (m2 + 1)(m2 ¡ 1) ´ (m2 ¡ 4)(m2 ¡ 1) (mod 5)

= (m+ 2)(m¡ 2)(m+ 1)(m¡ 1)

이므로 5 - m 일 때 언제나 m4 ¡ 1은 5의 배수가 된다.

따라서, m, n이 3의 배수가 아니고 5의 배수도 아니면 언제나 m4 ¡ n4 은 3의 배수이면서 5의 배수가되므로 문제가 성립한다. ¤

주 일반적으로, 자연수 n이 소수 p의 배수가 아닐 때

np¡1 ´ 1 (mod p)

가 된다. 이것은 페르마의 작은 정리라 하는 것이고 언젠가는 공부할 기회가 있을 것이다. 이것을 이용하면 위의 문제에서 필요한 m2 ´ 1 (mod 3), m4 ´ 1 (mod 5) 가 바로 얻어진다.

Page 13: 실전수학올림피아드 1400제 해답

1.1 수론 중급문제 13

38. n이 3의 배수가 아닐 때, n7 ¡ n 은 27 ¡ 2 의 배수임을 보여라. (1998 겨울학교 모의고사 중등부)

증명 27 ¡ 2 = 126 = 2 ¢ 32 ¢ 7 이므로 n7 ¡ n 이 항상 2의 배수, 9의 배수, 또 7의 배수가 됨을 보이면

된다.

n7 ¡ n = n(n6 ¡ 1) = n(n¡ 1)(n+ 1)(n2 ¡ n+ 1)(n2 + n+ 1)

로 인수분해된다.(1) n(n+ 1)은 연속한 정수의 곱이므로 2의 배수이다.(2) n(n¡1)(n+1)은 연속한 세 정수의 곱이므로 3의 배수이다. 또, n이 3의 배수가 아니므로 n = 3k§1로 둘 수 있고, 이 때 n2 § n+ 1 = (3k § 1)2 § (3k § 1) + 1 = 3(¢ ¢ ¢ ) + 1 + 1 + 1 이 3의 배수가 된다.

따라서, 9의 배수이다.(3) n = 7k 혹은 7k § 1 일 때 n(n ¡ 1)(n + 1)이 7의 배수가 되고, n = 7k § 2 일 때는 n2 § n + 1 =7(¢ ¢ ¢ ) + 4 + 2 + 1 이 7의 배수가 된다. n = 7k § 3 일 때는 n2 ¨ n+ 1 = 7(¢ ¢ ¢ ) + 9¡ 3 + 1 이 7의 배수가 된다. ¤

39. p와 p+ 2가 모두 3보다 큰 소수이면 6이 p+ 1의 약수임을 보여라. (캐나다 1973-3)

증명 p를 6으로 나눈 나머지에 따라 분류하자. p = 6k, 6k § 2, 6k + 3꼴일 때는 p가 3보다 큰 소수일

수 없다. p = 6k+ 1꼴일 때는 p+ 2 = 6k+ 3이 3보다 큰 소수일 수 없다. 따라서, p = 6k¡ 1꼴이고, 그럼 6 j p+ 1. ¤

별해 p와 p+ 2가 둘다 홀수이므로 p+ 1은 짝수. 연속한 세 정수에는 반드시 3의 배수가 하나 있어야

하는데 p와 p+ 2는 둘다 3의 배수가 아니므로 p+ 1이 3의 배수. 따라서, p+ 1은 6의 배수이다. ¤

40. n은 an ´ 1 (mod m) 을 만족하는 가장 작은 자연수이다. ak ´ 1 (mod m) 이면 n j k 임을 증명하여라.

증명 k를 n으로 나눗셈을 하여 k = qn+r (0 · r < n) 이라 하자. 그럼 ak = (an)qar ´ ar (mod m)

이므로 ar ´ 1 (mod m) 이다. 만일 r 6= 0 이면 r은 n보다 더 작은 자연수이므로 n의 최소성에 모순이다. 따라서, r = 0 이고, 즉 n j k 이다. ¤

41. a+ b+ c = 1998 와 0 < a < b · c < 2a 를 만족하는 정수 a, b, c 중에서 a, b, c의 최 공약수 g가 최가 되는 해를 하나 구하여라. 그리고, 이런 해가 유일한지 그렇지 않은지 확인하여라. (플란더즈 1998-1)

풀이 g = 111 (a; b; c) = (555; 666; 777), (444; 777; 777) 등으로 유일하지 않은듯. }

42. n = 1; 2; 3; : : : ; 1992 에 해, m!이 정확히 n개의 0으로 끝나는 자연수 m이 존재하지 않는 n은 모두 몇개인가? (AIME 1992-15)

풀이 There are always enough powers of 2, we are constrained by powers

of 5. We jump powers of 10, at multiples of 25. So 24! ends in 4 zeros

(one 5 each from 5, 10, 15, 20), but 25! ends in 6 zeros. Similarly,

multiples of 125 skip two, multiples of 625 skip three etc. So

consider 7980! It has 1596 multiples of 5, including 319 multiples of 25,

63 multiples of 125, 12 multiples of 625, and 2 multiples of 3125.

Hence total of 1596+319+63+12+2 = 1992. It has skipped 319+63+12+2 = 396.

¢ ¢ ¢ 답 }

43. 9997보다 큰 9997의 배수 중에서 각 자릿수가 모두 홀수인 가장 작은 정수를 구하여라. (IMTS R2-1)

풀이 구하는 수를 9997n이라 하자. 우선 일의 자리가 홀수라야 하므로 n은 홀수. 만일 3n · 10000 이

면9997n = 10000n¡ 3n = 10000(n¡ 1) + (10000¡ 3n)

에서 만의 자리는 n ¡ 1의 일의 자리이므로 짝수가 된다. 따라서, 3n > 10000, 즉 n > 3333 이고, n은홀수이므로 n ¸ 3335. n = 3335 일 때 9997n = 33339995 ¢ ¢ ¢ 답 }

44. 연속한 네 자연수의 곱은 어떤 정수의 제곱이 될 수 없음을 증명하여라. (헝가리 1926-2)

Page 14: 실전수학올림피아드 1400제 해답

14 수론

증명 연속한 네 자연수를 n¡ 1, n, n+ 1, n+ 2 로 두자. 이들의 곱 A는

A = (n¡ 1)n(n+ 1)(n+ 2)

= f(n¡ 1)(n+ 2)gfn(n+ 1)g

= fn2 + n¡ 2gfn2 + ng

= f(n2 + n¡ 1)¡ 1gf(n2 + n¡ 1) + 1g

= (n2 + n¡ 1)2 ¡ 1

이므로 어떤 제곱수보다 1이 작다. A도 제곱수라면 두 제곱수의 차가 1이 되어야 하는 상황이다. 그런데, 제곱수들은 0, 1, 4, 9, 16, 25, : : : 등으로 차가 1이 되는 것은 0과 1뿐이다. 즉, A = 0 이어야 한다.

그러나, A는 네 자연수의 곱이므로 자연수이고, 이것은 모순이다. 따라서, A는 제곱수가 될 수 없다. ¤

45. (m2 + n)(m+ n2) = (m+ n)3 을 만족하는 모든 정수해 (m;n)을 구하여라. (아일랜드 2003-1)

풀이 ( 원외국어고 2학년 최일규)

mn+m2n2 = 3m2n+ 3mn2

mn(mn¡ 3m¡ 3n+ 1) = 0

mn = 0 혹은 (m¡ 3)(n¡ 3) = 8

이 된다. 오른쪽 부정방정식에서 (m¡ 3; n¡ 3)은³m¡ 3n¡ 3

´=³81

´;³¡8¡1´;³18

´;³¡1¡8´;³42

´;³¡4¡2´;³24

´;³¡2¡4´

가 가능하고, 그럼 모든 정수해 (m;n)은 다음과 같다.

³mn

´=³k0

´;³0k

´;³114

´;³¡52

´;³ 411

´;³ 2¡5´;³75

´;³¡11

´;³57

´;³ 1¡1´

}

46. 수열 a1; a2; a3; : : : 은 a1 = 2, 그리고 n ¸ 2 일 때 an은 a1a2 ¢ ¢ ¢ an¡1 + 1 의 가장 큰 소인수로 정의된다. 이 수열에 5가 나타나지 않음을 보여라. (호주 1982-5)

증명 a1 = 2, a2 = 3 이므로 이후 2의 배수나 3의 배수는 나타나지 않는다. 그런데 5가 나타난다면,

가장 큰 소인수가 5인데 2나 3은 소인수가 아니므로 an = a1a2 ¢ ¢ ¢ an¡1 + 1 = 5m꼴이다. 1 넘기면 우변은 4의 배수가 되는데 좌변은 짝수가 오직 a2 = 2 한 번만 나타나므로 모순. 따라서 5는 나타나지 않는다. ¤

47. 1; 2; 3; : : : ; n 의 최소공배수를 Mn이라 하자. 예를 들어, M1 = 1, M2 = 2, M3 = 6, M4 = 12, M5 = 60,M6 = 60 등이다. Mn¡1 =Mn 이 되는 n은 어떤 수들인가? 자신의 주장을 증명하여라. (호주 1991-2)

풀이 Mn¡1 · Mn 임은 자명. 등호가 성립할 때는 n이 이미 Mn¡1의 약수일 때이다. n이 소수의 거

듭제곱꼴일 때에는 1; 2; : : : ; n ¡ 1 중에 어느 것도 n만큼 그 소수의 지수 이상을 포함하는 경우가 없으므로 n - Mn¡1 로 성립하지 않는다. n이 소수의 거듭제곱꼴이 아닐 때에는 n = ab꼴(a, b는 서로소이고 1보다 큼)로 나타낼 수 있고, a와 b는 1; 2; : : : ; n¡ 1 중에 있으므로 a; b j Mn¡1, 즉 n j Mn¡1 이다.

¢ ¢ ¢ 답 소수의 거듭제곱꼴이 아닌 수 }

48. f(x) = x2 + x라 하자. a, b가 양의 정수일 때 4f(a) = f(b) 가 성립할 수 없음을 증명하여라.(캐나다 1977-1)

Page 15: 실전수학올림피아드 1400제 해답

1.1 수론 중급문제 15

증명 4a(a+ 1) = b(b+ 1) 의 양의 정수해를 묻는 문제이다.

b · 2a =) b(b+ 1) · 2a(2a+ 1) < 4a(a+ 1)

b ¸ 2a+ 1 =) b(b+ 1) ¸ (2a+ 1)(2a+ 2) > 4a(a+ 1)

로 각각 성립하지 않는다. 따라서, 이 식을 만족하는 정수해는 없다. ¤

49. P는 5보다 큰 모든 소수들의 집합이다. 모든 p 2 P 에 해 p8 ¡ 1꼴의 정수들의 최 공약수를 구하여라.(플란더즈 1996-2)

풀이 p > 5 에 해선 p8 ¡ 1 6´ 0 (mod p) 이므로 공약수가 될 수 없고, 2, 3, 5의 지수는 p = 11 을

입하면 25 ¢ 3 ¢ 5 가 최 임이 확인됨. p8 ¡ 1 을 인수분해하여 항상 25, 3, 5의 배수가 된다는 것도 증명할 수 있음. }

50. k개의 연속한 정수들의 합으로 나타나는 수들의 집합을 Ak라 하자. A9 \ A10 \ A11 의 원소 중에 가장작은 자연수를 구하여라. (AIME 1993-6)

풀이 구하는 수를 a라 하면 a = (l¡4)+ ¢ ¢ ¢+(l+4) = (n¡4)+ ¢ ¢ ¢+(n+5) = (m¡5)+ ¢ ¢ ¢+(m+5)꼴이다. 정리하면, a = 9k = 10n+ 5 = 11m. 99의 배수 중에서 일의 자리가 5인 것이므로, 최소의 자연수는 ¢ ¢ ¢ 답 495 }

51. 분모가 2005 이하인 기약분수 중에서9

25와 가장 가까운 분수를 구하여라. (한국 2005-J1)

풀이 구하려는 분수를qp라 하면(단, gcd(p; q) = 1)¯

9

25¡ q

p

¯=j9p¡ 25qj25p

가 최소일 때를 구하면 된다.

(i) 9p¡ 25q = §1 일 때; 2q ´ §1 (mod 9) 이므로 q ´ §5 (mod 9). 따라서, 이 방정식의 일반해는

q = 9t+ 5

p = 25t+ 14or

q = 9t+ 4

p = 25t+ 11

이 각각의 경우에 해 25p가 최 일 때를 구해보면 p = 1989, q = 716. 즉, 이 때 최소이다.

(ii) j9p¡ 25qj ¸ 2 일 때; 이 때 25p · 25£ 2005 에서

j9p¡ 25qj25p

¸ 2

25p¸ 2

25£ 2005 >1

25£ 1989

이므로, (i)에서 구한 최소의 경우보다 항상 크다.

이상에서 구하는 분수는q

p=716

1989이다. }

52. 세 정수 a, b, c가 ab = c2 을 만족하면, 적당한 정수 t, u, v에 해 a = tu2, b = tv2 이 됨을 보여라.

증명 소인수분해로 생각하자. 자연수 n을 소인수분해했을 때, 지수가 홀수인 소인수들만 모아 곱한 것

을 n0이라 하자. 즉, n = (p2e11 ¢ ¢ ¢ p2err )(q2f1+11 ¢ ¢ ¢ q2fs+1s ) 이 n의 소인수분해라 하면(지수가 짝수인 것

들과 홀수인 것들을 분류하였음), n0 = q1 ¢ ¢ ¢ qs 이다. 그럼

n

n0= p2e11 ¢ ¢ ¢ perr q2f11 ¢ ¢ ¢ q2fss = (pe11 ¢ ¢ ¢ perr qf11 ¢ ¢ ¢ qfss )2

으로 제곱수이므로 n = n0m2꼴이 된다. 즉, 모든 자연수 n은 n = n0m2꼴로 나타낼 수 있다. 여기서n0은 어떤 소인수도 두 번 곱해져 있지 않으므로(소인수에 지수가 붙지 않으므로) n의 무승파트라 부르기로 하자.

Page 16: 실전수학올림피아드 1400제 해답

16 수론

ab = c2 을 만족하는 자연수 a와 b도 각각 a = a0u2, b = b0v2꼴이 되도록 하는 무승파트 a0, b0이 있다.

그럼 a0b0(uv)2 = c2 이 된다. 양변의 소인수분해를 생각하면 (uv)2과 c2으로부터 모든 소인수에 지수가짝수가 되는 것들만 생기므로, a0b0의 소인수분해에도 모든 소인수의 지수는 짝수가 되어야 한다. 그런데, a0과 b0은 무승파트이므로 각각 소인수는 한 번씩만 나타나고, 그럼 지수가 짝수가 되려면 a0의 소인수는 b0에도 있고, 또 b0의 소인수는 a0에도 있어야 한다. 즉, a0과 b0의 소인수는 완전히 일치해야 하고,

그럼 a0 = b0 이다. 즉, t = a0 = b0 이라 하면 a = tu2, b = tv2 꼴이 된다. ¤

53.x+ 1

y,y + 1

x가 둘다 자연수가 되도록 하는 두 자연수 x, y는 서로소임을 보여라.

(폴란드 1994/1995 1차-1 변형)

증명 d가 공약수라면 d j x j y + 1, d j y 이므로 d j 1. ¤

주 이런 조건을 만족하는 해도 사실 몇 개 없다. WLOG x · y 라 하면 y = x 또는 x+ 1뿐. 각 경우

를 풀어보면 (x; y) = (1; 1), (1; 2), (2; 3)뿐이다.

54. p(q ¡ r) = q + r 을 만족하는 모든 소수해 (p; q; r)을 찾아라. (호주 2003-1)

풀이1 p(q ¡ r) = q + r () q(p¡ 1) = r(p+ 1).

) q : r = p+ 1 : p¡ 1

만약 p = 2 이면 q : r = 3 : 1 이므로 q = 3r. 이것은 모순이므로, ) p = 2k + 1 꼴이다.

) q : r = 2k + 2 : 2k = k + 1 : k

q, r 모두 소수이므로 q = k+ 1, r = k, p = 2k+ 1. p, q, r이 모두 소수가 되게 하는 자연수 k = 2 뿐이다. ) p = 5, q = 3, r = 2. }

풀이2 (q ¡ r) j (q + r) =) (q ¡ r) j 2r .

) q ¡ r = 1; 2; r; or 2r

(1) q ¡ r = r 혹은 2r일 때: q = 2r 또는 3r. ) 모순.

(2) q ¡ r = 2일 때: 2p = 2r + 2. ) p = r + 1. ) 연속한 세 수가 소수 ) 모순.

(3) q ¡ r = 1일 때: q = r + 1. ) r = 2, q = 3 ) p = 5.

) 답은 (5; 3; 2)뿐. }

55. 3k+1꼴의 모든 자연수들을 모은 집합을 A라 하자. n 2 A 일 때 n이 1과 자기 자신 이외에는 A에 속하는약수를 갖지 않으면 n을 A-소수라고 한다. 그리고, A의 수들을 A-소수들의 곱으로 나타내는 것을 A-소인수분해라고 한다. 곱하는 순서를 무시할 때, A-소인수분해는 보통의 소인수분해처럼 항상 유일한가?

풀이 유일하지 않다. 100 = 10 ¢ 10 = 25 ¢ 4 이고, 4, 10, 25는 모두 A-소수들이다. }

56. m£ n 크기의 방에 1£ 1 크기의 타일을 겹치거나 빈 곳이 없이 말끔히 깔았더니(n > m), 방의 가장자리에 깔린 타일의 개수가 방에 깔린 전체 타일의 개수의 절반이었다. 가능한 방의 크기를 구하여라.

(플란더즈 예선 1995/1996 1차)

풀이 방의 가장자리를 제외한 곳에 깔린 타일의 개수도 방에 깔린 전체 타일 개수의 절반이다.(m¡ 2)(n¡ 2) = 1

2mn) mn¡ 4m¡ 4n+ 8 = 0

) (m¡ 4)(n¡ 4) = 8) (m¡ 4; n¡ 4) = (1; 8); (2; 4)) (m;n) = (5; 12); (6; 8):

¢ ¢ ¢ 답 5£ 12 or 6£ 8 }

57. n2이 n!을 나누지 않는 자연수 n을 모두 구하여라. (소련 1964-8)

풀이 모든 소수들과 4가 답이다. 우선 n이 소수이면 n!에는 소인수 n이 한 번밖에 들어있지 않으므로

n2은 n!을 나누지 않는다. n이 합성수일 때, n = ab, 1 < a; b < n 인 두 자연수 a, b가 있고, 그럼 n!에는 a; b; n이 들어있으므로 a 6= b 라면 n2 j n! 이다. n = ab 이면 a = b 일 수밖에 없는 수는 p2꼴의 수들이므로 이제 이것들만 확인하면 된다. p ¸ 3 이면 n!에 p; 2p; p2이 들어있으므로 n2 j n! 이고, p = 2 일때는 42은 4!을 나누지 않는다. }

Page 17: 실전수학올림피아드 1400제 해답

1.1 수론 중급문제 17

58. m, n은 m2 + 3m2n2 = 30n2 + 517 을 만족하는 정수이다. 3m2n2은 얼마인가? (AIME 1987-5)

풀이 (3x^2 + 1)(y^2 - 10) = 507 = 3·13^2. The first factor is not

divisible by 3, so the second must be. It cannot be 3, since 13 is

not square, or 507 (nor is 517). So it must be 39, hence 3x^2 = 12,

y^2 = 49.

¢ ¢ ¢ 답 588 }

59. 두 제곱수의 합이 또다른 두 제곱수의 합의 절반이 되는 경우가 무한히 많음을 보여라.

증명 (m+n2)2 + (m¡n

2)2 = (m2 + n2)=2 ¤

60. 두 유리수의 합과 곱이 정수이면 본래의 두 수는 모두 정수임을 증명하여라.(1999 KAIST 대전.충남 영재수학교실 2차평가)

증명 두 유리수를qp, sr이라고 하자. p와 q, 그리고 r과 s는 서로소이다. 두 수의 합

qr + ps

pr

에 s를 곱하고 두 수의 곱qs

pr

에 r을 곱하여 빼면,qrs+ ps2 ¡ qrs

pr=

ps2

pr=

s2

r

은 정수가 되므로 r과 s가 서로소임에서 r = 1일 수 밖에 없다. p도 마찬가지로 1이다. ¤

61. 6m + 2n + 2 가 완전제곱수가 되는 음이 아닌 정수쌍 (m;n)을 모두 구하여라. (호주 2004-5)

풀이 m;n ¸ 2 이면 4k + 2꼴이므로 완전제곱수가 될 수 없다. m = 0, n ¸ 2 일 때와 m ¸ 2, n = 0일 때는 4k + 3꼴이므로 역시 완전제곱수가 될 수 없다. 남은 경우는 다음과 같다.

(i) m;n · 1 일 때: (m;n) = (0; 0), (1; 0) 만 해가 된다.

(ii) m = 1, n ¸ 2 일 때: 준식은 2n + 8 이다. n ¸ 4 이면 16k + 8꼴이어서 완전제곱수가 아니고,n = 2; 3 일 때 검토하면 n = 3 일 때만 해가 된다.

(iii) m ¸ 2, n = 1 일 때: 준식은 6m + 4 이다. m = 2; 3 일 때는 직접 검토하면 해가 안 되고, m ¸ 4일 때는 6m + 4 = 4(2k + 1)2꼴이므로 정리하면 2m¡43m = k(k + 1). 여기서 k와 k + 1은 홀짝이 다르며 서로소이고 k + 1이 더 크므로 (k; k + 1) = (2m¡4; 3m) 이 될 수밖에 없는데, 2m¡4보다 3m이 훨씬 커서 차이가 1일 수 없으므로 해가 없다.

이로부터 해는 (m;n) = (0; 0), (1; 0), (1; 3) 뿐이다. }

62. abc는 세 자리의 수이다. acb+ bca+ bac+ cab+ cba = 3194 일 때, abc를 구하여라. (AIME 1986-10)

풀이 Put n = abc. Then N + n = 222(a+b+c). We have 222·14 = 3108 which

is too small. 222·15 - 136 = 3194, which does not work (sum of digits

of 136 is not 15), 222·16 - 358 = 3194, which does work,

222·17 - 580 = 3194, which does not work, 222·18 - 802 = 3194,

which does not work and 222·19 is too big.

¢ ¢ ¢ 답 358 }

63. abcdef는 a 6= 0, d 6= 0 인 6자리의 자연수이고 abcdef = (def)2 을 만족한다. 이런 수를 모두 구하여라.(플란더즈 1999-1)

풀이 def(def ¡ 1) = abc000 에서 def와 def ¡ 1 둘 중 하나는 일의 자리가 5, 즉 53의 배수라야 함.

n = 125; 126; 250; 251; 375; 376; 500; 501; 625; 626; 750; 751; 875; 876 이 후보가 되고, 그 중 실제로 가능한 것은 ¢ ¢ ¢ 답 376, 625 }

Page 18: 실전수학올림피아드 1400제 해답

18 수론

64. 각 자릿수의 합을 자기 자신과 곱하면 각 자릿수의 세제곱의 합과 같아지는 두 자리의 수를 모두 찾아라.(플란더즈 예선 1999/2000 2차)

풀이 두 자리 수를 10a+ b 라고 하면 (a+ b)(10a+ b) = a3 + b3 = (a+ b)(a2 ¡ ab+ b2). 즉,

10a+ b = a2 ¡ ab+ b2

이것은 정계수 이차방정식이므로 정수해를 가지려면 판별식이 완전제곱수라야 한다. a에 한 식으로생각했을 때

m2 = D = (b+ 10)2 ¡ 4(b2 ¡ b) (m ¸ 0)정리하면

m2 + 3(b¡ 4)2 = 148이것을 만족하는 해는 148 = 121 + 3 ¢ 9 = 100 + 3 ¢ 16 = 1 + 3 ¢ 49 뿐이고, 각 경우를 풀어주면 만족하는 답은 37, 48뿐이다. }

주 b = 0; 1; : : : ; 9 일 때로 나눠 각각 막노동으로 풀어주는 게 더 마음이 편할지도...;

65. 1, 2, 3, 4, 5, 6, 7, 8, 9의 숫자를 각각 2번씩 사용하여 합이 가장 작게 되는 서로 다른 소수들을 만들려고한다. 그 가장 작은 합은 얼마인가? (IMTS R4-1)

풀이 2를 제외한 짝수는 일의 자리에 올 수 없으므로 십의 자리에 있어야 한다. 2와 5는 적어도 하나씩은

십의 자리에 쓰여야 한다.따라서,이 합은 최소 2(40+60+80)+(2+20)+(5+50)+2(1+3+7+9) = 477

이상이다. 실제로 2, 5, 23, 41, 47, 59, 61, 67, 83, 89로 하면 딱 477이 가능하다. ¢ ¢ ¢ 답 477 }

66. 1부터 n까지의 자연수를 모두 곱한 것을 n!로 나타낸다. abc = a! + b! + c! 을 만족하는 세 자리의 자연수abc를 모두 구하여라.

풀이 7!, 8!, 9!은 네 자리 이상이므로 7, 8, 9는 나타날 수 없다. 6! = 720 이므로 6이 나타난다면 백의

자리가 7 이상이 되어 곤란하다. 따라서, 6도 나타나지 않는다. 만일 5도 나타나지 않는다면 최 의 수는4!+4!+4! = 72 로 세 자리의 수가 아니므로 곤란하다. 따라서, 5는 반드시 나타난다. 이 때, 최 의 수는5!+ 5!+ 5! = 360 이므로 백의 자리는 3 이하이다. 또 그 때, 최 의 수는 3!+ 5!+ 5! = 246 이므로 백의자리는 2 이하이다. 만일 5가 2개 나타난다면 백의 자리는 반드시 2이고, 그 때 2! + 5! + 5! = 242 6= 255이므로 성립하지 않는다. 따라서, 5는 1개이다. 그 때, 최 의 수는 2! + 4! + 5! = 146 이므로 백의 자리는 1이다. 1과 5를 제외한 나머지 한 자리를 d라 하자(1 · d · 4). 그럼 1! + d! + 5! = d! + 121 = 1d5

또는 15d 이다. 즉, d! + 21 = d5 또는 5d. d = 1; 2; 3; 4 일 때, d! + 21 = 22; 23; 27; 45 이므로 만족하는것은 d = 4 일 때뿐이고, abc = 145 ¢ ¢ ¢ 답 }

67. k는 1보다 큰 주어진 자연수이다. 다음 방정식의 모든 정수해를 찾아라.

yk = x2 + x (Towns 1981-J1)

풀이 x = ¡(n+ 1) 일 때는 x = n 일 때와 같으므로 x ¸ 0 일 때만 보자. x와 x+ 1은 서로소이므로

yk = x(x+ 1) 에서 x = ak, x+ 1 = bk꼴이 되어야 한다. k ¸ 2 이므로 k제곱수 0; 1; 2k; 3k; 4k; : : : 중에서 차이가 1이 나는 쌍은 0과 1뿐이다. 따라서, x = 0 일 때뿐이고, 짝이 되는 음수일 때도 고려하면x = ¡1 일 때도 같다. ¢ ¢ ¢ 답 (x; y) = (0; 0), (¡1; 0) }

68. 각 자리의 숫자의 합이 9 이상인 두 자연수들의 합으로 표시할 수 없는 최 의 자연수를 구하여라.(통신강좌 1992-5-4)

풀이 90 이상의 자연수들은 81; 82; : : : ; 90에 일의 자리가 9인 수를 더하여 모두 만들어낼 수 있으므로

문제의 조건을 만족하지 않는다. 89가 문제의 조건을 만족하며 실제로 해보면 된다. 따라서 89가 답이다. }

별해 9; 19; 29; : : : ; 89는 문제의 조건을 만족하는(자릿수의 합 9 이상인 두 수의 합으로 나타낼 수 없

는) 수이면서 또한 그 자신은 자릿수의 합이 9 이상인 수이고, 9로 나눈 나머지가 각각 0; 1; 2; : : : ; 8이다.9의 배수는 자릿수의 합이 9의 배수이므로 자릿수의 합이 9 이상이다. 따라서, 9; 19; 29; : : : ; 89에 적당한 9의 배수를 더하면 90 이상의 수는 모두 문제의 조건을 만족하지 않음을 알 수 있다. 즉, 89가 최 이다. }

Page 19: 실전수학올림피아드 1400제 해답

1.1 수론 중급문제 19

69. 자연수 n에 하여 1n + 2n + 3n + 4n 을 십진법으로 표현했을 때, 일의 자리에서부터 연속하여 나타나는 0의 개수를 f(n)이라 하자. f(n)의 최 값은 얼마인가? (KMC예선 2002전기 고1)

풀이 1n+2n +3n +4n를 나누는 가장 큰 2k에 해 생각해보자. 1n은 항상 1이고, 3n을 8로 나눈 나

머지는 1; 3; 1; 3; : : :으로 반복되므로 1n + 2n + 3n + 4n는 8의 배수가 될 수 없고 n이 3 이상 홀수일 때4의 배수이다. 즉 f(n) · 2이므로, f(n) = 2인 n을 찾아낸다.

n = 3일 때 1n + 2n + 3n + 4n = 1 + 8 + 27 + 64 = 100. ¢ ¢ ¢ 답 2 }

70. n+ 10 이 n3 + 100 의 약수가 되는 가장 큰 자연수 n을 구하여라. (AIME 1986-5)

풀이 n+10 divides n^3+10n^2, so 10n^2-100. Also 10n2+100n, so 100n+100.

Also 100n+1000, so 900. Hence largest n+10 is 900. Check

(900-10)^3 + 100 = -10^3 + 100 mod 900.

¢ ¢ ¢ 답 890 }

71. 세제곱했을 때 마지막 세 자리가 888로 끝나는 가장 작은 자연수를 구하여라. (AIME 1988-9)

풀이 Let number be N. Only single digit cube ending in 8 is 2^3. So

must be (10a + 2)^3 = 120a + 8 mod 100. Only single digit multiples

of 12 ending in 8 are 4·8 = 48 and 9·12 = 108, so a = 10b + 4 or

10b + 9, and N = 100b + 42 or 100b + 92. Hence N^3 = 3·42^2·100b + 42^3

or 3·42^2·100b + 92^3 mod 1000 = 200b + 88 or 200b + 688 mod 1000.

So smallest is 192.

¢ ¢ ¢ 답 192 }

72. ( )( )( )( )의 ( )( )%는 2000이다. 괄호 안을 1에서 9까지의 서로 다른 숫자로 채워넣어라.(플란더즈 예선 1999/2000 1차)

풀이 abcd ¢ ef = 200000 = 26 ¢ 55 이다. 여기서 abcd와 ef가 모두 10의 배수일 수 없으므로 둘중 어느

한 쪽이 26 또는 55을 다 갖는다. 어느 한 쪽이 100의 배수일 수 없다는 것과 각각 두 자리 수, 네 자리 수라는 것 등도 참고하면, 가능한 경우는 (26; 55)과 (25; 2 ¢ 55)뿐이다. 전개해보면 (64; 3125), (32; 6250)에서 전자만 가능하다. }

73. n!은 1부터 n까지의 자연수들의 곱을 나타낸다.예를 들어, 5! = 5¢4¢3¢2¢1 = 120이다. 1!+2!+3!+¢ ¢ ¢+n!

이 완전제곱수가 되는 n을 모두 구하여라. (플란더즈 예선 2000/2001 2차)

풀이 n ¸ 4 이면 mod 5로 말이 안 되어서... n = 1; 3뿐. }

74. M = ab 은 두 자리의 수이고 N = cde 는 세 자리의 수이다. 9MN = abcde 일 때, M , N을 구하여라.(AIME 1997-3)

풀이 Put f(M, N) for the number abcde = 1000M + N. We have 9·M·111

< f(M, 111). Also if N >= 123, then 9·M·N >= 9·M·123 = 1000M + 107M

>= 1000M + 1070 > f(M,N), so N must be 112, 113, ... , or 122.

Now bootstrap. If N >= 113, then 9·M·N >= 1000M + 17M >= 1000M + 170

> f(M,N). So the only possibility is N = 112. Hence 1000M + N = 1008 M,

so M = N/8 = 14.

¢ ¢ ¢ 답 14, 112 }

75. 어떤 정수 A가 있다. 이 때 f(x) =x¡ 12, g(x) =

x

2이다. 만약 A에 f를 두 번, g를 세 번을 적용했더

니(적용시킨 순서는 모른다) 1이 되었고 변환 과정에서 항상 정수였다. A가 될 수 있는 정수들 전부의 합은 얼마인가? (2007 IT꿈나무 올림피아드)

풀이 ¢ ¢ ¢ 답 444

이진법으로 생각하면 좋다. 과정을 거꾸로 생각해보면 f를 적용하여 1이 되는 것은 11(2), g를 적용하여

1이 되는 것은 10(2)이다. 즉, f를 거꾸로 적용하는 것은 이진법으로 쓴 마지막에 1을 추가하는 것이고,

g를 거꾸로 적용하는 것은 0을 추가하는 것이다.따라서, A가 될 수 있는 수는 1xxxxx꼴들이다(x들 중에1은 딱 2개). 이런 꼴은 10개 있고, 각 자릿수별로 합하면 10 ¢25+4 ¢(24+23+22+2+1) = 10 ¢32+4 ¢31이 된다. }

Page 20: 실전수학올림피아드 1400제 해답

20 수론

76. 자연수 n이 두 제곱수의 합으로 나타내어질 수 있다는 것과 2n이 두 제곱수의 합으로 나타내어질 수 있다는 것이 동치임을 증명하여라. (헝가리 1938-1)

증명 n = a2 + b2 이면2n = (a+ b)2 + (a¡ b)2

이다. 역으로 2n = c2 + d2 이면, c와 d는 홀짝이 같고,

n =

µc+ d

2

¶2+

µc¡ d

2

¶2임에서

c+ d

2와

c¡ d

2가 모두 정수임을 확인할 수 있다. ¤

77. 연속한 39개의 자연수는 그 중 자릿수의 합이 11로 나누어지는 수를 적어도 하나 포함함을 증명하여라.(소련 1961-3)

증명 끝자리 0 : : : 9 를 최소한 세 번 포함한다. 이 세 덩어리를 옮겨갈 때 자리올림이 발생하는데, 두

번 중 적어도 한 번은 십의 자리에서는 자리올림이 발생하지 않는다. 그때의 두 덩어리를

n 0; n 1; : : : ; n 9; n+1 0; n+1 1; : : : ; n+1 9

이라 하자. 앞쪽 덩어리가 11의 배수가 아니려면 10개의 수가 11로 나눈 나머지가 각각 차례 로 1, 2,

: : : ; 10이 되어야 한다. 뒷쪽 덩어리도 마찬가지이다. 즉, n 9와 n+1 0은 11로 나눈 나머지가 각각 10,

1이다(1). 그런데, 십의 자리에서는 자리올림이 발생하지 않으므로, n보다 n + 1이 자릿수의 합이 1 크

고, 따라서, n 9보다 n+1 0이 자릿수의 합이 8 작다. 이것은 (1)과 맞지 않으므로 모순. 따라서, 이 두

덩어리에 11의 배수가 반드시 있다. ¤

주 38개의 자연수 중에는 11의 배수가 없는 것이 있다(예: 999981, : : : ; 1000018).

78. 한 아이가 한 변의 길이가 1 cm인 42개의 정육면체꼴 블록들을 쌓아 커다란 직육면체꼴을 만들었다. 이직육면체의 밑면의 둘레의 길이가 18 cm라고 한다. 그럼 높이는 얼마인가?

(영재교육원 선발시험 기출문제)

풀이 가로, 세로, 높이를 각각 a, b, c (cm)라 하자. 그럼

abc = 42; a+ b = 9

이다. 그럼fa; bg = f1; 8g; f2; 7g; f3; 6g; f4; 5g

가 후보가 되는데, 이 중에서 ab가 42의 약수가 되는 것은 f2; 7g뿐이다. 그럼 높이는 3 cm ¢ ¢ ¢ 답 }

79. 양의 정수열 a1; a2; : : : ; a2006은 1; 2; : : : ; 2006을 적당히 재배열한 수열이다. 이 때, 정수 j(a21 ¡ 1)(a22 ¡2) ¢ ¢ ¢ (a22006 ¡ 2006)j 이 항상 3의 배수임을 보여라. (한국 2006-J1)

증명 (고양 대화중 2학년 한석원)

1; 2; 3; : : : ; 2006의 수들 중 3k+1꼴의 수는 669개, 3k꼴의 수는 668개이므로, a3k+1꼴의 항 669개가 모

두 3의 배수일 수 없다. 그 항을 am이라 하자. 그럼 a2m ´ 1 (mod 3), m ´ 1 (mod 3) 이므로 a2m ¡m

은 3의 배수이고, 따라서 준식도 3의 배수이다. ¤

80. 아래 시계의 시계바늘은 1부터 12까지의 숫자를 f5; 6; 7; 8g과 f9; 10; 11; 12; 1; 2; 3; 4g의 두 집합으로 나누고 있는데 각 집합의 원소의 합은 한 쪽이 다른 쪽의 두 배이다.

Page 21: 실전수학올림피아드 1400제 해답

1.1 수론 중급문제 21

이와 같이 시계는 시계바늘에 의해 두 집합으로 나뉘어진다. 두 부분으로 나뉘어진 두 집합의 원소의 합이 한 쪽이 다른 쪽의 배수가 되는 경우는 하루에 몇 분 동안 나타나는가? (단, 시계바늘이 정확히 숫자를가리키는 경우는 배제한다.) (IT꿈나무 올림피아드 2006 1차)

풀이 합이 작은 집합의 합 m은 1 + 2 + 3 + ¢ ¢ ¢ + 12 = 78 = 2 ¢ 3 ¢ 13 의 약수이어야 한다. 즉,

m = 1; 2; 3; 6; 13; 26; 39 만이 후보가 된다.

(1) m = 1 일 때: f1g(2) m = 2 일 때: f2g(3) m = 3 일 때: f3g, f1; 2g(4) m = 6 일 때: f6g, f1; 2; 3g(5) m = 13 일 때: f12; 1g, f6; 7g.(6) m = 26 일 때: f5; 6; 7; 8g, f11; 12; 1; 2g.(7) m = 39 일 때: f4; 5; 6; 7; 8; 9g [ f10; 11; 12; 1; 2; 3g. 칭적이므로 1가지로 봐야 한다.

따라서, 모두 11가지가 가능하다. 위의 각 경우마다 분침이 5분간 움직일 수 있고, 시침과 분침이 서로바뀌는 경우가 있으며, 하루에 12시간을 두 번 도니까, 각 경우마다 하루에 20분씩 해당된다. 따라서,11£ 20 = 220(분). }

81. 0 < a < b < c < d < 500, a+ d = b+ c, bc¡ ad = 93 을 만족하는 정수해 (a; b; c; d)는 모두 몇 개나 있는가? (AIME 1993-4)

풀이 Put b = a + n, c = a + n + m. Then d = a + 2n + m. So 93 = bc - ad

= n(n + m). So n = 1, n+m = 93 or n = 3, n+m = 31. There are 405 tuples

(a, a+1, a+93, a+94) and 465 tuples (a, a+3, a+31, a+34)

¢ ¢ ¢ 답 870 }

82. 한 원 위에 2000개의 점이 있는데, 다음과 같이 1, 2, 3, ..., 1993 의 수를 각 점에 부여한다: 먼저 아무 점에나 1을 부여하고, 거기서 시계방향으로 두 번째 점에 2를 부여한다. 거기서 다시 시계방향으로 3번째점에 3을 부여하고, 이런 식으로 계속한다. 어떤 점들에는 둘 이상의 수가 중복되어 부여되었을 수 있다.1993이 부여된 점에 함께 부여된 수들 중에서 가장 작은 것은 무엇인가? (AIME 1993-9)

풀이n(n+1)2

´ 1993¢19942

(mod 2000), 즉 (1993¡ n)(1994 + n) ´ 0 (mod 4000) 이다. 1993¡ n 과

1994 + n 둘 중에 하나만 짝수이고 또한 하나만 5의 배수이므로, so either 1993 ¡ n or 1994 + n is amultiple of 125 and the other is a multiple of 16(32로 수정해야 할듯?). If 1993 ¡ n is a multipleof 125, then n ´ 118 (mod 125) and 6 (mod 16), and smallest is obviously 118. If 1993 ¡ n is a

multiple of 16, then n ´ 9 (mod 16) and 6 (mod 125), so smallest is > 118. ¢ ¢ ¢ 답 118 }

83. 10진법의 자연수 n의 각 자릿수의 합을 s(n)으로 쓰기로 하자. 다음 각각의 경우에 해 n=s(n)의 최소값을 구하여라: (i) 10 · n · 99 (ii) 100 · n · 999 (iii) 1000 · n · 9999 (iv) 10000 · n · 99999

(IMTS R1-1)

풀이 (1) 10 · n · 99(solution)

모든 n에 하여 n > s(n) > 1이므로 n¡1s(n)¡1 > n

s(n)이다. 따라서 n=s(n)을 최소로 하는 (10이상

인) n의 1의 자리수는 9이다.

일반적으로 n의 10k의 자리가 1씩 바뀔 때 n값은 10k, s(n)은 1씩 바뀐다.n¡10ks(n)¡1 > n

s(n)이라고 가정하면 아래 식과 동치가 된다.

) n ¢ s(n)¡ 10ks(n) > n ¢ s(n)¡ n) n > 10ks(n):

즉, n ¡ 10ks(n)이 양수라면 10k 자리수가 클수록, 음수라면 10k 자리수가 작을수록 n=s(n)값이작아진다.n = 10a+ 9(1 · a · 9)라고 하면 n¡ 10s(n) = 10a+ 9¡ 10(a+ 9) < 0) n¡ 10s(n) < 0이므로 10 · n · 99인 경우 n = 19이다.

답: 19:

Page 22: 실전수학올림피아드 1400제 해답

22 수론

(2) 100 · n · 999(solution)n의 10의 자리를 구한다. n ¸ 100일 때 n = 100k+10a+9라고 표현하면(k ¸ 1; 1 · a · 9) 이 때s(n) = s(k) + a+ 9이다.n¡10s(n) = 100k+10a+9¡10(s(k)+a+9) = 100k+9¡10s(k)¡90 = 10(10k¡s(k))¡81 ¸10(10k ¡ k)¡ 81 ¸ 90¡ 81 > 0따라서 n ¸ 100일 때 n의 10의 자리가 클수록 n=s(n)이 작아진다.n = 100k + 99를 입한다.n¡ 100s(n) = 100k + 99¡ 100(s(k) + 18) = 100(k ¡ s(k))¡ 1701:100이상의 n = 100k + 99에서 k ¡ s(k) > 17이면 n의 100의 자리는 9, 아니면 0 또는 1인데,k ¸ 20일 때 k ¡ s(k) > 17임을 쉽게 알 수 있다.n이 세자리 수일 때 k < 10이므로 세자리수 n = 199:

답: 199:

(3) 1000 · n · 9999(solution)

앞선 결과에서 n = 1000a+ 100b+ 99(a; b는 한자리 자연수)의 꼴임을 알 수 있다. n의 1000 자리수에 따라 100 자리수가 달라진다.n¡ 1000s(n) = 1000s+ 100b+ 99¡ 1000s(a)¡ 1000b¡ 1800 = 1000¡ 900b¡ 1701 < 0:즉 a = 1, b = 0이다.

답: 1099:

(4) 10000 · n · 99999(solution)n = 10000a+1000b+100c+99라고 할 때, n¡ 100s(n) = 10000a+1000b+100c+99¡ 100(a+b+ c+ 18) > 10000a¡ 100(9 + 9 + 9 + 18) > 0이므로 c = 9이다.n ¡ 1000s(n) = 10000a + 1000b + 999 ¡ 1000(a + b + 27) = 9000a ¡ 18001 이므로 a ¸ 3일 때b = 9, 아니라면 b = 0이다.n¡ 10000s(n) = 10000a+ 1000b+ 999¡ 10000(a+ b+ 27) < 0이므로 a = 1: 답: 10999:

}

84. 6개의 숫자 1, 2, 3, 4, 5, 6을 어떤 순서로 써도 그 결과를 십진법으로 읽었을 때 11의 배수가 될 수 없음을 보여라.

증명 11의 배수판정법을 이용하자. 6개의 숫자를 배열한 6자리 수를 abcdef라 하면

j(a+ c+ e)¡ (b+ d+ f)j = j(a+ b+ c+ d+ e+ f)¡ 2(b+ d+ f)j = j21¡ 2(b+ d+ f)j = X

가 11의 배수가 되어야 한다. 1 + 2+ 3 · b+ d+ f · 4+ 5+ 6 이므로 12 · 2(b+ d+ f) · 30, 즉 X는0 · X · 9 범위의 수이다. 여기서 11의 배수인 것은 0뿐이고, 그럼 2(b + d + f) = 11 로 홀짝이 맞지않으므로 모순. 따라서, 11의 배수가 될 수 없다. ¤

85. 2006 ¢ 2005 ¢ 2004 ¢ ¢ ¢ ¢ ¢ 1004 를 계산하여 팔진법으로 쓰면 마지막에 0이 연속하여 여러 개가 나타난다.

이 연속한 0은 모두 몇 개인가? (IT꿈나무 올림피아드 2006 1차)

풀이 이 수가 8n의 배수일 때 n이 최 얼마인지를 묻는 문제이다. 소인수분해했을 때 2의 지수를 파

악하면 된다.2006 ¢ 2005 ¢ ¢ ¢ ¢ ¢ 11003 ¢ 1002 ¢ ¢ ¢ ¢ ¢ 1 =

2006

1003¢ 20041002

¢ ¢ ¢ ¢ ¢ 21¢ (2005 ¢ 2003 ¢ 2001 ¢ ¢ ¢ ¢ ¢ 1) = 21003 ¢ (홀수) 가

된다. 따라서, 소인수분해했을 때 2의 지수는 1003이고, 그럼 8n = 23n 임에서 3n · 1003. 즉, n · 334이다. ¢ ¢ ¢ 답 334개 }

86. 7진법으로 쓴 수를 10진법으로 읽으면 원래 수의 두 배가 되는 자연수를 더블이라 부르기로 하자. 예를 들어, 51은 7진법으로 쓰면 102이므로 더블이다. 가장 큰 더블을 구하여라. (AIME 2001 1차-8)

풀이 a_0+10a_1+10^2a_2 + ... = 2(a_0+7a_1+7^2a_2+...) or

a_0 + 4a_1 = 2a_2 + 314a_3 + 7599a_4 + ... . But a_i ≤ 6, so lhs ≤ 30,

so a_3, a_4, ... = 0 and a_0 + 4a_1 = 2a_2. Largest soln is evidently

a_2 = 6, a_1 = 3, a_0 = 0, giving 630_7 = 315_10 (note that a_i must be ≤ 6).

¢ ¢ ¢ 답 315 }

Page 23: 실전수학올림피아드 1400제 해답

1.1 수론 중급문제 23

87. 다음과 같은 성질을 갖는 두 개의 1999자리의 수 A, B가 존재하는가?: `B의 자릿수들은 A의 자릿수들의순서를 재배열한 것이고, A+B 의 자릿수들은 모두 9이다.' 또, 이런 성질을 갖는 한 쌍의 2000자리의 수는 존재하는가? (소련 1967-2a)

풀이 우선 이런 두 수의 덧셈에서 자리올림은 생기지 않음을 보이자. 일의 자리의 합이 19가 될 수는

없으므로 그냥 9이고, 따라서 일의 자리의 합에서는 자리올림이 생기지 않는다. 다음 십의 자리의 합을보면 역시 9가 되려면 자리올림이 생기지 않는다. 이런 식으로 계속 생각하면 모든 자리에서 자리올림이발생하지 않음을 알 수 있다.

자리올림이 없으므로 A의 모든 자릿수는 자신 안에 9에 한 보수를 갖는다. 즉, 예를 들어 A에 3이 n개있으면 6도 n개 있다. 그렇게 보수끼리 짝지어질 수 있으므로 A는 짝수 자리의 수라야 한다. 따라서,1999자리의 수에서는 만족하지 않는다.

A = 1818 ¢ ¢ ¢ 18

B = 8181 ¢ ¢ ¢ 81

2000자리의 수에는 위와 같은 예가 있다. }

88. 연속한 n개의 자연수가 모두, 서로 다른 소인수가 2개 이하인 수들이라고 한다. n의 최 값을 구하여라.(IT꿈나무 올림피아드 2006 1차)

풀이 30 = 2 ¢ 3 ¢ 5 임에 주목하면 30개 이상의 연속한 자연수에는 반드시 30의 배수가 있으므로 3개

이상의 소인수를 갖는 수가 있다. 또, 1, 2, 3, ..., 29는 모두 서로 다른 소인수가 2개 이하이므로 29가n의 최 값이다. }

89. 양의 정수쌍 (a; b)에 하여 유리수 Q(a; b) =a2b+ 2ab2 ¡ 5

ab+ 1을 생각하자. 유리수 Q(a; b)를 정수가 되

게 하는 모든 양의 정수쌍 (a; b)를 (a1; b1), (a2; b2), (a3; b3), : : : ; (an; bn)이라 할 때, a1 + a2 + ¢ ¢ ¢+ an의 값을 구하여라. (한국 2006 1차-J14)

풀이

Q(a; b) = a+ 2b¡ a+ 2b+ 5

ab+ 1

이므로 K = a+2b+5ab+1 이 정수가 되어야 한다. K의 분모 분자가 모두 양수이므로 a+ 2b+ 5 ¸ ab+ 1 이

고, 인수분해하면

(a¡ 2)(b¡ 1) · 6a ¸ 5, b ¸ 4 이면 좌변이 9 이상이 되어 성립하지 않으므로, a · 4 또는 b · 3 이다. 먼저 a · 4 일 때를 보면 다음과 같다.

a = 1 일 때: K = 2b+6b+1

= 2 + 4b+1 이 정수이므로 b = 1 또는 3.

a = 2 일 때: K = 2b+72b+1

= 1 + 62b+1 이므로 b = 1.

a = 3 일 때: K = 2b+83b+1

, 3K = 6b+243b+1

= 2 + 223b+1 이 정수이므로 b = 7.

a = 4 일 때: K = 2b+94b+1

, 2K = 4b+184b+1

= 1 + 174b+1 이므로 b = 4.

다음 b · 3 일 때를 보는데, a ¸ 5 인 경우만 생각하면 된다.

b = 1 일 때: K = a+7a+1

= 1 + 6a+1 이고 a ¸ 5 이므로 a = 5.

b = 2 일 때: K = a+92a+1

, 2K = 2a+182a+1

= 1 + 172a+1 이 정수이므로 a = 8.

b = 3 일 때: K = a+113a+1

, 3K = 3a+333a+1

= 1 + 323a+1 이고 a ¸ 5 이므로 a = 5.

이상을 종합하면 해는 (1,1), (1,3), (2,1), (3,7), (4,4), (5,1), (8,2), (5,3)이 전부이고, 구하는 답은

1 + 1 + 2 + 3 + 4 + 5 + 5 + 8 = 29 ¢ ¢ ¢ 답 }

90. 다섯 개의 자연수 1, 1, 1, 1, 2는 이 중 어느 세 개의 수를 택해도 그 수들의 합이 나머지 두 수의 합으로나누어떨어지는 성질이 있다. 이런 성질을 갖는 서로 다른 다섯 개의 자연수들이 있는가?

(뉴질랜드 2003-8)

Page 24: 실전수학올림피아드 1400제 해답

24 수론

풀이 서로 다른 다섯 개의 자연수를 0 < a < b < c < d < e 라 하자. d + e j a + b + c 인데,

a+ b+ c < d+ e+ e < 2(d+ e) 로 2배가 안 되므로 a+ b+ c = d+ e 이다. 비슷하게 c+ e j a+ b+ d

인데 a+ b+ d < c+ e+ e < 2(c+ e) 이므로 역시 a+ b+ d = c+ e 이다. 여기서 연립하면 c = d 가 되고 이것은 모순이다. 따라서, 그런 서로 다른 다섯 개의 자연수는 존재하지 않는다. }

주 `서로 다르다'는 조건을 없앴을 때 문제를 만족하는 해가 (a; a; a; a; 2a)꼴뿐임을 보일 수도 있다.

해를 0 < a · b · c · d · e 라 하자. 그럼 위의 풀이에서와 비슷하게

d+ e j a+ b+ c : a+ b+ c · d+ e+ e < 2(d+ e) ) a+ b+ c = d+ ec+ e j a+ b+ d : a+ b+ d · c+ e+ e < 2(c+ e) ) a+ b+ d = c+ eb+ e j a+ c+ d : a+ c+ d · b+ e+ e < 2(b+ e) ) a+ c+ d = b+ e

가 된다. 여기서 b = c = d 임을 알 수 있고, 또 e = a + b 이다. 즉, 다섯 개의 수는 (a; b; b; b; a + b) 이다. 다시 b+ b j a + b+ (a + b) 인데, 그럼 2b j 2a, b j a 이므로 a = b 가 된다. 따라서, 다섯 개의 수는(a; a; a; a; 2a)가 된다.

91. 적당한 정수 m, n에 하여 m(5m+ 4n)꼴로 쓸 수 있는 양의 정수들 중 2006 이하인 것은 모두 몇 개인가? (한국 2006 1차-J18)

풀이 (1) m이 홀수일 때: 5m2 + 4mn 은 4k + 1꼴이고, 또한 m = 1 을 입하면 4k + 1꼴은 모두

다 됨을 알 수 있다.

(2) m이 짝수이고 4의 배수일 때: m(5m + 4n)은 16의 배수이고, 또한 m = 4 를 입하면 16n + 80

이므로 16의 배수는 모두 다 된다.

(3) m이 짝수이지만 4의 배수는 아닐 때: m(5m+4n)은 4의 배수이지만 8의 배수는 아니므로 8k+4꼴이고, 또한 m = 2 를 입하면 8n+ 20 이므로 8k + 4꼴의 수는 다 된다.

총정리하면, 원하는 꼴로 쓸 수 있는 것은 4k + 1, 8k + 4, 16k꼴들이다. 이 중 2006 이하인 것은 각각502, 251, 125개다. 따라서 총 878개 ¢ ¢ ¢ 답 }

92. 1, 2, 3, ..., 1000 중에서 두 완전제곱수의 차가 될 수 없는 것은 모두 몇 개인가? (AIME 1997-1)

풀이 2n+1 = (n+1)^2 - n^2 and 4n = (n+1)^2 - (n-1)^2. Odd squares = 1 mod 4

and even squares = 0 mod 4, so we cannot get 2 mod 4 by the difference of

two squares. Thus we can get all except 2 = 1·4-2, 6=2·4-2, 10=3·4-2,

... , 998=250·4-2.

¢ ¢ ¢ 답 750 }

93.(2m+ n)!(m+ 2n)!

(m!)2(n!)2(m+ n)!이 정수임을 보여라.

증명 자연수 a; b가 있을 때(a+b)!a!b! 가 정수임을 보인다. 어떤 소수 p와 자연수 k가 pkjq!과 pk+1 6 jq!을

만족할 때 pkjjq라고 표기한다.

(a+ b)!을 소인수분해했을 때 어떤 소수 p에 해 pkjj(a+ b)!이고 pj jja!b!이라면 k ¸ j임을 증명한다.(a+ b)와 p가 주어져 있을 때 k = b a+b

pc+ b a+b

p2c+ b a+b

p3c+ ba+b

p4c+ ¢ ¢ ¢로 구한다.

bx+yzc ¸ bx

zc+ b y

zc이므로 다음 부등식이 성립한다.

k = b a+bpc+ b a+b

p2c+ ba+b

p3c+ b a+b

p4c+ ¢ ¢ ¢ ¸ (b a

pc+ b b

pc) + (b a

p2c+ b b

p2c)+ (b a

p3c+ b b

p3c) + ¢ ¢ ¢ = j

임의의 소수에 해 위의 부등식이 성립하므로(a+b)!a!b! 는 정수이다.

이를 이용하여 주어진 식을 증명한다.(2m+n)!(m+2n)!(m!)2(n!)2(m+n)!

=(2m+n)!(m!)2n!

¢ (m+2n)!n!(m+n)!

=(2m+n)!(2m)!n!

¢ (2m)!(m!)2

¢ (m+2n)!n!(m+n)!

(2m+n)!(2m)!n!

,(2m)!(m!)2

,(m+2n)!n!(m+n)! 각 항이 정수이므로 원식

(2m+n)!(m+2n)!(m!)2(n!)2(m+n)!이 정수이다. ¤

별증 준식은³2m+ n

m+ n

´³m+ n

m

´³2n+m

m+ n

´과 같다. ¤

Page 25: 실전수학올림피아드 1400제 해답

1.1 수론 중급문제 25

94. 선생님이 칠판에 50000보다 작은 자연수를 적었다. 첫 번째 학생은 이 수가 2의 배수라고 말했다. 두 번째학생은 이 수가 3의 배수라고 말했다. 이와 같이 차례로, 12번째 학생은 이 수가 13의 배수라고 말했다. 선생님은 잘 관찰을 하다가, 두 명의 학생을 제외하고 모두가 옳은 주장을 했고, 그 두 명은 연이어 말을 했다고 하였다. 칠판의 수는 무엇인가? (IMTS R12-1)

풀이 a = 2; 3; 4; 5; 6 각각에 해, a의 배수라고 말한 학생이 틀렸다면 2a의 배수라고 말한 학생도

틀렸고 그럼 그 두 학생이 연이어 있지 않아서 모순. 따라서, 이런 a에 한 언급은 모두 참이다. 그럼10 = 2 ¢ 5, 12 = 3 ¢ 4 의 배수임도 참이고, 그럼 남은 수 중에서 연이어 있는 것은 (7; 8)일 때와 (8; 9)일때뿐이다. (7; 8)일 때는 칠판의 수가 4 ¢ 9 ¢ 5 ¢ 11 ¢ 13 = 25740의 배수일 때 가능하고(그 중 50000 이하인 것은 25740뿐), (8; 9)일 때는 칠판의 수가 최소한 4 ¢ 3 ¢ 5 ¢ 7 ¢ 11 ¢ 13 = 60060 이상이어야 해서 모순.

¢ ¢ ¢ 답 25740 }

95. 10진법으로 1000 자리의 자연수가 있는데, 5가 아닌 자릿수가 많아야 한 자리뿐이다. 이 자연수가 완전제곱수가 될 수 없음을 보여라. (독일BW 1973 1차-1)

증명 (한석원) 이 자연수가 완전제곱수 k2이라 하자. k2 ´ 0; 1 (mod 4) 만 가능하므로, k2의 마지막

두 자리가 55일 수 없다. 따라서, 5가 아닌 자릿수는 십의 자리 또는 일의 자리이다. 그 자릿수를 a라 하고, k의 마지막 두 자리를 mn이라 하자. (1) a가 일의 자리에 있을 때: k2 ´ (10m+ n)2 ´ 20mn+ n2

(mod 100) 이므로 십의 자리가 홀수이려면 n2의 십이 자리가 홀수여야 하고 그럼 n = 4 또는 6이다.

둘 모두 제곱했을 때 일의 자리는 6이 되므로 a = 6. 이 때 k2 ´ 5 ¢ 999 + 6 ´ 6 (mod 9). 그럼 k2는3의 배수인데 9의 배수는 아니므로 모순. (2) a가 십의 자리에 있을 때: n = 5 인 상황이므로 k2 ´ 25(mod 100) 이고 a = 2. 그럼 k2 ´ 2 (mod 3) 이 되어 역시 모순. ¤

96. a, b, c, d는 ab = cd 를 만족하는 자연수들이다. 이 때, a2 + b2 + c2 + d2 은 절 소수가 될 수 없음을 보여라. (독일BW 1971 2차-1a)

증명1 gcd(a; c) = g 라 하고 a = gA, c = gC 라 하면 Ab = Cd 이고 d = hA꼴, 그럼 b = hC 가 된

다. 이것을 입하면 a2 + b2 + c2 + d2 = (A2 + C2)(g2 + h2) 으로 2 이상의 두 자연수의 곱이므로 합성수. ¤

증명2 a2b2 = c2d2 이므로 a2 + b2 + c2 + d2 =(a2 + c2)(a2 + d2)

a2인데, 이 수는 자연수이므로 완전

히 약분되어져야 해서, 적당한 두 자연수의 곱 a2 = mn 에 해a2 + c2

m,a2 + d2

n이 자연수가 된다.

그런데 m;n · a2 < a2 + c2; a2 + d2 이므로 이 두 수는 1보다 큰 자연수여서 이 곱은 합성수. ¤

97. 3의 거듭제곱은 십의 자리가 항상 짝수임을 보여라. (Towns 1987가을 J1)

증명 3의 거듭제곱은 일의 자리가 1, 3, 7, 9만 가능하다. 이들에 해 각각 3배를 하면 3 ¢ 1 = 03,

3 ¢ 3 = 09, 3 ¢ 7 = 21, 3 ¢ 9 = 27 로 일의 자리의 3배가 십의 자리의 홀짝에 영향을 미치지 않음을 알 수있다. 따라서, 십의 자리가 짝수로 출발했으면 아무리 3을 여러 번 곱해도 계속 짝수. ¤

98. a, b, c는 a = b+ c 인 임의의 세 자연수이다. a4 + b4 + c4 이 어떤 완전제곱수의 2배임을 보여라.(Towns 1988봄 JA1)

증명 (b+ c)4 + b4 + c4 = 2(b4 + 2b3c+ 3b2c2 + 2bc3 + c4) = 2(b2 + bc+ c2)2. ¤

99. 다음을 만족하는 두 개의 여섯 자리 수를 모두 찾아라: 두 수를 이어쓴 12자리 수는 원래 두 수의 곱의 배수이다. (Towns 1989봄 SA1)

풀이 문제의 의미 로 mn j 106m + n 라 하자. 그럼 m j 106m + n 이므로 m j n. n = mk 라 하

면 m, n은 둘다 여섯 자리 수이므로 1 · k < 10. 그리고, mk j 106 + k. 여기서 k j 106 이 되는데,1 · k < 10 에서 만족하는 것은 k = 1; 2; 5뿐. (1) k = 5 일 때: m j 200001, 즉 200001 = md 이고 d는홀수. m, n = 5m이 여섯 자리 수이려면 100000 · m < 200000 이어야 하는데 1 < d < 3 이 되어 만족하는 d 없음. (2) k = 2 일 때: 500001 = md 이고 100000 · m < 5000000. 1 < d < 5 이므로 만족하는홀수 d는 d = 3뿐. 그 때 m = 166667, n = 333334 실제로 가능. (3) k = 1 일 때: 1000001 = md 이고100000 · m < 1000000. 1 < d < 10 인데, 1000001은 한 자리의 소인수를 갖지 않음을 확인할 수 있어서 모순. ¢ ¢ ¢ 답 166667과 333334 }

Page 26: 실전수학올림피아드 1400제 해답

26 수론

100. p+ q = 198 을 만족하는 방정식 x2 + px+ q = 0 의 가능한 모든 정수해를 구하여라.(러시아 1989 4차-y8-2)

풀이 ®¯ ¡ (®+ ¯) = 198, 즉 (®¡ 1)(¯ ¡ 1) = 199 인수분해법. 199는 소수이므로 해는 2쌍뿐. }

101. 1 ¢ 2 ¢ 3 ¢ ¢ ¢ ¢ ¢ 2001 + 2002 ¢ 2003 ¢ ¢ ¢ ¢ ¢ 4002 가 4003으로 나누어떨어짐을 보여라. (러시아 1989 4차-y9-1)

증명 뒷항을 (¡2001)(¡2000) ¢ ¢ ¢ (¡1)로 보면 끝. ¤

102. xy = 19951996 인 자연수 x, y에 해, x+ y는 1996의 배수가 될 수 없음을 증명하여라.(몰도바 1996 최종-y8-5)

증명 x, y는 홀수이고 x + y는 4의 배수이므로 xy ´ 3 (mod 4) 라야 한다. 그런데 19951996 ´ 1

(mod 4) 임. ¤

103. 양의 유리수들이 다음과 같은 규칙으로 나열되어 각각 무한번 나타난다.

1

1;2

1;1

2;3

1;2

2;1

3;4

1;3

2;2

3;1

4;5

1;4

2;3

3;2

4;1

5; : : :

예를 들어 23는 9; 42; : : : 번째 위치에 나타난다.

(a) 12이 나타나는 처음 다섯 위치를 찾아라.

(b) 23가 n번째로 나타나는 위치를 식으로 표현하여라.

(c) p, q가 서로소이고 p < q 일 때,pq가 처음으로 나타나는 곳의 위치를 식으로 표현하여라.

(호주 1983-1)

풀이 분모와 분자의 합이 k + 1이 되는 항들을 k번째 그룹이라고 말하자. 약분을 하지 않을 상태의ab는 a+ b¡ 1번째 그룹의 b번째 항이다. k번째 그룹은 k개의 항을 가지므로 a

b는 맨앞에서 1+ 2+ ¢ ¢ ¢+(a+ b¡ 2) + b =

(a+b¡1)(a+b¡2)2

+ b번째 항이다.

(a) 12이 나타나는 처음 다섯 위치는 n = 1; 2; 3; 4; 5일 때의 n

2n이 나타나는 위치를 말하므로(3n¡1)(3n¡2)

2+

2n 으로 계산하여 각각 3, 14, 34, 63, 101번째이다.

(b) 23가 n번째로 나타나는 위치는 2n

3n이 나타나는 위치이므로(5n¡1)(5n¡2)

2+ 3n = 25n2¡9n+2

2.

(c)(p+q¡1)(p+q¡2)

2+ q }

104. n ¸ 2 인 자연수 n에 해, n의 양의 약수들의 곱(1과 n 포함)을 P (n)이라 하자. P (n) = n10 인 가장 작은 n을 구하여라. (IMTS R18-2)

풀이 n의 약수들을 순서 로 1 = d1 < ¢ ¢ ¢ < dk = n 라 하면 n = nd1

> ¢ ¢ ¢ > ndk= 1 들도 n의 약

수들을 역순으로 쓴 것. 따라서, P (n)2 = (d1 £ ¢ ¢ ¢ £ dk)(nd1£ ¢ ¢ ¢ £ n

dk) = nk. 즉, P (n) = nk=2 이

다(k는 n의 양의 약수의 개수). 즉, 문제는 약의 약수가 20개인 수 중에 가장 작은 것을 묻는 것이다. 약수가 20개인 수는 p19, p9q, p4q3, p4qr 등의 꼴이 전부이고, 각 꼴마다 가장 작은 수는 219, 29 ¢ 3, 2433,24 ¢ 3 ¢ 5 이며, 이들 중에서 가장 작은 것은 24 ¢ 3 ¢ 5 = 240 ¢ ¢ ¢ 답 }

105. 2n!을 소인수분해했을 때의 2의 지수를 구하여라. (헝가리 1939-2)

풀이 m!을 소인수분해했을 때 p의 지수는 bmpc+ bm

p2c + bm

p3c + ¢ ¢ ¢ 의 공식으로 구할 수 있으므로,

구하는 값은 2n¡1 + 2n¡2 + ¢ ¢ ¢+ 2 + 1 = 2n ¡ 1 ¢ ¢ ¢ 답 }

106. n1; n2; : : : ; n1998은 다음을 만족하는 자연수들이다.

n21 + n22 + ¢ ¢ ¢+ n21997 = n21998

적어도 두 개의 항이 짝수임을 보여라. (주니어발칸 1997-5)

Page 27: 실전수학올림피아드 1400제 해답

1.1 수론 중급문제 27

증명 홀수는 짝수개이므로, 모두 홀수일 수 없음만 확인하면 됨. 모두 홀수이면 mod 8로 5 ´ 1 이 되

어 모순. ¤

107. n이 자연수일 때, n(n+ 1)(2n+ 1)꼴의 모든 수를 나누는 가장 큰 자연수 d를 구하여라.(몰도바 1997 최종-y7-1)

풀이 2 j n(n+ 1) 이고 3 j 2 ¢ n(n+ 1)(n+ 2)¡ 3n(n+ 1) = n(n+ 1)(2n+ 1) =: f(n) 이므로 준식

은 항상 6의 배수. 또한, d j f(1) = 6 이므로 더 큰 수는 없음. ¢ ¢ ¢ 답 6 }

108. 좌표평면에서 점 (p2;p3 )을 중심으로 하는 원은 격자점(정수 좌표의 점)을 많아야 하나만 지남을 증명

하여라. (이탈리아 1986-4)

증명 둘 있다고 하고 무리수의 상등으로 둘이 같을 수밖에 없음을 증명. ¤

109. x가 정수이면 3x5 + 5x3 ¡ 8x 는 항상 120의 배수임을 보여라. (이탈리아 1987-1)

증명 8, 3, 5의 배수임을 보이면.. 뭐 쉽.. ¤

110. 9진법으로 나타낸 수열 1; 11; 111; 1111; : : : 의 모든 항이 삼각수임을,즉 적당한 정수 m에 해m(m+1)

2 꼴

의 수임을 증명하여라. (이탈리아 1988-4)

증명9n ¡ 18

=

3n + 1

2¢ 3

n ¡ 12

2. 혹은 9

m(m+ 1)

2+ 1 =

(3m+ 1)(3m+ 2)

2의 점화법으로도. ¤

111. 정수 계수의 두 삼차다항식이 공통의 무리수근을 가지면, 공통의 무리수근이 하나 더 있음을 증명하여라.(폴란드 1966 3차-1)

증명 켤레로 뻔한 거지만, 중등중급 수준에서 증명연습을 해볼 필요는 있... ¤

112. p는 2보다 큰 소수이다. 2p는 x > y 인 두 자연수 x, y에 해 1x+ 1

y 의 꼴로 나타내는 방법이 딱 한 가지

가 있음을 증명하여라. (헝가리 1931-1)

증명 2p= x+y

xy. 분모 넘기고 2 곱하고 인수분해법으로 정리하면 (2x ¡ p)(2y ¡ p) = p2. (x; y) =

(p+p2

2; 1+p2). ¤

113. m!n! = k! 은 1보다 큰 정수해를 무한히 많이 가짐을 증명하여라. (Towns 1982가을 J3)

증명 (m;n; k) = (n!¡ 1; n; n!) ¤

114. a, b, c가 1 이외의 공약수를 가지지 않는 자연수로, a, b, c를 세 변의 길이로 가지는 삼각형의 넓이도 정수일 때, 우리는 (a; b; c)를 원시적 헤론 세짝이라고 한다. a = 96 일 때, b와 c는 모두 홀수여야 함을 증명하여라. (IMTS R19-2)

증명 최 공약수가 1이므로 둘다 짝수일 순 없고, 하나만 홀수이면 헤론의 공식에서 S가 정수가 될 수

없음. ¤

115. 다음 세 등차수열에서 모두 등장하는 가장 작은 자연수를 찾아라. 그리고, 이 세 등차수열에서 모두 등장하는 자연수는 무한히 많음을 증명하여라. (IMTS R21-2)

5, 16, 27, 38, 49, 60, 71, : : :7, 20, 33, 46, 59, 72, 85, : : :8, 22, 36, 50, 64, 78, 92, : : :

증명 각각 11k ¡ 6, 13k ¡ 6, 14k ¡ 6꼴의 수열. 11 ¢ 13 ¢ 14k ¡ 6꼴은 늘 나타남. ¤

Page 28: 실전수학올림피아드 1400제 해답

28 수론

116. 각각의 수가 다른 두 수의 합을 나누는 세 자연수의 순서쌍 (a1; a2; a3)을 모두 구하여라(단, a1 ¸ a2 ¸ a3).

(호주 1993-3)

풀이 a2 + a3 = 2a1 일 때와 a2 + a3 = a1 일 때로 구분. 전자일 때는 (n; n; n)뿐. 후자일 땐 다시

2a3 = a2 일 때와 2a3 = 2a2 일 때로 구분하여 각각 (3n; 2n; n), (2n; n; n). }

117. 2n+ 1 과 3n+ 1 이 모두 완전제곱수인 자연수 n이 있다. 5n+ 3 이 소수일 수 있는가?(러시아 1993 최종-y9-1)

풀이 2n+ 1 = a2, 3n+ 1 = b2 이라 하면 5n+ 3 = 4a2 ¡ b2. 인수분해법 }

118. 자연수 x, y, z가 (x¡ y)(y¡ z)(z ¡ x) = x+ y+ z 를 만족한다. x+ y+ z 가 27로 나누어떨어짐을 보여라. (러시아 1993 최종-y9-5)

증명 x, y, z가 mod 3으로 모두 서로 다를 수 없음(우변만 3의 배수). 둘은 같고 하나만 다를 수도 없

음(좌변만 3의 배수). 즉, 모두 mod 3으로 같음. ¤

119. x2 + 3x+ 24 가 완전제곱수가 되는 정수 x를 모두 구하여라. (유고슬라비아 1980 고2-1)

풀이 x2 + 3x + 24 = k2 이라 두면 정수해를 가지므로 판별식이 제곱수, 즉 D = 9 ¡ 4(24 ¡ k2) =

4k2 ¡ 87 = m2. k;m ¸ 0 이고 (2k +m)(2k ¡m) = 87 = 3 ¢ 29 이므로 2k +m ¸ 2k ¡m; 0 임에서(2k+m; 2k¡m) = (87; 1), (29; 3)만 가능. 각각 (k;m) = (22; 43), (8; 13) 이 되고 그 때 이차방정식을

풀면 x = 12(¡3§m) 임에서 x = 20;¡23; 5;¡8 ¢ ¢ ¢ 답 }

120. 합과 곱이 모두 n이 되는 n개의 정수(양수든 음수든 상관없고, 서로 다를 필요도 없다)가 존재하면 n을좋은 자연수라고 부른다. 4k + 1꼴과 4l꼴(l ¸ 2)의 자연수는 모두 좋은 자연수임을 보여라.

(인도지역예선 1995-2)

증명 4k + 1 : 4k + 1; 1;¡1; 1;¡1; :::. 4l : 2l;§2; 1; :::; 1; 1;¡1; 1;¡1; :::. ¤

121. 모든 자연수 n에 해 an = 20 + n2 이고 dn = gcd(an; an+1) 이라 하자. dn이 취할 수 있는 모든 값들의 집합을 구하고, 각각의 값에 한 예를 하나씩 찾아라. (인도지역예선 1997-2)

풀이 d j 2n+1, d j n(2n+1)¡2(n2+20) = n¡40, d j (2n+1)¡2(n¡40) = 81. d = 1; 3; 9; 27; 81각각 예를 찾아주면. }

122. n은 자연수이고 p1; p2; p3; : : : ; pn은 n개의 5보다 큰 소수들이며 p21 + p22 + p23 + ¢ ¢ ¢ + p2n 은 6으로 나누

어떨어진다. n도 6으로 나누어떨어짐을 증명하여라. (인도지역예선 1998-2)

증명 다 6k § 1꼴이니까 뻔함. ¤

123. 방정식 y3 = x3 + 8x2 ¡ 6x+ 8 을 만족하는 자연수해 x, y를 구하여라. (인도지역예선 2000-2)

풀이 부등식적 부정방정식. (x+1)3 < y3 < (x+3)3 이므로 y3 = (x+2)3 이라야 함. ¢ ¢ ¢ 답 (9,11)}

124. 다음 방정식의 모든 정수해 (m;n)을 구하여라: (인도지역예선 1999-6)

(m¡ n)2 =4mn

m+ n¡ 1

풀이 분모 이항하고 정리하면 (m¡n)2(m+n) = (m+n)2. m+n = 0 일 때는 항상 성립. m+n 6= 0일 때는 m = n+ k 입해서 풀면... ¢ ¢ ¢ 답 (m;n) = (t;¡t), ( k2+k

2; k2¡k2). 단, k 6= §1. }

125. 한 정육면체를 99개의 작은 정육면체로 분할했는데, 이 99개의 정육면체들은 하나만 빼고 한 변의 길이가1인 정육면체들이라고 한다. 원래 정육면체의 부피를 구하여라. (몰도바 1997 최종-y11-1)

Page 29: 실전수학올림피아드 1400제 해답

1.1 수론 중급문제 29

풀이 m3 ¡ n3 = 98. m과 n은 홀짝이 같아야한다. m = n+ 2k 라 두고 입하여 정리하면 k(4k2 +6nk + 3n2) = 49. k = 1, n = 3, m = 5. }

126. 갑과 을은 13환(통화 단위)짜리 은행수표 여러 장으로 봉급을 받았다. 한 식당에서, 갑은 빵 9조각, 주스10잔, 소시지 7개를 주문했고, 을은 빵 5조각, 주스 7잔, 소시지 한 개를 주문했다. 빵, 주스, 소시지의 낱개 가격은 각각 정수 환이다. 갑이 거스름돈 없이 몇 장의 은행수표로 지불할 수 있으면 을도 그렇게 할수 있음을 보여라. (몰도바 1998 최종-y7/8-3)

증명 둘다 17로 나누어떨어진다는 옛날 헝가리 문제랑 비슷한... 13 j 2(9x+10y+7z)¡ (5x+7y+ z)¤

127. 1보다 큰 자연수 a에 해, a보다 작은 모든 자연수로 a를 나누었을 때의 나머지들을 모은 집합을 M이라하자. M의 (서로 다른) 모든 원소들의 합이 a일 때, a를 구하여라. (몰도바 1997 최종-y9-2)

풀이 a ¸ 11 이면 S(M) ¸ 1 + 2 + ¢ ¢ ¢+ b a¡12c > a 임을 금방 알 수 있음. a · 10 만 각각 검사해보

면... a = 10 만 가능함(집합의 원소의 합은 같은 수는 한 번만 더하게 됨을 유의할 것). ¢ ¢ ¢ 답 10 }

128. 소수 0:a1a2a3 ¢ ¢ ¢이 있는데, a1과 a2는 임의의 숫자이고 그 다음 자릿수들은 바로 앞의 두 자릿수의 합을 10으로 나눈 나머지라고 하자. 이 소수가 순환소수임을 보여라. (몰도바 1997 최종-y9-6)

증명 (an; an+1)의 패턴이 유한 가지뿐이므로. 유한-무한의 비둘기집. 순환잉여수열 ¤

129. 10과 1029 사이의 수들 중, 십진법으로 나타내었을 때 각 자릿수가 1만으로 이루어지는 소수는 많아야 9개임을 보여라. (호주 1995-1)

증명 1의 개수가 소수개라야 하고, 그런게 2, 3, 5, 7, 11, 13, 17, 19, 23, 29로 10개인데, 111은 제외할

수 있음 ¤

130. 각 자연수 n에 해, n! + 1 과 (n+ 1)!의 최 공약수 f(n)의 공식을 구하여라. (아일랜드 1996-1)

풀이 유클리드 호제법으로 g j n+ 1. n+ 1 이 prime이면 n+ 1 이 최 공약수. 그렇지 않으면 1. }

131. 19971997의 각 자릿수의 합의 각 자릿수의 합의 각 자릿수의 합을 구하여라. (몰도바 1997 최종-y10-2)

풀이 자릿수의 합은 mod 9로 불변. a ! b ! c ! d 가 되었다면, d ´ ¡1 (mod 9). a < 1010000,

b < 90000, c < 45, d · 12. ¢ ¢ ¢ 답 8 }

132. 1이 아닌 서로소인 두 자연수의 합으로 나타낼 수 있는 모든 자연수를 찾아라. (Towns 1983가을 J2)

풀이 5 이상의 홀수는 2k + 1 = k + (k + 1). 8 이상의 짝수는 4k = (2k ¡ 1) + (2k + 1) 과 4k + 2 =

(2k ¡ 1) + (2k + 3). (by 김규완) 1, 2, 3, 4, 6만 불가능. }

133. a1; a2; a3; : : : 는 자연수들의 단조 증가 수열이다. 임의의 k에 해 aak = 3k 가 항상 성립한다고 할 때,

a100을 구하여라. (Towns 1983가을 J4)

풀이 a81 = aa54 = 162 이고 a162 = aa81 = 243. 162 ¡ 81 = 243 ¡ 162 이므로 a81부터 a162까지

162...243의 연속한 자연수로 가득. a100 = 181 ¢ ¢ ¢ 답 }

134. 자연수 n의 (양의) 약수들(1과 n을 모두 포함한)의 합이 2n일 때 n을 완전수라 한다. n ¡ 1 과 n + 1 이모두 소수인 완전수 n을 모두 구하여라. (주니어발칸 2006-3)

풀이 쌍둥이 소수는 (3,5)를 제외하곤 6k § 1꼴이고, 4는 완전수가 아니므로 n = 6k꼴. k ¸ 2 이면

¾(n) ¸ 1 + k + 2k + 3k + 6k = 2n+ 1 로 불가능. n = 6뿐. }

135. 선생님이 사탕이 많이 가져와서 13명의 남학생과 10명의 여학생들에게 나눠주려고 한다. 남학생들끼리같은 개수의 사탕을 받고 여학생들끼리도 같은 개수의 사탕을 받도록 나누어주길 원한다(각자 적어도 하나씩은 받는다). 이렇게 할 수 있는 방법이 딱 한 가지뿐이라면, 선생님이 갖고 있는 사탕은 최 몇 개인가? (몰도바 1997 최종-y10-6)

Page 30: 실전수학올림피아드 1400제 해답

30 수론

풀이 260개. 그보다 많으면 방법이 두 가지 이상이고, 260개일 때 방법이 유일함을 말하면 됨. }

136. 무게가 각각 1; 2; : : : ; 1998인 1998개의 추가 있다. 이 추들을 총 무게가 각각 같은 n개의 집합으로 분할하려고 한다. n = 2; 3; 4; 5 중에서 가능한 것을 모두 찾아라. (몰도바 1998 최종-y9-6)

풀이 1에서 1998까지의 합은 1999 ¢ 999로 2, 4, 5의 배수가 아니므로 n = 2; 4; 5 는 불가능하고, n = 3

일 때는 합이 1999인 쌍이 999개 있으므로 이것을 333쌍씩 분할하면 됨. }

137. n은 자연수이다. A는 숫자 4로만 이루어진 2n자리의 수이고, B는 숫자 8로만 이루어진 n자리의 수이다.A+ 2B + 4 는 완전제곱수임을 증명하여라. (주니어발칸 2003-1)

증명 4 ¢ 102n¡19

+ 16 ¢ 10n¡19

+ 4 = ( 2¢10n+43

)2 ¤

138. 다음을 만족하는 자연수 x, y를 모두 구하여라. (주니어발칸 2005-1)

9(x2 + y2 + 1) + 2(3xy + 2) = 2005

풀이 판별식이 제곱수임을 이용. fx; yg = f7; 11g. }

139. a, b, c는 자연수로 a+ c, b+ c가 연속한 두 자연수의 제곱수가 된다. 그러면 ab+ c 와 ab+ a+ b+ c 도연속한 두 자연수의 제곱수가 됨을 보여라. (유고슬라비아 1981 고1-1)

증명 a+ c = n2, b+ c = (n+ 1)2 이라 하면 b = a+ 2n+ 1, c = n2 ¡ a 를 입하여 그냥 정리해보

면 ab+ c = (a+ n)2, ab+ a+ b+ c = (a+ n+ 1)2. ¤

140. 다음 식의 모든 해를 구하여라.2n + 7 = x2

단, n과 x는 둘다 정수이다. (Towns 1984가을 JO3)

풀이 n · ¡1 이면 정수가 아니라서, n ¸ 2 이면 mod 4로 3이라서 곤란. n = 0; 1 일 때만 체크하면

됨. ¢ ¢ ¢ 답 n = 1, x = §3 }

141. 각각의 n ¸ 2 에 해 n번째 항이 n¡ 1번째 항의 수와 그 수의 각 자릿수를 모두 합한 것으로 결정되는수열이 있다. 이 수열의 첫항이 1일 때, 이 수열에 123456이라는 항이 나타나는가? (Towns 1985봄 JO5)

풀이 3으로 나눈 나머지가 계속 2배씩 되므로, 3의 배수는 나타날 수 없음. }

142. n은 3으로 나누었을 때 2가 남는 5 이상의 자연수이다. n의 제곱이 어떤 소수와 어떤 완전제곱수의 합이될 수 없음을 증명하여라. (몰도바 2000 최종-y7-6)

증명 p = n2 ¡m2 = (n+m)(n¡m). n¡m = 1 이라야 하는데 그럼 3 j n+m. ¤

143. a2000 ¡ 1 이 10의 배수가 되는 자연수 a를 모두 구하여라. (몰도바 2000 최종-y8-1)

풀이 a는 일단 10과 서로소, 즉 5의 배수가 아닌 홀수. 2 j ak ¡ 1 은 자명하고, 5 j a4k ¡ 1 이므로 이

런 a는 항상 됨. }

144. m;n ¸ 2 은 정수이고 m+ n¡ 1 이 m2 + n2 ¡ 1 의 약수라고 한다. m+ n¡ 1 은 소수가 아님을 증명하여라. (몰도바 2000 최종-y8-3)

증명 귀류법으로, p := m+n¡1 ¸ 3이 소수라면, p j m2+n2¡1 = (p+1)2¡2mn¡1 = p2+2p¡2mn.

즉, p j mn. 그런데, p = m+ n¡ 1 > m;n 이므로 p - m;n 이 되어 모순. ¤

145. 번계승 N !!은 N이 홀수일 때 N(N¡2)(N¡4) ¢ ¢ ¢ 5 ¢3 ¢1 로, N이 짝수일 때는 N(N¡2)(N¡4) ¢ ¢ ¢ 6 ¢4 ¢2로 정의된다. 예를 들어 8!! = 8 ¢ 6 ¢ 4 ¢ 2 이고 9!! = 9 ¢ 7 ¢ 5 ¢ 3 ¢ 1 이다. 1986!! + 1985!! 이 1987의 배수임을 보여라. (Towns 1986가을 S4)

Page 31: 실전수학올림피아드 1400제 해답

1.2 수론 고급문제 31

증명 1986!! ´ (¡1)(¡3)(¡5) ¢ ¢ ¢ (¡1985) ´ ¡1985!! (mod 1987) ¤

146. n은 다음을 만족하는 서로 다른 네 소수 a, b, c, d의 곱이다.

(i) a+ c = d

(ii) a(a+ b+ c+ d) = c(d¡ b)

(iii) 1 + bc+ d = bd

n을 구하여라. (아일랜드 2002-7)

풀이 (i)에서 a = 2 또는 c = 2. 두 경우에 해 각각 (iii) 먼저 고려하고 (ii)를 풀면 금방 풀림. ¢ ¢ ¢답 (2,7,11,13) }

147. 1, 2, 5, 10, 20, 50센트 및 1달러 짜리 동전들이 많이 있다. B개의 동전으로 A센트를 만들었다. A개의 동전으로 B달러를 만들 수 있음을 보여라. (레닌그라드 1987-20)

증명 x 2 M 이면 100x2 M 임에 주목. x센트 짜리 b개로 a센트를 만들었다면 100

x 센트 짜리 a개로

b달러를 만들 수 있음. 이것들을 선형으로 조립하면 됨. ¤

148. x2 + xy + y2 = 2 의 정수해 (x; y)를 모두 구하여라. (이탈리아 1989-1)

풀이1 x가 홀수이면 x2 + (x+ y)y 는 홀수가 되어 모순. x, y가 모두 짝수이면 4의 배수가 되어 모순.

해없음. }

풀이2 mod 3으로 (x¡ y)2 ´ 2 가 되어 모순. }

149. 어떤 정수 x에 해서도 x2 + 5x+ 16 은 169로 나누어떨어지지 않음을 증명하여라. (이탈리아 1990-5)

증명 13 j (x¡ 4)2. 캐나다의 121의 배수가 아니더란 문제와 비슷. ¤

150. 462n+1 + 296 ¢ 132n+1 이 1947의 배수임을 증명하여라. (헝가리 1947-1)

증명 462 ´ 132 (mod 1947) 이므로 132n(46 + 296 ¢ 13) ´ 0 (mod 1947) ¤

1.2 수론 고급문제

1. 10101은 몇 진법으로 읽어도 항상 합성수임을 증명하여라. (캐나다 1972-3b)

증명 다음과 같이 인수분해되므로 합성수이다.

10101(n) = n4 + n2 + 1 = (n2 + 1)2 ¡ n2 = (n2 + n+ 1)(n2 ¡ n+ 1)

n ¸ 2 이므로 n2 ¡ n+ 1 > 1 은 틀림없이 성립한다. ¤

2. 네 개의 홀수인 양의 정수 a, b, c, d (a · b · c · d) 중에서 임의로 세 수를 뽑아서, 그 합을 남아 있는 한수로 나누면 그 나머지가 언제나 1이라고 한다. 이러한 네 홀수의 순서쌍 (a; b; c; d)를 모두 구하여라.

(한국 2003-J5)

Page 32: 실전수학올림피아드 1400제 해답

32 수론

풀이 a+ b+ c+ d = S 라 하면, 적당한 정수 k1; k2; k3; k4에 해

S ¡ a = ak1 + 1; S ¡ b = bk2 + 1; S ¡ c = ck3 + 1; S ¡ d = dk4 + 1

S ¡ a = ak1 + 1, S ¡ b = bk2 + 1에서 다음이 성립한다: a(k1 + 1) = b(k2 + 1). 이와 같이 풀면

a(k1 + 1) = b(k2 + 1) = c(k3 + 1) = d(k4 + 1) | ①

식 ①의 값을 k라고 하자. 4d ¸ S = d(k4 + 1) + 1, d(3¡ k4) ¸ 1 (* d ¸ c ¸ b ¸ a ¸ 1). 3¡ k4 ¸ 1,2 ¸ k4. 그런데 S는 짝수이므로 d(k4 + 1) + 1 은 짝수이다. 즉, k4 6= 1,

k4 = 2

) d(k4 + 1) = k에서 k = 3d. c(k3 + 1) = d(k4 + 1) = k에서 다음이 성립한다.

3c ¸ a+ b+ c = S ¡ d = 2d+ 1 =2

3c(k3 + 1) + 1

3:5¡ 3

2c¸ k3, k3은 정수이므로 3 ¸ k3. 그런데 3d = c(k3 + 1)에서 k3은 짝수이므로

k3 = 2

즉, c = d 이다. b(k2 + 1) = c(k3 + 1)에서 다음이 성립한다.

2b ¸ a+ b = S ¡ 2c = 3c+ 1¡ 2c = c+ 1 =1

3b(k2 + 1) + 1; 4 ¸ k2

그런데 3c = b(k2 + 1)에서 k2는 짝수이므로 k2 = 2 또는 4이다. k2 = 2 이면 b = c = d에서 S =3d+ 1 = a+ 3d 이므로 a = 1이다. 그러나 이것은 S ¡ a를 a로 나눠 나머지가 1인 것에 모순이다. 따라서,

k2 = 4

즉, a(k1 + 1) = 5b = 3c = 3d 이고, a + b = c + 1에서 2c = 5a ¡ 5, 15(a ¡ 1) = 6c = 2a(k1 + 1),15a¡ 15 = 2ak1 + 2a.

(13¡ 2k1)a = 15 = 15£ = 5£ 3앞에서 a = 1이면 모순이므로, a = 15 또는 5또는 3이다.

a = 15 ¡! b = 21; c = 35; d = 35 (성립)

a = 5 ¡! b = 6; c = 10; d = 10 (b; c; d가 짝수)

a = 3 ¡! b = 3; c = 5; d = 5 (성립)

) (a; b; c; d) = (3; 3; 5; 5), (15; 21; 35; 35). }

3. 다음 등식을 만족하는 양의 정수해 (a; b; c)는 존재하지 않음을 보여라.

(2a+ b)(2b+ a) = 2c (뉴질랜드 2003-9)

증명 귀류법으로, 양의 정수해 (a; b; c)가 존재한다고 가정하자. 그러면 좌변에서 2a+ b; 2b+ a ¸ 3 이

므로 둘다 2k꼴, 즉 짝수이고, b, a 모두 짝수여야 한다.

a = 2na0; b = 2nb0

인 최 의 n을 잡으면 a0과 b0 중 적어도 하나는 홀수이다. 그러면, 준식은 다음과 같다.

22n(2a0 + b0)(2b0 + a0) = 2c

여기서 a0, b0 중 적어도 하나는 홀수이므로, 2a0 + b0, 2b0 + a0 중 적어도 하나는 3 이상의 홀수이다. 그것은 우변 2c이 3 이상의 홀수를 약수로 갖는다는 것이므로 모순. 따라서, 양의 정수해 (a; b; c)는 존재하지 않는다. ¤

Page 33: 실전수학올림피아드 1400제 해답

1.2 수론 고급문제 33

4. f1; 16; 27g 중에서 어떤 두 수의 곱에 9를 합한 것은 항상 제곱수임에 주목하자. n+ 9, 16n+ 9, 27n+ 9

가 모두 제곱수가 되도록 하는 유일한 자연수 n을 구하여라. (IMTS R3-1)

풀이 n+ 9 = a2, 16n+ 9 = b2 이라 하면 16(a2 ¡ 9) = b2 ¡ 9, 16a2 ¡ b2 = 135, 즉

(4a+ b)(4a¡ b) = 33 ¢ 5

이렇게 인수분해법으로 풀고 그 중에서 27n+ 9 도 완전제곱수가 되는 것을 찾으면... n = 280. 그런데문제에 힌트처럼 제시한 것은 무슨 도움이 되는 걸까... }

5. 17명의 해적이 금화 한 상자를 훔쳐서 공평하게 나누다 보니 3개가 남았다. 그 남은 것을 서로 차지하려고 싸우다가 한 명이 죽었다. 그래서 다시 나누어 보니 이번에는 10개가 남았다. 또 싸움이 벌어져서 세명이 죽고 나서야 비로소 공평하게 나누어 가질 수 있었다. 각자 1000개 이상씩의 금화를 가졌다면 훔친금화는 모두 몇 개인가? 그 최소값을 구하여라. (통신강좌 1990-1-6)

풀이 x ´ 3 (mod 17), x ´ 10 (mod 16), x ´ 0 (mod 13) 을 동시에 만족시키며 x ¸ 13000 인 것들

중 최소인 x를 찾으면 된다.

m1 = 17; m2 = 16; m3 = 13; a1 = 3; a2 = 10; a3 = 0

으로 놓고 중국인의 나머지정리를 이용하자.

m = 17 ¢ 16 ¢ 13 = 3536; M1 =m

m1= 16 ¢ 13; M2 =

m

m2= 17 ¢ 13; M3 =

m

m3= 17 ¢ 16

k = 1; 2; 3 에 하여 MkXk ´ 1 (mod mk) 를 계산하여

X1 ´ ¡4 (mod 17); X2 ´ 5 (mod 16); X3 ´ ¡1 (mod 13)

을 얻는다. 따라서

x = a1M1X1 + a2M2X2 + a3M3X3

´ 3 ¢ 16 ¢ 13 ¢ (¡4) + 10 ¢ 17 ¢ 13 ¢ 5 + 0 ¢ 17 ¢ 16 ¢ (¡1) ´ 8554 (mod m):

따라서 구하는 x는 15626이다. }

6. 1보다 큰 모든 자연수 n에 해, (n¡ 1)2이 nn¡1 ¡ 1 의 약수임을 증명하여라. (플란더즈 2001-1)

풀이 n 신 n + 1 로 치환하면 n2 j (n + 1)n ¡ 1 임을 보이는 문제가 되고, 우변을 인수분해하거나

낮은 차수의 항을 간단히 이항전개해보면 금방 확인된다. }

7. 임의의 정수 n > 1 에 해, n의 가장 큰 소인수를 p(n)으로 나타내자. 다음을 만족하는 서로 다른 양의정수 x, y, z의 순서쌍을 모두 구하여라.

(i) x, y, z는 등차수열을 이룬다.

(ii) p(xyz) · 3. (영국 2003 2차-1)

풀이 gcd(x; y; z) = g 라 하고 (x; y; z) = (ga; gb; gc) 라 놓자. 그럼, gcd(a; b; c) = 1 이고, g = 2r3s꼴

이며, a, b, c는 여전히 등차수열이다. a+ c = 2b 이므로 a와 c는 홀짝이 같다.

(1) a, c가 모두 홀수일 때;a > 1이라면 a와 c는 모두 3의 거듭제곱수이고,그럼 b = (a+c)=2도 3의 배수가 되어 gcd(a; b; c) =1 이라는 데에 모순이다. 따라서, a = 1.c = 3n 이라 놓으면 공차 d가 3의 배수일 수 없으므로 b도 3의 배수가 아니다. 즉, b는 짝수.b = (a+ c)=2 이므로 a+ c(= 1 + c) 가 4의 배수이고,

c = 3n ´ 3 (mod 4)

여야 하므로, n은 홀수. n = 2m+ 1로 두면

c = 3 ¢ 9m; a+ c ´ 4 (mod 8); b ´ 2 (mod 4)

가 된다. 그러므로, b = 2, c = 3. (x; y; z) = (g; 2g; 3g).

Page 34: 실전수학올림피아드 1400제 해답

34 수론

(2) a, c가 모두 짝수일 때;b도 짝수이면 안 되므로, b는 3의 거듭제곱수이다. a나 c가 3으로 나누어진다면 모두 3의 배수가 되어 모순이다. 따라서, a, c는 짝수, 즉 2의 거듭제곱수이다. a와 c가 모두 4 이상이면 b = (a+ c)=2

가 짝수가 되어 곤란하므로, a = 2.c = 2n+1 이라 놓으면 b = (a + c)=2 = 2n + 1 = 3m꼴이다. (a; b; c) = (2; 3; 4), (2; 9; 16)은 해가 되고, m ¸ 3 이면 더 이상 해가 없음을 보이자. n ¸ 4 이므로 b = 3m ´ 1 (mod 16) 이다.m = 4k + r 로 두면 b = 3r81k ´ 3r (mod 16). 따라서 r = 0, 즉 m = 4k꼴일 때에만 가능하다.

그런데, 2n = 81k ¡ 1 ´ 0 (mod 5) 이므로 5 j 2n. 이것은 모순.

(1), (2)에 의해 답은(x; y; z) = (g; 2g; 3g); (2g; 3g; 4g); (2g; 9g; 16g)

가 전부이다. 단, g = 2r3s꼴의 수이다. }

8. 어떤 양의 정수가 연속한 둘 이상의 양의 정수들의 합이 된다는 것과 그 수가 2의 거듭제곱수가 아니라는것이 필요충분임을 증명하여라. (캐나다 1976-5)

증명 임의의 연속한 둘 이상의 양의 정수들의 합은 n; k ¸ 1 에 해

n+ (n+ 1) + ¢ ¢ ¢+ (n+ k) =(k + 1)(2n+ k)

2

로 나타낼 수 있다. 분자의 (k+ 1)과 (2n+ k)는 서로 홀짝이 다르고 둘다 2 이상이므로 이 합은 홀수인소인수를 갖는다. 따라서 2의 거듭제곱일 수 없다.

한편, N이 2의 거듭제곱이 아닌 임의의 양의 정수라 하자. 그럼 2N도 2의 거듭제곱이 아니고,

2N = ab (a는 2의 거듭제곱수, b는 홀수)

와 같이 분해할 수 있다. a와 b 모두 2 이상이고 홀짝이 다르므로, 둘 중 작은 것을 k+1로 두고 둘 중 큰것을 k + 2n으로 둘 수 있다(n; k ¸ 1). 그럼

N =(k + 1)(2n+ k)

2= n+ (n+ 1) + ¢ ¢ ¢+ (n+ k)

로 연속한 둘 이상의 양의 정수들의 합으로 나타낼 수 있다. ¤

9. 1990은 오른쪽에 자릿수 하나를 붙이고 왼쪽에 몇 개의 자릿수를 붙임으로써 `제곱수로 만들 수 있다'. 예를 들면, 419904 = 6482 이다. 같은 과정을 1991에 적용시켰을 때, 1991은 제곱수로 만들 수 없음을 보여라. 즉, ¢ ¢ ¢ yx1991d 가 제곱수가 되도록 하는 자릿수 d; x; y; : : : 는 존재하지 않음을 보여라.

(IMTS R3-2)

증명 십의 자리가 홀수임에서 일의 자리는 6. 그리고나서 mod 16으로 체크하면 12가 되어 제곱수가

아님을 확인. ¤

10. p와 p2 + 8 이 모두 소수이면 p3 + 4 도 소수임을 증명하여라. (통신강좌 1993-7-20)

풀이 p = 3 이면, p2 + 8 = 17, p3 + 4 = 31 로 모두 소수이다. p 6= 3 이면 3 - p 이므로 p ´ §1(mod 3), 결국 p2 + 8 ´ 0 (mod 3) 이 되고, 3의 배수. 따라서, p와 p2 + 8 이 모두 소수이면 p3 + 4 도소수이고, 이런 경우는 p = 3 일 때 뿐이다. }

11. m1;m2; : : : ;ms가 임의의 자연수라고 하자. [mi;mj ]를 mi, mj의 최소공배수, (mi;mj) 를 mi, mj의 최

공약수라 할 때 다음을 증명하여라. (통신강좌 1991-3-6)

[(m1;ms); (m2;ms); : : : ; (ms¡1;ms)] = ([m1;m2; : : : ;ms¡1];ms)

풀이 a =Q

peii , b =

Qpfii 라 하자. 단, pi는 i번째 소수이다. 그러면

[a; b] =Y

pmax(ei;fi)i ; (a; b) =

Ypmin(ei;fi)i (1)

Page 35: 실전수학올림피아드 1400제 해답

1.2 수론 고급문제 35

이 된다. 준식의 좌변과 우변을 각각Q

p®ii ,Q

p¯ii 라 하고,

mj =Yi

pnj;ii (j = 1; 2; ¢ ¢ ¢ ; s)

라 하자. 그러면 (1)로 부터

®i = max[min(n1;i; ns;i);min(n2;i; ns;i); ¢ ¢ ¢ ;min(ns¡1;i; ns;i)];¯i = min[max(n1;i; n2;i; ¢ ¢ ¢ ; ns¡1;i); ns;i]

이 성립하며, 임의의 i에 해 ®i = ¯i임을 보이면 된다. 편의를 위해 마지막 첨자 i를 생략하기로 하고,max(n1; n2; ¢ ¢ ¢ ; ns¡1) = nk라 하자.(i) ns ¸ nk일 때; ® = max(n1; n2; ¢ ¢ ¢ ; ns¡1) = nk이고, ¯ = min(nk; ns) = nk이므로 성립한다.(ii) ns < nk일 때; ® = max(t1; t2; ¢ ¢ ¢ ; ts¡1)라 할 때, tj = min(nj ; ns) · ns이고 tk = ns이므로

® = ns이다. 또 ¯ = ns = ®이므로 이 경우에도 성립한다.(i), (ii)에 의해 임의의 i에 해 ®i = ¯i가 성립하므로 문제의 등식이 성립한다. }

12. x2 + y 와 x+ y2 이 모두 완전제곱수가 되게 하는 자연수 x와 y는 존재하지 않음을 보여라.(소련 1966-3)

증명 x2 + y 는 x2보다 큰 완전제곱수이므로 x2 + y ¸ (x+ 1)2, 따라서

y ¸ 2x+ 1; 마찬가지로 x ¸ 2y + 1

그럼 y > x, x > y 이므로 이것은 모순이고, 이런 자연수해는 존재하지 않는다. ¤

13. 임의의 양의 정수 n에 해 항상 n3(n2 ¡ 1)(n2 ¡ 4) 의 약수가 되는 가장 큰 정수를 구하여라.(1989 뉴욕주 수학리그)

풀이

n3(n2 ¡ 1)(n2 ¡ 4) = n2(n¡ 2)(n¡ 1)n(n+ 1)(n+ 2) = A

가 된다. 이때 연속한 n개의 수는 n!임의 배수임을 이미 알고 있다.

이 값을 실제 n = 3, 4일 때 계산해 보면 9*5!, 32*3*5! 이 된다. 이 둘의 공약수는 3*5!이므로 우리는A가 항상 3*5!을 약수로 가진다고 추측할 수 있다.

만약 n = 3k라면 A = n2*(5!의 배수)이므로 A는 3*5!의 배수이다.

또한 n 6= 3k라면 n¡ 2, n¡ 1, n+ 1, n+ 2 중 적어도 두개는 3의 배수이고, A는 9의 배수가 된다.5!의 배수이며 동시에 9의 배수이므로 3*5!의 배수가 된다.

따라서 항상 A는 3*5!을 약수로 가진다.

만약 A가 3*5!보다 큰수를 약수로 가진다고하면 그 수는 k ¤ 3 ¤ 5!로 표현가능할 것이나 (k 6= 1) n = 3,4일 때에 입해보면 k는 3의 약수이면서 32의 약수여야 하는데 이는 불가능하다. 따라서 3*5!가 항상A의 약수가 되는 가장 큰 정수이다. }

14. w! = x! + y! + z! 의 모든 양의 정수해 w, x, y, z를 구하여라. (캐나다 1983-1)

풀이 x; y; z < w 이다.

² w ¸ 4 이면 w! > 3(w ¡ 1)! ¸ x! + y! + z! 이므로 해가 없다.

² w · 2 이면 w! · 2 < 3 · x! + y! + z! 이므로 해가 없다.

² w = 3 이면 3! = 3 ¢ 2! ¸ x! + y! + z! 이고 등호는 x = y = z = 2 일 때 성립.

따라서, 해는 유일하다. ¢ ¢ ¢ 답 (w; x; y; z) = (3; 2; 2; 2) }

15. 다음과 같은 꼴로 나타낼 수 있는 양의 정수를 모두 구하여라.

b

a+

c

a+

c

b+

a

b+

a

c+

b

c

단, a, b, c는 쌍마다 서로소(즉, gcd(a; b) = gcd(b; c) = gcd(c; a) = 1)인 양의 정수이다. (한국 2006-J2)

Page 36: 실전수학올림피아드 1400제 해답

36 수론

풀이 a · b · c 라 해도 되겠다①.

준식 =(a+ b)ab+ (b+ c)bc+ (c+ a)ca

abc

이고 이것이 양의 정수이려면

abc j (a+ b)ab+ (b+ c)bc+ (c+ a)ca

여야 한다. (b+ c)bc+ (c+ a)ca 는 c의 배수이므로 나머지항 (a+ b)ab도 c의 배수이다. ab와 c는 서로소이므로

c j (a+ b)

이다. ①에 의해 a+ b · 2c 이므로 a+ b = c 또는 2c이다.

(1) a + b = 2c 일 때: 이 때 a = b = c 이며, 쌍마다 서로소이므로 그 값들은 모두 1이다. 그럼 준식 = 1 + 1 + 1 + 1 + 1 + 1 = 6.

(2) a+ b = c 일 때: c j (a+ b) 를 이끌어낸 과정과 같은 방법으로 b j (a+ c) 가 된다. 즉, b j (2a+ b)

이고, b j 2a 이다. ①에 의해 2a · 2b 이므로 2a = b 또는 2b이다.

(2a) b = 2a 일 때: a = 1, b = 2, c = 3 이고 준식 = 1+23+ 2+3

1+ 3+1

2= 8.

(2b) b = a 일 때: a = 1, b = 1, c = 2 이고 준식 = 1+12+ 1+2

1+ 2+1

1= 7.

답은 6, 7, 8이다. }

16. a, b, c, d는 고정된 정수들이고 d는 5로 나누어지지 않는다고 하자. m을

am3 + bm2 + cm+ d

가 5로 나누어지도록 하는 정수라고 가정하자. 그럼

dn3 + cn2 + bn+ a

도 5로 나누어지도록 하는 정수 n이 존재함을 증명하여라. (헝가리 1900-1)

증명 m이 5의 배수이면 am3 + bm2 + cm + d은 5의 배수가 아니므로, 5 - m. 그럼 m은 5와 서로소

이므로 mod 5로 m의 잉여역수 n이 존재한다(mn ´ 1 (mod 5) 가 되는 수를 말함). 그럼 n3(am3 +bm2 + cm+ d) ´ a+ bn+ cn2 + dn3 ´ 0 (mod 5). ¤

17. 어떤 자연수의 자릿수근이란 각 자리의 수들을 모두 곱하는 것을 계속 반복하여 얻은 한 자리의 수를 말한다. 예를 들어 24378! 1344! 48! 32! 6 이므로 24378의 자릿수근은 6이다. 자릿수근이 1인 수는모든 자릿수가 1인 수뿐임을 보여라. (아일랜드 1992-7)

증명 a! 111 ¢ ¢ ¢ 1! 1 에서 111 ¢ ¢ ¢ 1은 10보다 작은 소인수만 갖는다. 그중 2나 5는 있으면 안 되고,

3과 7뿐인데, 3은 111의 약수이므로 3이 있으려면 37이 있어야 해서 안 된다. 고로 7뿐. 111 ¢ ¢ ¢ 1 = 7®라 하면, 7n들은 mod 100으로

01! 7! 49! 343! 2401

와 같이 순환함. 따라서, ® = 0뿐. ¤

18. an이 13 + 23 + ¢ ¢ ¢+ n3 의 일의 자리 숫자이다. 이 때, 0:a1a2 ¢ ¢ ¢ an ¢ ¢ ¢ 이 유리수임을 보여라.(셈본중등초급 도전문제 2.3.4)

증명 0:a1a2 ¢ ¢ ¢ an ¢ ¢ ¢이 순환소수임을 확인하면 된다. (10k + r)3 ´ r3 (mod 10) 이므로, 03 + 13 +

¢ ¢ ¢+ 93 ´ a (mod 10) 이라 하면 an+10 ´ an + a, 즉 an+100 ´ an + 10a ´ an (mod 10) 이다. 즉, 순환마디가 100(혹은 100의 약수)인 순환소수가 된다. ¤

주 r3 + (10¡ r)3 ´ r3 ¡ r3 = 0 (mod 10) 임을 관찰하면 a ´ 5 임을 알 수 있고 순환마디도 보다 짧

은 20임을 관찰할 수도 있다.

Page 37: 실전수학올림피아드 1400제 해답

1.2 수론 고급문제 37

19. (1) x2 + y2 + 1 = z2 이 무한히 많은 정수해를 가짐을 보여라.(2) 임의로 주어진 정수 a에 해 x2 + y2 + a = z2 이 무한히 많은 정수해를 가짐을 보여라.

(셈본중등고급 도전문제 2.3.2 및 확장, 스웨덴 1974-5 유사)

증명 (1) 임의의 정수 m에 해 (x; y; z) = (2m2; 2m; 2m2 + 1) 이면 항상 해가 된다.

(2) 주어진 a의 홀짝에 따라 x2+a 가 항상 홀수가 되도록 x를 무한히 많이 택할 수 있다. x2+a = 2k+1

로 두면 y = k, z = k + 1 이면 항상 해가 된다. ¤

20. pp+1 + qq+1이 소수가 되는 소수 p, q를 모두 구하여라. (2000 대전.충남 영재수학교실 2차평가)

풀이 우선 p = 2, q = 3 (또는 p = 3, q = 2) 일 때 pp+1 + qq+1 = 89 로 성립한다. p, q가 둘다 홀수

인 소수이면 pp+1 + qq+1 은 2보다 큰 짝수가 되어 소수가 될 수 없다. 둘다 2일 때도 마찬가지. 따라서둘 중에 하나는 2가 되고 나머지 하나는 홀수인 소수가 되어야 한다. 홀수인 소수(q라 하자)가 3이 아닐경우

22+1 = 8 ´ 2 (mod 3)

qq+1 = (3k § 1)짝 ´ 1 (mod 3)

즉, 22+1 + qq+1 ´ 0 (mod 3) 이 되어 소수가 아니다. 따라서 구하려는 쌍은 (2; 3), (3; 2) 둘뿐이다. }

21. n은 음이 아닌 정수이고, d0; d1; : : : ; dn 은 0, 1, 2로만 이루어진 수열이다. d0+3d1+¢ ¢ ¢+3kdk+¢ ¢ ¢+3ndn이 어떤 자연수의 제곱이라면, di = 1 이 되는 i (1 · i · n) 가 적어도 하나 존재함을 증명하여라.

(호주 1989-5)

개요 귀류법. 3m(3k + 2)꼴이라서 제곱수가 아님. }

22. 연속한 1984개의 양의 정수들의 제곱의 합은 정수의 제곱이 될 수 없음을 보여라. (캐나다 1984-1)

증명 연속한 1984개의 제곱의 합은

S =1984Xi=1

(n+ i)2 =1984Xi=1

(n2 + 2ni+ i2)

= 1984n2 + 2n ¢ 1984 ¢ 19852

+1984 ¢ 1985 ¢ 3969

6

= 992(2n2 + 2 ¢ 1985n+ 1985 ¢ 1323)

이 식은 25 £ (홀수) 꼴이므로 2의 지수가 짝수가 아니어서 제곱수가 될 수 없다. ¤

23.x+ y

2+pxy = 40 을 만족하는 서로 다른 양의 정수쌍 fx; yg는 꼭 하나 있음을 증명하여라.

(루마니아 지역예선 2005-y7-3b)

증명 x+y2+pxy = 40, 2배하면 (

px+

py )2 = 80 와 같은 식이다. 즉,

px+

py =

p80 (¤)

한 항을 넘긴 식py =

p80¡px 를 제곱하면 y = 80+x¡2p80x. 따라서 2

p80x은 정수. 2

p80x = m

으로 두고 제곱하면 82 ¢ 5x = m2. 따라서

x = 5a2 마찬가지로 y = 5b2

꼴이다(a, b는 적당한 양의 정수). (¤)에 입하면 a + b = 4. x 6= y 라 했으므로 a = b = 2 일 수 없고, 따라서 순서를 무시하면 (a; b) = (3; 1) 뿐이다. 즉, 서로 다른 양의 정수쌍 (x; y)는 순서를 무시하면(45; 5)뿐이다. ¤

Page 38: 실전수학올림피아드 1400제 해답

38 수론

24. 다음과 같은 꼴로 나타낼 수 있는 양의 정수를 모두 구하여라.

m2 + 20mn+ n2

m3 + n3

단, m, n은 서로소인 양의 정수이다. (한국 2006-J5)

풀이 (서울 당산서중 2학년 박민재)

준식 =m2 + 20mn+ n2

(m+ n)(m2 ¡mn+ n2)

이므로 (m+ n) j (m2 + 20mn+ n2), 즉

(m+ n) j ((m2 + 20mn+ n2)¡ (m+ n)2) = 18mn

이어야 한다.그런데 gcd(m;n) = 1이므로 gcd(mn;m+n) = 1 (* gcd(m;m+n) = gcd(n;m+n) = 1)

이고 결국 m+ n j 18 을 얻는다.

(i) m+ n = 9 or 18 일 때:

준식 =m2 + 20mn+ n2

(m+ n)(m2 ¡mn+ n2)=

(m+ n)2 + 18mn

(m+ n)((m+ n)2 ¡ 3mn)

이다. m+n 이 9의 배수이면 분모는 27의 배수이므로 분자 역시 27의 배수이어야 한다. 그러기 위해서 m 또는 n이 3의 배수이어야 한다. 이것은 gcd(m;n) = 1 이라는 것에 모순이다. 즉, 이 경우에 해는 없다.

(ii) m+ n = 2 or 3 or 6 일 때: gcd(m;n) = 1 이 되는 (m;n)은 (1,1), (1,2), (1,5)뿐이다(순서 상관없음). 세 경우 모두 입해보면 해가 된다.

m, n은 양의 정수이므로 m+ n = 1 이 될 수 없고 (i), (ii)에 의해서는 해는 세 가지 뿐이다. 각각의 경우 원래 식의 값을 구해보면 11, 5, 1이 나온다. }

25. 20052을 2개의 (양의) 완전제곱수의 합으로 나타내는 방법이 네 가지 이상 있음을 증명하여라.(플란더즈 2005-J3/S3)

증명 2005 = 5 ¢ 401 = (22 + 1)(202 + 1) 이다. Lagrange 항등식

(a2 + b2)(c2 + d2) = (ac¡ bd)2 + (ad+ bc)2

에 의해(c2 + d2 이 d2 + c2 으로 순서가 바뀔 수도 있으므로)

2005 = (22 + 1)(202 + 1) = 392 + 222

= 412 + 182

이 된다. 다시 이로부터

20052 = (412 + 182)(412 + 182) = (412 ¡ 182)2 + (2 ¢ 41 ¢ 18)2

20052 = (392 + 222)(392 + 222) = (392 ¡ 222)2 + (2 ¢ 39 ¢ 22)2

20052 = (412 + 182)(392 + 222) = (41 ¢ 39¡ 18 ¢ 22)2 + (41 ¢ 22 + 18 ¢ 39)2

20052 = (412 + 182)(392 + 222) = (41 ¢ 39 + 18 ¢ 22)2 + (41 ¢ 22¡ 18 ¢ 39)2

등 네 가지 방법으로 나타낼 수 있다(물론 위의 네 가지 계산에서 나타나는 수들이 모두 다르다는 것은확인할 필요가 있다). ¤

26. 서로 다른 임의의 세 정수 a, b, c가 주어졌을 때, a+ n, b+ n, c+ n 이 둘씩 서로소가 되도록 하는 정수n을 항상 찾을 수 있을까? (셈본중등고급 도전문제 2.2.2 변형)

증명 임의의 서로 다른 양의 정수 a, b에 해 n, n + a, n + b 가 서로소가 되도록 n을 잡아보자.

a = da0, b = db0, gcd(a0; b0) = 1 이라 할 때, a0 ¡ b0 은 a0, b0과 모두 서로소이다. 따라서, kb0 ´ 1(mod a0 ¡ b0) 인 k를 잡을 수 있다. 이제 n = 1¡ da0b0k 라 두면 세 수가 모두 서로소가 된다. ¤

Page 39: 실전수학올림피아드 1400제 해답

1.2 수론 고급문제 39

27. 임의의 자연수 n에 해(2n)!

(n+ 1)!n!이 항상 정수가 됨을 보여라.

증명 임의의 소수 p에 해, p가 분자에 곱해져있는 횟수가 분모에 곱해져있는 횟수 이상임을 보이면

된다. m!을 소인수분해했을 때 소수 p의 지수는 bmpc+ bm

p2c+ bm

p3c+ ¢ ¢ ¢ 의 공식으로 구할 수 있음을

이용하자. 즉, 임의의 소수 p와 임의의 자연수 k에 해¹2n

pk

º¸¹n+ 1

pk

º+

¹n

pk

º

임을 보이면 충분하다(이런 식을 모든 k에 해 변변 더하면 됨). a가 b의 배수가 아닐 때는 babc = b a¡1

bc

가 성립한다. 즉, n이나 n+ 1 중에 pk의 배수가 아닌 것이 있으면 둘 중 하나를 1만큼 줄여도 우변이 변하지 않고, 그럼 ba+ bc ¸ bac+ bbc 의 당연한 부등식에 의해 성립한다. n과 n+1 모두 pk의 배수가 되는 경우만 염려하면 되는데, 그럼 pk j (n+ 1)¡ n = 1 이므로 이런 경우는 없다. ¤

주 이항계수를 안다면¡2nn

¢ ¡ ¡ 2nn+1

¢을 정리했을 때 준식이 된다는 것을 확인할 수도 있다. 그럼 정

수의 차이므로 당연히 정수. 한편, 준식은 n개의 개괄호와 n개의 폐괄호를 잘 정렬하는 경우의 수인Catalan 수로 알려져있는 식이기도 하다.

28. §1 § 4§ 9§ : : : § n2 꼴의 합을 생각하자. 임의의 정수를 이 꼴로 표현할 수 있음을 증명하여라. (예를들어, 3 = ¡1 + 4, 8 = 1¡ 4¡ 9 + 16 + 25¡ 36¡ 49 + 64 이다.) (이탈리아 1991-3)

증명 k2 ¡ (k + 1)2 ¡ (k + 2)2 + (k + 3)2 = 4 임을 이용. 0, 1, 2(= ¡1¡ 4¡ 9 + 16), 3= ¡1 + 4 가

가능하므로, 4씩 증가시켜서 모두 다 가능함. 홀수는 (a+ 1)2 ¡ a2 = 2a+ 1 로도 할 수 있음. ¤

29. a, b, c는1

a+1

b=1

c를 만족하는 최 공약수가 1인 양의 정수들이다. (a+ b)가 완전제곱수임을 증명하

여라. (인도지역예선 1992-2)

풀이 a와 b의 최 공약수를 d라 하자. a = dx; b = dy라 하자. 그러면 x와 y는 서로소이다. 양변을 통

분하고 d를 오른쪽으로 넘기면,x+ y

xy=

d

c

이 된다. x와 y가 서로소이므로 x+ y와 xy도 서로소이고, c와 d도 서로소이므로 양변이 모두 기약분수이다. 즉, x+ y = d, xy = c이다. 따라서 a+ b = dx+ dy = d2으로 a+ b는 완전제곱수이다. }

30. 맨앞의 자릿수를 맨마지막으로 이동시키면 2배가 되는 정수가 존재하는지 그렇지 않은지 밝혀라.(캐나다 1985-2)

증명 주어진 조건을 만족하는 수를 a ¢ 10n + b 라 하자(a는 맨앞의 자릿수이고, b는 n¡ 1자리의 수).

10b+ a = 2£ (a ¢ 10n + b)

8b = (2 ¢ 10n ¡ 1)a

좌변은 8의 배수이고 (2 ¢ 10n ¡ 1)은 홀수이므로 8 j a, 즉 a = 8 이어야 한다. 그럼 b = 2 ¢ 10n ¡ 1 이고,

이것은 n자리의 수이므로 자리수가 넘어 모순. 따라서, 이런 정수는 존재하지 않는다. ¤

31.pn+

pn+ 2005 가 정수가 되도록 하는 정수 n을 모두 구하여라. (플란더즈 2005-4)

풀이pn이 유리수이면

pn은 정수이다(¤). 즉,

pn은 정수이거나 무리수이다.

(i)pn,pn+ 2005 중 하나만 정수인 경우: 정수(유리수)와 무리수의 합은 무리수이므로 불가능.

(ii)pn,pn+ 2005 가 둘다 무리수인 경우:

pn = a

pb,pn+ 2005 = c

pd 라 하자(단, b, d는 무승

수, 즉 제곱수를 약수로 갖지 않는 수).

pn+

pn+ 2005 = a

pb+ c

pd = k(정수)

Page 40: 실전수학올림피아드 1400제 해답

40 수론

라 하면a2b+ c2d+ 2ac

pbd = k2

이므로pbd는 유리수, 즉 (¤)에 의해 정수이다. 그럼 bd는 완전제곱수인데, b와 d는 무승수이므로

소인수분해를 생각하면 모든 소인수를 공통으로 갖고 있어야 해서 b = d 임을 알 수 있다. 이 때

pn+

pn+ 2005 = (a+ c)

pb 6= 정수

이므로 모순.

(iii)pn,pn+ 2005 가 둘다 정수인 경우:

pn = m,

pn+ 2005 = k 라 하면 m2 = n, k2 = n+ 2005,

즉 k2 ¡m2 = 2005 이므로(k +m)(k ¡m) = 2005 = 5 ¢ 401

여기서 가능한 경우는 (k ¡ m; k + m) = (1; 2005) 혹은 (5; 401)뿐이다. 각각 풀면 (k;m) =

(1003; 1002), (203; 198), 즉 n = 1982 과 10022만 가능하다.

(i){(iii)에서 답은 n = 1982, 10022. }

32. 옛날 옛날에 서코나라가 있었다. 그 나라의 임금님은 이름을 남기는 것을 좋아했다. 어느날 자신의 이름을 어디다 남길까 고민하다가

\그래 나만의 화폐를 만드는 거야!"

하고 결심했다. 그래서 그 다음날 그 나라의 화폐는 모두 서코임금의 초상화가 그려진 화폐로 모두 바뀌었다.

발행된 동전은 5원짜리, 9원짜리, 12원짜리의 3가지였다. 어느날 심심한 서코임금은 신하들에게 이런 문제를 냈다.

\내가 만든 3가지 금화로 거스름을 받지 않고는 정확하게 지불할 수 없는 가장 큰 금액은 얼마이겠는가?"

과연 얼마일까? (셈본중등초급 도전문제 2.2.2)

풀이 먼저 12원짜리는 고려하지 말고 5원짜리와 9원짜리 동전만 생각해보자. 9원짜리 5개를 쓰는 것

은 5원짜리 9개를 쓰는 것으로 바꿀 수 있으므로 9원짜리 5개 이상을 쓰는 것은 고려하지 않아도 된다.9원짜리를 0개, 1개, 2개, 3개, 4개 쓸 때를 각각 생각해보면 두 동전으로 만들 수 없는 금액은

5n꼴 9원짜리 0개 : 없음

5n+ 1꼴 9원짜리 4개 : 1 6 11 16 21 26 31

5n+ 2꼴 9원짜리 3개 : 2 7 12 17 22

5n+ 3꼴 9원짜리 2개 : 3 8 13

5n+ 4꼴 9원짜리 1개 : 4

들이다. 이외의 수는 5원짜리와 9원짜리 두 동전으로 다 만들 수 있다. 이제 12원짜리를 함께 고려하자.12원짜리를 1개 쓰면 위의 목록 중 12, 17, 21, 22, 26, 31원은 만들 수 있게 된다. 12원짜리를 2개 써서더 만들 수 있는 금액은 없다. 따라서, 세 가지 금화로 정확하게 지불할 수 없는 금액은 1, 2, 3, 4, 6, 7,8, 11, 13, 16원. ¢ ¢ ¢ 답 16원 }

33. 성식이는 각각 1; 2; 4; 8; : : : 의 2의 거듭제곱수가 차례로 한 장에 하나씩 적혀있는 카드를 갖고 있었다. 성식이가 갖고 있는 이 카드의 수는 30장에서 35장 사이였는데, 며칠 전에 그 중 몇 장을 잃어버렸다. 남은카드의 수의 합은 원래 전체 카드의 수의 합의 정확히 1=5이라고 한다. 잃어버린 카드는 모두 몇 장인가?

(1994 미국 지역별 수학리그)

풀이 원래 카드의 수가 n장이었다면 카드의 수의 합은 1+2+¢ ¢ ¢+2n¡1 = 2n¡1. 이것이 일부 카드를

잃어버린 후의 5배이므로 5의 배수. 2n은 mod 5로 2, 4, 3, 1을 반복하므로, n은 4의 배수. 30 · n · 35라 했으므로 n = 32.

11111111111111111111111111111111(2)=101(2) = 110011001100110011001100110011(2)

이므로 나중의 카드 개수는 16개, 잃어버린 카드의 개수도 16개이다. ¢ ¢ ¢ 답 }

Page 41: 실전수학올림피아드 1400제 해답

1.2 수론 고급문제 41

34. 7 j 3n + n3 은 7 j 3n ¢ n3 + 1 이 되기 위한 필요충분조건임을 보여라.(통신강좌 1997-14-2, 몰도바 1996 최종-y9-5 변형)

증명 7 j (3n ¡ 1)(3n + 1)(n3 ¡ 1)(n3 + 1) 만 보이면 됨. ¤

35. 잉여식 x ´ a (mod n), x ´ b (mod m) 이 해를 가질 필요충분조건은 gcd(n;m) j (a¡ b) 임을 보여라.

또, 해가 존재하는 경우, 그 해는 lcm(n;m)에 한 잉여계로 유일함을 보여라. (통신강좌 1993-7-3)

풀이 x ´ a(mod n), x ´ b(mod m)의 해가 존재한다는 것은 x = pn+a = qm+ b를 만족하는 p; q가

존재한다는 것이다. a¡ b = qm¡ pn을 만족하는 p; q가 존재할 필요충분조건은 gcd(m;n)j(a¡ b)이다또 다른 정수 y가 y ´ a(mod n), y ´ (mod m)을 만족한다면, x ´ y(mod n), x ´ y(mod m), 즉nj(x¡ y), mj(x¡ y)이므로 lcm(n;m)j(x¡ y)이다.

따라서 x ´ y(mod lcm(n;m))이므로 해의 유일성이 증명되었다. }

36. 자연수 n의 막자리합 D(n)은 다음과 같이 반복적으로 정의된다:

D(n) =

(n; 1 · n · 9 일 때

D(a0 + a1 + ¢ ¢ ¢+ am); n > 9 일 때

단, a0; a1; : : : ; am 은 n을 십진법으로 썼을 때의 각 자릿수들이다. 예를 들어, D(989) = D(26) = D(8) =8 과 같이 되는 것이다.

임의의 n = 1; 2; 3; : : : 에 해D(1234 ¢ n) = D(n)

이 성립함을 증명하여라. (셈본중등초급 도전문제 2.4.3)

증명 n을 십진법으로 썼을 때의 각 자릿수의 합을 f(n)이라 하자. 그럼 D(n)은 결과가 한 자리수가

될 때까지 f(f(¢ ¢ ¢ f(n) ¢ ¢ ¢ )) 와 같이 f를 반복 적용하는 것이다.

D(n) ´ n (mod 9)

임을 보이자. f(n) ´ n (mod 9) 임만 보이면 충분하다. n을 십진법으로 쓴 것이 n = am ¢ ¢ ¢ a1a0 이라면,

n¡ f(n) = (10mam + ¢ ¢ ¢ 10a1 + a0)¡ (am + ¢ ¢ ¢+ a1 + a0)

= 9 ¢ ¢ ¢ 99| {z }m개

am + ¢ ¢ ¢+ 9a1 + 0a0

로 9 j n¡ f(n) 임을 알 수 있다. 따라서, f(n) ´ n (mod 9) 이다. 따라서 D(n) ´ n (mod 9) 이고,

D(n) =

(n을 9로 나눈 나머지 (9 - n 일 때)

9 (9 j n 일 때)

가 된다. 이로부터A ´ B (mod 9) 이면 D(A) = D(B)

가 됨을 알 수 있다. 이제 1234n ´ n (mod 9) 이므로 문제가 증명되었다. ¤

37. a2 + b2 = n! 의 모든 양의 정수해 a, b, n을 구하여라. 단, a · b 이고 n < 14 이다. (캐나다 1987-1)

풀이 우선 n ¸ 7 일 때를 보자. 7 j n! 이므로 7 j a2 + b2. 완전제곱수는 n2 ´ 0; 1; 2; 4 (mod 7) 만이

가능하므로, a2 + b2 ´ 0 (mod 7) 이 될 수 있는 쌍은 (a2; b2) ´ (0; 0) (mod 7) 뿐이다. 따라서, 7 j a; b이고, 72 j a2 + b2 = n!. 그러나, n < 14 이므로 72 - n! 이고, 그럼 이 경우에는 해가 없다.

다음으로 3 · n · 6 일 때를 보자. 이 때 3 j n! = a2 + b2 이 되고, n2 ´ 0; 1 (mod 3) 만이 가능하므로,

앞에서와 비슷하게 3 j a; b, 즉 32 j a2 + b2 = n! 이다. n · 5 이면 32 - n! 이므로 n = 6 이어야 한다.mod 4로 생각하면 a, b 모두 짝수, 즉 6 j a; b 임도 알 수 있다. (6a0)2 + (6b0)2 = 6! = 720 의 해를 구하면, a02 + b02 = 20 에서 (a0; b0) = (2; 4) 뿐이므로 (a; b) = (12; 24) 이다.

마지막으로 n = 1; 2 일 때는 그냥 구해보면 (a; b; n) = (1; 1; 2) 만이 가능하다.

¢ ¢ ¢ 답 (a; b; n) = (1; 1; 2), (12; 24; 6) }

Page 42: 실전수학올림피아드 1400제 해답

42 수론

38. 자연수 m, n, k가 주어져 있다. 이 때, rm+ sn 이 k의 배수가 되도록 하는 서로소인 두 수 r, s를 항상 찾을 수 있음을 증명하여라. (소련 1961-9)

증명 Care is needed. Although easy, this is more awkward than it looks. Let d = (m;n), thegreatest common divisor of m and n. Let r = n=d, s = nhk ¡ m=d, where h is any integersu±ciently large to ensure that s > 0. Now rm + sn = mn=d + nnhk ¡mn=d = nnhk, which is amultiple of k. If e divides r, then it also divides rdhk = nhk. So if e divides r and s, then it alsodivides s¡nhk = ¡m=d. But n=d and m=d are relatively prime, so e must be 1. Hence r and s arerelatively prime. ¤

39. 다음 방정식을 만족하는 자연수 a, b, c를 모두 구하여라. (인도지역예선 1996-2)µ1 +

1

a

¶µ1 +

1

b

¶µ1 +

1

c

¶= 3

관찰 좌변에서 a; b; c 등이 충분히 크면 1+ 1a, 1 + 1

b, 1c 등이 1보다 얼마 크지 않은 수가 되어 그 곱이

3보다 작을 것이다. 즉, a; b; c가 충분히 작을 때만 해가 존재할 가능성이 있다. 따라서, 이 부정방정식은부등식법으로 접근하는 것이 좋겠다.

풀이 일반성을 잃지 않고 a · b · c 라 하자. a ¸ 3 이면 준식은

3 =

µ1 +

1

a

¶µ1 +

1

b

¶µ1 +

1

c

¶·µ1 +

1

3

¶3=64

27< 3

으로 성립하지 않는다. 따라서, a · 2.(i) a = 2 일 때: 준식은 다음과 같이 바뀐다.µ

1 +1

b

¶µ1 +

1

c

¶= 2

양변에 bc를 곱하고 정리하면

(b+ 1)(c+ 1) = 2bc

bc¡ b¡ c¡ 1 = 0(b¡ 1)(c¡ 1) = 2

따라서, 2의 약수를 생각하면 1 · b¡ 1 · c¡ 1 임에서 (b¡ 1; c¡ 1) = (1; 2)뿐이고, 즉 (a; b; c) =(2; 2; 3) 이다.

(ii) a = 1 일 때: 준식은 다음과 같이 바뀐다.µ1 +

1

b

¶µ1 +

1

c

¶=3

2

양변에 2bc를 곱하고 정리하면

2(b+ 1)(c+ 1) = 3bc

bc¡ 2b¡ 2c¡ 2 = 0(b¡ 2)(c¡ 2) = 6

따라서, 6의 약수를 생각하면 0 · b ¡ 1 · c ¡ 1 임에서 (b ¡ 2; c ¡ 2) = (1; 6), (2,3)뿐이고, 즉(a; b; c) = (1; 3; 8) 또는 (1; 4; 5)이다.

(i), (ii)에서 구하는 해는 ¢ ¢ ¢ 답 fa; b; cg = f1; 3; 8g, f1; 4; 5g, f2; 2; 3g }

주 위의 (i), (ii)에서도 인수분해법 신에 부등식법을 계속 쓸 수도 있다. 예를 들어 (i)에서 b ¸ 3 이

면 (1 + 1b)(1 + 1

c) · 16

9< 2 이므로 해가 없고, 따라서 b = 2, 이것을 입하면 c = 3 이 된다. (ii)에서

도 b ¸ 5 이면 (1 + 1b)(1 + 1

c) · 36

25< 32 이므로 해가 없고 b · 2 일 때도 (1 + 1

b)(1 + 1

c) > 3

2 로 해가

없어서 b = 3 또는 4이고, 이것을 입하면 각각 c = 8, 5가 된다.

Page 43: 실전수학올림피아드 1400제 해답

1.2 수론 고급문제 43

40. 어떤 2000자리의 자연수의 모든 자리의 숫자의 합이 3이다. 또한 이 자연수는 두 완전제곱수의 합이다. 이러한 자연수를 모두 구하여라. (셈본중등중급 도전문제 2.2.2)

풀이 2000자리 자연수의 모든 자릿수 숫자의 합이 3이므로 이 수는 3의 배수이며, 9의 배수가 아니다.

이때 이 수(A라고 하자)가 다른 홀수 b, c의 제곱의 합이 된다고 하면 A = b2 + c2인데 b, c가 각각 3으로 나눈 나머지가 1, 2 중 하나라고 할 때 A의 3으로 나눈 나머지는 1 or 2가 되어 A가 3의 배수라는데모순이다.

따라서 b, c 모두 3의 배수여야 하는데 그럴 경우 A가 9의 제곱이 되어서 모순이다.

따라서 조건을 만족하는 A는 존재하지 않는다. }

41. m = 3; 4; 5 혹은 6일 때, 연속한 m개의 완전제곱수의 합은 완전제곱수가 될 수 없음을 보여라.(아일랜드 1991-6a)

증명 연속된 세 수는 0; 1; 2( mod 3)에 하나씩 응되므로 연속된 세 완전제곱수의 합은 항상 02 +

12 + 22 ´ 2( mod 3)이다. 그런데 임의의 완전제곱수는 0; 1( mod 3)의 값밖에 가질 수 없으므로, 연속한 3개의 완전제곱수의 합은 완전제곱수가 될 수 없다.

한편, ( mod 4)에 해 연속된 완전제곱수들을 살펴보면 1; 0; 1; 0; ¢ ¢ ¢ ( mod 4)의 반복된 수열이 된다.

이때 연속된 네 완전제곱수의 합은 2( mod 4)이고,연속된 다섯 완전제곱수의 합은 2또는 3( mod 4)이고, 연속된 여섯 완전제곱수의 합은 3( mod 4)이 된다. 그런데 임의의 완전제곱수는 0; 1( mod 4)의 값밖에 가질 수 없으므로 m = 4; 5; 6일때 연속한 m개의 완전제곱수의 합은 완전제곱수가 될 수 없다. ¤

42. 네 정수 a1, a2, b1, b2가 a1b2 ¡ a2b1 = §1 을 만족하면a1 + a2

b1 + b2가 기약분수임을 증명하여라.

(통신강좌 1995-11-1)

증명 (a1 + a2)b2 ¡ (b1 + b2)a2 = §1) (a1 + a2; b1 + b2) j §1) (a1 + a2; b1 + b2) = 1

즉, a1 + a2와 b1 + b2는 서로소이고,a1 + a2

b1 + b2는 기약분수이다. ¤

43.1

p+1

q+1

r=1

n을 만족하는 소수 p, q, r과 자연수 n을 모두 구하여라. (아벨콘테스트 1994 결선 2a)

풀이 p; q; r이 모두 다르다고 하자. 이때 통분하여 정리하면,

pqr = n(pq + qr + rp)

가 된다. (p; pq + qr + rp) = 1이므로 pjn이고, 같은 방법으로 qjn; rjn이다. 즉, n = pqr이어야 하고,pq + qr + rp = 1인데 p; q; r ¸ 2이므로 모순이다. 그러므로 서로 다른 해는 없다.p = q 6= r이라 해보자. 통분하여 정리하면,

pr = n(2r + p)

이다. 그런데 p가 2r+ p를 나누지 않으므로 pjn이고, 같은 방법으로 rjn이다. 이때 n = pr; 2r+ p = 1이어야 하는데 p; r ¸ 2이므로 모순이다. 즉, p = q = r인 해만 존재할 수 있다.p = q = r이라고 하면, 3n = p에서 p가 소수이므로 n = 1; p = 3이다. 결론적으로 n = 1; p = q = r =3의 해가 유일하다. }

44. 자연수 m, n이 서로소이다. m+ n 과 m2 + n2 의 최 공약수는 1이거나 2임을 증명하여라.(소련 1963-4)

증명 m + n과 m2 + n2이 소수 p > 2를 공약수로 갖는다고 하자 (앞으로 사용할 ´는 mod p에

한 것이라고 약속하자). 그러면, m + n ´ 0이다. 즉, m ´ ¡n이고, 양변을 제곱하면 m2 ´ n2이다. 한편, m2 + n2 ´ 0이므로 2m2 ´ 0이고 따라서 m ´ 0, 즉 m은 p의 배수이다. m ´ ¡n이므로 n도 p의 배수가 되고 이것은 m;n이 서로소라는 것에 모순이다. 즉, m+ n과 m2 + n2의 최 공약수는 1 또는 2이다. ¤

45. m = pq 이고 p와 q는 소수이다. (a;m) = 1 을 만족하는 모든 a에 해 am¡1 ´ 1 (mod m) 이라고 하자. p 6= q 임을 보여라. (통신강좌 1995-10-3 변형)

Page 44: 실전수학올림피아드 1400제 해답

44 수론

증명 p = q, 즉 m = p2이라 하자. 그러면 Á(p2) = p2 ¡ p이고 오일러의 정리에 의해 ap2¡p ´ 1(

mod p2)이므로

ap2¡1 ´ ap

2¡pap¡1 ´ ap¡1 ´ 1( mod p2)

이 성립한다. 이제 (a; p) = 1이면서 ap¡1 6´ 1( mod p2)인 a가 존재함을 보이면 충분하다. 어떤 b가(b; p) = 1이며 bp¡1 ´ 1( mod p2)를 만족시킨다고 하자. 이때 a = b+ p라 하면

(b+ p)p¡1 ´ bp¡1 + (p¡ 1)pbp¡2 ( mod p2)

´ 1¡ pbp¡2 ( mod p2)

이다. (b; p) = 1이므로 1¡ pbp¡2 6´ 1( mod p2)이고 따라서 (a; p) = 1이며 ap¡1 6´ 1( mod p2)인 a가존재한다. 즉, p 6= q이다. ¤

46. 상수다항식이 아닌 정수계수다항식 P (x)가 있다. 모든 정수 n에 하여 P (n)의 값이 항상 소수를 취하는것은 불가능함을 증명하여라. (통신강좌 1995-11-3)

증명 문제의 조건을 만족시키는 다항식 P (x)가 존재한다고 가정하자. 조건에 의해서 P (0)는 소수이

어야 하므로 그 값을 P0라 하자.

이때

P (kP0) ´ P (0) ´ 0 (mod P0)

이므로, P (kP0)도 P0의 배수이다. 그런데 P (x)는 항상 소수이므로, P (kP0)도 소수다. 즉 P0가 소수이므로 P (kP0) = P0이다. (k = 1; 2; ¢ ¢ ¢ )

F (x) = P (x)¡ P0

라 하자. x가 P0, 2P0, 3P0, ¢ ¢ ¢일때 F (x)는 0이고 P0 < 2P0 < 3P0 < ¢ ¢ ¢이므로, identity theorem에서

F (x) = 0

즉, P (x) = P0인데 이는 P (x)가 상수다항식이 아님에 모순.) 모든 정수 n에 해 P (n)이 항상 소수를 취하는 것은 불가능하다. ¤

47. 완전수란 자기 자신을 제외한 양의 약수의 합이 자기 자신과 같은 수를 말한다. 예를 들어, 6 = 1+ 2+ 3,28 = 1 + 2 + 4 + 7 + 14 이므로 6과 28은 완전수이다. 2p ¡ 1 이 소수일 때 2p¡1(2p ¡ 1)은 완전수이고,

짝수인 완전수는 이런 꼴들뿐임을 증명하여라. (플란더즈 예선 1990/1991)

증명 x = 2p¡1(2p ¡ 1), 2p ¡ 1은 소수라고 하자. 그러면 x의 약수의 합은

¾(x) = (1 + 2 + ¢ ¢ ¢+ 2p¡1)(1 + 2p ¡ 1)= (2p ¡ 1)2p = 2x

이 된다. 즉, x는 완전수이다.x가 짝수인 완전수라고 하자. 그러면 어떤 홀수 m과 k ¸ 2에 해 x = 2k¡1m꼴로 나타낼 수 있다. 이때,

¾(x) = (2k ¡ 1)¾(m) = 2km

이 되고, (2k ¡ 1; 2k) = 1이므로 (2k ¡ 1)jm이 된다.m = (2k ¡ 1)m0이라 하자. 그러면, 위의 식에서 m의 약수의 합

¾(m) = 2km0 = (2k ¡ 1)m0 +m0 = m+m0

이고, 따라서 m의 약수는 m과 m0밖에 없음을 알 수 있다. 곧 m0 = 1이고 m = 2k ¡ 1은 소수이다. ¤

48. 두 자리의 자연수 n = ab가 있다. n = a2 + b3 을 만족하는 모든 n을 구하여라. (1999 교육청경시)

Page 45: 실전수학올림피아드 1400제 해답

1.2 수론 고급문제 45

풀이 a2 + b3 = 10a+ b에서, a2 ¡ 10a+ (b3 ¡ b) = 0이 된다. 근의 공식에 의해

a = 5§q52 ¡ (b3 ¡ b)

가 된다. 여기서 근호 안의 내용이 0 이상이어야 하므로 0 · b · 3이 된다. 이때 52 ¡ (b3 ¡ b)가 제곱수가 되려면 b = 0; 1; 3의 경우 밖에 없다. b가 0 또는 1일때, a는 10 또는 0이어야 하는데, a는 0이 아닌 한자리수의 정수이므로 불가능하다. 즉, b = 3이어야 하고, 그러면 a는 4 또는 6이 된다. 그러므로 가능한n은 43과 63밖에 없다. }

49. k는 1335 + 1105 + 845 + 275 = k5 을 만족하는 정수이다. k는 얼마인가? (AIME 1989-9)

풀이 The last digits of n and n^5 are the same. Hence last digit of lhs

is same as that of 3 + 0 + 4 + 7 = 4. Hence last digit of k is 4. Also

133 = 1 mod 3, 110 = -1 mod 3, 84 = 0 mod 3, 27 = 0 mod 3, so lhs = 0 mod 3.

Obviously k > 133. So smallest possibility is 144, next is 174. Now

11^5 = (10 + 1)^5 = 10^5 + 5·10^4 + 10·10^3 + 10·10^2 + 5·10 + 1 = 161051,

so 110^5 < 2 10^10. Obviously, 27 and 84 < 100, so 27^5 and 84^5 < 10^10.

Similarly, 133^5 < (1331/10)^5 = 11^15/10^5 < 5 10^10. Hence lhs < 10^11.

But 170^2 = 28900 > 28000, 170^4 > 780000000 > 7 10^8 and 170^5 > 10^11.

So only possibility is 144.

¢ ¢ ¢ 답 144 }

50. x3 + 5y3 = 9z3 을 만족하는 정수해 (x; y; z)를 모두 구하여라. (아벨콘테스트 1994 결선 2b)

풀이 만일 해 x; y; z가 d > 1를 공약수로 갖는다면 x=d; y=d; z=d또한 해가 되기 때문에, (x; y; z) = 1인

해가 존재함을 알 수 있다. 자명하지 않은 해 x; y; z가 존재하여 서로소라고 가정하자.

정수는 3k; 3k+1; 3k¡ 1꼴의 세가지로 나눌 수 있는데 이를 세제곱하면 각각 27k3; 27k3+27k2+9k+

1; 27k3 ¡ 27k2 + 9k ¡ 1가 되므로 모든 정수의 세제곱은 mod 9에 해 0; 1;¡1의 값만을 가질 수 있다. 따라서 x3 + 5y3 ´ 0( mod 9)이려면 x; y가 모두 3의 배수일 수 밖에 없다. 이때 x3 + 5y3이 27의배수이므로 9z3도 27의 배수이고, 따라서 3jz3이다. 결국 3jz이고 x; y; z가 서로소라는 가정에 위배된다.

즉, x3 + 5y3 = 9z3의 정수해는 x = y = z = 0의 자명한 해 밖에 없다. }

51. x3 + y3 + z3 = 2002 를 만족하는 정수들의 순서쌍 (x; y; z)를 모두 구하여라. (통신강좌 1995-11-20)

풀이 (서울과학고 1학년 우지철) x3 + y3 + z3 = 2002 ´ 4 (mod 9)

a ´ 0 (mod 3)일 때 a3 ´ 0 (mod 9)

a ´ 1 (mod 3)일 때 a3 ´ 1 (mod 9)

a ´ 2 (mod 3)일 때 a3 ´ ¡1 (mod 9)

) x3 + y3 + z3 ´ 0; 1; 2; 3;¡1;¡2;¡3이므로 해는 존재하지 않는다. }

52. 1 +1

2+1

3+ ¢ ¢ ¢+ 1

n은 정수가 아님을 보여라. (n ¸ 2)

증명 n ¸ 2이므로 2k · n · 2k+1인 자연수 k를 찾을 수 있다. 이때 1; 2; ¢ ¢ ¢ ; 2k ¡ 1; 2k + 1; ¢ ¢ ¢ ; n의

최소공배수는 2k¡1 ¢m(m은 홀수)꼴이 된다.

1 +1

2+ ¢ ¢ ¢+ 1

n= H

라 하자. 양변에 2k¡1 ¢m을 곱하면

m£ 2k¡1µ1 +

1

2+ ¢ ¢ ¢+ 1

n

¶= Hm ¢ 2k¡1

이다. 이를 정리하면 m ¢ 2k¡1 ¢ 12k= m

2 를 제외한 좌변의 모든 항이 정수가 된다. m이 홀수이므로 이

항은 정수가 될 수 없고, 따라서 좌변은 정수가 아니다. 즉, H는 정수가 아니다. ¤

Page 46: 실전수학올림피아드 1400제 해답

46 수론

53. 순순환소수란 소수점 직후의 k개의 숫자 마디가 계속 반복되는 소수이다. 예를 들면 다음과 같은 것이다.

0:243243243 ¢ ¢ ¢ = 9

37

혼순환소수란 순환 마디가 있긴 하지만 순순환은 아닌 소수이다. 예를 들면 다음과 같은 것이다.

0:011363636 ¢ ¢ ¢ = 1

88

혼순환소수를 기약분수pq꼴로 나타냈을 때, 분모 q는 2나 5, 혹은 둘다로 나누어떨어짐을 증명하여라.

(미국 1988-1)

풀이 혼순환소수 0:a1a2 ¢ ¢ ¢ ak _b1b2 ¢ ¢ ¢ _bl를 기약분수 p=q로 나타냈다고 하자. 이 때

x = b1b2 ¢ ¢ ¢ bl ¡ a1a2 ¢ ¢ ¢ an

라고 하면,p

q10l ¡ p

q=

x

10k

가 된다. 이를 정리하면 p(10l ¡ 1)10k = qx 가 된다. p, q가 서로소이므로 p j x 이고, 따라서 정수x0 = x=p 에 해 (10l ¡ 1)10k = qx0 라 할 수 있다.q가 2나 5로 나누어떨어지지 않는다고 가정하자. 그러면 q와 10k는 서로소이다. 그러면 q j (10l ¡ 1) 이고, 10k j x0 이 된다. 곧 x=10k가 정수라는 것이고 주기가 a1a2 ¢ ¢ ¢ akb1b2 ¢ ¢ ¢ bl¡k 인 순순환소수가 되

어 가정에 위배된다. 그러므로 q는 2나 5로 나누어떨어진다. }

54. 이차방정식 x2 + ax¡ 66a = 0 의 두 근이 모두 정수일 때, 두 근을 구하여라. 단, a는 소수이다.(1995 서울시)

풀이 x2¡ax¡66a = 0의 근이 모두 정수이므로 판별식 D = a2+264a가 완전제곱수여야만 한다. a가

소수이므로 a(a+264)가 제곱수이려면 aj(a+264)이고 aj264이다. 264 = 23 ¢3 ¢11이므로 a = 2; 3; 11이고 각각에 해 a(a+ 264)를 계산하면 a = 11일때만 a(a+ 264) = 552으로 제곱수가 된다.

따라서 a = 11이고 두 근은 ¡33과 22이다. }

55. 모든 양의 실수는 십진법으로 전개했을 때 모든 자리의 수가 0 또는 7뿐인 9개의 수의 합으로 나타낼 수있음을 증명하여라. (Towns 1981 S3)

증명 먼저 모든 양의 실수는 십진법으로 전개했을 때 모든 자리의 수가 0 또는 1뿐인 9개의 수의 합으

로 나타낼 수 있음을 확인하자(¤). 십진법으로 전개한 임의의 양의 실수를

a = ¢ ¢ ¢ a3a2a1a0:a¡1a¡2a¡3 ¢ ¢ ¢

라 하자. 이제 0과 1로만 이루어진 9개의 수 b1; b2; : : : ; b9 을 정하는데(지수가 아니고 첨자임), bk는 십진법 전개로 a와 마찬가지로

bk = ¢ ¢ ¢ bk3bk2bk1bk0 :bk¡1bk¡2bk¡3 ¢ ¢ ¢

인 것으로 보자(k = 1; 2; : : : ; 9). 이 때, bki 들을 정하는 방법은 다음과 같다: b1i ; b2i ; : : : ; b

9i 중에서 정확히

ai개를 1로 한다(좀더 구체적으로 한다면, b11부터 b1ai까지를 1로 한다). 그럼

ai = b1i + b2i + ¢ ¢ ¢ b9i

가 되므로, a = b1 + ¢ ¢ ¢ + b9 도 성립함을 알 수 있다. (예를 들어, a = 320:1 이라면 b1 = 110:1,b2 = 110:0, b3 = 100:0, b4 = ¢ ¢ ¢ = b9 = 000:0 과 같이 하는 것이다.) 따라서, (¤)이 확인되었다.

이제 x를 임의의 양의 실수라 하면 x=7도 양의 실수이므로, (¤)에 의해

x

7= b1 + ¢ ¢ ¢+ b9

를 만족하는 각 자리가 0과 1로만 이루어진 b1; : : : ; b9가 존재한다. 이제 양변에 7을 곱하면 x를 각 자리가 0과 7로만 이루어진 9개의 수 7b1; : : : ; 7b9의 합으로 표현한 것이 된다. ¤

Page 47: 실전수학올림피아드 1400제 해답

1.2 수론 고급문제 47

56. 94000은 3817자리의 수이고 최고자리의 숫자가 9라는 것이 알려져있다. 0 · n · 4000 에 해 모든 9n꼴의 수들 중에 최고자리의 숫자가 9인 것은 모두 몇 개인가? (AIME 1990-13)

풀이 If 9^n starts with 9, then 9^{n-1} must start with 1 and have the

same number of digits. If 9^n starts with any other digit, then it has

one more digit than 9^{n-1}. Now 9^4000 has 3816 more digits than 9^1.

So in 3816 cases from 2, 3, ... , 4000, the first digit of 9^n is not 9

and in the other 183 cases it is 9. Add in the case 9^1 (but not 9^4000)

and we get 184.

¢ ¢ ¢ 답 184 }

57. 2진법으로 쓴 자연수에서 1을 모두 0으로, 0은 모두 1로 바꿔쓰는 조작을 반전이라고 하자. 또, 2진법으로쓴 자연수를 뒤에서부터 거꾸로 읽는 조작을 뒤집기라고 하자. 예를 들어, 2진수 11010010을 반전시킨 것과 뒤집은 것은 각각 101101과 1001011이다. 우리에게 허락된 조작이 이 두 가지뿐일 때,

(1) 387은 최소 몇 번의 조작으로 0으로 만들 수 있는가? 그리고,

(2) 5번 이상의 조작을 해야만 0으로 만들 수 있는 자연수 중에서 가장 작은 것은 무엇인가?(IT꿈나무 올림피아드 2006 1차)

풀이 0이나 1이 연속하여 나온 부분을 `덩어리'라고 부르기로 하자. 반전은 덩어리의 수를 1개 감소시

키고, 뒤집기는 덩어리의 수를 1개 감소시키거나 혹은 똑같이 유지시킨다. 따라서, 한 번의 조작으로 최1개의 덩어리를 없앨 수 있으므로, 덩어리가 n개인 수는 최소 n번의 조작이 있어야 0으로 만들 수 있

다. 387은 2진법으로 쓰면 110000011이므로 덩어리가 3개이다. 그리고, 덩어리가 5개인 자연수 중에서가장 작은 것은 10101, 즉 21이다. ¢ ¢ ¢ 답 (1) 3번 (2) 21 }

58. 자릿수의 합이 7의 배수가 되는 수를 행운의 수라고 부르기로 하자. 25 바로 다음에 나타나는 행운의 수는 34이고, 이럴 때 두 행운의 수 25와 34가 이웃해 있다고 말하기로 하자. 25와 34의 차는 9이다. 이렇게,

이웃한 두 행운의 수의 차가 될 수 있는 수를 모두 구하여라. (KAIST Cyber영재교육 2005 가을)

풀이 우선 13 이하의 모든 자연수는 다음과 같이 모두 이웃한 두 행운의 수의 차가 될 수 있다: 1 =70000 ¡ 69999, 2 = 61 ¡ 59, 3 = 500002 ¡ 499999, 4 = 403 ¡ 399, 5 = 7000000 ¡ 6999995,6 = 2005 ¡ 1999, 7 = 59 ¡ 52, 8 = 10006 ¡ 9998, 9 = 16 ¡ 7, 10 = 100006 ¡ 99996, 11 = 106 ¡ 95,12 = 1000006¡ 999994, 13 = 1006¡ 993. 이제 14 이상의 자연수는 이웃한 두 행운의 수의 차가 될 수없음을 보이자. 즉, M을 임의의 행운의 수라 하면 M +14 이전에 다른 행운의 수가 존재함을 보이면 된다(¤). 십의 자리 이상과 일의 자리로 구분하여 M = 10a + b 라 하자. b · 2 일 때는 M + 7 또한 행운의 수이다. b ¸ 3 일 때에는 10(a+1); 10(a+1)+ 1; : : : ; 10(a+ 1)+ 6 중에 행운의 수가 반드시 있으므로 그 수와 M과의 차이는 기껏해야 16¡ b · 13 이하이다. }

59. 자연수 n의 서로 다른 양의 약수 중 홀수인 것들의 개수를 N(n)으로 쓰기로 하자. 예를 들어, 24의 양의약수 중 홀수는 1, 3뿐이므로 N(24) = 2 이다. 합 N(1)+N(2)+ ¢ ¢ ¢+N(1989) 가 홀수인지 짝수인지 알아내어라. (호주 1989-1 변형)

풀이 N(n) = 홀수 , N = 2m® (® = 홀수부) 일 때 ® = 제곱수. ® = 1; 32; 52; 72; 92; : : : 에 해

좀 막노동을 뛰면... }

60. 1 + p+ p2 + p3 + p4 이 완전제곱수가 되는 소수 p를 모두 구하여라. (호주 1995-4)

풀이 부등식법: p ¸ 5 이면 (p2 + p¡12)2 < 준식 < (p2 + p+1

2)2 임을 이용. }

61. n개의 자릿수 a1; a2; : : : ; an이 임의로 주어져 있다. 제곱근의 소숫점 이하 십진전개가 이 n개의 숫자로(주어진 순서 로) 시작하는 자연수가 항상 존재하는가? (독일BW 1973 1차-3)

풀이 항상 존재함을 보이겠다. 0:a1a2 ¢ ¢ ¢ an = h 라 할 때 k + h <pm < k + h + 1

10n 을 만족하는

자연수 m, k가 존재함을 보이면 된다. 각 변을 제곱하면

(k + h)2 < m < (k + h)2 + 2(k + h) ¢ 1

10n+

1

102n

2(k + h) ¢ 110n 이 1보다 커지도록 k를 충분히 크게 잡자(k = 10n 쯤으로 잡으면 충분). 그럼 부등식의

좌변과 우변의 차이가 1보다 커지므로 그 사이에 정수 m이 존재함은 분명하다. 굳이 구체적으로 써준자면 m = b(10n + h)2 + 1c 로 잡으면 된다. }

Page 48: 실전수학올림피아드 1400제 해답

48 수론

62. 21917 + 1 에서 21991 ¡ 1 까지의 모든 정수의 곱이 제곱수가 아님을 증명하여라. (Towns 1992봄 SA1)

증명 n!의 소인수분해에서 소수 p의 지수는 f(n!; p) = bnpc + b n

p2c + b n

p3c + ¢ ¢ ¢ 로 구할 수 있음이

잘 알려져 있다. 특히 f(2n!; 2) = 2n¡1 + 2n¡2 + ¢ ¢ ¢ = 2n ¡ 1 의 간단한 형태로 얻을 수 있다. 그럼 문제에서 제시한 수 m = (21917 + 1) ¢ ¢ ¢ (21991 ¡ 1) 의 2의 지수는

f(m; 2) = f(21991!; 2)¡ f(21917!; 2)¡ 1991 = (21991 ¡ 1)¡ (21917 ¡ 1)¡ 1991

로 홀수가 된다. 따라서, m은 제곱수가 아니다. ¤

주 n과 2n 사이에는 항상 소수가 적어도 하나 있음이 베르트랑의 공준(Bertrand's Postulate)이라 하

여 잘 알려져 있다. 만일 이것을 사용할 수 있다면, m에 곱해진 수들 중 가장 큰 소수의 지수가 1임을 금방 알 수 있어서 문제가 너무 쉬워진다. 이것이 `어려운 정리'를 함부로 쓰지 말라는 경고를 한 이유이다.

베르트랑의 공준은 페르마의 작은 정리를 아주 교묘하게 적용하여 고등 경시 수준에서도 증명을 할 수있지만, 그 아이디어는 결코 쉽다고 할 수 없다. 따라서, 비교적 쉬운 수준에서 다른 방법으로 풀어낼 수있는 시험문제를 굳이 베르트랑의 공준으로 풀어낸다면, 별다른 경고 문구가 없더라도 충분히 상당한 감점이 적용되거나 심지어 점수를 거의 받지 못할 수 있으니 주의하여야 한다. 그것은 마치 빈 한 마리잡으려다 초가 삼간 불태우는 격이다. 삼각형의 넓이를 구하기 위해 미적분까지 사용한다거나, 아폴로니우스의 중선정리를 증명하라는데 더 어려운 스튜어트의 정리를 사용한다거나, 체바의 정리를 증명하라는데 동급의 메넬라우스의 정리를 사용한다거나 하는 것은 엉뚱한 일이 아니겠는가?

63. n이 1보다 큰 자연수이고 1 + 2n + 4n 이 소수이면, n은 3의 배수임을 보여라.(불가리아 1981 3차-4 변형)

증명 mod 7로 하면 됨. ¤

64. 2의 거듭제곱수 중에 자릿수를 재배열하여 또다른 2의 거듭제곱수를 만들 수 있는 것이 있는가? 단, 재배열할 때 맨 앞자리에 0이 나올 수 없다. (Towns 1988가을 SO1)

풀이 맨앞에 0이 나올 수 없다는 조건에 의해 하나가 다른 것의 10배 이상이 될 수 없으므로, 두 수는

(k; 2k), (k; 4k), 아니면 (k; 8k)가 된다. 그런데, 자릿수의 재배열로는 9로 나눈 나머지가 변하지 않는데,k는 2k, 4k, 8k 어느 것과도 9로 나눈 나머지가 같을 수 없으므로 모순(k는 2의 거듭제곱수이므로 9와서로소, 따라서 차인 k, 3k, 7k는 어느 것도 9의 배수가 될 수 없다). 따라서, 불가능. }

65. 다음을 증명하여라.

(a) n = 4k + 1꼴이면 합과 곱이 같은 n개의 양의 홀수들이 존재한다.

(b) n이 이런 꼴이 아니면 그런 홀수들도 존재하지 않는다. (Towns 1990봄 SO2)

증명 (a) k = 0 일 때는 자명. k ¸ 1 일 때는 2k + 1; 3; 1; : : : ; 1 (1이 4k ¡ 1개) 로 하면 곱은

3(2k + 1) = 6k + 3, 합도 (2k + 1) + 3 + (4k ¡ 1) = 6k + 3 이 된다. 따라서, 모든 음 아닌 정수 k에해 합과 곱이 같은 4k + 1개의 양의 홀수들이 존재한다.(b) 합과 곱이 같은 n개의 홀수들 중 4t ¡ 1꼴의 수가 b개라 하자. 그럼 mod 4로 곱 ´ (¡1)b 이고합 ´ b ¢ (¡1) + (n¡ b) ¢ 1 이므로 n ´ 2b+ (¡1)b (mod 4) 이다. 이 식은 b가 짝수일 때나 홀수일 때나둘다 n ´ 1 (mod 4) 가 됨을 확인할 수 있다. ¤

66. 21989과 51989을 10진법으로 이어썼다. 그럼 전체는 몇 자리 수가 되는가? (Towns 1989가을 SO4)

풀이 두 수를 각각 a자리 수, b자리 수라 하자. 그럼 10a¡1 < 21989 < 10a 이고 10b¡1 < 51989 < 10b

(10의 거듭제곱수가 아니므로 등호는 성립 불가). 따라서, 변변 곱하면 10a+b¡2 < 101989 < 10a+b 이고 a + b ¡ 2 < 1989 < a + b. a + b는 정수이므로 1989 = a + b ¡ 1 만 가능하다. 그럼 a + b = 1990¢ ¢ ¢ 답 }

67. N개의 정수가 주어져 있다. 이 중 임의의 N ¡ 1개의 곱에서 나머지 하나를 뺀 것이 항상 N의 배수이면,N개 전체의 제곱의 합도 N의 배수임을 증명하여라. (Towns 1991봄 JO1)

Page 49: 실전수학올림피아드 1400제 해답

1.2 수론 고급문제 49

증명 N의 배수인 항이 있으면 다른 항이 나머지 하나가 될 때 그 다른 항도 N의 배수. 즉, 모두 N의

배수이므로 OK. N의 배수인 항이 없으면 0도 일단 없고, 그럼 전체 곱을 P라 하면 P ¡ a2i 이 늘 N의

배수이므로 a2i ´ P (mod N) 으로 일정. 따라서, 다 더하면 N의 배수. ¤

68. 다음의 방정식을 만족하는 모든 자연수 n과 정수 x, y (x 6= y) 를 찾아라.

x+ x2 + x4 + ¢ ¢ ¢+ x2n= y + y2 + y4 + ¢ ¢ ¢+ y2

n

(Towns 1991봄 SO1)

풀이 넘기고 x¡ y로 나눠주면 1 + (x+ y) + (x+ y)(x2 + y2) + ¢ ¢ ¢+ (x+ y)(x2 + y2) ¢ ¢ ¢ (x2n¡1 +y2

n¡1) = 0. 여기서 x + y > 0 일 수 없고 x + y j 1 이므로 x + y = ¡1. 입하고 ¡1 곱하면

(x2 + y2) + (x2 + y2)(x4 + y4) + ¢ ¢ ¢ = 0. n ¸ 2 이면 이 식에 항이 남아있고 x 6= y 라서 x2 + y2 등이모두 양수이므로 좌변> 0 으로 모순. 즉, n = 1 이어야 하고, 그 때 x+ y = ¡1 인 모든 정수쌍에 해성립. }

69. 자릿수가 오름차순으로 나타나고(꼭 모두 다를 필요는 없고) 5로 끝나는 수들을 찾고 있다. 이 수들의 제곱도 역시 같은 성질을 가져야 한다.

(a) 이런 수를 4개만 찾아라.

(b) 이런 수가 무한히 많음을 증명하여라. (Towns 1991봄 JA3)

풀이 352 = 1225 를 확장하면 됨. 3352 = 90000 + 21000 + 1225 = 112225 등의 꼴을 관찰하여 귀납

법. }

70. 4545 + 5454 가 합성수임을 보여라. (러시아 1989 4차-y8-5)

증명 m4 + 4n4 은 인수분해되므로 합성수. ¤

71. 20052005는 두 완전제곱수의 합이지만 두 완전세제곱수의 합은 아님을 보여라. (아일랜드 2005-1)

증명 완전제곱수의 합임은 Lagrange 항등식으로부터. 세제곱수의 합이 아님은 mod 7로. ¤

72. m, k, n은 자연수이고 n > 1 이다. 이 때 m(m+ 1) = kn 이 성립할 수 없음을 보여라. (소련 1964-2)

증명 m과 m+ 1은 서로소이므로 각각이 n제곱수. 같은 n제곱수끼리 차이가 1인 경우는 자연수 범위

에서는 없음. ¤

73. 방정식 x4 + 131 = 3y4 을 만족하는 정수해 (x; y)가 존재하지 않음을 보여라. (호주 1984-1)

증명 mod 5로 해 없음. ¤

74. 각각의 자연수 n마다 음이 아닌 정수 f(n)을 배정하여 다음 조건이 만족되도록 하였다.

(i) 임의의 자연수 m, n에 하여 f(mn) = f(m) + f(n).

(ii) n의 마지막 자릿수가 3이면 f(n) = 0.

(iii) f(10) = 0.

f를 구하여라. (호주 1984-6 변형)

풀이 (iii)에서 f(2) = f(5) = 0, 즉 소인수 2와 5는 무시됨. 즉, 10과 서로소인 수들만 생각하면 됨. 그

럼 일의 자리가 1, 3, 7, 9 중의 하나이고, 이런 수는 적당한 홀수(3, 1, 9, 7)를 곱해 일의 자리를 3으로만들 수 있음. 그럼 (ii)에서 0. }

75. 자연수로 이루어진 세 무한등차수열이 있다. 1, 2, 3, 4, 5, 6, 7, 8 각각에 해 그 수가 나타나는 수열이적어도 하나씩은 있을 때, 1980이 나타나는 수열이 있음을 보여라. (오스트리아-폴란드 1980-1)

Page 50: 실전수학올림피아드 1400제 해답

50 수론

증명 2, 4, 6, 8을 포함하는 집합을 상으로 경우를 조금만 나누면; ¤

76. 세제곱의 합이 1991이 되는 두 정수가 존재하는가? (러시아 1991 4차-y9-5)

풀이 mod 7 }

77. 2x+1 + y2 = z2 을 만족하는 소수 (x; y; z)를 모두 구하여라. (러시아 1992 4차-y9-6)

풀이1 2x+1 = (z + y)(z ¡ y). y; z ¸ 5 일 때는 소수가 6k§ 1꼴뿐이므로 6 j (y+ z)(y¡ z). 모순. 나

머지만 검토하면 z = 5, y = 3, x = 3. }

풀이2 z + y = 2a, z ¡ y = 2b (a > b) 라 하면 2z = 2b(2a¡b + 1), 즉 z = 2b¡1(2a¡b + 1). 소수이므

로 b = 1, z = 5, y = 3. }

풀이3 mod 3 으로 그만. }

78. x3 + y5 = z7 을 만족하는 세 자연수의 순서쌍 (x; y; z)가 무한히 많음을 증명하여라. (IMTS R13-4)

증명 (230m35; 218m21; 213m15) ¤

79. 칠판에 2와 3이라는 수가 적혀있다. 그리고 이 칠판에 새로운 자연수를 보충해서 쓰는데, 단 이미 a와 b의수(a = b라도 상관없음)가 적혀있을 때 ab¡ a¡ b의 수를 쓰는 것을 규칙으로 한다. 이 칠판에 쓰여질 수있는 수를 모두 찾아라. (1999 KAIST 대전.충남 영재수학교실 2차평가 변형)

풀이 b = 2 넣으면 a! a¡ 2 를 쓸 수 있으므로, 2와 3으로부터 시작하여 3 이하의 모든 정수를 쓸 수

있고 b = 0 넣으면 a에 의해 ¡a를 항상 쓸 수 있으므로 모든 자연수를 쓸 수 있다.주 만일 문제의 의미가 칠판에 오직 자연수만 써야 하고 0이나 음수는 쓸 수 없는 거라면 1, 2, 3만 적은 후 더 이상 쓸 수 없다는 것이 답이 된다. }

80. c가 어떤 값이든, 방정식 x(x2 ¡ 1)(x2 ¡ 10) = c 가 다섯 개의 정수해를 가질 수 없음을 보여라.(러시아 1989 4차-y10-5)

증명 y = x(x2 ¡ 1)(x2 ¡ 10) 의 그래프와 y = c 의 그래프의 교점을 관찰하자고 따지면, c > 0 이면

(0; 1)에, c > 0 이면 (¡1; 0)에 근을 가지게 됨. ¤

별해 (김 현) 근과 계수와의 관계에서 다섯 해의 합은 0, 둘씩 곱한 합은 ¡11이므로, 제곱의 합은 22.

c = 0 일 땐 안 되고, c 6= 0 이면 일단 0, §1은 해가 아니므로 모든 해는 절 값이 2 이상. 즉 제곱이 4

이상. 모두 22일 때도 안 되고, 하나라도 32 이상이 있어도 안 되니까 끝. ¤

81. 방정식 x3 + y3 = 4(x2y + xy2 + 1) 이 정수해를 갖지 않음을 보여라. (러시아 1993 4차-y9-5)

증명 (x+ y)3 ´ 4 (mod 7) 이 됨. ¤

82. 음이 아닌 정수들 e1 > e2 > ¢ ¢ ¢ > er 에 해 n = 2e1 +2e2 + ¢ ¢ ¢+2er 이라 두자. n! = n(n¡ 1) ¢ ¢ ¢ 2 ¢ 1이라 할 때, n!=2n¡r은 홀수임을 보여라. (호주 1988-3)

증명 그냥 n!의 소인수분해 ¤

83. 자연수 n을 십진법으로 썼을 때 나타나는 0이 아닌 모든 자릿수들의 곱을 P (n)이라 하자. P (n)이 n을 나눌 때, n을 `굉자리한(prodigitious)' 수라 한다. 14개의 연속한 굉자리한 자연수들은 없음을 증명하여라.

(IMTS R16-2)

증명 일의 자릿수 k가 두 번 나오면 k j 10 이거나 k = 0. 이런 k들 중에 연속된 4개의 숫자는 없어서.

(0 1 2 5이므로 0 1 2의 3개 연속이 최 라서 아마 13개가 한계) ¤

Page 51: 실전수학올림피아드 1400제 해답

1.2 수론 고급문제 51

84. 십진법으로 모든 자릿수가 1인 자연수 n에 해서 p(n)이 다시 모든 자릿수가 1인 자연수가 되는 이차 정계수 다항식 p가 존재하는가? (러시아 1994 4차-y9-3)

풀이 1-반복수 다항식. 9x2 ¡ 2x+ 1 }

85. a는 임의의 양의 정수이다. x+ 12(x+ y ¡ 1)(x+ y ¡ 2) = a 를 만족하는 양의 정수 x, y가 유일하게 존

재함을 증명하여라. (헝가리 1936-3)

증명 유사진법. x + y ¡ 1 = n(¸ 1) 으로 두면 1부터 n ¡ 1까지의 합을 f(n)이라 할 때 f(n) < a ·f(n+ 1) 을 만족하는 n이 유일하게 존재할 테고 x = a¡ f(n) · n 은 그 편차. ¤

86. (a1; a2; : : : ; an)과 (b1; b2; : : : ; bn)는 각각 1; 2; : : : ; n을 재배열한 것이다. n이 짝수이면, ai + bi 들(i =1; 2; : : : ; n) 중에 n으로 나눈 나머지가 같은 것이 있음을 보여라. (몰도바 1997 최종-y8-5)

증명 나머지가 모두 다르다면, 모두 합했을 때 2(1 + 2 + ¢ ¢ ¢ + n) ´ 0 + 1 + ¢ ¢ ¢ + (n ¡ 1) (mod n),

즉n(n¡1)2

´ n(n+ 1) ´ 0 (mod n). 그럼n(n¡1)2

= nk, 즉 n = 2k + 1 이므로 모순. ¤

87. n!의 자릿수의 합이 9인 자연수 n을 모두 구하여라. (유고슬라비아 1979 고1-4)

풀이 9의 배수여야 하므로 n ¸ 6. 만일 11의 배수라면, 홀수번째 자릿수의 합을 A, 짝수번째 자릿수

의 합을 B라 할 때 A ´ B (mod 11). A = B 라면 A + B가 짝수이므로 9일 수 없고, A 6= B 이면큰 쪽이 11보다 크므로 A + B도 11보다 커서 불가능. 즉, n < 11 일 때만 보면 되고, 실제 계산해보면n = 6; 7; 8 만 답이 됨. }

88. 다음 방정식의 정수해를 모두 구하여라. (폴란드 1966 3차-2)

x4 + 4y4 = 2(z4 + 4u4)

풀이 x; z; y; u가 차례로 짝수가 되어 무한내림으로 모두 0뿐. }

89. xy = yx¡y 을 만족하는 자연수 순서쌍 (x; y)를 모두 구하여라. (주니어발칸 1998-3)

풀이 y = 1 이면 x = 1. x = y 일 때도 x = 1. 따라서, x > y ¸ 2 일 때만 보자. y = mnx 입하

고(m < n 은 서로소) 양변을 nx제곱하면 xm = (m

nx)n¡m. 즉 x = (m

n)n¡m2m¡n 이고 y = (m

n)

m2m¡n .

(단, 2m¡ n = 0 일 때는 (m;n) = (1; 2) 이고 그럼 1 = 12 로 성립하지 않음) (i) 2m¡ n > 0 이면 y쪽

의 지수가 0보다 크므로 정수가 되려면 n = 1. 그럼 m < n 임에서 모순. (ii) 2m¡n < 0 이면 y쪽의 분

자와 분모를 바꿔 생각할 때 m = 1. x = nn¡1n¡2 , y = n

1n¡2 이고 n ¸ 3. n = yn¡2 ¸ 2n¡2 에서 n ¸ 5

이면 이 부등식은 성립하지 않으므로 n = 3; 4 일 때만 확인하면 되고, 각각 (x; y) = (9; 3), (8; 2). ¢ ¢ ¢답 (x; y) = (1; 1), (8; 2), (9; 3). }

90. 음이 아닌 정수 n에 하여 An = 23n + 36n+2 + 56n+2 으로 정의하자. A0; A1; : : : ; A1999 의 최 공약수를 구하여라. (주니어발칸 1999-2)

풀이 A0 = 1+9+25 = 35 이므로 5와 7에 해서만 살피면 됨. 5 - A1 이므로 5는 아님. 23; 36; 56 ´ 1(mod 7) 이므로 An ´ 1 + 9 + 25 ´ 0 (mod 7) 으로 7은 항상 됨. ¢ ¢ ¢ 답 7 }

91. n2 + 3n 이 어떤 정수의 제곱이 되는 자연수 n을 모두 구하여라. (주니어발칸 2000-2)

풀이 3n = (m + n)(m ¡ n) 에서 m + n = 3a, m ¡ n = 3b 꼴일 수밖에 없고(a + b = n, a > b),

2p · n · 2p+ 1 일 때 2n = 3a ¡ 3b ¸ 3p+1 ¡ 3p = 2 ¢ 3p. 즉, 2p+ 1 ¸ 3p 인데, 이것은 p > 1 일 때3p = (1 + 2)p > 1 + 2p 이므로 모순. p = 0; 1 일 때만 확인하면 n = 1; 3 만 답이 됨. }

92. x, y가 자연수이고 3x+ 4y, 4x+ 3y 가 모두 완전제곱수이면 x와 y는 모두 7의 배수임을 증명하여라.(주니어발칸 2004-3)

Page 52: 실전수학올림피아드 1400제 해답

52 수론

증명 m2+n2 ´ 0 (mod 7)인데,제곱수는mod 7로 0, 1, 2, 4뿐이므로 만족하는 경우는 m, n 모두 7의

배수일 때뿐. 그럼 72 j m2+n2 = 7(x+y) 이므로 7 j x+y 이고, 한편 7 j 2(4x+3y)¡7(x+y) = x¡y.

따라서, 7 j 2x; 2y, 즉 7 j x; y. ¤

93. m과 n이 서로 다른 자연수일 때, 22m+ 1 과 22

n+ 1 은 서로소임을 증명하여라. (헝가리 1940-2)

증명 m > n 이라 하고 공통소인수 p가 존재한다고 하자. p j 22n + 1 이므로 p j 22n+1 ¡ 1 =(22

n+1)(22

n ¡ 1), 마찬가지로 p j 22n+2 ¡ 1 = (22n+1 ¡ 1)(22n+1 +1), 이렇게 계속하면 p j 22m ¡ 1이 되고, p j 22m + 1 과 빼면 p j 2, 즉 p = 2. 이것은 모순. ¤

94. 서로 다른 두 자연수 x, y의 산술평균은 두 자리의 수이다. 이 산술평균의 십의 자릿수와 일의 자릿수의자리를 서로 바꾸면 x, y의 기하평균이 된다.

(a) x, y를 구하여라.

(b) 10진법에서 해는 유일함을 보이고, 12진법에서는 해가 없음을 보여라.

(c) 해가 있는 진법과 해가 없는 진법의 예를 하나씩 더 구해보아라. (독일BW 1972 2차-3)

풀이 (a) 1 · a · 9, 1 · b · 9, a와 b는 자연수, x > y 라 가정한다. 12 (x + y) = 10a + b,pxy = 10b + a. 산술평균은 항상 기하평균 이상이고 a 6= b 이므로 a > b. 산술평균에서 x =

2(10a+ b)¡ y. 이를 기하평균에 입하여 정리하면 y = (10a+ b)§ 3p11(a+ b)(a¡ b). 똑같은 방법

으로 x = (10a+ b)§ 3p11(a+ b)(a¡ b). x 6= y 이고 x > y 이므로

x = (10a+ b) + 3p11(a+ b)(a¡ b)y = (10a+ b)¡ 3

p11(a+ b)(a¡ b)

1 · a; b · 9 이므로 a¡ b의 최 값은 8, a+ b의 최 값은 17, 그리고p11(a+ b)(a¡ b)는 자연수여야

하므로 a+ b = 11. 이런 순서쌍 (a; b)를 구하여 비교하면 오직 a = 6, b = 5 만이 식을 만족. 계산하면x = 98, y = 32.(b) 10진법에서는 오직 (6,5)만이 (a; b)를 만족하므로 이미 끝. 12진법의 수 x, y, a, b에서는 1 · a; b ·11, 1

2(x+ y) = 12a+ b,

pxy = 12b+ a 이므로 (a)에서와 똑같은 과정을 거칠 때

x = (12a+ b) +p13 ¢ 11(a+ b)(a¡ b)y = (12a+ b)¡

p13 ¢ 11(a+ b)(a¡ b)

1 · a; b · 11에서 (a+ b)(a¡ b)는 13 ¢11 을 인수로 가져야 하는데 (a+ b)(a¡ b) = a2¡ b2 · 112¡1 <13 ¢ 11 로 불가능하므로 위의 식을 만족시키는 순서쌍 (a; b)는 존재하지 않음.(c) 일반적인 n진법의 수 a, b, x, y에서 1 · a; b · n ¡ 1, 1

2(x + y) = na + b,

pxy = nb + a 이므로

(a)와 똑같은 과정을 거칠 때

x = (na+ b) +p(n+ 1)(n¡ 1)(a+ b)(a¡ b)y = (na+ b)¡

p(n+ 1)(n¡ 1)(a+ b)(a¡ b)

(a+ b)(a¡ b) · a2¡ b2 · (n¡ 1)2¡ 1 은 무조건 (n+1)(n¡ 1)보다 작으므로 (n+1)(n¡ 1)은 1을 제외한 완전제곱수를 인수로 가져야함. 이에 맞는 조건의 n값은 8 등이 있음(그리고, 8일 때 a = 4, b = 3,x = 56, y = 14 의 해가 실제 존재). 이에 맞는 않는 조건의 n값은 7 등이 있음. }

95. p가 소수이면 7p+ 3p ¡ 4 는 완전제곱수가 아님을 증명하여라. (주니어발칸 2007-4)

증명 p ¸ 5 일 때 먼저 mod p로 ¡1임을 보여 p = 4k + 1꼴임을 확인하고, 그것을 이용해 mod 4로

2가 됨을 확인. ¤

96. 자연수 n에 하여, 다음을 만족하는 실수 x (1 · x < n) 의 개수를 구하여라.

x3 ¡ bx3c = (x¡ bxc)3

(단, bxc는 x를 넘지 않는 최 의 정수이다.) (호주 1992-2)

풀이 x = n+ h 라 하고 입해서 정리하면 3nh(n+ h)가 정수일 때. }

Page 53: 실전수학올림피아드 1400제 해답

1.2 수론 고급문제 53

97. K와 L은 자연수이다. 적당한 자연수 M이 존재하여, M보다 큰 자연수 n에 해서는µK +

1

2

¶n+

µL+

1

2

¶n이 정수가 될 수 없음을 보여라. (호주 1992-4)

증명 2n으로 통분하고 분자만 생각. n이 짝수일 때는 mod 4로, n이 홀수일 땐 인수분해했을 때 복잡

한 항이 홀수... ¤

98. 다음을 만족하는 모든 정수해 (x; y)를 구하여라. (호주 1993-5)

(x+ 2)4 ¡ x4 = y3

풀이 y는 짝수이고 인수분해하면 (x + 1)((x + 1)2 + 1) = y31 . 좌변 두 항이 서로소임을 이용. mod

8도 도움이 될 수도. }

99. 자연수 a, b가 주어졌을 때, 방정식 xa+b + y = xayb 을 만족하는 자연수 x, y를 모두 찾아라.(오스트리아 1983-5)

풀이 y = kxa 입하고 정리. 다시 k = mxb 입하고 정리. 1 = m(xab ¡ 1). m = 1, x = 2,

a = b = 1 일 때만 해 있음. }

100. 1; 12; 123; 1234; : : : ; 1234567890; 12345678901; : : : 의 수들 중에 적어도 하나는 1981로 나누어떨어짐을보여라. (유고슬라비아 1980 고1-2)

증명 1234567890(1 + 1010 + ¢ ¢ ¢ + 1010k) 중에 1981의 배수가 있음을 보이면 충분. 즉, 1 + 1010 +

¢ ¢ ¢+ 1010k중에 1981의 배수가 있음을 보이면 충분. 1010, 1010 ¡ 1 이 모두 1981과 서로소임을 확인했으면 이건 금방... ¤

101. 다음 방정식을 만족하는 음이 아닌 정수들의 순서쌍 (a; b; c; d)를 모두 구하여라.

a2 + b2 + c2 + d2 = a2b2c2 (오스트리아 1986-1)

풀이 모두 홀수면 안 되고, d만 짝수여도 mod 8로 안 되니까 a, b, c 중에 짝수가 존재. 그럼 우변은

4의 배수. 좌변이 4의 배수이려면 모두다 짝수. 그럼 무한내림... }

102. q가 정수일 때, 이차방정식 x2 + 7x¡ 14(q2 + 1) = 0 은 정수해를 갖지 않음을 보여라.(인도지역예선 1995-4)

증명 mod 7, 근과 계수 ¤

103. 다음 식을 만족하는 자연수 순서쌍 (n;m)을 모두 찾아라. (오스트리아-폴란드 1982-1)

gcd((n+ 1)m ¡ n; (n+ 1)m+3 ¡ n) > 1

풀이 p를 최 공약수의 한 소인수라 하자. n+ 1, n 모두 p의 배수가 아님. p j (n+ 1)3 ¡ 1 을 얻을 수

있음. 그담엔 m을 mod 3으로 분류해서 풀면 m = 3k 일 때 (m;n) = (3k; 7l + 1)꼴만 답이 됨. }

104. a2+ b2 = c2 을 만족하고, a, b, c의 최 공약수가 1인 세 정수의 쌍 (a; b; c)가 무한히 많음이 알려져있다.(rs)2 + (st)2 = (tr)2 을 만족하고, r, s, t의 최 공약수가 1인 세 정수의 쌍 (r; s; t)도 무한히 많음을 증명하여라. (IMTS R41-2)

증명 (r; s; t) = (bc; ca; ab) 의 치환이면 끝. ¤

105. n은 자연수이다. 2n + 1과 3n + 1 모두 완전제곱수일 필요충분조건은 n + 1이 연속된 두 제곱수의 합인동시에, 한 제곱수에다 그 바로 다음 제곱수의 2배를 더한 수임을 보여라. (호주 1994-6)

Page 54: 실전수학올림피아드 1400제 해답

54 수론

증명 (1) 동치변형: 2n + 1 = (2m + 1)2 () n + 1 = m2 + (m + 1)2. (2) 동치변형: 3n + 1 =(3k ¡ 1)2 = (¡3k + 1)2 () n+ 1 = (k ¡ 1)2 + 2k2. ¤

106. p2 + 7pq + q2 이 완전제곱수가 되는 소수 p, q를 모두 구하여라. (인도지역예선 2001-2)

풀이 (류길현)

p2 + 7pq + q2 = k2

(p+ q)2 + 5pq = k2

k2 ¡ (p+ q)2 = 5pq

(k + p+ q)(k ¡ p¡ q) = 5pq

k + p+ q > p; q; 5임이 자명하므로 다음의 경우들만 생각하면 된다.

(1) k ¡ p¡ q = 1, k + p+ q = 5pqk = p+ q + 12p+ 2q + 1 = 5pq(5p¡ 2)(5q ¡ 2) = 9(p; q) = (1; 1)Ã소수가 아니라서 안 됨.

(2) k ¡ p¡ q = 5, k + p+ q = pqk = p+ q + 52p+ 2q + 5 = pq

(p¡ 2)(q ¡ 2) = 9(p; q) = (3; 11); (5; 5); (11; 3)

(3) k ¡ p¡ q = p, k + p+ q = 5qk = 2p+ q3p+2q = 5q ! p = q일 때 가능(k¡ p¡ q = q일 때는 p, q를 바꾸면 되니까 이것과 결과가 같다.)

답: (p; q) = (3; 11); (11; 3); (r; r) (r은 소수) }

107. 합이 소수이고, p2 + qs 와 p2 + qr 이 서로 다른 완전제곱수인 소수 p, q, r, s를 모두 구하여라.(러시아 1994 4차-y9-7)

풀이 합은 2보다 큰데 소수라 했으므로 홀수이고, 그럼 p, q, r, s 중에 2가 홀수개 있음. 2가 3개인 경

우에는, p2 + qs 와 p2 + qr 이 서로 다르므로 r 6= s. WLOG, 2가 아닌 것을 s라 하면, p2 + qs = 8 이므로 모순. 따라서, 2가 1개인 경우만 보면 됨. p 6= 2 라면 p2 + 2a꼴의 완전제곱수가 있는데(a는 q, r,s 중 하나이고 2가 아님), 2a는 홀제곱수의 차이므로 4의 배수, 즉 a가 짝수이고 이것은 모순. 따라서,p = 2 일 때만 보면 됨. 4 + qs = a2, 4 + qr = b2 이라 하면 qs = (a¡ 2)(a+ 2), qr = (b+ 2)(b¡ 2). q,s는 소수이므로 qs = 1 ¢ qs 또는 q ¢ s꼴로만 분해됨. 전자의 경우에는 qs = 5 가 되어 모순이므로 후자의 경우임. qs도 마찬가지. 따라서, jq ¡ sj = 4 = jq ¡ rj. r 6= s 라 했으므로 WLOG: r < q < s는 공차4의 등차수열. 그럼 셋 중 하나는 3의 배수이므로 r = 3 일 수밖에 없고 그럼 q = 7, s = 11. 이 경우 조건 모두 만족함. ¢ ¢ ¢ 답 (p; q; r; s) = (2; 7; 3; 11) 또는 (2; 7; 11; 3) }

108. 방정식 x2 + x = y4 + y3 + y2 + y 의 정수해 x, y를 모두 구하여라. (소련 1967-6)

풀이 4배하고 1더하면 좌변은 (2x+ 1)2 이고 우변은 (2y2 + y)2과 (2y2 + y + 2)2의 사이. 즉 부등식

법으로 (2x+ 1)2 = (2y2 + y + 1)2 이 유일한 가능성. }

109. 모든 자연수 x에 하여, 자연수 x를 십진법으로 나타내었을 때 각 자리의 숫자들을 모두 합한 것을 Q(x)

라고 하고 모두 곱한 것을 P (x)라고 하자. 이 때, 모든 자연수 n에 하여

Q(Q(x)) + P (Q(x)) +Q(P (x)) + P (P (x)) = n

를 만족하는 x는 무한히 많음을 보여라. (오스트리아 1983-1)

증명 rn =19(10n¡ 1) = 11 ¢ ¢ ¢ 1 (1이 n¡1개) 로 정의하고, x = 1

9(10rn ¡1) ¢ 10m = 11 ¢ ¢ ¢ 1 00 ¢ ¢ ¢ 0

(1이 rn개, 0이 임의의 m개) 로 두자. 그럼 Q(x) = rn 이고 P (x) = 0. Q(Q(x))+P (Q(x))+Q(P (x))+P (P (x)) = (n¡ 1) + 1 + 0 + 0 = n 이다. ¤

Page 55: 실전수학올림피아드 1400제 해답

1.2 수론 고급문제 55

110. 다음은 9에서 시작하여 자릿수를 두 배씩 불려나가며 곱한 수이다.

9£ 99£ 9999£ ¢ ¢ ¢ £ (102n ¡ 1)

이 계산 결과의 각 자릿수의 합을 (n에 한 식으로) 구하여라. (미국 1992-1)

풀이 점화식. ¢ ¢ ¢ 답 9 ¢ 2n }

111. 다음과 같은 성질을 갖는 자연수 a, b, c, d가 존재하도록 하는 자연수 순서쌍 (x; y; z) 중에서 z가 최소인것을 찾아라.

(i) xy = ab = cd 이고 x > a > c.

(ii) z = ab = cd.

(iii) x+ y = a+ b. (오스트리아-폴란드 1982-7)

풀이 x = an, a = cm (m;n ¸ 2). 그럼 d = bm. c는 1일 수 없으므로 c ¸ 2. (i)에 입해서 풀면

cm¡1 ¸ 2m¡1 ¸ m 이 되어 등호는 c = 2, m = 2 일 때만 성립. a = 4. x = 2k (k ¸ 3). k = 3 일 때체크해보면 b = 12 일 때 z가 최소값 48을 가짐. k = 4; 5 일 때 체크해보면 z값이 더 커짐. }

112. 십진법에서 xyxy꼴의 수는 거듭제곱수(제곱수, 세제곱수, 네제곱수, ...)가 될 수 없음을 보여라. 한편,b진법에서 xyxy꼴의 수가 거듭제곱수가 되는 경우가 있는 가장 작은 b > 1 를 구하여라.

(아일랜드 1998-3 변형)

풀이 101 j xyxy 이므로 1012 j xyxy < 104. 이것은 모순. b진법에서는 b2 + 1 이 무승수(square-free

number)이면 똑같은 모순이 발생. b = 2; 3; 4; ::: 을 차례로 확인해보면 무승수가 아닌 것은 b = 7 이 최초. 이 때, 2626(7) = 1000 = 10

3 으로 성립. }

113. 정확히 16개의 약수 1 = d1 < d2 < ¢ ¢ ¢ < d15 < d16 = n 을 갖고, d6 = 18, d9 ¡ d8 = 17 인 자연수 n을모두 구하여라. (아일랜드 1998-6)

풀이 18 j n. 18의 약수가 벌써 1, 2, 3, 6, 9, 18로 6개이므로 6번째 약수가 18이려면 n의 18이하의 약수

는 이것이 전부.그럼 4 - n 이므로 n = 2132+bprqs¢ 꼴이고,약수의 개수는 16 = 2(3+b)(1+r)(1+s) ¢ ¢ ¢꼴.가능한 것은 2¢4¢2 = 2¢8이 전부. 2¢8은 n = 2137 인데 1, 2를 제외하고는 약수가 모두 3의 배수이므로 3 j d9¡d8 = 17에서 곤란. 즉 2¢4¢2뿐이고, n = 2¢33 ¢p꼴. p ¸ 19. fd7; d8; d9; d10g = f27; 54; p; 2pg이고 여기서 경우를 잘 따져보면 p = 37. ¢ ¢ ¢ 답 2 ¢ 33 ¢ 37 }

114. 7x ¡ 3 ¢ 2y = 1 을 만족하는 자연수 x, y를 모두 구하여라. (러시아 1990 4차-y11-5)

풀이 x = 1 일 때는 y = 1. x ¸ 2 이면 y ¸ 3 이고 그럼 mod 8로 x는 짝수. (7X +1)(7X ¡1) = 3 ¢2y에서 7X + 1 = 2a+1, 7X ¡ 1 = 3 ¢ 2b+1꼴. 2a ¡ 3 ¢ 2b = 1. 홀수이므로 b = 0. 그럼 a = 2, X = 1,y = 4, x = 2. }

115. 자연수 n에 해 an =hn1

i+hn2

i+ ¢ ¢ ¢+

hnn

i이라고 정의하자. n이 소수일 때, 또 그 때만 an = 2+an¡1

임을 증명하여라. (몰도바 1997 최종-y10-8)

증명 가우스 항 중에서 새로 다음 정수가 되는 것들의 개수만큼 추가되므로 an = an¡1 + dn. ¤

116. 다음 방정식을 만족하는 정수쌍 (x; y)를 모두 구하여라. (유고슬라비아 1981 고2-3)

y4 ¡ x(x+ 1)(x+ 2)(x+ 3) = 1

풀이 x = 0;¡1;¡2;¡3 일 때는 y = §1. y4 = (x2 + 3x+ 1)2 이므로 x · ¡4 이거나 x ¸ 1 일 때는

(x+ 1)4 과 (x+ 2)4 사이에 낀다는 부등식법. }

Page 56: 실전수학올림피아드 1400제 해답

56 수론

117. a는 정수이다. 방정식x2 + axy + y2 = 1

이 무한히 많은 서로 다른 정수해 (x; y)를 갖도록 하는 a를 모두 구하고 그것을 증명하여라.(아일랜드 1995-2)

증명 jaj · 1 일 때는 각각 특수해를 구해주고, jaj ¸ 2 일 때를 풀자. a 신 ¡a를 입한 식과 원래

의 식의 해가 일 일 응(y ! ¡y)하므로 ¡a = b > 0 일 때만 보아도 충분. (x; y) = (1; b) 의 특수해로부터 (x; y)! (y; ay ¡ x) 의 점화법으로 새로운 해를 계속 만들 수 있다. ¤

118. 자연수 n을 십진법으로 쓴 각 자릿수의 합을 S(n)으로 나타내자. 모든 n에 해 다음을 증명하여라.

S(2n) · 2S(n) · 10S(2n)

또한, S(n) = 1996S(3n) 을 만족하는 자연수 n이 있음을 증명하여라. (아일랜드 1996-2)

증명 m+ n 을 계산할 때 자리올림이 한 번도 없으면 S(m+ n) = S(m) + S(n). 각 자릿수는 2배하

면 앞자리에 많아야 1이 올라가고, 앞자리는 2배가 되어 짝수가 되었으니 1을 받아도 자리올림이 발생하지 않는다. 즉, 예를 들어 세 자리의 수 n = 100a+ 10b+ c 라 하면 200a, 20b, 2c를 합할 때 자리올림이 생기지 않으니 S(2n) = S(200a+ 20b+ 2c) = S(200a) + S(20b) + S(2c) = S(2a) + S(2b) + S(2c).S(2a) = 2a 또는 2a ¡ 9 이므로 S(2a) · 2a 이다. 이로부터 S(2n) · 2a + 2b + 2c = 2S(n). 한편,ai (0 · i · k) 를 n의 각 자릿수라 하고, ai가 5 이상일 때 °i = 1, 그렇지 않으면 °i = 0 라 할 때,5S(2n) = 5

Pi S(2ai) = 5

Pi(2ai ¡ 9°i) =

Pi ai +

Pi 9(ai ¡ 5°i) ¸ S(n) 이다. ¤

119. 양의 약수의 개수의 네제곱이 자기 자신이 되는 모든 자연수를 구하여라. (아일랜드 1999-9)

풀이 소인수분해에서 지수가 모두 4의 배수. 네제곱근한 등식의 양변을 비교. 소인수가 7 이상이면

pe > 4e+ 1. 따라서, 소인수 3, 5의 지수들을 살펴 경우를 좀 따져주면 됨. }

120. 피보나치 수열은 F0 = F1 = 1, 그리고 n ¸ 0 에 해 Fn+2 = Fn+1 + Fn 으로 정의된다. 피보나치 수열에서 연속한 2000개의 항의 합은 피보나치 수가 될 수 없음을 증명하여라. (몰도바 2000 최종-y7-7)

증명 Fn+Fn+1+¢ ¢ ¢+Fn+1999 = Fn+2+Fn+4+¢ ¢ ¢+Fn+1998+Fn+2000 = X 라 하면 X+Fn+1 =

Fn+2001 이 됨을 귀납적으로 확인할 수 있고, 그럼 Fn+2000 < X < Fn+2001 이라서. ¤

121. 다음을 만족하는 모든 자연수해 (a; b; c; n)을 찾아라. (아일랜드 2001-1)

2n = a! + b! + c!

풀이 WLOG a · b · c 라 하면 a! j 2n 이므로 a · 2. (i) a = 1 이면 우변이 짝수가 되기 위해 b = 1

이고, 그럼 c! = 2n¡ 2 = 2(2n¡1¡ 1) 은 2 ¢홀수 이므로 c = 2; 3. 해는 (1; 1; 2; 2), (1; 1; 3; 3). (ii) a = 2

이면 우변이 4의 배수가 되기 위해 b = 2; 3. b = 2 이면 2n¡2 = 1 + c!4 이고, c!4 가 홀수가 될 수 없어서

모순. b = 3 이면 2n¡3 = 1 + c!8 이고, c!8이 홀수인 경우는 c = 4; 5뿐. 해는 (2; 3; 4; 5), (2; 3; 5; 7). ¢ ¢ ¢

답 (1; 1; 2; 2), (1; 1; 3; 3), (2; 3; 4; 5), (2; 3; 5; 7) }

122. 0이 아닌 자릿수가 딱 둘뿐이고 그 중 하나는 3인 완전제곱수를 모두 찾아라. (중미 2001-3)

풀이 마지막 0은 짝수개. 102n으로 나누어주면, 일의 자리가 3일 수는 없으므로, 300 ¢ ¢ ¢ 0x꼴. 문제 조

건에 의해 x는 0은 안 되고, mod 10에서 2, 3, 7, 8도 안 됨. 현재 x = 1; 4; 5; 6; 9가 남았는데, mod 25에서(혹은 mod 3에서) 5도 안 됨. mod 9에서 9도 안 됨. x = 6 이면 유명한 명제에 의해 십의 자리가 홀수,

즉 36만 가능하고 이것은 실제로 됨. x = 1; 4의 경우만 남았다. (i) x = 1 일 때: 3 ¢10n = (k+1)(k¡1),차가 2인 정수이므로 둘다 5의 배수일 수 없어서 5n은 한쪽에 몰림. 또 둘다 짝수. 그럼 한쪽의 최소값2 ¢ 5n은 반 쪽의 최 값 3 ¢ 2n¡1보다 훨씬(차이가 2 초과) 크므로 해가 없음. (ii) x = 4 일 때: 역시

3 ¢ 10n = (k + 2)(k ¡ 2) 임에서 2 ¢ 5n은 3 ¢ 2n¡1보다 훨씬 큼. ¢ ¢ ¢ 답 36 }

123. a1 = A, an+1 = an + d(an) 으로 주어지는 수열 a1; a2; a3; : : : 이 있다. 단, d(m) 은 m의 자신을 제외한가장 큰 약수를 나타낸다. 이 수열에 2002가 나타나도록 하는 정수 A > 1 은 어떤 값인가? (중미 2002-3)

Page 57: 실전수학올림피아드 1400제 해답

1.2 수론 고급문제 57

풀이 an+1 = 2002 라 하고 an=d(an) = p 라 하면 2002 = (p+ 1)d(an) 이므로 p+ 1 j 2002. 2002의양의 약수들 중에서 1를 뺐을 때 소수가 되는 것을 찾으면 14와 182뿐, 즉 p = 13 또는 181. 그런데, 이때 둘다 11 j an 이라서 p가 an의 가장 작은 소인수임에 모순. ¢ ¢ ¢ 답 2002 }

124. 홀수인 소수 p가 적당한 정수 x, y에 해 x5 ¡ y5꼴로 나타내어질 때, 적당한 홀수 v에 해 다음이 성립함을 증명하여라. (아일랜드 2001-3)r

4p+ 1

5=

v2 + 1

2

증명 인수분해하여 (x¡ y)(x4 + :::)꼴로 보면 (x4 + :::) 부분이 1보다 크다는 걸 금방 확인한 후 그럼

소수이려면 x¡ y = 1. x = y + 1 을 입하여 정리하면q4p+15

= 2y2 + 2y + 1 =(2y+1)2+1

2. ¤

125. 적당한 홀수 n에 해 55n + a ¢ 32n 이 2001의 배수가 되게 할 수 있는 최소의 자연수 a를 찾아라.(아일랜드 2001-6)

풀이 2001 = 3 ¢ 23 ¢ 29 이고, a ´ 1;¡1; 1 (mod 3; 23; 29). 그럼 나머지 정리나 정수 분류로 a ´ 436(mod 2001). }

126. 다음 식의 값이 정수가 되도록 하는 음 아닌 실수 x를 모두 구하여라.

3q13 +

px+

3q13¡px (아일랜드 2001-9)

풀이 준식을 t라 하면 0 < t < 2 3p13(< 5) 임을 금방 확인할 수 있음. 세제곱하여 3

p169¡ x = f(t)

꼴로 정리한 후 t = 1; 2; 3; 4 를 각각 입하여 풀면 끝. }

127. 다음을 만족하는 자연수해 (p; q; n)을 모두 구하여라. 단, p와 q는 소수이다.

p(p+ 3) + q(q + 3) = n(n+ 3) (아일랜드 2002-3)

풀이 mod 3으로 보면 p, q 둘 중 하나는 3의 배수, 즉 3. 그 후 인수분해법. ¢ ¢ ¢ 답 (2,3,4), (3,7,8) }

128. 방정식 a2b2 + b2c2 + 3b2 ¡ c2 ¡ a2 = 2005 는 정수해를 갖지 않음을 보여라. (중미 2005-2)

증명 b가 짝수일 때, 홀수일 때, 각각 mod 4로 생각하면 끝. ¤

별증 양변에서 3을 빼면 좌변이 b2 ¡ 1 로 인수분해되어 인수분해법으로 풀 수도 있지만 노동이 됨. ¤

129. n은 자연수이고, d는 2n2의 양의 약수이다. n2 + d 는 제곱수가 될 수 없음을 증명하여라.(헝가리 1953-2)

증명 dk = 2n2 이라 두면, A2 = n2 + d = n2 ¢ k+2k. 여기서 k와 k+ 2는 최 공약수가 1 또는 2이다.

최 공약수가 1이면 k + 2와 k 둘다 제곱수라야 하고 이것은 불가능. 최 공약수가 2이면 k+22 와 k

2 둘

다 제곱수라야 하고 이것도 불가능. ¤

130. y2 = x3 + 16 의 정수해를 모두 찾아라. (이탈리아 1994-2)

풀이 (y + 4)(y ¡ 4) = x3. y가 홀수일 땐 gcd(y + 4; y ¡ 4) = 1 이므로 y + 4, y ¡ 4 둘다 세제곱수이

고, 차이가 8인 두 세제곱수는 0과 §8뿐이므로 y = §4, x = 0. y가 짝수일 땐 y ´ 2 (mod 4) 이면 좌변은 4의 배수지만 8의 배수는 아니라서 모순. 즉 y = 4Y 꼴이고, 그럼 x도 4X꼴. (Y + 1)(Y ¡ 1) = 4X3에서 Y 는 홀수, 즉 Y = 2Z + 1꼴이고, Z(Z + 1) = X3 에서 차이가 1인 두 세제곱수는 0과 §1뿐. 정리하면 됨. }

131. x2 + 615 = 2y 의 자연수해 (x; y)를 모두 구하여라. (이탈리아 1995-6)

Page 58: 실전수학올림피아드 1400제 해답

58 수론

풀이 x2 ´ (¡1)y (mod 3) 이므로 y는 짝수. y = 2z 라 하면 (2z + x)(2z ¡ x) = 615 = 3 ¢ 5 ¢ 41.(2z + x; 2z ¡ x) = (41; 15), (123; 5), (205; 3), (615; 1)을 각각 풀면 됨. ¢ ¢ ¢ 답 59, 12? }

132. 방정식 2x2 ¡ 3x = 3y2 의 자연수해는 무한히 많은지, 아니면 없는지, 있기는 있는데 유한개뿐인지를 판정하여라. (yMO 2008-1-42)

풀이 3x, 3y2 이 모두 3의 배수이므로 3 j 2x2, 즉 3 j x. x = 3z 를 입하고 양변을 3으로 나눠주면

6z2 ¡ z = y2. 즉 z(6z ¡ 1) = y2. z와 6z ¡ 1 은 서로소이고 z는 자연수이므로 둘다 완전제곱수. 그런데 6z ¡ 1 꼴의 완전제곱수는 없다. 따라서, 이 식은 자연수해를 전혀 갖지 않는다. }

133. 임의의 서로 다른 정수 x, y, z에 하여 (x¡ y)5 + (y¡ z)5 + (z¡ x)5 이 5(x¡ y)(y¡ z)(z¡ x)로 나누어짐을 증명하여라. (소련 1962-12)

증명 x¡ y = r, y ¡ z = s 로 두면 z ¡ x = ¡(r + s) 이고 (x¡ y)5 + (y ¡ z)5 + (z ¡ x)5 은

r5 + s5 ¡ (r + s)5 = ¡5r4s¡ 10r3s2 ¡ 10r2s3 ¡ 5rs4 = ¡5rs(r + s)(r2 + rs+ s2)

¤

134. 자연수 수열 (An)은 A1 < 1987 이고 임의의 자연수 i에 해 Ai +Ai+1 = Ai+2 를 만족한다. A1 ¡An

과 A2 +An¡1 이 모두 1987의 배수이면, n은 홀수임을 증명하여라. (레닌그라드 1987-48)

증명 1987 j f(k) + (¡1)kf(n¡ k + 1) 임을 점화적으로 알 수 있음. 여기서 k = n 일 때 n이 짝수라

하면 1987 j f(n) + f(1). 이걸 1987 j f(1)¡ f(n) 과 변변 더하면 1987 j 2f(1), 즉 1987 j f(1) 로 모순.¤

135. 3x2 ¡ px+ q = 0 이 서로 다른 두 유리근을 가지는 모든 자연수쌍 (p; q)를 찾아라. (이탈리아 1993-2)

풀이 판별식 D = p2 ¡ 12q 가 양의 완전제곱수(d2)면 됨. p2 ´ d2 (mod 12) 인 경우들. WLOG d의

부호를 적절히 결정하면 p ´ d (mod 6) 이라 할 수 있음. 즉, d = p¡ 6n 꼴이고, 그럼 12q = p2 ¡ d2 =12pn¡ 36n2, 즉 q = pn¡ 3n2 = n(p¡ 3n). 단, p > 3n > 0. }

136. n은 주어진 정수이다. 연립방정식 xy+ yz+ zx = 3n2¡1, x+ y+ z = 3n 을 만족하는 정수해 x ¸ y ¸ z

는 (x; y; z) = (n+ 1; n; n¡ 1) 이 유일함을 보여라. (스웨덴 1977-3)

증명 (x¡ y)2 + (y ¡ z)2 + (z ¡ x)2 = 2(x+ y + z)2 ¡ 6(xy + yz + zx) = 6. ¤

137. a3 + 3a2 + a 가 완전제곱수인 자연수 a는 존재하지 않음을 증명하여라. (이탈리아 1991-2)

증명 a(a2 + 3a + 1) 이 제곱수임에서 a와 a2 + 3a+ 1 은 서로소이므로 각각 제곱수. a = m2 을

입하면 m4 + 3m2 + 1 = n2. 양변 4 곱해서 정리하면 (2m2 + 3)2 ¡ (2n)2 = 5. 두 양의 제곱수의 차가5가 되는 경우는 9¡ 4 뿐인데, 그럼 m = 0, 즉 a = 0 이므로 모순. ¤

138. p는 소수이다. p j n2 ¡ n + 3 을 만족하는 정수 n이 존재하는 것과 p j m2 ¡m + 25 을 만족하는 정수m이 존재하는 것은 필요충분임을 보여라. (폴란드 1994 2차-6)

증명 p = 2 일 때는 둘다 해가 없어서 ok. p = 3 일 때는 둘다 해가 있어서 ok. 이제 p는 5 이상의 소

수. p j (2n ¡ 1)2 + 11 과 p j (2m ¡ 1)2 + 99 에 해당하므로(그리고, 홀수들의 제곱은 모든 제곱수와mod p로 같은 집합을 이루므로) x2 ´ ¡11 (mod p) 인 x의 존재여부와 y2 ´ ¡99 (mod p) 인 y의 존재여부에 해당. 이것은 y = 3x 또는 x = 3¤y (잉여역수) 의 응으로 끝. ¤

139. x+ y + z,1

x+1

y+1

z, xyz가 모두 정수가 되도록 하는 양의 유리수들의 순서쌍 (x; y; z)을 모두 구하여

라. (폴란드 1994 3차-1)

풀이 x = qp 입해서 각각 정수 a, b, c라 두고 첫식과 막식의 결과를 중간식에 입하면 p j q3 임을

얻어낼 수 있음. 그럼 p = 1. 칭적으므로 y, z도 마찬가지. }

Page 59: 실전수학올림피아드 1400제 해답

1.2 수론 고급문제 59

주 유리해정리를 참조할 수도 있음.

140. 각 자연수 n에 해 (십진법에서의) n의 자릿수들을 재배열하여 만들 수 있는 자연수들의 집합을 An이라하자. 그리고, An의 원소들의 최 공약수를 dn이라 하자 예를 들어, A1120 = f112; 121; : : : ; 2101; 2110g이고 d1120 = 1 이다. n이 십진수로 나타냈을 때 모든 자릿수가 다 같지는 않은 자연수일 때, dn의 가능한 최 값을 구하여라. (이탈리아 1997-5)

풀이 a와 b가 n의 서로 다른 자릿수일 때 dn j ¢ ¢ ¢ ab ¡ ¢ ¢ ¢ ba = 9(a ¡ b). 따라서, dn · 9 ¢ 9. 실제로

n = 9999999990 일 때는 어떻게 재배열해도 92의 배수이므로 dn = 81 이 가능함. }

141. 십진법에서 모든 자릿수가 같은 자연수들 중에서 세 연속한 홀수의 제곱수의 합이 되는 수를 특별한 수라고 하자.

(a) 4자리의 특별한 수를 모두 찾아라.

(b) 2000자리의 특별한 수가 존재하는가? (이탈리아 2000-1)

풀이 세 연속한 홀수의 제곱수의 합은 3(2m+ 1)2 + 8꼴. (a) 5555. (b) mod 3, mod 10, mod 8 등을

적절히 사용해보면 없음. }

142. 자연수 n에 해, m = 5n + 3n + 1 이 소수이면 n은 12의 배수임을 증명하여라. (이탈리아 2002-5)

증명 n이 홀수이면 6 j 5n + 1 이므로 3 j m 이 되어 모순. n이 짝수인데 4의 배수는 아니면 10 j3n + 1 = 9m + 1 로 5 j m 이 되어 모순. 고로, n은 4의 배수. n = 4k 로 두면 m ´ 252k + 92k + 1 ´42k + 22k + 1 ´ 2k + 4k + 1 (mod 7) 인데, 23 ´ 43 ´ 1 (mod 7) 이므로 k = 1; 2 일 때만 체크하면3 - k 일 때는 7 j m 이 되어 모순임을 알 수 있음. ¤

143. a, b는 합이 1인 양의 실수이다. a3과 b3이 유리수이면, a와 b도 유리수임을 보여라.(폴란드 1994/1995 1차-9)

증명 a3, b3의 분모에 따라 적당히 a, b에 몇 배 해주면 a+ b는 자연수, a3, b3도 자연수. ab는 유리수

인데 그 세제곱이 자연수이므로 역시 자연수. 그럼 둘은 각각 자연수가 아니라면 켤레무리수. 다시 세제곱 싸바싸바하면 좀 노동해서 끝. ¤

144. P는 정계수 다항식인데, P (5)는 2의 배수이고 P (2)는 5의 배수이다. P (7)는 10의 배수임을 증명하여라.(폴란드 1995 2차-1)

증명 x¡ y j p(x)¡ p(y) 임을 이용. 7¡ 5 j P (7)¡P (5), 7¡ 2 j P (7)¡P (2) 에서 각각 2; 5 j P (7). ¤

145. m£ n 크기의 체스판에서 각 격자점들과 각 칸의 중심마다 졸을 하나씩 놓았다.

(a) 정확히 500개의 졸이 놓이는 격자의 크기 (m;n)을 모두 찾아라.

(b) 정확히 k개의 졸이 놓이는 크기의 격자가 존재하지 않는 자연수 k가 무한히 많음을 증명하여라.(이탈리아 2003-5)

증명 (2m+ 1)(2n+ 1) = 2k ¡ 1. (a)는 그냥 적당히 풀고, (b)는 2k ¡ 1 이 소수이면 해가 없으니까.¤

146. 20042005를 두 제곱수의 합으로 표현할 수 있는가? (이탈리아 2004-3b)

풀이 3 j a2 + b2 이면 a, b 모두 3의 배수임에서 계속 32으로 나눠주면 3의 홀수승이라서 곤란해짐. }

147. pn + 144 = m2 인 자연수해 (m;n; p)를 모두 찾아라. 단, p는 소수여야 한다. (이탈리아 2006-2)

풀이 pn = (m+ 12)(m¡ 12) = papb꼴. 만일 b = 0 이면 m = 13 이고 pn = 52. 이제 b > 0 일 때를

보자. 24 = pa ¡ pb = pb(pa¡b ¡ 1) 에서 두 인수는 서로소이므로 pb = 23 또는 31, 즉 m = 20 또는 15.

각각 pn = 28 또는 34. }

Page 60: 실전수학올림피아드 1400제 해답

60 수론

148. a, b, c, d는 실수이다. a = b = c = 0 이고 d가 유리수일 때만, 임의의 실수 x에 해 ax3 + bx2 + cx+ d

가 항상 유리수가 됨을 증명하여라. (몰도바 2001 최종-y7-5)

증명 준식을 f(x)라 할 때 f(0) = d 는 틀림없이 유리수. f(x) + f(¡x) 를 생각하면 bx2도 항상 유리

수. 즉, b = 0. f(x) ¡ f(¡x) 를 생각하면 A(x) = x(ax2 + c)이 항상 유리수. f(2x) ¡ f(¡2x) 로 하면 B(x) = x(4ax2 + c)도 항상 유리수. B ¡ A 나 4A ¡ B 를 생각하면 ax3, cx도 항상 유리수이므로a = c = 0. ¤

149. 임의의 정수 n에 해, 2 + 2p28n2 + 1 이 정수이면, 이 수는 제곱수임을 보여라. (헝가리 1969-1)

증명 준식을 m이라 하면 m은 짝수,또 좀더 살펴보면 m은 4의 배수. m = 4r 이라 두면 7n2 = r(r¡1)로 정리됨. r과 r ¡ 1은 서로소이므로 다음 두 가지뿐. (1) r = 7a2 = b2 + 1 꼴일 때: mod 4 또는 mod7로 따져보면 불가능. (2) r = 7a2 + 1 = b2 꼴일 때: 이 때는 m이 제곱수이므로 ok. ¤

150. 방정식 xx = y3 + z3 이 무한히 많은 자연수해 x; y; z를 가짐을 보여라. (폴란드 1995/1996 1차-10)

증명 x = 2n, y = z = 2k꼴만 생각. 2n¢2n= 23k+1 이므로 n ¢ 2n = 3k + 1꼴이 되는 n이 무한히 많

음을 보이면 충분. n = 6r ¡ 2꼴이면 됨. ¤

Page 61: 실전수학올림피아드 1400제 해답

제 2 장

2.1 수 중급문제

1. 함수 f는 임의의 m;n 2 N 에 해

f(mn) = mf(n) + nf(m)

을 만족한다고 한다. f(12) = f(15) = f(20) = 60 일 때 f(8)의 값을 구하여라. (2002 플란더즈 예선)

풀이 주어진 조건을 f의 성질에 입하면 다음과 같다.

f(12) = 3f(4) + 4f(3) = 60

f(15) = 5f(3) + 3f(5) = 60

f(20) = 4f(5) + 5f(4) = 60

f(3), f(4), f(5)를 각각 미지수로 보고 연립방정식을 풀면 f(4) = 8 를 얻는다.

f(4) = 2f(2) + 2f(2) = 8

로부터 다시 f(2) = 2 를 얻고, 그럼

f(8) = 4f(2) + 2f(4) = 24 ¢ ¢ ¢ 답

이다. }

2. 양수 x, y의 산술 평균과 기하 평균을 각각 ma, mg로 나타내자. ma +mg = y¡ x 일 때x

y의 값을 구하

여라. 단, 두 수의 산술 평균은 두 수의 합을 2로 나눈 것이고, 기하 평균은 두 수의 곱의 제곱근이다.(루마니아 2005 지역예선 y7-3a)

풀이 x+y2+pxy = y ¡ x, 2배하여 정리하면 2

pxy = y ¡ 3x, 양변 제곱하면 4xy = y2 ¡ 6xy + 9x2,

즉 y2 ¡ 10xy + 9x2 = 0 이고, 인수분해하면 (y ¡ x)(y ¡ 9x) = 0 이다. y = x 일 때는 준식의 좌변은

2x로 양수, 우변은 0이므로 성립하지 않고, y = 9x 일 때는 8x = 8x 로 잘 성립한다. 따라서, ¢ ¢ ¢ 답xy= 19

}

3. 합 1 ¢ 1!+2 ¢ 2!+3 ¢ 3!+ ¢ ¢ ¢+(n¡ 1) ¢ (n¡1)!+n ¢n! 의 값을 구하여라. 단, n! = n(n¡ 1)(n¡2) ¢ ¢ ¢ 2 ¢ 1이다. (캐나다 1969-6)

Page 62: 실전수학올림피아드 1400제 해답

62 대수

풀이 k ¢ k! = [(k + 1)¡ 1] ¢ k! = (k + 1)!¡ k! 이다. 즉,

1 ¢ 1! + 2 ¢ 2! + 3 ¢ 3! + ¢ ¢ ¢+ (n¡ 1) ¢ (n¡ 1)! + n ¢ n!= (2!¡ 1!) + (3!¡ 2!) + (4!¡ 3!) + ¢ ¢ ¢+ (n!¡ (n¡ 1)!) + ((n+ 1)!¡ n!)

그럼 2!과 ¡2!, 3!과 ¡3!, : : : ; n!과 ¡n!들이 상호 소거되고, 처음과 마지막의 ¡1!과 (n+ 1)!만 남는다.

¢ ¢ ¢ 답 (n+ 1)!¡ 1 }

4. 다음 연립방정식을 만족하는 모든 실수해 순서쌍 (x; y; z)를 찾아라. (캐나다 2004-1)8><>:xy = z ¡ x¡ y

xz = y ¡ x¡ z

yz = x¡ y ¡ z

풀이 우선 x = y = z 이면 (1)에서 x2+x = 0, 즉 (x; y; z) = (0; 0; 0) 이거나 (¡1;¡1;¡1) 이고, 이들

은 모두 해가 된다. 이제 x, y, z가 모두 같지는 않다고 하자. (1), (2), (3) 세 식을 둘씩 더하거나(7,8,9)

빼면(4,5,6) 다음을 얻는다.

(x+ 2)(y ¡ z) = (y + 2)(z ¡ x) = (z + 2)(x¡ y) = 0 ¢ ¢ ¢ (4); (5); (6)x(y + z + 2) = y(z + x+ 2) = z(x+ y + 2) = 0 ¢ ¢ ¢ (7); (8); (9)

x; y; z 중에는 서로 같지 않은 쌍이 최소한 둘은 있으므로, (4){(6)에서 x; y; z 중 적어도 둘은 ¡2이다.

일반성을 잃지 않고 y = z = ¡2 라 하면 (7)에서 x = 0. 따라서, 칭적으로 (x; y; z) = (0;¡2;¡2),(¡2; 0;¡2), (¡2;¡2; 0) 만이 가능하고 이들은 모두 해가 된다. 따라서, 답은 (0; 0; 0), (¡1;¡1;¡1),(0;¡2;¡2), (¡2; 0;¡2), (¡2;¡2; 0) ¢ ¢ ¢ 답 }

5. 임의의 실수 x, y에 해 다음을 만족하는 함수 f : R! R 를 모두 구하여라.

f(x)f(y)¡ f(xy) = x+ y (통신강좌 1997-15-10)

풀이 y = 0을 입하면f(0) ¢ f(x)¡ f(0) = x

f(0) 6= 0임은 자명하다.

f(x) =x

f(0)+ 1

f(x) = kx+ 1을 입해 풀어보면 k = 1

고로 f(x) = x+ 1 }

6. y =p2 +

p2, z =

p2¡p2 일 때 x = y + z 의 값을 구하여라. (아벨콘테스트 1994 예선)

풀이 x2 = (y + z)2 = (2 +p2) + (2¡p2) + 2p4¡ 2 = 4 + 2p2 = 2£ (2 +p2)

) x =p2p2 +

p2 =

p2y: }

7. 다음 조건을 만족하는 순서쌍 (x; y; z)를 모두 찾아라: 이 중 임의의 한 수를 다른 두 수의 곱에 더하면 그결과는 언제나 2가 된다. (캐나다 1970-1)

풀이 식으로 옮기면 다음과 같은 연립방정식이다.

(1) x+ yz = 2; (2) y + zx = 2; (3) z + xy = 2

(1)¡ (2), (2)¡ (3), (3)¡ (1) 하여

(x¡ y)(z ¡ 1) = (y ¡ z)(x¡ 1) = (z ¡ x)(y ¡ 1) = 0 (4)

을 얻는다. x = 1 이면 (1), (2)에서 yz = 1, y + z = 2 이고 이것은 t2 ¡ 2t + 1 = 0 의 두 근이므로y = z = 1. 칭적으로 y = 1 이거나 z = 1 일 때에도 x = y = z = 1 을 얻는다. x, y, z 모두 1이 아니면 (4)에서 x = y = z. 그럼 (1)은 x2+x¡2 = 0 이 되고, x 6= 1 이므로 x = ¡2. 즉, (x; y; z) = (1; 1; 1)

또는 (¡2;¡2;¡2) ¢ ¢ ¢ 답 }

Page 63: 실전수학올림피아드 1400제 해답

2.1 대수 중급문제 63

8. a+ b = 1 을 만족시키는 양수 a, b에 하여1

1 + a+

a

1 + b의 최소값은 얼마인가? (한국 2004 1차-J4)

풀이 준식을 x라 하면 a, b 모두 양수이므로 우선 x > 0 이다. b = 1¡ a 를 입하여 b를 소거하여 통

분하자.

x =1

1 + a+

a

2¡ a=

2 + a2

2 + a¡ a2

분모를 이항하여 a에 한 내림차순으로 정리하면

f(a) = (1 + x)a2 ¡ xa+ 2(1¡ x) = 0 (1)

a에 한 이 이차방정식이 해를 가지므로 판별식

D = x2 ¡ 4 ¢ 2(1¡ x) ¢ (1 + x) ¸ 0

즉, 9x2 ¸ 8, 그럼 x > 0 이므로 x ¸ 2p23. 따라서, x의 최소값은 2

p23 일 것으로 기 할 수 있는데, 다만

이 때 (1)의 해가 0 < a < 1 범위에 있어야 진짜 원하는 최소값이 된다. x = 2p23 일 때는 판별식이 0일

때이고 (1)이 중근을 가져 완전제곱식이 될 때이다. 이 때 a = x2(1+x) 로 0 < a < 1 을 틀림없이 만족함

을 알 수 있다. }

별해 이차방정식 (1)의 그래프를 생각해보자. 축 a = x2(1+x) 는 0 < a < 1

2 의 범위에 있으므로

f(0) < f(1) 이다. 따라서, (1)이 0 < a < 1 에서 해를 가질 필요충분조건은 다음과 같다.

f(축) · 0; f(1) > 0

이것을 풀면 2p23· x < 3

2 이 된다. }

9. 두 개의 가방에 몇 개의 수가 들어있고, 전체 수의 개수는 어떤 소수이다. 가방1에서 가방2로 수 170을 옮겼더니 각 가방의 수들의 평균값이 둘다 1씩 증가하였다. 전체 수의 합이 2004라 할 때, 전체 수의 개수를구하여라. (플란더즈 2004-2)

풀이 가방 1 속의 수의 개수를 m, 가방 2 속의 수의 개수를 n이라 하고, 가방 1 속의 수의 합을 a, 가

방 2 속의 수의 합을 b라고 하자. 조건에 맞게 식을 써보면,

a¡ 170m¡ 1 =

a

m+ 1;

b+ 170

n+ 1=

b

n+ 1

두 식을 각각 정리하면

m2 + 169m¡ a = 0

n2 ¡ 169n+ b = 0

두 식을 빼면

m2 ¡ n2 + 169(m+ n)¡ (a+ b) = 0

(m¡ n)(m+ n) + 169(m+ n)¡ 2004 = 0(m¡ n+ 169)(m+ n) = 2004 = 22 ¢ 3 ¢ 167

m+ n 이 소수이고 m+ n = 2, 3, 167 중에 하나이다. 그런데, m¡ n+ 169 가 m+ n 보다 169 이상 클수 없으므로 m+ n = 167 만 가능하다. }

10. x, y, z는 적당한 수 a, b에 해 a + b + x + y + z = 2004 와 0 · x · a · y · b · z 를 만족시킨다.x+ y + z 의 최 값과 최소값을 구하여라. (호주 2004-2)

Page 64: 실전수학올림피아드 1400제 해답

64 대수

풀이 (1) x + y + z · a + b + x + y + z · 2004 이고, 등호가 a + b = 0 일 때, 즉 (x; a; y; b; z) =

(0; 0; 0; 0; 2004) 일 때 성립하므로 최 값은 2004.

(2) a+ b · y + z · x+ y + z 이므로 2(x+ y + z) ¸ a+ b+ x+ y + z = 2004, 즉 x+ y + z ¸ 1002이고, 등호가 a+ b = x+ y + z 일 때, 즉 (x; a; y; b; z) = (0; a; a; b; b) (a+ b = 1002) 일 때 성립하므로 최소값은 1002.

}

11. x4 + x3 + x2 + x+ 1 = 0 는 몇 개의 실근을 갖는가? (플란더즈 예선 1991/1992)

풀이 양변에 (x¡ 1)을 곱하면 x5 ¡ 1 = 0 이 된다. x > 1 이면 x5 > 1 이고 x < 1 이면 x5 < 1 이므

로 x = 1 만 가능한데, 이것은 원래의 식은 성립시키지 않는다. 즉, x4 + x3 + x2 + x+ 1 = 0의 실근은0개. }

12. 다음 연립방정식을 보자.

x1 + 4x2 + 9x3 + 16x4 + 25x5 + 36x6 + 49x7 = 1

4x1 + 9x2 + 16x3 + 25x4 + 36x5 + 49x6 + 64x7 = 12

9x1 + 16x2 + 25x3 + 36x4 + 49x5 + 64x6 + 81x7 = 123

이 때, 16x1 + 25x2 + 36x3 + 49x4 + 64x5 + 81x6 + 100x7 의 값을 구하여라. (AIME 1989-8)

풀이 Label equations given (1), (2), (3). Then

(1) - 2 (2) + (3) = 2(x_1 + ... + x_7), (3) - (2) = 5x_1 + 7x_2 + 9x_3 + ... .

Hence 7x_1 + 9x_2 + 11x_3 + ... = (1) - 3 (2) + 2 (3) and

required value = (1) - 3 (2) + 3(3).

¢ ¢ ¢ 답 334 }

13. 자연수 함수 f가 다음과 같이 정의된다.

f(n) =

(n¡ 3 (n ¸ 1000 일 때)

f(f(n+ 6)) (n < 1000 일 때)

f(1992)¡ f(1) 은 얼마인가? (플란더즈 예선 1991/1992)

풀이 n = 999부터 몇 개의 값을 구해보면 f(999) = f(996) = 999, f(998) = f(995) = 998, f(997) =

f(994) = 997로 일정한 규칙성을 짐작할 수 있다. 수학적 귀납법으로 n = 997 ¡ 6k (1 · n · 1000)일때 f(n) 값이 같음을 증명한다.

² k = 0일 때 f(997) = 997.

² k = 0 : : : n¡ 1일 때 f(997 ¡ 6k) = 997이라면, f(997¡ 6n) = f(f(997 ¡ 6n + 6)) = f(f(997 ¡6(n¡ 1)) = f(997) = 997.

따라서 f(1) = f(997¡ 6 ¢ 166) = 997이고, f(1992)¡ f(1) = 1989¡ 997 = 992: }

14. x와 y는 x+ y = 1 을 만족하는 양의 실수라 한다. 이 때 다음 부등식이 성립함을 보여라.(캐나다 1971-2)µ

1 +1

x

¶µ1 +

1

y

¶¸ 9

증명 좌변을 전개해보자.µ1 +

1

x

¶µ1 +

1

y

¶= 1 +

1

x+1

y+1

xy= 1 +

x+ y + 1

xy= 1 +

2

xy

산술-기하평균 부등식에서 x+ y ¸ 2pxy, 즉 xy · 14 이므로

1 +2

xy¸ 1 + 2

14

= 9

등호는 x = y = 12 일 때 성립한다. ¤

Page 65: 실전수학올림피아드 1400제 해답

2.1 대수 중급문제 65

15. x = (1 + 1n)n 이고 y = (1 + 1

n)n+1 일 때, yx = xy 임을 보여라. (캐나다 1974-1a)

증명 다음과 같이 서로 같다.

xy =

µ1 +

1

n

¶n(n+1n)n+1

=

µ1 +

1

n

¶ (n+1)n+1nn

yx =

µ1 +

1

n

¶(n+1)(n+1n)n

=

µ1 +

1

n

¶ (n+1)n+1nn

¤

16. 방정식 [x] +2004

[x]= x2 +

2004

x2의 1이 아닌 해를 a라 할 때, a2의 값을 구하여라. (단, [x]는 x를 넘지 않

는 가장 큰 정수이다.) (한국 2004 1차-J15)

풀이 우변이 양수이므로 좌변도 양수, 즉 [x] > 0 이라야 하고, 또 1이 아닌 해라 했으므로, a > 1 이

다. 이 때, a2 ¡ [a] ¸ a2 ¡ a > 0 이므로

[a] +2004

[a]= a2 +

2004

a2() 2004

[a]¡ 2004

a2= a2 ¡ [a]

() 2004 ¢ a2 ¡ [a][a]a2

= a2 ¡ [a]

() a2[a] = 2004

a =p167 일 때 이 식이 성립하고, 좌변은 a가 증가함에 따라 단조증가하므로 해는 이것 하나뿐이다.

따라서, a2 = 167. }

17. S(n) = 1! + 2! + ¢ ¢ ¢+ n! 을 나타내는 것으로 할 때, 3£ 1! + 4£ 2! + 5£ 3! + 6£ 4! + ¢ ¢ ¢+ (n+ 2)£ n!

의 값을 S(n)을 이용하여 나타내어라.

풀이 (k + 2) £ k! = (k + 1)! + k! 이므로 준식은 S(n + 1) + S(n) ¡ 1 = 2S(n) + (n + 1)! ¡ 1 ¢ ¢ ¢답 }

18. 음이 아닌 세 실수 x1, x2, x3의 합은 1=2보다 크지 않다. 다음을 증명하여라.

(1¡ x1)(1¡ x2)(1¡ x3) ¸ 1

2(뉴질랜드 2000-2)

풀이 전개하면 1¡ (x1 + x2 + x3) ¸ 12, x1x2 ¡ x1x2x3 = x1x2(1¡ x3) ¸ 0, x2x3 + x3x1 ¸ 0. 등호

조건은 하나가 12, 나머지 둘이 0일 때. }

19. 실수 x는 [x+ 0:19] + [x+ 0:20] + [x+ 0:21] + ¢ ¢ ¢+ [x+ 0:91] = 546 을 만족한다. [100x]의 값을 구하여라. (AIME 1991-6)

풀이 %EEE

546 = 7·73 + 35 = 38·7 + 35·8. Hence [x + 0.19] = [x + 0.20] = ...

= [x + 0.56] = 7, [x + 0.57] = ... = [x + 0.91] = 8. So 7.43 <= x < 7.44.

¢ ¢ ¢ 답 743 }

20. x =p19 +

91p19 +

91p19 +

91p19 +

91p19 +

91

x

을 만족하는 실수해 x의 절 값을 모두 합한 값을 k라 하

자. k2을 구하여라. (AIME 1991-7)

Page 66: 실전수학올림피아드 1400제 해답

66 대수

풀이 %EEE

It is clear that simplifying will give a quadratic in x.

One can slog through to find it, but it is easier to note that if

x = √19 + 91/x, then x will satisfy the equation, so the quadratic

must be simply x2 - √19 x - 91. A minor trap is that one root is

negative: x = (√383 + √19)/2 or -(√383 - √19)/2, so sum of

abs values is √383.

¢ ¢ ¢ 답 383 }

21. 모든 양의 정수 n에 해, 다음을 증명하여라. (캐나다 1974-1b)

12 ¡ 22 + 32 ¡ 42 + ¢ ¢ ¢+ (¡1)n(n¡ 1)2 + (¡1)n+1n2 = (¡1)n+1(1 + 2 + ¢ ¢ ¢+ n)

증명 n이 짝수일 때

(12 ¡ 22) + (32 ¡ 42) + ¢ ¢ ¢+ ((n¡ 1)2 ¡ n2)

= (1¡ 2)(1 + 2) + (3¡ 4)(3 + 4) + ¢ ¢ ¢ ((n¡ 1)¡ n)((n¡ 1) + n)

= ¡(1 + 2 + 3 + 4 + ¢ ¢ ¢+ (n¡ 1) + n)

n이 홀수일 때

12 ¡ (22 ¡ 32)¡ ¢ ¢ ¢ ¡ ((n¡ 1)2 ¡ n2)

= 1¡ (2¡ 3)(2 + 3)¡ ¢ ¢ ¢ ¡ ((n¡ 1)¡ n)((n¡ 1) + n)

= 1 + 2 + 3 + 4 + ¢ ¢ ¢+ (n¡ 1) + n

로 항상 성립한다. ¤

별해 수학적 귀납법을 이용한다. n = 1 일 때는 자명. n = k 일 때 성립한다고 가정하면

12 ¡ 22 + ¢ ¢ ¢+ (¡1)k+1k2 = (¡1)k+1 k(k + 1)2

12 ¡ 22 + ¢ ¢ ¢+ (¡1)k+2(k + 1)2 = (¡1)k+1 k(k + 1)2

+ (¡1)k+2(k + 1)2

= (¡1)k+2 k + 12

f2(k + 1)¡ kg

= (¡1)k+2 (k + 1)(k + 2)2

이므로 n = k + 1 일 때도 성립한다. 그러므로 모든 양수 n에 해 준식은 성립. ¤

22. x, y, z가 모두 실수일 때,

x+ y + z = 5

xy + yz + zx = 3

을 만족하는 z의 최 값을 구하여라. (캐나다 1978-3)

풀이 x+ y = 5¡ z 이고,

xy = 3¡ z(x+ y) = 3¡ z(5¡ z) = z2 ¡ 5z + 3따라서, x, y를 두 실근으로 갖는 방정식은

t2 ¡ (5¡ z)t+ (z2 ¡ 5z + 3) = 0이며, 판별식

D = (5¡ z)2 ¡ 4(z2 ¡ 5z + 3) = ¡3z2 + 10z + 13 ¸ 0임에서 3z2 ¡ 10z ¡ 13 = (z + 1)(3z ¡ 13) · 0, 즉

¡1 · z · 13

3

이다. z = 133 일 때 x = y = 1

3 임을 확인할 수 있으므로 z의 최 값은 133

¢ ¢ ¢ 답 }

Page 67: 실전수학올림피아드 1400제 해답

2.1 대수 중급문제 67

23. 방정식 (x2 ¡ x+ 1)(x2 ¡ x+ 2) = 12 의 실근은 모두 몇 개인가? (플란더즈 예선 1995/1996 2차)

풀이 x2 ¡ x = t라고 하면 x가 실수인 범위에서 t ¸ ¡ 14이다.

(t+ 1)(t+ 2) = 12) t2 + 3t¡ 10 = 0) (t¡ 2)(t+ 5):이 때 t ¸ ¡ 1

4인 해는 t = 2이고 이 때 x2 ¡ x = t를 만족하는 x는 두 개 있다.

¢ ¢ ¢ 답 2개 }

24. 함수 f(x)는 임의의 실수 x에 하여

f(1 + x) + xf(1¡ x) = x2 + x

를 만족시킨다. f(¡6)의 값을 구하여라. (한국 2003 1차-J6)

풀이 (1 + x)와 (1¡ x)는 1에 칭적인 값이다. x = §7 을 입하면 f(¡6)과 f(8)의 값을 함께 구할

수 있다.

(1) x = 7 입: f(8) + 7f(¡6) = 72 + 7 = 56.(2) x = ¡7 입: f(¡6)¡ 7f(8) = (¡7)2 ¡ 7 = 42.

아래 식의 f(8)에 위 식을 입한다.

f(¡6)¡ 7f56¡ 7f(¡6)g = 4250f(¡6) = 42 + 392 = 434

f(¡6) = 8:68

¢ ¢ ¢ 답 8.68 }

25. x가 양의 실수일 때, 방정식 3x3 ¡ [x] = 3 을 풀어라. (IMO-LL 1989, 포르투갈 출제)

풀이 x ¸ 2 이면 3x3 = x3 +2x3 > x+3 ¸ [x] + 3 이므로 성립하지 않는다. [x] = 0 일 때도 3x3 = 3

에서 x = 1 이 되어 성립하지 않는다. 따라서, 1 · x < 2 이고 [x] = 1. 이 때 x3 = 43, 즉 x = 3

q43¢ ¢ ¢

답 }

26. 임의의 네 양의 실수 a, b, c, d에 해, 다음을 증명하여라. (인도지역예선 1991-2)

a

b+

b

c+

c

d+

d

a¸ 4

풀이 A.M ¸ G.M 으로 ab+ b

c+ c

d+ d

a¸ 4 4q

ab¢ bc¢ cd¢ da= 4. }

27. 이차방정식 x2 + bx+ 6b = 0 이 정수해만을 갖도록 하는 실수 b는 모두 몇 개인가? (AIME 1991-8)

풀이 두 정수근을 m · n 이라 하면 mn = 6b,m+n = ¡b. mn+6(m+n) = 0이므로 (m+6)(n+6) =

36. 36의 양의 약수가 9개임에서 36을 두 정수의 곱으로 표현하는 방법은 (양수, 음수 모두 세었을 때)

2d 92e = 10가지이고 실제로 각 경우에 m+ n의 값이 모두 다르므로 b는 모두 10개 ¢ ¢ ¢ 답 }

28. 1000, n, 1000¡ n, n¡ (1000¡ n), : : : 는 n번째 항에서 n+ 1번째 항을 뺀 것을 n+ 2번째 항으로 하는수열이다. 이 수열은 처음으로 음수가 나타나면 종료되는 것으로 한다. 이 수열의 길이를 최 로 하는 자연수 n을 구하여라. (AIME 1998-8)

풀이 항을 계속 구하면 1000, n, 1000¡ n, 2n¡ 1000, 2000¡ 3n, 5n¡ 3000, 5000¡ 8n, 13n¡ 8000,13000 ¡ 21n, 34n ¡ 21000, 34000 ¡ 55n, 89n ¡ 55000, : : : 등이다. 제3항부터 각각이 0 이상임을 부등식으로 풀면 n · 1000, n ¸ 500, n · 666:6 ¢ ¢ ¢ , n ¸ 600, n · 625, n ¸ 615:3 ¢ ¢ ¢ , n · 619:0 ¢ ¢ ¢ ,n ¸ 617:6 ¢ ¢ ¢ , n · 618:1 ¢ ¢ ¢ , n ¸ 617:9 ¢ ¢ ¢ 등이 된다. ¢ ¢ ¢ 답 618 }

29. A£B £ C 크기의 상자의 부피가 (A+ 2)£ (B + 2)£ (C + 2) 크기의 상자의 부피의 절반이 된다고 한다. A · B · C 가 자연수들일 때, C의 최 값은 얼마인가? (AIME 1998-14)

Page 68: 실전수학올림피아드 1400제 해답

68 대수

풀이 %EEE

c = 2(a+2)(b+2)/(2ab-(a+2)(b+2)) = 2(a+2)(b+2)/(ab-2a-2b-4)

= 2 + (8a+8b+16)/(ab-2a-2b-4). So (c-2)/8 = (a+b+2)/(ab-2a-2b-4).

To make progress we need to realise that it helps to put c'=c-2, b'=b-2,

a'=a-2, so c'/8 = (a'+b'+6)/(a'b'-8). Now we cannot have a or b = 1 or 2,

because (1+2)/1 and (2+2)/2 are already ≥ 2 (so multiplying by (c+2)/c

gives > 2). So a', b' ≥ 1, so a'+b' ≤ a'b'+1. Hence

c'/8 ≤ (a'b'+7)/(a'b'-8) = 1 + 15/(a'b'-8) ≤ 1 + 15. Hence c ≤ 130

and it is easy to check this is realised.

¢ ¢ ¢ 답 130 (3,11) }

30. (3x2 ¡ 2x¡ 1)4 = a8x8 + a7x7 + ¢ ¢ ¢+ a1x+ a0 일 때, a8 + a6 + a4 + a2 는 얼마인가?(플란더즈 예선 1995/1996 2차)

풀이 x = 0을 입하여 a0를 구한다. (¡1)4 = 0 + 0 + ¢ ¢ ¢+ a0 = a0 = 1:

x = 1을 입하면: 04 = 0 = a8 + a7 + ¢ ¢ ¢+ a1 + a0 ¢ ¢ ¢ ¢ ¢ ¢ (a)x = ¡1을 입하면: 44 = 256 = a8 ¡ a7 + ¢ ¢ ¢ ¡ a1 + a0 ¢ ¢ ¢ ¢ ¢ ¢ (b)(a) + (b) = 256 = 2(a8 + a6 + a+ 4 + a2) + 2a0 ) a8 + a6 + a4 + a2 =

256¡22

= 127

¢ ¢ ¢ 답 127 }

31. a와 b는 서로 다른 실수이고, 두 점 (a; a3)과 (b; b3)을 지나는 직선이 y = x3 의 그래프와 또 다른 점에서다시 만난다. 이 점의 y-좌표는 얼마인가? (플란더즈 예선 1996/1997 1차)

풀이 두 점 (a; a3)과 (b; b3)을 지나는 식은 다음과 같다.

y ¡ a3 = b3¡a3a¡b (x¡ a) = (a2 + ab+ b2)(x¡ a)

이 직선이 y = x3과 만나는 또다른 점 (p; p3)을 입하고 a 6= p이므로 양 변을 (p¡ a)로 나눈다.p3 ¡ a3 = (a2 + ab+ b2)(p¡ a)) p2 + ap+ a2 = a2 + ab+ b2 ) p2 + ap = b2 + ab

) (p+ a2)2 = (b+ a

2)2 ) p = b;¡(a+ b)

이 때 p 6= b이므로 p = ¡(a+ b)이다.

¢ ¢ ¢ 답 ¡(a+ b)3 }

32. 함수 f : R ! R 가 f(x) + f(1 ¡ x) = 7 과 x + f(x3) = 1

2f(x) 를 만족시킬 때, f( 19 )의 값은? (단, R은

실수 전체의 집합이다.) (한국 2005 1차-J9)

풀이 뒷식에 x = 0 을 입하면 f(0) = 0. 다시 앞식에 x = 0 을 입하면 f(1) = 7 이 된다. 이제 뒷

식에 x = 1, x = 13 을 차례로 입하면 f( 1

3) = 1

2f(1)¡ 1 = 5

2, f( 1

9) = 1

2f( 13)¡ 1

3= 1112

¢ ¢ ¢ 답 }

33. 다음 연립방정식을 주어진 상수 A에 관해 풀어라. (인도지역예선 1992-7)

(x+ y)(x+ y + z) = 18

(y + z)(x+ y + z) = 30

(z + x)(x+ y + z) = 2A

풀이 세 식을 더하면 (2x + 2y + 2z)(x + y + z) = 48 + 2A이고 정리하면 (x + y + z)2 = 24 + A이

다. t = x + y + z = §p24 +A 라고 두고 z를 A에 한 식으로 표현해보자. (x + y)(x + y + x) =(t ¡ z)t = t2 ¡ zt = (24 + A) ¡ zt = 18, 즉 z = 6+A

t. 같은 방법으로 x = ¡6+A

t, y = 24¡A

t.

t가 0일 때는 해가 없고, t가 양일 때나 음일 때나 모두 준식을 잘 만족함을 확인할 수 있다. ¢ ¢ ¢ 답(x; y; z) = pm1p

24+A(¡6 + A; 24¡ A; 6 +A) (단, A > ¡24 일 때만) }

34. 함수 f는 모든 x에 해 f(x) + f(x¡ 1) = x2 을 만족한다. f(19) = 94 일 때, f(94)를 구하여라.(AIME 1994-3)

풀이 %EEE

f(94)=94^2-f(93)=94^2-93^2+f(92)=94^2-93^2+92^2-f(91)= ... =

(94^2-93^2) + (92^2-91^2) +...+ (22^2-21^2)+ 20^2-f(19) = 94+93+...+21+400-94

= 115·37+306 = 4561.

¢ ¢ ¢ 답 4561 }

Page 69: 실전수학올림피아드 1400제 해답

2.1 대수 중급문제 69

35. 음이 아닌 임의의 실수 x, y에 해 함수 f가 f(xy) = f(x) + f(y) 를 만족한다. f(0:5) + f(1) + f(2) 의값은 얼마인가? (플란더즈 예선 1999/2000 2차)

풀이 x = y = 1 입하면 f(1) = 0 준식은 f(0:5 ¢ 1 ¢ 2) = f(1) = 0 }

36. 0 < b < 10¡6 일 때, 다음 중p1¡ b에 가장 가까운 근사값은 어느 것인가? (1) 1¡ b (2) 1¡ b

2(3) 1¡ b2

(4) 1¡ b2

2(5) 1 (플란더즈 예선 2000/2001 1차)

풀이 제곱해서 비교하면 (1) 1 ¡ 2b + b2 = 1 ¡ b + (¡b + b2) < 1 ¡ b, (2) 1 ¡ b + b2

4> 1 ¡ b 이

다. b의 범위에 의해 b2> b2 > b2

2> 0 이므로 (2) < (3) < (4) < (5) 이다. 따라서, (1)과 (2) 중에서p

1¡ b에 더 가까운 것을 찾으면 된다. (1)과 (2)의 평균인 1 ¡ 34b와

p1¡ b를 비교하면, (1 ¡ 3

4b)2 =

1¡ 32b+ 9

16b2 = (1¡ b)¡ b( 1

2¡ 916b) < 1¡ b 로

p1¡ b쪽이 더 크다. 따라서, 더 큰 쪽인 (2)가 더 가

까운 근사값이다. ¢ ¢ ¢ 답 (2) }

37. 실수 x에 하여, x2 +16

x2 + 1의 최소값을 구하여라. (한국 2005 1차-J11)

풀이 (대전 대덕중 3학년 고기혁) x2 + 1 > 0 이므로 산술-기하평균 부등식에서

(준식) = (x2 + 1) +16

x2 + 1¡ 1 ¸ 2

r(x2 + 1)

16

x2 + 1¡ 1 = 2 ¢ 4¡ 1 = 7

x = §p3 일 때 실제 등호가 성립하므로, 최소값은 7 ¢ ¢ ¢ 답 }

38.4pk에 가장 가까운 정수를 f(k)로 쓰기로 하자.

1

f(1)+

1

f(2)+

1

f(3)+ ¢ ¢ ¢+ 1

f(1995)를 계산하여라.

(AIME 1995-13)

풀이 %EEE

f(k) must take one of the values 1, 2, ... , 7. The boundary values

are (n + ½)^4 = n^4 + 2n^3 + ½(3n^2 + n) + 1/16, giving 5 1/16, 39 1/16,

150 1/16, 410 1/16, 915 1/16, 1785 1/16. So the sum is 5 + 34/2 + 111/3

+ 260/4 + 505/5 + 870/6 + 210/7 = 400.

¢ ¢ ¢ 답 400 }

39. (x¡ 1)(x¡ 2) ¢ ¢ ¢ (x¡ 99) 를 전개했을 때, x98의 계수를 구하여라.(2000 대전.충남 영재수학교실 1차평가)

풀이 ¡(1 + 2 + 3 + ¢ ¢ ¢+ 99) = ¡4950 ¢ ¢ ¢ 답 }

40. a와 b는 a 6= b 인 주어진 두 실수이다. 상수 k > 0 가 어떻게 주어지느냐에 따라 x에 한 방정식jx¡ aj+ jx¡ bj = k 의 해가 달라질 수 있다. 이에 해 다음 중 맞는 진술을 모두 골라라.

(1) 해가 딱 하나 있도록 하는 k값은 딱 하나 있다.

(2) 해가 딱 하나 있도록 하는 k값은 무한히 많다.

(3) 해가 무한히 많이 있도록 하는 k값은 딱 하나 있다.

(4) 해가 무한히 많이 있도록 하는 k값은 여러 개 있다.

(5) 어떤 k값에 해서도 해는 항상 딱 두 개이다. (플란더즈 예선 1999/2000 2차)

풀이 x의 범위를 a; b보다 클 때, a; b 사이에 있을 때, a; b보다 작을 때로 나누어 생각하면 쉽게 다음과

같은 사실을 알아낼 수 있다.k > ja¡ bj일 때 해는 x = a+b+k

2; a+b¡k

2 의 두개이다.

k = ja¡ bj일 때 해는 a, b 사이에 있는 모든 x로 무한히 많다.k < ja¡ bj일 때 해는 없다.

그러므로 (3)만 옳은 답이다.

¢ ¢ ¢ 답 (3) }

Page 70: 실전수학올림피아드 1400제 해답

70 대수

41. 임의의 x 2 R0 에 해

x ¢ fµx

2

¶¡ f

µ2

x

¶= 1

를 만족하는 함수 f : R0 ! R 을 모두 찾아라. 단, R0은 0을 제외한 실수 전체의 집합이다.(플란더즈 2002-2)

풀이 x에 4x를 입하면

4

xf

µ2

x

¶¡ f

µx

2

¶= 1

이 식에 x를 곱하고 준식과 더하면 3f( 2x) = 1 + x. 여기에 x 신 2

x를 입하면 f(x) = x+23x 임을 알

수 있다. (이 함수는 준식에 입하면 만족한다.) }

42. 다음 부등식을 증명하여라. (KAIST Cyber 2002 겨울캠프평가시험)

1

1+

1p2+

1p3+ ¢ ¢ ¢+ 1p

n¸ pn

증명 분모가 커질수록 분수는 작아지므로,

1

1+

1p2+ ¢ ¢ ¢+ 1p

n¸ 1p

n+

1pn+ ¢ ¢ ¢+ 1p

n= n ¢ 1p

n=pn

등호는 n = 1 일 때에만 성립. ¤

43. 모든 양의 실수 a, b, c에 해 다음의 부등식이 성립함을 보여라.

a

bc+

b

ca+

c

ab¸ 2

a+2

b¡ 2

c

그리고, 이 식에서 등호가 성립할 a, b, c의 조건을 구하여라. (플란더즈 2002-1/J1)

증명 양변에 abc를 곱하여 정리하면

a2 + b2 + c2 ¸ 2bc+ 2ca¡ 2ab(a+ b¡ c)2 ¸ 0

따라서 부등식이 성립하고, 등호가 성립할 조건은 a+ b = c 이다. ¤

44. 임의의 실수 t에 해, [t]은 t보다 작거나 같은 가장 큰 정수를 나타낸다. 예를 들어, [8] = 8, [¼] = 3,[¡5=2] = ¡3 등이다. 다음 방정식이 실근을 갖지 않음을 보여라.

[x] + [2x] + [4x] + [8x] + [16x] + [32x] = 12345 (캐나다 1981-1)

증명 좌변의 식을 S(x)라 하자. S(x)는 증가함수이다.

² x ¸ 196 이면 S(x) ¸ S(196) = 63 ¢ 196 = 12348² x < 196 이면 좌변의 각 항에서 최소 1씩 줄어들어 S(x) · S(196)¡ 6 = 12342

이 되므로, 주어진 방정식의 실수해는 존재하지 않는다. ¤

45. 다음 방정식의 실근 a의 개수를 구하여라.·1

2a

¸+

·1

3a

¸+

·1

5a

¸= a

단, [x]는 x보다 작거나 같은 가장 큰 정수를 나타낸다. (캐나다 1998-1)

Page 71: 실전수학올림피아드 1400제 해답

2.1 대수 중급문제 71

풀이 좌변은 정수이므로 우변의 a도 정수. a = 30n+ r, 0 · r < 29 (n; r은 정수) 로 두자. 이것을

입하면

15n+

·1

2r

¸+ 10n+

·1

3r

¸+ 6n+

·1

5r

¸= 30n+ r

n+

·1

2r

¸+

·1

3r

¸+

·1

5r

¸= r

따라서, 각각의 r = 0; 1; : : : ; 29 이 주어지면 n이 유일하게 결정된다. 즉, 각각의 r = 0; 1; : : : ; 29 마다서로 다른 해가 하나씩 있으므로, 해는 모두 30개이다. }

46. 2 < x2 < 3 인 음 아닌 실수 x가 있는데,1

x과 x2의 소수부가 같다고 한다. x12 ¡ 144

x를 구하여라.

(AIME 1997-9)

풀이 %EEE

Since 2 < x^2 < 3, we have 1 < x < 2, so 1/2 < 1/x < 1 and the

fractional part of 1/x is just 1/x. The fractional part of x^2 is

x^2 - 2, so x^3 - 2x - 1 = 0. Taking out the factor x + 1, and solving

x^2 - x - 1 = 0, we get x = (√5 + 1)/2. Now using x^2 = 1+x, we get

successively x^4 = 2+3x, x^8 = 13+21x, x^12 = 89+144x, 1/x = x-1,

so x^12 - 144/x = 233.

¢ ¢ ¢ 답 233 }

47. a, b, c, d는 0이 아닌 실수이고 f(x) =ax+ b

cx+ d이다. f(19) = 19, f(97) = 97 이고, x 6= ¡d

c인 모든 x에

해 f(f(x)) = x 일 때, f(x) = y 인 x가 존재하지 않는 유일한 y를 구하여라. (AIME 1997-12)

풀이 %EEE

f(f(x)) = x simplifies to (a+d)(cx^2 + (d-a)x - b) = 0, so d = -a.

There is no loss of generality in taking c = 1. Then f(19) = 19,

f(97) = 97 give 38a + b = 361, 194a + b = 9409. Solving, a = 58,

b = -1843. The unattained value is a/c.

¢ ¢ ¢ 답 58 }

48. a1 = 211, a2 = 375, a3 = 420, a4 = 523, 그리고 an = an¡1 ¡ an¡2 + an¡3 ¡ an¡4 로 정의된 수열fang이 있다. a531 + a753 + a975 의 값을 구하여라. (AIME 2001 2차-3)

풀이 %EEE

We find a_5 = 267, a_6 = -a_1, a_7 = -a_2, a_8 = -a_3, a_9 = -a_4,

a_10 = -a_5, a_11 = a_1, a_12 = a_2, a_13 = a_3, a_14 = a_4 etc.

So a_531 = a_1, a_753 = a_3, a_975 = a_5 and a_1 + a_3 + a_5 =

211 + 420 + 267 = 898.

¢ ¢ ¢ 답 898 }

49. 모든 실수 x에 해 f(¡x) = f(x) 를 만족하는 함수 f를 우함수, g(¡x) = ¡g(x) 를 만족하는 함수 g를기함수라고 한다. f가 우함수, g가 기함수이고, f와 g는 실수값을 가지며 항등적으로 0이 되지는 않는 함수들이다. 다음 중 기함수인 것을 모두 골라라. f ± g, g ± f , f ± f , g ± g, f + g. (플란더즈 예선 2002 2차)

풀이 각 함수에 ¡x값을 넣어본다.

f ± g(¡x) = f(g(¡x)) = f(¡g(x)) = f(g(x)) = f ± g(x): 우함수.g ± f(¡x) = g(f(¡x)) = g(f(x)) = g ± f(x): 우함수.f ± f(¡x) = f(f(¡x)) = f(f(x)) = f ± f(x): 우함수.g ± g(¡x) = g(g(¡x)) = g(¡g(x)) = ¡g(g(x)) = ¡g ± g(x): 기함수.(f + g)(¡x) = f(¡x) + g(¡x) = f(x)¡ g(x): f(x) 6= 0인 x 값을 입하면 기함수가 아님을 알 수 있다. }

50. 함수 f : Z ! Z 는 n > 999 일 때 f(n) = n ¡ 3 을 만족하고 n < 1000 일 때는 f(n) = f(f(n + 5)) 를만족한다. f(84)의 값을 구하여라. (AIME 1984-7)

Page 72: 실전수학올림피아드 1400제 해답

72 대수

풀이 %EEE

For n < 1002, f(n) = 997 for n even, 998 for n odd.

¢ ¢ ¢ 답 997 }

51. n > 0 에 해 an+2 = an+1 ¡ an 을 만족하는 정수들의 수열 a1; a2; a3; : : : 이 있다. 이 수열의 처음1492개항의 합은 1985이고, 처음 1985개항의 합은 1492라고 한다. 처음 2001개항의 합을 구하여라.

(AIME 1985-5)

풀이 %EEE

The sequence is a_1, a_2, a_2-a_1, -a_1, -a_2, a_1-a_2, a_1, a_2, ... ,

so it is periodic with period 6. Also the sum of a complete period of terms

is 0. 1492 = 4 mod 6, 1985 = 5 mod 6, 2001 = 3 mod 6, so the sum of the

first 1492 terms is a_1 + a_2 + a_3 + a_4 = 2a_2 -a_1, the sum of the

first 1985 is a_1 + ... + a_5 = a_2 - a_1. Hence a_2 = (2a_2 -a_1) -

(a_2 - a_1) = 493, and the sum of the first 2001 = 2a_2 = 986.

¢ ¢ ¢ 답 986 }

52. n이 자연수일 때, 다음 식을 간단히 하여라. (플란더즈 예선 1998/1999 1차)

1 + 3 + 5 + 7 + ¢ ¢ ¢+ (2n¡ 1)(2n+ 1) + (2n+ 3) + ¢ ¢ ¢+ (4n¡ 1)

풀이 n2=(4n2 ¡ n2) = 1=3 ¢ ¢ ¢ 답 }

53. a는 a3 = a + 1 을 만족하는 양의 실수이다. 다음 중 맞는 식을 모두 골라라. (1) a4 = a2 + a (2)

a4 = a5 ¡ 1 (3) a4 = a3 + a2 ¡ 1 (4) a4 = 1

a¡ 1 (플란더즈 예선 2000/2001 1차)

풀이 (1) a3 = a+ 2 식의 양 변에 a를 곱하면 a4 = a2 + a를 얻을 수 있다.

(2) a4 = a2 + a의 양 변에 a를 곱한다. 얻어진 a5 = a3 + a2의 식에 a3 = a+ 1, a2 = a4 ¡ a((1)의 식에서 얻어진 것)을 입하면 a5 = a+ 1 + a4 ¡ a = a4 + 1) a4 = a5 ¡ 1을 얻을 수 있다.(3) (1)에서 얻은 식 a4 = a2 + a에서 a = a3 ¡ 1을 입한다. a4 = a3 + a¡ 1.(4) a는 1이 아니므로 (2)의 식을 정리하고 양변에 (a ¡ 1)을 나눈다. a4 = a5 ¡ 1 ) a5 ¡ a4 = 1 )(a¡ 1)a4 = 1) a4 = 1

a¡1 :

¢ ¢ ¢ 답 (1), (2), (3), (4) }

54. 방정식 x3 ¡ 3x2 + 3x+ 1 = 0 의 모든 실근의 합을 구하여라. (플란더즈 예선 2002 2차)

풀이 (x ¡ 1)3 = ¡2. x ¡ 1 = ¡ 3p2. 근과 계수와의 관계에 한 문제로 혼동할 수 있음. ¢ ¢ ¢ 답

1¡ 3p2 }

55. f(x)를 4, 9, 7, 5, x의 평균값이라 정의하고 g(x)를 4, 9, 7, 5, x를 순서 로 늘어놓았을 때 가운데 놓이는 수라 하자. f(x) = g(x) 가 되는 실수 x를 모두 구하여라. (IT꿈나무 올림피아드 2006 1차)

풀이 평균 f(x) = 5+ x5 이다. x < 5 이면 g(x) = 5 이므로 f(x) = g(x) 이려면 x = 0 이다. 5 · x · 7

이면 g(x) = x 이므로 f(x) = g(x) 이려면 x = 254 이다. x > 7 이면 g(x) = 7 이므로 f(x) = g(x) 이려

면 x = 10 이다. ¢ ¢ ¢ 답 0, 254, 10 }

56. a, b, c는 모두 양수이고, a < 5 이다. 이 수들이 a2 ¡ a¡ 2b¡ 2c = 0, a+ 2b¡ 2c+ 3 = 0 을 만족할 때,a, b, c 사이의 소 관계를 밝혀라. (1995 교육청경시)

풀이 a2 ¡ a¡ 2b¡ 2c = 0 ¢ ¢ ¢ ¢ ¢ ¢ (1)a+ 2b¡ 2c+ 3 = 0 ¢ ¢ ¢ ¢ ¢ ¢ (2)(1)¡ (2) : a2 ¡ 2a¡ 4b¡ 3 = 0) a2 ¡ 2a = 4b+ 3 > 3) a2 ¡ 2a¡ 3 > 0) (a¡ 3)(a+ 1) > 0) a > 3:

이 때 4b+ 3 = a2 ¡ 2a < 15(a < 5)) b < 3: (1) + (2) : a2 ¡ 4c+ 3 = 0.a2 ¡ 4a + 3 = (a ¡ 1)(a ¡ 3) > 0 = a2 ¡ 4c + 3 = 0 ) c > a: 종합하면 c > a > 3 > b이므로,c > a > b의 소관계가 있다. }

Page 73: 실전수학올림피아드 1400제 해답

2.1 대수 중급문제 73

57.4

3

½1

a¡·1

a

¸¾= a 를 만족하는 유리수 a의 개수를 구하여라. (단, 0 < a < 1, [x]는 x를 넘지 않는 최

정수)

풀이 a = qp라고 두고 위의 식에 입한다.(이 때 p > q)

43f pq¡ b p

qcg = q

p) 4

3qfp¡ qb p

qcg = q

p

이 때 (p¡ qbpqc)는 p를 q로 나눈 나머지이다. p > q이므로 p = kq + r(r < q)라고 하고 입하면

4r3q= q

p) 4rp = 3q2 ) 4r(kq + r) = 3q2 ) 4r2 + 4kqr ¡ 3q2 = 0 ) r =

¡2kq§p4k2q2+12q2

4=

q2(¡k §pk2 + 3):

r ¸ 0이므로 r = q2(¡k + pk2 + 3)인데 r이 자연수이려면 (k2 + 3)이 제곱수여야 하고 (k + 2)2 >

k2 +3 > k2 이므로 k2 +3 = (k+1)2이다. 따라서 k = 1이고 이 때 r = q2, p = kq+ r = q+ q

2= 3q

2 이

다. 즉 a로 가능한 값은 2=3 하나 뿐이다.

답: 1개. }

58. xi들은 ¡1 < xi < 1 이고 jx1j + jx2j+ ¢ ¢ ¢ + jxnj = 19 + jx1 + ¢ ¢ ¢ + xnj 을 만족하는 실수들이다. n의가능한 최소값은 얼마인가? (AIME 1988-4)

풀이 %EEE

lhs < n, so we need n ≥ 20. Taking x_i = alternately ±19/20 works for

n = 20.

¢ ¢ ¢ 답 20 }

59. f1(x) = 1¡ jxj 이고, n = 1; 2; 3; : : : 에 해 fn+1(x) = 1¡ jfn(x)j 로 정의된다. 임의의 자연수 m에해, fk(m) = 0 이 되는 최소의 자연수 k를 구하여라. (플란더즈 예선 1998/1999 2차 변형)

풀이 jfk(x)j 의 절 값이 1보다 클 때 fk+1(x) = 1¡ jfk(x)j의 절 값은 jfk(x)j보다 1 작아진다. 임

의의 자연수 m이 있을 때 f1(m) = 1 ¡ jmj = 1 ¡m이고 fn(m)에서 n = 1; 2; 3; : : :로 바뀌어 갈수록fn(m)의 절 값은 1씩 작아진다.jf1(m)j = m¡ 1이므로 jfm(m)j = m¡m = 0이 최초로 0이고, 그 후 1과 0이 반복된다.

답: k = m: }

60. 다음 식을 계산하여라. (플란더즈 예선 1999/2000 1차)µ1 +

1

2

¶µ1 +

1

3

¶¢ ¢ ¢µ1 +

1

1999

¶µ1 +

1

2000

풀이 32¢ 43¢ ¢ ¢ 2000

1999¢ 20012000

= 20012

:

답: 20012 : }

61. 1 +1

4+1

9+1

16+1

25+ ¢ ¢ ¢ = ¼2

6임을 알고 있다고 할 때, 1 +

1

9+1

25+1

49+1

81+ ¢ ¢ ¢ 의 값을 구하여

라. (플란더즈 예선 2003 2차)

풀이 1 + 19+ 125+ 149+ 181+ ¢ ¢ ¢

= 1+( 13)2+( 1

5)2+( 1

7)2+( 1

9)2 = f1+( 1

2)2+( 1

3)2+( 1

4)2+( 1

5)2+ ¢ ¢ ¢ g¡f( 1

2)2+( 1

4)2+( 1

6)2+ ¢ ¢ ¢ g

= f1 + ( 12)2 + ( 1

3)2 + ( 1

4)2 + ( 1

5)2 + ¢ ¢ ¢ g ¡ 1

4f( 11)2 + ( 1

2)2 + ( 1

3)2 + ¢ ¢ ¢ g

= ¼2

6¡ 14¢ ¼26= ¼2

8:

¢ ¢ ¢ 답 ¼2=8 }

62. 정의역이 자연수인 함수 f(x)가 있다. f(1) = 1 이고 모든 자연수 m, n에 해서 f(m + n) = f(m) +f(n)¡ 2f(mn) 을 만족시킬 때 f(2005)와 f(2006)의 값을 차례로 구하여라.

(IT꿈나무 올림피아드 2006 1차)

풀이 n = 1 을 입하면 f(m + 1) = f(m) + 1 ¡ 2f(m) = 1 ¡ f(m). 따라서, f(1) = 1, f(2) = 0,f(3) = 1, f(4) = 0, : : : ; f(2005) = 1, f(2006) = 0. }

Page 74: 실전수학올림피아드 1400제 해답

74 대수

63. 정수의 집합에서 유리수의 집합으로의 두 함수 f , g가 임의의 정수 n에 해 다음 조건을 만족한다:f(0) = 4, f(1) = 12, g(n) = f(n+ 1), f(n) = 2g(n+ 1)¡ 2g(n¡ 1).(1) f(2), g(2)의 값을 구하여라.

(2) f(2n), g(2n)을 각각 n의 식으로 나타내어라.

(3) f(1995) + g(1995) 의 값을 구하여라. (1995 서울시)

풀이 (1) f(n) = 2g(n+ 1)¡ 2g(n¡ 1) = 2f(n+ 2)¡ 2f(n)) 3f(n) = 2f(n+ 2)) f(n+ 2) = 3

2f(n)

f(2) = 32f(0) = 6:

g(2) = f(3) = 32f(1) = 18:

(2) f(2n) = ( 32)nf(0) = ( 3

2)n ¢ 4 = 3n

2n¡2

g(2n) = f(2n+ 1) = ( 32)n ¢ f(1) = ( 3

2)n ¢ 12 = 3n+1

2n¡2 :

(3) f(1995) + g(1995) = g(1994) + f(1996) = ( 32)997 ¢ 12 + ( 3

2)998 ¢ 4 = 3999

2996}

64. 다음 등식을 관찰하여라.

13 = 1

23 = 3 + 5

33 = 7 + 9 + 11

43 = 13 + 15 + 17 + 19

.

..

위와 같은 방식으로 503 = m+ (m+ 2) + (m+ 4) + ¢ ¢ ¢+ n 이 된다고 할 때, (m;n)을 구하여라.(플란더즈 예선 2003 2차)

풀이 규칙을 살펴보면 1부터 홀수를 늘어놓고 앞에서부터 각각 한 개, 두 개, 세 개씩 묶어 더하면 세

제곱수가 되는 규칙을 알 수 있다. 그러므로 503은 (1 + 2 + 3 + ¢ ¢ ¢ 49 + 1(번째 홀수부터 (1 + 2 + 3 +¢ ¢ ¢ 49) + 50번째 홀수까지의 합이다.1+2+3+ ¢ ¢ ¢ 49+1번째 홀수: (1+2+3+ ¢ ¢ ¢ 49+1) ¢2¡1 = (49 ¢50=2+1) ¢2¡1 = 49 ¢50+1 = 2451:1 + 2 + 3 + ¢ ¢ ¢ 49 + 50번째 홀수: (51 ¢ 50=2) ¢ 2¡ 1 = 2550¡ 1 = 2549:¢ ¢ ¢ 답 (2451; 2549) }

65. 임의의 실수 a, b, c, d, e에 해 다음 부등식을 증명하여라. (몰도바 1998 최종-y9-1)

a2 + b2 + c2 + d2 + e2 ¸ a(b+ c+ d+ e)

증명 다음 식을 전개하면 준식과 같다.

( 12a¡ b)2 + ( 1

2a¡ c)2 + ( 1

2a¡ d)2 + ( 1

2a¡ e)2 ¸ 0

따라서, 준식은 성립하고, 등호는 12a = b = c = d = e 일 때 성립한다. ¤

주 산술-기하평균 부등식에서 14a2 + b2 ¸ 2

q14a2b2 = jabj ¸ ab 임 등을 이용해도 된다. 산술-기하

평균 부등식은 음이 아닐 경우에만 적용되므로 이와 같이 절 값으로 풀어놓은 후 비교해야 한다는 점을 주의할 것.

별증 a에 한 식으로 보고(b; c; d; e는 상수로 보고) 내림차순으로 정리하자.

a2 ¡ (b+ c+ d+ e)a+ (b2 + c2 + d2 + e2) ¸ 0이 a에 한 이차식이 모든 실수 a에 해 항상 0 이상이어야 하므로 그래프가 x-축과 접하거나 그보다위쪽에 놓여야한다. 즉, 이 부등식이 항상 성립할 필요충분조건은 판별식 D · 0 이다.

D = (b+ c+ d+ e)2 ¡ 4(b2 + c2 + d2 + e2)

인데, 코시-슈바르츠 부등식에서 (1 + 1+ 1+ 1)(b2 + c2 + d2 + e2) ¸ (b+ c+ d+ e)2 이므로 D · 0 은틀림없이 성립하고, 따라서, 문제도 성립한다. ¤

Page 75: 실전수학올림피아드 1400제 해답

2.1 대수 중급문제 75

66. m이 임의의 자연수이고

x = m2; y = (m+ 1)2; z = 2(x+ y + 1)

이면 다음 여섯 개의 수는 모두 제곱수임을 보여라. (셈본중등초급 도전문제 3.1.1)

(1) xy + x+ y (2) yz + y + z (3) zx+ z + x(4) xy + z (5) yz + x (6) zx+ y

증명 (1)은 다음과 같이 확인된다.

xy + x+ y = fm(m+ 1)g2 +m2 + (m+ 1)2 = (m2 +m)2 +m2 + (m+ 1)2

= m4 + 2m3 +m2 +m2 +m2 + 2m+ 1

= m4 + 2m3 + 3m2 + 2m+ 1 = (m2 +m+ 1)2

(2){(6)도 다 m에 한 식으로 풀어보면 다음과 같다.

(2) yz + y + z = 4m4 + 12m3 + 21m2 + 18m+ 9 = (2m2 + 3m+ 3)2

(3) zx+ z + x = 4m4 + 4m3 + 9m2 + 4m+ 4 = (2m2 +m+ 2)2

(4) xy + z = m4 + 2m3 + 5m2 + 4m+ 4 = (m2 +m+ 2)2

(5) yz + x = 4m4 + 12m3 + 17m2 + 12m+ 4 = (2m2 + 3m+ 2)2

(6) zx+ y = 4m4 + 4m3 + 5m2 + 2m+ 1 = (2m2 +m+ 1)2

따라서, 모두 제곱수이다. ¤

주 (1)을 (px¡py )2 + (

pxy ¡ 1)2 ¡ 1 = (pxy + 1)2 등으로 좀더 요령있게 정리할 수도.

67. 1, 2, 3, : : : ; 1000 중에서 적당한 실수 x에 해 [2x] + [4x] + [6x] + [8x]꼴로 나타나는 것은 모두 몇 개인가? (AIME 1985-10)

풀이 %EEE

Put f(x) = [2x] + [4x] + [6x] + [8x]. We find f(1/8) = 1, f(1/6) = 2,

f(1/4) = 4, f(1/3) = 5, f(3/8) = 6, f(1/2) = 10, and f(x + n/2) =

f(x) + 10n. So we realise 6 out of 10, or 600 out of 1000.

¢ ¢ ¢ 답 600 }

68. 수직선에서 좌표가 10인 점 P (10)을 중심으로 양쪽에 칭으로 놓여 있는 24개의 서로 다른 정수 좌표를갖는 점 P1(x1), P2(x2), P3(x3), : : : ; P24(x24)에 하여

(jx1 ¡ 1j+ jx1 + 1j) + (jx2 ¡ 1j+ jx2 + 1j) + ¢ ¢ ¢+ (jx24 ¡ 1j+ jx24 + 1j)

의 최소값을 구하여라. (한국 2006 1차-J15)

풀이 한 점의 좌표를 k라 하면 P (10)에 칭인 점의 좌표는 20¡ k가 된다. 이렇게 쌍으로 묶어 생각

하면 k < 10 으로 둘 수 있다. 정수 n 6= 0 에 해 jn¡ 1j+ jn+ 1j = 2jnj 이므로

(i) 1 · k < 10 이면: 20 ¡ k ¸ 1 이므로 A = (jk ¡ 1j + jk + 1j) + (j20 ¡ k ¡ 1j + j20 ¡ k + 1j) =2k + 2(20¡ k) = 40.

(ii) k = 0 이면: 20¡ k = 20 이므로 A = 2 + 2 ¢ 20 = 42.(iii) k · ¡1 이면: 20¡ k ¸ 1 이므로 A = ¡2k + 2(20¡ k) = 40¡ 4k. k · ¡1 이므로 k가 작아질수

록 A의 값은 커지게 된다. 즉 A ¸ 44 가 된다.

즉, k가 클수록 A값이 작고, 가장 작은 242= 12개를 합하면 9£ 40 + 42 + 44 + 48 = 494 ¢ ¢ ¢ 답 }

69. 가우스함수와 사칙연산만으로, 0이면 1을, 아니면 0을 돌려주는 함수를 구현해보아라.(ML프로포절 136-2)

풀이

·1

1 + a2

¸(a2 = a£ a 로 구현한다.) 답은 이외에도 여러 가지가 있을 수 있다. }

Page 76: 실전수학올림피아드 1400제 해답

76 대수

70. 수열 1; 2; 4; 5; 10; 11; 22; 23; 46; 47; : : : 는 다음과 같이 만들어진 것이다: 1에 1을 더해 2를 얻고, 2를 2배하여 4를 얻고; 4에 1을 더해 5을 얻고, 5를 2배하여 10을 얻고; : : : ; 이렇게 1을 더하는 것과 2배하는 것을 계속하여 수열을 얻는다. 이 수열의 100번째 항은 3 ¢ 2k ¡ 1 꼴이다. k를 구하여라.

(1989 뉴욕주 수학리그)

풀이 a2 = 2, a2k+2 = 2a2k + 1 이므로

a2k+2 + 1 = 2(a2k + 1) = ¢ ¢ ¢ = 2k(a2 + 1) = 3 ¢ 2k

따라서, a2k+2 = 3 ¢ 2k ¡ 1 이다. 2k + 2 = 100 일 때이므로 k = 49 ¢ ¢ ¢ 답 }

71. 임의의 실수들의 수열 A = (a1; a2; a3; : : : ) 에 해, ¢A를 수열 (a2 ¡ a1; a3 ¡ a2; a4 ¡ a3; : : : ) 로 정의한다. ¢(¢A)의 모든 항이 1이고 a19 = a92 = 0 일 때, a1은 얼마인가? (AIME 1992)

풀이 ¢(¢A) = (1; 1; 1; 1; : : : ) 이므로 ¢A = (b; b+ 1; b+ 2; b+ 3; : : : ) 가 된다. 그럼 또

A = (a; a+ b; a+ b+ (b+ 1); a+ b+ (b+ 1) + (b+ 2); : : : )

즉,

an = a+ b+ (b+ 1) + ¢ ¢ ¢+ (b+ (n¡ 2)) = a+ (n¡ 1)b+ (n¡ 2)(n¡ 1)2

이다. a19 = a92 = 0 임에서

a+ 18b+ 17 ¢ 9 = 0a+ 91b+ 45 ¢ 91 = 0

이고, 여기서 b를 소거하기 위해 두 식에 각각 91, 18을 곱하여 빼면

73a+ 91 ¢ 9 ¢ (17¡ 90) = 0

즉, a = 91 ¢ 9 = 819 ¢ ¢ ¢ 답 }

72. 방정식 x6 ¡ x5 + x4 ¡ x3 + x2 ¡ x+ 34= 0 이 실수해를 갖지 않음을 보여라. (스웨덴 1986-1)

증명1 준식 = (x¡ 12)2x4 + 3

4(x¡ 2

3)2x2 + 2

3(x¡ 3

4)2 + 3

8¸ 38> 0. ¤

증명2 준식 = x(x ¡ 1)(x4 + x2 + 1) + 34. x ¸ 1 또는 x · 0 이면 x(x ¡ 1) ¸ 0 이므로 준식 ¸ 3

4.

0 < x < 1 에서는 jx(x¡ 1)j = j(x¡ 12)2 ¡ 1

4j · 1

4, x4 + x2 + 1 < 3 이므로 준식 > ¡ 1

4¢ 3 + 3

4= 0. ¤

73. a ¸ b ¸ c 와 a+ b+ c · 1 을 만족하는 양수 a, b, c가 있다. a2 + 3b2 + 5c2 · 1 임을 보여라.(Towns 1989봄 JO1)

증명 a2 + 3b2 + 5c2 · a2 + (2ab+ b2) + (c2 + 2ac+ 2bc) = (a+ b+ c)2 · 1. ¤

74. 다음 방정식이 무한히 많은 해를 갖게 되도록 하는 a의 값을 모두 구하여라.

jx¡ aj+ jx+ 3a¡ 8j = 4 (오클랜드 2001-4)

풀이 b = ¡3a+ 8 이라 하고 jx¡ aj+ jx¡ bj = 4 의 방정식으로 생각하자. 또, 수직선에서 x, a, b의

위치의 점을 각각 X, A, B라 하자. 그럼 방정식은 XA+XB = 4 로 다시 쓸 수 있다.

² AB > 4 이면 XA+XB ¸ AB 이므로 해가 없다.

² AB < 4 이면 선분 AB 내에서는 XA+XB = AB 이므로 해가 없고, 선분 AB의 밖에서는 이 선분에서 멀어질수록 XA+XB 가 단조증가하므로 해가 양쪽에 각각 하나뿐이다. 즉, 해를 유한 개밖에 갖지 않는다.

² AB = 4 일 때 선분 AB 내의 모든 점이 XA+XB = AB = 4 를 만족하므로, 이것이 무한히 많은해를 가질 필요충분조건이 된다.

그러므로, AB = ja¡ bj = ja¡ (¡3a+8)j = 4 를 풀면 j4a¡ 8j = 4, ja¡ 2j = 1, 따라서 ¢ ¢ ¢ 답 a = 1

또는 3 }

Page 77: 실전수학올림피아드 1400제 해답

2.1 대수 중급문제 77

주 y = jx¡ aj+ jx¡ bj 의 그래프를 이용하여 풀 수도 있고(y = c 와 무한히 많은 교점을 갖는 경우

는 a와 b 사이의 평평한 선분이 y = c 가 되는 경우임), x¡ a 와 x¡ b 의 부호에 따라 경우를 나누어 풀수도 있다.

75. n£ n 표의 각각의 칸에 실수가 하나씩 채워져 있다. 임의의 십자교차영역(한 행과 한 열의 합집합)의 수를 모두 합하면 항상 A 이상이다. 모든 칸에 적힌 수의 총합의 가능한 최소값은 얼마인가?

(독일BW 1972 1차-1)

풀이 모든 칸에 해 그 칸을 교차점으로 포함하는 십자교차영역을 모두 합해보자. 그럼 n2개의 십자

교차영역을 합하므로 그 값은 n2A 이상이고, 또한 각 칸은 그 칸을 지나는 십자교차영역이 n+n¡1개씩

있으므로 (2n¡ 1)S가 된다(S는 모든 칸에 적힌 수의 총합). 즉 (2n¡ 1)S ¸ n2A 이므로 S ¸ n2A2n¡1 . 실

제로 모든 칸의 수가 A2n¡1이면 문제의 조건을 충족하면서 등호가 성립하는 경우가 된다. ¢ ¢ ¢ 답 n2A

2n¡1}

76. 다음을 증명하여라. (Towns 1991가을 JO3)

1

2 +1

3 +1

4 +1

¢ ¢ ¢+ 1

1991

+1

1 +1

1 +1

3 +1

4 +1

¢ ¢ ¢+ 1

1991

= 1

증명 n =1

3 +1

4 +1

¢ ¢ ¢+ 1

2007

이라 하자. 그럼

준식 =1

2 + n+

1

1 +1

1 + n

=1

2 + n+

12 + n

1 + n

=1

2 + n+1 + n

2 + n= 1

로 문제의 식이 성립함을 알 수 있다. ¤

77. n, m은 m > n 인 자연수이다.qn+

pm+

pn 과

qm+

pn+

pm 중 어느 것이 더 큰가?

(Towns 1992봄 JA3 변형)

풀이 둘다 양수이므로 둘을 제곱한 A2 = n+pm+

pn 과 B2 = m+

pn+

pm 을 비교하면 된다.

B2 ¡A2 = (m¡ n) + (pn+

pm¡

pm+

pn )

= (m¡ n) +(n+

pm )¡ (m+

pn )p

n+pm+

pm+

pn

= (m¡ n)¡ m¡ n

X+

pm¡pn

X(X :=

pn+

pm+

pm+

pn )

> (m¡ n)

µ1¡ 1

X

¶(* pm¡pn > 0 이므로)

> 0 (X > 1 임이 자명하고 m > n 이므로)

로 B2 > A2 이다. 즉, 후자가 항상 더 크다. }

78. 양의 실수 2개가 주어져있다. 두 수의 합이 두 수의 곱보다 작다면 그 합은 4보다 큼을 증명하여라.(Towns 1990가을 JO1)

증명 (a+ b)2 ¸ 4ab > 4(a+ b) 이므로 a+ b > 4. ¤

Page 78: 실전수학올림피아드 1400제 해답

78 대수

79. 1 <p3 < 2 임을 이용하여, 6 < 3

p3 < 7 임을 증명하여라. (IMTS R10-2)

증명 0 <p3¡ 1 < 1 이고 제곱하면 0 < 4¡ 2p3 < 1, 즉 3

2<p3 < 2. 다시 0 < 2¡p3 < 1

2 이고

제곱하면 0 < 7¡ 4p3 < 14, 즉 27

16<p3 < 7

4. 이제 327=16 < 3

p3 < 37=4 이므로 616 < 327 과 37 < 74

을 확인하면 충분함. 전자는 216 < 311 과 동치인데 216 = (23)5 ¢ 2 < (32)5 ¢ 3 = 311 로 확인되고, 후자는 37 = (33)2 ¢ 3 < (4 ¢ 7)2 ¢ 3 < 74 으로 확인됨. ¤

80. 방정식 x¡ [x] = [ 12x¡ 2] 의 모든 실수해를 구하여라. (몰도바 1996 최종-y8-6)

풀이 x는 두 가우스식의 합이므로 정수. 그럼 좌변은 0. 그럼 0 · 12x¡ 2 < 1 이고 4 · x < 6. 정수

인 것은 x = 4; 5뿐이고 각각 입해보면 실제로 성립함. ¢ ¢ ¢ 답 4, 5 }

81. 1보다 큰 임의의 자연수 n에 해 다음을 증명하여라. (인도지역예선 2002-6)

1

2<

1

n2 + 1+

2

n2 + 2+ ¢ ¢ ¢+ n

n2 + n<1

2+1

2n

증명 분모를 n2이나 n2 + n으로 통일해보면 끝. ¤

82. 1월에 K와 V 는 학교에서 20번 평가를 받았고 20회씩 점수를 받았다. 각 점수는 2에서 5까지의 정수이다.K가 5점을 받은 횟수는 V 가 4점을 받은 횟수와 같고, K가 4점을 받은 횟수는 V 가 3점을 받은 횟수와 같으며, K가 3점을 받은 횟수는 V 가 2점을 받은 횟수와 같고, K가 2점을 받은 횟수는 V 가 5점을 받은 횟수와 같다. 두 사람의 평균점이 같다면, K가 2점을 받은 횟수는 몇 번인가? (Towns 1988봄 JO1)

풀이 K가 5점, 4점, 3점, 2점을 받은 횟수를 각각 a, b, c, d라 하자. 그럼 V 가 5점, 4점, 3점, 2점을 받

은 횟수는 각각 d, a, b, c. 두 사람의 총점이 같으므로 5a + 4b + 3c + 2d = 4a + 3b + 2c + 5d, 여기서a+ b+ c = 3d, 즉 4d = a+ b+ c+ d = 20 이고 d = 5 ¢ ¢ ¢ 답 }

83. 양의 실수 a, b에 해 다음 부등식을 증명하여라. (러시아 1993 4차-y9-1)

a2 + ab+ b2 ¸ 3(a+ b¡ 1)

증명 (a¡ 1)2 + (a¡ 1)(b¡ 1) + (b¡ 1)2 ¸ 0 ¤

84. 포물선 y = x2 에 접하는 모든 포물선 y = ¡x2 + bx+ c 의 꼭지점의 자취를 구하여라.(러시아 1991 4차-y9-1)

풀이 같다고 놓고 D = 0 이용하면 c = ¡ b2

8. 꼭지점은 ( b

2; b2

8). ¢ ¢ ¢ 답 y = 1

2x2. }

85. 방정식 ax5 + bx4 + c = 0 이 서로 다른 세 근을 갖는다. 방정식 cx5 + bx+ a = 0 도 서로 다른 세 근을가짐을 보여라. (러시아 1994 4차-y9-5)

증명 c 6= 0 이고, 그럼 양변을 x5으로 나눠보면 끝. ¤

86. 다음 식을 인수분해하여라. (방글라데시 2004 2차-1)

(a+ b)2(b+ c)2(c+ a)2 + abcf2(a+ b)(b+ c)(c+ a) + abcg

풀이 f(a+ b)(b+ c)(c+ a) + abcg2 = (ab+ bc+ ca)2(a+ b+ c)2 }

87. a0 = ® > 0 이고 an+1 =an

1 + an으로 주어진 수열 (an)의 일반항을 구하여라. (이탈리아 1986-2)

풀이 역수를 생각하면 쉽... 1an+1

= 1an+ 1 의 등차수열이므로 1

an= 1

®+ (n¡ 1). }

88. a, b, c, x, y는 a3 + ax+ y = 0, b3 + bx+ y = 0, c3 + cx+ y = 0 을 만족하는 다섯 개의 실수이다. a, b,c가 모두 서로 다르면 그 합이 0임을 보여라. (주니어발칸 1999-1)

Page 79: 실전수학올림피아드 1400제 해답

2.1 대수 중급문제 79

증명 f(t) = t3 + xt+ y = 0 의 세 근이 a, b, c이므로 근과 계수와의 관계로 끝. ¤

89. 어떤 정수 m;n ¸ 2 에 해서도, mpn과 n

pm 중 적어도 하나는

3p3보다 크지 않음을 증명하여라.

(몰도바 1996 최종-y10-5)

증명 k ¸ 2 일 때 3k ¸ k3 임을 이용. 귀류법을 쓰면 m3n3 > 3n3m ¸ m3n3 으로 모순. ¤

90. 다음 등식을 증명하여라. (헝가리 1941-1)

(1 + x)(1 + x2)(1 + x4)(1 + x8) ¢ ¢ ¢ (1 + x2k¡1

) = 1 + x+ x2 + x3 + ¢ ¢ ¢+ x2k¡1

증명 지수를 2진법으로 전개한다고 생각. ¤

91. f가 다음 함수 방정식을 만족한다고 하자.

2f(x) + 3f

µ2x+ 29

x¡ 2¶= 100x+ 80

f(3)의 값을 구하여라. (IMTS R19-4)

풀이 g(x) = 2x+29x¡2 라 하면 g(g(x)) = x. g(3) = 35, g(35) = 3. x = 3; 35 입하고 연립방정식 풀면

f(3) = 1996. }

92. 모든 실수 x에 해 다음을 만족하는 함수 f : R! R 를 모두 구하여라.

x2f(x) + f(1¡ x) = 2x¡ x4 (오스트리아 1985-6)

풀이 연립방정식으로 풀면 될테고... }

93. 삼차방정식 x3 ¡ 3px2 + 3q2x¡ r3 = 0 의 세 근이 p, q, r일 때 p = q = r 임을 증명하여라.(인도지역예선 1999-4)

증명 근과 계수와의 관계에서 (1) p+ q + r = 3p, (2) pq + qr + rp = 3q2, (3) pqr = r3. 만일 r = 0

이면 (1)에서 q = 2p 이고 (2)에서 2p2 = 12p2 이므로 p = 0 이 되어 성립. p, q 중에 0인 것이 있으면(3)에서 r = 0 이 되어 마찬가지. 이제 p; q; r 6= 0 일 때를 보자. (3)에서 pq = r2 이고 이것과 (1)을 (2)에

입하면 3pr = 3q2, 즉 pr = q2. 이것을 다시 (2)에 입하면 q(p+ q+ r) = 3q2, 즉 p+ q+ r = 3q 이고 (1)에 의해 p = q. 그럼 r도 같게 된다. ¤

94. 실수 a, b, c, d가 부등식 (a+ b+ c+ d)2 ¸ 4(a2 + b2 + c2 + d2) 을 만족할 때, 방정식 (a¡ x)(b¡ x) +(c¡ x)(d¡ x) = 0 에 실수해가 있음을 증명하여라. (몰도바 1999 최종-y8/9-2)

증명 D = (a+ b+ c+ d)2 ¡ 8(ab+ cd) ¸ (a+ b+ c+ d)2 ¡ 4(a2 + b2 + c2 + d2). ¤

95. 용량이 각각 a리터와 b리터인 두 통이 각각 다른 주스로 채워져 있다. 단, 두 주스는 리터당 단가가 서로 다르고, a와 b는 자연수이다. 각각의 통으로부터 동시에 999리터의 주스를 빼내서 각각 상 편의 통에 옮겨부었더니 두 통 안의 최종 혼합물의 리터당 단가가 같게되었다. 이런 상황이 가능한 a와 b 중에서 a+ b를최소로 하는 것을 찾아라. (몰도바 1998 최종-y7/8-4)

풀이(a¡ 999)x+ 999y

a=(b¡ 999)y + 999x

b를 정리하면 (a ¡ 999)(b ¡ 999) = 9992. a + b =

(a¡ 999) + (b¡ 999) + 2 ¢ 999 ¸ 2p(a¡ 999)(b¡ 999) + 2 ¢ 999 = 4 ¢ 999. 등호는 a = b = 2 ¢ 999 일

때. }

96. 양수 x, y에 해 다음 부등식을 증명하여라. (러시아 1995 4차-y9-1)

x

x4 + y2+

y

y4 + x2· 1

xy

증명 양변에 xy를 곱한 후 분자를 기하-산술 부등식으로 전개하면 끝. 곱하지 않고 그냥 분모만 산

술-기하로 풀어도 마찬가지네. ¤

Page 80: 실전수학올림피아드 1400제 해답

80 대수

97. 다음 식을 증명하여라. (러시아 1994 최종-y9-5)

a1a2(a1 + a2)

+a2

a3(a2 + a3)+¢ ¢ ¢+ an

a1(an + a1)=

a2a1(a1 + a2)

+a3

a2(a2 + a3)+¢ ¢ ¢+ a1

an(an + a1)

풀이1 왼쪽 식에서 오른쪽 식을 뺀 후 좀 정리해보면 금방... }

풀이2 부분분수로 각 변을 전개해도 금방. }

98. 모든 자연수 n에 해 다음을 보여라. (아일랜드 1996-7)

21=2 ¢ 41=4 ¢ 81=8 ¢ ¢ ¢ ¢ ¢ (2n)1=2n < 4

증명 S = 12+ 24+ 38+ ¢ ¢ ¢ + n

2n< 2 를 증명하면 됨. 등비수열과 등차수열이 복합된 꼴. 2S ¡ S =

1 + 12+ 14+ ¢ ¢ ¢+ 1

2n¡1 ¡ n2n, S = 2¡ 1

2n¡ n2n

< 2. ¤

99. 다음 연립방정식의 모든 실수해 x1; x2; : : : ; x1999을 구하여라. (몰도바 1999 최종-y7-3)

1 + x21 = 2x2; 1 + x22 = 2x3; : : : ; 1 + x21999 = 2x1

풀이 다 합하여 좌변으로 다 넘기면P(xi ¡ 1)2 = 0 이므로 x1 = ¢ ¢ ¢ = x1999 = 1. }

100. 다음 연립방정식을 만족하는 실수 순서쌍 (x; y; z)를 모두 구하여라. (남미 1989-1)

x+ y ¡ z = ¡1x2 ¡ y2 + z2 = 1

¡x3 + y3 + z3 = ¡1

풀이 세 식을 각각 (1), (2), (3)이라 할 때 (1)2 ¡ (2) 하면 (y + x)(y ¡ z) = 0 으로 거의 끝. ¢ ¢ ¢ 답(¡1;¡1;¡1), (1;¡1; 1) }

101. 자연수 a, b, c에 해 다음 부등식을 증명하여라. (몰도바 1999 최종-y7-7)

a2

a2 + 2bc+

b2

b2 + 2ca+

c2

c2 + 2ab¸ 1

증명 AM-GM으로 분모를 a2 + b2 + c2 으로 통일하면 끝. ¤

102. (x+px2 + 1 )(y +

py2 + 1 ) = 1 이면 x+ y = 0 임을 보여라. (러시아 1994 최종-y9-1)

증명 (x+px2 + 1 )(y +

py2 + 1 ) = (x¡px2 + 1 )(y ¡

py2 + 1 ) 을 전개해서 소거. ¤

103. a는 a3 = 6(a+ 1) 인 양의 실수이다. 방정식 x2 + ax+ a2 ¡ 6 = 0 는 실수해를 갖지 않음을 증명하여라.(주니어발칸 2007-1)

증명 a > 2p2 만 보이면 됨. ¤

104. 0이 아닌 임의의 실수 x에 해 x8 ¡ x5 ¡ 1

x+1

x4¸ 0 임을 증명하여라. (아일랜드 1998-1)

증명 그냥 인수분해 (x9 ¡ 1)( 1x¡ 1

x4) = 1

x4(x3 ¡ 1)2(x6 + x3 + 1) ¸ 0 ¤

Page 81: 실전수학올림피아드 1400제 해답

2.1 대수 중급문제 81

105. 서로 다른 실수 a1; a2; : : : ; an이 주어졌을 때, 다음 함수의 최소값을 구하여라. (폴란드 1969 3차-2)

y = jx¡ a1j+ jx¡ a2j+ ¢ ¢ ¢+ jx¡ anj; x 2 R

풀이 WLOG a1 < a2 < ¢ ¢ ¢ < an 이라 할 때, 삼각부등식으로 y ¸ ja1 ¡ anj+ ja2 ¡ an¡1j+ ¢ ¢ ¢ +jabn

2c ¡ adn

2ej. }

106. 다음 연립방정식의 모든 실수해를 구하여라. (Towns 1985봄 JO3)

(x+ y)3 = z

(y + z)3 = x

(z + x)3 = y

풀이1 양변에 1=3승을 취하면

x+ y = z1=3

y + z = x1=3

z + x = y1=3

즉,

x+ y + z = z + z1=3 = y + y1=3 = x+ x1=3

함수 f(x) = x+ x1=3 은 일 일 응이다. 따라서

x = y = z

이다. 입해보면 8x3 = x 에서 x = 0 또는 8x2 = 1. 따라서, (x; y; z) = (0; 0; 0), §(1=p8; 1=

p8; 1=

p8)

}

풀이2 첫번째 식에서 두번째 식을 빼보면

z ¡ x = (x¡ z)f(x+ y)2 + (x+ y)(y + z) + (y + z)2g

이 되어서 x = z 가 아니라면

¡1 = f(x+ y)2 + (x+ y)(y + z) + (y + z)2g

가 된다.그런데 우변은 x, y, z가 실수일 때 절 로 0보다 작아질 수 없다.따라서 x = z. 결국 x = y = z가되고, 이제 앞의 풀이처럼 입하여 해를 구하면 된다. }

풀이3 개요 (1) 하나가 0이면 나머지도 0.

(2) x > 0 > y 이면 x+ z < 0 그럼 x+ z < 0 그럼 z < 0 또 x < 0 모순.(3) x; y; z > 0 이면 x ¸ y ¸ z 가정. 그럼 y ¸ x, 즉 x = y = z. }

107. 임의의 실수 a에 해서 다음 부등식을 증명하여라. (몰도바 2000 최종-y8-7)

(a3 + a2 + 3)2 > 4a3(a¡ 1)2

증명1 전개해서 몇 개씩 완전제곱의 합으로 나타내기. ¤

증명2 a < 0 일 땐 L ¸ 0 > R 이라서 당연. 0 · a · 1 일 땐 L ¸ 9 > 4 > R 로 역시 당연. a > 1 일

땐 우변의 8a4을 좌변으로 이항해놓고, 좌변의 항들 중 a6 + 4a4 ¸ 4a5 과 6a3 > 4a3 만을 생각하면 당연. ¤

Page 82: 실전수학올림피아드 1400제 해답

82 대수

108. 다음 방정식의 실수해 x는 둘뿐임을 증명하여라. (아일랜드 2004-4)

(x¡ 1)(x¡ 2)(x¡ 3)(x¡ 4)(x¡ 5)(x¡ 6) = 720

증명 720 = 6! 이므로 0과 7은 해가 되고 나머지는 x에 한 증가감소를 생각하면 더 이상 없음이 금

방... ¤

109. 두 이차다항식 x2 + ax+ b 와 x2 + bx+ a 가 모두 삼차다항식 x3 + px2 + qx+72 의 인수라고 한다. 단,a, b, p, q는 상수이고, a 6= b 이다. 이 삼차다항식의 세 근을 구하여라. (뉴멕시코 1992 최종-7)

풀이 두 이차식의 공통근 ®가 있어야 하고, 두 식을 변변 빼면 ® = 1. 두 식의 나머지 근은 각각 b와

a. 근과 계수와의 관계로 충 풀면 세 근은 1, 8, ¡9. }

110. 함수 f(x) =1

1¡ x+ x2를 모든 실수 x에 해 g(¡x) = g(x), h(¡x) = ¡h(x) 를 만족하는 두 함수의

합 f(x) = g(x) + h(x) 로 나타내어라. (뉴멕시코 1995 1차-7)

풀이 f(¡x) = g(x)¡h(x) 이므로 g(x) = 12(f(x)+ f(¡x)) = 1+x2

1+x2+x4, h(x) = 1

2(f(x)¡ f(¡x)) =

x1+x2+x4

. }

111. (1) 임의의 실수 x 6= 1 에 해 f( xx¡1 ) = x 를 만족하는 함수 f를 구하여라.

(2) 임의의 실수 x 6= 1 에 해 g( xx¡1 ) = g(x) 를 만족하는 함수 g가 무한히 많음을 보여라. (단, 하나

가 다른 것의 상수배가 되는 것은 다시 세지 않는다.) (뉴멕시코 1998 최종-7)

증명 (1) y = xx¡1 로 치환하면 f(y) = y

y¡1 . (2) f(f(x)) = x 임에서, f와 항등함수 I를 칭적으로

사용한 함수는 모두 g가 됨. 예를 들어 (f(x)I(x))n 이나, f(x)n + I(x)n 등. ¤

112. 0 · x · 1, 0 · y · 1 인 실수 x, y에 해 항상x

1 + y+

y

1 + x· 1 이 성립함을 증명하여라.

(레닌그라드 1988-y7-1)

증명x

1 + y+

y

1 + x· x

x+ y+

y

x+ y= 1. x+ y = 0 일 때도 당연. ¤

113. 임의로 주어진 실수 상수 a < b < c 에 해, 다음 등식이 해를 딱 두 개 가짐을 증명하여라.(레닌그라드 1988-y9-3)

1

x¡ a+

1

x¡ b+

1

x¡ c= 0

증명 구간 (a; b), (b; c)에 각각 한 개씩 해가 있음. 중간값 정리의 개념을 풀어서 서술해줄 것. ¤

114.p2 +

p3 과

p2 + 3

p3 을 모두 근으로 갖는 정계수 다항식을 하나 찾아라. (스웨덴 1964-3)

풀이 각 근을 갖는 다항식을 구한 후 곱하면 되겠고, 그냥 열심히 (x¡p2 )2 = 3 등으로 근호를 차근

차근 벗겨가면 됨. ¢ ¢ ¢ 답 (x4 ¡ 10x2 + 1)(x6 ¡ 6x4 ¡ 6x3 + 12x2 ¡ 36x+ 1) }

115. 1이 아닌 실수 x에 해 (1 + x+ x2)2 < 3(1 + x2 + x4) 이 성립함을 증명하여라. (스웨덴 1971-1)

증명 전개해서 이항/소거한 후 2로 나누면 준식은 x4 ¡ x3 ¡ x + 1 > 0 과 동치이고, 인수분해하면

(x¡ 1)(x3 ¡ 1) > 0 과 동치. 이것은 참. ¤

별증 1 + x2 + x4 = (1 + x+ x2)(1¡ x+ x2) 으로 인수분해되고 1 + x+ x2 > 0 이므로 이것으로 준

식의 양변을 나눠주면 1 + x+ x2 < 3(1¡ x+ x2) 과 동치. 이것은 정리하면 0 < (1¡ x)2 과 동치. ¤

Page 83: 실전수학올림피아드 1400제 해답

2.1 대수 중급문제 83

116. 임의의 자연수 n과 양의 실수 r에 해 다음 부등식이 성립함을 증명하여라.

1¡ 1r· n(r1=n ¡ 1) · r ¡ 1 (스웨덴 1974-2)

증명 r1=n = a 로 치환하자. a > 1 이면 an¡1a¡1 = an¡1 + an¡2 + ¢ ¢ ¢ + a + 1 > n 임에서, a < 1

일 때도 비슷하게 하여 오른쪽 부등식이 성립. 왼쪽 부등식도 비슷하게, a > 1 이면 1an

¢ an¡1a¡1 =

a¡1 + a¡2 + ¢ ¢ ¢+ a¡n < n 임에서, a < 1 일 때도 비슷하게 하여 성립. ¤

별증 r; 1; : : : ; 1 에 산술-기하평균 부등식을 쓰면 r1=n · r+(n¡1)n

, 정리하면 n(r1=n ¡ 1) · r ¡ 1 로

오른쪽 부등식이 성립. 왼쪽 부등식은 r(n+1)=n; : : : ; r(n+1)=n; 1 (총 n+ 1개항) 에 산술-기하평균을 사용하면 됨. ¤

117. 양수 a, b, c에 해 다음 부등식이 성립함을 보여라. (러시아 1990 4차-y10-1)pab(a+ b) +

pbc(b+ c) +

pca(c+ a) >

p(a+ b)(b+ c)(c+ a)

증명 한 번 제곱하여 항들을 적당히 소거하고 나면 우변이 2abc만 남고,양변 2로 나누면 좌변의 각 항이

우변의 abc보다 큼. 혹은 제곱하여 소거한 후 좌변에 남은 세 항 중 한 항만으로도 2pab2c(a+ b)(b+ c) >

2pab2c(a+ 0)(0 + c) = 2abc 이므로 끝. ¤

118. a; b; : : : ; n이 3보다 큰 어떤 소수로도 나누어지지 않는 서로 다른 자연수들일 때, 다음 부등식을 증명하여라. (헝가리 1922-3)

1

a+1

b+ ¢ ¢ ¢+ 1

n< 3

증명 1a+ 1

b+ ¢ ¢ ¢+ 1

n< (1 + 1

2+ 122+ ¢ ¢ ¢ )(1 + 1

3+ 132+ ¢ ¢ ¢ ) = 2 ¢ 3

2= 3 ¤

119. 연립방정식 x = a¡ y2, y = a¡ x2 이 서로 다른 실수해 x, y가 존재하는 실수 a를 모두 구하여라.(스웨덴 1976-2)

풀이 두 식을 변변 빼면 x ¡ y = x2 ¡ y2, x ¡ y 로 나누면 x + y = 1. 즉 x, y는 둘 다 이차식

t2 ¡ t+ (1¡ a) = 0 을 만족. 판별식이 양이어야 하므로 1 > 4(1¡ a), 즉 a > 34. }

120. cd = 1 인 양수 a, b, c, d가 주어져 있다. 폐구간 [ab; (a+ c)(b+ d)] 에 완전제곱수가 적어도 하나 포함됨을 증명하여라. (레닌그라드 1987-38)

증명 완전제곱수가 포함되지 않는다면, n2 < ab < (a+c)(b+d) < (n+1)2 을 만족하는 음 아닌 정수

n이 존재. 그런데, (n+1)2 = n2 +2n+1 < ab+2pab+1 = ab+2

pabcd+ cd · ab+ ad+ bc+ cd =

(a+ c)(b+ c) 로 모순. ¤

121. 다음 연립방정식을 풀어라. (스웨덴 1979-1)

x1 + 2x2 + 3x3 + ¢ ¢ ¢+ (n¡ 1)xn¡1 + nxn = n

2x1 + 3x2 + 4x3 + ¢ ¢ ¢+ nxn¡1 + xn = n¡ 13x1 + 4x2 + 5x3 + ¢ ¢ ¢+ xn¡1 + 2xn = n¡ 2

..

.

(n¡ 1)x1 + nx2 + x3 + ¢ ¢ ¢+ (n¡ 3)xn¡1 + (n¡ 2)xn = 2nx1 + x2 + 2x3 + ¢ ¢ ¢+ (n¡ 2)xn¡1 + (n¡ 1)xn = 1

풀이 모든 식을 다 더하면 x1+x2+¢ ¢ ¢+xn = 1. 이웃한 두 식끼리 빼면 x1+x2+¢ ¢ ¢+xn¡nxk = ¡1(k = 2; : : : ; n). 그럼 xk =

2n(k = 2; : : : ; n) 이고 x1 =

2n¡ 1. }

Page 84: 실전수학올림피아드 1400제 해답

84 대수

122. x, y는 x¡px · y¡ 14· x+

px 를 만족하는 양의 실수들이다. y¡py · x¡ 1

4· y+

py 임을 증명하

여라. (스웨덴 1991-2)

증명 (px¡ 1

2)2 · y · (px+ 1

2)2 이므로

px¡ 1

2· py · px+ 1

2. 즉

py¡ 1

2· px · py+ 1

2. ¤

123. x4 ¡ 2ax2 + x+ a2 ¡ a = 0 의 근이 모두 실수가 되도록 하는 실수 a의 값을 모두 구하여라.(인도지역예선 2000-7)

풀이 (x2 + x¡ a)(x2 ¡ x¡ (a¡ 1)) 로 인수분해 됨. ¢ ¢ ¢ 답 a ¸ 3=4 }

124. u와 v는 이차방정식 ax2 + bx + c = 0 의 두 근이다(단, ac 6= 0). px2 + qx + 1 = 0 이 u=v와 v=u를 두근으로 갖는 이차방정식이라면, q는 얼마인가? a, b, c에 한 식으로 나타내어라.

(플란더즈 예선 1998/1999 2차)

풀이 2¡ b2

ac}

125. a, b, c가 양의 실수들일 때 abc ¸ (a+ b¡ c)(b+ c¡ a)(c+ a¡ b) 임을 증명하여라. (스웨덴 1982-2)

증명 (x; y; z) = (b+ c¡ a; c+ a¡ b; a+ b¡ c) 로 치환하면 준식은 (x+ y)(y + z)(z + x) ¸ 8xyz 와

동치. AM-GM. ¤

126. R은 실수 전체의 집합, S는 0과 1을 제외한 실수 전체의 집합이다.

임의의 x 2 S 에 해 f(x) +1

2xf

µ1

1¡ x

¶= 1

을 만족하는 함수 f : S ! R 를 모두 구하여라. (호주 2003-2)

풀이 x 신에 x,1

1¡ x,x¡ 1x

을 각각 입하면,

f(x) +1

2xf

µ1

1¡ x

¶= 1 (1)

f

µ1

1¡ x

¶+1¡ x

2f

µx¡ 1x

¶= 1 (2)

f

µx¡ 1x

¶+

x

2(x¡ 1)f(x) = 1 (3)

(1)£ 2x¡ (2) 를 하면

2xf(x)¡ 1¡ x

2f

µx¡ 1x

¶= 2x¡ 1 (4)

(3)£ 1¡ x

2+ (4) 를 하면

7x

4f(x) =

3

2x¡ 1

2) f(x) =

6x¡ 27x

이것을 다시 문제의 식에 입해보면 성립하므로 이것이 해가 된다. }

127. 다음 연립방정식을 풀어라:

ab+ c+ d = 3; bc+ d+ a = 5; cd+ a+ b = 2; da+ b+ c = 6

단, a, b, c, d는 실수이다. (영국 2003/2004 1차-1)

풀이 ¢ ¢ ¢ 답 2, 0, 0, 3 }

Page 85: 실전수학올림피아드 1400제 해답

2.1 대수 중급문제 85

128. 임의의 c > 1 에 해, 두 수pc+ 1¡pc 와

pc¡pc¡ 1 중 어느 쪽이 큰지 결정하여라.

(캐나다 1969-2)

풀이 부등호의 자리를 비워두고 다음과 같이 정리해보자.

pc+ 1¡pc°p

c¡pc¡ 1() p

c+ 1 +pc¡ 1° 2

pc

() (c+ 1) + 2pc2 ¡ 1 + (c¡ 1)° 4c

() 2pc2 ¡ 1° 2c

() c2 ¡ 1° c2

따라서, ° 자리에 들어갈 부등호는 <이고,pc¡pc¡ 1 이 항상 크다. }

별해 무리식의 유리화를 이용해보자.

pc+ 1¡pc =

1pc+ 1 +

pc<

1pc+

pc¡ 1 =

pc¡pc¡ 1

이므로pc¡pc¡ 1 이 더 크다. }

주 y =px 의 그래프가 위로 볼록함을 이용해도 된다.

129. 조건 2 · x · 4, x · y, 3 · y · 5 를 만족시키는 실수 x, y에 하여,2 + xy

x+ y의 최 값은?

(한국 2005 1차-J4)

풀이 x = a 로 일정할 때

2 + ay

a+ y=

a(y + a)¡ (a2 ¡ 2)y + a

= a¡ a2 ¡ 2y + a

이므로 이것은 y가 최 일 때(즉, y = 5 일 때) 최 값을 갖는다. 따라서, y = 5 일 때 중에만 최 값이있고, 그럼

5x+ 2

x+ 5= 5¡ 23

x+ 5

도 역시 x가 최 일 때 최 값을 갖는다. 따라서, 최 값은 (x; y) = (5; 4) 일 때 ¢ ¢ ¢ 답 229. }

130. 자연수 n에 하여pn 의 정수부분을 a, 소수부분을 b라 할 때 a3 ¡ 9ab + b3 = 0 이 만족된다고 한다.

an의 값을 구하여라. (1991 서울시)

풀이 a3 + b3 = 9ab 에서 a ¸ 3 이면 좌변이 크고 a · 1 이면 우변이 커서 해없음. a = 2 일 때는

(b¡ 4)(b2 + 4b¡ 2) = 0 으로 인수분해되고 n = 6 이 답이 됨. }

131. a1 > a0, a1 > 0, ar+2 = 3ar+1 ¡ 2ar (r = 0; 1; : : : ; 98) 을 만족시키는 정수 a0; a1; : : : ; a100이 주어져있다. a100 > 299 임을 증명하여라. (소련 1962-3)

증명 점화식을 풀어 ar = (2r ¡ 1)a1 ¡ (2r ¡ 2)a0 ... ¤

132. a, b, c가a2 + b2 + c2 = 1

을 만족하는 실수일 때, 다음 부등식을 증명하여라. (헝가리 1910-1)

¡12· ab+ bc+ ca · 1

Page 86: 실전수학올림피아드 1400제 해답

86 대수

증명

0 · (a+ b+ c)2 = (a2 + b2 + c2) + 2(ab+ bc+ ca) = 1 + 2(ab+ bc+ ca)

로부터 왼쪽 부등식이 얻어지고,

0 · (a¡ b)2 + (b¡ c)2 + (c¡ a)2 = 2(a2 + b2 + c2)¡ 2(ab+ bc+ ca) = 2¡ 2(ab+ bc+ ca)

로부터 오른쪽 부등식이 얻어진다. ¤

133. f(x) + xf(1¡ x) = 1 + x 를 만족시키는 함수 f : R! R 을 구하여라. (통신강좌 1991-2-18)

풀이

f(x) + xf(1¡ x) = 1 + x (1)

에서 x 신 1¡ x를 입하면

f(1¡ x) + (1¡ x)f(x) = 2¡ x (2)

를 얻는다. (1)¡(2)£x를 하면 f(x)¡x(1¡x)f(x) = 1+x¡x(2¡x). 따라서 (x2¡x+1)f(x) = x2¡x+1.x2 ¡ x+ 1 > 0이므로, (x2 ¡ x+ 1)로 양변을 나누면, f(x) = 1, x 2 R. }

134. 정의역이 0 이상 11 이하인 실수들의 집합이고 함수값이 항상 0 이상인 함수 f가 다음 두 조건을 만족시킬 때, f(2)2 + f(10)2 의 값을 구하여라.

(1) f(0) = 0, f(6) = 1;

(2) x ¸ 0, 0 · y; x+ y · 11 이면 f(x+ y) =f(x) + f(y)

1¡ f(x)f(y). (한국 2006 1차-J19)

풀이 f(6)에 한 식을 구하여 6차식을 적당히 인수분해하면 뭔가 나옴. f(2)2+f(10)2 = (2¡p3 )2+(2 +

p3 )2 = 14 ¢ ¢ ¢ 답 }

135. a+ b+ c = 6, a2 + b2 + c2 = 18, abc = 4 일 때, 다음 식의 값을 구하여라.

(1) (a+ b)2 + (b+ c)2 + (c+ a)2

(2) a2b2 + b2c2 + c2a2

풀이 ¢ ¢ ¢ 답 (1) 54 (2) 33 }

136. 이차방정식 x2 ¡ (3m+ 1)x+ 8m = 0 의 두 근의 비가 2 : 3 일 때, 정수 m의 값을 구하여라.

풀이 두 근을 2a, 3a라고 하면 근과 계수의 관계에 의해서 2a + 3a = 3m + 1, 6a2 = 8m 이다. 이를

연립하면 8(5a ¡ 1) = 3(6a2) 이고 이를 정리하면 (9a ¡ 2)(a ¡ 2) = 0 이다. 따라서 a는 2 혹은 29이고

m이 정수가 되려면 a는 2가 되어야 한다. 그러면 m은 3이다. }

137. a, b, c를 a2+ b2+ c2 6= 0 인 실수라 하자. x, y, z가 ax+ by+ cz = 0, bx+ cy+az = 0, cx+ay+ bz = 0

을 동시에 만족하면 x = y = z 또는 x+ y + z = 0 이 성립함을 증명하여라. (1991 서울시)

증명 세 식을 모두 합하면 (a+ b+ c)(x+ y+ z) = 0. x+ y+ z 6= 0 이라면 a+ b+ c = 0. a = ¡b¡ c

를 세 식에 입하면 b(x¡ y) = c(z ¡ x), b(z ¡ x) = c(y ¡ z), b(y ¡ z) = c(x¡ y). x, y, z 중 어느 둘이 같은 것이 있으면 모두 같음. x 6= y 6= z 6= x 라 하면 이 세 식을 모두 곱하여 b3 = c3, 즉 b = c 를 얻음. 칭적이므로 a = b = c 임을 알 수 있고, 그럼 a = b = c = 0 이 되어 모순. ¤

138.1

2!+2

3!+3

4!+ ¢ ¢ ¢+ 9

10!의 값은 얼마인가?

풀이 telescoping 으로 1¡ 110!

}

Page 87: 실전수학올림피아드 1400제 해답

2.1 대수 중급문제 87

139. 함수 f(x)가 다음 두 조건을 만족할 때, f(1999)를 구하여라.

(i) f(3) = 5

(ii) f(x+ 2) =f(x)¡ 1f(x) + 1

(1999 교육청경시)

풀이 5; 23;¡ 1

5;¡ 3

2; 5; : : : 로 f(2n+ 1)은 주기수열. ¢ ¢ ¢ 답 ¡ 1

5}

140. 함수 f(x) = x3 ¡ x 를 생각하자. f(x+ a) = f(x) 를 만족하는 x가 존재한다면 jaj · 2 임을 증명하여라.(아일랜드 1988-7)

증명 (x+ a)3 ¡ (x+ a) = x3 ¡ x 를 정리하면 (x+ a)3 ¡ x3 = (x+ a)¡ x, a(3x2 + 3ax+ a2) = a.

a = 0 일 땐 틀림없이 결론이 성립하므로 a 6= 0 이라 하면 3x2 + 3ax+ a2 ¡ 1 = 0. 해를 가지므로, 판별식 D = 9a2 ¡ 12(a2 ¡ 1) ¸ 0, 즉 3a2 · 12. ¤

141. 다음 연립방정식이 실수해를 갖지 않음을 증명하여라. (레닌그라드 1989-y8-1)

x+ y + z = 0

1

x+1

y+1

z= 0

증명 1x= ¡ 1

y¡ 1

z= ¡y¡z

yz= x

yz 이므로 x2 = yz 로 x, y, z는 모두 같은 부호임을 알 수 있다. 그럼

합이 0이 될 수 없다. ¤

142. abc = 1 이고 a+ b+ c >1

a+1

b+1

c인 세 양의 실수 a ¸ b ¸ c 가 있다. a > 1 > b 임을 보여라.

(스웨덴 1995-4)

증명 a · 1 이면 b; c · 1 이고 3 ¸ a + b + c > 1a+ 1

b+ 1

c¸ 3 으로 모순이므로 a > 1. a +

b + 1ab= a + b + c > 1

a+ 1

b+ 1

c= 1

a+ 1

b+ ab 이므로 ab ¡ a ¡ b < 1

ab¡ 1

a¡ 1

b, 정리하면

(a¡ 1)(b¡ 1) < (1¡ 1a)(1¡ 1

b) =

(a¡1)(b¡1)ab

, 즉 (a¡ 1)(b¡ 1)(1¡ 1ab) < 0. b ¸ 1 이면 이것은 성립

하지 않음. ¤

143. 0이 아닌 세 실수 a, b, c가 abc = 1 과 a+ b+ c =1

a+1

b+1

c을 만족하면, a, b, c 중에 1인 것이 있음을

증명하여라. (레닌그라드 1988-y8-1)

증명 a+ b+ c = ab+ bc+ ca 이므로 (a¡1)(b¡1)(c¡1) = abc¡ (ab+ bc+ ca)+(a+ b+ c)¡1 = 0.¤

144. 실수에서 정의되는 실함수 f가 모든 실수 x에 해 다음의 (i), (ii)를 만족한다.

(i) f(10 + x) = f(10¡ x)

(ii) f(20 + x) = ¡f(20¡ x)

f는 주기함수이며 f(¡x) = ¡f(x) 임을 증명하여라. (이탈리아 1997-2)

증명 f(x+ 40) = f(x) 등을 뭐 적당히 하면 얻어낼 수 있음. 셈PS 에도 칭성 쪽에 비슷한 문제 있

었던 것 같은데. ¤

145. 임의의 양수 a, b에 하여 다음 두 조건이 동치임을 증명하여라:

(i)pa+ 1 >

pb.

(ii) 임의의 x > 1에 하여 ax+x

x¡ 1 > b. (폴란드 1994/1995 1차-5)

증명 (ii)를 이차식 f(x)로 전개한 후 축이 1보다 크면 D < 0, 축이 1 이하이면 f(1) ¸ 0 임을 이용

함(근데 항상 f(1) = 1). 두 경우를 각각 (i)과 비교. ¤

Page 88: 실전수학올림피아드 1400제 해답

88 대수

146. x[x[x[x]]] = 88 을 만족하는 실수 x를 모두 구하여라. 단, [x]는 x를 넘지 않는 가장 큰 정수를 나타낸다.(체코슬로바키아 1998-1)

풀이 f(x) = x[x[x[x]]] 는 (단조가 아닌) 증가함수. f(3) = f(¡3) = 81 이므로 그 근처에서 찾아

보자. x < ¡3 이면 [x] · ¡4 이고, [x[x]] ¸ 12, [x[x[x]]] · ¡37, x[x[x[x]]] > 111 이므로 해가 없다.x = 88

28= 22

7 만 해가 됨. [x[x[x]]] = 88x

< 883= 29 1

3 임에서 x = 8827

; 8828; 8829 만 후보가 되는데, 각각 검

산해보면 x = 8828= 22

7 만 실제 가능함. }

147. (1)px+

py +

pxy =

px+

py + xy + 2y

px 임을 증명하여라.

(2)p3 +p10 + 2

p3 =

p5 +

p22 +

q8¡p22 + 2

p15¡ 3p22 임을 보여라. (스페인 1990-1)

증명 (1)py + xy + 2y

px = (1 +

px)py. (2)

q8¡p22 + 2

p15¡ 3p22 = p3 +

p5¡p22. 그

다음은p3 소거하고 양변 제곱해서 비교하면 끝. ¤

148. a 6= c 이고 p(x) = x4 + ax3 + bx2 + cx + 1, q(x) = x4 + cx3 + bx2 + ax + 1 이라 하자. p(x) = 0 과q(x) = 0 이 두 개의 공통근을 가질 a, b, c의 조건을 구하고, 그 공통근을 구하여라. (스페인 2000-1)

풀이 p(x)¡ q(x) = (a¡ c)(x3 ¡ x) = 0. a 6= c 이므로 x3 = x, x = 0; 1;¡1. p(0) 6= 0 이므로 0은 해

가 아니고, 따라서 §1이 두 공통근일 수밖에. 입하면 a+ b+ c+ 2 = ¡a+ b¡ c+ 2 = 0, 즉 b = ¡2이고 a+ c = 0. }

149. 연립방정식 x2 ¡ y2 = 0, (x¡ k)2 + y2 = 1 이 딱 두 개의 실근을 가질 k의 조건을 구하여라. 딱 세 개의실근을 가질 조건도 구하여라. (브라질 1981-1)

풀이 두번째 식이 2x2 ¡ 2kx + k2 ¡ 1 = 0 이 되고, 이것은 판별식을 조사하면 k2 > 2 일 때는 해가

없고, k2 = 2 일 때는 중근, k2 < 2 일 때는 서로 다른 두 실근. k2 = 2 일 때, 즉 k = §p2 일 때를 각각조사하면 y가 2개씩 존재하므로 두 실근의 경우가 됨. k2 < 2 일 때는 각각의 x에 해 y = §x 로 y가2개씩 존재하므로 x = 0 일 때가 아니면 근은 항상 4개가 됨. x = 0 일 때를 조사하면 세 실근. }

주 그래프를 그려보아서 판단해도 쉽다. 첫 식이 y = §x 두 직선이고 두 번째 식은 x-축 위에 중심을

둔 원이므로 딱 두 개의 실근을 가질 때는 두 직선에 모두 접할 때.

150. x3 + ax+ b = 0 이 서로 다른 세 실근을 가지면 a < 0 임을 증명하여라. (브라질 1992-1)

증명 세 실근을 p; q; r이라 하면 p+q+r = 0이므로 a = pq+qr+rp = pq+r(p+q) = pq¡(p+q)2 =

¡(p2 + pq + q2) · 0. a = 0 일 때는 p = q = 0 일 때이므로 서로 다른 세 실근이 아님. ¤

2.2 수 고급문제

1. abcd = 1 인 네 양의 실수 a, b, c, d에 해, (1 + a)(1 + b)(1 + c)(1 + d) ¸ 16 임을 증명하여라.(인도지역예선 1993-6)

증명 산술-기하. ¤

2. n은 n ¸ 2 인 자연수이다. 다음을 증명하여라. (캐나다 1998-3)

1

n+ 1

µ1 +

1

3+ ¢ ¢ ¢+ 1

2n¡ 1¶

>1

n

µ1

2+1

4+ ¢ ¢ ¢+ 1

2n

Page 89: 실전수학올림피아드 1400제 해답

2.2 대수 고급문제 89

증명 양변에 n+ 1을 곱한 후, 다음과 같이 양변에 같은 값을 더하거나 빼면서 동치변형을 해보자.

(준식) () 1 +1

3+ ¢ ¢ ¢+ 1

2n¡ 1 >

µ1 +

1

n

¶µ1

2+1

4+ ¢ ¢ ¢+ 1

2n

¶() 1 +

1

2+1

3+ ¢ ¢ ¢+ 1

2n>

µ2 +

1

n

¶µ1

2+1

4+ ¢ ¢ ¢+ 1

2n

¶()

µ1 +

1

2+ ¢ ¢ ¢+ 1

2n

¶¡µ1 +

1

2+ ¢ ¢ ¢+ 1

n

¶>1

n

µ1

2+1

4+ ¢ ¢ ¢+ 1

2n

¶() 1

n+ 1+

1

n+ 2+ ¢ ¢ ¢+ 1

2n>1

2n+1

4n+ ¢ ¢ ¢+ 1

2n2

여기서 n+ k < nk + nk = 2nk 이므로 1n+k

> 12nk 가 되어 이 부등식은 성립한다. ¤

3. a, b, c를 음이 아닌 실수라 하자. 이 때 a+ b+ c ¸ abc 이면 a2 + b2 + c2 ¸ abc 임을 증명하여라.(아일랜드 1997-4)

증명 a2 + b2 + c2 < abc 라고 하면 a2 < abc 이므로 a < bc, 비슷하게 얻은 세 식을 모두 합하면

(abc ·)a+ b+ c < bc+ ca+ ab(· a2 + b2 + c2) 으로 모순. ¤

4. 만일 n개의 자연수 x1; x2; : : : ; xn이 있어 다음 n개의 방정식

2x1 ¡ x2 = 1

¡xk¡1 + 2xk ¡ xk+1 = 1 (k = 2; 3; : : : ; n¡ 1)¡xn¡1 + 2xn = 1

을 만족한다면, n은 짝수임을 증명하여라. (스웨덴 1983-3)

증명 점화식을 열심히 풀면

xn = nx1 ¡ n(n¡ 1)2

= n¡ x1

을 얻을 수 있다. 여기서 x1 =n2 이므로 n은 짝수. ¤

5. 다음 식을 간단히 하여라. ·[m®]n

®

¸단, m과 n은 양의 정수이며, ®는 n보다 큰 임의의 무리수이다.

풀이 ®는 무리수이므로 m® = k + e(0 < e < 1)(k는 정수)로 표현이 가능하다.

따라서 [m®]n = nk이다.

m =m®

®=

k + e

®

이고[m®]n

®=

nk

®

이다. 이 때 n < ®이므로

[m®]n

®=

nk

®<

nk + ne

®<

nk

®+ 1(e < 1; n < ®이므로 ne < a)

따라서[m®]n

®< mn <

[m®]n

®+ 1

이므로

mn¡ 1 < [m®]n

®< mn

이고 ·[m®]n

®

¸= mn¡ 1

이다. }

Page 90: 실전수학올림피아드 1400제 해답

90 대수

6. 함수 f : (0;1)! R 가 성질 f(x) = f

µ1

x

¶을 만족한다. 모든 x > 0 에 해서

u

Ãx+ 1

x

2

!= f(x)

를 만족하는 함수 u : [1;1)! R 가 존재함을 증명하여라. (통신강좌 1989-C14)

풀이 t =1

2

µx+

1

x

¶, x > 0라 하면

) x2 ¡ 2tx+ 1 = 0) x = t§pt2 ¡ 1) u(t) = f(t§pt2 ¡ 1) for x ¸ 0.cf) f(t+

pt2 ¡ 1) = f

µ1

t+pt2¡1

¶= f(t¡pt2 ¡ 1)

}

7. a, b, c; p, q, r이 실수이고, 임의의 실수 x에 해 부등식

ax2 + 2bx+ c ¸ 0 와 px2 + 2qx+ r ¸ 0

이 성립할 때, 임의의 실수 x에 해 다음 부등식이 성립함을 증명하여라.

apx2 + bqx+ cr ¸ 0 (헝가리 1918-3)

증명 ax2 + 2bx+ c ¸ 0이므로 판별식 D = b2 ¡ ac · 0이다. 마찬가지로 q2 ¡ pr · 0이다. 즉,

0 · b2 · ac; 0 · q2 · pr

이다. 그러므로 (bq)2 · acpr이다.apx2+bqx+cr의 판별식 D0 = (bq)2¡4apcr을 생각하자. apcr ¸ 0이므로 (bq)2¡4apcr · (bq)2¡apcr이고 (bq)2 · acpr에서 D0 · 0이다. 판별식이 0보다 작거나 같으므로 모든 x에 해 apx2+bqx+cr ¸ 0이다. ¤

8. 방정식 4x2 ¡ 40[x] + 51 = 0 의 모든 실근을 구하여라. 단, [x]는 x보다 작거나 같은 가장 큰 정수를 나타낸다. (캐나다 1999-1)

풀이 x¡ 1 < [x] · x 이므로 4x2 ¡ 40x+ 51 · 4x2 ¡ 40[x] + 51 < 4x2 ¡ 40x+ 91.

4x2 ¡ 40x+ 51 · 0 < 4x2 ¡ 40x+ 91(2x¡ 3)(2x¡ 17) · 0 < (2x¡ 7)(2x¡ 13)3

2· x <

7

2또는

13

2< x · 17

2

따라서, 가능한 [x]는 1, 2, 3, 6, 7, 8뿐이다.

² [x] = 1 일 때: 4x2 + 11 = 0 으로 곤란.

² [x] = 2 일 때: 4x2 ¡ 29 = 0, 여기서 [x] = 2 를 만족하는 것만 구하면 x =p292.

² [x] = 3 일 때: 4x2 ¡ 69 = 0, 여기서 [x] = 3 을 만족하는 것은 없음.

² [x] = 6 일 때: 4x2 ¡ 189 = 0, 여기서 [x] = 6 을 만족하는 것만 구하면 x =p1892.

² [x] = 7 일 때: 4x2 ¡ 229 = 0, 만족하는 해는 x =p2292.

² [x] = 8 일 때: 4x2 ¡ 269 = 0, 만족하는 해는 x =p2692.

따라서, 구하는 모든 실근은 x =p292,p1892,p2292,p2692

¢ ¢ ¢ 답 }

Page 91: 실전수학올림피아드 1400제 해답

2.2 대수 고급문제 91

9. 모든 실수 a, b, c, d에 하여 다음의 부등식을 증명하여라. (한국 2006-J6)

a+ b+ c+ d

(1 + a2)(1 + b2)(1 + c2)(1 + d2)< 1

증명 (서울 당산서중 2학년 박민재)

A.M-G.M 부등식에 의해서 어떤 양수 x에 해서

x = 2

rx2 £ 1

4· x2 +

1

4(¤)

가 성립한다. 이로부터

준식 =a+ b+ c+ d

(1 + a2)(1 + b2)(1 + c2)(1 + d2)· jaj+ jbj+ jcj+ jdj(1 + a2)(1 + b2)(1 + c2)(1 + d2)

· a2 + b2 + c2 + d2 + 1

(1 + a2)(1 + b2)(1 + c2)(1 + d2)

· a2 + b2 + c2 + d2 + 1

a2 + b2 + c2 + d2 + 1= 1

가 된다. 마지막 부등식은 분모를 전개해보면 쉽게 알 수 있고, 마지막 부등식의 등호는 a, b, c, d 중에3개 이상이 0일 때 성립한다. 그러나, 그 경우 (¤)에 의한 두 번째 부등식이 성립하지 않으므로 결국 원래 부등식의 등호는 성립하지 않는다. ¤

10. a, b, c는 a+ b+ c = 1 을 만족하는 양의 실수들이다. 다음을 증명하여라.

(1 + a)(1 + b)(1 + c) ¸ 8(1¡ a)(1¡ b)(1¡ c) (러시아 1991 4차-y11-3)

관찰 제한식 a + b + c = 1 을 무효화시키기 위해 동차식으로 변형해주고 싶은 것이 인지상정. 그래

서, 1 + a = (a+ b+ c) + a, 1¡ a = (a+ b+ c)¡ a 등으로 체하면 준식은

(a+ b+ c+ a)(b+ c+ a+ b)(c+ a+ b+ c) ¸ 8(b+ c)(c+ a)(a+ b)

가 된다. 이제 a, b, c를 똑같이 k배를 해줘도 이 부등식은 똑같은 식이 되므로 더 이상 a+ b+ c = 1 의제한식은 의미를 갖지 못하고 없는 것이나 마찬가지이다. 이 식은 또 잘 관찰하면 b+ c = X, c+ a = Y ,a+ b = Z 로 치환하고 싶은 유혹을 느낄 것이다. 유혹에는 넘어가주는 것이 또한 인지상정. 그럼

(Y + Z)(Z +X)(X + Y ) ¸ 8XY Z

의 식이 된다. 이것은 Y + Z ¸ 2pY Z 등의 A.M ¸ G.M 부등식으로 바로 풀 수 있음이 이제 눈에 띌것이다. 그렇게 관찰된 최종적인 구조를 보니 굳이 치환을 하고 말고 할 것이 없었다는 생각이 든다. 그래서, 증명만 다시 정리한다면 다음과 같이 하면 되겠다.

증명 a+ b+ c = 1 이므로

1 + a = (1¡ b) + (1¡ c)

1 + b = (1¡ c) + (1¡ a)

1 + c = (1¡ a) + (1¡ b)

모든 항이 다 양수이므로, 여기에 A.M ¸ G.M 부등식을 적용하여 변변 곱하면

(1 + a)(1 + b)(1 + c) = [(1¡ b) + (1¡ c)][(1¡ c) + (1¡ a)][(1¡ a) + (1¡ b)]

¸ 2p(1¡ b)(1¡ c) ¢ 2

p(1¡ c)(1¡ a) ¢ 2

p(1¡ a)(1¡ b)

= 8(1¡ a)(1¡ b)(1¡ c)

따라서, 준식이 증명되었다. 등호는 1¡ a = 1¡ b = 1¡ c 일 때, 즉 a = b = c = 13 일 때 성립. ¤

Page 92: 실전수학올림피아드 1400제 해답

92 대수

11. 다음 식을 증명하여라.

[x] + [y] + [x+ y] · [2x] + [2y]

증명 x = [x] + a, y = [y] + b일 때 0 · a · b <이라 두자. 그럼 2b ¸ a+ b 이다.

[2x] + [2y] = [2[x] + 2a] + [2[y] + 2b] = 2[x] + 2[a] + 2[y] + 2[b]

¸ 2[x] + 2[y] + [2b] ¸ 2[x] + 2[y] + [a+ b]

= [x] + [y] + [[x] + [y] + a+ b] = [x] + [y] + [x+ y]

이다. ¤

12. az3 + bz2 + cz + d = 0 의 세 근은 모두 실수부가 음수라고 한다. ab > 0 이고 bc > ad > 0 임을 보여라.(아일랜드 1992-8)

주 세 근을 모두 음의 실근이거나, 아니면 세 근 중 하나가 음근(¡p이라 하자)이고 나머지 둘은 실

수부가 음수인 켤레허근(¡q § ir라 하자). 후자의 경우 세 근의 합은 ¡p ¡ 2q 로 음수, 세 근의 곱은¡p(q2+r2)으로 음수, 세 근을 둘씩 곱해 합한 값은 ¡p(¡2q)+(q2+r2)으로 양수. 전자의 경우도 마찬가지로, 세 근의 합,세 근의 곱은 항상 음수, 세 근을 둘씩 곱해 합한 값은 항상 양수이다. 즉, ba ;

ca; da> 0

이고, a, b, c, d는 모두 같은 부호. 이제 bc > ad 만 보이면 됨. ba¢ ca> d

a 와 동치. 세 근을 ®, ¯, ° 라 하

면 (®+ ¯ + °)(®¯ + ¯° + °®) < ®¯° 와 동치. 넘겨서 (®+ ¯)(¯ + °)(° + ®) < 0 의 인수분해식과 동치가 됨. 세 근 모두 음근일 때는 당연히 성립하고, 켤례허근이 있을 때는 ((p + q)2 + r2)(¡2q) < 0 로역시 성립.

13. 실수의 수열 u1; u2; : : : 는

u1 = 1; un =1

u1 + ¢ ¢ ¢+ un¡1(n > 1)

의 점화식을 만족한다.

u1 + u2 + ¢ ¢ ¢+ uN > 1989

가 되는 양의 정수 N이 존재함을 증명하여라. (호주 1989-3)

증명개요 귀류법 u1 + ¢ ¢ ¢+ un · 1989 이면 un+1 ¸ 11989

. ) N · 19892. ¤

별증 ( 전 전민중 3학년 정명진)

1 ¸ ui > 0 이므로 u1+ ¢ ¢ ¢+un¡1 · (n¡ 1), 즉 un ¸ 1n¡1 . u1+ ¢ ¢ ¢+uN ¸ 1+ ¢ ¢ ¢+ 1

N¡1 !1. ¤

14. a1, a2, : : : ; an 을 음이 아닌 실수들이라 하자. 모든 가능한 짝의 곱 aiaj (i < j) 들을 모두 합한 값을

M으로 정의하자. 즉,

M = a1(a2 + a3 + ¢ ¢ ¢+ an) + a2(a3 + a4 + ¢ ¢ ¢+ an) + ¢ ¢ ¢+ an¡1an

이다. a1, a2, : : : ; an 중 적어도 하나는 그 제곱이 2Mn(n¡1) 을 넘지 않음을 보여라. (캐나다 1972-2)

증명 ai들 중 가장 작은 것을 a라 하자. 그럼

M =Xi<j

aiaj ¸Xi<j

a2 =³n2

´a2

따라서, a2 · 2M

n(n¡ 1) 이다. ¤

15. 관계식 x2 + y2 = 2 를 만족시키는 실수 x, y에 하여,15x2

2+ 4xy 의 최 값을 구하여라.

(한국 2006 1차-J11)

Page 93: 실전수학올림피아드 1400제 해답

2.2 대수 고급문제 93

풀이 (KAIST 05학번 이은정)

y = §p2¡ x2 을 k = 152x2 + 4xy 에 입하면

2k ¡ 15x2 = §8xp2¡ x2

4k2 ¡ 60kx2 + 225x4 = 64x2(2¡ x2)

289x4 ¡ 2(30k + 64)x2 + 4k2 = 0

이 식을 x2에 한 이차방정식으로 볼 때, 실수해를 가져야 하므로 판별식이 0 이상이다. 즉,

D0 = (30k + 64)2 ¡ 289 ¢ 4k2 = (30k + 64)2 ¡ (34k)2= (30k + 64 + 34k)(30k + 64¡ 34k) = 64(k + 1) ¢ (¡4)(k ¡ 16) ¸ 0

따라서, ¡1 · k · 16 이다. x2 = 544289(< 2) 일 때 실제로 k = 16 이 성립하므로, 최 값은 ¢ ¢ ¢ 답 16

}

별해1 (김해 장도한)

G.M · A.M 부등식에 의해

15

2x2 + 4xy · 15

2x2 +

1

2(x2 + (4y)2) = 8(x2 + y2) = 16

이다. 등호가 성립하는 경우는 x2 = 4y2 = 25 이고 xy > 0 일 때로 존재한다. 따라서, 최 값은 16 ¢ ¢ ¢

답 }

별해2 (광주 동성고 3학년 방재혁)

x2 + y2 = 2 이므로, x =p2 cos µ, y =

p2 sin µ 로 치환하자.

준식 =15£ 2 cos2 µ

2+ 4

p2 cos µ

p2 sin µ

= 15 cos2 µ + 8 sin µ cos µ

=15(1 + cos 2µ)

2+ 4 sin 2µ

=15

2+15

2cos 2µ +

8

2sin 2µ

=15

2+

s82 + 152

4sin(2µ + ®) (tan® =

8

15)

=15

2+17

2sin(2µ + ®) · 16

즉 2µ + ® = 90±일 때, 최 값 16을 가진다. }

16. 모든 자연수 n에 해 다음 부등식을 증명하여라. (플란더즈 1986-2)

n! ·µn+ 1

2

¶n힌트 ((n+ 1)=2 + ®)((n+ 1)=2¡ ®) · ((n+ 1)=2)2 만 확인하면 충분 }

17. 실수 a, b, x, y가 ax+ by = 3, ax2 + by2 = 7, ax3 + by3 = 16, ax4 + by4 = 42 를 만족한다. ax5 + by5

의 값을 구하여라. (AIME 1990-15)

풀이 %EEE

This needs a trick:

(ax^{n+1} + by^{n+1})(x+y) - (ax^n + by^n)xy = ax^{n+2} + by^{n+2}.

Hence 7(x+y) - 3xy = 16, 16(x+y) - 7xy = 42. Solving x+y = -14, xy = -38.

Applying again: ax^5 + by^5 = 42(x+y) - 16xy = 20.

¢ ¢ ¢ 답 20 }

Page 94: 실전수학올림피아드 1400제 해답

94 대수

18. 어떤 자연수를 3x2+ y2꼴로 나타낼 수 있으면, 이 수는 u2+uv+ v2꼴로도 나타낼 수 있음을 보여라. 또한, 그 역도 성립함을 보여라. 단, x, y, u, v는 음 아닌 정수이다. (IMTS R4-3)

풀이 u = x + y, v = x ¡ y 라고 하면, 3x2 + y2 = u2 + uv + v2 이 된다. 즉, x; y가 정수이면

3x2 + y2 = u2 + uv + v2 인 u; v가 존재한다.

역방향에서 u, v가 짝수일 때는 위와 같이 하면 되고, u, v의 홀짝이 다를 때는 WLOG u = 짝수 = 2w

라 하면 u2 + uv + v2 = 4w2 + 2wv + v2 = 3w2 + (w + v)2. }

19. b < c < d 일 때 다음 부등식이 임의의 a에 해 성립함을 증명하여라. (아벨콘테스트 1993 결선 2)

(a+ b+ c+ d)2 > 8(ac+ bd)

풀이 (a+ b+ c+ d)2 ¡ 8(ac+ bd)는 a에 관한 이차식이다. 이것을 a에 관해 묶으면

a2 + 2(b¡ 3c+ d)a+ (b+ c+ d)2 ¡ 8bd

이 된다. 판별식을 살펴보면,

D=4 = (b¡ 3c+ d)2 ¡ (b+ c+ d)2 + 8bd

= (b¡ 3c+ d+ b+ c+ d)(b¡ 3c+ d¡ b¡ c¡ d) + 8bd

= (2b¡ 2c+ 2d)(¡4c) + 8bd= 8(c2 ¡ bc+ bd¡ cd) = 8(c¡ b)(c¡ d)

< 0

이므로 식은 항상 양수가 된다. 그러므로 모든 a에 해 (a+ b+ c+ d)2 > 8(ac+ bd)이다. }

20. 모든 실수 x에 해 f(x) = f(398¡x) = f(2158¡x) = f(3214¡x)가 성립한다. f(0); f(1); f(2); : : : ; f(999)에나타나는 서로 다른 실수는 최 몇 개인가? (AIME 2000 1차-12)

풀이 %EEE

From the first two, f(x) = f(1760+x). From the second two,

f(x) = f(1056+x). gcd(1056, 1760) = 352, so f(x) = f(x+352).

Then f(x) = f(398-x) gives f(x) = f(46-x). Conversely, it is easy to

check that f(x) = f(46-x) and f(x) = f(352+x) imply the relations given.

Evidently, f(x) = f(352+x) means the values repeat after any block of 352.

Take that block centered on x = 23. We can choose separately the values

at x = 23-176, 23-175, ... , 23-1, 23. Then the reflection determines

the values at 23+1, 23+2, ... , 23+176. The period then determines all

other values. So we get at most 177 different values. If we take them

all different, they certainly appear in x = 0, 1, ... , 999, which

contains two complete periods.

¢ ¢ ¢ 답 177 }

21. 다음을 만족하는 실수 x가 존재하는 가장 큰 자연수 n을 구하여라:

21 < x1 + x2 < 22

22 < x2 + x3 < 23

.

..

2n < xn + xn+1 < 2n+1 (아일랜드 1990-4)

풀이 x2 + x¡ 4 = 0 은 만족하는 1과 2 사이의 근을 ®라 하면 1 < x < ® < 1:6.

x4 + x5 < ®4 + ®5 = 4®3 = 4®(4¡ ®) = 16®¡ 4(4¡ ®) = 20®¡ 16 < 24

이므로 n ¸ 4 이면 성립하지 않는다. 한편, x = 1:5 를 입하면 n = 3 일 때 잘 성립한다. }

Page 95: 실전수학올림피아드 1400제 해답

2.2 대수 고급문제 95

22. 모든 자연수 k는 k = a11! + a22! + ¢ ¢ ¢ + amm!꼴로 승수전개할 수 있고 그 방법은 유일하다. 단, 0 <

am < m+1, 0 · ai < i+1. 16!¡32!+48!¡64!+ ¢ ¢ ¢+1968!¡1984!+2000! = a11!+a22!+ ¢ ¢ ¢+ann!

이 승수전개한 것일 때, a1 ¡ a2 + a3 ¡ a4 + ¢ ¢ ¢+ (¡1)j+1aj 의 값을 구하여라. (AIME 2000 2차-14)

풀이 %EEE

We have (n+1)! = n·n! + n! = n·n! + (n-1)(n-1)! + (n-1)! etc.

Hence 2000! - 1984! = 1999·1999! + 1998·1998! + ... + 1984·1984!,

and 1999 - 1998 + 1997 - 1996 + ... + 1985 - 1984 = 8. Thus the 62 pairs

(2000! - 1984!) + (1968! - 1952!) + ... + (48! - 32!) give 8·62 = 496.

Finally + 1·16! gives -1. Total 495.

¢ ¢ ¢ 답 495 }

23. 두 실수 x, y가2x2 ¡ 15x+ 26 · y · x2 ¡ 8x+ 20

을 만족시킬 때, 4(x2 + y)의 최 값과 최소값의 합을 구하여라. (한국 2006 1차-J7)

풀이 (한국과학영재학교 1학년 김민기)

우선 2x2 ¡ 15x+ 26 · x2 ¡ 8x+ 20 이어야 한다. x2 ¡ 7x+ 6 · 0, (x¡ 6)(x¡ 1) · 0, 즉

1 · x · 6

이다. 이제 준식을 변형하면

3x2 ¡ 15x+ 26 · x2 + y · 2x2 ¡ 8x+ 204(3x2 + 5x+ 26) · 4(x2 + y) · 8(x2 ¡ 4x+ 10)

이고, 이 식은 원래의 식과 동치이다. 여기서 좌변과 우변은 각각

L = 12x2 + 20x+ 104

= 12(x¡ 52)2 + 29 ¸ 29 (최소)

R = 8(x¡ 2)2 + 48· 176 (x = 6 일 때 최 )

이 된다. 실제로 등호가 성립하는 경우를 구하면 (x; y) = ( 52; 1), (6; 8) 일 때 4(x2 + y) = 29, 176이 되

므로 이것이 틀림없는 최소값과 최 값이다. 따라서, 구하는 합은 29 + 176 = 205 ¢ ¢ ¢ 답 }

주 2x2 ¡ 15x+ 26 · x2 ¡ 8x+ 20 을 만족하는 영역의 그래프를 그려 풀 수도 있다.

24. bxc는 내림 기호이고, x보다 작거나 같은 가장 큰 정수를 뜻한다. 비슷하게, dxe은 올림 기호이고, x보다크거나 같은 가장 작은 정수를 뜻한다. x는 1 · x · 100 인 실수일 때,

bxcdxe = x2

이 성립하는 x는 모두 몇 개인가? (IT꿈나무 올림피아드 2006 1차)

풀이 우선, x가 정수일 때는 bxc = dxe = x 이므로 항상 성립한다. x가 정수가 아닐 때를 보자. 좌변

이 정수이므로 우변 x2도 정수이다. x =pm 이라 하고, n <

pm < n+ 1 이라 하자(m, n은 정수). 그

럼 bxcdxe = n(n+1) 이므로 m = n(n+1) 이다. 즉, x =p1 ¢ 2, p2 ¢ 3, : : : ; p99 ¢ 100 등 99개의 값에

서 성립한다. x가 정수일 때는 x = 1; 2; : : : ; 100 등 100개의 값이 있으므로, 모두 100+99 = 199(개). }

25. 다항식 f와 그 계수조건이 다음과 같이 주어져있다.

f(x) = a0 + a1x+ a2x2 + ¢ ¢ ¢+ anx

n; 0 · ai · a0 (i = 1; 2; : : : ; n)

b0; b1; ¢ ¢ ¢ ; b2n 을 다항식 (f(x))2의 계수, 즉

(f(x))2 = (a0 + a1x+ a2x2 + ¢ ¢ ¢+ anx

n)2

= b0 + b1x+ b2x2 + ¢ ¢ ¢+ bn+1x

n+1 + ¢ ¢ ¢+ b2nx2n

라고 하자. 이 때, 다음을 증명하여라. (캐나다 1974-3)

bn+1 · 1

2(f(1))2

Page 96: 실전수학올림피아드 1400제 해답

96 대수

증명 bn+1은 xn+1의 계수이므로

bn+1 = a1an + a2an¡1 + ¢ ¢ ¢+ ana1

주어진 조건에서 0 · ai · a0 이므로

bn+1 · a0(a1 + a2 + ¢ ¢ ¢+ an)

이것과 비교하면

f(1)2 = (a0 + a1 + ¢ ¢ ¢+ an)2

= a20 + 2a0(a1 + ¢ ¢ ¢+ an) + (a1 + ¢ ¢ ¢+ an)2

¸ 2a0(a1 + ¢ ¢ ¢+ an)

따라서, f(1)2 ¸ 2bn+1 임을 알 수 있다. ¤

26. u는 0 < u < 1 이고,

u1 = 1 + u; u2 =1

u1+ u; : : : ; un+1 =

1

un+ u; n ¸ 1

로 정의된다. 모든 n = 1; 2; 3; : : : 에 해 un > 1 임을 보여라. (캐나다 1977-6)

증명 우선 u1 > 1, u2 > 1 임을 확인할 수 있다(산술-기하평균 부등식에서 u2 =11+u

+ (1+u)¡ 1 ¸2¡ 1 = 1 이고 등호는 성립하지 않음). n¡ 1과 n일 때 성립한다고 가정하면

un+1 =1

un+ u =

1

un+ un ¡ 1

un¡1¸ 2¡ 1

un¡1> 1

로 n+ 1일 때도 성립하므로, 수학적 귀납법으로 증명되었다. ¤

별증 수학적 귀납법으로 다음을 증명하자.

1 < un · 1 + u

n = 1 일 때는 자명하고, n = k 일 때 1 < uk · 1 + u 이 성립한다고 가정하면,

1 <1 + u+ u2

1 + u=

1

1 + u+ u · 1

uk+ u(= uk+1) < 1 + u

로 n = k + 1 일 때도 성립한다. ¤

주 uk > uk+1 이면 1uk

< 1uk+1

이므로 uk+1 < uk+2 가 된다. 또 그럼, 다시 비슷하게 uk+2 > uk+3

이 된다. 이렇게 n이 증가함에 따라 un은 증가와 감소를 반복하는 수열이다. 그리고, x = 1x+ u 가 되

는 한 근 ®(> 1)로 수렴할 것을 예상할 수 있다. 실제로,

jun+1 ¡ ®j < ju1 ¡ ®j®n

! 0

임을 어렵지 않게 증명할 수 있다.

27. 이차방정식 (bc¡ 1)x2 + (a¡ b+ c¡ abc)x+ ab¡ 1 = 0 이 정수근을 갖도록 하는 한 자리의 양의 정수a, b, c에 하여 100a+ 10b+ c 의 최 값을 구하여라. (한국 2005 1차-S17)

Page 97: 실전수학올림피아드 1400제 해답

2.2 대수 고급문제 97

풀이 (제주과학고 2학년 강진호, 수정됨)

정수계수 이차방정식이므로 정수근을 갖는다면 판별식이 완전제곱수라야 한다.

D = (a¡ b+ c¡ abc)2 ¡ 4(bc¡ 1)(ab¡ 1)= a2b2c2 ¡ 2abc(a+ b+ c) + (a2 + b2 + c2) + 2(ab+ bc+ ca)¡ 4= (abc)2 ¡ 2(abc)(a+ b+ c) + (a+ b+ c)2 ¡ 4= (a+ b+ c¡ abc)2 ¡ 4 = m2

즉, (a+ b+ c¡ abc)2 ¡m2 = 4 이다. 완전제곱수들 0, 1, 4, 9, 16, : : : 의 분포를 관찰하면 두 완전제곱수의 차가 4가 되는 경우는 4¡ 0 일 때뿐이다. 따라서, m = 0 이고

a+ b+ c¡ abc = §2 (¤)

이다. 그럼 a(bc¡ 1) = b+ c§ 2, 즉

bc¡ 1 j b+ c§ 2

이다. b+ c§ 2 = 0 일 때는 bc¡ 1 = 0 이기도 하므로, bc¡ 1 · b+ c§ 2 이 항상 성립한다. 즉,

(b¡ 1)(c¡ 1) · 4

이다. (¤)은 칭식이므로, 마찬가지로

(a¡ 1)(b¡ 1) · 4; (a¡ 1)(c¡ 1) · 4

이기도 하다. a > 5 이면 b = c = 1 이어야 하므로 문제의 식에서 최고차항이 0이 되어 이차방정식이 아니므로 곤란하다. a = 5 일 때는 b ¡ 1; c ¡ 1 · 1 이고, 이 경우 (a; b; c) = (5; 2; 1) 또는 (5; 1; 2)일 때(¤)이 만족된다. (a; b; c) = (5; 2; 1) 일 때 문제의 방정식은 x2 ¡ 6x+ 9 = 0 으로 x = 3 의 정수근을 갖는다. 따라서, 100a+ 10b+ c 의 최 값은 521이다. }

28. 다음을 증명하여라. (인도지역예선 1992-6)

1 <1

1001+

1

1002+

1

1003+ ¢ ¢ ¢+ 1

3001< 1

1

3

풀이1

2001<1

2

µ1

2000+

1

2002

¶< ¢ ¢ ¢ < 1

2

µ1

1001+

1

3001

¶<4=3

2001.

주 y = 1x 의 막 그래프에서 넓이를 적당히 비교하는 것을 생각한 것과 동치인 식임. }

29. a, b, c는 실수이고 abc > 0 이다. 3

µc

ab+

a

bc+

b

ca

¶+ 4

µ1

a+1

b+1

c

¶> 0 임을 증명하여라.

(1998 교육청경시)

풀이 abc > 0 이므로 왼쪽 식에 abc를 곱한 후 0과 비교하면 된다. abc를 곱하면

3(a2 + b2 + c2) + 4(ab+ bc+ ca) = 2(a+ b+ c)2 + (a2 + b2 + c2) > 0

이므로 준식이 성립한다. }

30. 집합 S에서 연산 ¤를 다음의 법칙이 만족하도록 정의한다.

x ¤ (x ¤ y) = y; (x; y 2 S)

(y ¤ x) ¤ x = y; (x; y 2 S)

이 때, 연산 ¤는 교환법칙은 성립함을 보여라. 그리고, 결합법칙이 성립하지 않는 경우가 있음을 보여라.(Putnam 1972)

Page 98: 실전수학올림피아드 1400제 해답

98 대수

증명 두 번째 법칙의 x 신 y ¤ x 를 입하면

y = (y ¤ (y ¤ x)) ¤ (y ¤ x) = x ¤ (y ¤ x)

이 된다. 이로부터x ¤ y = x ¤ (x ¤ (y ¤ x)) = y ¤ x

로 교환법칙이 성립함을 확인할 수 있다. 한편,

¤ a b c

a b a cb a c bc c b a

로 ¤를 정의하면 (a ¤ b) ¤ c = a ¤ c = c, a ¤ (b ¤ c) = a ¤ b = a 로 서로 다르므로 결합법칙이 성립하지않는 예가 된다. 각 원소 a, b, c는 왼쪽에서 곱할 때 어떤 두 원소를 서로 바꾸는 작용을 하므로 왼쪽에서 두 번 곱하면 항등원으로 작용한다. 오른쪽에서 곱할 때도 마찬가지이므로 문제의 조건을 만족한다는 것을 쉽게 알 수 있다. ¤

31. 다음을 만족시키는 함수 f(x)에 하여, f(¡9)의 값을 구하여라.

f(3) = 1; (x2 ¡ x+ 1)f(x2) = f(x) (한국 2004 1차-J16)

풀이 x = 3 을 입하면 f(9) = 17f(3) = 1

7.

x = 9 를 입하면 f(81) = 173f(9) = 1

73¢7 .x = ¡9 를 입하면 f(¡9) = 91f(81) = 13

73. }

32. a, b, c, d, e는 1 · a < b < c < d < e 인 정수이다.

1

[a; b]+

1

[b; c]+

1

[c; d]+

1

[d; e]· 15

16

임을 보여라. 단, [m;n]은 m과 n의 최소공배수를 나타낸다. (캐나다 1979-3)

증명 준식의 좌변을 S라 하자. x < y 일 때 [x; y] ¸ 2x 이고 등호는 y = 2x 일 때만 성립함을 기억해

두자.

(i) c = 3 일 때; a = 1, b = 2, d ¸ 4 이고, [a; b] = 2, [b; c] = 6, [c; d] ¸ 6, [d; e] ¸ 8.d 6= 6 이면 [c; d] ¸ 9 이므로 S · 1

2+ 16+ 19+ 18< 23+ 14= 1112

< 1516 으로 성립. d = 6 이면

[c; d] = 6, [d; e] ¸ 12 이므로 S · 12+ 16+ 16+ 112= 23+ 14= 1112

< 1516 으로 역시 성립.

(ii) c = 4 일 때; [a; b] ¸ 2, [b; c] ¸ 4, [c; d] ¸ 8, [d; e] ¸ 10.d 6= 8 이면 [c; d] ¸ 12 이므로 S · 1

2+ 14+ 112+ 110= 35+ 13= 1415

< 1516 으로 성립. d = 8 이면

[d; e] ¸ 16 이므로 S · 12+ 14+ 18+ 116= 1516 으로 역시 성립.

(iii) c ¸ 5 일 때; [a; b] ¸ 2, [b; c] ¸ 6, [c; d] ¸ 10, [d; e] ¸ 12 이므로 이 때도역시 성립.

따라서, 준식은 항상 성립한다. ¤

33. 모든 x 2 R 에 해 다음을 만족하는 함수 f : R! R 을 모두 구하여라.

f(x)3 = ¡ x

12¢ (x2 + 7x ¢ f(x) + 16 ¢ f(x)2) (플란더즈 1987-3)

힌트 모두 이항하여 인수분해할 수 있음. 연속성을 감안하면 f(x) = ¡x=2 또는 f(x) = ¡x=3 또는

원점에서 이 두 가지가 서로 교체되는 함수들. }

34. 0이 아닌 모든 실수에서 정의되고 다음 두 조건을 만족하는 함수 f는 유일하게 존재함을 증명하여라.

(i) 0이 아닌 임의의 실수 x에 해, f(x) = xf( 1x).

(ii) x 6= ¡y 인 임의의 0이 아닌 실수쌍 (x; y)에 해, f(x) + f(y) = 1 + f(x+ y). (호주 1991-4)

Page 99: 실전수학올림피아드 1400제 해답

2.2 대수 고급문제 99

증명 g(x) = f(x) ¡ 1 로 치환하면 두 조건은 g(x) + 1 = x(g( 1x) + 1) 과 g(x) + g(y) = g(x+ y) 로

바뀐다. 두 번째 식의 y에 x를 입하면 g(2x) = 2g(x) 이므로, 첫 번째 식과 연관시키면

g(2x) = 2xg(1

2x) + 2x¡ 1 = xg(

1

x) + 2x¡ 1

2g(x) = 2xg(1

x) + 2x¡ 2

의 두 식이 같아야 한다. 즉, xg( 1x ) = 1, g( 1x) = 1

x 이다. x 신 1x 를 입하면 g(x) = x. 따라서,

f(x) = x+ 1 뿐이다. ¤

35. 다음 방정식을 풀어라. (캐나다 1992-4)

x2 +x2

(x+ 1)2= 3

풀이 양변에 (x+ 1)2을 곱해주고 전개하여 정리하면 다음을 얻는다.

x4 + 2x3 ¡ x2 ¡ 6x¡ 3 = 0

이것은 다음과 같이 인수분해된다.

(x2 ¡ x¡ 1)(x2 + 3x+ 3) = 0

x2+3x+3은 판별식이 음이므로 항상 양의 값을 갖고, 따라서, x2¡x¡1 = 0의 근을 구하면 x = 1§p52

¢ ¢ ¢ 답 }

36. 실수 a, b, c가 임의의 실수 x, y, z에 하여 다음 부등식을 만족시킬 때, 3a+ 2b¡ c 의 최 값은 얼마인가? (한국 2004 1차-J3)

x2 + 4xy + 4y2 + axz + byz + cz2 = 0

풀이 x에 한 식으로 보고 x에 한 내림차순으로 정리하자.

x2 + (4y + az)x+ (4y2 + byz + cz2) ¸ 0

이 이차식이 임의의 x에 해 항상 성립해야 하므로 판별식

D = (4y + az)2 ¡ 4(4y2 + byz + xz2) · 0

이어야 한다. 이번에는 y에 한 내림차순으로 정리하면

(8az ¡ 4bz)y + (a2 ¡ 4c)z2 · 0

z = 0 일 때는 당연히 성립하고, z 6= 0 일 때도 임의의 y에 해 성립해야 하므로 8az ¡ 4bz = 0 이고a2 ¡ 4c · 0 이어야 한다. 즉,

b = 2a; c ¸ a2

4

그럼 문제의 식은

X = 3a+ 2b¡ c · 7a¡ a2

4= ¡

³a2¡ 7´2+ 49 · 49

로 최 값은 a = 14, b = 28, c = 49 일 때 49가 된다. }

관찰 준식을 (x+ 2y)2 ¸ ¡z(ax+ by + cz) 로 정리할 수 있고, 여기에 z = 1, x = ¡3, y = ¡2 를

입하면 49 ¸ 3a+2a¡ c 의 부등식을 얻는다. 여기서 등호조건만 찾아 말해줘도 답은 확실히 구할 수 있다.

Page 100: 실전수학올림피아드 1400제 해답

100 대수

37. a, b, c는 양의 실수이고 a ¸ b ¸ c 이다. 다음 부등식이 성립함을 보여라. (통신강좌 1997-14-6)

a2 ¡ b2

c+

c2 ¡ b2

a+

a2 ¡ c2

b¸ 3a¡ 4b+ c

증명 a+ b ¸ 2c이므로 양변에a¡ b

c을 곱하면

a2 ¡ b2

c¸ 2(a¡ b)이다.

a+ c > b이므로 양변에a¡ c

b를 곱하면

a2 ¡ c2

b¸ a¡ c이다.

b+ c · 2a이므로 양변에b¡ c

a를 곱하면

b2 ¡ c2

a· 2(b¡ c)이고

c2 ¡ b2

a¸ 2(c¡ b)이다.

위 세 결과를 더하면

a2 ¡ b2

c+

c2 ¡ b2

a+

a2 ¡ c2

b¸ 3a¡ 4b+ c이다. ¤

38. x는 1 · x · 3 범위의 실수이다. x2의 소수부가 x의 소수부와 같다고 한다. 이런 x는 모두 몇 개인가?(플란더즈 1995-2)

풀이 1 · x2 · 9이므로 x2의 정수부 크기에 따라 방정식을 9개 얻을 수 있다. 따라서 답은 9개 ¢ ¢ ¢답 }

39. 실수 x, y, z, w가 n = 2; 4; 6; 8 에 해 다음 식을 만족한다:

x2

n2 ¡ 12 +y2

n2 ¡ 32 +z2

n2 ¡ 52 +w2

n2 ¡ 72 = 1

x2 + y2 + z2 + w2 의 값을 구하여라. (AIME 1984-15)

풀이 %EEE

Solving is messy, x^2 = 11025/1024 etc. So trick needed. Write

equations as x^2/(t - 1) + y^2/(t - 9) + z^2/(t - 25) + w^2/(t - 49) = 1.

Multiply out to get quartic in t, namely

t^4 - (84+x^2+y^2+z^2+w^2)t^3 + ... = 0,

which has 4 known roots 4, 16, 36, 64, so must be t^4 - 120t^3 + ... = 0.

¢ ¢ ¢ 답 36 }

40. a, b, c, p는 실수 이고 a+ 1b= b+ 1

c= c+ 1

a= p 일 때, abc+ p = 0 임을 증명하여라. 단, a, b, c가 모

두 같지는 않다. (ML프로포절 161-1)

증명 (KAIST 과학영재센터 연구원 고봉균)

(1) a+ 1b= b+ 1

c 에서 (a¡ b) = (b¡ c)=bc, 그럼 순환적이므로

(a¡ b) =(b¡ c)

bc= ¢ ¢ ¢ = (a¡ b)

(abc)2:

따라서 (abc)2 = 1. jabcj = 1.(2) 문제에 주어진 식을 조금 변형하여 abc = pbc ¡ c = pca ¡ a = pab¡ b 를 얻을 수 있고, (1)에서처럼 둘씩 빼면

(a¡ b) = ¡pa(b¡ c) = ¢ ¢ ¢ = ¡p3abc(a¡ b):

따라서 p3abc = ¡1.(1), (2)의 결과에 의해 jp3j = 1, 즉 jpj = 1 이고, (2)의 결과에서 p와 abc가 부호가 달라야 함을 알 수있다. 따라서, abc+ p = 0. ¤

41. 실수 a1; a2; : : : ; a100 들이 다음을 만족한다.

a1 ¸ a2 ¸ ¢ ¢ ¢ ¸ a100 ¸ 0; a1 + a2 · 100; a3 + a4 + ¢ ¢ ¢+ a100 · 100a21+a

22+¢ ¢ ¢+a2100 이 가질 수 있는 최 값을 구하고,이 최 값일 때의 모든 가능한 수열 a1; a2; : : : ; a100을

찾아라. (캐나다 2000-5)

Page 101: 실전수학올림피아드 1400제 해답

2.2 대수 고급문제 101

풀이 아래 그림과 같이 200£ 100 크기의 사각형에서 a21 + ¢ ¢ ¢+ a2100 을 정사각형들의 넓이의 합으로

생각해보자.

a1 a2 a3 a4 a5 a6

¢ ¢ ¢

A

B

C

100

직사각형 A와 B는 높이가 같다. 0 < a2 < 50 일 때는 A보다 B가 더 넓으므로

A < B + C

따라서,a21 + ¢ ¢ ¢+ a2100 < a21 + ¢ ¢ ¢+ a2100 + (B + C ¡A) = 50 ¢ 200 = 10000

이 된다. a2 = 0 일 때와 a2 = 50 일 때는 각각

(a1; : : : ; a100) = (100; 0; : : : ; 0) 과 (50; 50; 50; 50; 0; : : : ; 0)

으로 모두 a21 + ¢ ¢ ¢+ a2100 = 10000 이고 이 때가 최 이다. }

42. 정수 a, b, c가a+ 2b+ 3c = 35; ab+ bc+ ca = 61

을 만족시킬 때, a의 최 값을 구하여라. (한국 2003 1차-J16)

풀이 (전남과학고 1학년 심민수)

a = 35¡ 2b¡ 3c 를 오른쪽 식 a(b+ c) + bc = 61 에 입하면 35(b+ c)¡ 2b2 ¡ 3c2 ¡ 5bc+ bc = 61 이고, 이것을 b에 관한 내림차순으로 정리하면

2b2 + (4c¡ 35)b+ (3c2 ¡ 35c+ 61) = 0

이 된다. 정수해 b가 존재해야 하므로 근의 공식

b =(35¡ 4c)§pD

4(¤)

꼴에서 §pD = 4b¡ (35¡ 4c)X 가 정수, 즉 D는 완전제곱수라야 한다.

D = (4c¡ 35)2 ¡ 8(3c2 ¡ 35c+ 61) = 737¡ 8c2

에서 D는 홀수임을 알 수 있으므로 D = (2n+ 1)2 이라 하면(n ¸ 0), 4n(n+ 1) = 736¡ 8c2, 즉

n(n+ 1)

2= 92¡ c2

여기서 jcj · 9 임을 알 수 있고, 이 범위에서 92¡ c2 이n(n+1)2

= 1 + 2 + ¢ ¢ ¢+ n꼴의 수가 되는 것을

찾으면 된다.

92¡ c2 = 92; 91; 88; 83; 76; 67; 56; 43; 28; 9

n(n+ 1)

2= 0; 1; 3; 6; 10; 15; 21; 28; 36; 45; 55; 66; 78; 91; : : :

등이 가능하므로, 겹치는 것은 28과 91뿐이다. 즉, jcj = 1 or 8이고, 그 때 각각 n = 13과 7이어서pD = 27, 15이다. (¤)에 입하여 b가 정수가 되는 경우들을 구하면

(b; c) = (1; 1); (3;¡1); (¡3; 8); (13;¡8)

이다. 각각의 경우 a = 35¡ 2b¡ 3c = 30, 32, 17, 33이므로 a의 최 값은 33이다. }

Page 102: 실전수학올림피아드 1400제 해답

102 대수

43. 음이 아닌 실수 a, b, c, d, e, f , g가 a+ b+ c+ d+ e+ f + g = 1 을 만족한다.

M = maxfa+ b+ c; b+ c+ d; c+ d+ e; d+ e+ f; e+ f + gg

라 할 때 M의 최소값을 구하여라. (통신강좌 1998-17-10)

풀이 (권익재)

M =1

3일 때, a =

1

3, d =

1

3, g =

1

3이면 성립한다.

이제 M <1

3이면 모순임을 보이자.

a+ b+ c; e+ f + g <1

3이므로 a+ b+ c+ e+ f + g <

2

3

a+ b+ c+ d+ e+ f + g = 1에 의해서 d >1

3

c+ d+ e >1

3이므로 이는 M <

1

3에 모순이 된다.

)M 의 최소값은1

3이다. }

별해 (손서연)

M = maxfa; a+ b; a+ b+ c; b+ c+ d; c+ d+ e; d+ e+ f; e+ f + g; f + g; gg 로도 쓸 수 있다. 여기에는 각 문자가 3번씩 포함되어 있으므로 이 집합의 평균은 3=9 = 1=3이다. 따라서, 그 중 최 값인 M은M ¸ 1=3 이다. 등호는 a = d = g = 1=3 이고 나머지가 0일 때 성립한다. }

44. 함수열 fn은 f1(x) = x 이고 fn(x) =pfn¡1(x)¡ 1

4 로 정의된다(n 2 N, n ¸ 2).

(a) 두 함수가 정의되는 모든 실수 x에 해 fn(x) · fn¡1(x) 임을 증명하여라.

(b) 각각의 n에 해, fn(x) = x 를 만족하는 x를 구하여라. (플란더즈 1994-4)

풀이 fn¡1(x)¡ fn(x) = (pfn¡1(x)¡ 1

2)2 ¸ 0이므로 항상 fn(x) · fn¡1(x)이다.

한편 위의 부등식에서 등호는 fn¡1(x) = 14일 때 성립하는데, f1(

14) = 1

4이고 fn¡1( 14 ) =14이면

fn(14) = 1

4이므로 귀납적으로 임의의 자연수 n에 해 fn(14) = 1

4이며 fn(x) = x의 유일한 해가 된다

는 알 수 있다. 즉 모든 n에 해 fn(x) = x의 해는 x = 14이다. }

45. a1; a2; : : : ; a121은 1000을 넘지않는 자연수들의 수열이다. 값 n은 이 수열에서 다른 수들보다 더 여러 번나타나고, m은 이 수열의 모든 항의 평균이다. [m¡ n]의 가능한 최 값은 무엇인가? (AIME 1989-11)

풀이 n에 해당하는 모든 항들을 1씩 줄이면 m은 1보다는 덜 줄어들고 n은 1 줄어들고 하니 m¡ n이

증가함. 따라서, n = 1 일 때만 보면 되고, 이 때 m의 최 값을 생각하면 됨.%EEE

Optimum is evidently n+1 terms = 1, n terms = 1000, 999, 998, ...

(as far as necessary). n = 1 gives 2 of 1, 1 each of 881, ... , 1000,

mean 924.9, diff 923. n = 2 gives 2 each of 942, ... , 1000, mean 946.9,

diff 945. n = 3 gives 3 each of 962, ... , 1000, mean 948.6, diff 947.

n = 4 gives 4 each of 972, ... , 1000, mean 945.3, diff 944. In the

general case the mean is (1001 - (120-n)/n + 1000) (120 - n)/2 + n+1)/121,

so we wish to maximise (2002 - 120/n)(120 - n)/2 + n and hence to maximise

-1000n - 7200/n, or equivalently to minimise 36/n + 5n. Easily seen this

is at n = 3.

¢ ¢ ¢ 답 947 }

46. F는 2개 이상의 원소를 갖는 유한집합이다. 모든 x; y; z 2 F 에 해 다음을 만족하는 연산 ¤가 존재하는지 증명 혹은 반증하여라.

(1) x ¤ z = y ¤ z =) x = y (오른쪽 소거법칙이 성립)

(2) x ¤ (y ¤ z) 6= (x ¤ y) ¤ z (결합법칙이 성립하는 경우가 없음) (Putnam 1984)

Page 103: 실전수학올림피아드 1400제 해답

2.2 대수 고급문제 103

풀이 존재한다. F의 원소를 원형으로 배열해놓고 각 원소 x에 해 그 오른쪽 원소를 x0으로 말하기

로 하자. 그럼

x ¤ y = x0

로 하면 (1), (2)를 모두 잘 만족함을 쉽게 알 수 있다. }

47. a1 = 1, a2 = 1,

an =an¡2an¡1

an¡2 + an¡1(n ¸ 3)

으로 주어진 수열 fang에서, a12를 구하여라. (1990 뉴욕주 수학리그)

풀이 양변에 역수를 취하면

1

an=

an¡2 + an¡1an¡2an¡1

=1

an¡1+

1

an¡2

즉, bn =1an

으로 치환하면 b1 = b2 = 1, bn = bn¡1 + bn¡2 의 피보나치 수열이 된다. 1, 1, 2, 3, 5, 8,

13, 21, 34, 55, 89, 144 에서 b12 = 144 이므로 a12 =1b12

= 1144

¢ ¢ ¢ 답 }

48. 모든 정수 x에 해 f(f(x)) = x+ 1 을 만족하는 함수 f : Z! Z 는 존재하지 않음을 보여라.(아벨콘테스트 1994 결선 3b)

증명 먼저 f가 일 일 응임을 보이겠다.만일 f(x) = f(y)라 하면 f(f(x)) = f(f(y))이므로 x+1 =

y + 1이 되어 x = y이다. 따라서 f는 일 일 함수이다. 또, 임의의 x에 해 f(f(x ¡ 1)) = x이므로f(y) = x인 y = f(x¡ 1)이 존재한다. 그러므로 f는 일 일 응이다.f가 일 일 응이므로 f(x) = 0인 정수 x가 유일하게 존재한다. 귀납적으로 f(x+n) = n임을 보이자.

먼저 n = 0인 경우, 가정에 의해 성립한다.

만일 n = k일때 f(x+ k) = k가 성립한다 하자. 그러면

f(f(x+ k)) = f(k) = x+ k + 1

이고 양변에 다시 f를 취하면

f(x+ k + 1) = f(f(k)) = k + 1

되어 f(x+ k + 1) = k + 1이 성립한다. 따라서 모든 정수 n ¸ 0에 해 f(x+ n) = n이다.

만일 n = ¡k일때 f(x¡ k) = ¡k가 성립한다 하자. 그러면

f(f(¡k ¡ 1)) = ¡k = f(x¡ k)

이다. f가 일 일 응이므로

f(¡k ¡ 1) = x¡ k

이고 x¡ k = f(f(x¡ k ¡ 1))에서

f(¡k ¡ 1) = f(f(x¡ k ¡ 1))

이다. 그러므로 f(x¡ k ¡ 1) = ¡k ¡ 1이다. 즉, 모든 정수 n · 0에 해 f(x+ n) = n이 성립한다.n = x+1이라 하자. 그러면 f(2x+1) = x+1이다. 그런데 f(x) = 0에서 f(0) = f(f(x)) = x+1이고,f가 일 일 응이므로 2x+ 1 = 0여야 한다. 이는 x가 정수라는데 모순이다. 즉, 조건을 만족하는 f가존재하지 않는다. ¤

49. 서로 다른 두 양의 홀수 a와 b가, (a¡ b)2 · 8pab 를 만족시킨다고 하자. n = ab 일 때, 방정식

x2 ¡ 2([pn ] + 1)x+ n = 0

의 해가 모두 양의 정수임을 보여라. 단, 임의의 실수 r에 하여 [r]은 r보다 크지 않은 가장 큰 정수를나타낸다. (한국 2003-J2)

Page 104: 실전수학올림피아드 1400제 해답

104 대수

증명 (부산 건국고 1년 신승현)

a = 2a1 + 1, b = 2b1 + 1 (a1; b1 2 N) | ①

ⅰ) (a+ b)2 > 4ab (* a 6= b)

①을 입하여 정리하면, (a1 + b1 + 1)2 > ab | ⓐ

ⅱ) (a¡ b)2 · 8pab < 4(a+ b) (* 산술-기하평균, a 6= b)(a¡ b)2 < 4(a+ b)에 ①을 입하여 정리하면,

a21 + b21 ¡ 2a1b1 < 2a1 + 2b1 + 2

양변에 적당히 더하고 빼면,

a21 + b21 + 2a1b1 ¡ 1 < 4a1b1 + 2a1 + 2b1 + 1(a1 + b1)

2 ¡ 1 < (2a1 + 1)(2b1 + 1) = ab

(a1 + b1)2 ¡ 1 < ab () (a1 + b1)

2 · ab | ⓑ

ⓐ, ⓑ에서

(a1 + b1)2 · ab < (a1 + b1 + 1)

2

a1 + b1 ·pab < a1 + b1 + 1

) [pab ] = [

pn ] = a1 + b1

이것을 주어진 방정식에 넣고 인수분해를 해서 답을 구해보면,

x = 2a1 + 1; 2b1 + 1 = a; b

따라서, 두 해는 a, b가 되어 해가 모두 양의 정수이다. ¤

50. 다음 조건을 만족하는 함수 f : R! R 과 g : R! R 를 모두 구하여라.

(1) 임의의 실수 s, t에 해, 2f(s)¡ g(s) = f(t)¡ t.

(2) 임의의 실수 s에 해, f(s)g(s) ¸ s+ 1. (플란더즈 1999-3)

풀이 (1)의 t에 s를 입하면 f(s)+s = g(s). 이것으로 g를 소거할 수 있음. (1') f(s)¡s = f(t)¡t. 즉

f(s)¡s = f(0)¡0으로 상수. f(s) = s+a. (2') f(s)(f(s)+s) ¸ s+1에 입하면 (s+a)(2s+a) ¸ s+1,

즉 2s2+(3a¡1)s+(a2¡1) ¸ 0. 판별식으로 풀면 D = (3a¡1)2¡8(a2¡1) = a2¡6a+9 = (a¡3)2 · 0.따라서, a = 3. }

51. 1보다 큰 모든 자연수 n에 해 다음을 증명하여라. (헝가리 1938-2)

1

n+

1

n+ 1+

1

n+ 2+ ¢ ¢ ¢+ 1

n2 ¡ 1 +1

n2> 1

증명 분모가 클수록 수는 작아지므로

준식 =1

n+

µ1

n+ 1+

1

n+ 2+ ¢ ¢ ¢+ 1

n2

¶>1

n+

µ1

n2+1

n2+ ¢ ¢ ¢+ 1

n2| {z }n2 ¡ n개

=1

n+

n2 ¡ n

n2=1

n+

n¡ 1n

= 1

로 성립함을 알 수 있다. 위 부등식에서 n ¸ 2 일 때 fn+ 1; : : : ; n2 ¡ 1g 6= ? 이므로 등호를 분명히 배제할 수 있다. ¤

Page 105: 실전수학올림피아드 1400제 해답

2.2 대수 고급문제 105

별증1 두 번째 항부터 n개씩 항을 묶어 다음과 같이 분석할 수 있다.

준식 =1

n+

µ1

n+ 1+ ¢ ¢ ¢+ 1

2n

¶+

µ1

2n+ 1+ ¢ ¢ ¢+ 1

3n

¶+ ¢ ¢ ¢+

µ1

(n¡ 1)n+ 1 + ¢ ¢ ¢+1

n2

¶¸ 1

n+³ n

2n

´+³ n

3n

´+ ¢ ¢ ¢+

³ n

n2

´=1

n+

µ1

2+1

3+ ¢ ¢ ¢+ 1

n

n ¸ 4 이면 괄호 안에 12+ 13+ 14 이 있으므로 1보다 항상 크고, n = 3 일 때도 1

3+ ( 1

2+ 13) 이 있으므

로 역시 1보다 크다. n = 2 일 때는 준식에서 따로 확인하여 12+ 13+ 14> 1 로 역시 성립한다. ¤

별증2 k ¸ 1 일 때 다음의 성질을 이용하자.

1

2k¡1 + 1+

1

2k¡1 + 2+ ¢ ¢ ¢+ 1

2k¸ 1

2k+1

2k+ ¢ ¢ ¢+ 1

2k= 2k¡1 ¢ 1

2k=1

2

등호는 항이 딱 하나뿐일 때인 k = 1 일 때만 성립한다. 즉, [2k¡1 + 1; 2k]꼴의 구간이 구간 [n; n2] 안에두 번 포함되면 1

2+ 12= 1 보다 크게 됨을 알 수 있다. 즉,

[2k¡1 + 1; 2k]; [2k + 1; 2k+1] ½ [n; n2]

인 k가 언제 존재하는지 살펴보자. n · 2k¡1 + 1 을 만족하는 가장 작은 자연수 k를 잡자(n ¸ 2 이므로 k ¸ 1). 그럼 k의 최소성에서 2k¡2 + 1 < n, 즉 2k¡2 + 2 · n 이다. 2를 이항하고 양변에 8을곱하면 2k+1 · 8n ¡ 16 이다. 따라서, 8n ¡ 16 · n2 이면 원하는 k가 틀림없이 존재하는데, 이것은n2 ¡ 8n+ 16 = (n ¡ 4)2 ¸ 0 으로 항상 잘 성립한다. 그러므로, 문제의 부등식은 항상 잘 성립하는 부등식이다. ¤

별증3 코시-슈바르츠 부등식에서

(n+ (n+ 1) + (n+ 2) + ¢ ¢ ¢+ n2)

µ1

n+

1

n+ 1+

1

n+ 2+ ¢ ¢ ¢+ 1

n2

¶¸ (1 + 1 + ¢ ¢ ¢+ 1)2

즉,

(n+ n2)(n2 ¡ n+ 1)

2(준식) ¸ (n2 ¡ n+ 1)2

(준식) ¸ 2(n2 ¡ n+ 1)

n2 + n

이것이 1보다 크면 충분하다.

2(n2 ¡ n+ 1)

n2 + n> 1

2n2 ¡ 2n+ 2 > n2 + n

n2 ¡ 3n+ 2 > 0(n¡ 1)(n¡ 2) > 0

으로 동치변형되므로, n > 2 이면 이 부등식은 항상 잘 성립한다. n = 2 일 때도 직접 확인하거나 코시부등식의 등호가 성립하지 않음 등을 관찰하면 역시 성립한다. ¤

52. 모든 음 아닌 정수 x, y (x ¸ y) 에 해 다음을 만족하는 함수 f : N0 ! R 을 모두 구하여라. (N0은 모든 음 아닌 정수들의 집합이다.) (오스트리아-폴란드 1979-4)

f(x+ y) + f(x¡ y) = f(3x)

Page 106: 실전수학올림피아드 1400제 해답

106 대수

풀이 x = y = 0 을 입하면 f(0) + f(0) = f(0) 이므로 f(0) = 0.

y = 0 을 입하면 2f(x) = f(3x) 이고, y에 x를 입하면 f(2x) + f(0) = f(3x), 여기서 f(0) = 0 이므로

f(3x) = f(2x) = 2f(x)

가 된다. 이제 x; y에 2x; x를 입하면

f(3x) + f(x) = f(6x)

2f(x) + f(x) = 2f(2x) = 4f(x)

f(x) = 0

따라서, 문제의 함수방정식을 만족하는 f(x)는 영함수뿐임을 알 수 있다. }

53. 두 문자 x, y에 한 이차식이 두 일차식의 곱으로 인수분해되려면, 한 문자에 한 식으로 보았을 때의판별식의 판별식이 0이 되어야 함을 증명하여라.

증명 두 일차식의 곱을

(ax+ by + c)(a0x+ b0y + c0)

= aa0x2 + (ab0 + a0b)xy + bb0y2 + (ac0 + a0c)x+ (bc0 + b0c)y + cc0

이라 하면, 이것을 한 문자 x에 한 식으로 보았을 때의 판별식은

D = ((ab0 + a0b)y + (ac0 + a0c))2 ¡ 4aa0(bb0y2 + (bc0 + b0c)y + cc0)

= (ab0 ¡ a0b)2y2 + 2(ab0 ¡ a0b)(ac0 ¡ a0c)y + (ac0 ¡ a0c)2

= ((ab0 ¡ a0b)y + (ac0 ¡ a0c))2

으로 y에 한 완전제곱식이 된다. ¤

54. n은 자연수이고 k는 자연수 a1; a2; : : : ; an들의 합보다 큰 자연수이다. a1! a2! ¢ ¢ ¢ an! < k! 임을 증명하여라. 단, n!은 1부터 n까지의 자연수를 모두 곱한 것이다. (헝가리 1937-1)

증명 좌변 a1! a2! ¢ ¢ ¢ an! 에는 a1+ a2+ ¢ ¢ ¢+an개의 수가 곱해져있는데, 개는(ai!의 안에서는) 1씩

증가시키면서 곱하다가(ai! = 1 ¢ 2 ¢ ¢ ¢ ¢ ¢ ai) 가끔(ai!의 ai를 곱한 후 다음의 ai+1!의 1을 곱하는 것으로

넘어갈 때) 1로부터 다시 곱셈을 시작하게 된다. 우변은 1부터 a1 + a2 + ¢ ¢ ¢ + an개(좌변에 곱해진 수의 개수)보다 더 많은 수들을 계속 1씩 증가시키면서 곱한 것이다. 따라서, 각각의 곱해진 항을 차례로비교하면 우변이 더 크거나 같고, 우변에는 남는 항도 있으므로 우변이 더 큼을 알 수 있다. ¤

별증 서로 다른 n종류의 물건이 각각 a1; a2; : : : ; an개씩 있을 때, 이들을 일렬로 나열하는 경우의 수

는(a1 + a2 + ¢ ¢ ¢+ an)!

a1! a2! ¢ ¢ ¢ an! 으로 세어진다(고등학교 과정). 따라서, 이것은 경우의 수이므로 자연수이고,

즉 1 이상이다. 이로부터, k > a1 + ¢ ¢ ¢+ an 이므로

k!

a1! a2! ¢ ¢ ¢ an!>(a1 + a2 + ¢ ¢ ¢+ an)!

a1! a2! ¢ ¢ ¢ an!¸ 1

임을 알 수 있다. ¤

55. a, b, c가 한 삼각형의 세 변일 때, 다음을 증명하여라: (인도지역예선 1999-5)

a

c+ a¡ b+

b

a+ b¡ c+

c

b+ c¡ a¸ 3

증명 a = y + z, b = z + x, c = x+ y 의 Ravi 치환을 하면, 좌변은y+z2y

+ z+x2z

+ x+y2x

= 12( zx+ x

z+

yx) + 3

2¸ 32¢ 3q

zx¢ xz¢ xy+ 32= 32+ 32. 마지막 부등식은 AM-GM. ¤

Page 107: 실전수학올림피아드 1400제 해답

2.2 대수 고급문제 107

56. n개의 수를 모았을 때(n > 2) 각 수가전체합

n¡ 1 보다 작으면 이 모음을 `북적 는' 모음이라 부른다.

fa; b; c; : : : g는 합이 S인 n개의 수들의 북적 는 모음이라 하자. 다음을 증명하여라.

(a) 이 수들은 모두 양수이다.

(b) 항상 a+ b > c 가 성립한다. (a, b, c는 이 모음의 임의의 세 수)

(c) 항상 a+ b ¸ S

n¡ 1 가 성립한다. (Towns 1992봄 JA1)

증명 임의의 항 a에 해, S에서 a를 제외한 나머지 n¡ 1개의 항을 더한 것을 생각해보자.

S ¡ a = b+ c+ ¢ ¢ ¢| {z }n¡1개

< (n¡ 1) S

n¡ 1 = S

따라서, S를 소거하고 a를 넘기면 a > 0 임을 알 수 있다. a는 임의의 항이었으므로, 모든 항은 양수. 즉(a)가 증명되었다.

이번에는 임의의 두 항 a, b에 해, S에서 a, b를 제외한 나머지 n¡ 2개의 항을 더한 것을 생각하자.

S ¡ (a+ b) = c+ d+ ¢ ¢ ¢| {z }n¡2개

< (n¡ 2) S

n¡ 1 = S ¡ S

n¡ 1

정리하면 a+ b > Sn¡1 이 되므로 (c)도 증명되었다(특히, 등호는 성립하지 않는다는 것도 알 수 있다).

(c)가 증명되었으면 a+ b ¸ Sn¡1 > c 이므로 (b)는 당연히 성립한다. ¤

57. 다음을 만족하는 함수 f : R+ ! R 를 모두 찾아라. (오스트리아-폴란드 1978-1)

임의의 x; y > 0 에 해 f(x+ y) = f(x2 + y2)

58. m1;m2; : : : ;mk는 합이 S인 양의 실수들이다. 다음을 증명하여라. (베트남 1980-2)

kXi=1

µmi +

1

mi

¶2¸ k

µk

S+

S

k

¶2

59. 다음 연립방정식의 모든 실수해를 구하여라. (Towns 1988봄 JA3)

(x3 + x4 + x5)5 = 3x1

(x4 + x5 + x1)5 = 3x2

(x5 + x1 + x2)5 = 3x3

(x1 + x2 + x3)5 = 3x4

(x2 + x3 + x4)5 = 3x5

풀이 윤환식이므로 WLOG: x1이 최 라 하자. 그럼 x3 + x4 + x5 · x4 + x5 + x1 이므로 그 5제

곱끼리도 부등호 방향은 불변이고 따라서 3x1 · 3x2. 즉, x2도 최 . 마찬가지로 x3, x4, x5들도차례로 모두 최 이므로 다섯 수가 모두 같은 값. (3x)5 = 3x 를 풀면 3x = 1; 0;¡1, 즉 ¢ ¢ ¢ 답x1 = x2 = x3 = x4 = x5 =

13 또는 0 또는 ¡ 1

3}

60. A, B, C는 다음을 만족하는 음 아닌 실수들이다.

A4 +B4 + C4 · 2(A2B2 +B2C2 + C2A2)

(a) A, B, C 중 어느 것도 나머지 두 수의 합보다 크지 않음을 증명하여라.

(b) A2 +B2 + C2 · 2(AB +BC + CA) 임을 증명하여라.

(c) 원래의 부등식이 (b)로부터 얻어질 수도 있는가? (Towns 1987가을 J5)

Page 108: 실전수학올림피아드 1400제 해답

108 대수

증명 이것은 헤론의 공식을 전개했을 때 나오는 꽤 알려진 인수분해식임(A2에 한 내림차순으로 정

리하거나 하면 풀만함).

(A+ B + C)(¡A+B + C)(A¡B + C)(A+B ¡ C) ¸ 0

A + B + C = 0 이면 모두 0이므로 (a)가 성립. B + C = A 이면 A + B = 2B + C ¸ C 등이 바로확인되므로 역시 (a)가 성립. 이제 0인 항이 없을 때만 보자. A ¡ B + C;A + B ¡ C < 0 이라 하면둘을 합했을 때 2A < 0 로 모순. 즉, 음수인 항은 많아야 하나뿐이고, 곱이 양이므로 그럼 음수인 항은 없음. 결국 (a)는 항상 성립. 혹은, (A2 ¡ (B + C)2)(A2 ¡ (B ¡ C)2) · 0 까지만 인수분해한 후(B ¡ C)2 · A2 · (B + C)2, 즉 jB ¡ Cj · A · B + C 등으로도 (a)가 확인됨.(b)는 A2 · A(B + C) 등의 식을 모두 합하면 바로 확인됨. (c)는 (A;B;C) = (1; 2; 4) 등의 반례가 있음. ¤

61. a, b, c는 삼각형의 세 변의 길이이고, p+ q + r = 0 이다. a2pq + b2qr + c2rp · 0 임을 증명하여라.(Towns 1988가을 SA2)

증명 p, q, r의 부호를 한꺼번에 바꿔도 상관없으므로, WLOG p ¸ q ¸ r 이라 할 때 q ¸ 0 일 때만 보

면 된다. r = ¡(p+ q) 를 입하면 준식은 a2pq · b2q(p+ q)+ c2(p+ q)p = (b2+ c2)pq+(bq)2+(cp)2

과 동치. a < b+ c 이므로 (b+ c)2pq · (b2 + c2)pq+ (bq)2 + (cp)2 만 보이면 충분하고, 이것은 소거하고 정리하면 0 · (bq ¡ cp)2 이므로 성립. ¤

62. fn(k) = k + 1 (k = 1; 2; 3; : : : ; n ¡ 1), fn(n) = 1 이 성립한다. 각 자연수 n ¸ 2 에 해서, an, bn, cn,dn이 n에 의해서만 결정되는 함수

fn(x) = an + bnx+ cnjx¡ dnj

을 구하여라. (IMTS R10-3)

63. a, b, c, d는 a · b · c · d 와 a+ b+ c+ d ¸ 1 을 만족하는 양수들이다. a2 + 3b2 + 5c2 + 7d2 ¸ 1 임을증명하여라. (Towns 1989봄 SO1)

증명 a2+3b2+5c2+7d2 ¸ a2+(2ab+b2)+(2ac+2bc+c2)+(2ad+2bd+2cd+d2) = (a+b+c+d)2 ¸ 1.¤

64. f(x) = x4 + 17x3 + 80x2 + 203x+ 125 라 하자. f(3 §p3 ) = g(3 §p3 ) 과 f(5 §p5 ) = g(5 §p5 )

를 만족하는 다항식 g(x) 중에서 최소 차수인 것을 구하여라. (IMTS R11-5)

풀이 f(x) ¡ g(x)는 3 §p3 과 5§p5 를 모두 근으로 가지므로 f(x) ¡ g(x) = ((x ¡ 3)2 ¡ 3)((x ¡5)2 ¡ 5)Q(x) 로 4차식 이상임. Q(x)가 상수 1일 때 g(x)는 3차식이고, Q(x)가 다른 상수이면 g(x)는

4차식이고, Q(x)가 1차 이상이면 g(x)는 5차 이상임. 따라서, 3차식인 것이 최소 차수. ¢ ¢ ¢ 답 f(x)¡((x¡ 3)2 ¡ 3)((x¡ 5)2 ¡ 5) }

65. 방정식 x2 ¡ 12[x2 ¡ 2] = 2 + 3[ 1

2x2 ¡ 3] 의 모든 실수해를 구하여라. (몰도바 1996 최종-y9-6)

66. 수열 (xn), (yn), (zn)이 다음과 같이 주어져있다.

xn+1 = yn +1

xn; yn+1 = zn +

1

yn; zn+1 = xn +

1

zn(n ¸ 0)

단, x0, y0, z0는 주어진 양수이다. 이 세 수열에는 얼마든지 큰 수가 있음을 증명하여라.(오스트리아-폴란드 1981-6)

증명 Mn = max(xn; yn; zn) 이라 하자. 그럼 Mn은 증가수열이고 Mn+1 ¸ Mn +1

Mn. 귀류법으로

이 수열이 위로 유계, 즉 항상 Mn < N 이라 하자. 그럼 Mn+1 > Mn +1N, 따라서 Mn > M0 +

nN 이

되고, 특히 MN2 > M0 +N > N . ¤

67. n은 2 이상의 정수이다.3pn3 + 2n2 + n 의 십진전개에서 소수점 이하 첫번째 자리의 수를 구하여라.

(아일랜드 2004-8)

Page 109: 실전수학올림피아드 1400제 해답

2.2 대수 고급문제 109

풀이 (n+ 0:6)3 < n3 + 2n2 + n < (n+ 0:7)3. ¢ ¢ ¢ 답 6 }

68. x1 ¸ x2 ¸ x3 ¸ ¢ ¢ ¢ ¸ xn ¸ ¢ ¢ ¢ 인 양의 실수들의 수열 fxngn¸1이 모든 n에 해 다음을 만족한다.

x1

1+

x4

2+

x9

3+ ¢ ¢ ¢+ xn2

n· 1

모든 k에 해 다음이 성립함을 보여라. (인도지역예선 2000-3)

x1

1+

x2

2+

x3

3+ ¢ ¢ ¢+ xk

k· 3

69. 모든 x에 해서 (2x¡ 1)20 ¡ (ax+ b)20 = (x2 + px+ q)10 을 만족하는 실수 p; q; a; b를 모두 찾아라.(소련 1963-12)

70. 자연수 c가 주어졌을 때, 수열 ffng은 다음과 같이 정의된다.

f1 = 1; f2 = c; fn+1 = 2fn ¡ fn¡1 + 2 (n ¸ 2)

각 첨자 k에 하여, 다음을 만족하는 첨자 r이 항상 존재함을 보여라.

fkfk+1 = fr (IMO-SL 1984, 캐나다 출제)

증명 fn = (n¡ 1)c+ (n¡ 2)2. 그 다음은 그냥 전개 확인. r = fk + k. ¤

71. ab+ cd = 2pq 인 수 a, b, c, d, p, q가 있다. ac ¸ p2 > 0 이면 bd · q2 임을 보여라.(러시아 1990 4차-y9-2)

증명 bd > q2 가정하면 (ab+ cd)2 = 4p2q2 < 4abcd 로 모순. ¤

72. a2x2 + ax+ 1¡ 7a2 = 0 의 두 해가 모두 정수가 되도록 하는 양의 실수 a의 값을 모두 구하여라.(러시아 1990 4차-y9-5)

풀이 두 근의 합이 정수이므로 a = 1b꼴. 입하고 b2 곱해주면 x2 + bx+ b2 ¡ 7 = 0. 판별식이 제곱

수임을 이용. 28¡ 3b2 = 제곱수 ¸ 0. b = 1; 2; 3뿐. }

73. 3p2 + 3

p4 가 어떤 정계수 삼차방정식의 근이면, 이 삼차방정식의 실근은 이것뿐임을 증명하여라.

(IMTS R12-4)

증명 x = a + a2, a3 = 2 라 두면 x3 = a3(1 + a)3 = 6(a2 + a + 1) = 6(x + 1). y = x3 그래프와

y = 6(x+ 1) 그래프를 그려서 한 점에서만 만남을 확인...; ¤

74. x, y, z는 서로 다른 양의 실수들이다. 다음의 세 수¯x

y¡ y

x

¯;

¯y

z¡ z

y

¯;¯ zx¡ x

z

¯들이 한 삼각형의 세 변의 길이가 될 수 있는지 그렇지 않은지 알아내어라. (인도지역예선 1997-5)

풀이 (김 현)

일반성을 잃지 않고 x ¸ y ¸ z 라고 하자. 그러면 위의 세 수는

x

y¡ y

x;

y

z¡ z

y;

x

z¡ z

x

삼각형의 세 변이 될 수 있는지 알아보기 위해서는 변의 비가 필요하므로 xyz를 곱해주면

(x2 ¡ y2)z; (y2 ¡ z2)x; (x2 ¡ z2)y

(x2 ¡ y2)z+ (y2 ¡ z2)x¡ (x2 ¡ z2)y = (y¡ x)(z¡ x)(z¡ y) < 0 로 인수분해 정리되어 삼각부등식을만족하지 않으므로 삼각형을 이루지 못한다. }

Page 110: 실전수학올림피아드 1400제 해답

110 대수

75. x > y ¸ 0 일 때 다음 부등식을 증명하여라. (유고슬라비아 1979 고2-2)

x+4

(x¡ y)(y + 1)2¸ 3

증명 분모를 x ¡ y; y+12

; y+12 로 분해하면 산술-기하로 (좌변) ¸ x + ( 3

x+1)3 이 되고... 왼쪽의 x를

다시 x+13+ x+1

3+ x+1

3¡ 1 로 분해하면 다시 산술-기하로 (좌변) ¸ 4 4

p1¡ 1 이 됨. ¤

76. 다음 연립 방정식을 풀어라. (러시아 1992 4차-y9-1)

x3 ¡ 5y2

x=6

y; y3 ¡ 5x

2

y=6

x

증명 xy 곱하고, 두 식을 더해보고 또 빼보고. ¤

77. a, b, c, d는 다음을 만족하는 서로 다른 실수들이다.

a+ b+ c+ d = 3; a2 + b2 + c2 + d2 = 45

이 때, 다음 식의 값을 구하여라. (IMTS R18-4)

a5

(a¡ b)(a¡ c)(a¡ d)+

b5

(b¡ a)(b¡ c)(b¡ d)+

c5

(c¡ a)(c¡ b)(c¡ d)+

d5

(d¡ a)(d¡ b)(d¡ c)

풀이

a5

(a¡ b)(a¡ c)(a¡ d)+

b5

(b¡ a)(b¡ c)(b¡ d)+

c5

(c¡ a)(c¡ b)(c¡ d)+

d5

(d¡ a)(d¡ b)(d¡ c)

통분하고, 분자의 인수들 찾고, 칭식임을 보이고, 그럼 결과는 약분되고 2차식만 남아서 수치 입법.}

78. a; b; c ¸ 0 이고 a+ b+ c · 3 이다. 다음 부등식을 증명하여라. (러시아 1989 4차-y10-4)

a

1 + a2+

b

1 + b2+

c

1 + c2· 3

2· 1

1 + a+

1

1 + b+

1

1 + c

증명 왼쪽은 분모 산술-기하. 오른쪽은 산술-조화(혹은 코시). ¤

79. Milo는 Mindbender고등학교의 학생이다. 각 과목의 시험을 치른 후, 그는 항상 누적평균을 계산하고, 그평균값은 반올림된 가장 가까운 정수로만 나타낸다. (즉, 85.49점은 내림을 하여 85로, 85.50점은 올림을하여 86으로 나타낸다.) 오늘 그는 2개의 시험을 쳤다. 먼저 프랑스어 시험에서 75점을 받았고, 이것은 평균을 1점 떨어뜨렸다. 두 번째 역사 시험에서는 83점을 받았고, 이것은 평균을 2점 더 떨어뜨렸다. 그의현재 평균점수는 얼마인가? (IMTS R13-1)

풀이 어제까지 과목수를 n, 총점을 S, 반올림된 평균을 m+ 3 이라 하자.

m+ 2:5 · S

n< m+ 3:5; m+ 1:5 · S + 75

n+ 1< m+ 2:5; m¡ 0:5 · S + 158

n+ 2< m+ 0:5

n, S, m이 정수임에 유의하여 S를 소거해주는 방향으로 이 부등식을 풀면 됨. (문제를 음미해보면 평균은 m+3:5, m+1:5, m+0:5 에 가까웠을테니까, 아마도 첫식의 오른쪽, 둘째식의 왼쪽, 세째식의 오른쪽 부등식이 서로 연결된 의미가 클 듯) }

Page 111: 실전수학올림피아드 1400제 해답

2.2 대수 고급문제 111

80.x2 + y2

x2 ¡ y2+

x2 ¡ y2

x2 + y2= k 일 때 다음 식을 k에 관한 식으로 나타내어라.

E(x; y) =x8 + y8

x8 ¡ y8¡ x8 ¡ y8

x8 + y8(주니어발칸 1997-2)

풀이 통분하면 k =2(x4 + y4)

x4 ¡ y4. 마찬가지로

k

2+2

k=2(x8 + y8)

x8 ¡ y8:= r. 즉, E =

r

2¡ 2

r=

k2 + 4

4k¡ 4k

k2 + 4=(k + 2)2(k ¡ 2)24k(k2 + 4)

. }

81. x1 + ¢ ¢ ¢+ xn · 12 인 음 아닌 실수 x1; x2; : : : ; xn 에 해 다음을 증명하여라.

(1¡ x1)(1¡ x2) ¢ ¢ ¢ (1¡ xn) ¸ 1

2(폴란드 1966 3차-3)

증명 (1¡ x)(1¡ y) ¸ (1¡ (x+ y)) 이므로 귀납법. 뉴질랜드 2000-2 의 확장 ¤

82. x, y는 다음을 만족하는 양의 실수들이다.

x3 + y3 + (x+ y)3 + 30xy = 2000

x+ y = 10 임을 보여라. (주니어발칸 2000-1)

증명 x+y = p, xy = q 라 하면 준식은 2p3¡3pq+30q¡2000 = 0, (p¡10)(2p2+20p+200¡3q) = 0.산술-기하평균 부등식에서 p2 ¸ 4q 이므로 오른쪽항은 양수이고, 고로 p¡ 10 = 0. ¤

83. 모든 양의 실수 a, b, c에 하여 다음 부등식이 성립함을 증명하여라.

1

b(a+ b)+

1

c(b+ c)+

1

a(c+ a)¸ 27

2(a+ b+ c)2(주니어발칸 2002-4)

증명 a; b; c와 b+ c; c+ a; a+ b는 크기 순서가 반 이므로 재배열부등식의 최소일 때에 의해 LHS ¸1

a(b+c)+ 1

b(c+a)+ 1

c(a+b). 그담은 산술-조화(혹은 코시)를 쓰면 LHS ¸ 9

2(ab+bc+ca) 가 되고 이것을

RHS와 비교하는 것은 껌. 혹은 그냥 처음부터 쭈욱 산술-기하 부등식만 반복해서 사용해도 됨. ¤

84. x, y가 둘 다 0은 아닌 실수일 때 다음 부등식을 증명하여라. (주니어발칸 2004-1)

x+ y

x2 ¡ xy + y2· 2

p2p

x2 + y2

증명 (x+ y)2 · 2(x2 + y2) 임에서 x+ y · p2px2 + y2 이고, 또 x2 ¡ xy + y2 ¸ 1

2(x2 + y2) 이므

로, LHS ·p2px2+y2

12 (x

2+y2)= RHS. ¤

85. a1, a2, b1, b2, c1, c2는 a1a2 > 0, a1c1 ¸ b21, a2c2 ¸ b22 을 만족하는 실수들이다. (a1 + a2)(c1 + c2) ¸(b1 + b2)2 임을 증명하여라. (헝가리 1939-1)

증명 a1; a2; c1; c2 는 모두 부호가 같다.모두 음수일 때는 모두 부호만 바꿔줘도 식은 똑같으므로,모두

양수일 때만 생각해도 된다. 그럼 코시부등식으로 직접 끝. 혹은 (a1+a2)x2¡2(b1+ b2)x+(c1+ c2) =(a1x2 + b1x+ c1) + (a2x2 + b2x+ c2) 가 0이 될 수 없음을 말해도 끝. ¤

86. 방정식 4x4 ¡ ax3 + bx2 ¡ cx + 5 = 0 의 네 근 r1; r2; r3; r4 가 모두 양의 실수이고 다음을 만족함이 알려져있다. 이 네 근을 구하여라. (남미 1985-3)

r1

2+

r2

4+

r3

5+

r4

8= 1

Page 112: 실전수학올림피아드 1400제 해답

112 대수

풀이 (김 현) 근과 계수와의 관계에서 r1r2r3r4 =54이다. 네 근이 모두 양의 실수이므로 A:M ¸

G:M에 의해

1 =r1

2+

r2

4+

r3

5+

r4

8¸ 4 4r

r1r2r3r4

2 ¢ 4 ¢ 5 ¢ 8 = 44

r1

44= 1

따라서 등호 성립조건에서r1

2=

r2

4=

r3

5=

r4

8=1

4

(r1; r2; r3; r4) = (12; 1; 5

4; 2) }

87. a2 + b2 = c2 과 x2 + y2 = z2 을 만족하는 자연수 a; b; c와 x; y; z를 생각하자.

(a+ x)2 + (b+ y)2 · (c+ z)2

임을 증명하고 등호조건을 구하여라. (IMTS R15-4)

풀이 양변 전개후 소거하면 ax + by · cz 만 증명하면 됨. 양변 제곱하면 정확히 코시부등식 or 완전

제곱꼴. }

88. x, y는 양의 정수이고 y > 3 이다.

x2 + y4 = 2[(x¡ 6)2 + (y + 1)2]

일 때, x2 + y4 = 1994 임을 증명하여라. (아일랜드 1994-1)

89. 양이 아닌 실수 a1; a2; : : : ; a1996에 해 다음 부등식을 증명하여라. (몰도바 1996 최종-y10-6)

2a1 + 2a2 + ¢ ¢ ¢+ 2a1996 · 1995 + 2a1+a2+¢¢¢+a1996

증명 2a + 2b ¸ 1 + 2a+b. 귀납적으로. 등호조건은 ab = 0 일 때이므로 1995개의 ai가 0일 때. ¤

90. 다음을 만족하는 함수 f를 모두 구하여라.

(i) 실수값을 갖는다.

(ii) x 6= 23 인 모든 실수에 해 f(x)가 정의된다.

(iii) 23를 제외한 모든 실수 x에 하여 다음을 만족한다.

498x¡ f(x) =1

2f

µ2x

3x¡ 2¶

(호주 1992-6)

풀이 xà 2x

3x¡ 2 입... 연립방정식 }

91. 모든 실수에 하여 정의되고, 다음 두 성질을 만족하는 각각의 함수 f에 하여 f(p5753 )의 값을 구하

여라. (호주 1993-2)

f(xy) = xf(y) + f(x)y

f(x+ y) = f(x1993) + f(y1993)

풀이 f(0) = 0, f(1) = 0, f(x+ 1) = f(x1993) = f(x). 고로 f(n) = 0. f(pnpn ) = ::: }

92. 임의의 자연수 n에 하여f(n) = b2pn c ¡ bpn¡ 1 +pn+ 1 c

라 할 때, f(n) = 1 을 만족시키는 n을 모두 구하여라. (단, bxc는 x를 넘지 않는 최 의 정수이다.)(호주 1993-4)

Page 113: 실전수학올림피아드 1400제 해답

2.2 대수 고급문제 113

풀이 (m ¡ 1 · )pn¡ 1 +pn+ 1 < m · 2pn(< m + 1) 이라 놓고 부등식 풀이. 제곱해서 풀다보

면 아마 n ¸ 2 일 때 2pn2 ¡ 1 > 2n¡ 1 이 성립함을 유용하게 써먹... }

93. 서로 다른 세 양수가 주어져 있다. 이 세 수에

a

b+

b

c>

a

c+

c

a

가 성립하도록 적당한 순서로 a, b, c의 이름을 붙일 수 있음을 보여라. (러시아 1991 4차-y9-3)

증명 a > c > b 로 하면 됨... ¤

94. P (x)를 n ¸ 3 차의 정수 계수 다항식이라고 하자.

P (x1) = P (x2) = ¢ ¢ ¢ = P (xm) = 1

를 만족하는 서로 다른 정수 x1; : : : ; xm (m ¸ 3) 이 있을 때, P는 정수근을 가질 수 없음을 증명하여라.(체코슬로바키아 1986-2)

증명 xi들중 3개만 택하여보면 충분할. P (x)¡ 1 = (x¡ a)(x¡ b)(x¡ c)Q(x) 라 두고 n이 정수근이

라 하면 ¡1 = (n¡ a)(n¡ b)(n¡ c)Q(n), 즉 n¡ a; n¡ b; n¡ c j 1, n¡ a; n¡ b; n¡ c 2 f¡1; 1g. 이것은 a, b, c가 서로 다름에 모순. ¤

95. x 6= 1, y 6= 1, x 6= y 인 실수 x; y; z는 다음 식을 만족한다.

yz ¡ x2

1¡ x=

zx¡ y2

1¡ y

이 분수의 값이 x+ y + z 와 같음을 보여라. (남미 1985-4)

증명 변은 x+(yz-x)/(1-x) 이므로 (yz-x)/(1-x)=y+z 임을 보이면 충분하다. 양변에 1-x를 곱하면,

x+y+z=xy+yz+zx 임을 보이면 된다. 그런데 주어진 조건에서 한변에 모두 넘기고 x-y로 나누면 아마바로 이 식이 나옴 ¤

96. 다음 등식을 증명하여라. (몰도바 1996 최종-y11-1)

1

666+

1

667+ ¢ ¢ ¢+ 1

1996= 1 +

2

2 ¢ 3 ¢ 4 +2

5 ¢ 6 ¢ 7 + ¢ ¢ ¢+2

1994 ¢ 1995 ¢ 1996

증명 2(n¡1)n(n+1) =

1(n¡1)n ¡ 1

n(n+1)= 1

n¡1 ¡ 2n+ 1

n+1 이므로 2(3n¡1)3n(3n+1) =

13n¡1 +

13n+

13n+1

¡ 1n. 이것으로 전개하면 끝. ¤

97. a1 = an = 0이고 k = 2; 3; : : : ; n¡1에 해 ak¡1+ak+1 ¸ 2ak 를 만족하는 실수 a1; a2; : : : ; an (n ¸ 3)이 있다. 이 수들이 모두 0 이하임을 증명하여라. (폴란드 1967 3차-1)

증명 양의 항이 있다고 가정하고, 최 항(중에서도 가장 오른쪽항)을 잡으면 그 좌우항이 역시 최 항

이 되어야 해서 모순. ¤

98. 다음을 만족하는 함수 f : N! N 를 모두 구하여라.

(a) 모든 n;m 2 N 에 해, f(n+m) = f(n)f(m).

(b) 방정식 f(f(x)) = f(x)2 이 적어도 하나의 자연수해를 갖는다. (몰도바 1997 최종-y11-5)

풀이 f(1) = a 라 할 때 f(n) = an. (b)는 aan= a2n 이 되고, a > 1 이면 지수가 같아야 하므로

an = 2n. 그런데 a ¸ 3 이면 an ¸ 3n > 2n 으로 해 없음. ¢ ¢ ¢ 답 f ´ 1, f(n) = 2n. }

Page 114: 실전수학올림피아드 1400제 해답

114 대수

99. 다음 성질을 만족하는 함수 f : R+ ! R 를 모두 구하여라(R+는 양의 실수의 집합).

(i) 임의의 x; y 2 R+ 에 해 f(xy) = f(x)f( 3y) + f(y)f( 3

x)

(ii) f(1) = 12

(호주 1995-8)

풀이 f(3) = 12. f(3x) = f(x) = f( 3

x) = f( 1

x). f(xy) = 2f(x)f(y) = f(x

y). f(x2) ´ 1. }

100. 임의의 삼각형의 세 변의 길이 a, b, c에 해 다음 부등식이 성립함을 증명하여라:(인도지역예선 1995-5)

a2 + b2 + c2 >p3 maxfja2 ¡ b2j; jb2 ¡ c2j; jc2 ¡ a2jg

증명 WLOG a ¸ b ¸ c 이면 우변은p3(a2 ¡ c2). cos 제2법칙으로 b2 소거하고 (2¡p3 )a2 + (2 +p

3 )c2 ¸ 2ac AM-GM 이용하면. ¤

101. p(x) = xn + an¡1xn¡1 + ¢ ¢ ¢ + a1x + a0 은 정수 계수의 다항식이고, (p(x))2은 음이 아닌 계수들만을갖는 다항식이라고 한다. p(x)의 계수도 모두 음이 아니어야 하는가? 당신의 주장을 증명하여라.

(IMTS R43-3)

증명 반례: p(x) = x4 + 2x3 ¡ 2x2 + 4x+ 4, p(x) = x4 + 100x3 ¡ x2 + 100x+ 100 등 ¤

102. 모든 실수 x에 해 a(x) > 0 이고 b(x)2 ¡ 4a(x)c(x) < 0 인 함수 a, b, c에 의해 정의된 함수 f(x) =a(x)x2 + b(x)x+ c(x) 는 모든 실수 x에 해 f(x) > 0 임을 증명하여라.

증명 이 식을 일반화해서 두 변수 x, y에 관한 식 f(x; y) = a(y)x2 + b(y)x+ c(y) 로 생각. 각각의 고

정된 y에 해 항상 성립하므로 OK. ¤

103. 모든 자연수 n에 해 다음을 증명하여라. (아일랜드 1995-6)

nn · (n!)2 ·µ(n+ 1)(n+ 2)

6

¶n

증명 왼쪽 부등식은 1:2 로 짝을 지어 n · (k+ 1)(n¡ k) 임을 보여서 끝. 오른쪽은 n이 충분히 클 때

충 n! · nn

8로 만들어서 비교. ¤

104. 2 이상의 각 정수 n에 해 다음을 만족하는 두 양의 실수 a와 b 중에서 어느 것이 더 큰지 밝혀라.(미국 1993-1)

an = a+ 1; b2n = b+ 3a

풀이 a; b > 1 임은 당연. f(a) = an ¡ a¡ 1, g(b) = b2n ¡ b¡ 3a 라 두면 f , g 모두 1보다 큰 실수 범

위에서 단조증가. g(1) = ¡3a < 0 이고 g(a) = a2n ¡ a ¡ 3a = (a + 1)2 ¡ 4a = (a ¡ 1)2 > 0 이므로g(b) = 0 의 근 b는 구간 (1; a) 에 존재. }

105. 다음의 방정식이 모든 실수 a1; a2; : : : ; an+1들에 해서 항상 실수해를 갖도록 하는 자연수 n을 모두 구하여라. (오스트리아 1985-4)

an+1x2 ¡ 2x

qa21 + a22 + ¢ ¢ ¢+ a2n+1 + a1 + a2 + ¢ ¢ ¢+ an = 0

풀이 판별식 두 번 사용. x에 해 한번, 다시 an+1에 해 한번. a1 = a2 = ¢ ¢ ¢ = an = 1 을 입해

보면 n = 1; 2; 3; 4 만 가능. 코시로 실제 됨을 확인. }

Page 115: 실전수학올림피아드 1400제 해답

2.2 대수 고급문제 115

106. x; y; z > ¡1 일 때, 다음을 증명하여라. (주니어발칸 2003-4)

1 + x2

1 + y + z2+

1 + y2

1 + z + x2+

1 + z2

1 + x+ y2¸ 2

증명 x · 1+x2

2 이용하여 분모를 풀어주고 1 + x2 = a, ... 로 치환하면 ca+2b

+ ab+2c

+ bc+2a

¸ 1 꼴

로 변형됨. 이것은 (a+ 2b)c+ (b+ 2c)a+ (c+ 2a)b 를 양변에 곱해주면 코시부등식으로 됨. ¤

107. a1 =12 이고, n > 1 에 해서

an =an¡1

2nan¡1 + 1

로 정의된 수열이 있다. a1 + a2 + ¢ ¢ ¢+ a1998 을 계산하여라. (몰도바 1998 최종-y11-5)

풀이 1an= 1

an¡1 + 2n = ¢ ¢ ¢ = n(n+ 1). an =1n¡ 1

n+1 로 telescoping }

108. f : N ! N 는 증가함수이고, f(2) = a > 2, 또 모든 자연수 m, n에 해 f(mn) = f(m)f(n) 을 만족한다. a의 가능한 최소값을 구하여라. (북유럽 1987-3)

풀이 f(n) = n2 일 때 모든 조건 만족하므로 a = 4 는 가능. a = 3 은 불가능함을 보이자. f(3) = b

라 하면 f(32) > f(23) = 27 임에서 b > 5. f(35) < f(28) = 6561 임에서 b < 6. }

109. w, a, b, c는 서로 다른 실수들이고, 다음 연립방정식을 만족하는 실수 x, y, z가 존재한다.

x + y + z = 1

xa2 + yb2 + zc2 = w2

xa3 + yb3 + zc3 = w3

xa4 + yb4 + zc4 = w4

w를 a, b, c에 한 식으로 나타내어라. (아일랜드 1994-9)

풀이 '서로 다른'에 주목하여 먼저 a, b, c 중에 0이 없음을 확인하고, 그담엔 아래 세 식에서 아무 생각

없이 x, y, z를 연립하여 푼 다음, 이것을 첫식에 입하여 w에 한 4차식을 얻고, 원래 이 연립방정식이 w = a; b; c 를 해로 가진다는 것과 이 4차식이 일차항이 없음에 착안하여 abc+ bcw+ caw+ abw = 0

에서 마지막 해 w = ¡abc=(bc+ ca+ ab) 를 구함. }

110. a, b, c는 실수이고 a; c 6= 0 이다. ax2+ bx+ c = 0 의 한 실근을 r이라 하고 ¡ax2+ bx+ c = 0 의 한 실근을 s라 할 때, r과 s 사이에 a

2x2 + bx+ c = 0 의 한 실근이 있음을 증명하여라. (몰도바 2000 최종-y9-1)

증명 f(x) = a2x2 + bx+ c 이라 하면 f(r)f(s) = (¡a

2r2)( 3a

2s2) = ¡ 3

4(ars)2 < 0. 중간값 정리 ¤

111. 주어진 세 실수 a, b, c가 a+ b+ c > 0, bc+ ca+ ab > 0, abc > 0 을 만족한다. a; b; c > 0 임을 보여라.(Towns 1985봄 SO3)

증명 근과 계수. p; q; r > 0 일 때, x가 음수이면 x3 ¡ px2 + qx¡ r < 0 으로 근이 될 수 없다. ¤

112. A1 = f1g, A2 = f2; 3; 4g, A3 = f5; 6; 7; 8; 9g, ... 등의 집합들을 생각하자. bn을 An의 최소 원소와 최원소의 산술평균이라고 하자. 다음 식의 값이 소수임을 보여라.

2000

b2 ¡ 1+2000

b3 ¡ 1+ ¢ ¢ ¢+ 2000

b2000 ¡ 1(몰도바 2000 최종-y7-3)

증명 bn =12((n ¡ 1)2 + 1 + n2) = n2 ¡ n+ 1. 1

bn¡1 =1

n(n¡1) =1

n¡1 ¡ 1ntelescoping. ¢ ¢ ¢ 답

1999 ¤

Page 116: 실전수학올림피아드 1400제 해답

116 대수

113. x1 =12, xk+1 = x2k + xk 로 주어진 수열이 있다.

1

x1 + 1+

1

x2 + 1+ ¢ ¢ ¢+ 1

x100 + 1

의 정수부를 구하여라. (Towns 1985가을 J6)

풀이 부분분수에서 1xk+1

= 1xk¡ 1

xk+1. telescoping }

114. n개의 실수 a1; a2; : : : ; an 이 있는데, 총합은 0이고, 처음 k개의 항은 양이 아니며, 그 이후의 항들은 음이 아니라고 한다. a1 + 2a2 + 3a3 + ¢ ¢ ¢+ nan ¸ 0 임을 증명하고, 등호는 ai들이 모두 0일 때만 성립함을 보여라. (스웨덴 1966-2)

증명 a1+2a2+ ¢ ¢ ¢+kak+(k+1)ak+1+ ¢ ¢ ¢+nan ¸ ka1+ka2+ ¢ ¢ ¢+kak+kak+1+ ¢ ¢ ¢+kan = 0

이고 등호는 모든 항이 0일 때만. ¤

115. 임의의 자연수 n과 음 아닌 실수 a, b, c에 해 다음을 증명하여라. (스웨덴 1975-3)

an + bn + cn ¸ abn¡1 + bcn¡1 + can¡1

증명1 AM-GM 관계에서 an + bn + ¢ ¢ ¢+ bn ¸ nabn¡1. ¤

증명2 재배열 부등식. ¤

116. (x; y) 6= (0; 0) 일 때, 함수

f(x; y) =x2 ¡ xy + y2

2x2 + y2

의 최소값을 구하여라. (대학생경시 2002-40)

풀이 f(x; y) = (x2 ¡ xy + y2)=(2x2 + y2) = k 로 두고 k의 최소값을 구하자. 분모를 넘기면 x2 ¡xy + y2 = 2kx2 + ky2, 이것을 정리하면 다음과 같다.

(1¡ k)y2 ¡ xy + (1¡ 2k)x2 = 0

k가 구하는 최소값일 때 이 y에 관한 2차식은 해를 가지므로, 판별식

D = x2 ¡ 4(k ¡ 1)(2k ¡ 1)x2 ¸ 0

이다. x = 0 이면 k = 1 이 되고, x 6= 0 이면 1¡ 4(k ¡ 1)(2k ¡ 1) ¸ 0, 즉 8k2 ¡ 12k + 3 · 0. 근의 공식으로 풀면

3¡p34

· k · 3 +p3

4; 따라서 최소값은

3¡p34

}

117. 다음 연립부등식을 풀어라. (스웨덴 1992-3)

2x1 ¡ 5x2 + 3x3 ¸ 02x2 ¡ 5x3 + 3x4 ¸ 0

...

2x23 ¡ 5x24 + 3x25 ¸ 02x24 ¡ 5x25 + 3x1 ¸ 02x25 ¡ 5x1 + 3x2 ¸ 0

풀이 다 더하면 0 ¸ 0 으로 모두 등호성립. 모두 똑같지 않다면, 윤환식이므로, 최 인 항과 그렇지 않

은 항이 이웃한 경우가 존재. 그런데, xk가 최 라면 5xk · 2xk¡1 + 3xk+1 로 xk¡1 = xk+1 = xk 여

야 함. 따라서, 모든 항이 똑같을 때뿐. }

Page 117: 실전수학올림피아드 1400제 해답

2.2 대수 고급문제 117

118. 0이 아닌 실수들에 해 정의되는 연산 ¤가 0 아닌 임의의 실수 a, b, c에 해 다음을 만족한다: (1)a ¤ a = 1, (2) a ¤ (b ¤ c) = (a ¤ b)c (a ¤ b 와 c의 곱셈). 방정식 x ¤ 36 = 216 을 풀어라. (스웨덴 1993-4)

풀이 216 ¢36 = (x¤ 36)36 = x ¤ (36¤ 36) = x¤ 1 = x ¤ (x ¤x) = (x ¤x)x = x. 일반적으로 a¤ b = a=b

임을 증명할 수 있음. }

119. 주어진 상수 a, b에 해 함수 f(x) =1

ax+ b를 생각하자. f(x1) = x2, f(x2) = x3, f(x3) = x1 인 서로

다른 세 실수 x1, x2, x3이 존재할 때, a와 b는 어떤 수인가? (스웨덴 1993-6)

풀이 x = f(f(f(x))) =a+ b2 + abx

b3 + 2ab+ (a2 + ab2)x를 풀면 (a2 + ab2)x2 + (b3 + ab)x ¡ (a + b2) = 0.

이 이차방정식이 서로 다른 세 실근을 가져야 하므로, 이 식은 항등식, 즉 계수가 모두 0이어야 한다. 고로, a+ b2 = 0. a = ¡b2 일 때는 f(f(f(x))) ´ x 로 항상 성립. }

120. 모든 실수 x에 해 p(x¡ 1) + p(x+ 1) = 2p(x) + 2 가 되는 실계수 다항식 p(x)를 모두 구하여라.(스웨덴 2003-4)

풀이 p가 1차 이하, 즉 p(x) = ax+ b 라 하면 2ax+ 2b = 2ax+ 2b+ 2 로 성립하지 않는다. p가 2차

이상이라면 p(x) = axn + bxn¡1 + cxn¡2 + ¢ ¢ ¢ 라 두자.

a((x¡1)n+(x+1)n)+b((x¡1)n¡1+(x+1)n¡1)+c((x¡1)n¡2+(x+1)n¡2)+¢ ¢ ¢ = 2axn+2bxn¡1+2cxn¡2+¢ ¢ ¢

여기서 좌변의 n¡ 2차항의 계수는 n(n¡ 1)a+ 2c 6= 2c 로, n > 2 일 때는 우변의 n¡ 2차항의 계수와같을 수 없다. 즉, n = 2 이고, 2a + 2c = 2c+ 2 로 a = 1 이다. p(x) = x2 + bx + c 일 때는 식이 항상잘 성립한다. }

121. x+ y = 2 인 음 아닌 실수 x, y에 해 다음을 증명하여라. (아일랜드 2000-6)

x2y2(x2 + y2) · 2

증명 동차식의 방법으로 하면 준식은 다음과 차례로 동치: 32x2y2(x2 + y2) · (x+ y)6, x6 + 6x5y¡17x4y2 + 20x3y3 ¡ 17x2y4 + 6xy5 + y6 ¸ 0, (x¡ y)2((x2 ¡ y2)2 + 8xy(x2 + y2)) ¸ 0. 그리고 이것은 성립. ¤

별증 xy · (x+y2)2 = 1 이므로 x2y2(x2 + y2) · xy(x2 + y2) · 1

8(x + y)4 = 2. 마지막 부등식은

(x¡ y)4 ¸ 0 과 (x+ y)4 ¸ 8xy(x2 + y2) 이 동치임에서. ¤

122. 다음 연립방정식이 모두가 0은 아닌 음 아닌 실수해 x1; x2; x3; x4; x5를 갖는다: x1+x3 = 2tx2, x2+x4 =2tx3, x3 + x5 = 2tx4. t의 최소값은 얼마인가? (스웨덴 1974-5)

풀이 만일 x3 = 0 이면 x2 + x4 = 0 이고 그럼 xi들이 모두 0임을 금방 알 수 있음. 즉, x3 6= 0. 연립

하여 4t2 ¡ 2 = x1+x5x3

의 식을 얻을 수 있고, 이것은 0 이상이므로 t2 ¸ 12. t도 0 이상이므로 t ¸ 1p

2.

실제 x1 = x5 = 0, x2 = x4 = 1, x3 =p2 이면 등호 성립. }

123. a; b ¸ 0 에 해 다음을 증명하고 등호조건은 a = b 임을 보여라. (아일랜드 2004-5)

p2

µqa(a+ b)3 + b

pa2 + b2

¶· 3(a2 + b2)

증명 (a+b)2 · 2(a2+b2)임을 이용하면 LHS · p2pa2 + b2(p2a(a+ b)+b) ·p2(a2 + b2)( 3a+b

2+

b), 즉, (a+ b) · p2pa2 + b2 와 동치이므로 끝. ¤

124. f(1) = 1 이고, 모든 실수 x, y에 해 다음을 만족하는 함수 f : R! R 를 모두 구하여라.(아일랜드 2006-5)

f(xy + f(x)) = xf(y) + f(x)

풀이 x = 1 을 입하면 f(y + 1) = f(y) + 1. (x; y) = (n; n), (n+ 1; n¡ 1) 을 입한 두 식을 비교

하면 됨. }

Page 118: 실전수학올림피아드 1400제 해답

118 대수

별해 (x; y) = (1; 0) 을 입하면 f(0) = 0. y = 0 을 입하면 f(f(x)) = f(x) 임을 이용하여 다음과

같이 할 수도 있다. x = 1y 를 입하면 1 + f( 1

y) = 1 + f(f( 1

y)) = f(1 + f( 1

y)) = 1

yf(y) + f( 1

y). 따라

서, 1 = 1yf(y), 즉 f(y) = y. }

125. 양의 실수 a, b, c에 해 다음을 증명하고, 각 부등식의 등호조건을 구하여라.

a+ b+ c

3·s

a2 + b2 + c2

abc+ bc

a+ ca

b

3(아일랜드 2007-7)

풀이 좌변은 체비셰프나 코시나 전개. 우변은 abc= Z 등으로 치환 후 전개하면 X2 + Y 2 + Z2 ¸

XY + Y Z + ZX 가 됨. }

126. a, b, c는 정수이고 a > 0 이다. 방정식 ax2 + bx+ c = 0 이 구간 (0; 1)에서 서로 다른 두 해를 갖는다고하자. a ¸ 5 임을 보이고, a = 5 일 때 그와 같은 방정식의 예를 하나 들어라. (스웨덴 1979-5)

증명 f(x) = ax2 ¡ dx + c 라 할 때 f(0) > 0, f(1) > 0, D > 0, 0 < 축 < 1 등에서 잘 정리하면 다

음 두 부등식을 얻음: d2 > 4ac 이고 0 < 2c < d < a + c. 정수이므로 a와 c는 2 이상의 차이가 있어야 함. 만일 a · 4 이면 c · 2. c = 2 이면 a = 4 이고 d = 5 인데 d2 > 4ac 성립하지 않음. 즉 c = 1.d2 > 4ac ¸ 12 이고 d < a + c · 5 이므로 d = 4. 그럼 a = 4 인데 역시 d2 > 4ac 성립하지 않으므로모순. a = 5 일 때 실례는 5x2 ¡ 5x+ 1 = 0. ¤

127. 임의의 실수 x, y에 하여 다음을 만족하는 함수 f : R! R 를 모두 구하여라.

f(x+ y)¡ f(x¡ y) = f(x)f(y) (폴란드 1992/1993 1차-4)

풀이 x = y = 0 입하면 f(0) = 0. x = 0 만 입하면 f(y) = f(¡y). y = x;¡x 입하여 비교하면

f(2x) = f(x)2 = ¡f(2x). 즉 f ´ 0. }

128. 0이 아닌 실수 a ¸ b ¸ c 에 해 다음 부등식을 증명하고, 등호가 성립할 조건을 구하여라.(북유럽 1988-2)

a3 ¡ c3

3¸ abc

µa¡ b

c+

b¡ c

a

증명 3배하고 다 왼쪽으로 넘기면 (a¡ b)3 + (b¡ c)3 ¸ 0 으로 정리됨. ¤

별증 a3¡b33

+ b3¡c33

= (a¡ b)a2+ab+b2

3+ (b¡ c) b

2+bc+c2

3¸ (a¡ b)ab+ (b¡ c)bc. ¤

129. 모든 실수 x에 해 다음 조건을 만족하는 함수 f : R! R 을 모두 구하여라. (폴란드 1993 3차-5)

f(¡x) = ¡f(x);f(x+ 1) = f(x) + 1;

f

µ1

x

¶=

f(x)

x2(x 6= 0)

풀이 마지막 식에서 f(x) = x2f( 1x) 임을 참조하여, x! 1

x! 1

x+ 1! x

x+1! ¡1

x+1! ¡(x+ 1)!

x+ 1! x 로 순환시키며 식을 연속적으로 변화시켜보면 그냥 f(x) = x 가 정리됨. }

130. a, b, x, y는 a+ b+ x+ y < 2 인 양의 실수들이다. a+ b2 = x+ y2 이고 a2 + b = x2 + y 이면, a = x 이고 b = y 임을 보여라. (스웨덴 1995-3)

증명 a ¡ x = (y ¡ b)(y + b) 이고 (a ¡ x)(a + x) = y ¡ b. 즉 a ¡ x = (a ¡ x)(a + x)(y + b). 만일

a 6= x 라면 (a+ x)(y+ b) = 1. 그럼 2 > (a+ x) + (y+ b) ¸ 2p(a+ x)(y + b) = 2 로 모순. 즉, a = x

이고 b = y. ¤

Page 119: 실전수학올림피아드 1400제 해답

2.2 대수 고급문제 119

131. 임의의 양의 실수 A, B, C에 해 다음 부등식을 증명하여라.

max(A2 ¡ B;B2 ¡ C;C2 ¡ A) ¸ max(A2 ¡A;B2 ¡B;C2 ¡ C)

단, max(x; y; z)는 x, y, z 중 가장 큰 수를 나타낸다. (레닌그라드 1991-y9-1)

증명 좌변을 X, 우변을 Y 라 하고, 귀류법으로 X < Y 라 하자. WLOG, Y = A2 ¡ A 라 할 수 있다.

A2 ¡ B · X < Y = A2 ¡ A 이므로 A < B. 그럼 또 A2 ¡ C < B2 ¡ C · X < Y = A2 ¡ A 이므로A < C. 그럼 X ¸ C2 ¡ A > A2 ¡A = Y 가 되어 모순. ¤

132. a, b, c가 삼각형의 세 변의 길이일 때, 다음 부등식이 성립함을 보여라.

1

a+1

b+1

c· 1

a+ b¡ c+

1

b+ c¡ a+

1

c+ a¡ b(폴란드 1993/1994 1차-3)

증명 Ravi 치환하면 금방. 1y+z

+ ¢ ¢ ¢ · 12pyz+ ¢ ¢ ¢ · 1

2x+ ¢ ¢ ¢ . ¤

133. 양의 실수 a, b, c, d에 해 다음 부등식을 증명하여라. (레닌그라드 1988-y9-E3)

1

a+1

b+4

c+16

d¸ 64

a+ b+ c+ d

증명 보조정리: 1a +1b¸ 4

a+b 를 반복하여 적용. ¤

134. 0과 1 사이의 세 실수 X, Y , Z에 해 다음을 증명하여라. (레닌그라드 1989-y9-E5)

2(X3 + Y 3 + Z3)¡ (X2Y + Y 2Z + Z2X) · 3

증명 X3 ¡X2Y + Y 3 · 1 임을 X ¸ Y , X < Y 일 때로 나누어 각각 확인된다. 이런 식 셋을 합하면

준식. ¤

135. x+ y + z = 12 인 x; y; z ¸ 0 에 해 다음을 증명하여라. (레닌그라드 1988-y8-6a)

1¡ x

1 + x¢ 1¡ y

1 + y¢ 1¡ z

1 + z¸ 1

3

증명 보조정리: a; b ¸ 0 일 때 1¡a1+a

¢ 1¡b1+b

= 1¡a¡b+ab1+a+b+ab

¸ 1¡(a+b)1+(a+b) 임을 이용하면 됨. ¤

136. 주어진 실수 a에 하여, 수열 fang을 다음과 같이 정의하자: a1 = a,

an+1 =

8<:1

2

µan ¡ 1

an

¶(an 6= 0 일 때)

0 (an = 0 일 때)

fang에 양수가 아닌 항이 무한히 많이 나타남을 증명하여라. (폴란드 1978 3차-5)

증명 양이 아닌 항을 하나만 찾을 수 있어도 충분. 귀류법으로 모두 양이라고 가정. 그럼 각 항의 앞항

을 생각하면 모두 1보다 커야함. 그럼 또 모두 1 +p2보다 커야함. 귀납적으로, 모든 항이 b보다 크다면

각 항의 앞항을 고려했을 때 모든 항은 b+pb2 + 1 > 2b 보다 커야함. 그럼 모든 항이 무한히 커야한다

는 것이므로 모순. ¤

137. x, y, z는 1보다 작은 양의 실수들이다. 다음 셋 중 적어도 하나는1

4를 넘지 않음을 보여라.

(헝가리 1961-2)

(1¡ x)y; (1¡ y)z; (1¡ z)x

증명 귀류법. 1 < 2p(1¡ x)y < (1 ¡ x) + y (GM-AM) 들을 다 더하면 3 < 3 의 모순. 항이 6개짜

리인 AM-GM으로 봐도 되고. ¤

Page 120: 실전수학올림피아드 1400제 해답

120 대수

138. 모든 x에 해 다음을 만족하는 함수 f : R! R 을 모두 구하여라.

xf(x) = [x]f(fxg) + fxgf([x])단, [x]와 fxg는 각각 x의 정수부와 소수부를 나타낸다. (몰도바 1999 최종-y10-5)

풀이 x에 정수를 입하면 f(n) = f(0). x에 0 < r < 1 을 입하면 f(r) = f(0). 마지막으로 n+ r

을 입하면 f(n+ r) = f(0). 상수함수. }

139. 모든 실수 x에 해 다음을 만족하는 정수 계수의 다항식 P를 모두 구하여라.

P (P (P (P (P (x))))) = x28 ¢ P (P (x)) (오스트리아-폴란드 2005-2)

풀이 우선 상수함수일 때는 없음. P (x) = axn+ bxn¡1+ ¢ ¢ ¢ 로 두면 좌우변의 최고차항의 차수를 비

교할 때 n5 = 28+ n2, 즉 n = 2. P (x) = ax2 + bx+ c 로 둘 때 최고차항의 계수를 비교하면 a31 = a2,

즉 a = 1. 다시 제2최고차항(31차항)을 비교하면 16b = 2b 에서 b = 0. 다시 제3최고차항(30차항)을 비교하면 16c = 2c 에서 c = 0. P (x) = x2뿐. }

140. 자연수 n ¸ 2 에 하여 다음 연립방정식을 풀어라. (폴란드 1994/1995 1차-2)

x1jx1j = x2jx2j+ (x1 ¡ 1)jx1 ¡ 1j;x2jx2j = x3jx3j+ (x2 ¡ 1)jx2 ¡ 1j;

.

..

xnjxnj = x1jx1j+ (xn ¡ 1)jxn ¡ 1j

풀이 xi가 어떤 값이어도(xi < 0, 0 · xi < 1, 1 · xi 등으로 경우를 나눠 따져보면 됨)항상 xi+1 > 0.

또한, xi들은 모두 한꺼번에 1 이상이거나 모두 한꺼번에 1 미만임. 각각의 경우 입하여 식을 모두 변변 합하면 제곱들의 합만 남고 그럼 모두 1일 때뿐. }

141. 다음을 만족하는 실수해 (x; y; z)를 모두 구하여라. (남아공 2001-2)

x(1¡ y2)(1¡ z2) + y(1¡ z2)(1¡ x2) + z(1¡ x2)(1¡ y2) = 4xyz = 4(x+ y + z)

풀이 xyz = x+ y + z 임에서 첫 번째 식은 전개해보면 그냥 당연한 식이 되어 버린다. 그래서 xyz =

x+ y + z 만 풀면 되고, 그럼 임의의 x, y에 해 (x; y; z) = (x; y; x+yxy¡1 ) 이 답. }

142. 수열 fxng이 다음과 같이 주어진다.

x1 =1

2; xn =

2n¡ 32n

xn¡1 (n ¸ 2)

임의의 n 2 N 에 하여 x1 + x2 + ¢ ¢ ¢+ xn < 1 이 성립함을 보여라. (폴란드 1994/1995 1차-12)

증명 귀납법으로 xn · 1n(n+1) 임을 증명할 수 있고, 그것으로 telescoping. ¤

143. a; b; c; d는 양의 무리수들이고 a + b = 1 이다. c + d = 1 일 필요충분조건은 모든 자연수 n에 하여[na] + [nb] = [nc] + [nd] 가 성립하는 것임을 증명하여라. (폴란드 1995 2차-3)

증명 정방향의 명제는 [na] + [nb] = n¡ 1 = [nc] + [nd] 로 쉬움. 역명제는 c+ d0 = 1 로 두면 (정명

제에 의해) 모든 자연수 n에 해 [nd] = [nd0]. 그럼 d = d0. ¤

144. 다음 연립방정식의 양의 실수해는 x = y = z = 1 뿐임을 증명하여라.

x+ y2 + z3 = 3

y + z2 + x3 = 3

z + x2 + y3 = 3 (스웨덴 1986-4)

증명 x; y ¸ 1 이면 x+ y2 · x2 + y3 이므로 z3 ¸ z, 즉 z ¸ 1. 그럼 x = y = z = 1 뿐. x; y · 1 일

때도 비슷하게 x+ y2 ¸ x2 + y3 에서 z · 1 로 x = y = z = 1 뿐. ¤

Page 121: 실전수학올림피아드 1400제 해답

2.2 대수 고급문제 121

145. 두 실계수 삼차방정식 x3 +mx¡ n = 0 과 nx3 ¡ 2m2x2 ¡ 5mnx¡ 2m3 ¡ n2 = 0 이 딱 하나의 공통근을 가질 때, 첫번째 식이 중근을 가짐을 보이고, 이 두 방정식의 근을 n에 한 식으로 모두 구하여라.

(스페인 1995-5)

풀이 삼차방정식이라 했으므로 우선 n 6= 0. m = 0 이면 두 식이 x3 = n 으로 일치하여 공통근

이 딱 하나가 아니므로 m 6= 0. 공통근을 a라 하고, a3 = n ¡ ma 를 둘째식에 입하여 정리하면

a2 + 3nm

a + m = 0. 이걸 다시 첫식에 입하여 정리하면 a2 = ¡m3 이고 a = ¡ 2m2

9n. 둘을 연

립하여 풀면 4m3 = ¡27n2, 즉 m = ¡3 3q

n2

4 이고 공통근은 a = ¡ 3q

n2, m = ¡3a2. 그럼 첫식은

(x¡a)(x2+ax+ na) = (x¡a)2(x+2a) = 0, 둘째식은 n으로 나눠주고나면 x3+9ax2+15a2x¡25a3 =

(x¡ a)(x+ 5a)2 = 0. }

146. a, b는 주어진 실수이다. 방정식 ax+ b[x] = ay + b[y] 가 x 6= y 인 실수해를 갖지 않을 a, b의 조건을 구하여라. (브라질 1985-5)

풀이 a = 0 이면 해가 무한히 많으므로 a 6= 0. WLOG x < y. ba([x] ¡ [y]) = y ¡ x 6= 0 에서

[y] = [x] + n 이라 할 때 n ¸ 1 임을 알 수 있다. n¡ 1 < y ¡ x < n+ 1 임에서 1¡ 1n< y¡x

n< 1 + 1

n,

즉 0 < y¡xn

< 2 이므로 0 < ba< 2 임을 알 수 있고, 실제 이 범위에서 n = 1 로 하면 b

a= x¡ y 는 항

상 해를 갖는다. ¢ ¢ ¢ 답 a 6= 0 이고, ba· 0 또는 b

a¸ 2. }

147. f(x) =ax+ b

cx+ d로 주어진 함수 f가 f(0) 6= 0, f(f(0)) 6= 0 을 만족한다. F (x) = f(¢ ¢ ¢ f(x) ¢ ¢ ¢ ) 이라 할

때(f를 n번 합성), F (0) = 0 이면 이 함수가 정의되는 모든 x에 해 F (x) = x 임을 증명하여라.(브라질 1990-5)

증명 f를 k번 합성한 것을 fk(x) = akx+bkckx+dk

라 하면, fk+1(x) = f(fk(x)) = fk(f(x)) 임에서 두 점

화식이 나오고, 그것을 비교하면 bck = cbk 이고 (a ¡ d)bk = b(ak ¡ dk). fn(0) = 0 이라면 bn = 0 인

데, 그럼 bcn = 0 이고, b 6= 0 이므로 cn = 0. 또 b(an ¡ dn) = 0 에서 an = dn. 그럼 fn(x) ´ x. ¤

148. 임의의 실수 a, b, c, d에 해 다음을 증명하여라. (레닌그라드 1987-y8-3)

(1 + ab)2 + (1 + cd)2 + (ac)2 + (bd)2 ¸ 1

증명 LHS = 1 + (1 + ab+ cd)2 + (ac¡ bd)2 ¸ 1 ¤

149. f(0) = 0 이고, 모든 음 아닌 실수 x에 해 f(2x+ 1) = 3f(x) + 5 를 만족하는, 실수값을 갖는 함수 f를하나만 찾아라. (브라질 1993-5)

풀이 구간 In = [2n ¡ 1; 2n+1 ¡ 1) 들은 음 아닌 실수들의 집합을 분할하고, x 7! 2x + 1 에 의해

In ! In+1 로 응된다. 따라서, I0 = [0; 1) 에서만 아무렇게나 정의하면 나머지 구간에서는 귀납적으로 역시 잘 정의된다. }

150. x+ y + z + t = 0 과 x2 + y2 + z2 + t2 = 1 을 만족하는 임의의 실수 x, y, z, t에 해 다음 부등식을 증명하여라. (오스트리아-폴란드 1996-4)

¡1 · xy + yz + zt+ tx · 0

증명 준식은 (x + z)(y + t) = ¡(x + z)2 이므로 오른쪽 부등식은 바로 나옴. 왼쪽 부등식은 j(x +z)(y + t)j · 1

2((x+ z)2 + (y + t)2) · (x2 + z2) + (y2 + t2) = 1 로 증명됨. ¤

Page 122: 실전수학올림피아드 1400제 해답

122 대수

Page 123: 실전수학올림피아드 1400제 해답

제 3 장

기하

3.1 기하 중급문제

1. 한 원 위에 세 점 A, B, C가 있다. 호 BC, CA, AB의 중점을 각각 M , N , K라 하자. 그럼 세 현 AM ,BN , CK가 한 점에서 만남을 보여라.

증명 만나는 한 점은 삼각형 ABC의 내심이다. ¤

2. ¤ABCD의 점 D에서 AB, BC에 내린 수선의 발을 각각 P , Q라 하고, B에서 AD, CD에 내린 수선의발을 각각 R, S라 하자. \PSR = \SPQ 이면 PR = QS 임을 보여라. (아일랜드 2003-7)

증명 ( 원외국어고 3학년 강성환)

\BPD = \BRD = \BQD = \BSD = 90± 이므로 지름 BD에 한 원주각이 되고, 다각형BQSDRP는 원에 내접한다. 그러므로 원주각 \PSR = \SPQ 이면 PR = QS 이다. ¤

3. 다음 세 가지가 주어졌을 때 사각형 ABCD를 구하는 방법을 설명하여라.

(1) 네 변의 길이

(2) 변 AB와 CD가 평행하다.

(3) 변 BC와 DA가 만나지 않는다. (캐나다 1972-4)

풀이 일반성을 잃지 않고 AB ¸ CD 라 하자. 선분 AB 위에 BE = CD 가 되는 점 E를 잡는다. A를

중심으로 반지름 AD인 원을 그리고, E를 중심으로 반지름 BC인 원을 그린다. 두 원의 교점 하나를

A

P R

B Q C

S

D

Page 124: 실전수학올림피아드 1400제 해답

124 기하

D라 한다.

조건을 만족하는 사각형이 존재한다면, D에서 BC와 평행한 직선을 그려 AB와 만나는 점 E가 위와 같은 점이므로, 두 원의 교점 D가 반드시 있다. 다만, AB = CD 여서 A와 E가 일치할 때는 두 원도 일치하므로 교점이 무한히 많고, 그 중 아무 것이나 하나 택하면 된다. 이제 평행사변형 BCDE가 되도록C를 잡으면 된다. }

4. 삼각형 ABC가 주어져있다. BE = EF = FC 가 되도록 변 AB, BC 위에 각각 점 E, F를 작도하여라.

풀이 \EFB = 2\ECB 임에서 착안한다. \ABC의 이등분선을 그려 BC의 수직이등분선과 만나는

점을 D라 하자. CD의 연장선이 AB와 만나는 점을 E라 한다. 그리고, E를 중심으로 하고 B를 지나는원이 BC와 만나는 점을 F라 하면 된다. }

5. 삼각형 ABC에서 세 내각 중에 \A가 가장 크면 세 변 중에 가장 긴 것은 BC임을 증명하여라.

증명 삼각형의 임의의 두 내각 \B, \C와 그에 응하는 두 변 CA, AB에 해, 둘 중 큰 각에 응

하는 변의 길이가 항상 더 큼을 증명하자. 남은 한 변 BC에서 수직이등분선 L을 올려, 남은 한 꼭지점A가 L에서 왼쪽에 있는지 오른쪽에 있는지에 따라 살피면 각의 크기와 변의 길이 사이의 상관관계를 관찰할 수 있다. 좀더 엄밀히 한다면 A에서 BC에 수선을 내려 피타고라스의 정리로 논하면 된다. ¤

주 고등부로 간다면 cosine 제2법칙으로 논할 수도 있다.

6. ¤ABCD는 한 변의 길이가 1인 정사각형이다. 변 AB, BC, CD, DA 위에 각각 점 A0, B0, C0, D0을 잡는데, AA0 = BB0 = CC0 = DD0 = 1=n 이 되도록 한다. 두 사각형 AA0CC0과 BB0DD0이 겹치는 영역은 정사각형이 되는데 넓이가 1=1985이라고 한다. n을 구하여라. (AIME 1985-4)

풀이 %EEE

AP = √(1-^2), so by similar triangles, x/1 = √(1-x^2)/(1 - 1/n).

Hence x = 1/√(1 + (1 - 1/n)^2), so the side of the small square is

x/n = 1/√(n^2 + (n-1)^2). It is easy to check that n^2 + (n-1)^2 = 1985

for n = 32.

[그림]

¢ ¢ ¢ 답 32 }

7. K, L, M , N을, 어떤 마름모의 네 변 위에 (바깥쪽으로) 세운 정사각형들의 중심이라고 하자. 다각형KLMN이 정사각형임을 증명하여라. (헝가리 1906-2)

힌트 (1) 이웃한 두 정사각형의 중심과 그 사이에 위치한 마름모의 한 꼭지점을 연결한 삼각형이 넷

있는데, 이들이 서로 합동이고 90± 회전한 위치에 있음을 보이면 된다.(2) 똑같은 마름모와 정사각형 타일들을 무한히 복제하여 평면을 가득 깔아나가는 것에서 칭성을 발견한다. }

8. AB < AC 인 삼각형 ABC가 외접원 S를 갖는다. A에서 BC에 내린 수선이 S와 P에서 다시 만난다.X가 선분 AC 위의 점일 때, BX가 S와 Q에서 다시 만난다. BX = CX 이면 PQ가 S의 지름이고 그 역도 성립함을 보여라. (영국 2002/2003 1차-2)

Page 125: 실전수학올림피아드 1400제 해답

3.1 기하 중급문제 125

증명

(1) ()의 증명) BX = CX 라 가정하자. XBC가 이등변삼각형이고 XAQ도 그와 닮았으므로 AQ는BC와 평행하고, 따라서 AP와 수직이다. 따라서, PQ는 원 S의 지름이다.

(2) ((의 증명) PQ가 원 S의 지름이라 가정하자. \PAQ = 90± 이므로 AQ와 BC가 평행. 따라서,XBC는 이등변삼각형이고 XB = XC 이다.

(1), (2)에 의해 두 조건이 동치임이 증명되었다. ¤

9. 두 원 C1, C2가 점 S에서 만난다. S에서의 C1의 접선이 C2와 A( 6= S) 에서 만나고, S에서의 C2의 접선이 C1과 B( 6= S) 에서 만난다. A, B, S를 지나는 원을 C3이라 하자. S에서의 C3의 접선이 C1, C2와 각각 P ( 6= S), Q( 6= S) 에서 만난다. PS = QS 임을 증명하여라. (플란더즈 2003-2)

증명

선분 AS, SB, PQ가 각각 원 C1, C2, C3의 접선이 되므로, 원과 접선의 성질에 의해

\QAS = \SAB = \PSB

\AQS = \ASB = \SPB

\QSA = \SBA = \PBS

가 성립한다. 이것에 의해 4QSA와 4SBA와 4PBS는 닮음임을 알 수 있다(AA닮음). 닮음인 삼각형의 각 변의 비는 같으므로 4QSA와 4SBA에서

AB : AS = SB : SQ |{ ①

4SBA 와 4PBS에서

AB : AS = SB : SP |{ ②

①, ②에서 AB : AS = SB : SQ = SB : SP 가 성립하고 SB : SQ = SB : SP 에서 SB는 같으므로SP = SQ 이다. ¤

10. 삼각형 ABC에서 BC의 중점을 D라 하자. 직선 AD 위에 \CEA가 직각이 되는 점 E를 잡았더니\ACE = \B 가 되었다. \A가 직각이 아니라면 AB = AC 가 됨을 보여라. (호주 2003-6)

Page 126: 실전수학올림피아드 1400제 해답

126 기하

증명 AB 6= AC 인 4ABC가 존재한다고 가정하자.

¡!CE 위에 CE = C0E 인 점 C0( 6= B)을 잡자. 4ACC0는 이등변삼각형이므로

\ACE = \AC0E (1)

4CED » 4CC0B (SAS) 이므로\BC0C = 90± (2)

문제의 조건 \ACE = \ABC 와 (1)에 의해,

\AC0E = \AC0C = \ABC

따라서, ¤AC0BC는 원에 내접. (2)에 의해

\BAC = \BC0C = 90±

이것은 조건에 모순. 따라서, AB = AC. ¤

11. 삼각형 ABC 내부의 점 O를 지나는 세 직선 DE, FK, MN을 각각 AB, AC, BC에 평행하게 그렸다.

단, F와 M은 AB 위에, E와 K는 BC 위에, D와 N은 AC 위에 있는 점이다.

AF

AB+

BE

BC+

CN

CA

의 값을 구하여라. (플란더즈 2003-J2)

풀이

AF

AB=4AFC

4ABC=4AOC

4ABC

BE

BC=4ABE

4ABC=4ABO

4ABC

CN

CA=4CBN

4ABC=4BOC

4ABC

세 식을 더하면

AF

AB+

BE

BC+

CN

CA=4AOC +4ABO +4BOC

4ABC=4ABC

4ABC= 1

}

BD

EC

A

C

Page 127: 실전수학올림피아드 1400제 해답

3.1 기하 중급문제 127

12. 다음 그림과 같이 세 변의 길이가 각각 5, 6, 7인 삼각형 ABC의 외부에 세 개의 정사각형을 그리면 세 개의 빗금친 삼각형을 얻는다. 그림에서 세 선분 AP , BQ, CR은 각각 빗금친 삼각형의 중선이다. 이 세 중선의 길이의 합은 얼마인가? (한국 2004 1차-J10)

풀이 Q가 지나는 선분의 양끝점을 D, G라 하자. 4BQG를 Q에 해 칭시킨 것을 4EQD라 하자.

그럼 도형 DEQB는 삼각형이 되고, DB = AB, DE(= BG) = BC,

\EDB = \EDQ+ \BDQ = \BGQ+ \BDQ

= 180± ¡ \DBG = \B

이므로 4BDE ´ 4ABC (SAS합동) 가 된다. 따라서,

BQ =1

2BE =

1

2AC

마찬가지로, AP =1

2BC, CR =

1

2AB 이고, 세 중선의 합은

AP +BQ+ CR =1

2(BC + CA+AB) =

1

2(7 + 6 + 5) = 9

이다. }

별해 4DBG를, DB가 AB에 일치하도록, B를 중심으로 90± 회전하자. 그럼 D0, B, C는 일직선을

이루고, BQ0은 중점연결선으로 AC의 길이의 12임을 알 수 있다. }

13. QB는 PA를 지름으로 하는 원의 PA와 평행한 현이다. 두 직선 PB와 QA가 점 R에서 만난다. 이 원의중심을 O라 하고, PORS가 평행사변형이 되도록 점 S를 잡자. 그럼 SP = SQ 가 됨을 보여라.

(아일랜드 2003-2)

증명 (부산과학고 1학년 이태희)

QB k PA 이므로 RO에 해 좌우 칭인 그림이 되고(\QAP = \QBP = \BPA), PORS는 직사각형이다. \RSP = \RQP = \R 이므로 QRPS는 한 원에 내접하는 사각형이고,

\SPQ = \SRQ = \PAQ = \APR = \SRP = \SQP

로 4SPQ는 SP = SQ 인 이등변삼각형이다. ¤

A

R

O P

B Q

S

Page 128: 실전수학올림피아드 1400제 해답

128 기하

14. AB = 12 이다. A를 중심으로 하고 반지름 8인 원과 B를 중심으로 하고 반지름 6인 원을 그렸을 때 만나는 교점 중 하나를 P라 하자. P를 지나는 직선이 두 원과 각각 Q, R에서 다시 만나는데, QP = PR 이된다고 한다. QP 2을 구하여라. (AIME 1983-14)

풀이 %EEE

Put QP = 2a, take perp dist A, B from QP be b, c. Take line

parallel to QP through B meeting perp bisector of QP at D.

Considering triangle ADB gives 4a^2 + (b-c)^2 = 144. Also

b^2 = 64 - a^2, c^2 = 36 - a^2. Hence 4a^2 = 130.

¢ ¢ ¢ 답 130 }

15. P를 평행사변형 ABCD 내부의 임의의 점이라 하고, 세 점 A, B, C를 지나는 원의 반지름을 R이라 하자. P로부터 가장 가까운 꼭지점에 이르는 거리는 R보다 크지 않음을 보여라. (헝가리 1907-2)

증명 삼각형 ABC와 CDA는 합동이므로 일반성을 잃지 않고 P가 삼각형 ABC 안에 있다고 할 수 있

다. 삼각형 ABC의 외심에서 세 꼭지점에 선분을 긋고 세 변에도 수선(수직이등분선)을 내린다. 그럼 삼각형 ABC는 여섯 개의 직각삼각형으로 영역이 분할되고, 이들 직각삼각형은 빗변의 길이가 모두 R이다. 따라서 P가 어느 직각삼각형에 있든지 그쪽 꼭지점에 이르는 거리가 R 이하가 된다. ¤

16. AB = BC 인 예각 이등변삼각형 ABC의 외접원의 반지름의 길이가 3이다. 점 A와 외접원의 중심 O를잇는 직선과 변 BC와의 교점 P에 하여 AP = 21

4 일 때, 선분 AB의 길이의 제곱은 얼마인가?

(한국 2004 1차-S6)

풀이 AP의 연장선이 원과 만나는 점을 G라 하자.

그럼

BP ¢ PC = AP ¢ PG =21

4¢ 34=63

16

한편, 각의 이등분선 정리에 의해

AB : BP = AO : OP = 3 :21

4¡ 3 = 4 : 3

따라서,

BP ¢ PC =3

4AB ¢ 1

4AB =

3

16AB2

이 되고, 그럼 AB2 = 21. }

17. 삼각형의 두 꼭지점 중 하나씩을 지나는 두 직선이 이 삼각형을 세 개의 작은 삼각형과 하나의 사각형으로 나눈다. 이 네 작은 도형의 넓이가 모두 같을 수 있는가? (영국 1989 주니어)

18. 원주 위에 세 점이 A, B, C 순으로 있을 때 호 BC 위에 AD = DC 가 되게 D를 정하고, D에서 현 BC에수선을 내려 그 발을 E라고 하면 AB +BE = CE 임을 증명하여라.

주 (1) BD에 해 A를 칭이동

(2) E에 한 B의 칭점을 B0으로 할 때 4ABD ´ 4CB0D

B

A

O

C

P G

Page 129: 실전수학올림피아드 1400제 해답

3.1 기하 중급문제 129

19. 두 개의 5£ 1 크기의 직사각형을 아래 그림과 같이 2쌍의 꼭지점이 일치하도록 겹쳐놓았다.

(a) 겹쳐진 부분의 넓이를 구하여라.

(b) 두 직사각형의 교점 중 꼭지점이 아닌 두 점 A와 B 사이의 거리를 구하여라. (플란더즈 2004-J1)

풀이 (a) KA = x, AN = y 라 하자. 그럼

x+ y = 5

칭성에 의해 KA = AR = PB = BQ = x, AN = NB = BL = LA = y 이다. 직각삼각형 ARN에서y2 = x2 + 1, 즉 (y + x)(y ¡ x) = 1 이고

y ¡ x = 0:2

이다. 연립하여 풀면 y = 2:6, x = 2:4. 겹쳐진 마름모 영역 ANBL의 넓이는 AN을 밑변으로 볼 때 높이가 1이므로 S = AN ¢ 1 = y = 2:6.(b) 마름모의 넓이는 두 각선의 곱에 1

2을 하여 구할 수도 있으므로,

2:6 = S =1

2LN ¢ AB =

p26

2AB

따라서, AB =110¢26¢2p26

=p265. }

20. 세 점 A, B, C가 주어져 있을 때, A를 중심으로 하고 두 이웃한 변(또는 그 연장선)이 각각 B와 C를 지나는 정사각형을 작도하여라.

21. 한 원의 둘레 위에 A, B, C, D 네 점이 차례로 있고, 호 AB, BC, CD, DA의 중점으로 역시 원주 위에있는 네 점 P , Q, R, S를 잡자. 이 때 PR과 QS가 서로 수직임을 보여라. (캐나다 1975-5)

풀이 PR과 QS의 교점을 M이라 하자.

현 XY 에 한 원주각을 \(XY )로 쓰기로 하자.

\PMS = \PQM + \QPM = \(PS) + \(QR)

=1

2(\BCD + \BAD) = 90±

따라서, PR ? QS 이다. }

Page 130: 실전수학올림피아드 1400제 해답

130 기하

22. 둘레의 길이와 넓이가 같은 삼각형이 있다. 이 삼각형의 내접원의 반지름을 구하여라.(플란더즈 예선 1990/1991)

풀이 S = 12(a+ b+ c)r 에서 S = a+ b+ c 이므로 r = 2 ¢ ¢ ¢ 답 }

23. Q를 원 위의 임의의 점이라 하고, P1P2P3 ¢ ¢ ¢P8을 이 원에 내접하는 정8각형이라 하자. Q에서 네 지름P1P5, P2P6, P3P7, P4P8에 이르는 거리들의 제곱의 합은 Q의 위치에 관계없음을 증명하여라.

(헝가리 1911-2 변형)

증명 원 위의 점 Q에서 P1P5, P3P7에 내린 두 수선은 서로 직교하며, 원의 중심 O까지 포함하여

각선의 길이가 원의 반지름 r이 되는 직사각형을 이룬다. 따라서, 피타고라스의 정리에 의해 두 수선의길이의 제곱의 합은 r2과 같다. P2P6, P4P8에 해서도 마찬가지이며, 따라서 네 거리들의 제곱의 합은2r2으로 일정하다. ¤

24. 한 변의 길이가 1인 정육면체에 두 정사면체 A와 B가 내접하고 있는데, 정사면체 A는 이 정육면체의 밑면에 있는 한 각선의 양끝점과 윗면에 있는 한 각선의 양끝점을 네 꼭지점으로 하고, 정육면체의 나머지 네 꼭지점은 정사면체 B의 꼭지점이 된다. A [B 의 부피를 구하여라. (플란더즈 1994-3)

풀이 교집합은 정팔면체인데 합집합은... 방사팔면체로군하! 1¡ 12 ¢ 13 18 = 12

}

25. 원에 정사각형이 내접하고 있고, 이 원과 정사각형 사이의 한 영역에 작은 정사각형이 내접하고 있다. 작은 정사각형은 큰 정사각형과 변으로 접한다. 작은 정사각형의 넓이가 1일 때, 큰 정사각형의 넓이를 구하여라. (AIME 2001 2차-6)

풀이 %EEE

Let the large square have side 2, the small square side x.

Then the radius is √2. The lines containing vertical sides of the

small square pass a distance x from the center. So by Pythagoras,

2 = (1 + 2x)^2 + x^2, so (5x-1)(x+1) = 0, so x = 1/5. So ratio = 1/x^2 = 25.

¢ ¢ ¢ 답 25 }

26. 지름이 AB인 원에서 \CAB = 22± 인 현 AC를 긋자. 주어진 원에 점 B에서 내접하고, 현 AC에도 접하는 작은 원을 그린다. 작은 원과 직선 AC의 접점 P에서 선분 AB에 그은 수선의 발을 M이라 할 때,\CMB의 크기는 얼마인가? (한국 2004 1차-S10)

풀이 BQ를 작은 원의 지름이라 하자. 접선과 현이 이루는 각은 원주각과 같으므로

x = \APQ = \QBP = \QPM

따라서, x = 12\APM = 1

2(90± ¡ 22±) = 34± 이다. \PMB + \PCB = 180± 로 ¤PMBC가 원에 내

접하는 사각형이므로,

\CMB = \CPB = 90± ¡ x = 56±

}

27. 두 점에서 만나는 두 원의 교점 중 하나를 P라 하자. 두 원의 공통현을 포함하지 않는 P를 지나는 직선중에서, 두 원에 의해 잘린 현의 길이가 같은 직선 l을 작도하여라.

풀이 P에 해 두 원 중 하나를 점 칭이동하여 얻은 교점과 P를 연결하면 됨. }

Page 131: 실전수학올림피아드 1400제 해답

3.1 기하 중급문제 131

28. ABCD는 볼록사각형이다. 변 AB, BC, CD, DA의 중점을 각각 A0, B0, C0, D0이라 하고, A0C0과B0D0의 교점을 P라 하자. 네 영역 PD0AA0, PA0BB0, PB0CC0, PC0DD0의 넓이를 각각 a, b, c, d라 할때, a+ c = b+ d 임을 증명하여라. (아벨콘테스트 1993 결선 1a)

증명 j4PAD0j = j4PDD0j 등을 모두 합하면 a+ c 와 b+ d 에 모든 넓이가 동등하게 기여하므로 둘

은 같다. ¤

29. 이등변삼각형 ABC의 꼭지각 \A는 20±이고, 이등변삼각형 DAC의 꼭지각 \D는 100±이다. 단, D는 직선 AC에 해 B의 반 쪽 영역에 있다. AB = BC + CD 임을 증명하여라. (플란더즈 1996-1)

증명 ABCD의 바깥으로 CDE가 정삼각형이 되는 점 E를 잡자. 그럼 BCE는 일직선을 이루고,

DEA는 이등변삼각형이다. \DAB = \DEB = 60± 이므로 칭성에 의해 BAE도 이등변삼각형이 된다. 따라서, AB = BC + CE = BC + CD. ¤

30. AB = 13, BC = 15, CA = 17 인 삼각형 ABC가 있다. D, E, F는 각각 변 AB, BC, CA 위의 점이고, AD=AB = p, BE=BC = q, CF=CA = r 이라 하면 p+ q + r = 2=3 이고 p2 + q2 + r2 = 2=5 이다.j4DEF j=j4ABCj 를 구하여라. (AIME 2001 1차-9)

풀이 %EEE

area ADC = α area ABC. area ADF = (1-γ)area ADC = α(1-γ) area ABC.

Similarly for the other two triangles, so area DEF/area ABC = 1 -

(α+β+γ) + (αβ+βγ+γα) = 1/3 + ½( (α+β+γ)^2 - (α^2+β^2+γ^2) )

= 1/3 + 1/45 = 16/45.

¢ ¢ ¢ 답 16=45 }

주 AB = 13, BC = 15, CA = 17 이라는 것은 불필요한 조건인듯.

31. A와 B를 원 k 위의 점이라 하자. 또 다른 원 l의 한 호 k0이 A와 B를 연결하고 k를 똑같은 넓이의 두 부분으로 나눈다고 한다. 호 k0이 원 k의 지름보다 더 길다는 것을 증명하여라. (헝가리 1914-1)

힌트 AB에 평행한 원 k의 지름, 혹은 A를 지나는 원 k의 지름을 그려 그것과 k0의 호 AB를 최단거

리부등식으로 비교한다. }

풀이

원호 k0가 원 k의 면적을 이등분하므로 k0는 임의의 지름의 한쪽에 위치할 수 없다. 그러므로 k의 모든지름은 원호 k0와 만나고 k의 중심 P는 원 L의 내부에 존재한다.

그러므로 k의 반지름 AO는 원 L안에 있고 k0와 지름 AC의 교차점 D는 반지름 OC상에 있다. k0는AD+BD보다 크므로 DB > DC임을 보이면 된다. 중심이 D이고 C를 지나는 원이 원 k안에 있으므로DB > DC이다. }

32. 한 직선 위에 있지 않은 세 점 P , Q, R이 주어져 있다. P , Q, R을 세 변의 중점으로 하는 삼각형을 찾아라.

A

k

B k'D

O

Page 132: 실전수학올림피아드 1400제 해답

132 기하

풀이 삼각형 PQR을 그리면 이것이 원래 삼각형의 중점삼각형. 따라서, P를 지나고 QR에 평행한 직

선, Q를 지나고 RP에 평행한 직선, R을 지나고 PQ에 평행한 직선을 그려, 세 직선이 둘씩 만나는 점을찾으면 됨. }

33. 서로 수직인 두 평면에 정사각형 ABCD와 ABEF가 각각 놓여있다. 직선 AE와 BF의 교점을 O라 하자. AB = 4 일 때 다음을 계산하여라:

(가) 점 B에서 평면 DOC와 평면 DAF의 교선에 이르는 거리

(나) 두 직선 AC와 BF 사이의 거리

단, 공간 상의 두 직선 사이의 거리는 양끝점이 각각 두 직선 위에 있고 두 직선과 동시에 수직인 선분의길이를 뜻한다. (루마니아지역예선 2005y8-2)

풀이 (가) 왼쪽 그림과 같이, BE, AF의 중점을 각각 G, H라 하자. 그럼 평면 DOC는 평면 CDHG와

같고, 이 평면과 평면 DAF와의 교선은 DH이다. B에서 DH에 내린 수선의 발을 P라 하고, 삼각형BDH의 넓이를 이용해 BP의 길이를 구해보자. BDH는 세 변의 길이가 2

p5, 2

p5, 4

p2 = BD 인 이

등변삼각형이므로, BD를 밑변으로 하는 삼각형으로 볼 때 넓이는 S = 12¢BD¢h = 2p2¢p20¡ 8 = 4p6

이 된다. 또, S = 12¢DH ¢BP 이기도 하므로 4

p6 =

p5 ¢BP , 즉 BP = 4

p305

¢ ¢ ¢ 답

(나) 오른쪽 그림과 같이, ACBI가 평행사변형이 되도록 하자. 그럼 두 직선 AC와 BF 사이의 거리는점 A에서 평면 FIB에 이르는 거리와 같다. IB의 중점을 M이라 할 때, 칭성에 의해 평면 FIB는 평면 FMA와 수직이므로, A에서 평면 FIB에 내린 수선의 발 Q는 직선 FM 위에 있다. 삼각형 AFM의

넓이로부터, FM ¢ AQ = AF ¢AM , 즉p16 + 8 ¢AQ = 4 ¢ 2p2, AQ = 4

p33

¢ ¢ ¢ 답 }

34. ABC는 삼각형이고 X, Y , Z는 각각 BC, CA, AB 위의 점으로 선분 AX, BY , CZ는 ABC 내부의 한점 D에서 만난다. 사각형 DY AZ, DZBX, DXCY 중 두 개가 원에 내접하면 나머지 하나도 또한 원에내접함을 보여라. (통신강좌 1989-C11 변형)

증명 내 각 얘기로 풀면 초간단. ¤

35. 원 K의 지름 AB의 연장선 위에 점 C를 잡고, C를 지나 원 K와 점 N에서 접하는 접선을 그리자. \ACN의이등분선이 직선 AN , BN과 만나는 점을 P , Q라 할 때, PN = QN 임을 증명하여라.

(아벨콘테스트 1994 결선 1b)

증명 \ACP = \PCN(= x) 이고 \BNC = \BAN(= y) (접현각) 이다. 이로부터 삼각형의 외각을

계산하면

\NPQ = \PAC + \PCA = x+ y

\NQP = \QNC + \QCN = x+ y

로 \NPQ = \NQP 이므로 4NPQ는 이등변삼각형이다. ¤

36. 한 변의 길이가 6p2인 정사각형 ABCD와, 이 정사각형이 놓인 평면에 평행하고 길이가 12

p2인 선분

EF가 있다. BCF와 ADE가 둘다 정삼각형이라고 할 때, 다면체 ABCDEF의 부피를 구하여라.(AIME 1983-11)

Page 133: 실전수학올림피아드 1400제 해답

3.1 기하 중급문제 133

풀이 ABEF는 등변사다리꼴이 되고, 이런 다면체를 2개 준비하여 정사각형 ABCD와 B0C0D0A0이맞붙도록 하면 한 변의 길이가 12

p2인 정사면체가 된다. ¢ ¢ ¢ 답 288 }

37. 두 점 A와 B에서 시작하여 다음의 순서로 그림처럼 원과 교점들을 작도하였다.² A를 중심으로 하고 B를 지나는 원² B를 중심으로 하고 A를 지나는 원² C를 중심으로 하고 A를 지나는 원² D를 중심으로 하고 B를 지나는 원² E를 중심으로 하고 A를 지나는 원² F를 중심으로 하고 A를 지나는 원² G를 중심으로 하고 A를 지나는 원

M이 AB의 중점임을 보여라. (플란더즈 2005-J2)

증명 AB = r 이라 하자. 정삼각형들에 의해 \ABC + \CBD + \DBE = 3 ¢ 60± 으로 평각이 되므

로 B는 AE의 중점이다.EF = EA = 2r = 2AF

이므로 4EAF는 세 변의 길이의 비가 2 : 2 : 1 인 이등변 삼각형이다. F와 G는 AE에 칭이므로 M은AE 위에 있고, AF = FM 이므로 4FAM도 이등변 삼각형이다. 특히 \A가 공통이므로

4FAM » 4EAF

이고, 따라서 AM = 12r 이다. 그러므로, M은 AB의 중점이다. ¤

38. 삼각형 ABC에서 B와 C로부터 그은 두 중선이 서로 직교한다. BC, CA, AB의 길이를 각각 a, b, c라 할때, b2 + c2 을 a에 관한 식으로 나타내어라. (플란더즈 예선 1992/1993)

풀이 파푸스의 중선 정리와 무게중심의 성질, GBC에서 피타고라스 정리를 응용하면 5a2. }

39. 한 변의 길이가 a인 정사각형의 네 꼭지점을 중심으로 반지름이 a인 네 원을 그렸을 때 사각형 중앙에 그려지는 영역의 넓이를 구하여라.

풀이 네 꼭지점을 ABCD라 하고, 각 변 AB, BC, CD, DA에 가장 가까운 교점을 각각 E, F , G,

H라 하자. 주어진 정사각형의 한 변의 길이를 a, 구하는 도형 EFGH의 넓이를 S, 돛꼴 DAEH의 넓이를 X라 하자. 삼각형 DEC가 정삼각형이 되므로, 각 ADE가 30±임을 먼저 확인해두자.

X = j부채꼴DAEj+ j정삼각형DECj ¡ j부채꼴CEHDj

12a2 +

p3

4a2 ¡ ¼

6a2

= (

p3

4¡ ¼

12)a2

따라서, S = a2 ¡ 4X = (1¡p3 + ¼3)a2. }

Page 134: 실전수학올림피아드 1400제 해답

134 기하

40. 정삼각형 ABC가 주어져있다.¡!BC의 연장선 위에 점 D,

¡!BA의 연장선 위에 점 E를 잡아 AE = BD 가

되게 한다. CE = DE 임을 증명하여라.

증명¡!BD 연장선 위에 BD = CF 가 되도록 점 F를 잡으면 AE = CF 이므로 EBF는 정삼각형. 그

리고, CD의 중점은 BF의 중점과 일치함. ¤

41. a, b, c는 어떤 직각삼각형의 세 중선의 길이이고 a ¸ b ¸ c 이다.a2 + b2

c2은 얼마인가?

(플란더즈 예선 1996/1997 1차)

풀이 세 변의 길이를 2x · 2y < 2z 라 하면 z2 = x2 + y2 이고,

a2 = x2 + (2y)2; b2 = (2x)2 + y2; c = z

따라서,a2 + b2

c2=(x2 + 4y2) + (4x2 + y2)

x2 + y2= 5

¢ ¢ ¢ 답 5 }

42. 직사각형 ABCD 내부에 두 점 P , Q가 있는데, PQ는 AB에 평행하다. X, Y 는 변 AB 위의 점으로 A,X, Y , B의 순서로 있고, Z, W는 변 CD 위의 점으로 C, Z, W , D의 순서로 있다. 네 영역 AXPWD,XPQY , BY QZC, WPQZ의 넓이는 모두 같다. BC = 19, PQ = 87, XY = Y B+BC+CZ =WZ =WD +DA+ AX 일 때 AB는 얼마인가? (AIME 1987-6)

풀이 %EEE

Let distance of PQ from XY be h and XY = x. Then area XPQY = area WPQX,

so (PQ + XY)h/2 = (PQ + WZ)(19-h)/2. Hence 2h = 19. Also AX + BY + DW + CZ

= 2x - 38, so 2 AB = 4x - 38, or AB = 2x - 19. Hence area ABCD = (2x - 19)19.

But 4 x area XPQY = (87 + x)19. Hence x = 106, so AB = 193.

¢ ¢ ¢ 답 193 }

43. P는 삼각형 ABC 내부의 점이고, 직선 PA, PB, PC는 각각 변 BC, CA, AB와 점 D, E, F에서 만난다. PD = PE = PF = 3 이고 PA+ PB + PC = 43 일 때, PA ¢ PB ¢ PC 를 구하여라.

(AIME 1988-12)

풀이 %EEE

Area PAB/area ABC = 3/(c+3). Hence 1 = 3/(a+3) + 3/(b+3) + 3/(c+3).

Multiplying out we get abc = 9(a + b + c) + 54.

¢ ¢ ¢ 답 441 }

44. 어떤 볼록사각형 ABCD가 지름 AB인 원에 내접한다. 각선 AC와 BD의 교점을 S라 하고, S에서AB에 내린 수선의 발을 T라 하자. ST가 각 CTD를 이등분함을 증명하여라. (뉴질랜드 2003-7)

증명 (과천고 2학년 홍성준)

\ADB = \ACB = \STA = \STB = 90± 이므로 ¤ADST , ¤BCST는 원에 내접하는 사각형들이다.

Page 135: 실전수학올림피아드 1400제 해답

3.1 기하 중급문제 135

따라서, 원주각에서

\STD = \SAD = \CAD = \CBD = \CBS = \CTS

이므로 ST는 \CTD의 이등분선이다. ¤

45. 중심이 O인 원에 볼록사각형 ABCD가 내접하고 있고, 각선은 서로 직교한다. 꺾은 선 AOC에 의해 이사각형이 같은 넓이의 두 영역으로 나뉘어짐을 증명하여라. (Towns 1981 J3)

증명1 (제주 대기고 1학년 김대철, 대전 어은중 1학년 임준혁)

O에서 현 BD에 내린 수선의 발을 M이라 하자.

OM과 AC가 모두 BD에 수직이므로 OM k AC 이다. 따라서, 등적변형으로 j4OACj = j4MACj 이고, 그럼

j¤OABCj = j¤MABCj

가 된다. 또한, M은 BD의 중점이므로, j4AMBj = j4AMDj, j4CMBj = j4CMDj 이다. 즉,j¤ABCM j = j¤ADCM j = 1

2j¤ABCDj. 따라서, j¤OABCj = 1

2j¤ABCDj = j¤OADCj 가 된

다. ¤

증명2 \AOB +\COD = 2\ADB + 2\CAD = 2 ¢ 90± 이므로 4OAB와 4OCD는 12r2 sin µ의 넓

이 공식에 의해 넓이가 같다. (혹은 4OCD를 O를 중심으로 적당히 회전시켜 4OAB와 변으로 접하도록 하면 넓이가 같다는 것이 직접 관찰된다.) 4OBC와 4ODA도 마찬가지이고, 따라서 두 사각형은 넓이가 같다. ¤

46. 정삼각형 ABC가 한 원과 6개의 점 D, E, F , G, H, J에서 만나고 있다. 이 점들은 삼각형의 둘레를 따라 A;H; J;B;D;E;C; F;G의 순서로 있다. CF = 1, FG = 13, GA = 2, HJ = 7 일 때 선분 DE의 길이를 구하여라.

¢ ¢ ¢ 답 2p22

47. 평면 위에 원 C와 점 P가 주어져 있다. P를 지나고 C와 만나는 직선들은 C의 현을 하나씩 결정한다. 이런 현들의 중점은 한 원 위에 있음을 보여라. (캐나다 1991-3)

증명 원의 중심을 O라 하고 현의 중점을 M이라 하자.

그러면 OM과 PM은 항상 직교한다. 따라서, M은 OP를 지름으로 하는 원 위에 있다. ¤

Page 136: 실전수학올림피아드 1400제 해답

136 기하

48. 예각삼각형 ABC의 세 수선이 외접원과 각각 점 D, E, F에서 만난다. 이 세 수선은 다음 중 삼각형DEF의 어떤 선에 해당하는가? (1) 수선 (2) 중선 (3) 각의 이등분선 (4) 변의 수직이등분선 (5) 앞의 어느 것도 아니다. (플란더즈 예선 1997/1998 1차)

¢ ¢ ¢ 답 (3)

49. 세 점 A(¡8; 5), B(¡15;¡19), C(1;¡7)로 이루어진 삼각형에서 각 A의 이등분선은 ax+ 2y + c = 0 이다. a와 c를 구하여라. (AIME 1990-7)

풀이 %EEE

AB has gradient (5+19)/(15-8) = 24/7, AC has gradient

(19-7)/(15+1) = 3/4.

[그림]

By Pythagoras, AB = 25, AC = 15. Extend AC to X with AX = 25,

then X must be (7,-15), so the midpoint of BX is (-4,-17), and the

equation of the line joining it to A is (x+8)/(y-5) = -4/22,

or 11x + 2y + 78 = 0. hence a = 11, c = 78.

¢ ¢ ¢ 답 11, 78 }

별해 각의 이등분선 정리를 이용하면 된다. }

50. 한 모서리의 길이가 1인 정이십면체의 한 꼭지점 A에서 그 칭인 꼭지점 B까지 표면을 따라 움직이는최단거리는 얼마인가? (플란더즈 예선 2000/2001 2차)

풀이 전개도를 생각하면... 정삼각형 4개를 이어붙인 도형에서 양끝점의 최단거리니까... ¢ ¢ ¢ 답 p7

}

51. ¤ABCD는 AB = 13p3, AD = 12

p3 인 직사각형이고 O는 이 사각형의 중심이다. 4OBC를 잘라내고

남은 도형에서, 선분 OA와 OD를 접어서 한 면이 트여있는 사면체를 만들었다. 이 사면체의 부피를 구하여라. (AIME 1990-14)

풀이 %EEE

I cannot see any elegant solution to this. It seems to be a horrible

slog. Take the base ADC to have sides m, n, n where m = √432, n = √507.

Then the three sides from the base to the apex O are all equal to

½√(m^2 + n^2). So if the foot of the altitude from O is X, then

OXA, OXC, OXD are congruent, so X is the circumcenter of ADC.

[그림]

By similar triangles CE/AC = CD/CY, so CE = n^2/√(n^2 - m^2/4). Hence

OX^2 = (m^2 + n^2)/4 - n^4/(4n^2 - m^2) = ½m^2(3n^2-m^2)/(4n^2-m^2).

Area ADC = ½m√(n^2 - m^2/4), so vol = (1/3) OX area ADC =

m^2/24 √(3n^2-m^2) = 594.

¢ ¢ ¢ 답 594 }

주 AB = 2a, AD = 2b, 사면체의 높이를 h라 하면

h2 = 3b2 ¡ b4

a2= b2

Ã3¡µb

a

¶2!

이 식의 결과가 잘 나오도록 a와 b를 좋은 값으로 주는 것이 어떨까.

52. 직사각형이 반원에 내접한다는 것은 직사각형의 밑변이 반원의 밑변 위에 있고 직사각형의 윗변이 반원의 원호에 내접함을 뜻한다. 반지름이 3인 반원에 높이가

p3인 직사각형이 내접한다. 이 반원에 내접하

는 또다른 직사각형이 있는데, 먼저 주어진 직사각형과 넓이가 같다고 한다. 이 두 번째 직사각형의 높이를 구하여라. (IT꿈나무 올림피아드 2006 1차)

Page 137: 실전수학올림피아드 1400제 해답

3.1 기하 중급문제 137

풀이 그림과 같이 직사각형의 밑변의 길이를 2x, 높이를 y라 하면, x2 + y2 = 32.

처음의 직사각형은 y =p3 일 때이므로 x =

p6. 이제 두 번째 직사각형의 x, y에 해 살펴보자. 처

음의 직사각형과 넓이가 같다고 했으므로 xy =p3 ¢ p6. 즉, x2y2 = 18. x2, y2에 한 연립방정식이

다. 합과 곱을 알고 있으므로 이차방정식으로 풀리고, 한 근(처음의 직사각형)을 알고 있으므로 나머지

한 근은 둘을 서로 바꾼 것이다. 즉, x2 = 3, y2 = 6. ¢ ¢ ¢ 답 p6 }

53. 원에 내접하는 볼록사각형 ABCD가 있다. 변 AB와 CD의 연장선이 만나는 교점을 P라 하고, AD와BC의 연장선이 만나는 교점을 Q라 하자. 그리고, 4PBC의 외접원과 4QAB의 외접원이 만나는 B 이외의 교점을 R이라 하자. P , Q, R이 한 직선 위에 있음을 보여라.

증명 원주각에 의해 \PRB = \PCB = \C, \QRB = \QAB = \A 이다.

그리고, ¤ABCD가 원에 내접하므로 각의 합 \A+\C = 180± 이다.따라서, \PRB+\QRB = 180±

이고, 이것은 P , R, Q가 한 직선을 이룸을 뜻한다. ¤

54. 삼각형 ABC에서 BC = 4, AC = 2, AB = 3 이고 삼각형 A0B0C0에서 A0B0 = 1 이다. \BAC +\B0A0C0 = 180± 이고 \ABC = \A0B0C0 일 때, 삼각형 ABC의 넓이를 S, 삼각형 A0B0C0의 넓이를

S0이라고 하자.S

S0의 값을 구하여라. (한국 2006 1차-J13)

풀이1 (KAIST 응용수학과 05학번 이준경)

4A0B0C0을 2배 확 한 삼각형 4A"B"C"을 4ABC에 AC와 A"B"이 일치하도록 맞붙이자. 그럼\A+ \A" = 180± 이므로 BA(A")C"은 일직선을 이룬다.

또, \B = \B" 이므로 4C"BC » 4C"B"A" 이다. A"C" = x, B"C" = y 라 하면 이 닮은삼각형의비례관계에서

x : y = y : x+ 3 = 2 : 4

Page 138: 실전수학올림피아드 1400제 해답

138 기하

가 되고, 그럼 y = 2x, x + 3 = 2y 이므로 이것을 풀면 x = 1 임을 알 수 있다. 따라서, j4ABCj :j4A"B"C"j = 3 : x = 3 : 1 이고, j4A"B"C"j = 4S0 이므로 S=S0 = 12 ¢ ¢ ¢ 답 }

풀이2 (KAIST 과학영재센터 연구원 고봉균)

\B = \B0 이므로 B와 B0이 일치시키고 C0 2 AB, A0 2 BC 가 되도록 놓자. 그리고, BC 위에 D를잡아 AD k C0A0 이 되도록 하자.

그럼 \A = 180± ¡ \A0 = \C0A0C = \ADC 가 되므로 4CAB » 4CDA. 여기서 CD : CA = CA :CB, 즉 x : 2 = 2 : 4 이므로 x = 1. 따라서, BD = 3 이고 j4ABDj = 3

4S. 4ABD와 4C0B0A0의 닮

음비가 BD : B0A0 = 3 : 1 이므로 j4A0B0C0j = 19j4ABDj = 1

12S 이다. ¢ ¢ ¢ 답 12 }

55. 삼각형 ABC에서 \ACB의 이등분선이 AB와 만나는 점을 D라 하자. 삼각형 ABC의 외심이 삼각형BCD의 내심과 일치한다고 한다. 이 때, AC2 = AD ¢AB 임을 증명하여라.

(루마니아 2005 지역예선 y7-4)

증명 4ABC의 외심을 O라 하자.

OBC는 OB, OC가 외접원의 반지름인 이등변 삼각형이므로 \OBC = \OCB. O는 4BCD의 내심이기도 하므로 \OBD = \OBC = \OCB = \OCD, 따라서 \DBC = \DCB 가 된다. CD는 \C의이등분선이므로 \ACD = \DCB. 이로부터

\ACD = \DBC

이다. \A는 공통이므로 4ABC » 4ACD 의 닮음이 되고, 따라서 AB : AC = AC : AD 이다. ¤

56. 네 면의 넓이가 모두 같은 사면체 ABCD가 있다. 이 사면체의 내부에 있는 임의의 점 P에서 이 사면체의 네 면에 이르는 거리의 합이 일정함을 보여라.

증명 사면체 PABC, PBCD, PCDA, PDAB의 부피의 합을 생각하면 간단. ¤

57. 수학적인 거미줄 기술이 있는 거미가 다음 그림과 같이 모든 선분의 길이가 자연수인 거미줄을 만들었다.

Page 139: 실전수학올림피아드 1400제 해답

3.1 기하 중급문제 139

x를 구하여라. (플란더즈 예선 2003 2차)

풀이 x+ 5 > 17 이고 x < 5 + 9 이므로 x = 13 ¢ ¢ ¢ 답 }

58. 평면 위에 볼록사각형 ABCD가 주어져있을 때, 네 꼭지점에 이르는 거리의 합 PA + PB + PC + PD

가 최소가 되는 점 P의 위치를 찾아라. (통신강좌 1993-7-6)

풀이 최단거리 부등식에 의해

PA+ PC ¸ AC; PB + PD ¸ BD

이고, 각각 등호는 P가 AC 위에 있을 때, BD 위에 있을 때 성립한다. 따라서, P가 두 각선 AC와BD의 교점일 때, PA+ PB + PC + PD 는 AC +BD 와 같게 되고 이 때가 최소이다. }

59. AB와 CD가 평행한 사다리꼴 ABCD에서, AB = 92, BC = 50, CD = 19, DA = 70 이라고 한다. P는변 AB 위의 점이고, P를 중심으로 하는 한 원이 변 AD와 BC에 접한다. AP의 길이를 구하여라.

(AIME 1992-9)

풀이 %EEE

Extend AD, BC to meet at X. Then B lies on the bisector of ∠AXB.

But AP/PB = AX/BX. By similar triangles AX/BX = DX/CX = AD/BC = 7/5.

Hence AP = (7/12)92 = 161/3.

¢ ¢ ¢ 답 161=3 }

60. AB = 1995, BC = 1994, CA = 1993 인 삼각형 ABC가 있다. CX를 한 수선이라고 하자. 두 삼각형ACX와 BCX의 내접원이 CX와 만나는 두 접점 사이의 거리를 구하여라. (AIME 1993-15)

풀이 %EEE

We have AC = AU + CU = AT + CR = (AX - r) + (CX - r), where r is

the inradius of ACX. Put AX = x, CX = z, AC = b. Then 2r = x + z - b.

Similarly, if r' is the inradius of BCX and BX = y, then

2r' = y + z - a, so 2RS = |2r - 2r'| = |x - y - b + a| = |x - y - 1|.

Now x^2 = b^2 - z^2, y^2 = a^2 - z^2, so x^2 - y^2 = b^2 - a^2,

and x - y = (b^2 - a^2)/c = 3987/1995. Hence RS = 996/1995.

[그림]

¢ ¢ ¢ 답 332=665 }

61. 중심이 O인 원의 접선 AD와 할선 ABC가 있다(B, C, D는 원 O 위의 점). D에서 AO에 내린 수선의 발을 E라고 하면 \AEB = \ACO 임을 증명하여라. (1993 도대회)

증명 직각삼각형의 닮음과 원의 방멱에 의해

AE ¢ AO = AD2 = AB ¢AC

따라서, 4ABE » 4AOC 이다(SAS닮음). ¤

Page 140: 실전수학올림피아드 1400제 해답

140 기하

62. 볼록사각형 ABCD의 변 AB, BC, CD, DA 위에 각각 점 P , Q, R, S를 잡는데,AP

PB=

BQ

QC=

CR

RD=

DS

SA= k 로 비가 일정하도록 한다. ¤PQRS의 넓이가 ¤ABCD의 넓이의 52%일 때, k의 값을 구하여

라. (IMTS R3-3)

풀이 j4APSj = kk+1

¢ 1k+1

j4ABDj 임 등을 이용하면

0:48j¤ABCDj = j4APSj+ j4BQP j+ j4CRQj+ j4DSRj = 2k

(k + 1)2j¤ABCDj

이 된다. 여기서 k에 한 이차방정식을 풀면 6k2 ¡ 13k + 6 = 0, (3k ¡ 2)(2k ¡ 3) = 0, 즉 k = 23or 3

2

¢ ¢ ¢ 답 }

63. 삼각형 ABC의 변 AB의 중점을 M이라 하자. \ABC의 이등분선이 AC와 만나는 점을 D라 하자.MD ?BD 이면 AB = 3BC 임을 증명하여라. (루마니아 2005 지역예선 y7-2)

증명 MD \BC = N , 즉 두 직선 MD와 BC의 교점을 N이라 하고, AC의 중점을 K라 하자.

4BNM은 \B의 이등분선이 밑변과 수직으로 만나므로 이등변 삼각형이다. 따라서, BM = BN 이고MD = DN . 삼각형의 합동과 중점연결정리에 의해 BC = 2MK = 2CN , 그럼

BC =2

3BN =

2

3BM =

1

3AB

가 된다. ¤

64. 삼각형 ABC의 각 B, C의 이등분선을 각각 BP , CQ라 하자. 그리고, 꼭지점 A에서 직선 BP , CQ에 내린 수선의 발을 각각 H, K라 하자. KH와 BC가 평행함을 증명하여라. (호주 1989-2)

풀이 AK, AH의 연장이 BC와 만나는 점을 L, M이라 하면 KH는 4ALM의 중점연결선. }

65. 중심이 O인 원에 두 현 AC와 BD가 원 내부의 점 X에서 만나는데, AC = BD = 78, OA = 42, OX = 18

이라고 한다. 부채꼴과 비슷하게 생긴 영역 CXD의 넓이를 구하여라. (AIME 1995-14)

풀이 %EEE

OY^2 = OA^2 - AY^2, so OY = 9√9. XY^2 = OX^2 - OY^2, so XY = 9

and ∠OXY = 60^o. Hence DX = AX = AY + XY = 48, BX = CX = 30.

Also ∠AXB = 180^o - 2·60^o = 60^o. So area triangle AXB =

½AX·BX sin 60^o = 360√3. ∠ACB = ∠XBC and their sum is ∠AXB = 60^o,

so ∠ACB = 30^o. Hence ∠AOB = 60^o. So area sector OAB = 422π/6 and

area between AB and minor arc AB = 294π - 422(√3)/4. Hence required

area = 294π - 81√3.

[그림]

¢ ¢ ¢ 답 294¼ ¡ 81p3 }

Page 141: 실전수학올림피아드 1400제 해답

3.1 기하 중급문제 141

66. 정삼각형 ABC의 변 BC, CA, AB의 중점을 각각 D, E, F라 하자. P , Q, R은 각각 선분 EF , FD, DE

위의 점으로, APR, BQP , CRQ가 각각 일직선을 이룬다고 한다. 4PQR의 넓이가 1일 때, 4ABC의 넓이를 구하여라. (AIME 1998-12)

풀이 %EEE

Compare the figures APRE and CRQD. If PE > RD, then RE > DQ, so

QF > RD. Similarly that implies QF > PE, and that implies PE < RD.

Contradiction. Similarly if PE < RD. So PE = RD. Similarly PE = QF.

Put PF = QD = RE = x, AF = 1. Then BQD, PQF are similar, so x/1 = (1-x)/x

or x^2 + x - 1 = 0, so x = (√5 - 1)/2 (we discard the negative root).

By cosine rule PQ^2 = x^2 + (1-x)^2 - x(1-x) = 3x^2 - 3x + 1

= 3(x^2+x-1)+4-6x = 7-3√5. Area ABC/area PQR = 22/x2 = 7 + 3√5.

¢ ¢ ¢ 답 7 + 3p5 }

67. M은 한 변의 길이가 4인 정삼각형 3개를 붙여 만든 등변사다리꼴이다. M 0은 M을 뒤집어놓은 사다리꼴이다. M과 M 0의 밑변을 같은 직선 위에 두고 두 사다리꼴을 이 직선 위에서 미끄러뜨릴 때, 두 사다리꼴이 겹치는 영역의 넓이는 최 얼마인가? (플란더즈 예선 1998/1999 2차)

¢ ¢ ¢ 답 10p3

68. AB = 8, BC = 6, BD = 10, \A = \D, \ABD = \C 인 사각형 ABCD가 있다. CD의 길이를 구하여라. (AIME 2001 2차-13)

풀이 %EEE

Extend AB, CD to meet at E. Then triangles ECB, EBD are similar.

Hence BD/ED = CB/EB, so 10/ED = 6/(ED-8). Hence ED = 20.

Also EC/CB = EB/BD, so EC = 7.2. Hence CD = 12.8 = 64/5.

¢ ¢ ¢ 답 64=5 }

69. 한 변의 길이가 1인 정육면체 ABCD-A1B1C1D1가 있다.모서리 BC의 중점을 E라 하고, 면 CDD1C1의중심을 M이라 하자. 평면 AEM이 이 정육면체와 만나는 단면의 넓이를 구하여라. (플란더즈 1998-2)

풀이 이 평면이 CC0, DD0과 만나는 점을 P , Q라 하자. 단위정육면체 CDGH-C1D1G1H1을 맞붙여

그리면, M은 AH1의 중점. D1G1의 중점을 F라 하면 EF의 중점도 M이고, 따라서 AEH1F는 평행사

변형이고 구하는 단면의 넓이는 이것의 절반. 충 구하면 넓이는p144

¢ ¢ ¢ 답 }

70. 사각형 ABCD에서 AB = 5, BC =p50, CD = 4, AD = 3 이고 AC = 5, BD =

p58 이다. X, Y 가 각

각 각선 BD와 AC의 중점일 때 XY 2을 구하여라. (2000 대전.충남 영재수학교실 2차평가)

풀이 파푸스의 중선정리에 의해

2(AX2 +BX2) = AB2 +AD2

2(AY 2 +XY 2) = AX2 + CX2

2(CX2 +BX2) = CD2 +BC2

첫 번째 식과 세 번째 식을 더한 후 두 번째 식을 이용해 2(AX2 + CX2)을 소거하면

AC2 +BD2 + 4XY 2 = AB2 +BC2 + CD2 +DA2

25 + 58 + 4XY 2 = 25 + 50 + 16 + 9

따라서, 4XY 2 = 17, 즉 XY 2 = 174

¢ ¢ ¢ 답 }

Page 142: 실전수학올림피아드 1400제 해답

142 기하

71. AB = 6, AC = 8 인 4ABC에서 \BAC의 이등분선이 BC와 만나는 점을 D라 했더니 AD = AB 가되었다. C에서 직선 AD에 내린 수선의 발을 M이라 할 때, AM의 길이를 구하여라.

(통신강좌 1990-1-1)

풀이 각의 이등분선 정리에서 BD : DC = BA : AC = 3 : 4. 따라서, BD = 3x 라 하면 DC = 4x.

BD의 중점을 N이라 하면, AN2 +ND2 = AD2, AN2 +NC2 = AC2 임에서

28 = AC2 ¡AD2 = NC2 ¡ND2 =

µ11

2x

¶2¡µ3

2x

¶2= 28x2

따라서, x = 1. 4AND와 4CMD가 AA-닮음이므로 CD : DM = AD : DN = 6 : 32= 4 : 1. CD = 4

이므로 DM = 1. 따라서, AM = 7. }

별해 AB, AC의 중점을 각각 E, F라 하고, EF와 AD의 교점을 N이라고 하면, EF k BC 이고,가정에

서 AB = AD 이므로, AN = AE = 12AB. 직각삼각형 AMC에서 F는 빗변의 중점이므로 AF = CF =

MF = 12AC 이다. 따라서 삼각형 FAM은 이등변삼각형이고, \FMA = \FAM = \MAB = 1

2\A.

그러므로 AB k FM . 한편 EF k BC 이므로, \EFM = \B = \ADB = \MNF 이고 삼각형MNF는 이등변삼각형. 고로 MN =MF = 1

2AC. 따라서 AM = AN +MN = 1

2(AB +AC). }

72. 직각삼각형 ABC의 넓이를 S라 하고 외접원과 내접원의 반지름을 각각 R, r이라 하자. r(2R + r) = S

임을 증명하여라.

증명 직각삼각형의 둘레의 길이는 2r + 4R. ¤

73. 한 변의 길이가 1인 두 개의 정사각형이 같은 중심을 갖고 공통의 팔각형 영역을 갖는다. 이 팔각형의 어느 한 변의 길이가 43=99일 때, 이 팔각형의 넓이를 구하여라. (AIME 1999-4)

풀이 %EEE

Let the sides of the triangle be x, y, √(x^2+y^2). It is clear

from the diagram that the perimeter of the triangle is the side of

the square 1. Solving, y = (1-2x)/(2-2x) and octagon side = 1-x-y

= (2x^2-2x+1)/(2-2x) = 43/99. Hence 198x^2 - 112x + 13 = 0. Product of

roots is 13/198. So area octagon = area square - 4 x area triangle

= 1 - 2xy = 1 - 26/198 = 86/99.

[그림]

¢ ¢ ¢ 답 86=99 }

74. P와 Q는 x축 위를 움직이는 점이고 PQ = 8 로 일정하다. A(0; 6), B(16; 10)에 해 사각형 ABQP의 둘레의 길이가 최소가 될 때, PQ의 중점 M의 x-좌표를 구하여라. (플란더즈 예선 1998/1999 1차)

힌트 최단거리부등식, 7

75. AB k DC, BC ? AB, AC ? BD 인 사다리꼴 ABCD가 있다. AB =p11, AD =

p1001 일 때, BC의

길이를 구하여라. (AIME 2000 2차-8)

Page 143: 실전수학올림피아드 1400제 해답

3.1 기하 중급문제 143

풀이 %EEE

Put BC = x. Then AC = √(x^2+11). ABC and BCD are similar.

So BD/BC = AC/AB, so BD = x(√(x^2+11))/√11. Also

cos ABD = cos ACB = x/√(x^2+11). Applying cosine rule to ABD, we get

1001 = AD^2 = AB^2 + BD^2 - 2AB·BD cos ABD = 11 + x^2(x^2+11)/11 - 2x^2.

Hence (x^2+99)(x^2-110) = 0, so x = √110.

¢ ¢ ¢ 답 p110 }

76. 4PQR의 세 변의 연장선¡!PQ,

¡!QR,

¡!RP의 연장선 위에

PA

PQ=

QB

QR=

RC

RP= a 가 되도록 각각 점 A, B,

C를 잡자. 4ABC의 넓이가 4PQR의 61배가 되도록 하려면 a를 얼마로 해야 하는가? (1991 도대회)

77. AB = 5, BC = 6, CA = 7 인 4ABC 내부에 한 점 P가 있다. P를 지나는 이 삼각형의 현을 각 변에 평행하게 하나씩 그렸더니, 이 세 현의 길이가 모두 같았다. 변 AB와 만나는 두 현이 변 AB의 길이를 분할하는 비를 구하여라. (1995 도대회)

78. 세 변의 길이가 90, 120, 150인 직각삼각형이 있다. 이 직각삼각형의 내접원을 그리고, 빗변이 아닌 두 변과 평행하며 이 내접원에 접하는 두 직선을 그리면, 원래 직각삼각형의 내부에 작은 직각삼각형 영역 두개를 얻는다. 이 두 작은 직각삼각형의 내심을 잇는 선분의 길이를 구하여라. (AIME 2001 2차-7)

풀이 %EEE

Let B be the vertex with angle 90^o. Suppose it is a distance

x from the points of contact of the incircle. Then chasing around

the triangle using the fact that the tangents from a point have

equal length, we get x = (90 + 120 - 150)/2 = 30. Evidently

this is also the radius of the incircle. The other two triangles

are similar. The top one has sides 30, 40, 50 and inradius 10,

and the right-hand one has sides 45, 60, 75 and inradius 15.

We can regard the dotted line as the hypoteneuse of a right-angled

triangle with vertical side 60+10-15 = 55, horizontal side 60+15-10

= 65. Hence length √(55^2 + 65^2) = 5√290.

¢ ¢ ¢ 답 5p290 }

79. 원 O를 중심으로 하는 원 C가 있다. AB는 이 원의 지름이고, BM은 B에서의 이 원의 접선이다. 원주위의 또 다른 점 E에서의 접선이 BM와 F에서 만난다. AE의 연장선이 BM과 만나는 점을 D라 할 때,BF = FD 임을 증명하여라. (1995 교육청경시)

80. 직각삼각형 ABC의 내접원의 반지름을 r이라 하자. r은 빗변의 1=4보다 짧고, 또 다른 두 변의 1=2보다짧음을 보여라. (헝가리 1925-3)

증명 직사각형 ABCD를 생각하면 내접원이 완전히 포함되므로 지름이 한 변의 길이보다 짧아서

2r < a; c. 직각점에서 빗변에 내린 높이를 h, 중선의 길이를 d라 하면, 직각점을 지나 빗변에 평행한 선에 해 직각삼각형이 완전히 한 쪽에 있으므로 내접원도 완전히 한 쪽에 있어서 두 평행선 사이의 거리h보다 지름이 짧으므로 2r < h · d = 1

2b. ¤

81. 다음 그림은 한 변의 길이가 a인 정사각형의 각 꼭지점과 그 꼭지점을 포함하지 않는 변의 중점을 연결한선분들을 그려 얻은 도형이다. 빗금친 영역의 넓이를 구하여라. (플란더즈 예선 1990/1991)

Page 144: 실전수학올림피아드 1400제 해답

144 기하

풀이 정사각형 ABCD라 하고, AB부터 각 변의 중점을 K, L, M , N . 또, 빗금친 팔각형의 꼭지점들

을 N에 가까운 것부터 PQRSTUVW라 하자. 그럼 구하는 넓이는 S = a2¡4(j4PAKj+ j4QKBj) =a2 ¡ 4( 1

8a2 + 1

12a2) = 1

6a2 ¢ ¢ ¢ 답 }

82. A, B, C는 AB = AC 를 만족하는 한 원 위의 세 점이다. 선분 BC 위의 임의의 점을 E라 하고, 직선AE가 원과 다시 만나는 점을 D라 하자. AD ¢ AE 는 E의 선택과 관계없이 항상 일정함을 증명하여라.

(아벨콘테스트 1997 결선 2b)

증명 이등변삼각형이므로 \ABC = \ACB 이고, 원주각에 의해 \ABC = \ADC 이다.

즉, \ACB = \ADC 가 되고 \CAD는 공통이므로

4ADC » 4ACE

따라서, AD : AC = AC : AE 이고 풀면 AD ¢AE = AC2 으로 일정한 값이 된다. ¤

별증1 A에서 BC에 내린 수선이 BC와 원과 만나는 점을 각각 M , N이라 하자. 이등변삼각형이므로

AN에 해 좌우 칭의 그림이 되고 따라서 AN은 지름이다.

그럼 지름에 한 원주각으로 \ADN = \R = \AME 이고, \DAN은 공통이므로

4ADN » 4AME

따라서, AD : AN = AM : AE 이고 풀면 AD ¢AE = AM ¢AN 이다. M , N은 4ABC에 의해 결정된점이므로 이 값은 일정한 값이다. ¤

Page 145: 실전수학올림피아드 1400제 해답

3.1 기하 중급문제 145

별증2 A에서 BC에 내린 수선의 발을 M이라 하고, 그림과 같이 AB = AC = a, AM = h, BM =

CM = b, ME = x 라 하자.

그럼 다음과 같이 식을 정리할 수 있다.

AD ¢AE = (AE + ED)AE

= AE2 +AE ¢ ED

= AE2 +BE ¢ EC (* 방멱)

= (h2 + x2) + (b+ x)(b¡ x)

= h2 + b2 = a2

따라서, 일정한 값이다. ¤

83. 두 원 k1, k2가 한 점 P에서 접한다. P를 지나는 한 직선이 원 k1, k2와 각각 A1, A2에서 다시 만나고,P를 지나는 또다른 직선이 원 k1, k2와 각각 B1, B2에서 다시 만난다. 삼각형 PA1B1과 PA2B2가 닮았음을 증명하여라. (헝가리 1933-3)

증명 먼저 두 원이 외접하는 경우를 생각하자. 두 원의 공통내접선을 XY 라 하자(X는 \A1PB2쪽에,

Y 는 \B1PA2쪽에 있는 점이다). 접선과 현이 이루는 각과 맞꼭지각에 의해

\A1B1P = \A1PX = \A2PY = \A2B2P

이 된다. 마찬가지로 \B1A1P = \B2A2P 가 된다( 신에 맞꼭지각 \A1PB1 = \A2PB2를 얘기해도된다). 따라서, 4PA1B1과 4PA2B2는 AA닮음이다.

두 원이 내접하는 경우에도 \A1B1P = \A1PX = \A2B2P 등으로 비슷하게 된다(맞꼭지각을 생각할 필요가 없으므로 더 쉽다). ¤

별증 반직선 PA1, PB1, 원 k1을 모아 집합 X라 하고, 반직선 PA2, PB2, 원 k2를 모아 집합 Y 라 하

자. 두 원이 외접하든 내접하든, 집합 X와 집합 Y 는 P를 중심으로 하는 확 -축소 닮음 관계에 있다. 따라서, 4PA1B1과 4PA2B2도 P를 중심으로 같은 닮음 관계에 있다. ¤

Page 146: 실전수학올림피아드 1400제 해답

146 기하

84. 한 변의 길이가 1인 정오각형 ABCDE가 주어져 있다. M을 이 도형의 내부 혹은 선분 위의 점이라 하자.

점 M으로부터 가장 가까운 꼭지점까지의 거리를 r1, 그 다음으로 가까운 꼭지점까지의 거리를 r2, : : : 이런 식으로 계속하여, M으로부터 다섯 꼭지점까지의 거리를 r1 · r2 · r3 · r4 · r5 이 되도록 정하자.

(a) r3이 최소값이 되는 점 M의 자취를 구하여라.

(b) r3이 최 값이 되는 점 M의 자취를 구하여라. (소련 1962-8)

풀이 Let X be the midpoint of AB and O the center of ABCDE. Suppose M lies inside AXO.Then ME = r3. So we maximise r3 by taking M at X, and we minimise r3 by taking M as theintersection of AO and EB. AXO is one of 10 congruent areas, so the required loci are (a) the 5midpoints of the diagonals, and (b) the 5 midpoints of the sides. }

85. 세 점 (¡6; 0), (0; 12), (16; 0)을 꼭지점으로 하는 삼각형의 둘레와 내부에 있는 격자점들 중에서, 두 점(0; 0), (5; 0)과 이루는 삼각형이 둔각삼각형이 되는 격자점의 개수를 구하여라.

(플란더즈 예선 1996/1997 1차 변형)

풀이 어떤 꼭지점이 둔각인가에 따라 나누어 (0; 0); (5; 0)과 둔각 삼각형을 이루는 격자점을 구한다.

² (0; 0)이 둔각인 경우 삼각형 내부나 둘레에 있는 점 중에서 x 좌표가 0보다 작은 점들이다.x = ¡1 : y = 0 ¢ ¢ ¢ 10 (11개)x = ¡2 : y = 0 ¢ ¢ ¢ 8 (9개)¢ ¢ ¢ x = ¡6 : y = 0 (1개)격자점 개수: 11 + 9 + ¢ ¢ ¢ 1 = 36개.

² (5; 0)이 둔각인 경우 삼각형 내부나 둘레에 있는 점 중에서 x 좌표가 5보다 큰 점들이다.(0; 12)와 (0; 16)을 잇는 기울기 ¡ 3

4인 직선 아래에 있는 점으로 x좌표가 4 단위로 변할 때 규칙성

이 있다.x = 5 : y = 0 ¢ ¢ ¢ 8 (9개) Ã 답에는 포함되지 않는다.x = 6 : y = 0 ¢ ¢ ¢ 7 (8개)x = 7 : y = 0 ¢ ¢ ¢ 6 (7개)x = 8 : y = 0 ¢ ¢ ¢ 6 (7개)x = 9 : y = 0 ¢ ¢ ¢ 5 (6개)¢ ¢ ¢격자점 개수는 (8 + 7 + 7) + (6 + 5 + 4 + 4) + (3 + 2 + 1 + 1) = 48이다.

² 나머지 한 점이 둔각인 경우는 그 점이 (0; 0)과 (5; 0)을 잇는 선을 지름으로 하는 원 내부에 있을 경우이다.원 내부와 둘레에 있는 점 중에서 y ¸ 0인 점은 삼각형 안에 포함되므로, (x¡ 52 )2+y2 · ( 5

2)2

인 점의 개수를 구한다.y = 0 : x = 0 ¢ ¢ ¢ 5y = 1 : x = 1 ¢ ¢ ¢ 4y = 2 : x = 1 ¢ ¢ ¢ 4) 6 + 4 + 4 = 14개.

총 36 + 48 + 14 = 98.

답: 98개. }

86. 양의 정수 a, b에 하여, y = (x¡ a)2 의 그래프와 y = b2 ¡ x2 의 그래프의 두 교점과 두 그래프의 y-축과의 교점들을 꼭지점으로 하는 사각형의 넓이가 12

p7 일 때, a+ b 의 값을 구하여라. (단, a < b 이다.)

(한국 2005 1차-J13)

풀이 두 그래프의 y-절편은 a2과 b2이고, 두 교점은 그래프를 그려보면 x-좌표의 부호가 서로 다르게

나타난다. 즉, (x¡ a)2 = b2 ¡ x2 의 두 근을 ®, ¯라 하면 구하는 사각형의 넓이는 삼각형 2개의 넓이의합으로 보아서

1

2(b2 ¡ a2)(j®j+ j¯j) = 1

2(b2 ¡ a2)j®¡ ¯j = 12

p7

이 된다. 2x2 ¡ 2ax+ a2 ¡ b2 = 0 에서 ®+ ¯ = a, ®¯ = 12(a2 ¡ b2) 이므로

(®¡ ¯)2 = (®+ ¯)2 ¡ 4®¯ = 2b2 ¡ a2

위의 넓이의 식을 제곱한 것에 이것을 입하면

(b2 ¡ a2)2(2b2 ¡ a2) = 26327

Page 147: 실전수학올림피아드 1400제 해답

3.1 기하 중급문제 147

이 된다. 만일 a와 b가 홀짝이 서로 다르면 b2 ¡ a2 은 홀수이고 2b2 ¡ a2 ´ 2 ¡ 0 or 0 ¡ 1 (mod 4)로 좌변이 4의 배수가 될 수 없어 모순. 따라서, a와 b는 홀짝이 같고, 그럼 b ¡ a ¸ 2, b + a ¸ 4 이므로 b2 ¡ a2 ¸ 8. (b2 ¡ a2)2 이 26327 의 약수임에서 b2 ¡ a2 = 8, 12, 24만이 가능하다. 이 때, 각각2b2 ¡ a2 = 63, 28, 7이고, b2 = 55, 16, ¡17 이 되어, 가능한 것은 b2 = 16뿐이다. 즉, b = 4, a = 2.a+ b = 6 ¢ ¢ ¢ 답 }

87. a£ 1995£ c 크기의 직육면체꼴의 나무토막이 있다(a · 1995 · c). 이 나무토막을 어느 한 면과 평행한면으로 두 조각으로 자르는데, 그 중 한 조각이 원래의 나무토막과 닮은꼴이 되도록 하려고 한다. 주어진(a; c)에 해, 이렇게 자르는 방법은 몇 가지가 가능한가? (AIME 1995-11)

풀이 %EEE

We must cut the longest edges, so the similar piece has dimensions

a x 1995 x k for some 1 ≤ k < c. The shortest edge of this piece cannot

be a, so it must be k. Thus a x 1995 x c and k x a x 1995 are similar.

Hence c = 1995^2/a, k = a^2/1995. Now 1995 = 3·5·7·19, so 1995^2 has

3^4 factors, of which (3^4-1)/2 = 40 are < 1995.

¢ ¢ ¢ 답 40 }

88. J씨는 직사각형꼴의 농장을 갖고 있는데, 그림과 같이 9개의 정사각형 구역으로 나누었다. 가장 작은 구역의 넓이는 1이다. 두 번째로 작은 영역 A의 넓이를 구하여라. (플란더즈 예선 2000/2001 2차)

A

풀이 A의 한 변의 길이를 a라 하고, 그 오른쪽 사각형의 한 변의 길이를 b라 하자. 그럼 그 오른쪽은

b+1,그 아래는 b+2,그 왼쪽은 b+3짜리가 된다.이 다섯 사각형 구역의 폭을 비교하면 a+2b+1 = 2b+5,

따라서 a = 4 ¢ ¢ ¢ 답 }

89. 중심 O인 원 위에 다섯 점 A, B, C, D, E가 차례로 있다. AOC, BOD, COE, DOA, EOB는 각각 B,

C, D, E, A를 내부에 포함하는 어떤 정다각형의 일부들(연속한 세 꼭지점들)이라고 한다. 이 5개의 정다

각형의 변의 개수를 각각 a, b, c, d, e라 할 때,1

a+1

b+1

c+1

d+1

e의 가능한 값을 모두 구하여라.

(IT꿈나무 올림피아드 2006 1차)

풀이 정n각형의 한 외각의 크기는 360±n 이므로, 한 내각의 크기는 180± ¡ 360±

n. 따라서,

720± = \AOC + \COE + \EOB + \BOD + \DOA

=

µ180± ¡ 360

±

a

¶+

µ180± ¡ 360

±

b

¶+

µ180± ¡ 360

±

c

¶+

µ180± ¡ 360

±

d

¶+

µ180± ¡ 360

±

e

¶2 =

µ1

2¡ 1

a

¶+

µ1

2¡ 1

b

¶+

µ1

2¡ 1

c

¶+

µ1

2¡ 1

d

¶+

µ1

2¡ 1

e

¶따라서, 1a +

1b+ 1

c+ 1

d+ 1

e= 5 ¢ 1

2¡ 2 = 1

2. ¢ ¢ ¢ 답 1

2}

Page 148: 실전수학올림피아드 1400제 해답

148 기하

90. 중점삼각형이란 원래 삼각형의 각 변의 중점을 세 꼭지점으로 하는 삼각형을 말한다. 부피가 1인 정사면체가 있다. 이 정사면체의 각 삼각형면에 중점삼각형을 그리고, 그 중점삼각형을 밑면으로 갖는 작은 정사면체를 각각 붙여 새로운 다면체를 얻는다. 이런 과정을 두 번 더 거치자(그래서 모두 세 번 거친다). 이결과로 얻어진 다면체의 부피는 얼마인가? (AIME 2001 2차-12)

풀이 %EEE

The side length of each small tetrahedron on the face of the original

tetrahedron is 1/2 the side length of the original, so its volume is 1/8.

There are 4 of them so total volume 1/2. The new figure has 24 faces.

The second time, each small tetrahedron has volume 1/64, so their total

volume is 24/64 = 3/8. The new figure has 3·24 = 72 faces which are

subsets of faces of the previous figure and another 72 outside, total 144.

The tiny tetrahedra have vol 1/512, so total 144/512 = 9/32. Hence

total vol = 1 + 1/2 + 3/8 + 9/32 = (32 + 16 + 12 + 9)/32 = 69/32.

¢ ¢ ¢ 답 69=32 }

91. 임의의 삼각형에서, 변보다 긴 수선은 많아야 하나뿐임을 증명하여라. (헝가리 1942-1)

증명 한 꼭지점 A에서 변에 내린 수선은 그 변에 이르는 최단거리이므로 변 AB, AC의 길이 이하

임. 즉, hA > a 라 하면, b; c ¸ hA > a ¸ hB ; hC 이므로 그런 경우는 많아야 하나뿐. ¤

92. 원 1의 중심이 원 2 위에 있고, 두 원은 두 점 A, B에서 만난다. B에서 그린 원 2의 접선이 원 1과 점 C에서 다시 만난다. AB = BC 임을 증명하여라. (Towns 1991가을 JO1)

증명 원 1의 중심을 O라 하자. 접현각의 성질로부터 \OBC = \OAB.

따라서, 4OAB와 4OBC는 같은 크기의 밑각을 갖는 OA = OB = OC 인 이등변삼각형이므로 합동이다. 이로부터 AB = BC. ¤

93. 한 각의 내부에 중심이 각각 A, B인 두 원이 있다. 두 원과 각의 두 변은 서로 접한다. AB를 지름으로 하는 원도 각의 두 변에 접함을 증명하여라. (Towns 1991가을 SO1)

증명 각을 XOY , AB의 중점을 M이라 하고, A, M , B에서 OY 에 내린 수선의 발을 각각 C, N , D라

하자. 그리고, 원 A와 B, M의 반지름을 각각 a, b, m이라 하자.

AB가 원 M의 지름이므로 m = AB2= a+b

2 이다. 그런데, 사다리꼴 ACDB에서 MN은 중점연결선

이므로 윗변 MN = AC+BD2

= a+b2 이다. 이것은 m과 같으므로, 원 M은 OY 에 접한다. 칭적으로,

OX에도 접한다. ¤

Page 149: 실전수학올림피아드 1400제 해답

3.1 기하 중급문제 149

94. 삼각형 ABC의 외심을 O, 수심을 H, 그리고 변 BC에 한 O의 칭점을 D라 하자. AHDO가 평행사변형임을 보여라.

증명 O에 한 B의 칭점을 E라 하면 중점연결정리에 의해 OD = CE. 그리고 각을 좀 고려해주

면 AHCE가 평행사변형... ¤

95. 볼록사각형 ABCD에서 변 BC와 DA의 중점을 각각 M , N이라 하자. 각선 AC가 MN을 이등분한다고 할 때, 두 삼각형 ABC와 ACD의 넓이가 같음을 증명하여라. (Towns 1988봄 JO2)

증명 MN \ AC = P 라 하자. j4ABCj = 2j4AMCj = 2(j4AMP j + j4CMP j) = 2(j4ANP j +j4CNP j) = 2j4ANCj = j4ACDj. ¤

96. 정사각형 ABCD의 내부에 점 M이 있다. \MAC = \MCD = x 라 할 때, \ABM 의 크기는 얼마인가? (Towns 1988가을 JA2)

풀이 MAC의 외접원은 CD와 C에서 접함. 즉 B를 중심으로 함. 따라서, \ABM = 2\ACM =

2(45± ¡ x). }

97. AM은 4ABC의 한 중선이다. 4ABM의 내접원의 반지름이 4ACM의 내접원의 반지름의 2배가 될 수있는가? (Towns 1989봄 JO2)

98. 사다리꼴 ABCD에 한 원이 내접한다. 빗변을 지름으로 하는 두 원이 서로 접함을 증명하여라.(Towns 1989봄 SO2)

99. 오각형 ABCDE는 내접원을 가지며 각선 AD와 CE가 내심 O에서 만난다. BO와 DE가 서로 수직임을 증명하여라. (Towns 1991봄 JA2)

100. 어떤 사다리꼴이 있고, 한 직선이 이 사다리꼴의 각 변, 각 각선과 6개의 점에서 만나는데, 교점으로 끊긴 이 직선의 5개의 선분의 길이가 모두 같다(윗변, 밑변과는 그 연장선에서 만난다). 이 사다리꼴의 윗변과 밑변의 길이의 비를 구하여라. (Towns 1988봄 SA1)

101. 4ABC의 수심을 N이라 하자. 4ABN , 4ACN , 4BCN의 외접원들은 모두 같은 크기임을 증명하여라.(Towns 1988가을 SO2)

102. 반지름이 각각 1, r, r인 세 원이 서로 외접하고 있다. 이 세 원에 외접하는(즉, 세 원을 내부에 포함하고,

각 원이 두 변에 접하며, 각 변도 두 원에 접하는) 삼각형이 존재하는 r의 값의 범위를 구하여라.(Towns 1991봄 JO2)

풀이 r의 크기를 연속적으로 움직여가며 관찰하자. r이 최소일 때는 r인 원과 1인 원의 공통외접선이

서로 평행에 이를 때의 직전. 즉, r > 12. r이 최 일 때는 두 공통외접선이 완전히 일치하여 한 직선을

이루게 될 때의 직전. 그 때의 상황은 피타고라스 정리로 (r + 1)2 = (r ¡ 1)2 + r2 일 때이고 구하면

r = 4. 즉, r < 4. ¢ ¢ ¢ 답 12< r < 4 }

103. 세 높이가 4, 7, 10인 삼각형은 존재하지 않음을 증명하여라. (IMTS R8-1)

증명 세 변의 길이는 x=4, x=7, x=10이고 1=4 > 1=7 + 1=10 이므로 삼각부등식 실패. ¤

104. 4ABC에서 \C의 이등분선이 외접원과 변 AB와 만나는 점을 각각 D, K라 하자. CA ¢CB = CK ¢CD

임을 보여라.

증명 \CDB = \A 이므로 4DBC » 4AKC. 따라서, CD : CB = CA : CK. ¤

105. 그림과 같이, 단위 정사각형을 7개의 닮은 직각삼각형으로 분할하는 실수 x (0 < x < 1)가 존재한다.

Page 150: 실전수학올림피아드 1400제 해답

150 기하

이와 같은 x는 최고차항의 계수가 1인 5차 다항식을 만족한다. 이 다항식을 구하여라. (IMTS R8-2)

풀이 직각삼각형들이 모두 닮았고 세 변의 길이의 비는 1 : x :px2 + 1. 이 비에 의해 오른쪽 변에 접

한 직각삼각형부터 차근차근 변의 길이들을 구해보면 윗변의 길이는 x + xx2+1

+ x(x2+1)2

= 1 이 됨.

(x2 + 1)2 곱해서 정리하면 x((x2 + 1)2 + (x2 + 1) + 1) = (x2 + 1)2, x5 + 3x3 + 3x = x4 + 2x2 + 1.

¢ ¢ ¢ 답 x5 ¡ x4 + 3x3 ¡ 2x2 + 3x¡ 1 }

106. 두 원이 두 점 M , N에서 만난다. M , N과 다른 점 A를 첫 번째 원 위에 잡는다. 직선 AM과 AN이 두 번째 원과 각각 점 B와 C에서 다시 만난다. 점 A에서 첫 번째 원의 접선이 BC와 평행함을 증명하여라.

(이탈리아 1986-1)

증명 접선 위에 점 K를 M쪽으로 하나 잡으면 \MAK = \MNA = \MBC. ¤

107. 삼각형의 세 변을 각각 p : 1¡ p, q : 1¡ q, r : 1¡ r 로 나누는 세 점을 잡자. 이 세 점을 이은 삼각형의 넓이를 구하여라. 단, 원래 삼각형의 넓이를 1로 한다.

풀이 pqr + (1 ¡ p)(1¡ q)(1¡ r). 이것은 칭식이어서 쓸모가 있을 수. 베트남 1980-3 의 보조정리

로 쓰이거나. 셈PS의 그림활용 부등식과 연관. }

108. 사각형의 세 내각의 이등분선이 한 점에서 만난다면, 네 내각의 이등분선이 모두 그 점에서 만남을 증명하여라. (IMTS R11-4)

증명1 (김지훈)

각 A, B, C의 이등분선이 한 점 P에서 만난다고 할 때 P와 D를 이었을 때 그 PD가 각 D의 이등분선임을 보이면 된다. P에서 AB, BC, CD, AD에 수선의 발을 내리고 순서 로 Q, R, S, T라 하자. 그러면 4AQP ´ 4ATP (RHA), 마찬가지로 4PQB ´ 4PRB 도 되고 또한 4PRC ´ 4PSC 도 성립.

따라서 PT = PS 이고 따라서 4TDP ´ 4SPD 도 성립(RHS). 따라서 \PDT = \PDS. ¤

증명2 (이광섭)

각 A, B, C의 이등분선이 한 점에서 만났다고 하자. DA와 CB의 연장선의 교점을 X라고 하고,문제에서의 교점을 I라고 하자. I는 삼각형 ABX의 외각의 이등분선의 교점이므로 방심이 되고, \AXI = \IXB.

따라서 XI는 \AXB의 이등분선이고, 삼각형 DXC에서 \BCI = \DCI 이므로 I는 삼각형 DXC의내심. 따라서 DI는 \ADC의 이등분선. DA k CB 일 때는 따로 간단히 확인해보면 역시 성립함. ¤

증명3 (김지욱)

사각형 ABCD에서 \A의 이등분선은 변 AB, 변 AD에서의 거리가 같은 점들의 집합이다. 따라서, 한점에서 만나는 세 내각의 이등분선들의 교점을 O라 하면 O에서부터 모든 변들까지의 거리는 같다. 그러므로 나머지 한 각에서의 이등분선은 그 각을 끼고 있는 변들에서의 거리가 같은 점들의 집합이다. 따라서 이 선은 O를 지난다. ¤

109. 삼각형 ABC가 주어져 있다. C를 지나고 \B의 이등분선과 평행한 직선이 \A의 이등분선과 점 D에서만난다. 또 점 C를 지나고 \A의 이등분선과 평행한 직선과 \B의 이등분선의 교점을 E라고 하자. 이 때,DE k AB 라면 CA = CB 임을 증명하여라. (소련 1963-9)

힌트 그냥 각 계산. or 원에 내접하는 사각형 }

110. S는 삼각형 ABC 내부의 점으로, 세 삼각형 ABS, BCS, CAS의 넓이가 모두 같다고 한다. S는 삼각형ABC의 무게중심임을 증명하여라. (헝가리 1936-2)

증명 꼭지점:변에 이르는 거리의 비가 2:1이므로 점이 유일하게 결정됨. 그럼 무게중심뿐. ¤

111. 평행사변형 ABCD에서, E는 AD의 중점이고 F는 B에서 CE로 내린 수선의 발이다. ABF가 이등변삼각형임을 보여라. (몰도바 1997 최종-y7-6)

증명 BC의 중점을 G라 할 때, AG는 4BCF의 중점연결선이 되어 BF를 수직이등분함. ¤

Page 151: 실전수학올림피아드 1400제 해답

3.1 기하 중급문제 151

112. ABC는 AB = AC 이고 \BAC = 30± 인 삼각형이다. BC에 한 A의 칭점을 A0; CA에 한 B의칭점을 B0; AB에 한 C의 칭점을 C0이라 하자. A0B0C0이 정삼각형임을 증명하여라.

(인도지역예선 1998-4)

증명 각 계산하고 직각이등변삼각형 몇 개 찾고 하면 각 변의 길이가 모두 ABp2. ¤

113. BE와 CF는 삼각형 ABC의 두 높이이고 O는 그 교점, 즉 수심이다. O를 지나는 임의의 직선 KL을 잡자. 단, K는 AB 위에, L은 AC 위에 있는 점이다. 또, KM ? BE 이고 LN ? CF 가 되도록 BE와 CF

위에 각각 점 M , N을 잡자. FM k EN 임을 증명하여라. (인도지역예선 2001-1)

증명 \FMO = \KFO = \NLO = \NEO ¤

114. 예각삼각형 ABC에서, 점 A, B, C에서 변에 내린 수선의 발을 각각 D, E, F라 하자. H를 수심이라할 때, 다음을 증명하여라. (호주 1993-6)

AH

AD+

BH

BE+

CH

CF= 2

증명 넓이로 따지면 간단. H 신 삼각형 내부의 임의의 점으로 해도 다 성립. 셈본 수준. ¤

115. 단위 길이의 두 선분 AB와 CD가 점 O에서 만나고 \AOC = 60± 이다. AC +BD ¸ 1 임을 보여라.(러시아 1993 최종-y9-2)

증명 BD를 평행이동해서 B0이 A와 만나도록 하면 정삼각형 한 변에 한 최단거리 부등식. ¤

116. 도시 A, B, C, D가 있는데, A에서의 거리가 D보다 C가 가깝고, B에서도 D보다 C가 더 가깝다. A에서B에 이르는 직선도로 위에 어떤 점에서도 D보다 C가 가까움을 보여라. (러시아 1994 4차-y9-2)

증명 CD의 수직이등분선 l에 해, A, B는 모두 C와 같은 쪽의 영역에 있음. 선분 AB도 그 영역에

있음(반평면은 볼록함). ¤

117. 삼각형 ABC에서 변 BC 위에 B;K;L;C의 순서가 되도록 두 점 K, L을 잡는데, BC ¢KL = BK ¢ CL

이고 AL이 \KAC의 이등분선이 되도록 하였다. AL ? AB 임을 증명하여라. (인도지역예선 1995-1)

증명 XK k AC 가 되도록 AB 위에 점 X를 잡자. XK=AC = BK=BC = KL=CL = AK=AC 이므

로 XK = AK. 이등변삼각형이므로 \XKA의 이등분선 KH가 AB와 직교하고 KH는 AL과 평행하니까. ¤

118. D는 삼각형 ABC의 선분 BC 위의 점이고, DC = 2BD 를 만족한다. \ABC = 45±, \ADC = 60± 일때, 삼각형 ABC의 세 각의 크기를 모두 구하여라. (유고슬라비아 1980 고1-3)

풀이 A에서 BC에 내린 수선의 발을 H라 하고, BD = 1, DH = x 라 할 때, x = 1p3¡1 =

p3+12. 싸

바싸바 계속 계산해서 구하면 ¢ ¢ ¢ 답 60±, 45±, 75± }

별해 C에서 AD에 내린 수선의 발을 O라 하면 OA = OB = OC 를 확인할 수 있음. }

119. 삼각형 ABC에서 P는 무게중심이고 M과 N은 각각 AC, AB의 중점이다. 만약 사각형 ANPM에 내접하는 원이 존재한다면 4ABC는 이등변삼각형임을 증명하여라. (남미 1987-2)

증명 AM + PN = AN + PM 이랑 파푸스의 정리 두 개를 함께 연립하여 풀면... ¤

120. 사각형 ABCD에서 AB는 CD와 평행하고 각선 AC와 BD는 서로 수직이다. 다음을 보여라: (1)AD ¢BC ¸ AB ¢ CD (2) AD +BC ¸ AB + CD (인도지역예선 1997-4)

증명 충 제곱해서 피타고라스로 정리하고 인수분해. (2)는 제곱하면 (1)과 동치 ¤

Page 152: 실전수학올림피아드 1400제 해답

152 기하

121. 평면 위의 서로 만나지 않는 두 원 k1, k2의 중심을 각각 O1, O2라 하자. 한 공통외접선이 k1과 A에서,k2와 B에서 접한다. 선분 O1O2는 k1, k2와 각각 C, D에서 만난다.

(a) 네 점 A, B, C, D가 한 원 위에 있음을 보여라.

(b) 직선 AC와 BD가 서로 수직임을 보여라. (몰도바 1996 최종-y11-3)

증명 \AO1C = 2®, \BO2D = 2¯ 라 하면 ®+ ¯ = 90±. 따라서, \CAB + \DBA = ®+ ¯ = 90±

로 (b)가 확인되고, \O1CA = ¯ = \DBA 로 (a)도 확인됨. ¤

122. 삼각형 ABC의 변 BC 위에 BM =MN = NC 를 만족하는 두 점 M , N을 잡자. AC에 평행한 한 직선이 직선 AB, AM , AN과 만나는 점을 각각 D, E, F라 하자. EF = 3DE 임을 보여라. (호주 1994-1)

증명 확 -축소변환에 의해, AC에 평행한 직선이 B를 지난다고 가정할 수 있다. 그럼 삼각형의 닮음

에서 BD : CA =MB :MC = 1 : 2, CA : BF = NC : NB = 1 : 2 이므로 BD : BF = 1 : 4. ¤

별증 평행사변형의 한 꼭지점에서 두 변에 그은 두 중선이 각선을 3등분함을 이용해도 같은 그림

에서 간단. ¤

123. 삼각형 ABC의 중선 BM(혹은 그 연장선) 위에 AN = BC 가 되도록 점 N을 잡자. 직선 AN이 BC와만나는 교점을 K라 할 때, BK = KN 임을 증명하여라. (몰도바 1999 최종-y7-4)

증명 ABCD가 평행사변형이 되는 점 D를 잡으면 AN = AD 이고 4AND » 4KNB. ¤

124. P를 이등변삼각형 ABC의 밑변 BC 위의 임의의 점이라 하자. P에서 AB, AC 위로 내린 수선의 발을각각 D, K라고 하고, 선분 AD와 AK 위에 2MD =MB 이고 2NK = NC 인 점 M과 N을 각각 잡자.MC = NB 임을 증명하여라. (몰도바 1999 최종-y7-8)

증명 DM = DB, KN = KC 이므로 4PBM » 4PCN » 4ABC 이고 4BPN ´ 4MPC ¤

125. \A가 직각인 직각삼각형 ABC에서, 한 원이 중선 AD와 A에서 접하고 있다. 그 원은 직선 BC와 두 점M , N에서 만난다. 이 삼각형의 한 옆변이 \MAN을 이등분함을 증명하여라.

(몰도바 1999 최종-y8/9-8)

증명 \DBA = \DAB 와 \NMA = \NAD 를 변변 빼면 끝. ¤

126. 두 원 S1과 S2가 점 F에서 서로 외접한다. 두 원의 한 공통외접선이 S1과 A에서 접하고 S2와 B에서 접한다. AB에 평행하고 S2와 C에서 접하는 직선이 S1과 두 점 D, E에서 만난다. A, F , C가 한 직선 위에있음을 보여라. (러시아 1994 최종-y9-2)

증명 AO2 k CO1 이므로 \AO2F = \CO1F , \AFO2 = \CFO1. ¤

127. ABC는 \C = 90± 이고 CA 6= CB 인 삼각형이다. CH는 수선, CL은 각의 이등분선이라 하자. 직선 CL

위의 C가 아닌 임의의 점 X에 하여 \XAC 6= \XBC 임을 보여라. 또한 직선 CH 위의 C가 아닌 임의의 점 Y 에 하여 \Y AC 6= \Y BC 임을 보여라. (주니어발칸 2001-2)

증명 (1) \XAC = \XBC 이면 4ACX ´ 4BCX (ASA). (2) WLOG BC > AC 라 하면 BY >

AY . 그럼 \CBA > \CAB 이고 \Y BA > \Y AB 라서 \Y BC > \Y AC. ¤

128. 정사각형 ABCD 내부에 점 M이 있다. 삼각형 ABM , BCM , CDM , DAM의 무게중심들이 정사각형의 네 꼭지점이 됨을 증명하여라. (Towns 1983가을 J1)

증명 M을 중심으로 3=2배 확 하면 중점연결사각형. 변환기하 ¤

129. 정삼각형 내부의 한 점 P로부터 세 꼭지점에 이르는 거리가 각각 3, 4, 5라고 한다. 이 정삼각형의 넓이를구하여라. (아일랜드 1998-2, 한국 1993-5 변형)

Page 153: 실전수학올림피아드 1400제 해답

3.1 기하 중급문제 153

풀이 P에 의해 나뉘어진 세 조각을 각각 한 꼭지점을 기준으로 60± 회전시키면 원래 삼각형의 넓이의

2배가 되는 육각형이 얻어지는데, 그 육각형의 넓이가 S(3; 3; 3) + S(4; 4; 4) + S(5; 5; 5) + 3S(3; 4; 5) 가

됨(S(a; b; c)는 세 변의 길이가 a, b, c인 삼각형의 넓이). ¢ ¢ ¢ 답 508

p3 + 9 }

130. CA = CB 인 삼각형 ABC의 외접원의 호 AB 위(직선 AB에 하여 C의 반 쪽)에 점 P가 있다. C에서 PB에 내린 수선의 발을 D라 할 때, PA+ PB = 2 ¢ PD 임을 보여라. (주니어발칸 2002-1)

증명 4CAP 를 C를 중심으로 회전하여 CA0과 CB가 일치하도록 하면 CPP 0은 이등변삼각형이고2 ¢ PD = PP 0 = PB +AP 0 = PB +AP . ¤

131. 두 원 S1, S2가 두 점 A, B에서 만나고 S2의 중심 O가 S1 위에 있다. S1의 현 OC가 S2와 점 D에서 만난다(D는 O와 C 사이의 점이다). D가 삼각형 ABC의 내심임을 보여라. (러시아 1990 4차-y10-6)

증명 \DAB = 12\DOB ´ 1

2\COB = 1

2\CAB 등. ¤

132. 삼각형 ABC에서 \B의 이등분선이 AC와 D에서 만나고 \C의 이등분선이 AB와 E에서 만난다. 이 두이등분선은 점 O에서 만나고 OD = OE 가 성립한다. \A = 60± 이거나 ABC가 이등변삼각형임을 증명하여라. (Towns 1984가을 JO1)

증명 D, E를 각각 이등분선에 해 칭시켜 BC 위에 얻은 점을 D0, E0이라 할 때 ID0E0이 이등변

삼각형임에서 ABC도. 만일 D0 = E0 이면 각 계산 좀 해보면 \A = 60±. ¤

133. ABCD는 AB k CD 와 AB + CD = AD 인 사각형이다. AD 위에 AP = AB 와 PD = CD 인 점 P를잡자.

(1) \BPC = 90± 임을 증명하여라.

(2) BC의 중점을 Q라 하고, BAQ의 외접원과 직선 AD의 교점을 R이라 하자. 네 점 B, P , R, C가 한원 위에 있음을 증명하여라. (중미 2004-5)

증명 (1) PX k AB 라 하면 PB, PC는 \APX, \DPX의 이등분선. (2) \QPR = \QRP 로

QPR은 이등변삼각형인데, Q는 직각삼각형 BPC의 외심이므로. ¤

134. 4ABC에서 \B = \C = 40± 이다. \B의 이등분선이 AC와 만나는 점을 D라 하자. BD +DA = BC

임을 증명하여라. (Towns 1984가을 SO3)

증명 A를 BD에 칭시킨 점을 A0, 이것을 다시 D에서 BC에 내린 수선을 축으로 칭시킨 점을

A00로 하면. ¤

135. 어떤 삼각형의 한 중선, 한 각의 이등분선, 한 수선이 내부의 한 점 O에서 만난다. 이 중 각의 이등분선의꼭지점으로부터 O까지 연결한 선분의 길이가 수선의 꼭지점으로부터 O까지 연결한 선분의 길이와 같다고 한다. 이 삼각형은 정삼각형임을 증명하여라. (Towns 1985봄 JO1)

증명 중선이 수선이 됨을 확인하면 됨. ¤

136. 사각형 ABCD가 AB = BC = l, \ABC = 100±, \CDA = 130± 를 만족한다. BD의 길이는 얼마인가?(Towns 1985봄 SO1)

풀이 100±=2 = 180± ¡ 130± 이므로, B를 중심으로 하고 A, C를 지나는 원 위에 D도 있음. ¢ ¢ ¢ 답l }

137. O를 중심으로 하는 원의 한 지름 AB와 그에 수직인 현 CD가 있다. 현 AE가 반지름 OC를 이등분할 때,

직선 DE가 현 BC를 이등분함을 보여라. (러시아 1995 최종-y9-2)

증명 \CAB = \CDB, \AOC = \DBC 로 4AOC » 4DBC (AA닮음). \CAE = \CDE 이므

로 4AOC 및 그 중선을 함께 C를 중심으로 회전확 하여 4DBC에 겹쳐주면. ¤

Page 154: 실전수학올림피아드 1400제 해답

154 기하

138. 예각삼각형 ABC의 변 BC, CA, AB 위에 각각 점 D, E, F가 있고

CD

CE=

CA

CB;

AE

AF=

AB

AC;

BF

BD=

BC

BA

를 만족한다. AD, BE, CF가 삼각형 ABC의 수선들임을 증명하여라. (인도지역예선 2002-1)

증명 문제의 조건에 의해 ABDE, BCEF , CAFD cyclic. 원주각에 의해 \BEF = \BCF (=

DCF ) = \DAF (= DAB) = \DEB 이므로 직선 BE는 삼각형 DEF의 한 내각의 이등분선. AD,CF도 마찬가지. 문제의 조건에서 또 \AEF = \B = \CED 이므로 BE는 \AEC의 이등분선, 즉 수선. AD, CF 등도 마찬가지. ¤

139. 정사각형 ABCD의 변 AB, CD 위에 각각 점 K, L을 잡고, 선분 KL 위에 점 M을 잡자. 4AKM과4MLC의 외접원이 다시 만나는 점은 각선 AC 위에 있음을 보여라. (Towns 1986봄 J4)

증명 \KAN = \KMN = \NCL. 정사각형일 필요는 없고 평행선 AB k CD 이기만 하면 충분. ¤

140. 볼록오각형 ABCDE에서, 삼각형 ABE, BCE, CDE, DAE의 무게중심을 각각 P , Q, R, S라 하자.PQRS가 평행사변형임을 보이고 그 넓이는 ABCD의 넓이의 2=9임을 보여라. (중미 2000-3)

증명 E를 중심으로 PQRS를 32배 확 . ¤

141. 삼각형 ABC에서 변 BC의 중점을 D라 하고, 변 AC 위에 BE = 2AD 를 만족하는 점 E가 있다. BE와AD의 교점을 F라 할 때 \FAE = 60± 라 한다. \FEA의 크기를 구하여라. (중미 2002-4)

풀이 BEE0B0을 E0 2 AC, D 2 B0E0 인 평행사변형이라 하자. 4DBB0 ´ 4DCE0 (ASA) 이므로AD = DE0. \FEA = \DE0A = 60±. }

142. 두 원이 두 점 P와 Q에서 만난다. P를 지나는 한 직선이 두 원과 각각 A와 A0에서 다시 만난다. 그와 평행한 Q를 지나는 직선이 두 원과 각각 B와 B0에서 다시 만난다. 두 삼각형 PBB0과 QAA0의 둘레의 길이가 같음을 보여라. (중미 2003-4)

증명 4PBB0 ´ 4QAA0 ¤

143. A와 B는 원 T 위의 서로 다른 점이다. BC는 B에서의 T의 접선이고 AB = AC 이다. \ABC의 이등분선이 AC와 점 D에서 만난다고 하자. D가 T의 내부에 있다면, \ABC > 72± 임을 보여라.

(아일랜드 2004-7)

증명 AC가 원 T와 다시 만나는 점을 E라 하고, \ABC = 2a 로 두자. 4ABC의 내각의 합과 접현각

에서 180± ¡ 4a = \BAC = \EBC < \DBC = a. 즉, 5a > 180±, 2a > 72±. ¤

144. 예각삼각형에서 세 수선의 발을 서로 연결하여 작은 삼각형을 얻었다. 이 삼각형이 두 변의 원래 삼각형의 변과 평행하다고 한다. 그럼 세 번째 변도 역시 평행임을 증명하여라. (Towns 1987봄 JO2)

증명 체바의 정리에 의해 평행변에 직교하는 수선의 발은 중점. 즉, 수심과 무게중심이 동일하게 됨.¤

145. 두 점 A, B에서 만나는 두 원이 있다. A에서의 두 원의 접선이 서로 직교한다. 한쪽 원 위에 있고 다른쪽원의 내부에 놓이는 임의의 점 M에 해, 직선 AM , BM이 나중의 원과 다시 만나는 점을 각각 X, Y 라하자. XY 는 지름임을 증명하여라. (레닌그라드 1987-26)

증명 M이 놓인 원의 중심을 O, 다른 원의 중심을 P라 하자. \XBY = \BMA¡\BXA = 12(360±¡

\BOA)¡ 12BPA = 180± ¡ 1

2(\BOA+ \BPA) = 90±. ¤

146. 중심이 각각 O, O0인 같은 크기의 원 ¡, ¡0의 두 교점 중 하나를 A라 하자. B는 ¡ 위의 한 점이고, AB가¡0과 다시 만나는 점을 C, 그리고 OBDO0이 평행사변형이 되도록 ¡0 위에 점 D를 잡자. CD의 길이는B의 위치에 상관없이 일정함을 보여라. (중미 2006-2)

Page 155: 실전수학올림피아드 1400제 해답

3.2 기하 고급문제 155

증명 OAEO0이 평행사변형이 되도록 하면 \BCD = \AED = \ABD 이므로 BD = CD. ¤

147. 예각삼각형 ABC의 외심을 O라 하자. \A의 이등분선이 BC와 D에서 만나고, D에서 AO에 내린 수선이 AC와 P에서 만난다. AB = AP 임을 보여라. (이탈리아 1995-4)

증명 \ABC = 12\AOC = 90± ¡ 1

2A = \APD 이므로 AD를 축으로 선 칭합동. ¤

148. 지름 AB가 원을 두 반원으로 나눈다. 점 P1; P2; : : : ; Pn이 한 반원 위에 이 순서 로 배열되어 있다. 다른 반원 위에서 점 C를 잡는데, 삼각형 CP1P2; CP2P3; : : : ; CPn¡1Pn 들의 넓이의 합이 최 가 되게 하려면 C는 어떤 점이어야 하는가? (폴란드 1970 3차-1)

풀이 P1P2 ¢ ¢ ¢Pn의 넓이는 일정하고, 삼각형 PnP1C의 넓이가 최 가 되도록 하면 됨. 즉, C에서의

접선이 P1Pn에 평행할 때. 혹은 P1Pn의 수직이등분선이 다른 반원과 만나는 점을 C. }

149. 점 O를 중심으로 하는 한 원의 내부에 점 A가 주어져 있고, 이 점을 지나는, 지름이 아닌 현 PQ가 있다.

점 P , Q에서의 원의 접선을 각각 p, q라 하자. 점 A를 지나고 OA에 수직인 직선 l이 p, q와 각각 점 K,

L에서 만난다. AK = AL 임을 보여라. (폴란드 1993/1994 1차-4)

증명 OAKP , OAQL cyclic. 4OAK ´ 4OAL. ¤

150. AB를 긴 밑변으로 하는 사다리꼴 ABCD에서 각선 AC와 BD는 서로 수직이다. 삼각형 ABC의 외심을 O라 하고, 직선 OB와 CD의 교점을 E라 하자. BC2 = CD ¢ CE 임을 증명하여라.

(이탈리아 1998-4)

증명 \CEB = \ABE = 90± ¡ 12\AOB = 90± ¡ \ACB = \CBD 이므로 4CBD » 4CEB ¤

3.2 기하 고급문제

1. 컴퍼스만을 사용하여(직선자는 사용하지 말 것) 수직으로 만나는 2개의 원을 그려라. 한 평면 위의 두 원이 수직으로 만난다는 것은 두 원의 교점에서 그은 각각의 접선이 서로 직교한다는 것을 뜻한다.

(IMTS R13-5)

풀이 X를 중심으로 하고 Y 를 지나는 원을 원 X(Y )로 쓰기로 하자. 우선 원 O(A)를 그리자. 원

A(O)를 그려 원 O와 만나는 한 점을 B라 하고, 원 B(O)를 그려 원 O와 만나는 A가 아닌 점을 C라 하고, 원 C(O)를 그려 원 O와 만나는 B가 아닌 점을 D라 하자. 그럼 OAB, OBC, OCD들은 정삼각형들이고, AD는 원 O의 지름이다.

그럼 지름의 원주각으로 AC ? CD 이므로, 원 A(C)와 원 D(C)를 그리면 이 두 원의 접선 CD와 CA는서로 직교한다. }

주 두 원을 똑같은 크기가 되도록 할 수도 있다. 위의 풀이에서처럼 O, A, B, C, D를 잡고, 원 A(C)와

원 D(B)의 한 교점을 E라 하자. OA = r 이라 할 때, AE = DE = AC = DB =p3 r 이므로

OE =p2 r 이다. 따라서, E를 중심으로 하고 반지름 r인 원을 그리면 원 O와 직교한다(두 중심과 두 교

점으로 이루어진 사각형이 한 변의 길이가 r인 마름모인데 각선의 길이가p2 r이므로 정사각형이다).

Page 156: 실전수학올림피아드 1400제 해답

156 기하

2. 원 O 위에 이 원의 중심 O를 지나지 않는 고정된 현 AB가 주어져 있다. XY 가 움직이는 지름일 때, 직선AX와 직선 BY 의 교점 P의 자취를 구하여라. (캐나다 1973-6)

풀이 P를 원의 안쪽에 생기는 점, P 0을 원의 바깥에 생기는 점이라 하자.

현 AB의 원주각을 ®라 하면

\APB = 90± + ®

\AP 0B = 90± ¡ ®

로 일정하고 두 각의 합이 180±이므로 A, P , B, P 0은 한 원 위에 있다. 즉, 위와 같은 원주각에 의한 원이 점 P의 자취가 된다. }

3. 직선 MN은 원 O와 A에서 접하고, 현 BC는 원 O의 지름 AD와 점 Q에서 만난다. 또, P는 PB ? BC

를 만족하는 직선 MN 위의 점이다.

(1) \CAN = 27±, \ACQ = 33± 일 때, \PBO의 크기를 구하여라.

(2) PQ ¢ AC = AQ2 +BQ ¢ CQ 임을 보여라. (1995 서울시)

풀이 (1) \BOC = \BOA+ \AOC = 2\ACB + 2\CAN = 2(27± + 33±) = 120±.(2) 방멱에 의해 AQ2 +BQ ¢CQ = AQ2 +AQ ¢QD = AQ ¢AD 이므로, 준식은 PQ ¢AC = AQ ¢AD,즉

PQ : QA = DA : AC

와 동치이다. 따라서 4PQA » 4DAC 임을, 이것은 직각삼각형이므로 \APQ = \CDA 임을 보이면충분하다. PBQA가 PQ를 지름으로 하는 원에 내접하므로, \APQ = \ABQ = \ADC 가 잘 성립한다. }

4. 삼각형의 꼭지점과, 내접원과 변의 접점을 이은 직선은 한 점에서 만남을 보여라.(통신강좌 1993-6-16)

풀이 A;B;C의 변에 한 접점을 각각 D;E; F라 하자.그러면 EA = AF; FB = BD;DC = CE이

다. 따라서AF

FB¢ BD

DC¢ CE

EA=

AF

BD¢ BD

CE¢ CE

AF= 1

이고 체바의 정리에 의해 AD;BE;CF는 한 점에서 만난다. }

5. 주어진 삼각형 ABC의 변 AC와 BC에 해 이 삼각형의 바깥쪽으로 평행사변형 AA0C0C와 BB00C00C를각각 그린다. 선분 A0C0과 B00C00의 연장선의 교점을 P라 하고, 선분 CP와 평행하면서 길이도 같도록 변AB의 바깥쪽에 선분 AP 0과 BP 00을 잡는다. 이 때

j¤ABP 00P 0j = j¤AA0C0Cj+ j¤BB00C00Cj

가 성립함을 보여라. 단, jSj는 도형 S의 넓이를 뜻한다. (셈본중등초급 도전문제 4.1.2)

Page 157: 실전수학올림피아드 1400제 해답

3.2 기하 고급문제 157

증명 CP가 AB와 만나는 점을 X라 하자.

등적변형으로j4ACC0j = j4ACP j = j4AP 0Cj = j4AP 0Xj

마찬가지로 j4BCC00j = j4BB00Xj. 따라서,

j¤ABP 00P 0j = 2j4AP 0Xj+ 2j4BB00Xj= 2j4ACC0j+ 2j4BCC00j= j¤AA0C0Cj+ j¤BB00C00Cj

가 성립한다. ¤

6. 삼각형 ABC에서, A와 B를 지나는 한 원이 변 AC, BC와 각각 D, E에서 만난다. 직선 AB와 DE는F에서 만나고 직선 BD와 CF는 M에서 만난다. MF =MC 이면, 또 그 때만 MB ¢MD =MC2 임을증명하여라. (미국 2003-4)

단계1 MF =MC 이면 MB ¢MD =MC2 임을 보이자.

DM의 연장선 위에 MD =MD0인 점 D0을 잡고 D0F , D0C, AE를 연결하자. 사각형 DCD0F는 평행사변형이다. D0F==AC, EF==D0C 삼각형 ADE와 삼각형 FD0C는 B를 닮음의 중심으로 닮음.

) AE==FC; \DBE = \EAD = \ACF

삼각형 BMC와 CMD는 닮음.

MD :MC =MC :MD; MB ¢MD =MC2

¤

단계2 MF = MC 일 때만 MB ¢MD = MC2. 즉, MB ¢MD = MC2 이면 MF = MC 임을 보이

자.MB :MC =MC :MD 이므로 삼각형 MBC와 MDC가 닮음. AE를 연결하면 \MBC = \AEC =\MCA 이므로

AE==MC; \EFM = \FEA = \ABM

따라서 삼각형 FBM과 FMD도 닮음.

) FM2 =MD ¢MB

즉, FM2 = CM2, MF =MC. ¤

별증 (박경태) ()) 체바의 정리에 의해서 FA=AB = EC=BE. 잘 정리하면 FA=EC = AB=BE. 가

비의 리에 의해서 FA=EC = AB=BE = FB=BC. 따라서 4BAE » 4BFC. 즉, AE k FC. 그러면\EBD = \EAD(원에 내접)= \ACF (엇각) 이므로 4MDC » 4MCB. 성립함.(Ã) 체바의 역정리 쓰면 됨. ¤

7. 원 O에 내접하고 \A < \B 인 예각삼각형 ABC를 생각하자. 원 외부의 어떤 점 P가

\A = \PBA = 180± ¡ \PCB

를 만족시킨다고 하자. 직선 PB가 원 O와 만나는 B가 아닌 점을 D라 하고, 점 A에서 원 O에 접하는 접선이 직선 CD와 점 Q에서 만난다고 하자. 이 때, CQ : AB = AQ2 : AD2 임을 보여라. (한국 2003-J3)

Page 158: 실전수학올림피아드 1400제 해답

158 기하

증명

\A = \PBA 이므로 \A = \DBA = a. 점 A는 접점이므로,

QA2= DQ ¢ CQ | ①

점 A;B;C;D는 한 원 위에 있으므로 \CAB = \CDB = a. a = \ABD 이므로 AB k CD.

) \DAB = \ADQ | ②

또한 A는 접점이므로 4ABD에서 \ABD = a = \DAQ이다. | ③②, ③에 의해서 4ABD » 4DAQ (AA닮음)

) AD : AB = DQ : AD

) AD2= AB ¢DQ | ④

①, ④에 의해서

AQ2: AD

2= DQ ¢ CQ : AB ¢DQ

= CQ : AB

가 성립한다. ¤

8. 직사각형 ABCD에서 AC의 길이는 e라 하고 점 A, B, C, D를 중심으로 하고 반지름이 각각 a, b, c, d인네 원이 있다. 이 때 a + c = b+ d < e 를 만족한다. 원 A, C의 두 공통 외접선과 원 B, D의 두 공통 외접선을 변으로 하는 사각형에 내접하는 원을 그릴 수 있음을 증명하여라. (소련 1961-2)

풀이 직사각형의 중심 O에서 각 공통외접선까지의 거리가 일정: r = (a+ c)=2 = (b+ d)=2. }

9. A, B, C, D가 다음을 만족하는 공간 위의 네 점이라 할 때,

\ABC = \BCD = \CDA = \DAB =¼

2

A, B, C, D는 한 평면 위에 있음을 증명하여라. (캐나다 1976-6)

증명 A에서 평면 BCD에 내린 수선의 발을 H라 하자.

Page 159: 실전수학올림피아드 1400제 해답

3.2 기하 고급문제 159

AB;AH ? BC 이므로 삼수선의 정리에 의해 HB ? BC, 마찬가지로 HD ? CD 이다. 따라서,HBCD는 직사각형. BAD와 BHD가 모두 직각삼각형이고 AHB와 AHD도 직각삼각형이므로, 피타고라스의 정리에 의해

BD2 = AB2 +AD2

= (AH2 +HB2) + (AH2 +HD2)

= 2AH2 + BD2

즉, AH = 0 으로 A = H. 따라서, 네 점은 한 평면 위에 있다. ¤

별증 피타고라스의 정리에서

AC2 = AB2 +BC2 = CD2 +DA2

BD2 = BC2 + CD2 = DA2 +AB2

따라서,

AC2 =1

2(AB2 +BC2 + CD2 +DA2) = BD2

이로부터 비슷한 방식으로 하여

AC = BD; AB = CD; AD = BC

임을 알 수 있다. 한편, B는 A를 직선 BC 위로 사영시킨 점이므로 두 평면 ADC와 BCD가 이루는 각을 µ라 하면

j4BCDj = j4ADCj cos µ이다. 4ADC ´ 4BCD 임에서 j4ADCj = j4BCDj 이므로 µ = 0 이어야 한다. ¤

10. 직선 `이 4ABC의 무게중심 G를 지난다. 각점 A, B, C에서 `에 그은 수선의 발을 X, Z, Y 라 하자.CY = AX +BZ 임을 보여라. 단, `은 AC, BC와 만난다. (통신강좌 1993-7-11)

풀이 D를 AB의 중점이라 하고, D에서 l에 그은 수선의 발을 Q라 하자.

GC : GD = 2 : 1이므로 CY = 2DQ

또, 사다리꼴 ABZX에서 D는 AB의 중점이므로, 2DQ = AX +BZ) CY = AX +BZ }

11. 단위원에 외접하는 사다리꼴에서 평행인 두 변의 길이를 x, y라 할 때, 부등식 x+ y ¸ 4 와 xy ¸ 4 가 성립하는지 각각 검토하고, 성립할 경우 등호 조건을 찾아라. (ML프로포절 147-3)

풀이 아래 그림처럼 x = AD = a+ b, y = BC = c+ d 라 두자.

직각삼각형 ABX에서 피타고라스 정리에 (a ¡ c)2 + 22 = (a + c)2 이 성립하므로, ac = 1 이 된다. 마찬가지로 bd = 1. 따라서, 산술-기하 평균 관계에 의해

x+ y = a+ b+1

a+1

b= a+

1

a+ b+

1

b¸ 2 + 2 = 4

xy = (a+ b)

µ1

a+1

b

¶= 2 +

a

b+

b

a¸ 2 + 2 = 4

로 두 부등식이 모두 성립함을 알 수 있다. x+ y = 4 일 등호조건은 a = b = 1 일 때이므로 정사각형일때, xy = 4 일 등호조건은 a = b 일 때이므로 등변사다리꼴일 때이다.

[주] \AOB = 12\EOF = \R 임으로부터 AG : GO = GO : GB, 즉 ac = 1 임을 알아낼 수도 있다. }

Page 160: 실전수학올림피아드 1400제 해답

160 기하

별해 x+ y ¸ 4 는 다음과 같이 증명할 수도 있다.

(1) x+ y = a+ b+ c+ d = AB+CD 이고, AB와 CD는 두 평행선 AD와 BC 사이를 잇는 선분이므로 각각 두 평행성의 거리 2보다 크거나 같다. 따라서, x+ y ¸ 2 + 2 = 4. 등호는 AB = CD = 2

일 때, 즉 AB와 CD가 높이가 될 때이므로 정사각형일 때이다.

(2) 사다리꼴 ABCD의 중단(높이의 12)을 가르는 직선을 그려, 내접원과 만나는 점을 E, F , 사다리꼴

의 옆변과 만나는 점을 G, H라 하자.

그럼 EF는 원의 지름이고, 사다리꼴이 원에 외접하므로 G, H는 원의 외부에 있어서 GH ¸ EF

이다. 따라서, x+ y = 2GH ¸ 2EF = 4. 등호는 GH = EF 일 때, 즉 옆변이 중단에서 접할 때이므로 정사각형일 때이다.

}

12. 삼각형 ABC에서 변 BC의 중점을 M이라 하고, P와 R을 각각 변 AB와 AC 위의 점이라 하자. AM과PR의 교점 Q가 선분 PR의 중점이라면, PR과 BC가 평행임을 증명하여라. (호주 1991-7)

풀이 PR0 k BC 인 점 R0을 변 AC 위에 잡고 Q0은 PR0의 중점이라 하면, 중점연결정리에 의해

QQ0 k RR0, 즉 AM k AC 가 되어 모순. }

13. 한 직선 l과 이 직선 밖의 한 점 P가 주어져 있다. 이 때, 자 없이 컴퍼스만을 이용하여, PQ가 l과 평행하게 되는 점 Q를 하나 작도하여라.

풀이 마름모법 }

14. 고정된 원 C와 그 중심 O를 지나는 직선 L이 있다. 직선 L 위를 움직이는 점 P를 중심으로 하고 점 O를지나는 원 K를 그린다. 원 C와 원 K의 공통접선이 원 K와 만나는 점을 T라고 할 때, 점 T의 자취를 구하여라. (소련 1962-2)

풀이 Let the common tangent meet C at S. Let X be the intersection of C and OP lying betweenO and P . PT = PO, hence \POT = \PTO, so \OPT = 180± ¡ 2\POT . But PT and OSare parallel, because both are perpendicular to the common tangent. Hence \POS = 2\POT , so\SOT = \XOT . Hence TX is tangent to C, in other words T lies on the (¯xed) tangent to C at X.Conversely, it is easy to see that any such point can be obtained (just take P such that PO = PT ).Thus the required locus is the pair of tangents to C which are perpendicular to L. }

15. O를 중심으로 하는 어떤 원의 내부에 O와는 다른 점 A가 주어져 있다. 각 OPA가 최 가 되는 원 O의원주 위의 점 P를 모두 구하고 작도하여라. (캐나다 1977-2)

풀이 O, A를 지나고 원 O와 접하는 작은 원 Q를 생각하자.

접점을 P라 하면, 원 O의 원주 위에 있고 직선 OA에 해 같은 쪽 영역에 있는 점 X ( 6= P )는 모두 원Q 밖에 있으므로 \OXA < \OPA 로 원주각보다 작게 된다. 따라서, 이 P가 구하는 점이다. 이런 P는OA에 칭인 것까지 2개 있다. }

Page 161: 실전수학올림피아드 1400제 해답

3.2 기하 고급문제 161

주 OP가 원 Q의 지름이므로, 작도는 A에서 OA에 수선을 올려 원 O와 만나는 점을 찾으면 된다.

16. A, B, C를 한 평면 위의 세 점이라 하자. 원 k2와 k3이 A에서 공통접선을 갖고, k3과 k1이 B에서, k1과k2가 C에서 공통접선을 갖는 세 원 k1, k2, k3을 작도하여라. (헝가리 1924-3)

풀이 A;B;C의 외심 O를 작도하면 jOAj = jOBj = jOCj이다. 그러면 OB;OC를 접선으로 갖고

B;C를 지나는 원을 그릴 수 있고, 마찬가지로 C;A, A;B에 해서도 그리면 그 원들이 각각 k1; k2; k3가된다. }

17. M은 AB = BC 인 이등변삼각형 ABC의 외접원의 호 AC 위의 한 점이다. P는 이 호의 중심이고, N은현 BM의 중심이다. P에서 MC에 내린 수선의 발을 K라 할 때, 삼각형 ANK가 이등변삼각형임을 보여라. (통신강좌 1991-3-12)

풀이 \OAB = \PAC = \OBA = \PCA에서

4OAB v 4PAC (1)

이고, \ONB = \PKC, \OBN = \PCK (PM에 한 원주각)에서

4ONB v 4PKC (2)

이다.

(1), (2)에서KC

NB=

PC

OB=

AC

AB이고, \NBA = \KCA (AM에 한 원주각)이므로

4NAB v 4KAC (3)

이다. 다시 (3)에서 \NAB = \KAC이므로 \BAC = \NAK가 되고,AC

AB=

AK

AN이므로,

4BAC v 4NAK (4)

가 된다. 4ABC는 이등변삼각형이므로, (4)에서 4ANK도 AN = NK인 이등변삼각형이 된다. }

18. \B = \C 이고, \A > 60± 인 이등변삼각형 ABC의 외접원을 O라 하자. 꼭지점 A에서 원 O에 접하는접선을 `이라 하고, ` 위의 점 P가 \BPA = \BAC 를 만족시킨다고 하자. 선분 PC가 원 O와 만나는점을 E라 하고, 직선 BE가 접선 `과 만나는 점을 F라 할 때, AF = FP 임을 보여라. 단, E 6= C.

(한국 2002 1차-S3)

B

A

N O

MP

K

C

B

A

O

MP

K

C

N

Page 162: 실전수학올림피아드 1400제 해답

162 기하

증명1

직선 `과 원이 점 A에서 접한다. 할선의 정리에 의해서,

AF 2 = FE ¢ FB (1)

문제에서 주어진 것처럼 \BAC = \BPA 이고, \BAC와 \BEC는 같은 현을 공유하고 있으므로 크기가 같다. 또한, \BEC와 \FEP는 맞꼭지각이므로 같다. 결국, \PEF = \BPF 이고, 그러므로4EPF와 4PFB는 닮았다(\PFE는 공통). 즉, PF : FE = FB : PF 이고,

PF 2 = FE ¢ FB (2)

(1), (2)에서 AF = PF . ¤

증명2 \BPA = \BAC, \ABC = \BAP 이므로 4ABC » 4PAB 의 닮은 이등변삼각형이다.

PA = PB = 1, AB = AC = a 라 하면 BC = a2 = 2AD. 피타고라스 정리와 PA2 = PE ¢PC 로부터

PC =p2a2 + 1; PE =

1p2a2 + 1

또 4EFP » 4EBC 에서 PF : PF +BC = PE : PC = 1 : 2a2 + 1. 따라서

PF + a2 = PF ¢ (2a2 + 1); 즉 PF =1

2=1

2PA

그러므로 AF = FP =1

2PA 이다. ¤

19. 삼각형 ABC의 수심을 H, 변 AC의 중점을 M이라 하자. M을 지나 \AHC의 이등분선에 평행한 직선을 l이라 하자. l은 이 삼각형의 둘레를 이등분함을 증명하여라. (중미 2005-5)

증명 높이 AD, CE, 그리고 변 AB, BC가 l과 만나는 점을 각각 I, J , S, T라 하고, A를 지나 l에 평

행한 직선이 CE와 만나는 점을 G라 하자. J는 CG의 중점이고 AI = JG. 따라서, 4AIS ´ 4CJT .¤

20. A, B, C는 x; y-좌표평면의 세 점이고, 선분 AB, BC, CA의 중점을 각각 X, Y , Z라 하자. 또, \CPZ =\Y XZ 를 만족하는 직선 BC 위의 한 점을 P라 하자. AP와 BC가 직교함을 증명하여라. (호주 1991-3)

풀이 Z는 ZA = ZP = ZC 로 4APC의 외심. }

21. AB와 BC의 길이가 주어져 있을 때 A에서 그은 중선과 B에서 그은 중선이 직교하도록 삼각형 ABC를작도하여라. (소련 1962-6)

풀이 Let M be the midpoint of AB and X the midpoint of MB. Construct the circle center B,radius BC=2 and the circle diameter AX. If they do not intersect (so BC < AB=2 or BC > AB)then the construction is not possible. If they intersect at N , then take C so that N is the midpointof BC. Let CM meet AN at O. Then AO=AN = AM=AX = 2=3, so the triangles AOM and ANXare similar. Hence \AOM = \ANX = 90±. }

Page 163: 실전수학올림피아드 1400제 해답

3.2 기하 고급문제 163

22. 원 k가 삼각형 ABC의 변 BC, CA, AB 들과 점 A1, A2; B1, B2; C1, C2 에서 만난다. 각각 A1, B1,

C1을 지나는 BC, CA, AB의 수선들이 한 점 M에서 만난다. 각각 A2, B2, C2를 지나는 BC, CA, AB의수선들도 한 점에서 만남을 증명하여라. (헝가리 1914-3)

힌트 구하는 점은 원 k의 중심에 한 M의 칭점이다. }

증명 원 k의 중심을 O라 하자. O에서 세 변 BC, CA, AB에 내린 수선의 발은 각각 A1A2, B1B2,

C1C2의 중점이 된다.

따라서, MN의 중점이 O가 되도록 하는 점 N을 잡으면 사다리꼴의 빗변을 같은 비율만큼 늘린 것이되어 NA2, NB2, NC2는 각각 MA1, MB1, MC1에 평행이 된다. 즉, A2, B2, C2를 지나는 BC, CA,AB의 수선들은 모두 N을 지난다. ¤

23. AB, CD는 한 원의 서로 수직인 두 현으로 각각 길이가 2, 1이고 AC + BD = AD +BC 일 때 그 원의반지름의 길이를 구하여라. (통신강좌 1994-9-4)

풀이 조건에서

AC +BD = AD +BC (5)

이고 피타고라스 정리에 의해

AC2 +BD2 = AD2 +BC2 (6)

이다. (1)을 제곱하여 정리하고 (2)를 사용하면

AC ¢BD = AD ¢BC (7)

이다. 톨레미 정리에 의해

AC ¢BD +AD ¢BC = AB ¢ CD = 2 (8)

가 된다. (3)과 (4)로 부터 AC ¢BD = AD ¢BC = 1이 되고 (1)에 의해 AC + 1=AC = AD + 1=AD가되어 AC = AD 또는 AC = 1=AD = BC가 된다.X를 AB와 CD의 교점이라 하면 방멱에 의해 AX ¢BX = CX ¢DX이다. AC = BC이면 CD는 AB를이등분하고 AX = BX = 1 = CX ¢DX이다.

그러나,이것은 CX;DX 둘 다 1보다 작기 때문에 불가능하다.그러므로 AC = AD;CX = DX = 1=2이다. AB는 CD의 수직 이등분선이므로 지름이 된다. 따라서 반지름의 길이는 1이다. }

풀이 (과기원 기계과 96학번 이원철) 피타고라스 정리에 의해

AC2 +BD2 = AD2 +BC2

조건식과 위 식을 이용하여AC ¢BD = AD ¢BC

Page 164: 실전수학올림피아드 1400제 해답

164 기하

톨레미 정리에 의해AC ¢BD +AD ¢BC = AB ¢ CD = 2

즉, AC ¢BD = AD ¢BC = 1

조건식과 위 식을 합치면

AC +1

AC= AD +

1

AD

AC = AD or AC =1

AD= BC

X를 AB와 CD의 교점이라 하면 방멱에 의해 AX ¢BX = CX ¢DX이다.AC = BC이면 CD는 AB를 이등분하고 AX = BX = 1 = CX ¢DX이다.

그러나 이것은 CX, DX < 1이므로 불가능하다.

그러므로, AC = AD, CX = DX =1

2이다.

AB는 CD의 수직이등분선이므로 지름이다.

즉, 반지름의 길이는 1이다. }

24. 평면 위에 세 원이 있다. 각 원의 중심을 O1, O2, O3, 반지름을 r1, r2, r3이라 하자. 임의의 두 원이 두 점에서 만나 공통현을 갖는다고 하자.

(1) 세 공통현은 한 점 P에서 만남을 증명하여라.

(2) 각각의 원 Oi에서(i = 1; 2; 3), P를 지나고 POi와 수직인 현을 생각하자. 이런 세 현의 길이가 모두 같음을 보여라. (ML프로포절 141-1)

증명 (1)은 두 현 AB, CD의 교점이 E라 하면 AE ¢EB = CE ¢ED 가 성립한다는 것을 이용하면 된

다. (셈본중등고급에 이 문제 있다. 몬즈의 문제)

(2)는 원의 중심에서 어떤 현 AB에 수선의 발 M을 내리면 M이 AB의 중점이 된다는 것과 (1)에서 이용했던 것을 다시 이용하면 된다. ¤

주 이 문제는 지구과학에서 진원의 깊이를 잴 때의 원리로 사용되는 방법이다.

25. 삼각형 ABC의 내심을 I, 직선 AI가 삼각형 ABC의 외접원과 만나는 점을 D, 선분 BI의 중점을 P라 하자. CI = 2AI 이면 AB = PD 임을 보여라. 단, D 6= A. (한국 2002-J7)

증명 각 A, B, C의 크기를 각각 2o, 2u, 2x라 하자. 그리고 직선 CI가 원과 만나는 점을 F , ID의 중

점을 G라 하자.

이제 FAB와 GPD가 합동인 이등변삼각형임을 보이겠다.

\DBI = \DBC + \CBI = \DAC + \CBI = o+ u

\DIB = \IAB + \IBA = o+ u

로 DBI는 이등변삼각형이다. \BDI = \BDA = \BCA = 2x 이고 \GPD = \PDB = \GDP 이

므로, 4GPD는 밑각이 x인 이등변삼각형이다. 또한 \FAB = \FBA =1

2\ACB = x 로 4FAB도

밑각이 x인 이등변삼각형이다. DCI도 DBI와 마찬가지로 DI = DC 인 이등변삼각형이고 4ICD와4IAF의 닮음비가 CI : AI = 2 : 1 이므로,

DG =1

2DI =

1

2DC = FA

FAB와 GPD의 두 이등변삼각형이 합동임이 확인되었으므로, PD = AB. ¤

Page 165: 실전수학올림피아드 1400제 해답

3.2 기하 고급문제 165

주 선분 BC의 중점을 M이라 할 때 삼각형 PMD와 AIB가 합동임을 보여 해결할 수도 있다. 또, 선

분 IA의 중점을 E라 할 때 삼각형 AIC와 EPD가 1 : 2의 변을 갖는 닮은 삼각형임을 보여 해결할 수도 있다.

26. 삼각형의 외심, 무게중심, 수심은 일직선 위에 있음을 보여라.

증명 D;E는 각각 A;B에서 BC, AC에 내린 수선의 발이고, M;M 0은 각각 BC;AC 의 중점이다.

H를 4ABC의 수심, O를 외심이라 하면 H는 AD와 BE의 교점이고, OM ? BC, OM 0 ? AC가 된다.AM과 HO의 교점을 X라 하자. AD k OM 이므로 4AHX와 4OMX는 닮음이고, 따라서

HX : XO = AH : OM ①

같은 방법으로 밑의 그림에서,

BE k OM 0, 즉 4BHY » 4M 0OY 이고,

HY : Y O = BH : OM 0 ②

한편, AB k MM 0(중점연결정리), AD k OM이므로 \HAB = \OMM 0 이고, AB k MM 0, BE kOM 0 이므로 \HBA = \OM 0M 이다. 따라서 4HAB » 4OMM 0 이고,

AH : OM = BH : OM 0 = AB :MM 0(= 2 : 1)

이로부터 ①, ②에 의해 HX : XO = HY : Y O = 2 : 1, 즉 X = Y 이다. 이 점은 두 중선 AM과 BM 0의교점이므로 4ABC의 무게 중심이다. 따라서, 무게 중심은 수심과 외심을 이은 선분을 2 : 1로 내분하며, 세 점은 한 직선 위에 있다. ¤

27. \C = 2\B 인 삼각형 ABC가 있다. 이 삼각형의 내부에 BD = CD 이고 AC = AD 인 점 D를 잡자.\A = 3\BAD 임을 보여라. (호주 2003-3)

증명 \ACE = \ECB = \EBC = ®, \DCB = \DBC = ¯ 라 하자.

A

C B

E

α

β

α

β

Page 166: 실전수학올림피아드 1400제 해답

166 기하

4ACE » 4ABC (AA)) AC2 = AE ¢ AB = AD2

) 4ADE » 4ABD (SAS).) \ADE = ®¡ ¯

\ACD = \ADC = 2®¡ ¯.

) \CAD = 180± ¡ 2\ACD = 180± ¡ (4®¡ 2¯) (1)

\CAB = 180± ¡ \ACB ¡ \ABC = 180± ¡ 3® (2)

\CDE에 해,

90± + ¯ = (2®¡ ¯) + (®¡ ¯) = 3®¡ 2¯) ®¡ ¯ = 30± (3)

그런데,

(1)£ 3 = (2)£ 2 () 540± ¡ (12®¡ 6¯) = 360± ¡ 6®() 180± = 6(®¡ ¯)

() 30± = ®¡ ¯

이것은 (3)에 의해 성립.

) 2\CAB = 3\CAD

) \CAD =2

3\CAB =) \DAB =

1

3\CAB

) \CAB = 3\DAB

¤

28. 세 변과 한 수선의 길이가 네 개의 연속한 양의 정수이고 이 수선이 원래의 삼각형을 각 변의 길이가 정수인 두 직각삼각형으로 나누는 삼각형을 하나 구하여라. 이런 삼각형은 당신이 찾은 것이 유일함을 보여라. (캐나다 1993-1)

풀이 그림과 같이 각 변과 수선의 길이를 a = x+ y, b, c, h로 두자.

h < b; c 이므로, 일반성을 잃지 않고 b < c 라 하면, 크기 순서는 다음과 같이 네 가지 경우가 나온다:ahbc, habc, hbac, hbca. 피타고라스의 정리를 응용하면

c2 ¡ b2 = (h2 + y2)¡ (h2 + x2) = y2 ¡ x2 = (y + x)(y ¡ x)

가 성립하고, 위의 네 가지 경우 각각에 해 여기에 y+x = a, b, c 등을 모두 h에 관한 식으로 입하면

(h¡ 1)(y ¡ x) = 2h+ 3 (i)

(h+ 1)(y ¡ x) = 2h+ 5 (ii)

(h+ 2)(y ¡ x) = 4h+ 8 (iii)

(h+ 3)(y ¡ x) = 2h+ 3 (iv)

의 식을 각각 얻는다. (i)의 경우는 h > 6 이면 2(h ¡ 1) < 2h + 3 < 3(h ¡ 1) 이므로 해가 없고,h = 1; 2; 3; 4; 5; 6일 때도 직접 구해보면 아무 것도 해가 되지 않는다. (ii)의 경우는 h > 2이면 2(h+1) <2h + 5 < 3(h+ 1) 이므로 해가 없고, h = 1; 2 일 때도 직접 구해보면 해가 되지 않는다. (iv)의 경우는

Page 167: 실전수학올림피아드 1400제 해답

3.2 기하 고급문제 167

언제나 h + 3 < 2h + 3 < 2(h + 3) 이므로 해가 없다. 이제 (iii)의 경우만 남았다. 이 때 y ¡ x = 4, 즉y = x+ 4 이고, 이것을 이용해 모든 값을 x로 나타내면 다음의 왼쪽 그림과 같다.

왼쪽 직각삼각형에서 x2 = 4x+ 5, 즉 (x+ 1)(x¡ 5) = 0 이므로, x = 5 일 때 오른쪽 그림과 같은 삼각형이 유일하다. }

29. 직선 ` 위에 네 점 A, B, C, D가 주어져 있다. 이 때 정사각형 하나를 작도하여 그 두 변이(또는 그 연장선이) 각각 A와 B를 지나도록 하고, 또 나머지 두 변이(또는 그 연장선이) 각각 C와 D를 지나도록 하여라. (헝가리 1898-3)

힌트 CD를 90± 회전하여 한 꼭지점이 B에 붙도록 한다. }

30. 정삼각형 ABC의 내접원의 한 접선이 변 AB, AC와 각각 점 D, E에서 만난다.AD

DB+

AE

EC= 1 임을 증

명하여라. (폴란드 1995/1996 1차-4)

증명 j4ADEj = 12(AD + EA¡DE)r = 1

2AD ¢DE ¢ sin 60± 와 준식이 동치. ¤

31. 4ABC는 AB = AC 인 이등변삼각형이다. \B의 이등분선이 변 AC와 D에서 만나고, BC = BD+AD

가 된다고 한다. \A를 구하여라. (캐나다 1996-4)

풀이 BC 위에 BD = BE 가 되는 점 E를 잡자. 그럼 문제의 조건에 의해 EC = AD 가 된다.

각의 이등분선의 성질에 의해 AB : BC = AD : DC = EC : CD 이고 \C가 공통이므로 4ABC와4ECD가 닮았다. 따라서, 4ECD도 이등변삼각형. \B = 2® 라 두면 \BDE = \BED = \ECD +\ECD = \B + \B = 4®. 즉, 4BCD의 내각의 합에서 ® + 6® + 2® = 9® = 180± 이므로 ® = 20±

이고, 4ABC의 내각의 합에서 \A = 180± ¡ 4® = 100± ¢ ¢ ¢ 답 }

32. AB = AC 인 이등변삼각형 ABC의 변 AB, AC와 각각 P , Q에서 접하고 4ABC의 외접원에 내접하는원이 있다. 선분 PQ의 중점은 4ABC의 내심임을 증명하여라. (셈본중등중급 도전문제 4.2.3)

증명 T를 두 원의 접점이라 하고, M을 선분PQ의 중점이라 하자.접선과 현의 성질에 의하여 \BPT =

\PQT , 이등변삼각형의 칭성에 의하여 \BPT = \PQT = \QPT가 된다. \PBT = \PMT = ¼2

이므로 4PBT와 4PMT는 RHA 합동이다.

Page 168: 실전수학올림피아드 1400제 해답

168 기하

) \PBM = \PMB = \MBC

따라서 M은 4ABC의 내심이다. ¤

33. 정삼각형 ABC에서 변 BC와 평행한 직선이 변 AB, AC와 D, E에서 만난다. 선분 CD의 중점을 F라하고, 정삼각형 ADE의 중심을 G라 하자. 삼각형 BFG의 세 내각의 크기를 구하여라. (호주 2004-4)

풀이 (KAIST 과학영재센터 연구원 고봉균) 점 E, A, G를 F에 해 점 칭시킨 점을 각각 I, J , H라

하자.

그럼 (A;D;E;G)와 (C; I; J;H)는 그 순서 로 점 B에 해 60± 회전이동한 점이 된다. 따라서, BG =BH, \GBH = 60± 가 되고, 그럼 BHG는 정삼각형이다. F는 변 GH의 중점이므로, 4BFG 는 세 내각의 크기가 30±, 60±, 90±인 직각삼각형이다. }

34. AB = AC 인 이등변삼각형 ABC에서 변 BC의 중점을 D라 한다. \ABC의 삼등분선이 AD와 만나는점을 A로부터 차례 로 M , N이라 하고, CN의 연장과 AB의 교점을 E라 하면 EM==BN 임을 증명하여라. (1991 한국 계절학교 선발시험)

풀이 \BEC = 180± ¡ 43\ABD = \BMC이므로 ¤EBCM은 원에 내접한다. 따라서 \EMB =

\ECB = \MBN(원주각)이고 따라서 EM k BN이다. }

35. \A = 90±인 직각삼각형 ABC가 있다. \B와 \C의 이등분선이 AC, AB와 만나는 점을 각각 D, E라 하고, BD와 CE의 교점을 O라고 하면, 4BOC 의 넓이는 사각형 BEDC의 넓이의 절반이 됨을 보여라.

증명 CO에 한 D의 선 칭점을 D0, BO에 한 E의 선 칭점을 E0이라 하자. 그럼 두 삼각형

ODE와 OD0E0의 넓이가 같다. ¤

36. 중심이 각각 O1, O2인 원 S1, S2가 두 점 A, B에서 만난다. O1, O2, A를 지나는 원이 S1, S2, 직선 AB와각각 D, E, C에서 다시 만난다. CD = CB = CE 임을 보여라. (러시아 1995 4차-y9-6)

증명 \CBD = ® 라 하면 \AO1D = 2®, \ACD = 180±¡2®. \CDB = 180±¡\CBD¡\ACD =®. ¤

A

Q P

B T

M

C

Page 169: 실전수학올림피아드 1400제 해답

3.2 기하 고급문제 169

별증1 O2A = O2C 이므로 \O2O1C = \O2O1A = \O2O1B, 즉 O1BC는 일직선. 마찬가지로

O2BD도 일직선. 그럼, \EBC = \ABO1 = \EAO1 = \ECO1. ¤

별증2 \DO1C = \DAC = \DAB = 12\DO1B 이므로 O1C는 BD를 수직이등분. 마찬가지로

O2C도 BE를 수직이등분. C는 외심. ¤

37. 점 P는 이등변삼각형의 밑변 위의 점이다. 옆변과 평행하고, 점 P를 지나는 두 직선이 삼각형과 만나는P가 아닌 점을 각각 Q, R이라 하자. 점 P를 직선 QR에 칭이동시킨 점은 삼각형의 외접원 위에 있음을 증명하여라. (헝가리 1949-2)

증명 P의 칭점을 P 0이라 하면 AQRP 0 cyclic. 180± ¡ \P 0AQ = \P 0RQ = 12\P 0RP (Q쪽) =

\P 0BP . (R이 P 0BP의 외심이라서) ¤

38. 삼각형 ABC의 내부에 \PBC = \PCA < \PAB 를 만족하는 점 P가 있다. 직선 BP가 삼각형ABC의 외접원과 만나는 점을 E라 하고, 직선 CE가 삼각형 APE의 외접원과 만나는 점을 F라 하자. 사각형 APEF와 삼각형 ABP의 넓이의 비는 점 P의 선택에 관계없이 일정함을 증명하여라.

(폴란드 1996 3차-2)

증명 \PCA = \PBC = \EAC 이므로 AE k PC. 등적변형으로 j4APEj = j4ACEj, 즉

j¤APEF j = j4ACF j. 4ACF » 4ABC 이고 닮음비는 AB : AC 로 일정. ¤

39. 삼각형 ABC에서 I는 내심이고 D, E, F는 각각 내접원이 변 BC, CA, AB와 만나는 접점이다. 직선AD는 P에서 내접원과 다시 만난다. M이 EF의 중점이라면, 네 점 P , I, M , D가 한 원 위에 있음을 증명하여라. (남미 1990-2)

증명 AP ¢ AD = AF 2 = AI ¢ AM ¤

40. \A = 2\C 인 4ABC가 있다. 변 BC의 길이 x를 다른 두 변 AC, AB의 길이 b, c에 관한 식으로 나타내어라. (1994 서울시)

풀이 \A의 이등분선과 BC의 교점을 D라 하자. 그러면 BDCD

= cb이다. 따라서,

BD =cx

b+ c; CD =

bx

b+ c

가 된다. 한편 \BAD = \BCA이고 \ABC는 공통이므로 4ABD와 4CBA는 닮음이다. 닮음비에 의해 b : x = bx

b+c: c이고 여기서 x =

pbc+ c2이다. }

41. 평행사변형 ABCD의 내부에\AOB + \COD = 180±

가 되도록 점 O를 잡는다. 그럼 \OBC = \ODC 임을 보여라. (캐나다 1997-4)

증명 4AOB ´ 4DPC가 되도록 O를 평행이동한 점을 P라 하자.

\COD+\DPC = 180± 이므로 OCPD는 원에 내접하는 사각형이고, 따라서, 원주각과 평행사변형의각에 의해 \ODC = \OPC = \OBC. ¤

42. 임의의 삼각형에서 둘레의 길이를 L, 세 중선의 길이의 합을 M이라 할 때M

L>3

4임을 증명하여라.

(한국 1993-9)

Page 170: 실전수학올림피아드 1400제 해답

170 기하

증명 AG+GB > AB 등을 모두 변변 더하면 됨. ¤

43. 임의의 예각삼각형이 주어져 있다. 몇 개의 선분을 그려 이 삼각형을 세 영역으로 분할하는데, 각각의 분할된 영역은 선 칭도형(어떤 직선에 해 칭이동시켰을 때 자기 자신이 되는 도형)이 되도록 하려고한다. 이런 분할 방법이 항상 3가지 이상 있음을 보여라. (IMTS R1-4)

풀이 (1) 외심과 각 꼭지점을 이으면 3개의 이등변삼각형이 생긴다.

(2) 내심에서 각 변에 수선을 내리면 3개의 선 칭사각형이 생긴다.(3) AB, AC의 중점을 M , N이라 하고 A에서 BC에 내린 수선의 발을 H라 하자. 그럼 선 칭사각형AMHN과 두 개의 이등변삼각형이 생긴다. }

44. 직사각형 ABCD가 있을 때, 어떤 직선 `이 각 변 AB, CD, 그리고 AD, BC (또는 그 연장선) 들과 만나는 점을 각각 M , N , 그리고 P , Q라 하자. 네 점 M , N , P , Q와 변 AB의 길이 p가 주어져 있을 때, 원래의 직사각형을 작도하여라. 이 문제는 어떤 조건에서 해를 구할 수 있고, 또 얼마나 많은 해가 있는가?

(헝가리 1897-3)

힌트 AB 즉 p를 평행이동하여 한쪽 끝이 P 혹은 Q에 오도록 한다. }

45. 한 직선 위에 있지 않은 세 점 M , N , P가 있다. M , N은 어떤 삼각형의 두 변의 중점이고 P는 그 삼각형의 수심이다. 이 조건을 만족하는 삼각형을 작도하여라. (남미 1991-6)

풀이 M에 한 P의 칭점을 Q라 하자. QN을 지름으로 하는 원과 P에서 MN에 그은 수선의 교점

을 A라 하면 됨. }

46. N은 \BAC의 이등분선 위의 점이다. P와 O는 AB와 AN 위의 점으로 \ANP = 90± = \APO 이다. Q는 NP 위의 임의의 점이고 Q를 지나는 임의의 직선이 AB, AC와 각각 E, F에서 만난다. 이 때\OQE = 90± 와 QE = QF 는 동치임을 보여라. (IMO 1994-2)

증명 먼저 \OQE = 90±라고 가정하자. PN의 연장선이 AC와 R에서 만난다고 하자.그러면 OEPQ와

ORFQ는 각각 동일 원주상의 사각형이므로 4OEQ ´ 4OFQ가 되어 QE = QF를 얻을 수 있다.

이제 \OQE 6= 90±라고 가정하자. 점 O를 지나고 EF에 수직인 직선이 NP와 Q0 6= Q에서 만난다고하자. Q0을 지나고 EF에 평행한 직선은 AB;AC와 각각 E0; F 0에서 만난다고 하자. 그러면 위에서 한것처럼 Q0E0 = Q0F 0을 얻을수 있다. AQ0이 EF와 M 6= Q에서 만난다고 하자. 그러면 ME = MF가되어 QE 6= QF이다. 즉 QE = QF이면 \OQE = 90± ¤

47. 사다리꼴 ABCD의 두 각선의 교점을 M이라 하자. AD k BC, AB ? AD 이고 이 사다리꼴의 내접원이 존재한다. 이 내접원의 반지름을 R이라 했을 때 4DCM의 넓이를 구하여라.

(셈본중등고급 도전문제 4.1.3)

Page 171: 실전수학올림피아드 1400제 해답

3.2 기하 고급문제 171

풀이 내점 원과 AB, BC, CD, DA의 교점을 각각 M , N , P , Q라 하자.

이때, AM =MB = BN = AQ = R이고, DQ = DP = a, CN = CP = b라 놓자.D에서 BC에 내린 수선의 발을 D0이라 하면 피타고라스 정리에 의해

(a+ b)2 = (b¡ a)2 + (2R)2

) R2 = ab

S4DCM = S4BCD £DM

BD=1

2¢ 2R(R+ b)£ AD

AD +BC

= R(R+ b)£ R+ a

2R+ a+ b

= R ¢ R2 +R(a+ b)ab

2R+ a+ b= R ¢ R(2R+ a+ b)

2R+ a+ b= R2

}

48. 세 점 X, Y , Z가 주어져 있다. X를 외심으로 하고, BC의 중점이 Y 이고, BZ가 한 수선이 되는 삼각형ABC를 작도하는 방법을 찾아라. (아일랜드 1991-1)

풀이 직선 BC를 그리고 원 Y (Z)를 그려 교점을 B, C. }

49. 삼각형 ABC에 하여 \BAC의 이등분선이 삼각형 ABC의 외접원과 만나는 점을 M , CM과 AB의 교점을 P라 하자. P를 지나고 AM과 수직인 직선이 AC와 만나는 점을 X, P를 지나고 AC와 수직인 직선이 AM와 만나는 점을 Y , P를 지나고 BC와 수직인 직선이 MB와 만나는 점을 Z라 하자. 세 점 X, Y ,Z가 한 직선 위에 있음을 보여라. (단, \B는 \C보다 크다.) (한국 2005-J5)

증명 (KAIST 과학영재센터 연구원 고봉균)

P에서 AC, BC에 내린 수선의 발을 각각 Q, S라 하고, XY 와 AB의 교점을 T라 하자.

A Q D

M P

C N B R b

b R

M

R

R a

a

Page 172: 실전수학올림피아드 1400제 해답

172 기하

Z, Y , X가 일직선을 이룸을 보이면 되겠다. ¤PQCS가 PC를 지름으로 하는 원에 내접하므로, 내 각과 원주각으로

\ZPY ´ \SPQ = \SCA ´ \BCA = \BMA ´ \ZMA

따라서, ¤ZPYM은 원에 내접하는 사각형이다. 또 이등변삼각형 APX는 축 AM에 해 칭이므로\XTP = \PQX = \R 이고, 따라서 ¤ATY Q도 원에 내접하는 사각형이다. 그럼 다시 원주각과 내각에 의해

\ZY P = \ZMP = \BAC ´ \TAQ = \XYQ

가 되고, 그럼 맞꼭지각에서 X, Y , Z가 일직선을 이룸을 확인할 수 있다. ¤

50. ABC는 \BAC = 40± 이고 \ABC = 60± 인 삼각형이다. D와 E는 각각 변 AC와 AB 위의 점으로서,\CBD = 40± 과 \BCE = 70± 을 만족한다. F는 두 직선 BD와 CE의 교점이다. 두 직선 AF와 BC가직교함을 증명하여라. (캐나다 1998-4)

증명 직선 BF에 한 C의 칭점을 G라 하고, GFX가 정삼각형이 되는 B에서 먼 점을 X라 하자.

그리고, F에서 BC에 내린 수선의 발을 H라 하자.

4BGF ´ 4BCF 이고 이들은 세 내각이 40±, 70±, 70± 인 이등변삼각형이다. 그럼 BX는 밑변이맞붙은 두 이등변삼각형 4BGF와 4XGF의 꼭지각을 이등분하므로, \XBC = 60± 이다(1). 한편,\XFH = 60± + 70± + 50± = 180± 로 XFH는 한 직선을 이루므로 \HFC는 4FCX의 외각이고,FX = FG = FC 로 이등변삼각형이므로 \FXC = 1

2\HFC = 10± 이다. 그러므로 \BXC = 40± 가

된다(2).(1), (2)에 의해 4XBC와 4ABC는 세 내각이 같고 BC가 일치하므로 합동, 즉 X = A 이다. 따라서,AF ? BC 가 확인된다. ¤

51. 볼록오각형 ABCDE에서 BC, CD, DE의 길이가 같고 각 각선이 이 오각형의 다른 어떤 변에 평행하다면(예를 들어 AC는 DE와 평행하다) 주어진 볼록오각형 ABCDE는 정오각형임을 보여라.

(통신강좌 1995-10-21, 미국 1972-5 유사)

증명 ( 전과학고 윤재문) 문제의 조건의 의해

BC = CD = DE

이므로, 4BCD, 4CDE는 이등변삼각형이다.

\CBE = \CDB

\DCE = \DEC

BE==CD이므로

\CBD = \CDB = \DBE

\DCE = \DEC = \CEB

그림처럼 ED==E0C가 되도록 BE상에 E0을 잡으면 ¤E0CDE는 평행사변형

E0C = ED = BC

Page 173: 실전수학올림피아드 1400제 해답

3.2 기하 고급문제 173

2\CBD = \CBE0 = \CE0B = \DEB = 2\DEC

) \CBD = \DEC

각 각선이 평행이므로 엇각을 이용하여 같은 각을 모두 표시하면 같다.그런데, ¤ABCD에서 \ABD =\ACD이므로 이 사각형은 원에 내접하고 그러면 원주각으로 \ADB = \ACB가 된다. 다시 엇각을 적용하면

\ADB = \BCA = \CAD = \ADE

그러면 4ABC, 4ADE도 이등변삼각형이 되어 5개 변, 각이 모두 같아 정5각형이 된다.

¤

52. P와 Q는 삼각형 ABC의 \BAC 내부에 있고(삼각형의 내부에 있을 필요는 없다) PQ의 연장선이 BC를수직이등분하며, \ABP + \ACQ = ¼ 이다. \BAP = \CAQ 임을 보여라. (통신강좌 1995-10-42)

증명 (상문고 이경용) 4ABP의 외접원과 직선 PQ의 교점을 R이라 하자. 점 A, B, P , R은 같은 원

주상에 있으므로 \ABP = \ARP , \ABP + \ACQ = ¼이므로 \ARP + \ACQ = ¼. 그러므로,¤ARQC도 원에 내접하는 사각형이다.

\BAP = \BRP

\CAQ = \CRQ

4RBC는 이등변삼각형이므로

\BRP = \CRQ) \BAP = \CAQ

¤

53. 원 O의 외부에 있는 점 P를 지나는 한 직선이 원 O와 두 점 A, B에서 만나고, P를 지나는 또 다른 직선이원 O와 두 점 C, D에서 만난다. 단, A는 P와 B 사이에, C는 P와 D 사이에 있다. 선분 AD와 BC의 교점을 L이라 하고, 반직선 PA 위에 BL ¢ PE = DL ¢ PD 가 되도록 점 E를 잡자. 직선 PL과 직선 DE의교점을 M이라 할 때, M은 선분 DE의 중점임을 보여라. (한국 2005-S5)

Ε ' ΕΒ

Χ ∆

ΕΒΕ

∆Χ

Β

ΑΑ

Χ ∆

P

B

Q

C

RA

Page 174: 실전수학올림피아드 1400제 해답

174 기하

증명 (고양 백석고 2학년 정윤범)

L; M에서 PB, PD에 내린 수선의 발을 각각 H, I; J , K라 하자.

4LAB와 4LCD가 닮았으므로 그 닮음비를 a : b 라 하자. 그럼 LH : LI = a : b. 또, P를 중심으로 한닮음에서 MJ : MK = a : b 이기도 하다. 그런데, 문제의 조건에서 PD : PE = BL : DL = a : b =MJ :MK 이므로, PD ¢MK = PE ¢MJ , 즉

j4PEM j = j4PDM j

으로 넓이가 같다. 그럼 이 두 삼각형을 각각 EM , MD를 밑변으로 하고 같은 높이를 갖는 삼각형으로보면 EM =MD 임을 알 수 있다. 따라서, M은 DE의 중점이다. ¤

54. 한 원이 ABCD에 내접한다. AB와 CD는 평행하고 BC = AD 이다. 각선 AC, BD는 E에서 만난다.삼각형 ABE, BCE, CDE, DAE의 내접원의 반지름은 각각 r1; r2; r3; r4이다. 1=r1+1=r3 = 1=r2+1=r4임을 증명하여라. (소련 1964-15)

증명 삼각형 ABE, BCE, CDE, DAE를 각각 X1, X2, X3, X4로 나타내고, 삼각형 Xi의 넓이와 둘

레의 길이를 각각 Si, di로 나타내자.일반적으로 S = dr 이 성립하고, 또 AB : CD = p : q 라 할 때 셈본을 참조하면 S1 : S2 : S3 : S4 = p2 : pq : q2 : pq 이다. 또한 삼각형의 닮음에서 d1 : d3 = r1 : r3 = p : q

이다. 원에 외접하는 사각형이므로 AB + CD = BC +DA 임에서

d1 + d3 = d2 + d4

S1

r1+

S3

r3=

S2

r2+

S4

r4S1

S2¢ 1r1+

S3

S2¢ 1r3=1

r2+1

r4p

q¢ 1r1+

q

p¢ 1r3=1

r2+1

r41

r3+1

r1=1

r2+1

r4

가 확인된다. ¤

55. AB = 36, AC = 72, \B = 90± 인 종이 삼각형 ABC가 있다. 이 삼각형의 내부에 점 P를 잡고, A, B,C가 각각 P에 오도록 종이를 한 번씩 접었다가 펴서 세 개의 접힌 자국의 선분이 생기도록 한다. 이 세선분이 서로 가로지르지 않도록 P를 잡을 수 있는 영역의 넓이를 구하여라. (AIME 1994-15)

풀이 %EEE

[그림]

The folds to bring A and B to P are the perpendicular bisectors

of AP and BP, which meet at O. We want O to lie below AB. Note that O

is the circumcenter of ABP, so ∠APB = ½∠AOB. We want ∠AOB > 180^o,

so ∠APB > 90^o, so P must lie inside the semicircle with AB as diameter.

[그림]

Similarly P must lie inside the semicircles on AC and BC.

Page 175: 실전수학올림피아드 1400제 해답

3.2 기하 고급문제 175

The semicircle on AC does not impose any constraint since B lies on it

and so the entire triangle lies inside it. Thus we want the area of

the intersection of the semicircles on AB and BC. The upper arc BC has

radius 18 and subtends an angle 120^o, so the area between it and the

chord BD is (1/3)π18^2 - 18^2cos 30^o sin 30^o = 108π - 81√3.

Similarly, the other arc has radius 18√3 and angle 60^o,

so area (1/6)π3·18^2 - 243√3. So total area 270π - 324√3.

¢ ¢ ¢ 답 270¼ ¡ 324p3 }

56. 삼각형 ABC의 내접원이 변 AB, AC와 각각 점 D, E에서 접한다. \ACB와 \ABC의 이등분선이 직선DE와 만나는 점을 각각 X와 Y 라 하고, 변 BC의 중점을 Z라 하자. 삼각형 XY Z가 정삼각형일 때, 또그 때만 \A가 60±임을 증명하여라. (발칸 2005-1)

증명 내심을 I라 하자. \DXI = \DEC+\ECX = 90±+ A2+ C2 이므로, \DXI+\DBI = 180±.

따라서, BIXD는 원에 내접하는 사각형이다. 그럼, ID ? AB 이므로, BX ? CI 가 된다. 즉 CXB가직각삼각형이므로, ZX = ZB = ZC 이다. 같은 방법으로 ZY = ZB 도 얻을 수 있고, 따라서 XY Z는항상 이등변 삼각형이다.ZX = ZC 이므로 \ZXC = \ZCX = \ACX 이고, 이로부터 ZX k AC 이다. 마찬가지로 ZY k AB도 확인할 수 있다. 이상으로부터 XY Z가 정삼각형일 필요충분조건은 \XZY = 60±, 즉 \A = 60± 임을 알 수 있다. ¤

57. ¡는 반지름 r인 원이다. A와 B는 AB <p3 r 인 ¡ 위의 서로 다른 두 점이다. B를 중심으로 하고 반지

름 AB인 원이 ¡와 C에서 다시 만난다고 하자. 그리고 P를 ABP가 정삼각형이 되는 ¡ 내부의 점이라고하자. 마지막으로, CP가 ¡와 Q에서 다시 만난다고 하자. PQ = r 임을 증명하여라. (캐나다 2002-4)

풀이 원 ¡의 중심을 O라 하고 \QCB = ® 라고 하자.

원 B와 원 ¡에서 원주각과 중심각의 관계를 고려하면

\QOA = 2\QCA = 2\PCA = \PBA

로 꼭지각이 60±인 이등변삼각형이므로 OQA는 정삼각형이다. 그럼 A를 중심으로 Q와 P를 60± 회전한 점이 각각 O와 B가 되므로 4QAP = 4OAB. 따라서 QP = BO = r. }

58. 서로 만나는 두 원의 교점의 하나를 O라 하자. O를 지나는 직선이 두 원과 만나는 점을 각각 P , Q라 하자. 선분 PQ의 중점의 자취를 구하여라. (통신강좌 1995-11-4)

풀이 (서울과학고 이지운) 두 원의 다른 교점 O0이라 하자. 우선 그림과 같이 P , Q가 각각 원의 바깥

Page 176: 실전수학올림피아드 1400제 해답

176 기하

쪽 호와 만난 경우를 생각하자. 선분 PQ이외의 다른 임의의 선분을 P 0Q0이라 하자.

선분 OQ에 한 원주각으로써

\OPO0 = \OP 0O0; \OQO0 = \OQ0O0

이다. 따라서4PO0Q 4P 0O0Q0(AA닮음)

이 된다. PQ; P 0Q0의 중점을 각각 M , M 0라 하면

O0P 0 : O0P = P 0M 0 : PM; \OP 0O0 = \OPO0

이므로4MPO0 » 4M 0P 0O0(SAS닮음)

이다. 따라서\OM 0O0 = \OMO0 = 일정

이다. 그러므로 M은 OO0을 지나는 원주위의 점이다. P , Q가 다른 쪽 호와 만날때의 경우도 비슷하게증명할 수 있다. }

59. r1 < r2 < r 은 각각 원 ¡1, ¡2, ¡의 반지름이다. ¡1과 ¡2가 각각 점 A, B에서 ¡에 내접하고 있으며, 둘은 서로 다른 두 점에서 만난다. 만약 AB가 ¡1과 ¡2의 한 교점을 지날 필요충분조건은 r1 + r2 = r 임을증명하여라. (이탈리아 1999-3)

증명 세 원의 중심을 O1, O2, O라 하자. O1, O2는 각각 OA, OB 위에 있다. AB가 지나는 두 원의

교점을 C라 하면 OAB, O1AC, O2CB는 모두 닮은 이등변삼각형. 역도 싸바싸바하면 성립. ¤

60. 주어진 한 원에 접하고 이 원 밖에 주어진 두 점을 지나는 원을 작도하여라.(셈본중등고급 도전문제 4.2.4)

풀이 원 O와 원 밖의 두점 A, B가 주어져 있다고 하자. A, B를 지나고 원 O와 두점에서 만나는 원

O0을 그린다.

A

O B

D

B

F

O

C

E

A

Page 177: 실전수학올림피아드 1400제 해답

3.2 기하 고급문제 177

O0과 O와의 교점을 C, D라 하고,Ã!CD와

Ã!AB의 교점을 E라 하자. (CD와 AB가 평행하면 다른 원을 잡

는다.)E에서 원 O에 접선을 그어 접점을 F라 하자. 4ABF의 외접원이 우리가 구하려는 원이다. (* 세원이각각 접하거나 두점에서 만날때, 각각의 교점을 잇는 세 직선은 한점에서 만남.) }

61. 원에 내접하는 사각형 ABCD에 하여 변 AB 위에 중심 O를 갖는 다른 원이 사각형 ABCD의 다른 세변 AD, DC, CB와 각각 E, F , G에서 접한다. FC =

p2, AO = 3AE 일 때 AE의 길이를 a라고 하자.

4a2의 값을 구하여라. (한국 2006 1차-J17)

풀이 4OFC를 O를 중심으로 회전하여 4OEH가 되도록 하자.

\OHE = \OCF = µ 라 하면 \A = 180±¡\C = 180±¡2µ. 따라서,4AOH는 \AHO = \AOH = µ

인 이등변삼각형이고, AH = AO 이다. 그럼p2 = FC = EH = AH ¡ AE = AO ¡ AE = 2a. 따라

서, 4a2 = 2 ¢ ¢ ¢ 답 }

주 `평면기하의 아이디어'(박승동 편저, 현재는 절판됨)라는 책의 41쪽 예10에 이와 거의 똑같은 문제

가 실려있다. 위의 풀이는 거기서 옮겨온 것이다. 출제가 잘못된 듯 하다.

별해 (광주 동성고 3학년 방재혁)

피타고라스의 정리에서 OE = OF = OG =p(3a)2 ¡ a2 = 2

p2 a.

\A = µ 라 하자. 사각형 ABCD가 원에 내접하므로, \C = ¼ ¡ µ. 따라서 \FCO = 12(¼ ¡ µ). 그러므

\FOC =µ

2

4AOE와 4OCF에서

tan µ =2p2a

a= 2

p2

tanµ

2=

p2

2p2 a

=1

2a

Page 178: 실전수학올림피아드 1400제 해답

178 기하

이것을 탄젠트 배각공식 tan µ =2 tan µ

2

1¡tan2 µ2

에 입하면

2p2 =

1a

1¡ 14a2

=4a

4a2 ¡ 14p2a2 ¡

p2 = 2a

a2 ¡p2

4a¡ 1

4= 0Ã

a¡p2

2

!Ãa+

p2

4

!= 0

이 된다. a > 0 이므로 a =p22. 따라서, 4a2 = 2 ¢ ¢ ¢ 답 }

62. ABC는 예각삼각형이다. B에서 내린 수선이 AC를 지름으로 하는 원과 점 P , Q에서 만나고, C에서 내린 수선이 AB를 지름으로 하는 원과 점 R, S에서 만난다. 네 점 P , Q, R, S는 한 원 위에 있음을 증명하여라. (체코슬로바키아 1992-6)

증명 BD, CE를 수선, H를 수심이라 하면 D, E는 각각 원 AC, AB 위에 있고 BCDE는 cyclic이므

로 방멱에 의해 PH ¢HQ = CH ¢HE = BH ¢HD = RH ¢HS. ¤

63. 점 A, B, C는 원 O 위의 서로 다른 점이다. A, B를 지나는 O의 두 접선이 P에서 만난다고 하자. C를 지나는 O의 접선은 AB와 Q에서 만난다. 다음을 증명하여라.

PQ2 = PB2 +QC2 (폴란드 2002/2003 1차-3)

증명 (여주 홍지현)

QC는 원 O의 접선이므로,QC2 = QB ¢QA (1)

직선 PQ 위에 PB = PX1, PA = PX2 인 점 X1, X2를 찍으면, PA = PB 이므로 PA = PB =PX1 = PX2 로 네 점 A, B, X1, X2는 P를 중심으로 하는 한 원 위에 있다. 그러면 할선의 비례에 관한 성질에서

QA ¢QB = QX1 ¢QX2 = (PQ¡ PB)(PQ+ PB) (2)

(1), (2)에 의해, QC2 = PQ2 ¡ PB2 으로 문제가 증명되었다. ¤

64. \ABC +\ABD = 180± 이고 AC ¢DE = BC ¢ CE 인 사각형 ABCD가 있다. 단, E는 두 각선의 교점이다. 2(AB2 +BC2) = AC2 +AD2 임을 증명하여라. (ML프로포절 158-3, 이탈리아 2000-2 변형)

증명 B에 한 C의 칭점 F를 잡자.

A

P

B

X1 X2

O

C

Q

Page 179: 실전수학올림피아드 1400제 해답

3.2 기하 고급문제 179

그럼 4AFC에서 AB가 중선이므로 파푸스의 중선정리에 의해

AC2 +AF 2 = 2(AB2 +BC2)

따라서, 문제의 식과 비교하면AF = AD (¤)

임을 증명하면 됨을 알 수 있다. E에서 AB에 평행한 직선을 그려 BC와 만나는 점을 H라 하자. 그럼

AC : CE = BC : CH

이므로 문제의 조건과 비교하여DE = CH (1)

임을 알 수 있다. 또,

\EHB = \ABF = 180± ¡ \ABC = \ABD = \HEB

이므로 4BHE가 이등변삼각형이 되어BE = BH (2)

이다. (1)과 (2)를 변변 더하면BC = BD

이다. 이로부터 BF = BD, \ABF = \ABD, AB는 공통이므로,

4ABD ´ 4ABF (SAS합동)

이고, 이로부터 (¤)이 확인된다. ¤

65. A, B, C, D는 한 원 위에 시계반 방향의 순서로 놓인 네 점이고 AB < AD 와 BC > CD 를 만족한다.

각 BAD의 이등분선이 이 원과 만나는 점을 X라 하고, 각 BCD의 이등분선이 이 원과 만나는 점을 Y 라하자. 이 원주 위의 6개의 점으로 이루어진 6각형을 생각하자. 이 6각형의 6개의 변 중에서 4개의 길이가같으면, BD가 이 원의 지름이 됨을 증명하여라. (캐나다 2004-3)

증명

원주각의 크기가 같으면 현의 길이도 같으므로, AX가 \A의 이등분선임에서 BX = XD, CY 가 \C의이등분선임에서 BY = Y D 가 된다. 즉,

AB;AY < Y D; CX;CD < BX

이고, 6각형의 변 중 4개의 길이가 같으려면 AB = AY = CX = CD 일 수밖에 없다. 그럼

\A = 2\(XD) = 4\(CD) = 4\(AB) = 2\(BY ) = \C

가 되고(\(PQ)는 현 PQ의 원주각을 의미함), \A = \C = 90± 일 수 밖에 없으므로 BD는 지름이 된다. ¤

66. 평행사변형 ABCD가 있다. P는 \PAB = \PCB 를 만족하는 이 평행사변형 외부의 점이다(단, 두 각의 방향은 서로 반 이다). \APB = \DPC 임을 증명하여라. (헝가리 1976-1)

Page 180: 실전수학올림피아드 1400제 해답

180 기하

증명 평행사변형 CDPQ 를 만들면, 4ADP ´ 4BCQ, \APB = \PBQ = \PCQ = \DPC, 그

리고 각 비교로 BCQP cyclic. ¤

67. ABC는 AC = BC 인 이등변삼각형이다. M은 AC의 중점이고, z는 AB와 수직이고 C를 지나는 직선이다. 세 점 B, C, M을 지나는 원이 직선 z와 두 점 C, Q에서 만난다. 삼각형 ABC의 외접원의 반지름을m = CQ 에 한 식으로 나타내어라. (주니어발칸 2004-2)

풀이 ABC의 무게중심과 외심을 각각 G, O, AB의 중점을 N이라 하자. AMGQ, AMON cyclic 이

므로 CG ¢ CQ = CM ¢ CA = CO ¢ CN . 따라서, CO = CGCN

m = 23m. }

68. 원 O의 원주 위에 반시계방향으로 여섯 개의 점 A, B, C, D, E, F가 있다. 선분 BD와 CF는 직교한다.

또한, 세 선분 CF , BE, AD는 한 점에서 만난다. 점 B에서 AC에 내린 수선의 발을 M , 점 D에서 CE에내린 수선의 발을 N이라 할 때, AE와 MN이 평행함을 보여라. (한국 2006-J3)

증명 (고양 백석고 3학년 조상영)

CF \BE \AD = L, BD \ CF = K 로 정의하자.

\BKC = \BMC = 90±이므로 B;M;K;C는 한 원 위에 있다. 따라서 원주각에 의해

\LAC = \DAC = \DBC = \KBC = \KMC

즉,LA k KM 동일한 원리로 LE k KN

이로부터 C를 중심으로 하여¤CALE » ¤CMKN

이고, 따라서 4AEC » 4MNC, 또 AE kMN 임을 알 수 있다. ¤

69. AB를 지름으로 하는 원의 B에서 그은 접선 위에 두 점 C와 D가 B에 해 서로 반 쪽에 있다. AC,AD가 원과 각각 E, F에서 다시 만난다. 또, CF , DE가 원과 각각 G, H에서 다시 만난다. AG = AH 임을 보여라. (중미 2003-2)

증명 AK를 접선이라 하자(K는 D와 같은 방향). \CDF = \FAK = \FEA 이므로 FDCE는

cyclic. 그럼 \FCD = \FED = \FGH 로 GH k CD. ¤

70. 삼각형 ABC의 점 C를 지나고 AB에 평행한 직선을 `이라 하자. \A의 이등분선이 BC와 점 D에서 만나고 `과 점 E에서 만난다. \B의 이등분선이 AC와 점 F에서 만나고 `과 점 G에서 만난다. DE = FG

일 때 CA = CB 임을 보여라. (아일랜드 1991-8)

증명 \A = 2®, \B = 2¯ 라 하고, ® > ¯ 라 가정하자. 그럼 BC > CA 이고 BG > AE.

BG

FG=

AB + CG

CG<

AB + CE

CE=

AE

DE

가 되어 모순. ¤

Page 181: 실전수학올림피아드 1400제 해답

3.2 기하 고급문제 181

71. 육각형 ABCDEF는 칭중심을 가진다. AB와 EF는 점 A0에서 만나고, BC와 AF는 점 B0에서, AB와CD는 점 C0에서 만난다. 다음을 증명하여라.

AB ¢BC ¢ CD = AA0 ¢BB0 ¢ CC0 (폴란드 1992/1993 1차-3)

증명 세 변의 길이가 a, b, c인 삼각형에서 ab¢ bc¢ ca= 1 임을 이용. 이것을 닮은삼각형에 적용하여 변

형. ¤

72. 삼각형 ABC와 점 P가 같은 평면 위에 있으며 점 P는 ABC의 외심이 아니다. P에서 삼각형 ABC의 각변의 수직이등분선에 내린 수선의 발을 세 꼭지점으로 하는 삼각형이 ABC와 닮았음을 증명하여라.

(이탈리아 1993-4)

증명 세 수선의 발이 모두 OP를 지름으로 하는 원 위에 있음. 세 각이 둘씩 같은 AA닮음. 경우를 좀

간략히 따지는 요령은 필요할 듯. ¤

73. 사각형 ABCD를 어느 두 변도 평행하지 않은 볼록사각형이라 하자. 변의 연장선에 의해 만들어지는두 각의 이등분선이 사각형 ABCD의 네 변과 각각 P , Q, R, S에서 만난다고 하자. 사각형 ABCD가 외접원을 가지는 것과 사각형 PQRS가 마름모라는 것이 동치임을 보여라. (통신강좌 1997-15-6)

증명 (과기원 97학번 이수인)

그림과 같이 AD와 BC의 연장선의 교점을 M , AB와 CD의 연장선의 교점을 N이라 하자. 각 \AMB와\BNC의 이등분선이 각 변과 만나는 점 P , Q, R, S가 그림과 같다고 하고 MP와 NQ의 교점을 O라하자.

\ABC = ¯; \CDA = µ; \MON = !;

\OMA = \OMB = Á; \ONB = \ONC = Ã

라 하자. 사각형 OMDN과 사각형 ONBM의 내각의 합은 360±이므로

! + Á+ (360± ¡ µ) + Ã = 360± = (360± ¡ !) + Ã + ¯ + Ã 2! = ¯ + ±

사각형 ABCD가 내접사각형이려면 ¯ + µ = 180±이어야 하므로 ! = 90±이다. 따라서 \NOR =\NOP = 90±, \ONR = \ONP = Ã이므로

) 4NOP ´ 4NOR (* ! = 90±)

따라서 OP = OR, OQ = OS ¢ ¢ ¢ (¤)각선이 서로 수직이고, (¤)을 만족하므로 ¤PQRS는 마름모이다. ¤

74. \A = 2\B 인 모든 삼각형 ABC에 해, C에서 A까지의 거리와 AB의 수직이등분선까지의 거리의 비가 일정함을 보여라. (체코슬로바키아 1991-4)

풀이 AB k CD 인 등변사다리꼴 ABCD를 그리고, 두 각선 AD와 BC의 교점을 P , AB와 CD의

중점을 각각 M , H라 하자. 그럼 PAB, PCD는 밑각이 \B 와 같고 P를 꼭지점으로 하는 닮은 이등변삼각형. 따라서, 삼각형 CAD와 DBC도 그와 닮은 이등변삼각형이고, CA = CD = 2CH. (CH가C에서 AB의 수직이등분선까지의 거리임) }

AN P Bβ

S

O

Q

ω

R

M

C

D

ψψ

φφ

δ

Page 182: 실전수학올림피아드 1400제 해답

182 기하

75. 원 O의 외부의 한 점 P를 지나는 직선이 원 O와 두 점 B, C에서 만난다고 하자(단, PB < PC). 직선PO와 원 O가 만나는 두 점을 Q, D라 하고(단, PQ < PD) 점 Q를 지나고 직선 BC에 수직인 직선이 원O와 만나는 또다른 점을 A라고 하자. 이 때, BD2 = AD £ CP 이면 직선 PA는 원 O의 접선임을 보여라. (한국 2006-J7)

증명 (서울 중평중 3학년 송하일)

\QAD = 90± 이므로 PC k AD 가 된다. 그러면 사각형 ABCD는 원에 내접하므로 등변사다리꼴이고BD = AC 가 된다.

문제의 조건으로부터 AC2 = AD ¢CP 가 되는데, AD : AC = AC : CP 이고 \PCA = \CAD 이므로

4PCA » 4CAD

가 된다. \PAC = \ADC 가 되고 접현각의 성질로부터 PA는 원의 접선이 된다. ¤

76. AB와 CD가 평행인 사다리꼴 ABCD가 있다. 이 사다리꼴에서 변 AD, BC 위에 각각 점 P , Q가 있는데, \APB = \CPD 이고 \AQB = \CQD 라고 한다. 사다리꼴의 두 각선의 교점에서 P , Q까지의거리가 같음을 보여라. (러시아 1994 최종-y9-7)

증명 B, C의 AD에 한 칭점 B0, C0을 잡으면 P는 세 직선 BC0, B0C, AD가 만나는 점. BB0,CC0이 AD와 만나는 점을 H, K라 하자. 4BAH와 4CDK는 AA닮음. 4BPH와 4CPK도 AA닮음. R = AC \ BD 라 하면 BP : PC0 = BP : PC = BH : CK = BA : CD = CR : RD 이므로RP k DC0. 즉 RP = DC0 ¢ BR

BD= CD ¢ AB

AB+CD= AB¢CD

AB+CD. 칭적으로 RQ도 같은 식. ¤

77. 삼각형 ABC의 변 BC, CA, AB의 바깥쪽에 점 P , Q, R을 잡자. 이 때, 4BPC » 4ACQ » 4RBA 이면 이 세 삼각형의 외심 P 0, Q0, R0로 이루어진 삼각형 R0P 0Q0도 이 세 삼각형과 닮았음을 증명하여라.

증명 BPC의 외접원과 CQA의 외접원의 교점을 F라 하자. 그럼

\AFB = 360¡ (180¡ \P + 180¡ \Q) = \P + \Q = 180¡ \R:

따라서, F는 ARB의 외접원 위에도 있다. 선분 P 0Q0은 두 원 P 0, Q0의 공통현(= CF )과 수직으로 만난다. 이 교점을 X라 하자. 선분 Q0R0과 선분 R0P 0의 경우에도 마찬가지이다. 이 교점들을 각각 Y , Z라하자. \P 0XF = \P 0ZR = 90± 이므로 P 0;X; F; Z도 한 원 위에 있다. 따라서

\P 0 = 180¡ \BFC = \P:

같은 이유로 Q0;X; F; Y 와 R0; Y; F; Z도 각각 한 원 위에 있다. 또 \Q0 = \Q, \R0 = \R 이다.

) 4Q0P 0R0 » 4BPC » 4ACQ » 4RBA

¤

78. ABCDEF는 서로 마주 보는 변이 평행한 볼록육각형이다. 삼각형 ACE와 BDF의 넓이가 같음을 보여라. (헝가리 1958-3)

Page 183: 실전수학올림피아드 1400제 해답

3.2 기하 고급문제 183

증명 삼각형 ACE를 제거했을 때 육각형에서 남는 세 작은 삼각형의 넓이를 각각 두 배로 키운 세 평

행사변형을 육각형 안쪽에 그리면, 육각형 안쪽에 작은 삼각형 XY Z가 하나 구성되는데 그 XY Z의 세변의 길이는 육각형의 마주보는 평행변의 길이의 차들임. 삼각형 BDF에 해서도 마찬가지이고, 그럼이렇게 구성된 XY Z와 X0Y 0Z0은 SSS합동으로 넓이가 같음. ¤

79. \ACB = 120± 인4ABC가 있다.이 삼각형의 외접원의 중심을 O,반지름을 R이라 하자.또한4ABC의수심을 H라 하자. ¤AOBH는 원에 내접하는 사각형이고 그 원의 중심 O0은 원 O 위에 있음을 증명하여라. (통신강좌 1998-16-14)

풀이 우선 다음과 같이 그림을 그리자.

\ACB = 120±이므로\CAB = µ라 두면 \ABC = 60± ¡ µ\ACK = \ABC + \CAB = 60±\BCJ = \ABC + \CAB = 60±

그런데 \AKC = \BJC = 90±이므로\CAK = \CBJ = 30±4HAB에서 \AHB = 180± ¡ \HAB ¡ \HBA

= 180± ¡ (\HAC + \CAB)¡ (\JBC + \CBA)= 180± ¡ (30± + µ)¡ (30± + 60± ¡ µ)= 60±

¤AOBH에서 \AHB + \AOB = 180±) ¤AOBH는 원에 내접하는 사각형\AOB = 360± ¡ \AOB

= 360± ¡ 2\ACB = 120±

(원 O에서)4AOB는 이등변 삼각형이므로

\ABO =180± ¡ 120±

2= 30±

\ABO는_AO의 원주각

따라서_AO의 중심각 \AO0O = 2\ABO = 60±

AO0 = OO0이므로 4O0AO는 정삼각형따라서 O0A = AO = R) O0O = O0A = RO0이 O에서 R만큼 떨어져 있으므로 O0은 원 O위에 있다. }

80. 4ABC에서 D를 A에서 BC로 내린 수선의 발이라 하고, E를 B에서 AC로 내린 수선의 발, F를 C에서AB로 내린 수선의 발이라 하자. 그리고 BE의 중점을 X, CF의 중점을 Y 라 하면 4DXY 의 외접원은이 삼각형의 수심을 지남을 증명하여라. 그리고 4DXY 는 4ABC와 닮음임을 보여라.

(통신강좌 1998-17-18)

증명 (중등부 이지훈)

(1)수심을 H;BC의 중점을 M이라 하자.i) X;Y 가 AD에 해 반 방향에 있을 때;

BE = 2BX; BC = 2BM; \MBX는 공통이므로4MBX v 4CBE ) \MXB = 90±

마찬가지로 4CMY v 4CBF에서 \CYM = 90±

A

B D M C

EF H

YX

(i)

A

B D M C

E

FH Y

X

(ii)

Page 184: 실전수학올림피아드 1400제 해답

184 기하

\HXM = \HDM = 90± ) H;X;D;M은 한 원 위에 있다.\HXM + \MYH = 180± ) H;X;M;Y 는 한 원 위에 있다.

따라서 H;X;D; Y;M은 한 원 위에 있다.

ii) X;Y 가 AD에 해 같은 방향에 있을 때;BE = 2BX; BC = 2BM; \MBX는 공통이므로4MBX v 4CBE ) \MXB = 90±

마찬가지로 4CYM v 4CFB에서 \CYM = 90±\MXH = \MYH = 90± )M;Y;X;H는 한 원 위에 있다.\MXH + \MDH = 180± )M;X;H;D는 한 원 위에 있다.

따라서 H;X;D; Y;M은 한 원 위에 있다.i), ii)에서 4DXY 의 외접원은 수심 H를 지난다.(2) i) \BCF + \FBC = \BCF + \CHD = 90±

\FBC = \CHD = \Y XD (* 원주각)\BCE + \CBE = \BHD + \CBE = 90±\BCE = \BHD = \XYD (* 원주각)

따라서 4DXY v 4ABC(AA닮음)

ii) \FBC + \BCF = \DHC + \BCF = 90±\FBC = \DHC = \DXY (* 원주각)\DCE + \DHX = \DYX + \DHX = 180±) \DCE = \DYX

따라서 4DXY v 4ABC(AA닮음)i), ii) 에서 4DXY v 4ABC ¤

81. 평면 위에 삼각형 ABC가 있고, P가 BC의 중점, Q는 CA 위의 CQ=QA = 2 를 만족하는 점, R은 AB

위의 AR=RB = 2 를 만족하는 점이다. 이 삼각형 ABC가 지워지고 세 점 P , Q, R만 남아있을 때, 원래의 삼각형 ABC를 다시 작도하는 방법을 설명하여라. (아일랜드 1995-9)

풀이 R에서 PQ의 중점을 이은 직선과 평행하게 P , R, Q를 지나는 등간격의 세 평행선을 그리고, 또

역시 등간격으로 Q 다음의 평행선을 하나 더 그리고, PQ, PR이 이 마지막 평행선과 만나는 점을 각각T , U라 할 때, TU를 1 : 4로 내분하는 점을 A라고 하면 됨. }

82. 정삼각형 ABC 내부의 점 M에서 각 변에 내린 수선의 발을 D, E, F라 하자. DEF가 직각삼각형이 되는 점 M의 자취를 구하여라. (Towns 1987봄 SA5)

풀이 각 변을 현으로 하여 정삼각형 내부에 원주각이 150±인 원호를 셋 그리면 됨. }

83. 4ABC에서 \A, \B의 이등분선이 변 BC, AC와 만나는 점을 각각 D, E라 하자. 점 C에서 직선 AD,BE에 내린 수선의 발을 각각 F , G라 할 때, AB k FG 임을 보여라. (1995 서울시)

증명 CFIG cyclic. \IGF = 90± ¡\FGC = 90± ¡\FIC = 90± ¡ (\IAC +\ICA) = \IBA. ¤

84. 직사각형 ABCD에서 E는 CD의 중점이고 P는 선분 BC 위를 움직이는 점이며 F는 AE와 BD의 교점이다. PA2 + PF 2 + PE2 이 최소일 때 BP : PC 를 구하여라. (1998 교육청경시)

풀이 B를 원점으로 두고 C(6c; 0), A(0; 6a)로 좌표를 두자. 그럼 E(6c; 3a) 이고, F는 4ACD의 무

게중심이므로 F (4c; 4a)이다. P (x; 0) 이라 하면(0 · x · 6), 준식 y = PA2 + PF 2 + PE2 = (x2 +(6a)2) + ((x ¡ 4c)2 + (4a)2) + ((6c ¡ x)2 + (3a)2) = 3x2 ¡ 20cx + 61a2 + 52c2. y가 최소일 때는

x = 103c 일 때, 즉 BP : PC = x : 6c¡ x = 10

3c : 8

3c = 5 : 4 ¢ ¢ ¢ 답 }

85. 삼각형 ABC에서 A0, B0, C0은 각각 변 BC, CA, AB 위의 점으로, AA0, BB0, CC0은 한 점 O에서 만

난다.AO

A0O+

BO

B0O+

CO

C0O= 92 일 때,

AO

A0O¢ BO

B0O¢ CO

C0O를 구하여라. (AIME 1992-14)

풀이 x+ y + z = 92, 1x+ 1

y+ 1

z= 1 일 때 xyz를 구하라는 상황인듯. }

Page 185: 실전수학올림피아드 1400제 해답

3.2 기하 고급문제 185

86. P는 어떤 예각삼각형 내부의 임의의 점이다. P에서 이 삼각형의 둘레 위의 점에 이르는 거리 중에서 가장 긴 것을 D, 가장 짧은 것을 d라 하자.

(a) D ¸ 2d 임을 증명하여라.

(b) 등호가 성립할 조건을 구하여라. (헝가리 1943-2)

풀이 (a) P에서 각 꼭지점을 잇고 또 각 변에 수선을 내리자. 그럼 주어진 예각삼각형이 6개의 직각삼

각형 영역으로 분할된다. 이 영역들의 꼭지점 쪽의 각(꼭지각이라 하자) 6개를 합하면 내각의 합이므로180±가 되고, 따라서 30± 이하의 꼭지각이 존재한다. 그런 꼭지각을 갖는 영역을 직각삼각형 PAH라 하자(A가 꼭지점, H가 수선의 발이다).

D는 가장 긴 거리이고 d는 가장 짧은 거리이므로

d · PH = PA sin\PAH · PA sin 30± =1

2PA · 1

2D (1)

로 D ¸ 2d 가 성립함을 알 수 있다.(b) 등호가 성립한다면 (1)식에서 모두 등호가 성립해야 한다. 즉, 30±보다 작은 꼭지각이 존재하면 안되고, 그럼 합이 180±이기 위해서는 모든 꼭지각이 정확히 30±가 되는 수밖에 없다. 따라서, 예각삼각형의 모든 내각이 60±이고, 즉 정삼각형이 된다. 정삼각형의 경우 중심을 P로 하면 실제로 D = 2d 가 성립한다. }

별해 (a) 예각삼각형을 ABC라 하고, AP , BP , CP가 변과 만나는 점을 각각 E, F , G라 하자.

PE

AE+

PF

BF+

PG

CG=j4PBCjj4ABCj +

j4PCAjj4ABCj +

j4PABjj4ABCj = 1

이므로PE

AE,PF

BF,PG

CG중에

1

3이하인 것이 존재한다. 일반성을 잃지 않고

PE

AE· 1

3이라 하자. 그럼

3PE · AE = AP + PE 이므로 2PE · AP , 이로부터

D ¸ AP ¸ 2PE ¸ 2d (¤)

즉, D ¸ 2d 임을 확인할 수 있다.

(b) 위의 증명에서 보았듯이PE

AE,PF

BF,PG

CG중에

1

3보다 작은 것이 하나라도 있으면 (¤)에서 등호가

성립하지 않는다. 따라서, 모두1

3이상, 즉 모두 정확히

1

3이어야 한다. 또, (¤)에서 모두 등호가 성립해

야 함을 생각하면

² D = AP = BP = CP 여야 하므로 P가 외심이어야 하며,

² PE = PF = PG = d 여야 하므로 P는 내심이기도 하고,

² PE가 그 변에 이르는 최단거리여야 하므로 E는 P에서 내린 수선의 발, 즉 AE 등이 모두 수선이므로 P는 수심이어야 하며,

² 각 변에서 높이의1

3이 되는 자취직선들을 그려보면 이 세 직선이 한 점 무게중심에서 만나므로 P는

무게중심일 수밖에 없다.

이 중에 원하는 로 두어 가지를 골라 잘 말하면 금방 ABC가 정삼각형이어야 함이 확인된다. }

87. 지름 AB를 갖는 반원과 이 반원의 원주 위의 한 점 X|A나 B와는 다른|가 주어져 있다. tA, tB , tX를각각 A, B, X에서의 이 원의 접선이라고 하자. 직선 AX가 tB와 만나는 점을 Z라 하고, 직선 BX가 tA와만나는 점을 Y 라 하자. 이 때, 세 직선 Y Z, tX , AB가 한 점에서 만나거나 모두 평행함을 증명하여라.

(캐나다 1974-5)

Page 186: 실전수학올림피아드 1400제 해답

186 기하

증명 AB와 Y Z가 P에서 만난다고 하자.

tX가 tA, tB와 만나는 점을 각각 C, D라 하자. 접선의 성질에 의해 CA = CX, DB = DX 가 되고, 그럼 직각삼각형의 성질에 의해 C, D는 각각 4AXY , 4BXZ의 외심이다. 즉, C, D는 각각 AY , BZ의중점. P가 4PAY 와 4PBZ의 닮음의 중심이므로 P , C, D는 일직선을 이룬다.AB와 Y Z가 한 점에서 만나지 않고 평행할 때는 ABZY 가 직사각형(정사각형)이고 X가 그 중심이므로 tX도 이들과 평행하다. ¤

88. 원 S 밖의 점 P에서 S에 그은 두 접선의 접점을 각각 A, B라 하자. AB의 중점을 M이라 하고, AM의 수직이등분선이 S와 삼각형 ABP 내부에서 만나는 점을 C라 하자. AC와 PM의 교점을 G라 하고, PM이S와 삼각형 ABP의 바깥에서 만나는 점을 D라 하자. BD와 AC가 평행하다면, G가 삼각형 ABP의 무게중심임을 보여라. (중미 2007-6)

증명 AC = CG 임은 당연하고, 약간의 각 계산으로 BAG와 BNC가 합동인 이등변삼각형임을 알아

내어 AG = CN 임을 확인하면(N은 AC가 BP와 만나는 점), G는 AN의 3등분점이므로 N은 BM의수직이등분선 위에 있을 수 밖에. ¤

89. 변 AB의 길이 c, 내접원의 반지름 r, 그리고 변 BC와 CA의 두 연장선 및 변 AB에 접하는 바깥 원의 반지름 rC 등이 주어졌을 때, 삼각형 ABC를 작도하여라. (헝가리 1900-2)

힌트 r과 rC가 연관된 닮은 삼각형을 찾을 수 있다. }

90. 정사각형 ABCD에서 \MDN = 45± 가 되도록 변 AB, BC 위에 각각 점 M , N을 잡았다. MN의 중점을 R이라 하고, 각선 AC가 선분 MD, ND와 만나는 점을 각각 P , Q라 할 때, RP = RQ 임을 증명하여라. (인도지역예선 1999-3)

증명 \PDN = 45± = \PCN 이므로 PNCD는 cyclic. 내 각으로 \MPN = \NCD = 90±. 고

로, B;P;Q는 MN을 지름으로 하는 원 위에 있고, 그 원의 중심은 R. ¤

91. BC k AD 인 사다리꼴 ABCD에서 AC = BC+AD 이고 두 각선의 교각은 60±라고 한다. AB = CD

임을 증명하여라. (Towns 1992봄 JO2)

증명 두 각선의 교점을 P라 하자. BC k AD 이므로 4PBC » 4PDA 이고 닮음비는 BC : AD 이

다.즉, PC : PA = BC : AD 인데, 문제에서 PC+PA = BC+AD 라 했으므로 PC = BC, PA = AD

가 된다. 따라서, 4PBC와 4PDA는 밑각이 60±인 이등변삼각형이므로(밑각이 120±일 수는 없으므로이 경우는 배제된다) 정삼각형이다. 그럼 4APB와 4DPC는 SAS합동이 되고, 따라서 AB = CD 이다. ¤

Page 187: 실전수학올림피아드 1400제 해답

3.2 기하 고급문제 187

별증 두 각선의 교점을 P라 하고 ¤AEBD가 평행사변형이 되는 점 E를 잡자.

그럼 AD = BE 이므로 EC = EB+BC = AD+BC = AC 로4CAE는 CA = CE 인 이등변삼각형이다. AE k PB 이므로 4CPB도 CP = CB 인 이등변삼각형이고, 이등변삼각형의 밑각은 90±보다 작아야 하므로 \CPB = 60± 이다(문제의 조건에서 두 각선의 교각이 60±라고 했는데 \CPB = 120±일경우를 배제할 수 있다). 따라서, 4PBC와 4PDA는 모두 정삼각형이고, 그럼 4APB와 4DPC는SAS합동이 된다. 이로부터 AB = CD 이다. ¤

92. 정사각형 ABCD의 변 AB, BC 위에 각각 점 E와 F를 잡아 BE = BF 가 되도록 하였다. BN을 삼각형 BCE의 한 수선이라 하자. \DNF = 90± 임을 증명하여라. (오스트리아-폴란드 1979-1)

93. 원호 AC와 꺾인 선 ABC로 둘러싸인 그림이 있다. 원호와 꺾인 선은 AC에 해 서로 반 쪽에 있다. 호AC의 중점을 지나 이 영역의 넓이를 이등분하는 직선을 작도하여라. (Towns 1987가을 J4)

94. 삼각형 ABC에서 직선 AC를 각각 직선 AB와 BC에 해 칭시킨 두 직선이 점 K에서 만난다. 직선BK가 외심을 지남을 증명하여라. (Towns 1988봄 JA2)

95. 4ABC 내의 점 M에서 각 수선(높이)에 수선의 발을 내렸다. 각 꼭지점에서 높이를 따라 앞에서 내린 수선의 발까지 잰 길이가 셋 모두 같다고 한다. 이 길이는 내접원의 지름과도 같음을 증명하여라.

(Towns 1987가을 S4)

96. 한 정사각형이 같은 크기의 정사각형과 중심을 공유하고 45± 회전되어 겹쳐져있다. 각 변은 상 편 정사각형에 의해 a : b : a 의 비로 세 부분으로 분할된다. 임의의 볼록사각형에 해, 각 변을 위의 비 a : b : a

로 세 부분으로 분할하고, 각 꼭지점에서 그 점에 이웃한 두 분할점을 찾아 그 두 점을 연결하는 직선을각각 그린다. 이 네 직선으로 결정되는 사각형이 원래의 사각형과 같은 넓이를 가짐을 증명하여라.

(Towns 1990봄 JA2)

97. 사각형 ABCD의 네 변 AB, BC, CD, DA는 각각 사각형 A0B0C0D0의 네 변 A0B0, B0C0, C0D0, D0A0과길이가 같다. AB k CD 이고 B0C0 k D0A0 임을 알 때, 두 사각형 모두 평행사변형임을 증명하여라.

(Towns 1990가을 JA4)

98. 한 원 위에 두 점 K, L이 있다. 꼭지점 C와 두 중선 AK, BL의 교점이 둘다 이 원 위에 있는 삼각형ABC를 작도하여라. (Towns 1991봄 SO2)

99. 주어진 각의 한 반직선 위에 한 고정점 A를 잡자. A에 접하고 다른 반직선과 두 점에서 만나는 모든 원을생각하자. 그 두 교점을 B, C라 할 때, 삼각형 ABC의 내심은 원의 크기에 상관없이 모두 한 직선 위에있음을 증명하여라. (러시아 1989 4차-y9-3)

증명 각의 꼭지점을 O라 하고 A의 (각의 이등분선에 한) 칭점을 A0이라 하면 AA0은 항상4ABC의

각 A의 이등분선이 됨. 간단한 각 계산으로 금방 확인됨. ¤

100. 세 원 k1, k2, k3가 서로 다른 세 점에서 각각 서로 외접한다. 서로 다른 두 원 위의 (접점과는 다른) 점A, B에 해, 직선 AB가 그 두 원의 접점을 지날 때 점 A, B가 서로 응한다고 하자. A 2 k1, B 2 k2,C 2 k3, D 2 k1 은 A와 B, B와 C, C와 D가 서로 응하는 점이다. AD가 k1의 지름임을 보여라.

(몰도바 1996 최종-y9-7, 헝가리 1950-2 변형)

증명 일반적으로 서로 접하는 두 원에 해 A와 A0이 B와 B0에 응한다면 AA0 k BB0 이고 방향

은 서로 반 이다. 또한, AA0이 지름이면 BB0도 지름이다. 즉, AA0이 k1의 지름이라면 AA0 k BB0 kCC0 k DD0 이고 DD0은 AA0과 방향이 반 인 지름이므로 끝. ¤

Page 188: 실전수학올림피아드 1400제 해답

188 기하

101. 삼각형 ABC의 세 변의 길이를 BC = a, CA = b, AB = c 라 하고, D와 E를 변 AC와 AB의 중점이라하자. 두 중선 BD와 CE가 직교할 때, 또 그 때만 b2 + c2 = 5a2 임을 증명하여라. (아일랜드 2001-2)

증명 PQ ? RS () PR2 +QS2 = PS2 +QR2 을 이용하면 끝. ¤

102. 주어진 정삼각형 ABC에 하여, 점 A의 반 편에 BC를 지름으로 하는 반원을 그리자. 선분 BC의 삼등분점을 P , Q라 하고, 직선 AP와 AQ가 반원의 호와 만나는 점을 각각 K, L이라 하자. 점 K, L이 반원의 호를 삼등분함을 증명하여라. (호주 1984-2)

증명 반원을 원으로 확장하면 삼각형의 옆변과 중점에서 만나므로... 2 : 1 닮음비를 발견하여 금방. ¤

103. B1, B2, B3은 각각 4A1A2A3의 세 변 A2A3, A3A1, A1A2 위에 위치하는, 꼭지점이 아닌 점들이다. 세선분 AiBi의 수직이등분선들은 절 로 서로 일치하지 않음을 증명하여라. (오스트리아-폴란드 1980-5)

증명 AiBi의 수직이등분선은 중점연결선 MjMk와 만난다. 따라서, 두 개의 이런 수직이등분선이 일

치한다면 그 직선은 두 중점연결선의 교점을 지나고, 따라서 어떤 변의 중점 Mj를 지나야한다. 그럼 Bi

두 개가 꼭지점이 될 수밖에... ¤

104. 두 원 S1, S2가 두 점 A, B에서 만나고 S1의 중심 O가 S2 위에 있다. S1의 현 AC가 S2와 점 D에서 다시 만난다. OD와 BC가 서로 수직임을 보여라. (러시아 1990 4차-y9-7)

증명 BP를 S1의 지름이라 하면 \PCD = \PBA = \ADO 이므로 OD k PC ? BC ¤

105. 아래 그림에서 l1과 l2는 평행선이고, AB는 이들에 수직인 직선이며, P , Q, R, S는 AB 위의 점 C를 중심으로 하고 AB보다 긴 지름을 갖는 원이 l1, l2와 만나는 점이다.

PR ¢ PS 의 값은 AB 위의 점 C의 위치와 상관없이 일정함을 증명하여라. (IMTS R12-5)

증명 피타고라스정리로 열심히 계산하면 끝. ¤

106. 볼록사각형 ABCD가 어떤 원과 8개의 점에서 만나 원주를 8개의 호로 나누고있다. 이 사각형의 내부에있는 네 호의 길이를 시계반 방향 순으로 p, q, r, s라 할 때, p+ r = q+ s 가 성립한다고 한다. ABCD가원에 내접하는 사각형임을 증명하여라. (독일BW 2002 1차-3)

증명 (김지훈)

원과 사각형 ABCD가 만나는 8개의 점을 E, F , G, H, I, J , K, L이라 하자. 그러면 p+ r = q+ s 에 의해 \EOF +\JOI = \LOK+\GOH 이다. 그러므로 \EOF부터 시계반 방향으로 각 8개를 2a, 2b,2c, 2d, 2e, 2f , 2g, 2h라 표현하자. 그러면 삼각형 EOF , 삼각형 EOL이 이등변삼각형이므로 \FEO,\OEL은 각각 90± ¡ a, 90± ¡ b 가 된다. 따라서 \AEF = a+ b. 같은 방법으로 \AFE = a+ h 이므로 \FAE = 180± ¡ 2a¡ b¡ h 가 되고 반 쪽 \ICJ에서도 마찬가지로 \ICJ = 180± ¡ 2e¡ d¡ f .

따라서 \ICJ + \FAE 가 180±임을 보인다. \ICJ + \FAE = 360± ¡ b¡ h¡ d¡ f ¡ 2a¡ 2e 이므로 QED. (2a+ 2b+ 2c+ ¢ ¢ ¢+ 2h = 360± 이고 2a+ 2e = 2c+ 2g) ¤

107. AB = BC, AD = DC 를 만족하는 사각형 ABCD에 내접하는 원이 이 사각형의 변 AB, BC, AD와 각각 점 K, M , N에서 접한다. 각선 AC가 MN과 점 P에서 만날 때, 네 점 A, K, P , N이 한 원 위에 있음을 보여라. (러시아 1992 4차-y9-3)

증명 \KAP = \BKM = \BMK = \MNK. (차례로: 좌우 칭평행, 이등변삼각형, 접현각) ¤

108. 삼각형 ABC의 변 BC 위에 BB1 = CC1, \BAB1 = \CAC1 을 만족하는 점 B1, C1을 잡을 수 있었다. ABC가 이등변삼각형임을 보여라. (러시아 1992 4차-y9-7)

Page 189: 실전수학올림피아드 1400제 해답

3.2 기하 고급문제 189

증명 두 원 ABB1과 ACC1의 크기가 같다. ¤

109. 삼각형 ABC의 변 AB와 BC 위에 각각 점 M , N이 있다.두 선분 AN , CM이 점 O에서 만나고 AO = CO

라 한다. (1) AM = CN 이면 ABC는 이등변삼각형인가? (2) BM = BN 이면 ABC는 이등변삼각형인가? (러시아 1993 4차-y9-3)

풀이 (1)은 반례가 있음. (2)는 YES }

110. C1과 C2는 두 점 A와 B에서 만나는 두 원이다. 점 B를 중심으로 하고 점 A를 지나는 원을 C0이라 하자.C0과 C1의 공통현이 C2에 접할 조건을 말하고, 그것을 증명하여라. (IMTS R15-5)

증명 C1과 C2의 크기가 같음. 각 계산 좀만 해보면 됨. ¤

111. 삼각형 ABC에서, 변 BC와 AC의 수직이등분선이 직선 AC, BC와 각각 점 M , N에서 만난다. O를 삼각형 ABC의 외심이라 할 때, 다음을 증명하여라.

(a) 다섯 점 A, B, M , N , O는 한 원 k 위에 있다.

(b) k의 반지름은 삼각형 MNC의 외접원의 반지름과 같다. (몰도바 1996 최종-y10-7)

증명 (a) \OMC = \OBA 임을 간단히 확인. (b) 현 MN에 한 원주각의 크기가 같으므로 원의 크

기도 같다. ¤

112. 삼각형 ABC의 내심을 I라 하자. 변 AB와 CA의 중점을 각각 N , M이라 하고, 직선 BI와 CI가 직선NM과 각각 점 K와 L에서 만난다고 할 때, AI +BI + CI > BC +KL 임을 증명하여라.

(주니어발칸 1997-3)

증명 BI + CI > BC 이므로 AI > KL 만 보이면 충분하다. MLC, NBK가 이등변삼각형이므로

KL =ML+NK ¡MN =MC +NB ¡MN = 12(AB +AC ¡BC) 이고 이것은 A에서 내접원에 그

은 접선의 길이. ¤

113. 예각삼각형 ABC의 외접원의 중심을 O라 하고 반지름을 R이라 하자. 직선 AO, BO, CO는 변과 각각점 D, E, F에서 만난다. 다음을 증명하여라. (남미 1985-6)

1

AD+

1

BE+

1

CF=2

R

증명 (임준혁) R : AD = AO : AD = j¤ABOCj : j4ABCj. 같은 식으로 R : BE = j¤BCOAj :j4ABCj. R : CF = j¤CAOBj : j4ABCj. j¤ABOCj+ j¤BCOAj+ j¤CAOBj = 2£ j4ABCj 이므로 R

AD+ R

BE+ R

CF= 2이고 여기에 R을 나누면 등식이 성립한다. ¤

114. 각 A와 C가 같은 사각형 ABCD가 있다. 각 B의 이등분선이 삼각형 BDC의 외접원과 점 C1 6= D 에서만난다. 또 각 D의 이등분선이 삼각형 BDA의 외접원과 점 A1 6= B 에서 만난다. 사각형 A1BC1D가 평행사변형임을 보여라. (몰도바 1997 최종-y8-3)

증명 \DA1B = \A = \C = \DC1B, \A1DC1 = \A1DC + \CDC1 = \A1DA + \CBC1 =

\A1BA+\ABC1. 문제에 별 도움은 되지 않지만, 참고로, 원주각이 같으므로 두 원 CBD와 ABD의크기는 같다. ¤

115. AB = BC 이고 \BCD = \EAB = 90± 인 볼록오각형 ABCDE가 있다. 이 오각형의 내부에 점 X가존재하여 AX는 BE에 수직이며, CX는 BD에 수직이다. BX는 DE에 수직임을 보여라. (호주 1992-3)

증명 BMXN은 cyclic. BM ¢ME = BN ¢BD 임에서 MNDE도 cyclic. 이후 각 계산. ¤

116. 주어진 선분 AB를 점 B 방향으로 점 D까지 연장하자. 이 때 BD의 길이는 임의로 정해진다. AD을 지름으로 하는 반원을 그리고 그 중심을 H라 하자. \ABG가 예각이 되는 반원 위의 한 점 G를 잡은 뒤,EH ¢ ED = EZ2 을 만족하도록 BG에 평행한 선분 EZ를 그리자(E는 AD 위의 점, Z는 반원의 호 위의 점이다). 또한 ZH와 TB가 평행이 되도록 반원 위에 점 T를 잡자. \ABG = 3\TBG 임을 증명하여라. (호주 1992-5)

Page 190: 실전수학올림피아드 1400제 해답

190 기하

증명 각 조금만 쌓아보면 금방... ¤

117. 각 ACB가 둔각인 삼각형 ABC가 있다. 점 C에서 AB에 내린 수선의 발을 D라 하고, AB의 중점을 M이라 하자. 또 AC의 연장선 위에 EM = BM 을 만족하는 점 E를 잡고, 두 직선 BC와 DE의 교점을 F라하자. BE = BF 라면 \CBE = 2\ABC 임을 증명하여라. (호주 1993-1)

증명 M은 4ABE의 외심. \AEB는 직각. BECD cyclic. EF의 중점을 N이라 하면 BNE, BNF ,

BDC가 모두 닮음 직각삼각형. ¤

118. 8개의 점 A1, A2, B1, B2, C1, C2, D1, D2가 한 원의 둘레 위에 차례로 놓여 있다. A1B2 = B1C2 =C1D2 = D1A2 이면, 네 직선 A1A2, B1B2, C1C2, D1D2로 이루어진 사각형이 원에 내접하는 사각형이됨을 보여라. (러시아 1993 최종-y9-6)

증명 위의 독일BW 2002 1차-3 와 비슷하게 각 계산을 좀 해보면... 일정한 같은 길이의 두 현이 있을

때 두 현의 사이에 놓이는 두 호의 길이의 합이 일정함을 이용함(간단한 불변량). ¤

119. 볼록육각형 ABCDEF의 각선 AD, BE, CF는 각각 이 육각형의 넓이를 이등분한다. 이 세 각선은한 점에서 만남을 증명하여라. (폴란드 1966 3차-5)

증명 (최재연) jABCDj = 12jABCDEF j = jBCDEj, 즉 jABDj = jBDEj 이므로 BD k EA. 마찬

가지로 AC k DF , CE k FB. 그럼 응되는 세 변이 모두 평행하여 4ACE » 4DFB. 이런 사실들로부터 AD \CF = P 라 하면 AP : PD = AC : DF = EA : BD 가 되고, 그럼 4PAE » 4PDB 가되어(SAS) BPE는 일직선이 됨. ¤

별해 한 점에서 만나지 않고 교점들이 서로 다를 때 중앙에 한 영역이 더 생기고 거기서 넓이 관계의

등식 등을 열심히 써보면... ¤

120. 한 직선 위에 세 점 A, B, C가 이 순서 로 놓여있다. 세 정삼각형 ABD, BCE, CAF를 그리는데, 직선AC에 해 D와 E는 같은 쪽에 있고 F는 반 쪽에 있도록 한다. 이 정삼각형들의 무게중심을 세 꼭지점으로 하는 삼각형은 정삼각형임을 증명하여라. 또, 이 새로운 정삼각형의 무게중심은 직선 AC 위에 놓임을 증명하여라. (아일랜드 1994-2)

증명 세 정삼각형 ABD, BCE, CAF의 무게중심을 각각 G, H, I라 하고, AC에 한 그 칭점들을

G0, H0, I0이라 하자. 그럼 I 0(G)A(G0)I(H0)C(H)는 마름모가 되고(괄호 안은 그 변이 지나는 점을 표기한 것), I 0GG0IH0H는 건너뛴 두 묶음의 세 변의 길이가 각각 같은 등각육각형이다. 여기서 GHI나G0H 0I 0이 정삼각형임은 자명(등각육각형에서 그 삼각형을 오려낸 나머지 세 삼각형이 합동). 또한, 이등각육각형의 중심 O는 당연히 AC 위에 있고, 정삼각형 GHI, G0H0I 0들의 중심도 O임은 자명하다. ¤

121. E는 마름모 ABCD의 각선 AC 위의 A, C와는 다른 임의의 점이다. N , M은 각각 직선 AB, BC 위의A, C와는 다른 점으로, AE = NE, CE = ME 를 만족한다. 직선 AM과 CN이 점 K에서 만날 때, 세점 K, E, D가 한 직선 위에 있음을 보여라. (러시아 1993 4차-y9-7)

증명 4NEC ´ 4AEM . ANKE, CMKE cyclic. \AKC = 2\NAE = \BAD 이므로 AKCD도

cyclic. KE는 \AKC를 이등분하므로 K 반 쪽의 호 AC의 중점 D를 지남. ¤

별증 4EAN » 4DAC, 이로부터 4CAN » 4DAE (SAS). 그럼 \ANK = \AED 로 내 각이

되므로 KED는 일직선. ¤

122. AC > AB 인 삼각형 ABC의 각 A의 이등분선이 외접원 ¡와 점 D에서 다시 만난다. D를 원 ¡의 중심O와 이었더니 직선 DO가 직선 AC와 점 E에서 만났다. BE와 AD가 수직일 때, AO와 BD가 평행함을보여라. (인도지역예선 2000-5)

증명 BC의 중점을 M , AD와 BC, BE의 교점을 각각 Z, X라 하자. EBC, OAD가 이등변삼각형이

고 BDMX가 cyclic이므로, \ADB = \ECB = \EBC = \XDM = \OAD. ¤

별증 ABE는 이등변삼각형인데 \A = 21\BOC = \BOD 이므로 이등변삼각형 OBD와 닮음. 그리

고 같은 이유로 내 각에서 ABOE cyclic. angleOBD = \AEB = \AOB. ¤

Page 191: 실전수학올림피아드 1400제 해답

3.2 기하 고급문제 191

123. D, E는 각각 직각삼각형 ABC의 두 옆변 AC, CB 위의 점이다. C에서 DE, EA, AB, BD에 내린 수선의 발들이 한 원 위에 있음을 증명하여라. (러시아 1989 4차-y10-3)

증명 수선의 발을 나열된 차례로 P , Q, R, S라 하자. \DPS = \DCS = \DBC = \SRC, \EPQ =

\ECQ = \EAC = \ERC 이므로 \SPQ+ \SRQ = 180± ¤

124. 마름모 ABCD 안에 내접하는 원이 있다. 이 원의 한 접선이 변 BC, CD와 각각 점 M , N에서 만난다.

어떤 접선을 그렸느냐에 상관없이 삼각형 AMN의 넓이가 일정함을 보여라. (유고슬라비아 1980 고2-2)

증명 내접원이 MN , BC, CD와 접하는 점을 각각 X, Y , Z라 하고, 중심을 O라 하자. jOMXj =jOMY j, jONXj = jONZj, jOAM j = jOCM j, jOAN j = jOCN 으로부터 jAMN j = jY OZC|. ¤

125. 내접원의 반지름과 외접원의 반지름이 주어졌을 때, 그에 해당하는 직각삼각형을 작도하는 방법을 설명하여라. (이탈리아 1987-3)

풀이 내심에서 빗변을 보는 각이 135±임을 이용. [참고] R, r과 한 각의 크기가 주어지는 것이 삼각형

의 작도조건 중 하나이다. }

126. 평행사변형 ABCD의 변 AB 위에 점 E가, 변 CD 위에 점 F가 있다. AF와 ED의 교점을 G, EC와FB의 교점을 H라 하자. 또 GH의 연장선이 AD, BC와 만나는 점을 각각 L, M이라 하자. DL = BM

임을 증명하여라. (호주 1994-8)

증명 E, F에서 각각 AD에 평행하게 그은 직선이 GH와 만나는 점을 각각 J , K라 하자. JE=DL =

GE=GD = GA=GF = LA=FK 이므로 DL ¢ LA = JE ¢ FK. 마찬가지로 BM ¢MC = JE ¢ FK =DL ¢ LA. ¤

별증 각선의 교점(평행사변형의 중심)을 O라 할 때, GOH가 일직선임을 메넬라우스로 증명해도

됨. 계산 막노동이긴 하지만. ¤

127. 한 직선과 두 동심원과의 교점을 순서 로 A, B, C, D라 하자. AE와 BF는 각 원에서 하나씩 택한, 서로 평행한 현이다. C에서 BF에 내린 수선의 발을 G라 하고, D에서 AE에 내린 수선의 발을 H라 하자.GF = HE 임을 증명하여라. (호주 1995-3)

증명 AG0C를 BH0D 로 평행이동. 평행이며 길이도 같다는 것을 ´로 쓰기로 할 때, AB ´ G0H0 =HG ´ EF 이므로 GF = HE. ¤

128. ABCD는 볼록사각형이고 P와 Q는 각각 변 AD와 BC 위의 점으로

AP

PD=

BQ

QC=

AB

CD

를 만족한다. 직선 PQ가 두 직선 AB, CD와 이루는 각이 같음을 증명하여라. (남미 1987-6)

증명 (이성곤)

Page 192: 실전수학올림피아드 1400제 해답

192 기하

APPD

= BQQC

= CDAB

= ba라고 두자. 이제

Ã!PQ와 직선 AB;CD의 교점을 각각 R; S라고 하자. 이제 문제는

\BRQ = \QSC를 증명하는 것과 동치이다.AC위에 AX : XC = a : b인 점 X를 잡으면, PX==CD;QX==AB이다.

) 4APX∽4ADC; 4CQX∽4CAB

4APX∽4ADC에서 PX = aa+b

£ CD. 4CQX∽4CAB에서 QX = ba+b

£ AB. 그런데 조건에서

AB : CD = a : b라고 하였으므로 AB = ak, CD = bk라 둔다면 PX = QX = abk

a+b.

) 4PQX는 PX = QX인 이등변삼각형이다. 한편, \PQX = \ARQ(엇각), \QPX = \QSC(동위각)이므로 \BRQ = \PQX = \QPX = \QSC. ¤

129. 볼록오각형 ABCDE에서 AE = AD, AB = AC 이고 \CAD는 \AEB와 \ABE의 합과 같다. 선분CD는 삼각형 ABE의 중선 AM의 길이의 두 배임을 증명하여라. (Towns 1984봄 JO2)

130. 볼록사각형 ABCD의 두 각선 AC와 BD가 서로 직교하고, 그 교점을 E라 하자. E를 AB, BC, CD,DA에 해 각각 칭시킨 점들이 모두 한 원 위에 있음을 증명하여라. (미국 1993-2)

증명 1=2만큼 축소하면 각 변에 내린 수선의 발이 한 원 위에 있음을 보이는 문제가 됨. 변환기하 그

담은 각의 합이 180±임을 보여서. ¤

131. 원 k에 내접하는 예각삼각형 ABC가 있다. 점 A에서 그은 접선이 직선 BC와 점 P에서 만난다. 선분AP의 중점을 M이라 하고, 원 k와 직선 BM의 두 번째 교점을 R이라 하자. 직선 PR이 원 k와 다시 만나는 점을 S라 할 때, AP와 CS가 서로 평행함을 증명하여라. (주니어발칸 2005-2)

증명 MP 2 = MA2 = MB ¢MR 이므로 4MPB » 4MRP 이고 \MPR = \MBP = \RBC =180± ¡ \RSC ¤

132. 원에 내접하는 사각형에서 각각 변의 중점에서 변에 수선을 내렸다. 이 네 수선이 한 점에서 만남을 보여라. (독일BW 1977 1차-4)

증명 중점사각형(평행사변형)의 중심에 해 O를 칭시킨 점. anticenter라고도 한다는 얘기있음.

IMO쇼트였다는 얘기도. ¤

133. 중심이 각각 O1, O2인 두 원이 두 점 A, B에서 서로 만난다. 두 원의 중심은 직선 AB에 하여 서로 반편에 위치하고 있다. 직선 BO1과 BO2는 각기 자신의 원과 B1, B2에서 다시 만난다. B1B2의 중점을

M이라 하자. M1, M2는 각각 원 O1, O2 위의 점으로 \AO1M1 = \AO2M2 이라 하자. 그리고, B1은작은 호 AM1 위에, B는 작은 호 AM2 위에 있다고 하자. \MM1B = \MM2B 임을 보여라.

(주니어발칸 2002-2)

증명 4AO1O2 ´ 4MO2O1 이므로 \MO1A = \MO2A. 그럼 \MO1M1 = \MO2M2. 또한

O1M1 = O1A = O2M , O1M = O2A = O2M2 이므로4MO1M1 ´ 4M2O2M . 고로 MM1 =MM2.

이제 B가 M1M2 위에 있음만 확인하면 된다.그건 각 계산 좀 해서 \M1BM2 = \M1BB1+\O1BO2+\O2BM2 = 180± 임을 확인하면 됨. ¤

134. 평행한 두 평면 ®와 ¯가 주어져있다. 구 ¾가 ®와 점 T에서 접한다. T를 지나는 서로 다른 두 직선 a, b가¯와 각각 C, D에서 만나고, 구 ¾와는 각각 A, B에서 만난다. 네 점 A, B, C, D가 한 원 위에 있음을 증명하여라. (몰도바 1998 최종-y11-3)

증명 a, b로 결정되는 평면 위에서만 생각하면 쉬움. 이 평면이 ®와 만나는 교선을 XY 라 하면(X가

C랑 같은 쪽), \BDC = \BTX = \BAT 등. ¤

135. 삼각형 ABC에서 변 AB, BC 위에 각각 점 D, E를 잡자. 선분 DE의 삼등분점을 K와 M이라 하자. 직선 BK와 BM이 변 AC와 만나는 점을 각각 T와 P라 할 때, TP · 1

3AC 임을 보여라.

(러시아 1990 4차-y11-6)

Page 193: 실전수학올림피아드 1400제 해답

3.2 기하 고급문제 193

증명 B를 중심으로 DE를 확 해도 삼등분점은 유지되므로, WLOG E = C 라 하고, D가 AB를 x : 1

로 내분한다고 하자. ATKC 순서라고 하자. 간단한 비례로 AT : TC = jKABj : jKBCj = 1 + x : 2,

AP : PC = jMABj : jMBCj = 2 + 2x : 1. 따라서, TPAC

= 1 ¡ 1+x3+x

¡ 13+2x

= 1 ¡ 1+x3+

23x

3+x¡

1+ 23x¡ 2

3x

3+2x= 13¡ 2x

3( 13+x

¡ 13+2x

) · 13. ¤

136. 삼각형 ABC의 변 BC의 중점을 F라 하자. 변 AB와 AC의 바깥쪽으로, D와 E에서의 꼭지각이 직각인이등변삼각형 ABD와 ACE를 그렸다. DEF도 직각이등변삼각형임을 증명하여라.

(아일랜드 1996-4, Towns 1991봄 JA4 변형)

증명 D는 BQ의 중점, E는 CG의 중점이라 하자. 4AQC ´ 4ABG. DF = 12QC = 1

2BG = FE.

AQBI는 cyclic. \QIB = \QAB = \R. ¤

137. 볼록육각형 ABCDEF에서

AB k CF; CD k BE; EF k AD

이다. 두 삼각형 ACE와 BDF의 넓이가 같음을 증명하여라. (Towns 1984가을 SO1)

증명 육각형에서 각 삼각형을 제외하고 남은 삼각형 조각들을 등적변형으로 비교. ¤

138. 한 선생님이 학급 학생들에게 다음과 같은 실습을 하도록 하였다:

반지름이 각각 1과 10인 두 동심원을 그리고, 작은 원에서 세 접선을 그려 둘씩 만나는 교점A, B, C가 모두 큰 원 안에 있도록 한다.

4ABC의 넓이를 S, 부채꼴과 비슷한 세 영역을 각각 S1, S2, S3이라 할 때(그림 참조),S1 + S2 + S3 ¡ S 를 구해보아라.

모든 학생의 결과가 똑같음을 증명하여라. (Towns 1985가을 J3)

증명 3£ (작은 반원)¡ 3£ (큰 반원) +원 = 2£ (준식) ¤

139. 마름모가 아닌 평행사변형 ABCD가 있다. \BAD의 이등분선이 직선 BC, CD와 각각 점 K, L에서 만난다. 세 점 C, K, L을 지나는 원의 중심은 B, C, D를 지나는 원 위에 있음을 보여라.

(Towns 1986봄 S5)

증명 원의 중심을 O라 하자. 4BKO ´ 4DCO 임을 보이면 됨(SAS). ¤

140. 예각삼각형 ABC에서 AB와 AC를 지름으로 하는 두 원을 각각 C1, C2라 하자. C2는 AB와 F에서 다시만나고, C1은 AC와 E에서 다시 만난다. 또한, 선분 BE와 원 C2의 교점을 P , 선분 CF와 원 C1의 교점을 Q라 하자. AP = AQ 임을 증명하여라. (중미 2000-5)

증명 C1과 C2의 A 아닌 교점을 D라 하면 AD ? BC. 4ABC의 세 수선 AD, BE, CF는 한 점(수심)

H에서 만난다. FBDH, ECDH는 cyclic이므로, AF ¢AB = AH ¢AD = AE ¢AC. 또, AC가 현 PEP 0을수직이등분하므로 \APH ´ \APP 0 = \ADP 로 4AHP » 4APD. 즉 AP 2 = AH ¢ AD = AQ2.¤

Page 194: 실전수학올림피아드 1400제 해답

194 기하

141. AD k BC 이고 AB ? AD 이며 내접원을 갖는 사다리꼴 ABCD가 있다. 내접원의 반지름을 R이라 하고각선의 교점을 M이라 하자. 4DCM의 넓이는 얼마인가? (Towns 1986가을 S1)

풀이 j4ABM j = j4CDM j 임을 보이고, 그로부터 열심히 풀면 넓이는 R2임을 알아낼 수 있다. 좀더

일반화하여, AB ? AD 의 조건이 없어도 이것은 성립함을 증명할 수 있다. 각선의 교점은 항상 내접원의 세로 지름 위에 있다. }

142. \A = 60± 인 예각삼각형 ABC가 있다. B와 C에서 그은 두 수선의 교각을 이등분하는 두 직선 중 하나가 외심을 지남을 증명하여라. (Towns 1987봄 JA4)

증명 두 수선의 교점을 F . 여러 개의 30±들이 발견됨. 각 F의 횡이등분선이 AB, AC와 만나는 점을

D, E라 하면 ADE는 정삼각형. DE 위에서 F와 칭이 되는 점을 O라 하면 O에서 AB, AC에 내린수선의 발이 중점이 되어 외심임을 확인할 수 있음. ¤

143. 예각삼각형 ABC에서 AD와 BE는 두 수선이다. [BDE] · [DEA] · [EAB] · [ABD] 일 때, 이 삼각형은 이등변삼각형임을 보여라. 단, [XY Z]는 삼각형 XY Z의 넓이를 나타낸다. (중미 2002-2)

증명 [BDE] · [ADE] 이면 [BDX] · [AEX] 이므로 [DAB] · [EAB]. 반 쪽 부등호도 마찬가지.

따라서, 등호 성립. 그럼 AB k DE. ABDE cyclic 이므로 등변사다리꼴. ¤

144. P와 Q는 각각 이등변삼각형 ABC의 두 등변 AB와 AC 위의 점으로, AP = CQ 를 만족한다. P , Q는이 삼각형의 꼭지점이 아니다. 삼각형 APQ의 외접원이 삼각형 ABC의 외심을 지남을 증명하여라.

(아일랜드 2006-2)

증명 ABC의 외심을 O라 할 때, APOQ가 cyclic임을 보이면 됨. 4OPA ´ 4OPQ (O를 중심으로

회전합동) 임에서 금방. ¤

145. P는 원 C 위의 고정점이고 Q는 직선 l 위의 고정점이다. R은 P , Q, R이 일직선을 이루지않는 한에서 원C 위를 움직이는 점이다. P , Q, R을 지나는 원이 직선 l과 다시 만나는 점을 V 라고 할 때, 직선 V R은 항상 한 고정점을 지남을 보여라. (아일랜드 2007-3)

증명 PQ, RV 와 원 C의 교점을 각각 S, X라 하자. PQV R, PSXR이 모두 cyclic이므로 \PSX =

180± ¡ \PRX = \PQV . 즉 X가 그 고정점. ¤

146. 어떤 반원의 지름 EF가 삼각형 ABC의 변 BC 위에 놓여있고, 이 반원이 변 AB, AC와 각각 점 P , Q에서 접한다. 직선 EP와 FQ의 교점 K가 A에서 BC로 내린 수선 위에 있음을 증명하여라.

(주니어발칸 2000-3, 알바니아 출제)

증명 E와 P가 같은 쪽에 있을 때를 K, 반 쪽에 있을 때를 L이라 하자. \PAQ = 180± ¡ \POQ =

180± ¡ 2\PEQ = 2(90± ¡ \PEQ) = 2\PKQ 이므로 K는 A를 중심으로 하고 P , Q를 지나는 원 위에 있다. L도 마찬가지이고 KL은 지름이므로 A 2 KL. 그럼 L은 KEF의 수심이므로 끝. ¤

147. 평면에 평행사변형 ABCD가 있다. A와 B를 지나면서 반지름 R인 원을 그리고, B와 C를 지나면서 반지름 R인 원을 또 그리자. 이 두 원의 또 하나의 교점을 E라 하고, E는 평행사변형의 꼭지점이 아니라고하자. ADE의 외접원도 반지름이 R임을 증명하여라. (북유럽 1987-2)

증명 BCE를 평행이동하여 ADF가 되도록 하면 AEDF cyclic by 원주각 or 원주 각 ¤

148. 삼각형 ABC에서 각 C의 이등분선이 변 AB와 만나는 점을 D라 하자. 삼각형 ABC의 넓이를 F라 할때, 다음 부등식을 증명하여라. (오스트리아-폴란드 2004-2)

2F

µ1

AD¡ 1

BD

¶· AB

증명 A, B를 지나 CD에 평행한 직선이 각각 직선 BC, AC와 만나는 점을 A0, B0이라 하자. 그럼

ABB0A0은 등변사다리꼴. 또, 2FAD ,2FBD는 각각 A0, B0에서 직선 AB에 이르는 거리. 그 두 거리의 차는

직각삼각형의 빗변 A0B0보다 작다. ¤

Page 195: 실전수학올림피아드 1400제 해답

3.2 기하 고급문제 195

149. 원 ¡ 위에 서로 다른 두 점 A, B가 있으며, AB는 지름이 아니다. P가 ¡ 위를 자유롭게 움직이는 점일때(단, P 6= A;B), 삼각형 ABP의 수심의 자취를 구하여라. (이탈리아 2006-3)

풀이 원 ¡와 P를 통째로 직선 AB에 해 선 칭시켜서 원 ¡0과 점 P 0이라 하면, PP 0\¡0 이 ABP의

수심이 됨. 즉, 자취는 ¡0. }

150. E는 정사각형 ABCD의 변 AB의 중점이고, 점 F , G는 EF와 AG가 평행이 되게 하는 선분 BC, CD 위의 점이다. FG는 정사각형의 내접원에 접함을 증명하여라. (헝가리 1960-3)

증명 BC, CD의 중점을 각각 H, K라 하면 HE+FK = EF 만 증명하면 충분함. AB = 2, HF = a

라 두고 피타고라스와 닮음을 이용해 걍 계산하면 됨. 좀더 좋은 방법이 있을까나? ¤

Page 196: 실전수학올림피아드 1400제 해답

196 기하

Page 197: 실전수학올림피아드 1400제 해답

제 4 장

조합

4.1 조합 중급문제

1. 체스판(8£ 8)에서 나이트는 가로 두 칸과 세로 한 칸, 혹은 가로 한 칸과 세로 두 칸이 떨어진 위치로 움직일 수 있다. 체스판의 한 귀퉁이에서 나이트가 출발하여 모든 칸을 정확히 한 번씩 들르고 각선 방향의 귀퉁이칸에 도착하는 것이 가능한가? (오클랜드 1997-7)

증명 흑칸 ! 백칸 ! 흑칸 으로 이동하게 되므로, 시작과 끝이 같은 색 칸이 되려면 이동은 짝수 번,

지나온 칸은 홀수 개. 모순. ¤

2. 1에서 n¡ 1까지의 자연수들 중에서 중복을 허용하여 n개의 수를 골랐는데 그 합이 2n보다 작았다. 이 수들 중에서 몇 개를 잘 고르면 합이 n이 됨을 증명하여라. (아일랜드 1988-6)

증명 a1, a1 + a2, a1 + a2 + a3, : : : ; a1 + a2 + ¢ ¢ ¢+ an 들 중 n으로 나눈 나머지가 같은 것이 있으면

그 둘의 차이가 역시 ai들의 부분합이면서 n의 배수가 된다. 나머지가 모두 다르다면 이들은 n개의 수이므로 n으로 나눈 나머지가 각각 하나씩 응되고, 그럼 이중에 n으로 나눈 나머지가 0인 것이 있다. ¤

3. 셈뛰기는 M가지 종목으로 이루어진 시합이다. A, B, C 세 명만이 참가하여 셈뛰기 시합이 열렸다. 각 종목에서 1등에게는 p1점이 수여되고, 2등에게는 p2점, 3등에게는 p3점이 수여된다. p1 > p2 > p3 > 0 이고 p1, p2, p3은 정수이다. 최종점수는 A가 22점, B가 9점, 그리고 C도 9점이었다. B는 100미터 달리기에서 우승했다. M은 얼마이고 또 높이뛰기에서 2등을 한 것은 누구인가? (캐나다 1986-2)

풀이 s = p1 + p2 + p3 이라 하자. 그럼 모든 점수의 합계 Ms = 22 + 9 + 9 = 40 이 된다. s ¸ 6,

M ¸ 2 이므로 (M; s) = (2; 20), (4; 10), (5; 8) 만 가능하다.

(i) M = 2 이면; 22 = A < 2p1 < 2B = 18 이므로 모순.

(ii) M = 4 이면; B ¸ p1 + 3, A < 4p1 이므로 p1 = 6 이어야 한다. 그럼 s = 10, p1 > p2 > p3 에서p2 = 3, p3 = 1 이다. 이 때 A · 3p1 + p2 = 21 이므로 모순.

(iii) M = 5이면; B ¸ p1+4, A < 5p1 이므로 p1 = 5. 그럼 p2 = 2, p3 = 1. 이 때, A = 5+5+5+5+2,B = 5 + 1 + 1 + 1 + 1 이므로 다음 표와 같다.

(1) (2) (3) (4) (5) 계A 2 5 5 5 5 22B 5 1 1 1 1 9C 1 2 2 2 2 9

따라서, 100미터 달리기 이외의 종목에서 2등을 한 것은 모두 C ¢ ¢ ¢ 답 }

별해 M ¸ 2 이고 B > p1 이므로 p1 · 8 이다.

(i) p1 = 8 이면 B = 8 + 1 이므로 M = 2인데 이는 22 = A < 2p1 에 모순.

Page 198: 실전수학올림피아드 1400제 해답

198 조합

(ii) p1 = 7 이면 B ¡ 7 = 2 이므로 M · 3 이고, 그럼 A < 3p1 에 역시 모순.

(iii) p1 = 6 이면 B에서 M · 4, A < Mp1 에서 M = 4. 그럼 B = 6 + 1 + 1 + 1 이고 p3 = 1.A = 6 + 6 + 6 + 4 또는 6 + 6 + 5 + 5 인데 어느 경우도 C가 9보다 커져서 모순.

(iv) p1 · 4 이면 B에서 M · 6, A < Mp1 에서 M = 6. 그러나, M - 22 + 9 + 9 이므로 p1 + p2 + p3이 정수가 될 수 없어 모순.

(v) 따라서, p1 = 5 이다. B에서 M · 5, A < Mp1 에서 M = 5. 그럼 B = 5 + 1 + 1 + 1 + 1, p3 = 1

이고, B 때문에 C는 3등을 기껏해야 1번밖에 안 했으므로 C ¸ 1 + 2 + 2 + 2 + 2 = 9. 등호가 성립할 때이므로 p2 = 2 이고, A = 2 + 5 + 5 + 5 + 5 가 된다.

높이뛰기 2등은 C이다. }

4. n(A)는 A의 원소의 개수, kAk는 A의 부분집합의 개수이다. kAk + 2kBk = kA [ Bk 가 성립할 때,n(A) = 2000이면 n(A \B)는 얼마인가? (셈본중등초급 도전문제 1.1.1)

풀이 n(A), n(B), n(A[B), n(A\B)를 각각 a, b, c, d라 하자. 그럼 문제의 조건에서 2a+2b+1 = 2c,

a = 2000 이다. 2의 거듭제곱에 관한 위의 식이 성립하려면 a = b+ 1 이고 c가 이들보다 1만큼 클 때뿐이다(2진법 전개를 생각해보면 쉽다). 따라서, a = 2000, b = 1999, c = 2001 이고, d = a+ b¡ c = 1998

¢ ¢ ¢ 답 }

5. 어떤 상자에 24개의 공이 있다. 그 중 2개는 빨간 공이고 나머지 22개는 파란 공이다. 준경이가 1에서 24까지의 자연수 중 하나를 골라 `n개'라고 선언하면, 정욱이는 상자 안에서 보지 않고 n개의 공을 하나씩 차례로 꺼낸다. 그래서 n번째 꺼낸 공이 처음으로 나온 빨간 공이면 준경이가 이기는 것으로 한다. 이길 확률이 최 가 되도록 하려면 준경이는 얼마를 불러야 하는가? (플란더즈 예선 1990/1991)

풀이 일종의 telescoping ¢ ¢ ¢ 답 1 }

6. 5행 N열의 5N개의 점이 있다.첫 행의 왼쪽부터 1; 2; : : : ; N ,그리고 다음 행의 왼쪽부터 N+1; : : : ; 2N ,이런 식으로 왼쪽에서 오른쪽으로 번호를 붙이고 다음 행으로 넘어가 다시 번호를 붙이는 방식을행우선번호라고 하자. 비슷하게, 열우선번호라는 것은 첫 열의 맨위부터 1; 2; 3; 4; 5, 다음 열의 맨위부터 6; 7; 8; 9; 10,

이런 식으로 위에서 아래로 번호를 붙인 후 다음 열로 넘어가는 방식이다. i번째 행에서 점 Pi를 각각 골라,

다섯 점 P1; P2; : : : ; P5를 뽑았다. 점 Pi의 행우선번호를 xi, 열우선번호를 yi라 하면, x1 = y2, x2 = y1,x3 = y4, x4 = y5, x5 = y3 이 된다는 것을 발견하였다. 이것이 가능한 N의 최소값을 구하여라.

(AIME 2001 1차-11)

풀이 %EEE

Suppose the row 1 point is in col a, the row 2 point in col b and

so on. Then a = 5b-3, b+N = 5a-4, c+2N = 5d-1, d+3N = 5e, e+4N = 5c-2.

First two equs give a = 5b-3, N = 24b-19. Last three give 124e = 89N+7.

Hence 124e = 2136b-1684. Now 2136b-1684 = 28b+52 mod 124, and by trial

smallest possible solution (for b and hence for N) is b = 7, giving

N = 149, e = 107. Continuing, we get a = 32, d = 88, c = 141. So it is

indeed a solution.

¢ ¢ ¢ 답 149 }

7. 집합 A는 n(A) ¸ 1000, A ½ f0; 1; 2; 3; : : : ; 1997g 을 만족한다. A의 원소 중에 2k (k ¸ 0 인 정수) 꼴이있거나 아니면 a+ b 가 2k (k ¸ 0 인 정수) 꼴이 되는 A의 서로 다른 두 원소 a, b가 존재함을 보여라.

(아일랜드 1997-8)

풀이 0에서 1997까지의 수를 한 집합 안에 2k 꼴인 수만 있거나, 집합에 포함된 두 수를 더하면 2k꼴

이 되도록 적절히 999개 묶음으로 나누는 방법을 찾는다.1997과 더하여 2k꼴을 만들 수 있는 수는 2048¡1997 = 51이다. 1996; 1995; : : :도 짝을 찾으면 (51; 1997); (52; 1996); : : : (1023; 1025)의973개의 묶음을 만들 수 있다.

같은 방법으로 50과 짝이 될 수 있는 숫자는 14이므로 (14; 50); (15; 49); : : : (31; 33)의 18개의 묶음을 만들 수 있다.13의 짝은 3이므로 (3; 13); : : : (7; 9)의 5개 묶음을 만들 수 있다.

종합하면 0; 1; : : : ; 1997의 집합을(51; 1997); (52; 1996); : : : ; (1023; 1025): 973개 묶음(1024): 1개.(14; 50); (15; 49); : : : ; (31; 33): 18개.

Page 199: 실전수학올림피아드 1400제 해답

4.1 조합 중급문제 199

(32): 1개.(3; 13); : : : ; (7; 9): 5개.(0; 1; 2; 4; 8): 1개.

총 973 + 1 + 18 + 1 + 5 + 1 = 999개의 묶음으로 나누었다. 즉 어떻게 A를 고르더라도 원소가 하나인2k 꼴의 묶음을 선택하게 되거나, 한 묶음 안에서 두 개의 숫자를 고르게 되어 그 합이 2k 꼴이 된다. 마지막 묶음에서 둘 이상의 숫자가 포함된 경우 (0; 1; 2; 4; 8)에서는 0만 2k꼴이 아니므로 이 경우에도 A의원소 중 2k꼴의 원소가 포함되게 된다. }

8. 1부터 64까지의 수를 가로, 세로 각각 8칸인 바둑판에 채워 넣되, 아래 형태의 그림에 든 수의 합은 항상5로 나누어지도록 할 수 있는가? (아래와 방향만 다른 형태의 그림도 모두 고려할 것.)

(통신강좌 1990-E3)

표 4.1:

풀이

A F G (H)a b c f g

B d C D Ee

O O O O

O O O O

문제에서 요구하는 로 64개의 수를 표에 채워넣을 수 있다고 한다면, 위 그림에서 A+ a+ b+ c ´ 0 ´B + a+ b+ c (mod 5)이므로, A ´ B (mod 5)이다. B + c+ d+ e ´ 0 ´ C + c+ d+ e (mod 5)이고,

비슷한 방법으로 B ´ C ´ D ´ E (mod 5)이다.F,G는 C,D를 이용하여 A와 5에 한 합동임을 알 수 있다. H ´ A (mod 5)도 어렵지 않게 보일 수 있으나 증명에서 꼭 필요하지 않으므로 생략한다.

결국 A에서 G 7칸과, O로 표시한 8칸 등 총 15개의 칸이 5에 해 합동이어야 하는데 1에서 64까지의수 중 5로 나눈 나머지가 같은 수는 최 13개 있으므로 문제에서 요구하는 로 표에 숫자를 채우는 것이 불가능함을 알 수 있다.

답: 불가능하다. }

9. 세 자리의 정수 중 2 또는 3의 배수이고 5의 배수도 7의 배수도 아닌 것들은 몇 개가 있는가?(셈본중등초급 도전문제 1.1.3)

풀이 집합적으로 나타내어 보자. 세 자리 정수 전체의 집합을 U라 하고, 그 중 2의 배수의 집합을

A, 3의 배수의 집합을 B, 5의 배수의 집합을 C, 7의 배수의 집합을 D라 하자. 그럼 우리가 원하는 것은 (A [ B) ¡ (C [ D) 의 원소의 개수이다. 이것은 다음과 같이 구할 수 있다. 단, E = C [ D 이고,jA \Bj = jABj 와 같이 표현하도록 하겠다.

X = j(A [B)¡ Ej = jA [Bj ¡ j(A [B)Ej= (jAj+ jBj ¡ jABj)¡ (jAEj+ jBEj ¡ jABEj)= jAj+ jBj ¡ jABj ¡ (jACj+ jADj ¡ jACDj)¡ (jBCj+ jBDj ¡ jBCDj)

+ (jABCj+ jABDj ¡ jABCDj)= jAj+ jBj ¡ jABj ¡ jACj ¡ jADj ¡ jBCj ¡ jBDj

+ jABCj+ jABDj+ jACDj+ jBCDj ¡ jABCDj

Page 200: 실전수학올림피아드 1400제 해답

200 조합

세 자리의 정수 중 n의 배수인 것은 f(n) = [ 999n]¡ [ 99

n]개가 되므로,

X = f(2) + f(3)¡ f(6)¡ f(10)¡ f(14)¡ f(15)¡ f(21)

+ f(30) + f(42) + f(70) + f(105)¡ f(210)

= 450 + 300¡ 150¡ 90¡ 64¡ 60¡ 43 + 30 + 21 + 13 + 9¡ 4

따라서, ¢ ¢ ¢ 답 412 }

주 jF ¡ Ej = jF [ Ej ¡ jEj 로 계산해도 된다. 그럼 X = jA [ B [ C [Dj ¡ jC [Dj 가 되고, 만일

포함-배제의 원리를 알고 있다면 위의 결과식이 더 쉽게 이해될 수 있다.

10. 태우에게는 서로 다른 5개의 수가 적힌 카드가 있다. 태우가 이 카드들을 적당한 순서로 한 장씩 지용이에게 보여줄텐데, 지용이는 카드가 보여지면 그 카드를 가질 것인가 버릴 것인가 바로 결정해야 한다. 지용이가 어떤 카드를 가지겠다고 결정하면 게임은 끝나고, 지용이가 택한 카드의 수가 태우가 가진 카드 중에서 가장 큰 수이면 지용이가 이긴다. 지용이는 처음 두 장의 카드는 그냥 버리고, 세 번째로 보여지는 카드부터 지금까지 나왔던 카드보다 큰 수가 나오면 그것을 갖는 것으로 전략을 세웠다. 지용이가 이길 확률은 얼마인가? (플란더즈 1992-2)

풀이 최 수가 k번째 나와서 지용이가 가질 확률을 ak라 하면, p = a1 + a2 + a3 + a4 + a5 =0 + 0 + 1

5+ 15¢ 23+ 15¢ 12= 1330

}

11. 여러 사람이 모인 자리에서 서로 아는 사람끼리만 한 번씩 악수를 하였다고 하자. 각 사람마다 악수한 횟수가 모두 같고 전체 악수의 횟수가 2005번일 때, 모인 사람의 수가 될 수 있는 가장 작은 수를 구하여라.

(한국 2005 1차-J15)

풀이 모인 사람의 수를 n이라 하고, 각 사람마다 악수한 (일정한) 횟수를 k라 하자. 각각의 악수는 두

사람 사이에 이루어지므로 각 사람마다 악수한 횟수를 모두 합하면 각각의 악수가 2번씩 세어져 전체 악수의 횟수의 2배가 된다. 즉, nk = 4010 이고, 4010 = 2 ¢ 5 ¢ 401 로 소인수분해되므로, n > k 의 조건을생각하면 n은 최소 401이 되어야 한다.

그리고 n = 401, k = 10 의 경우는 다음과 같이 실제로 구성될 수 있다: 401명을 원형으로 앉혀놓고, 각자 좌우로 거리 5 이하인 사람들끼리만 아는 것으로 한다. ¢ ¢ ¢ 답 401 }

12. 10명의 사람이 있는데, 각 사람에게 그 사람을 제외한 나머지 사람들의 나이의 합을 쓰라고 하였다. 그10개의 합으로 집합을 만들었더니 9개의 원소를 갖는 집합 f82; 83; 84; 85; 87; 89; 90; 91; 92g가 되었다. 이10명 각각의 나이를 모두 구하여라. (인도지역예선 1993-7)

풀이 10명의 나이를 a1, a2, : : : ; a10이라 하고, 이들의 나이의 합을 S = a1 + a2 + ¢ ¢ ¢+ a10 이라 하

자. 그럼

fS ¡ a1; S ¡ a2; : : : ; S ¡ a10g = f82; 83; 84; 85; 87; 89; 90; 91; 92g

로 10개의 수가 서로 다른 9개의 수로 나타나야 하므로 우변의 수들 중에서 어느 하나는 두 번 나타난다.

그것을 m이라고 하자. 그럼 두 집합의 수의 합이 같아야 하므로

(S ¡ a1) + (S ¡ a2) + ¢ ¢ ¢+ (S ¡ a10) = 82 + 83 + 84 + 85 + 87 + 89 + 90 + 91 + 92 +m

10S ¡ (a1 + a2 + ¢ ¢ ¢+ a10) = 9 ¢ 87 +m

9S = 9 ¢ 87 +m

따라서, m은 9의 배수이고, 주어진 수들중에서 9의 배수인 것은 90뿐이므로 m = 90 이다. 그럼 S = 97.

이로부터 열 명의 나이는 각각 5, 6, 7, 7, 8, 10, 12, 13, 14, 15살임을 알 수 있다. }

13. 어떤 풀리그 토너먼트에 5개의 팀이 참가했는데, 각 팀은 다른 팀과 꼭 한 번씩 결하고, 각 결에서는어느 한 팀의 승리가 반드시 결정된다. 각 팀이 다른 팀과의 결에서 승리할 확률은 모두 1

2로 같다고 하

자. 각 팀이 승리한 횟수가 모두 다를 확률을 구하여라. (AIME 1999-13 변형)

Page 201: 실전수학올림피아드 1400제 해답

4.1 조합 중급문제 201

풀이 %EEE

(원래 문제 풀이)

Team totals must be 0, 1, 2, ... , 39. So we must be able to order

the teams as T_1, T_2, ... , T_40, so that T_i loses to T_j for i < j.

In other words, this order uniquely determines the result of every game.

There are 40! such orders and 780 games, so 2780 possible outcomes for

the games. Hence prob = 40!/2^780.

¢ ¢ ¢ 답 15=128? }

14. 어떤 반의 학생들이 급우들에게 각자 적어도 한 통 씩의 연하장을 보냈다.그럼 x1 ! x2 ! ¢ ¢ ¢ ! xn ! x1과 같이 꼬리를 물며 연하장을 보낸 몇 명의 학생이 있음을 보여라. (통신강좌 1991-2-29)

증명 아무나 한 학생을 골라 x1이라 하자. x1이 연하장을 보낸 친구 중 아무나 한 명을 골라 x2라 하

자. 이렇게 x1 ! x2 ! x3 ! ¢ ¢ ¢ 를 계속 잡을 수 있다. 그러다가 처음으로 이미 위의 목록에 등장했던학생이 다시 등장하는 경우가 반드시 생긴다(비둘기집). 그 학생이 xi에서 처음으로 등장하고 xj에서

다시 등장했다면, xi ! xi+1 ! ¢ ¢ ¢ ! xj¡1 ! xi 가 우리가 찾는 연하장 꼬리물기의 한 예가 된다. ¤

별증 (조철현)

우를 증명하자. D가 기본회로를 가지지 않으면 D에는 길이가 가장 긴 기본 통로가 존재하므로 그 통로가 지나는 정점의 집합을 X라 하자. 이 때 D가 방향그래프이므로 X에 끝 정점이 존재하는데, 이 정점에서 X 중의 한 정점으로 나아간다면 기본 회로가 생겨 모순, X 이외의 정점으로 나아간다면 가장 긴통로라는 조건에 모순이 되므로 가장 긴 기본 통로의 끝 정점의 나가는 차수는 1이상일 수 없다. 따라서,D의 각 정점에서 나가는 차수가 모두 1이상일 수 없다. 즉, 방향그래프 D의 각 정점에서 나가는 차수가모두 1이상이면 D는 기본 회로를 갖는다. ¤

15. 1; 2; 3; : : : ; 9 의 9개의 숫자로 두 수를 만들어서 곱했을 때, 최소가 되는 두 수를 구하여라. 단, 아홉 개의숫자를 모두 정확히 한 번씩 사용해야 한다. (셈본중등초급 도전문제 3.3.2)

풀이 (1) 한 자리 수가 있을 경우:

1£ 23456789 = 23456789 (¤)2£ 13456789 = 26913578

이고 나머지 경우들은 (1£ 3) ¢ 10000000 = 30000000 보다 크므로 이들보다 크다. 따라서 이 경우(¤)이 최소이다.

(2) 두 수 모두 두 자리 이상의 수일 경우:

두 수 A, B의 맨 앞 두 자리만 생각해보면

A£B ¸ (13£ 23) ¢ 100000 > 29000000으로 모두 (¤)보다 훨씬 크다.

따라서, 최소의 곱을 만드는 두 수는 ¢ ¢ ¢ 답 1, 23456789 }

16. 첫 네 항이 1, 9, 8, 1로 주어진 수열이 있다. 이후 모든 항은 그 바로 앞의 네 항의 합의 일의 자리 숫자가된다. 이 수열에 1, 2, 3, 4가 연속하여 나오는 경우가 있는가? (유고슬라비아 1981 고1-3a)

풀이 홀 홀 짝 홀 홀 꼴로 주기 5로 반복됨. 불가능. }

17. 12개의 꼭지점과 16개의 선분이 아래 그림과 같이 주어져 있다.

² ² ² ²

² ² ² ²²

²² ²

꼭지점을 지나지 않는 임의의 곡선을 그린다. 곡선이 선분의 한 쪽에서 다른 쪽으로 지날 때 `교차한다'고한다. 예를 들어 원은 한 개의 선분을 가지고 있고 이 때 접선은 이 선분과 `교차'하지 않는다. 모든 선분을 딱 한 번씩 교차하는 곡선을 그릴 수 없음을 증명하여라. (소련 1961-1)

Page 202: 실전수학올림피아드 1400제 해답

202 조합

증명 각각의 영역마다 내부에 점을 하나씩 찍고, 각각의 선분마다 그 선분을 가로지르는 곡선을 그리

는데 양끝점이 인접한 두 영역에 그려진 점이 되도록 한다. (이렇게 그려진 그래프를 원래 그래프의 쌍(dual) 그래프라고 한다.) 그럼 차수가 홀수인 점이 4개 있으므로 한붓그리기가 안 된다. ¤

18. 어떤 선거 운동에서 여러 정당들에 의해 p가지 서로 다른 공약들이 선언되었다(p > 0). 여러 정당이 같은공약을 선언할 수 있고, 어떤 두 정당도 최소한 하나의 공통 공약을 갖는다. 그러나 어떤 두 정당도 그 공약들의 집합이 완전히 같지는 않다. 정당의 수는 2p¡1을 넘지 않음을 보여라. (캐나다 1972-8)

증명 전체 공약의 집합을 T라 하자. T의 두 부분집합 A와 T ¡A를 하나로 묶는 식으로 하면 T의 모

든 부분집합은 둘씩 묶여 총 2p=2 = 2p¡1 묶음을 이룬다. 만일, 정당의 수가 2p¡1을 넘으면, 비둘기집의 원리에 의해 같은 묶음에 속하는 두 정당이 있고, 그 두 정당은 서로소이므로 모순. 따라서, 정당의수는 2p¡1을 넘을 수 없다. ¤

19. 한중학교의 학생수는 500명이고 동아리의 수는 20개이다. 이제 학생들에게 1부터 500까지의 고유번호를 부여하고 20개의 동아리를 C1; C2; : : : ; C20 으로 나타낼 때, 동아리 Ci에 속한 학생수는 (i + 10)이라고 한다. 고유번호가 j인 학생이 속한 동아리의 수를 dj라고 할 때, 다음 값은 얼마인가? (단, 한 학생이

여러 개의 동아리에 동시에 가입할 수도 있고, 또는 아무 동아리에도 가입하지 않을 수도 있다.)

d1 + d2 + ¢ ¢ ¢+ d500 (한국 2003 1차-J9)

풀이 구하는 값이 각 학생이 활동하고 있는 동아리의 수를 모두 합친 것인데 이 값은 각 동아리에 등

록된 회원수를 모두 더한 것과 같다. 그러므로

d1 + d2 + ¢ ¢ ¢+ d500 = C1 + C2 + ¢ ¢ ¢+ C20 = 11 + 12 + ¢ ¢ ¢+ 30 = 410:

¢ ¢ ¢ 답 410 }

20. A에서 B까지 최단경로로 갈 때 가는 방법은 모두 몇 가지인가? (2000 대전.충남 영재수학교실 2차평가)

¢ ¢ ¢ 답 42(가지)

21. 지름이 1인 100개의 공을 10 £ 10 £ 1 크기의 상자에 넣을 수 있음은 명백하다. 같은 상자에 105개의 공을 넣으려면 어떻게 할까? 106개의 공은 어떻게 넣을 수 있을까? (플란더즈 2005-2)

풀이 (과천고 2학년 홍성준) (1) 105 = 9£ 5 + 10£ 6 에서 힌트를 찾는다.

그림과 같이 3개의 공이 서로 접하는 방식으로 10, 9, 10, 9, 10, 9, 10, 9, 10, 9, 10개 의 11줄로 공을 넣으면 가로로는 10이므로 문제 없고, 세로로는

5£p3 + 1 ; 9:660 < 10

A B

Page 203: 실전수학올림피아드 1400제 해답

4.1 조합 중급문제 203

이므로 넣을 수 있다.(2) 또, 106 = 9£ 4 + 10£ 7 이므로 10개씩 넣는 줄을 늘려본다. 즉, 10, 10, 9, 10, 9, 10, 9, 10, 9, 10,10개로 넣으면 세로가

4£p3 + 3 ; 9:928 < 10

이므로 역시 넣을 수 있다. }

22. 7£ 7 크기의 판의 두 칸을 검은색으로 칠하고 나머지 칸은 흰색으로 남겨둔다. 이렇게 해서 만들 수 있는판의 모양은 모두 몇 가지인가? 단, 판을 돌려서 같은 모양이 되는 경우는 한 가지로 본다.

(AIME 1996-7)

풀이 %EEE

There are 49·48/2 = 1176 ways of choosing 2 cells. In 24 cases the

two cells are diametrically opposite. In the other 1152 they are not.

The 1152 form groups of 4 related by rotation, so 288 distinct. The 24

form 12 groups of 2. So total 300.

¢ ¢ ¢ 답 300 }

23. 임의의 자연수 n에 해, n의 자릿수의 합과 n+ 2의 자릿수의 합의 차를 t(n)으로 정의하자. 예를 들어,t(199) = j19¡ 3j = 16 이다. t(n)이 가질 수 있는 값 중에 2000보다 작은 것은 모두 몇 개인가?

(AIME 1999-5)

풀이 %EEE

If there are k carries in going from n to n+2, then the digit sum of

n+2 is 9k-2 smaller than that of n. So possible values of t(n) are 2 and

7+9k, for k = 0, 1, 2, ... . Largest < 2000 is k = 221: 7 + 221·9 = 1996.

Hence 223 values < 2000.

¢ ¢ ¢ 답 223 }

24. 다음과 같이 다섯 개씩의 |와 }가 있다.

|}|}|}|}|}

이들을 서로 이웃한 둘의 자리를 바꾸는 움직임만으로, |들을 모두 오른쪽으로 모으고, }들은 모두 왼쪽으로 모으고 싶다. 몇 번의 움직임이면 되겠는가? 일반적으로 두 모양이 각각 n개씩 있다고 하면 몇 번의움직임이면 되겠는가? 당신이 구한 답이 최소임을 증명하여라. (셈본중등초급 도전문제 5.1.1)

풀이 편의상 | = X, } = O 이라고 하자. XOXOXOXOXO를 OOOOOXXXXX로 만들면 된다. 다

음과 같이 옮기면 된다.OXOXOXOXOX 5번OOXOXOXOXX 4번OOOXOXOXXX 3번OOOOXOXXXX 2번OOOOOXXXXX 1번

그럼 총 1 + 2+ 3+ 4+ 5 = 15번 움직였다. O 왼쪽에 X가 모두 k개 있다면 O가 이 X들 모두의 자리를넘어가려면 최소 k번의 움직임이 필요하다. 처음 각각의 O에 해 이런 k들을 계산하면 1, 2, 3, 4, 5이므로, 총 1+ 2+ 3+ 4+ 5 = 15번보다 더 적게 움직여서는 O를 모두 왼쪽으로 모으는 것이 불가능하다.}

25. 45각형의 모든 꼭지점에 0; 1; : : : ; 9 중 하나씩의 번호를 매기는데, i 6= j 인 모든 쌍 fi; jg에 해서, 양끝점이 i, j로 번호가 붙은 변이 있도록 할 수 있는가? (소련 1963-11)

풀이 i = 0 일 때 모든 j = 1; : : : ; 9에 해 양끝점이 i, j인 변이 있으려면 0으로 이름붙은 점은 적어

도 5개 있어야 한다. 다른 번호를 i로 생각해도 모두 마찬가지이므로, 각 번호마다 5개씩, 적어도 총 50개의 점이 필요한데 꼭지점은 45개뿐이므로 불가능하다. }

주 셈본에도 비슷한 문제가... 한붓그리기. 꼭지점이 50개라면 딱 된다(가능하다). 10개의 번호를 꼭

지점으로 하여 K10을 그리고, 두 점씩 짝을 지어 5개의 변을 추가로 그린다. 그럼 모든 점의 차수가 짝수이므로 출발점으로 돌아오는 한붓그리기가 가능하고, 이 한붓그리기를 펼쳐그리면 원하는 50각형이된다.

Page 204: 실전수학올림피아드 1400제 해답

204 조합

26. 1972년부터 몇 년간 달력을 모으면 모든 해의 요일을 정확히 표시할 수 있는 달력을 모두 수집한 것이 될까? (호주 수학경시)

풀이 1972년은 윤년이고 4년마다 윤년이 반복되며 다시 2000년은 윤년이 아니다. 또한 윤년이 아닌

해는 365 = 7 ¢ 52+ 1이므로 부분 해가 지나면 요일이 하루씩 미뤄진다. 윤년인 경우에는 이틀씩 미뤄지는데, 1월 1일부터 2월 28일까지는 윤년에서 다음해로 넘어갈 때 요일이 이틀 미뤄지고, 3월 1일부터12월 31일까지는 윤년인 해에 요일이 이틀 미뤄진다.

모든 해의 달력을 정확히 표시하려면 모든 윤달(2월)의 요일을 표시할 수 있어야 한다. 윤년에서 다음 윤년이 될 때 요일 변화를 살핀다.1972년의 2월 1일이 월요일이라고 하면 1973년 수요일, 1974년 목요일, 1975년 금요일, 1976년 토요일로 요일이 이틀 당겨지게 되는데 7과 2는 서로소이므로 일곱번째 윤년이 돌아오면 2월 1일이 월요일부터 일요일까지 모든 요일이 순환하게 된다.1972년부터 4 ¢ 6 + 1 = 25년간, 즉 1996년까지 달력을 모으면 윤년을 모두 모을 수 있다. 이 때 4년마다윤년인 규칙에 예외가 없다.25년간 달력을 모으면 평년 달력도 요일별로 모을 수 있음을 보이자. 1973년 1월 1일이 월요일이라면4년 뒤에는 토요일인 식으로 월,토,목,화,일,금(1993년)요일이 있고, 1월 1일이 화요일인 1985년 다음해인 1986년의 1월 1일은 수요일이므로 모든 종류의 평년 달력을 모을 수 있다.

답: 25년. }

27. 음이 아닌 정수들의 수열 fakg가 모든 k ¸ 1 에 해 ak ¸ a2k + a2k+1 을 만족한다.

(가) 모든 자연수 n에 해, 수열 fakg에는 n개의 연속한 0이 항상 있음을 보여라.

(나) 문제의 조건을 만족하면서 0이 아닌 항을 무한히 많이 갖는 수열의 예를 하나만 찾아 제시하여라.(루마니아 2005 지역예선 y9-4)

풀이 (가) ak가 0이 아니면 a2k와 a2k+1 중에 적어도 하나는 ak보다 작다. 즉, 0이 아닌 항에 해서

그보다 작은 항을 항상 찾을 수 있으므로, 음 아닌 정수들에서는 더 작은 항을 계속 쫓아가다보면 유한번 안에 0인 항을 만날 수 밖에 없다. 그리고, ak = 0 이면 a2k = a2k+1 = 0 으로 2개의 연속한 0이 나

타나게 되고, 그럼 다시 a4k = a4k+1 = a4k+2 = a4k+3 = 0 으로 4개의 연속한 0이 나타나게 되고, 귀

납적으로 임의의 자연수 m에 해 2m개의 연속한 0이 항상 나타나게 된다. 따라서, 임의의 자연수 n에해 n개의 연속한 0이 항상 있다.

(나) 다음의 수열이 한 예가 된다.

1; 1; 0; 1; 0; 0; 0; 1; 0; 0; 0; 0; 0; 0; 0; 1; 0; 0; 0; 0; 0; 0; 0; 0; : : :

즉, k = 2n 꼴일 때 ak = 1 이고 그외에는 모두 0인 수열. (이외에도 예는 얼마든지 많다.) }

28. 8학년 학생들의 장기 토너먼트에 두 명의 7학년 학생이 참가하도록 승인되었다. 각 경기자는 다른 참가자와 꼭 한 번씩 경기를 하여, 이긴 경우에 1점, 비긴 경우에 0.5점, 진 경우에 0점 씩을 받았다. 두 명의 7학년 학생은 둘이 합해 모두 8점을 얻었고, 각 8학년 학생들은 서로 똑같은 점수를 얻었다고 한다. 이 장기

회에 참가한 8학년 학생들은 모두 몇 명인가? 답은 유일한가? (캐나다 1976-3)

풀이 8학년 참가학생 수를 n명, 이들이 얻은 똑같은 점수를 k점이라고 하자. 매 경기마다 두 사람이

받는 점수의 합이 1로 일정하므로 참가자들이 얻는 총점은 전체 경기의 수와 같게 된다.

kn+ 8 =³n+ 22

´=(n+ 2)(n+ 1)

2

즉, n2 + (3 ¡ 2k)n = 14 이다. 좌변이 n의 배수이므로 n j 14, 즉 n = 1; 2; 7; 14 말고는 불가능하다.n = 1; 2 일 때에는 k < 0 이 되어 곤란. n = 7 이면 k = 4 이고, n = 14 이면 k = 8 이다. (n; k) = (7; 4)

는 9명의 학생이 모든 경기에서 비긴 경우의 예가 있고, (n; k) = (14; 8) 은 같은 학년 학생끼리는 모두비기고 8학년은 7학년에게 1승 1무씩을 거둔 경우의 예가 있다. 따라서, n = 7; 14 둘다 가능하고 답은유일하지 않다. }

29. 4£ 10£ 19 크기의 합동인 94개의 벽돌이 있다. 이 벽돌들을 한 장씩 계속 쌓아올려서(즉, 높이는 4 또는10 또는 19씩 계속 높아진다) 만든 탑의 높이는 몇 가지가 생길 수 있는가? (AIME 1994-11)

Page 205: 실전수학올림피아드 1400제 해답

4.1 조합 중급문제 205

풀이 %EEE

Suppose x bricks are oriented to add 4 to the height, y to add 19

and z to add 10. Since 5·10 = 3·4 + 2·19 we can take z = 0, 1, 2, 3,

or 4. Also we must have y = 94-x-z, so height = 4x+10z+19(94-x-z) =

1786-15x-9z. If z=0, height = 1786-15x = 1 mod 5; if z=1, height =

1777-15x = 2 mod 5; if z=2, height = 1768-15x = 3 mod 5; if z=3,

height = 1759-15x = 4 mod 5; if z=4, height = 1750-15x = 0 mod 5.

So these values of x,z all give different heights. We can check that

1750 > 15·94 = 1510, so for z=0 there are 95 possible values of x

(0, 1, ... , 94), for z=1, 94 etc. Hence in total 95+94+93+92+91 = 465

possible heights.

¢ ¢ ¢ 답 465 }

30. 상식이는 가위바위보를 하는 로보트를 만들었다. 이 로보트는 가위, 바위, 보를 낼 확률 p, q, r을 주인이입력하면 그 확률 로 결정하여 가위바위보를 한다. 근영이가 친구들과 가위바위보를 하는 것을 관찰했더니 가위를 자주 내는 버릇이 있었고, 그 확률은 가위가 1

2, 바위와 보는 각각 1

4씩이었다. 상식이는 가위

바위보 로보트를 근영이와 결시키려고 한다. 단판 승부로 할 때(비겨도 다시 가위바위보를 하지 않는다고 할 때), 이길 확률이 최 가 되도록 하는 p, q, r을 입력시키고 싶다.

(1) q의 값은 얼마로 해야 하는가?

(2) 그 때, 이길 확률은 얼마가 되는가? (IT꿈나무 올림피아드 2006 1차)

풀이 상식이의 로보트가 이길 확률 X는 p+ q+ r = 1 임에서 X = 14p+ 1

2q+ 1

4r = 1

4+ 14q 이고, 이

것은 q = 1 일 때 최 값 12을 갖는다. ¢ ¢ ¢ 답 (1) 1 (2) 1

2}

31. 한 학급에서 3문제짜리 수학시험을 보았다. A문제를 푼 학생은 전체의 1=6, B문제를 푼 학생은 전체의1=2, C문제를 푼 학생은 전체의 2=3이었다. A와 B문제를 동시에 푼 학생은 A를 푼 학생의 2=3이었고, A,B, C를 모두 푼 학생은 전체의 1=12이었다. 아무것도 풀지 못한 학생은 전체의 1=12이었고, B와 C만을푼 학생수는 A와 C를 동시에 푼 학생수와 같았다. A문제만 푼 학생은 없고, B문제만 푼 학생이 9명이었다면, 전체 학생은 모두 몇 명인가? (2000 대전.충남 영재수학교실 2차평가)

풀이 전체 학생 수를 x라고 둔다.

A만 푼 학생이 없으므로n(A) = n(A \B) + n(A \ C)¡ n(A \B \ C)) x

6= x9+ n(A \ C)¡ x

12

) n(A \ C) = 5x36

:

다음 n(B \ C ¡A) = n(A \ C)를 이용한다.n(B) = n(B \ A) + n(B \ C ¡ A) + n(B ¡A¡ C) = x

9+ 5x36+ 9 = x

2) x4= 9) x = 36:

¢ ¢ ¢ 답 36(명) }

32. 어떤 아파트 블록에는 아이들이 있는 부부들만이 살고 있다. 이 아파트의 모든 남자아이에게는 여자 형제가 있고, 이 아파트에는 여자아이보다 남자아이가 더 많다. 이 아파트에 아이들보다 어른들이 더 많을 수있는가? (뉴질랜드 2000-1)

풀이 모든 집에는 여자아이가 있다. 남자아이가 여자아이보다 더 많다. 끝. }

33. 평면은 유한개의 직선에 의해서 몇 개의 영역으로 나뉜다. 이 때 같은 경계선에 접해 있는 영역이 서로 다른 색깔로 칠해지기 위해서는 2가지 색깔만 써도 됨을 보여라. (통신강좌 1991-2-33)

풀이 (서울과학고 박종원)

직선의 갯수를 n이라 할 때 수학적 귀납법으로 증명해 보자.(1) n = 1일 때는 명백하다.(2) n = p (p ¸ 1)일 때 성립한다고 하고 n = p+ 1일 때를 생각해 보자.

Page 206: 실전수학올림피아드 1400제 해답

206 조합

일단 p개의 직선만 그으면 가정에 의해 성립한다. 이제 하나의 직선 l을 추가하자. 그리고 l을 기준으로한 쪽에 있는 영역들은 색깔을 반전시키자.

그러면 l의 한 쪽 부분씩만 보면 그것들은 반전되어도 같은 경계선에 접해있는 영역이 서로 다른 색깔로칠해져 있음은 명백하다. 이제 문제가 되는 것은 l을 경계선으로 하는 영역들이다. 그런데 l을 경계로 하는 영역들은 l이 없을 때는 한 영역이었으므로 같은 색이었는데 l의 한 쪽을 반전시켰으므로 다른 색이되었다.) n = p+ 1일 때도 성립.) 수학적 귀납법에 의해 모든 n(¸ 1)에 해 성립. }

34. 한 변의 길이가 1인 정사면체 ABCD의 꼭지점 A에서 한 마리의 개미가 출발하여 모서리를 따라 기어간다. 이 개미는 어느 꼭지점에 다다를 때마다, 어느 모서리를 따라왔는지에 해서는 잊어버리고, 세 개의 모서리 중에서 각각 1

3의 동등한 확률로 선택하여 계속 기어간다. 거리 7을 기어간 후 도착한 꼭지점이

A일 확률은 얼마인가? (AIME 1985-12)

풀이 %EEE

Let p_n be the prob of return after distance n. Then p_{n+1} =

(1-p_n)/3, because to return after n+1, it must have been at a different

vertex after n, and then chosen the correct edge to return. Obv p_1 = 0,

so p_2 = 1/3, p_3 = 2/9, p_4 = 7/27, p_5 = 20/81, p_6 = 61/243, p_7 = 182/729.

¢ ¢ ¢ 답 182=729 }

35. 다음은 가로 세로 각선의 합이 모두 같은 마방진이다. 이 마방진을 완성하여라.(플란더즈 예선 1999/2000 1차)

2 ¡1x

풀이 한 줄의 합이 3x. ... 셈PS에 있는 풀이를 이용... }

36. 4 £ 4 크기의 표의 각 칸에 숫자 1 또는 2를 썼다. 이 표의 3 £ 3 부분영역에 있는 숫자들을 모두 합하면항상 4의 배수가 되는데, 표 전체의 숫자의 합은 4의 배수가 아니라고 한다. 이 16개의 숫자의 합의 가능한 최 값과 최소값을 구하여라. (이탈리아 2005-3)

풀이 (KAIST 06학번 김태우)

3£ 3 부분영역의 수의 합은 최소 9, 최 18이므로, 4의 배수가 될 수 있는 합은 최소 12, 최 16이다.

최소값 12가 9보다 3이 큼에서 2가 3개 이상 있어야 하고, 최 값 16이 18보다 2가 작음에서 1이 2개 이상 있어야 함을 알 수 있다. 그리고, 2가 3개 있는 경우나 1이 2개 있는 경우는 아래와 같이 실제로 가능하다.

1 1 1 1

1 1 2 1

1 2 2 1

1 1 1 1

2 2 2 2

2 1 2 2

2 2 1 2

2 2 2 2

따라서, 16개 전체의 숫자의 합은 최소 19, 최 30이다. }

ll

Page 207: 실전수학올림피아드 1400제 해답

4.1 조합 중급문제 207

37. 어떤 상자에 900장의 카드가 있는데, 각각 100부터 999까지의 수가 하나씩 적혀있다. 영락이는 이 상자에서 몇 장의 카드를 보지 않고 꺼내어 각 카드에 적힌 수의 자릿수의 합을 계산한다. 영락이가 꺼낸 카드중에 자릿수의 합이 같은 세 장의 카드가 있다는 것을 확신할 수 있으려면 꺼낸 카드는 최소 몇 장이 되어야 하는가? (플란더즈 예선 1994/1995)

풀이 세 자리 수의 자릿수의 합으로 가능한 것은 최소 1(100인 경우)부터 최 27(999인 경우)까지이

다. 1에서 27까지 중 자릿수를 더했을 때 그 합이 되는 수가 3개 이하인 것은 1과 27(각각 한 개씩만 있음)뿐이다. 그러므로 2에서 26까지의 수 중 같은 것이 적어도 세 장 있으려면 25 ¢ 2 + 1 = 51장이 있어

야 하고, 100, 999가 포함되었을 경우를 생각하면 51 + 2 = 53장이 필요하다. ¢ ¢ ¢ 답 53장 }

주 자릿수의 합이 2인 경우는 2가지 있으므로 사실 고려하지 않아도 상관없다. 1가지뿐인 1과 27의 경

우만 특별히 취급하면 충분하다.

38. 1에서 1000000까지의 자연수 중에서, 완전제곱수 하나와 완전세제곱수 하나의 합으로 표현될 수 있는 자연수(예를 들어 33은 52 + 23 과 같으므로 그런 자연수)와 그렇지 못한 자연수는 어느 쪽이 더 많은가?

(뉴질랜드 2001-4)

풀이 완전제곱수는 1000개, 완전세제곱수는 100개가 있다. 이 둘이 조합하여 만들어지는 수가 모두

다르다고 하더라도 최 1000 ¢ 100 = 100000개의 수밖에 만들어지지 않는다. 즉, 그렇지 못한 수가900000개 이상이므로 더 많다. }

39. 몇 가지 크기의 공 65개가 두 개의 상자에 나뉘어 담겨있다. 각각의 공의 색깔은 하양, 검정, 빨강, 노랑중 한 가지이다. 같은 색의 공으로 다섯 개를 어떻게 골라도 그 중 어느 두 개의 공은 크기가 같다. 같은상자 안에 담긴 같은 크기 같은 색의 세 개의 공이 있음을 증명하여라. (인도지역예선 1990-1)

증명 같은 색의 공으로 다섯 개를 어떻게 골라도 그 중 어느 두 개의 공은 크기가 같다고 하였으므로,

각각의 색의 공은 크기가 네 종류씩뿐이다. 어느 한 쪽 상자에 33개 이상의 공이 있고, 그 중 어느 한 가지 색의 공은 9개 이상 있고(모두 8개 이하이면 4£ 8 = 32 < 33 으로 모순), 그럼 그 중 어느 한 크기의공은 3개 이상 있다(모두 2개 이하이면 4£ 2 = 8 < 9 로 모순). ¤

40. 어떤 토너먼트시합에서 각각의 두 사람은 서로 꼭 한 번씩 결한다. 각 결에서 이기면 1점을, 비기면12점을, 지면 0점을 받는다. 가장 낮은 점수를 받은 10명의 집합을 S라 하자. 모든 선수가 자기가 받은 점

수의 정확히 절반을 S의 선수와 결하여 얻었음이 확인되었다. 이 토너먼트에 참가한 사람은 모두 몇 명인가? (AIME 1985-14)

풀이 %EEE

n+10 players. The 10 worst played 45 games amongst themselves.

They must have got between them 45 points from these games and hence 90

points in total. The top n players played n(n-1)/2 games amongst themselves,

giving them n(n-1)/2 points. Hence they got n(n-1) points in total (the

rest from the 10 worst). So the total points scored by everyone were 90 +

n(n-1). But total is (n+10)(n+9)/2. Hence n^2 - 21n + 90 = 0, n = 6 or 15.

But the top n get n(n-1) points in total, an average of n-1 each, and the

bottom 10 get an average of 9 each. Hence n ≥ 10. So n = 15.

¢ ¢ ¢ 답 25 }

41. 2명의 소년이 정육면체의 12모서리를 빨강과 파랑의 색으로 색칠한다. 먼저 첫 번째 소년이 세 모서리를골라 빨강으로 색칠하고, 다음 두 번째 소년이 파랑으로 다른 세 모서리를, 다시 첫 번째 소년이 또 다른세 모서리를 골라 빨강 칠을 하고, 마지막으로 두 번째 소년이 남은 세 모서리를 파랑으로 색칠한다. 첫번째 소년이 어느 한 면의 네 모서리가 모두 빨강 칠이 되도록 할 수 있는가? (통신강좌 1991-3-32)

풀이 면이 모두 6개이므로 변 3개를 잘 고르면, 어떤 면에도 그 중 한 변이 속하게 할 수 있겠다(1). 이

제 아래 그림에서 a, b, : : : , `의 12개의 모서리를 다음과 같이 네 집합으로 분할하자.

fa; f; lg; fb; g; ig; fc; h; jg; fd; e; kg:

각 집합의 세 원소는 서로 꼬인 위치에 있어 (1)을 만족시키게 된다. 첫 번째 소년이 어떻게 세 변을 골라도 두번째 소년의 차례에 이 네 집합 중 빨강 칠이 되지 않은 세 변을 포함하는 집합이 적어도 하나 있

Page 208: 실전수학올림피아드 1400제 해답

208 조합

다. 이 집합의 세 변을 모두 파랑으로 칠하면, 색칠이 끝났을 때 빨강 칠의 모서리로 둘러싸인 면이 없게된다.

그러므로, 두번째 소년이 최선의 방해를 한다면 첫번째 소년은 결코 빨강 모서리의 면을 얻을 수 없다.}

42. 서로 다른 실수들 x1; x2; x3; : : : ; x40 이 주어져있다. 처음 두 항 x1과 x2를 비교하여 만일 x2 < x1 이면두 항을 서로 바꾼다. 그 다음, 두 번째 항과 세 번째 항을 비교하여 뒷항이 더 작다면 두 항을 서로 바꾼다. 이런 식으로 39번째 항과 40번째 항에 이를 때까지, 이웃한 두 항을 비교하여 뒷항이 더 작다면 두 항을 바꾸는 것을 계속하여 반복한다. 처음에 수열의 순서가 임의로 주어졌을 때, x20이 30번째 항이 되어끝날 확률을 구하여라. (AIME 1987-13)

풀이 문제의 조건은 x1; x2; : : : ; x31 중에서 x31이 가장 크고 x20이 두 번째로 크다는 것과 같다. 따라

서, 이럴 확률은 130¢31 =

1930 이다. }

43. 모든 정수들에 빨간색 또는 파란색을 칠한다고 하자. 어떤 두 빨간점도 거리 d만큼 떨어져 있지 않고, 어떤 두 파란점도 거리 1만큼 떨어져 있지 않도록 하려면, d는 어떤 수가 되어야하는가? (IMTS R13-3)

풀이 빨간점이 n과 n+1에 두 번 연속하여 나타났다고 하자. 그럼 이들과 거리 d만큼 떨어진 n+ d와

n + d + 1은 빨간점이 될 수 없으므로 둘다 파란점이다. 그럼 거리 1만큼 떨어진 파란점이 생겼으므로모순. 따라서,

² 빨간점도 파란점도 같은 색 점이 연속하여 나타날 수 없다.

그렇다면 빨간점 다음은 파란점, 파란점 다음은 빨간점, ... 이렇게 빨파빨파빨파빨파빨파... 로 칠하는방법만이 유일하다. 이 때, 두 빨간점 사이의 거리는 모든 짝수가 가능하고 홀수는 나타나지 않으므로,d는 모든 홀수가 될 수 있다. }

44. 어떤 주사위에는 1, 2, 3의 눈만이 각각 a, b, c개의 면에 그려져 있다. 이런 주사위 2개를 동시에 계속 던지면 두 주사위의 눈의 합이 3이 될 때가 3회에 1회 꼴로 나온다고 한다. c는 얼마인가?

(1999 교육청경시)

풀이 합이 3이 되려면 한 쪽이 1, 한 쪽이 2여야 하고, 이 확률이 1=3이므로 다음과 같이 계산한다.a6¢ b6¢ 2 = 1

3) a ¢ b = 6:

이러한 자연수 a; b는 순서에 상관없이 1; 6이거나 2; 3인데, a + b · 6이어야 하므로 2; 3이다. 따라서c = 6¡ a¡ b = 1:

답: 1: }

45. 네 쌍의 부부가 모인 어느 모임에서 두 사람씩 조를 이뤄 네 경기의 체스게임을 동시에 했다.

여B씨는 남E씨와 뒀다.

여A씨는 여C씨의 남편과 뒀다.

남F씨는 남G씨의 부인과 뒀다.

여D씨는 여A씨의 남편과 뒀다.

남G씨는 남E씨의 부인과 뒀다.

남H씨는 누구와 결혼했는지 알 수 있겠는가? 당신의 답이 옳음을 증명하여라. (플란더즈 1995-1)

f

k

c

g

j l

i

e

a

bd

h

Page 209: 실전수학올림피아드 1400제 해답

4.1 조합 중급문제 209

풀이 남E씨는 여B씨와 경기했으므로, E씨는 여C씨의 남편(여A씨와 경기한)도 아니고 여A씨의 남

편(여D씨와 경기한)도 아니며 또한 E씨의 부인은 남G씨와 경기했으므로 여B씨일 수도 없다. 따라서,E씨의 부인은 D씨다. 그럼 G씨와 경기한 것도 D씨이다. 따라서, A씨의 남편은 D씨와 경기한 G씨이다.G씨의 부인인 A씨는 F씨와 경기하였고, 그럼 C씨의 남편은 A씨와 경기한 F씨이다. 따라서, 남은 짝은B씨와 H씨뿐이다. ¢ ¢ ¢ 답 여B씨 }

46. 집합 N = f1; 2; : : : ; ng 에 해 조건 X ½ Y ½ N 이 만족되도록 집합 X, Y 를 구성할 수 있는 모든 경우의 수를 구하여라.

풀이 N의 각각의 원소에 해 (1) X에도 포함되지 않는 것, (2) X에는 포함되지만 Y 에는 포함되지

않는 것, (3) Y 까지 모두 포함되는 것 등 3가지를 독립적으로 결정할 수 있다. 따라서, 구하는 경우의 수

는 3n ¢ ¢ ¢ 답 }

47. 상자 안에 빨간 구슬 m개와 파란 구슬 n개가 있고, m + n · 1991 이다. 두 개의 구슬을 임의로 꺼내면,

그 두 구슬이 같은 색일 확률이 12이라고 한다. m의 가능한 최 값을 구하여라. (AIME 1991-13)

풀이 %EEE

Simplifying, (m-n)^2 = m+n. Put N = m+n. Then larger of m, n is

(N + √N)/2, so we also want N to be as large as possible. Hence N

must be largest square <= 1991, namely 44^2 = 1936, giving 990, 946.

¢ ¢ ¢ 답 990 }

48. 다음의 4£ 4 크기의 숫자판에는 각 칸마다 숫자가 하나씩 적혀있고, 가로 혹은 세로로 연속한 세 칸의 숫자의 합은 모두 같다고 한다.

33

23

0 17 (가)

(나)

(가)와 (나)에 들어갈 숫자를 차례로 써라. (IT꿈나무 올림피아드 2006 1차)

풀이 연속한 네 칸의 수를 a, b, c, d라 하면 a+ b+ c = b+ c+ d 이므로 a = d 이다. 즉, 가로나 세로

로 세 칸 이동한 곳에 서로 위치하는 두 수는 같다. 그럼 우선 다음을 알 수 있다.

33 33

23 23

0 17 (가) 0

33 33

그럼 연속한 세 칸의 합은 33 + 23 + 0 = 56 이다. 따라서, 0 + 17 + (가) = 56 이므로 ¢ ¢ ¢ 답 (가) 39(나) 33 }

49. 반지름 1인 원에 내접하는 임의의 사각형에 해, 가장 짧은 변의 길이가p2보다 작거나 같음을 보여라.

(캐나다 1969-9)

증명 원의 중심을 O라 하고, 사각형의 네 꼭지점을 차례 로 A, B, C, D라 하자.

\AOB, \BOC, \COD, \DOA 등 네 각의 합은 360± = 4 ¢ 90± 이므로, 비둘기집의 원리에 의해 이네 각 중에는 90±보다 작거나 같은 것이 있다. 일반성을 잃지 않고 \AOB · 90± 라 하면, A와 B는 반지름 1인 4분원에 포함되고, 따라서 AB · p2 이다. ¤

Page 210: 실전수학올림피아드 1400제 해답

210 조합

50. 어떤 다면체가 있는데, 임의의 꼭지점에서 출발해서 변을 따라 움직이다가 처음의 꼭지점으로 돌아오면어떻게 움직여도 그동안 변을 따라 꼭지점을 이동한 횟수가 항상 짝수번이 된다고 한다. 이 다면체의 꼭지점을 두 가지 색으로 칠하는데, 변으로 이웃한 점은 항상 서로 다른 색이 되도록 할 수 있음을 보여라.

(통신강좌 1996-12-19)

증명 먼저 주어진 그래프 G가 연결된 그래프 G1; G2; ¢ ¢ ¢ ; Gr로 분리된다고 하자.

그리고 각 선의 길이를 1이라고 하고, 임의의 점 P (2 Gi)를 기준으로 통로의 길이가 n만큼 떨어진 점에(¡1)n이라는 색을 칠하자. (n은 0 또는 자연수)

그렇다면 이때 임의의 Gi에서 점 X(2 Gi)에서 Y (2 Gi)로 가는 통로가 2개 이상 있고, 그중 한 통로 I에해 I의 길이가 ¯라 하자. 이제 X의 색이 ®(¡1 또는 1)라 하면 Y 의 색은 ® ¢ (¡1)¯이다. 그런데 다른

통로 II에 해 II의 길이가 °이고 ¯ 6´ ° (mod 2)라면 XI¡! Y

II¡! X라는 회로의 길이 ¯ + °는 홀수가 되어 모순이다. 따라서 ¯ ´ ° (mod 2)이고 임의의 점 X에서 Y 로 가는 모든 통로의 길이는 모두 짝수이거나 홀수이다. 그러므로 각 점의 색깔은 유일하게 1 또는 ¡1로 정해지고 인접한 점은 서로 다른 색으로 칠해지게 된다. ¤

51. 1 이상 126 이하의 자연수 중에서 8개의 서로 다른 자연수를 임의로 택하였다. 그 중에서 다음 조건을 만족하는 두 수 x, y를 항상 찾을 수 있음을 보여라.

1 <y

x· 2 (통신강좌 1996-13-26)

풀이 비둘기 집의 원리를 이용한다. 한 묶음에서 임의의 두 수를 꺼냈을 때 x < y · 2x의 관계가 성립

하도록 1에서 126까지의 수를 나눌 수 있을까?

한 묶음에서 가장 작은 수를 x라고 하면 같은 묶음의 모든 수는 2x 이하여야 한다.(1; 2)

(3; 4)(5; : : : ; 8)(9; : : : ; 24): : :(26 + 1; : : : ; 126)

위와 같이 7개의 묶음으로 나누면 8개의 숫자를 골랐을 때 적어도 한 묶음에 두 수가 있어 그 두 수가x < y · 2x이다.}

52. G는 11개의 꼭지점을 갖는 다면체이다. G의 어떤 꼭지점도 다른 모든 꼭지점과 변으로 연결되어 있는 경우는 없다. G의 변을 모두 지우고 G에서 서로 연결되어 있지 않았던 두 점을 모두 새로운 변으로 이어 그래프 G를 만들자. G와 같은 연결구조를 갖는 다면체는 존재하지 않음을 보여라. (통신강좌 1997-14-19)

증명 G의 변의 수를 e, G의 변의 수를 e0이라 하면 e+ e0 =¡112

¢= 55 이다. 그런데 다면체(혹은 평면

그래프)는

(변의수) 5 3 ¢ (꼭지점의수)¡ 6

가 되므로(셈본 중급 참조), e; e0 · 3 ¢ 11¡ 6 = 27 이 된다. 여기서 55 = e+ e0 · 54 이므로 모순. ¤

53. 집합 A = fa; b; cg 에서의 함수 f : A ! A 중에서, f(f(x)) = f(x) 를 항상 만족하는 것은 모두 몇 개인가? (플란더즈 예선 1996/1997 1차)

풀이 먼저 f(x) = x; x = a; b; c인 함수는 f(f(x)) = f(x) = x를 항상 만족한다.

f(x) = y(x 6= y)라고 가정한다. 이 때 f(f(x)) = f(x) ) f(y) = y 이므로, f(x) = y이면 f(y) = y이다. 이 때 x, y와 다른 z가 있을 때, f(z) = x이면 f(x) = x여야 하므로 모순이고, f(z) = y, f(z) = z인경우에는 모순이 생기지 않는다. 즉, (f(x); f(y); f(z)) = (x; y; z); (y; y; y); (y; y; z)가 가능하다.x; y; z에 a; b; c를 다르게 입할 수 있으므로 이러한 함수 f는 1 + 3 + 3 ¢ 2 = 10개 있다.¢ ¢ ¢ 답 10개 }

54. S는 6개의 원소로 이루어진 집합이다. 합집합이 S가 되도록 S의 두 부분집합을 택하는(두 부분집합은 서로 같을 수도 있다) 방법은 모두 몇 가지인가? (AIME 1993-8)

Page 211: 실전수학올림피아드 1400제 해답

4.1 조합 중급문제 211

풀이 %EEE

Let the subsets be A and B. For each element we have three choices

(A, B or both). That gives each pair of subsets twice except for the

case A = B = S. Hence (3^6 + 1)/2.

¢ ¢ ¢ 답 365 }

55. 각 면이 삼각형 또는 오각형인 32개의 면을 갖는 다면체가 있다. V 개의 전체 꼭지점 각각에는 T개의 삼각형과 P개의 오각형이 만난다. 100P + 10T + V 의 값은 얼마인가? 오일러의 공식 V + F = E + 2 를이용해도 좋다(단, F는 면의 개수, E는 모서리의 개수이다). (AIME 1993-10)

풀이 %EEE

V + 30 = E (Euler). VT + VP = 2E (counting edges). So V(T+P-2) = 60.

Also, there are VT/3 triangles and VP/5 pentagons, so V(T/3 + P/5) = 32.

Hence 32V(T+P-2) = 32·60 = 60V(T/3 + P/5). Hence 3T + 5P = 16.

Hence P + T = 2. Hence V = 30.

¢ ¢ ¢ 답 250 }

56. 1부터 100까지 숫자가 하나씩 적힌 구슬 100개가 일렬로 배열되어있다. 우리에게는 특별한 정리기계가있는데, 이 정리기계는 우리가 100개 중에서 50개를 적당히 골라주면, 그 기계는 이 50개의 구슬만을 오름차순으로 정리하여 원래의 50개의 위치에 순서 로 둔다. 예를 들어, 6개 중에서 3개를 정리해주는 기계라면, 1 3 2 4 6 5에서 3 2 6을 골라 그 기계에게 정리시키면 1 2 3 4 6 5가 된다. 다음은 이 정리기계를다섯 번 사용하여 1부터 100까지 오름차순으로 항상 정리할 수 있음을 증명하여라.

(IT꿈나무 올림피아드 2006 1차)

증명 100개의 구슬의 위치를 왼쪽부터 25개씩 묶어 A, B, C, D그룹이라고 하고, 두 그룹 X, Y에 있는

50개의 구슬을 정리시키는 것을 (X,Y)로 나타내기로 하자. 그럼 (A,B), (C,D), (A,C), (B,D), (B,C) 순으로 정리시키면 된다. 처음 두 번의 시행을 거치면 1부터 25까지의 수는 A그룹 또는 C그룹에 있게 되고, 76부터 100까지의 수는 B 또는 D그룹에 있게 된다. 그 다음 두 번의 시행을 거치면 A그룹은 1부터25까지 오름차순으로 정리되고 D그룹은 76부터 100까지 오름차순으로 정리된다. 마지막시행을 거치면모두 오름차순으로 정리된다. ¤

57. 어떤 나라는 몇 개의 `주'로 지역이 분할되어 있다. 각 주는 하나의 폐곡선으로 경계지어져 있다(한 주가여러 덩이로 분할되어 여기저기 존재하거나 하지 않는다). 이 나라의 지도를 만들고자 하는데, 변으로 인접한 두 주는 항상 서로 다른 색으로 나타내고자 한다. 이 나라에서는 둘씩 서로 이웃한 네 주를 찾을 수없지만, 지도를 만들기 위해서는 네 가지 색이 필요하다고 한다. 이런 지도도 있음을 예를 들어서 보여라.

(통신강좌 1991-2-34)

풀이 (서울과학고 박종원) 다음과 같은 그래프를 생각해 보자.

D;E; F는 같은 색일 수 없다. 따라서 °(G) ¸ 3. °(G) = 3이라고 하자. D는 1색, E는 2색, F는 3색이라고 하면 A는 3색, B는 2색일 수 밖에 없고, C는 1색, 2색, 3색인 점과 모두 연결되어 있으므로 결국3가지 색으로 칠하는 것은 불가능하다. 따라서 °(G) = 4이다.(이 때 C는 4색으로 하면 된다.) 또 G의어떤 부분그래프도 K4를 포함하지 않는다. }

58. (1) 어떤 나라에는 2개의 도시와 n개의 소도시가 있다. 이 나라에 통신망을 구성하는데, 하나의 통신선은 두 도시를 연결하며, 2n ¡ 1개의 통신선만으로 모든 도시를 직간접적으로 연결하고자 한다. 단, 도시끼리 혹은 소도시끼리 통신선이 연결되어서는 안 된다. 가능한 통신망의 연결구조는 모두 몇 가지인가?(2) n개의 도시가 있다고 하고 같은 문제를 풀어라. (통신강좌 1997-14-21)

A

E

F

B

C

D

Page 212: 실전수학올림피아드 1400제 해답

212 조합

풀이 (1) 각 통신선은 도시와 소도시 사이만을 연결해야 한다. 도시와 소도시 사이를 연결하는 통

신선은 2n종류가 있는데, 그 중에 딱 하나만을 제외해야 하는 것이므로 방법은 2n가지. n = 1 일 때는이것은 불가능하고, n ¸ 2 일 때는 항상 모두 연결되어있다. (2) 2n개의 모든 도시가 연결되기 위해서는적어도 2n¡ 1개의 선이 있어야 하는데, 딱 그 개수만큼만 선을 쓸 수 있으므로 연결구조는 나무가 된다.

즉, 두 도시 a와 b를 연결하는 경로가 있어야 하므로 a, b에 동시에 연결된 소도시 x가 있다.

만일 x 이외에 다른 소도시가 a, b와 연결되어 있다면, 꼭지점 4개로 구성된 회로가 존재하게 되어 나무임에 모순이다. 따라서, x가 될 소도시를 하나 택하고, 나머지 소도시들은 a에 연결할지 b에 연결할지를각각 선택하면 된다. 즉, 총 경우의 수는

¡n1

¢ ¢ 2n¡1 = n ¢ 2n¡1 이다. }

59. 어떤 파티에 20명의 남자와 20명의 여자가 있다. 각각의 남자는 x명의 여자를 좋아하고 각각의 여자는y명의 남자를 좋아한다. 둘이 서로 좋아하는 남녀는 한 쌍도 없다면, x+ y 의 최 값은 얼마인가?

(IT꿈나무 올림피아드 2006 1차)

풀이 (남자A,여자B) 쌍을 모으는데, A가 B를 좋아하거나 B가 A를 좋아하는 쌍을 모두 모은다고 하

자. 서로 좋아하는 경우는 없으므로, 그럼 이런 쌍은 모두 20x+ 20y개가 된다. 그런데, 가능한 (남자,여자) 쌍은 총 20£ 20개이므로 20(x+ y) · 202, 즉 x+ y · 20 이다. 실제 x = 0, y = 20 일 때 등호가 성

립하는 경우가 있음을 쉽게 알 수 있다. ¢ ¢ ¢ 답 20 }

60. 네 개의 동전이 있는데 세 개는 똑같은 무게의 진짜 동전이지만, 나머지 하나는 가짜로 무게가 다르다. 저울을 이용해 가짜를 찾고 싶은데, 이 저울은 동전을 하나만 올려놓으면 저울의 측정범위보다 가벼워서 무게를 측정하지 못하고, 2개 이상 올려놓아야 정확한 무게를 수치로 나타내준다고 한다. 이 저울을 4번만사용하여 가짜도 찾고 그것이 진짜 동전보다 가벼운지 무거운지까지 알아내는 방법을 찾아라.

(오클랜드 2001-5)

풀이1 네 동전을 a, b, c, d라 하고

a+ b+ c; a+ b+ d; a+ c+ d; b+ c+ d

의 무게를 각각 재자. 그럼 가짜가 포함되지 않은 한 경우만 무게가 다르고(X라 하자), 나머지 세 경우는가짜가 하나 포함되어 무게가 모두 같다(Y 라 하자). 즉, X에 포함되지 않은 동전이 가짜이고, X > Y

이면 가짜가 가벼운 경우, 반 는 가짜가 무거운 경우이다. }

주 위의 풀이에서 네 번 잰 무게를 모두 합한 후 3으로 나누어주면 전체 무게의 합 a+ b+ c+ d가 된

다. 여기서 위에서 잰 각각의 무게를 빼어주면 a, b, c, d의 무게를 알 수 있고, 그로부터 결론을 얻어도된다.

풀이2 네 동전을 a, b, c, d라 하고

a+ b; a+ c; a+ d

를 각각 재자. 만일 이 세 번의 무게가 모두 같으면(Y 라 하자) b, c, d는 모두 같은 무게이고 따라서 a가가짜이다. 그럼 진짜 동전 2개의 무게 X = b+ c 를 재어 X > Y 이면 가짜가 가볍고 그 반 는 가짜가무거운 경우가 된다. 만일 a가 진짜이면 가짜는 b, c, d 중에 하나이므로 세 번 중 진짜만으로 잰 두 번은무게가 같고(X라 하자) 가짜가 섞인 한 번은 무게가 다르다(Y 라 하자). 역시 X, Y 의 무게 비교에 따라같은 방식으로 결론을 내리면 된다(이 때는 저울을 세 번만 사용하게 된다). }

풀이3 P = a + b + c + d, Q = b + c + d, R = c + d, S = b + d 를 각각 재자. 그럼 a = P ¡ Q,

b = Q¡R, c = Q¡ S, d = R+ S ¡Q 로 각각 직접 계산해낼 수 있다. 그럼 진짜와 가짜가 분명히 드러나고 어느 것이 무거운지도 확인된다. }

주 이외에도 많은 방법이 가능하다.

ξ

βα

Page 213: 실전수학올림피아드 1400제 해답

4.1 조합 중급문제 213

61. 양팔저울과 몇 개의 추를 이용해 1 g, 2 g, : : : ; n g의 질량을 모두 측정할 수 있게 하고 싶다. 예를 들어,1 g의 추와 3 g의 추가 있으면 n = 4 까지 가능하다. 이 때, 2 g의 질량은 1 g의 추와 3 g의 추를 각각 서로다른 팔에 올리는 방법으로 측정할 수 있다. (1) 3개의 추로 가능한 최 의 n을 구하여라. (2) 4개의 추로가능한 최 의 n을 구하여라. (IT꿈나무 올림피아드 2006 1차)

풀이 (1) 숫자 0, 1, 2 신 숫자 ¡1, 0, 1을 이용한 3진법에 연관시킬 수 있다. 그럼 ¡13부터 13까지

측정가능하고 이 27가지를 측정하는 경우가 최 임을 확실히 알 수 있다. (왼쪽 접시와 오른쪽 접시의추를 모두 서로 바꾸어놓는 경우를 생각하면 §k의 측정이 항상 쌍으로 함께 존재해야 한다.) 실제로 1 g,3 g, 9 g의 추로 최 13 g까지 측정할 수 있다.(2) 마찬가지로 1 g, 3 g, 9 g, 27 g의 추로 최 40 g까지 측정할 수 있다.

¢ ¢ ¢ 답 (1) 13 (2) 40 }

62. 다음과 같이 주어진 3£ 3 크기의 부호판이 있다.

+ ¡ +

¡ + ¡+ ¡ +

여기서 한 가로줄 혹은 한 세로줄을 마음 로 택해 그 줄에 있는 세 칸의 부호를 모두 바꾸어 놓을 수 있

다. 예를 들어 가운데 세로줄을 택하면+ + +

¡ ¡ ¡+ + +

와 같이 바뀐다. 이런 작업만을 반복할 수 있을 때, 다

음 중 만들어낼 수 있는 부호판을 모두 골라라. (IT꿈나무 올림피아드 2006 1차)

(1)¡ + ¡+ ¡ +

¡ + ¡(2)

+ + +

+ + +

+ + +

(3)+ ¡ +

¡ ¡ ¡+ ¡ +

(4)¡ ¡ ¡¡ + ¡¡ ¡ ¡

(5)+ ¡ +

+ ¡ +

¡ + ¡

풀이 (1), (2), (5)는 쉽게 만들 수 있다. (3), (4)는 만들 수 없는데, 2£ 2 부분영역을 택했을 때 어떻

게 작업해도 그 부분영역 안의 +부호의 개수의 홀짝이 변하지 않아야 함을 관찰하면 증명된다. ¢ ¢ ¢ 답○○××○ }

63. 한 변의 길이가 1인 정삼각형 타일과 정사각형 타일이 각각 30개, 10개가 있다. 이것들을 이용해 정사면체나 정육면체, 사각뿔 등 여러 가지 다면체를 여러 개 만들 수 있다. 최 몇 개의 다면체를 만들 수 있는가? (같은 모양의 다면체를 여러 개 만들어도 되고, 몇 종류인지를 세는 것이 아니라 몇 개인지를 세는 것이다.) (IT꿈나무 올림피아드 2006 1차)

풀이 면의 개수가 가장 적은 다면체는 사면체뿐이고, 따라서 하나의 다면체를 만들 때 각각 적어도 4개

의 타일이 필요하다. 타일이 모두 40개이므로 많아야 10개의 다면체를 만들 수 있다. 그런데, 10개를 만들 수 있다면 모두 사면체로 만들어야 하는데 사각형 타일도 있으므로 이것은 안 된다. 9개를 만드는 것은 예를 들어 다음과 같이 가능하다: 정사면체 6개와 삼각기둥 3개. ¢ ¢ ¢ 답 9개 }

64. 정육면체의 각 면에 서로 다른 자연수를 하나씩 써넣는다. 또, 각 꼭지점에는 그 점에서 만나는 세 면의수의 합을 써넣는다. 그리고, 여덟 꼭지점에 부여된 이 합들을 다시 모두 합하여 S라고 한다. 각 면에 여섯 개의 자연수를 처음에 어떻게 써넣었더라도 이 S는 항상 a의 배수가 된다. a의 최 값을 구하여라.

(IT꿈나무 올림피아드 2006 1차)

풀이 각 면의 수는 그 면과 접한 4개의 꼭지점에 포함된다. 즉, 6면의 수를 u; v; w; x; y; z라 하면 S =

4(u+ v + w + x+ y + z) 가 된다. ¢ ¢ ¢ 답 4 }

65. 볼록다각형에 모든 각선을 그었다. 이 도형의 변이나 각선에 각각 방향성을 적당히 잘 줘서 루프가 생기지 않게 할 수 있음을 보여라. (A ! B ! C ! A 와 같이 선의 방향성 로 따라가서 원래 자리로 돌아오는 것을 루프라고 한다.) (독일BW 1974 1차-4)

증명 꼭지점에 순서를 줘서 항상 순서가 빠른 점에서 늦은 점을 향하는 방향이 되도록 하면 됨. ¤

Page 214: 실전수학올림피아드 1400제 해답

214 조합

66. 정육면체의 각 면을 4개의 작은 정사각형으로 4등분하고, 각각의 작은 정사각형을 3가지 색 중에서 골라색칠한다. 공통변을 갖는 인접한 두 정사각형은 서로 다른 색이어야 한다. 각각의 색이 꼭 8번씩 쓰였음을 증명하여라. (Towns 1988가을 JO4)

증명 정육면체의 각 꼭지점 X에 해, X에서 만나는 작은 세 정사각형의 모임을 생각. 모두 8개의 모

임이 있으므로 한 가지 색이 9번 이상 쓰이면 비둘기집의 원리에 의해 같은 모임에 그 색이 두 번 쓰였고 그럼 인접한 정사각형이 생기므로 모순. ¤

67. 다면체의 각 꼭지점마다 자연수를 하나씩 붙이는데, 한 모서리의 양끝점의 두 수는 서로소가 아니고 이웃하지 않은 두 점의 수는 서로소가 되도록 할 수 있음을 증명하여라. (소수는 무한히 많음을 가정해도 좋다.) (Towns 1988가을 SO3)

증명 각 변마다 소수 하나씩 주면 끝. ¤

68. (a) 정18각형의 꼭지점들을 번갈아 흑백으로 색칠하였고, 여기서 두 사람이 게임을 한다. 두 사람은 번갈아서, 같은 색의 두 꼭지점을 잇는 각선을 하나씩 그린다. 이 각선들은 서로 도중에 교차할수 없다. 마지막으로 각선을 그린 사람이 이기는 것으로 할 때, 두 사람 모두 최선의 전략으로 게임을 한다면 누가 이길까?

(b) 정20각형에서 같은 게임을 하면 결과가 어떻게 될까? (Towns 1988봄 JO3 변형)

69. 컴퓨터 화면에 수 123이 나타나있다. 매분마다 컴퓨터는 화면의 수에 102를 더한다. 컴퓨터 전문가 미샤는 그가 원할 때마다 화면에 나타난 수의 자릿수의 순서를 마음 로 바꿀 수 있다. 화면에 네 자리의 수가나타나지 않도록 할 수 있는가? (Towns 1991봄 JO4)

풀이 할 수 있다. 예를 들어 123 : 312 ! 414 ! 516 ! 618 ! 720 : 027 ! 129 ! 231 : 312 ! ¢ ¢ ¢로 반복시킬 수 있다. }

70. 모든 10자리 수들의 집합을 다음의 두 부분집합으로 분할할 수 있다: M은 두 5자리 수의 곱으로 나타낼수 있는 모든 10자리 수들의 집합이고, N은 나머지 10자리 수들의 집합이다. M과 N 중에 어느 집합이큰가? (Towns 1988봄 SO3)

풀이 두 5자리의 수를 104 · a · b < 105 이라 하면 이런 쌍의 개수는 a = 104; 104 + 1; : : : ; 105 ¡ 1일 때를 차례로 생각하면 (105 ¡ 104) + ¢ ¢ ¢ + 3 + 2 + 1 = 1

2(105 ¡ 104)(105 ¡ 104 + 1) ¸ jM j. 즉,

2jM j < 1010 ¡ 109(= jM j+ jN j) 임을 확인할 수 있어서 jM j < jN j. }

71. 1; 2; : : : ; n의 재배열 중에서, 첫항이 아닌 각각의 k에 해(1 · k · n) 그보다 왼쪽에 k ¡ 1, k + 1 중에적어도 하나가 항상 있는 것은 모두 몇 개인가? (Towns 1988가을 SA3)

풀이 1:::n의 배열의 처음에 수 하나를 더 끼워넣는 것을 생각하면 pn+1 = 2pn. 즉 pn = 2n¡1. }

72. 1989개의 수로 된 집합이 있는데, 여기서 10개의 수를 임의로 골라 합하면 항상 양수가 된다. 이 집합의모든 원소의 합도 양수임을 증명하여라. (Towns 1989가을 JO3)

증명 모든 가능한 10개 세트를 모두 합하면 각 수는 동일한 횟수(a라 하자. a는 그 한 수를 제외한 나

머지 1988개의 수에서 그 수와 세트를 이룰 9개의 수를 고르는 경우의 수임)씩 더해지므로, aS > 0 이됨. 즉, S > 0. ¤

73. 10명의 친구들이 서로에게 연하장을 보냈다. 각자 5통씩 보냈다고 한다. 그럼 서로 연하장을 보낸 두 명이 있음을 증명하여라. (Towns 1989가을 SO1)

증명 두 친구의 쌍은 12¢ 10 ¢ 9 = 45가지. 연하장은 5 ¢ 10 = 50통으로 친구쌍보다 더 많으므로 어떤 두

연하장은 같은 쌍 사이에 오갔음(비둘기집). ¤

74. 9£ 9 판의 각 칸에 파리가 한 마리씩 앉아있다. 어떤 신호가 떨어지자, 각 파리가 각방향으로 이웃한 칸으로 동시에 움직였다. 그럼 한 칸에 여러 마리의 파리가 있거나 아무 파리가 없을 수도 있게 된다. 빈칸은 최소 몇 개인가? (러시아 1989 4차-y9-2)

Page 215: 실전수학올림피아드 1400제 해답

4.1 조합 중급문제 215

풀이 흑백으로 판을 칠하면(코너가 흑색) 백색 칸끼리는 짝을 이룰 수 있으므로 OK. 흑색칸들은 다시

각방향으로 인접한 것이 서로 다른 색이 되도록 흑청으로 칠하면(코너가 흑색) 흑색칸이 청색칸보다9개가 많아서 빈칸은 최소 9개... }

75. \날으는 룩"은 보통의 체스 룩(차)처럼 가로 또는 세로 방향으로 몇 칸이든 움직이지만 인접한 칸으로는바로 갈 수 없다. 4£ 4 체스판에서 날으는 룩이 16번 움직여 모든 칸을 들러 원위치로 돌아오는 것이 가능한가? (Towns 1991가을 JO2)

풀이 가능한 것으로 기억함. }

76. 어떤 왕국에 32명의 기사가 있다. 이 중 몇몇은 다른 누군가의 가신이다. 한 명의 가신은 한 명의 주인만섬기며, 주인은 누구나 그의 가신보다 부자이다. 4명 이상의 가신을 둔 기사를 바론이라 한다. 바론은 최

몇 명인가? 단, 이 왕국의 법에 의하면, 내 가신의 가신은 내 가신으로 보지 않는다.(Towns 1991가을 JA1)

풀이 가신은 최 31명이므로 바론은 최 7명. 7명일 때 실제 가능함이 쉽게 확인됨. }

77. 1에서 12까지의 수가 원주 위에 임의의 순서로 배열되어있다. 이웃한 두 수의 차가 1보다 크면 그 두 수의자리를 바꿀 수 있다. 이런 과정을 유한번 반복하여 이 수들을 크기 순서 로 배열할 수 있음을 보여라.

(러시아 1989 4차-y8-3)

증명 1을 2에 이웃할 때까지 옮기고, (1 2)를 3에 이웃할 때까지 옮기고, (1 2 3)을 4에 이웃할 때까지

옮기고... 하면 됨. ¤

78. 같은 크기의 한 세트의 책 20권이 책꽂이에 임의의 순서로 일렬로 꽂혀있다. 도서관 사서는 이를 제1권부터 제20권까지 왼쪽에서 오른쪽으로 순서 로 재배열하고 싶다. 사서는 제 위치에 있지 않은 책 중 한 권을 그 책의 자리에 있는 책과 바꿔꽂는 것만 할 수 있다. 이렇게 재배열을 완료할 때까지 필요한 교환 횟수는 사서의 작업 순서에 상관없이 일정함을 증명하여라. (러시아 1989 4차-y8-7)

증명 무조건 20 - (cycle의 개수) 번 작업하게 됨(고정점도 길이 1짜리 cycle로 간주). ¤

79. 정육각형 ABCDEF의 각 꼭지점을 중심 O와 선분으로 이었다. 이 7개의 점 중 A, C, E는 파란색으로,

나머지 점들은 모두 흰색으로 색칠되어있다. 우리는 이 7개의 점 중 하나를 택해 그 점과 그 점에 이웃한점들의 색을 모두 바꾸는 조작을 할 수 있다(파란색은 흰색으로, 흰색은 파란색으로). 이런 조작을 유한번시행하여

(a) 점 B, D, F만 파란색이고 나머지는 흰색이 되도록 할 수 있음을 보여라.

(b) 점 B, D만 파란색이고 나머지는 흰색이 되도록 할 수 없음을 보여라. (러시아 1989 4차-y9-5)

증명 (a) A, B, C, D, E, F를 한 번씩 조작하면 됨. (b) 항상 짝수개의 칸이 동시에 변하므로 파란색

점의 개수의 홀짝이 불변. ¤

80. 1에서 n2까지의 자연수를 n £ n 체스판의 각 칸에 임의로 적었다. 두 수의 차가 n + 1보다 작지 않은 이웃한(꼭지점 또는 변을 공유하는) 두 칸이 존재함을 증명하여라. (Towns 1990가을 SO1)

증명 귀류법. 1에서 n2까지 건너가는 거리가 항상 n¡ 1번 이하이면 충분한데, 그럼 차이가 (n¡ 1)n이하라야 해서 모순. ¤

81. 똑같이 생긴 25개의 동전 중에 3개는 가짜이다. 모든 진짜동전의 무게는 서로 같고, 모든 가짜 동전의 무게도 서로 같지만 가짜동전은 진짜동전보다 가볍다. 천칭을 두 번만 사용해서 6개의 진짜동전을 가려내고싶다. 어떻게 하면 될까? (러시아 1990 4차-y9-1)

풀이 12개씩 달면 무겁거나 평형인 쪽에 가짜동전은 1개 이하. 그 12개를 6개씩 나눠 달면 무겁거나

평형인 쪽에 가짜동전이 0개. }

Page 216: 실전수학올림피아드 1400제 해답

216 조합

82. 가방에 1993개의 빨간 공과 1993개의 까만 공이 들어있다. 우리는 반복적으로 매 시간마다 공을 2개씩 꺼내는데,

(1) 2개의 공이 같은 색이면, 모두 버린다.

(2) 2개의 공이 다른 색이면, 까만 공은 버리고, 빨간 공은 가방에 다시 집어넣는다.

이 과정을 모두 마쳤을 때, 가방 안에 빨간 공 하나만 남게 될 확률은 얼마인가? (IMTS R10-4)

풀이 빨간 공은 항상 홀수개. 확률은 1 }

83. f1; 2; : : : ; 10g을 두 집합 fa1; a2; : : : ; a5g와 fb1; b2; : : : ; b5g로 분할하였다. a1 < a2 < a3 < a4 < a5 이고b1 > b2 > b3 > b4 > b5 일 때,

(1) 각각의 쌍 faj ; bjg (1 · j · 5) 에서 둘 중에 큰 것은 항상 6 이상임을 보여라.

(2)5X

i=1

jai ¡ bij = 25 가 항상 성립함을 보여라. (인도지역예선 2002-4)

증명 어디서 본듯... 통신강좌에서였나... (1)을 보이면 (2)는 바로 딸려나옴. ¤

84. 원 위에 몇 개의 점을 잡아 다음과 같이 번호를 매긴다. 먼저, 한 지름의 양끝점을 잡아 둘 다 1로 번호를매긴다. 그 다음부터는 그 전 단계에서 생긴 호의 중점을 찾아 양끝 두 점의 번호를 더한 값으로 번호를매긴다. 이 과정을 n번 시행했을 때 원 위의 점의 번호들의 합은 얼마인가? 예를 들어, 4번의 시행 뒤에는(1,4,3,5,2,5,3,4,1,4,3,5,2,5,3,4)와 같이 번호가 매겨진다. (소련 1963-13)

풀이 n단계의 합을 an으로 하면 우선 a0 = 2 이고, 전 단계의 각 수는 두 곳에 더해지므로 an+1 =

an + 2an = 3an. 따라서, an = 2 ¢ 3n. }

85. n이 5보다 큰 양의 정수라 하자. 집합 fn+ 1; n+ 2; : : : ; n+30g에서 소수는 많아야 8개임을 증명하여라.(IMTS R11-2)

증명 mod 30으로 1, 7, 11, 13, 17, 19, 23, 29인 수들만 소수일 가능성이 있고 나머지는 모두 합성수.¤

86. 3£ 3£ 3 루빅 큐브의 표면에 폐곡선을 그리는데, 꼭지점은 지나지 않게 하면서 각각의 작은 사각형을 딱한 번씩 지나게 그릴 수 있는가? (Towns 1989봄 JO4)

풀이 쉽게 할 수 있다. 각 면을 차근차근 지나가기만 해도. }

87. 99£ 99 판에 두 명이 번갈아가면서 비어있는 칸을 골라 체커를 놓는다. 단, 이웃한 칸이 모두 비어있거나이웃한 칸들 중 딱 한 칸에만 상 편의 체커가 있을 때만 그 칸에 체커를 놓을 수 있다. 이웃한 칸이란 변을 공유하는 상하좌우의 네 칸을 말한다. 더 이상 체커를 놓을 수 없는 사람이 진다고 할 때, 먼저 하는 사람과 나중에 하는 사람 둘 중에 어느 쪽이 필승법을 갖게 되는가? (러시아 1992 4차-y9-2)

풀이 천원에 놓은 후 칭플레이 }

88. 공간 상에 어느 네 점도 한 평면 위에 있지 않은 다섯 점이 주어져 있다. 이 중 두 점을 잇는 각 선분을 흰색, 초록, 또는 빨강으로 칠하는데, 모든 색이 다 사용되어야 하고 같은 색의 세 선분이 삼각형을 구성하는 경우는 없어야 한다. 이 다섯 점 중에 세 가지 색이 모두 만나는 점이 있음을 증명하여라.

(불가리아 1981 4차-1)

증명 귀류법. 한 점에서 같은 색 세 변이 있을 수 없음을 보이고, 모든 점에서 2:2이면 전체에 두 가지

색만 사용됨을 확인. ¤

89. 1997개의 연속된 양의 짝수들을 생각하자. 이들 중 41개를 택해 만든 부분집합에는 차가 100보다 작은 두수가 언제나 존재함을 증명하여라. (몰도바 1997 최종-y7-5)

Page 217: 실전수학올림피아드 1400제 해답

4.1 조합 중급문제 217

증명 40개의 집합 fa; :::; a+ 98g, fa+ 100; :::; a+ 198g, ..., fa+ 3900; :::; a+ 3992g 을 두고 비둘기

집. 또는 41개의 수를 b1 < b2 < ¢ ¢ ¢ < b41 이라 할 때, bn+1 ¸ bn + 100 임을 이용. ¤

90. 한 직선이 8£ 8 체스판을 가로지르고 있다. 직선이 어떤 칸의 내부의 점을 지날 때, 그 직선이 그 칸을 가른다고 말한다. 이 직선은 64개의 칸 중 최 몇 개의 칸을 가를 수 있는가? (헝가리 1930-2)

풀이 WLOG, 기울기는 0 이상이라 할 수 있고, 위로 혹은 오른쪽으로 인접한 칸을 하나씩 추가하는

방법으로 생각하면 최 1 + 7 + 7 = 15개. }

91. 11명의 수학자들과 그 남편들이 함께 회의에 참석했다. 때때로 남편들은 회의장에서 서로를 지나치는데,

한 번 서로 지나친 짝은 다시는 서로 지나치지 않는다. 서로 지나칠 때에는, 한 사람만 다른 사람을 알아보거나, 서로 알아보거나, 아니면 서로 아예 못 알아본다. 한 사람이 알아보는 경우를 `목격'이라 하고, 서로 알아보는 경우에는 `잡담'이라고 하자. 잡담일 경우에는 서로 잠깐 서서 얘기를 나눈다. 잡담은 두 번의 목격으로도 세어진다. 61번의 목격이 있었다면, 남편 중 한 명은 적어도 2번의 잡담을 나누었다는 것을 증명하여라. (IMTS R20-3)

증명 목격이면 A ! B를, 잡담이면 A! B 와 B ! A 를 둘다 그리는 graph를 생각하자. 그럼 이런

directed edge가 61개. 잡담이 1번 이하인 남자는 나가는 변의 개수와 들어오는 변의 개수의 합이 11개이하. 그럼 변은 모두 11¢11

2= 60:5개 이하. 모순. ¤

92. 홀에는 100명의 사람들이 있고, 이들 각각은 적어도 66명의 다른 사람들을 알고 있다. 서로 아는 네 명이반드시 존재한다고 확신할 수 없음을 증명하여라. [A가 B를 알면 B도 A를 안다.] (폴란드 1967 3차-2)

증명 33, 33, 34명의 세 그룹으로 분할한 후, 서로 다른 그룹끼리만 항상 아는 것으로 하면 됨. ¤

93. 홀에는 100명의 사람들이 있고, 이들 각각은 적어도 67명의 다른 사람들을 알고 있다. 서로를 모두 알고있는 4명이 존재함을 보여라. (폴란드 1967 3차-4)

증명 각자가 모르는 사람은 많아야 32명. a가 아는 사람이 적어도 67명, 이 집합을 B. B의 한 사람

b는 B 안에서 많아야 32명을 모르므로 B 안에서 적어도 34명을 안다. 이 34명의 집합을 C. C의 한 사람c는 C 안에서 적어도 한 명 D를 아니까, ABCD가 그 네 사람. ¤

94. 현재 미국에서는 1억 2천만 의 전화가 사용되고 있다고 가정하자. 모든 전화기에 10자리의 전화번호(각자릿수는 0부터 9까지 사용)를 배정하는데, 한 자리만을 실수로 틀리게 걸었을 때는 항상 이것을 알아내고 수정할 수 있도록 할 수 있는가? (예를 들어, 812-877-2917 번호의 전화기로 전화를 거는데, 812-872-2917이라고 잘못 걸었을 때, 812-872-2917과 한 자리만 다른 번호는 812-877-2917 이외에 존재하지 않도록 배정해야한다.) (IMTS R22-3)

증명 ABC-DEF-GHIJ가 사용하는 번호일 때 xBC-DEF-GHIJ, AxC-DEF-GHIJ, ... 등은 모두 사용

할 수 없고 또 ABC-DEF-GHIJ에만 소속되어야 한다. 즉, 사용번호 하나당 사용될 수 없는 번호 90개가함께 배정되고, 그럼 필요한 번호는 1억2천만£ 91개인데 이것은 10자리 번호의 개수 1010을 넘으므로모순. ¤

95. 1999£ 1999 표에서 몇 개의 칸을 칠했는데, 모든 행과 열마다 색칠된 칸이 딱 하나 있다. 1000£ 1000 크기의 부분영역에는 색칠된 칸이 항상 적어도 하나 포함됨을 증명하여라. (몰도바 1999 최종-y8/9-6)

증명 그 영역에 색칠된 칸이 없다면 그 영역과 같은 행의 나머지칸들을 모은 1000£ 999 의 각 행마다

색칠된 칸이 하나씩 있어야 하는데 그 1000개의 칸이 999개의 열에 있을 수 없... ¤

96. 한 반에 16명의 학생이 있다. 매월 선생님은 학생들을 두 그룹으로 나눈다. 선생님이 항상 같은 그룹에 있었던 두 학생이 없도록 하고 싶다면 최소 몇 달이 지나야 하는가? (러시아 1994 4차-y9-8)

풀이 1달 후 어느 8명은 같은 그룹. 2달째 후 그 8명 중 어느 4명은 아직 여전히 계속 같은 그룹. 3달

째 후 그 4명 중 어느 2명은 아직 여전히 계속 같은 그룹. 따라서, 적어도 4달은 필요. 실제 4달이면 충분함: 각 학생의 번호를 2진법으로 쓴 후 k달째에 2k¡1의 자리가 같은 학생들끼리 한 그룹이 되도록 하면 됨. ¢ ¢ ¢ 답 4달 }

Page 218: 실전수학올림피아드 1400제 해답

218 조합

97. A는 집합 f1; 2; : : : ; 100g에서 50개의 수를 골라 만든 부분집합이다. A의 어떤 두 원소도 합이 100이 아니라면 A는 완전제곱수를 적어도 하나 포함함을 증명하여라. (인도지역예선 1996-7)

증명 36, 64의 짝과 100이 핵심. 비둘기집 약간 비틀기 ㅎ ¤

98. 탁자 위에 성냥개비 세 더미가 있다. 한 더미는 성냥 100개, 다른 한 더미는 성냥 200개, 또 다른 한 더미는 성냥 300개로 되어있다. 두 명이 번갈아가면서 한 더미를 골라서 그 더미를 두 개의 더미로 나누는 게임을 한다(나눈 더미는 비어있을 수 없다). 더 이상 나눌 수 있는 더미가 없는 사람이 진다. 먼저 하는 사람과 나중에 하는 사람 중 누가 필승법을 갖겠는가? (러시아 1994 최종-y9-3)

풀이 먼저 하는 사람이 300개 더미를 100개, 200개로 나눈 후 칭 플레이. }

99. 이웃한 두 자연수의 비가 항상 소수가 되도록, 서로 다른 1995개의 자연수를 한 원의 둘레 위에 적을 수있는가? (러시아 1995 4차-y9-2)

풀이 소인수 하나를 추가하거나 제외하며 원을 돈 후 자기 자신으로 돌아와야 하는데, 1995가 홀수라

서 곤란함. 홀짝성. 혹은 곱셈을 먼저 다 하고 나눗셈을 나중으로 몰아서 해도 되는데, 그럼 곱셈 횟수와나눗셈 횟수가 같아야 하니까. }

100. N £N개의 칸 배열에서 둘레의 4(N ¡ 1)개의 칸을 생각하자. 이 칸들에 4(N ¡ 1)개의 연속한 (꼭 양수일 필요는 없는) 정수들을 하나씩 써넣으려고 한다. 각선에 평행한 변을 갖는 직사각형에 네 꼭지점의위치에 있는 수를 합하면 그 값은 항상 일정해야 한다. 각선 자신도 양끝점에 있는 두 수의 합이 역시같은 값이어야 한다. 이것이 가능한지 다음의 경우에 해 답하여라.

(a) N = 3

(b) N = 4

(c) N = 5 (Towns 1984봄 JO3)

풀이 (a)와 (c)는 가능함. (a)는 1...8을, (c)는 0...15를 이용하면 됨. (b)는 불가능함. (n ¡ 5) + ¢ ¢ ¢ +(n+ 6) = 12n+ 6 은 4의 배수가 아니라서. }

101. n2명의 학생으로 구성된 어떤 학급이 있다. 이 학급에서는 매주마다 한번씩 모든 학생들이 참가하는 퀴즈회가 열린다. 담임선생님은 학생들을 n명씩 n개팀으로 편성하는데, 어느 한 주에 같은 팀에 있었던 어

떤 두 학생도 다른 주에 다시 같은 팀에 속하지 않는 방식으로 가능한한 오랜 기간이 되도록 편성하려고한다. 이 기간이 n+ 2주보다 짧음을 증명하여라. (아일랜드 1995-1)

증명n2 ¡ 1n¡ 1 = n+ 1 < n+ 2. ¤

102. 한 변의 길이가 n인 정사각형꼴의 종이를 n2개의 단위칸으로 분할하였다. 일부 이웃한 두 칸의 사이에는단위 길이의 벽들을 그려 미로를 만드는데, 임의의 한 칸에서 출발하여 벽을 가로지르지 않고 이 미로를따라 임의의 다른 칸으로 항상 갈 수 있도록 한다. 가능한 벽의 총 길이는 최 얼마인가?

(이탈리아 1987-7)

풀이 나무 문제네. connected graph가 되려면 벽을 가로지로는 변의 개수는 최소 몇 개냐 하는. 2n(n¡1)¡ (n2 ¡ 1) = (n¡ 1)2개가 최 . }

103. N은 n자리의 자연수이고, (a) 모든 자릿수가 다 다르며, (b) 연속한 세 자릿수의 합은 항상 5의 배수가 된다. n · 6 임을 증명하여라. 또한, 최고 자리의 숫자를 무엇으로 시작하더라도 항상 이런 6자리 수를 찾을 수 있음을 증명하여라. (인도지역예선 1996-4)

증명 a1 ´ a4 ´ a7 (mod 5) 이므로 a7까지 존재할 수 없음. 항상 찾을 수 있다는 것은 (0; 1; 4)짝과

(0; 2; 3)짝을 첫 자리가 0, 1, 2, 3, 4 중 무엇이냐에 따라 적당히 돌려쓰면 됨 ¤

104. (a) 0에서 9까지의 10개의 자연수를 한 원주 위에 배열하는데 인접한 두 자연수의 차가 항상 3, 4, 5 중하나가 되게 할 수 있는가? (예를 들어서 0에서 6까지의 수는 0, 3, 6, 2, 5, 1, 4 순서로 배열하면 가능하다.)

(b) 0에서 13까지의 14개의 수로 한다면 어떤가? (소련 1967-4)

Page 219: 실전수학올림피아드 1400제 해답

4.1 조합 중급문제 219

풀이 (a) A = f0; 1; 2g, B = f3; 4; 5; 6g, C = f7; 8; 9g 로 두자. 집합의 A의 수 옆에는 항상 B의 수.

따라서, AB꼴의 변이 6개. CB꼴의 변도 6개. 그러나 B는 변에 8번만 사용될 수 있으므로 모순. (b) 가능하다. 2 5 1 4 0 3 6 10 13 9 12 8 11 7 }

105. N £N 체스판에 N2개의 말(`마(knight)'로 생각한다)이 놓여있다. 서로 잡히는 위치에 있었던 말들이 항상 서로 이웃한(적어도 한 점에서 만나는) 위치의 칸에 있게 되도록 말을 재배열할 수 있는가?

(a) N = 3 일 때 풀어라.

(b) N = 8 일 때 풀어라. (Towns 1983가을 J5)

풀이 (a) 가능함. 123/456/789 를 167/852/349 로 재배열하면 됨. (b) 불가능함. 먼저 순서 로 번호

를 매겼을 때, C3과 서로 잡는 위치는 8곳(그 집합을 A라 하자)이므로 그 8곳이 C3의 이웃으로 옮겨져야 함(게다가 한 칸의 이웃을 최 8칸이므로 A를 제외한 녀석이 C3의 이웃으로 올 수 없다). 그 A 중E4를 생각하면, E4와 서로 잡는 위치에 있었던 8곳도 모두 E4의 이웃으로 옮겨져야 함. 그럼 C3와 E4의새로운 공통이웃이 (적어도 2개) 생기는데, 옮기기 전에 C3와 E4의 공통 잡는 위치는 없었으므로 모순.}

106. 단위정사각형칸으로 분할된 무한히 펼쳐진 종이가 있다. 두 칸 사이의 `거리'란 가로세로로 움직여 두 칸사이를 연결하는 경로의 최단 길이(칸의 중심에서 중심까지)를 말한다. 거리가 6인 칸들은 항상 서로 다른 색이 되도록 모든 칸을 색칠한다면, 필요한 색의 개수는 최소 몇 개인가? 그 개수로 색칠하는 예를 들고, 더 적은 개수로는 불가능함을 증명하여라. (Towns 1984봄 JO5)

증명 한 칸을 기준으로 상하좌우 3의 거리에 있는 4개의 칸을 생각하면, 이 4개의 칸은 서로 거리 6에

있는 칸들이므로 K4가 구성되어 모두 색이 다름. 즉 4색 이상이 필요함. 4색으로는 실제로 간단히 구성가능함. ¤

107. (1) 차가 2이거나 5인 두 자연수는 항상 서로 다른 집합에 속하도록, 자연수 전체를 세 부분집합으로 분할할 수 있음을 증명하여라.

(2) 차가 2, 3, 또는 5인 두 자연수는 항상 서로 다른 집합에 속하도록, 자연수 전체를 네 부분집합으로분할할 수 있음을 증명하여라. 한편, 세 부분집합으로 분할하는 것은 곤란함을 보여라.

(아일랜드 1998-8)

증명 (1) mod 3으로 분할 (2a) mod 4로 분할. (2b) 세 부분집합으로 곤란하다는 것은 fa; a + 2; a +3; a+ 5g 를 고려해보면 a+ 2와 a+ 3은 항상 같은 집합에 있어야 함을 말할 수 있어서... ¤

108. n ¸ 4 인 볼록n각형을 생각하자. 이 다각형의 꼭지점들로 이루어진 삼각형들로 이 다각형을 분할하였다.

원래 다각형의 변을 두 개 갖는 삼각형들은 검은색으로 칠하고, 하나만 갖는 삼각형들은 붉은 색으로 칠하며, 다각형의 변을 하나도 갖지 않는 삼각형들은 흰색으로 칠하였다.

하얀 삼각형보다 검은 삼각형이 두 개 더 많음을 증명하여라. (주니어발칸 2004-4)

증명 삼각형은 총 n¡ 2개. 검정, 빨강, 하양 삼각형의 개수를 각각 a, b, c라 하면 a+ b+ c = n¡ 2.각 삼각형이 갖는 n각형의 변의 개수를 모두 합하면 2a+ b = n. 변변 빼면 a¡ c = 2. ¤

109. 5명 이상의 사람이 원탁에 둘러앉아 있을 때, 모든 사람이 두 명의 새로운 사람과 이웃하여 앉도록 재배열할 수 있음을 보여라. (폴란드 1968 3차-2)

증명 일단 n이 홀수일 때는 1; 3; 5; : : : ; n; 2; 4; : : : ; n ¡ 1 로 끝남. n이 짝수일 때는 1; 3; 5; : : : ; n ¡1; 2; n; n¡ 2; : : : ; 4 로 끝남. ¤

별증 n명이라 하자. 2 · a · n¡ 2 인 n과 서로소인 수 a가 존재하면 번호를 a만큼씩 증가하며 배열

하면 끝. 이런 a가 존재하지 않을 때는 Á(n) = 2 일 때이고, 이것을 소인수분해와 부등식으로 적당히 풀면 5 이상에서는 n = 6뿐. n = 6 일 때는 1 2 3 4 5 6을 1 5 2 4 6 3 등으로 재배열하면 됨. ¤

110. 쥐가 한 변의 길이가 3인 정육면체 모양의 치즈를 갉아먹고 있다. 이 치즈는 단위정육면체 27개로 나누어져있고, 이 쥐는 다음의 규칙에 따라 치즈를 갉아 먹는다: 한 코너에 있는 단위조각부터 먹기 시작해서, 그조각을 다 먹은 후 방금 먹은 단위조각과 면으로 이웃한 다른 단위조각을 먹기 시작한다. 이런 과정을 통해서 모든 치즈를 다 먹는데, 중심에 있는 단위조각 치즈를 마지막으로 먹는 것이 가능한가?

(유고슬라비아 1981 고1-4)

Page 220: 실전수학올림피아드 1400제 해답

220 조합

풀이 이웃한 칸을 흑백이 번갈아 되도록 색칠하면(코너가 흑), 흑칸이 하나 더 많은데 흑칸에서 시작

하여 백칸으로 끝날 수 있는지를 묻는 것이니. }

111. 10000개의 칸으로 이루어진 100£ 100 크기의 마룻바닥을 1£ 3 크기의 타일로 깔려고 한다.

(1) 이 마룻바닥의 중앙에 2£ 2 크기의 타일을 하나 깔면 나머지 바닥을 1£ 3 타일들로 잘 깔 수 있음을 증명하여라.

(2) 마룻바닥의 한 코너에 2£ 2 크기의 타일을 하나 깔면 나머지 바닥을 잘 깔 수 없음을 증명하여라.(아일랜드 1999-4)

증명 (1) 중앙 타일을 제외한 나머지 영역을 바람개비 형태로 4개의 합동인 직사각형 영역으로 나누

어보면 끝. (2) ABCABC... 색칠 이용 ¤

112. n개의 변을 갖는 다면체가 존재하는 자연수 n을 모두 구하여라. (폴란드 1969 3차-6)

풀이 (1) k각뿔은 2k개의 변을 가지므로 6이상의 짝수는 모두 가능함.한 삼각형면에 사면체를 새로 쌓

으면 3개의 변이 늘어나므로, 9이상의 홀수도 모두 가능함. (2)오일러의 공식 e = v+f¡2 ¸ 4+4¡2 = 6이므로 1, 2, 3, 4, 5는 불가능. 이제 e = 7 일 때만 확인하면 되는데, v 또는 f가 4가 되어 사면체밖에 안되고 그럼 또 수가 맞지 않으므로 불가능. }

113. 집합 f1; 2; : : : ; Ng의 공집합이 아닌 모든 부분집합을 생각하자. 각 부분집합에서 원소들의 역수의 곱을구해, 이것을 모두 합하면 얼마가 되는가? (Towns 1986가을 J4)

풀이 (1 + 12)(1 + 1

3) ¢ ¢ ¢ (1 + 1

N) 으로 인수분해에다 telescoping... }

114. 돌 세 더미가 있다. 그 중 한 더미에서 다른 두 더미의 돌의 수의 합만큼 새로 돌을 제거하거나 추가할수 있다. 예를 들어, [12,3,5]에서 12 + 5 = 17 을 두 번째 더미에 추가하여 [12,20,5]로 만들 수도 있고,3 + 5 = 8 을 첫 번째 더미에서 제거하여 [4,3,5]로 만들 수도 있다. [1993,199,19]로부터 시작하여 이런 시행만으로 빈 더미가 하나 생기도록 할 수 있는가? (Towns 1993봄 SO4)

풀이 계속 셋 다 홀수라서 불가능. }

115. 한 항공사가 한 나라의 64개의 마을을 연결하는 2000개의 양방향 항로를 운항한다. 어떤 두 도시 사이에도 이 항공사의 항로를 통하여 오갈 수 있음을 증명하여라. (몰도바 2000 최종-y8-5)

증명 가능한 모든 변의 개수는 64¢632

= 2016 이므로 K64에서 오직 16개의 변만 빠졌다. 그런데 64점

의 그래프에 cut이 존재하려면(두 도시 A와 B가 연결되지 않았다면, 도시 A에 연결된 모든 도시의 집합과, 나머지 도시들의 집합 간에 변이 없어야 하므로) min k(64 ¡ k) = 63 개 이상의 변이 제거되어야한다. ¤

116. 서로 다른 무게의 68개의 동전이 있다. 천칭을 100번만 써서 가장 무거운 동전과 가장 가벼운 동전을 찾아내는 방법을 구하여라. (Towns 1985봄 JO2)

풀이 34쌍으로 분할하여 각 쌍을 잰 후(34번) 무거울 가능성의 그룹(H)과 가벼울 가능성의 그룹(L)으

로 34:34로 나눈다. H에 있는 34개들로 승자승 토너먼트를 벌여 가장 무거운 것을 찾고(33번), L에서도마찬가지로 가장 가벼운 것을 찾으면(33번) 됨. }

117. 한 변의 길이가 l인 정사각형꼴로 생긴 마을이 있는데, 3£ 3 형태의 9블록으로 구성되어 있다. 각 블록은아스팔트 도로로 경계지어져 있다. 이 마을의 한쪽 구석에서 출발하여 아스팔트 도로를 따라 마을을 돌다처음의 위치로 돌아오려고 한다. 모든 아스팔트 도로 구간을 각각 한 번 이상 지나가고자 한다면 움직여야 하는 최단거리는 얼마가 되는가? (Towns 1984가을 JO2)

풀이 이 순환로에서 코너의 각 점은 1번 들르면 되고, 다른 12점은 적어도 두 번씩은 들러야 하므로 방

문횟수는 1 ¢ 4 + 2 ¢ 12 = 28번 이상. 즉, 달리는 거리도 28 이상이고, 실제로 이만큼만 달리는 길을 구성할 수 있다. ¢ ¢ ¢ 답 28 }

Page 221: 실전수학올림피아드 1400제 해답

4.1 조합 중급문제 221

118. 먼 바다의 어떤 섬에서 사용되는 언어의 낱말은 문자 a, b, c, d, e, f , g로만 구성된다고 한다. 한 낱말을다음과 같은 변형을 통해 다른 낱말로 만들 수 있을 때, 그 두 낱말을 동의어라고 말하기로 하자.

(i) 다음과 같은 규칙으로 한 문자를 두 문자로 체시킬 수 있다:

a! bc; b! cd; c! de; d! ef; e! fg; f ! ga; g ! ab

(ii) 한 문자가 다른 똑같은 두 문자 사이에 끼어있으면 그 두 문자는 제거할 수 있다.예를 들면, dfd! f .

이 언어의 모든 낱말은 동의어임을 보여라. (중미 2007-4)

증명 a ! bc ! cdc ! d 로 체되므로, 같은 방법으로 a! d ! g ! c ! f ! b ! e ! a 로 모든

문자들은 서로 체될 수 있다. ¤

119. 15£ n 크기의 판을 다음 두 가지 모양의 타일로 잘 깔 수 있는 자연수 n을 모두 구하여라. (중미 2000-2)

풀이 5£ 3 atom이 있으므로 5 j n 또는 3 j n 이면 잘 깔 수 있음. n = 5A+3B꼴일 때도 잘 깔림. 이

런 꼴이 아닌 수들은 n = 1; 2; 4; 7뿐인데, 첫행부터 차근히 따져보면 이것들은 다 불가능. }

120. 8개의 축구팀이 풀리그를 벌인다. 비기는 경우는 없고, 각 두 팀은 꼭 한 번씩 경기를 한다. 회가 끝났을때, A가 B, C, D를 이기고, B가 C, D를 이기고, C가 D를 이긴 네 팀 A, B, C, D를 항상 찾을 수 있음을보여라. (Towns 1985가을 J1)

증명 4팀 이상을 이긴 팀 A가 있다. A에게 진 B, C, D, E를 생각하면 B, C, D, E들끼리의 경기에서

2팀 이상을 이긴 B가 있다. B에게 진 C, D를 생각하면 끝. ¤

121. 한 여행객이 어떤 나라를 여행하고 있다. 우선 수도 A에서 출발하여 비행기를 타고 거기서 가장 먼 도시인 B로 간다. 다음 B에서 역시 비행기를 타고 거기서 가장 먼 도시인 C로 간다(C는 A가 될 수 없다고 한다). 이런 식으로 계속 여행한다면 이 여행객은 다시 수도로 돌아오지는 않게 됨을 증명하여라.

(오클랜드 2001-7)

증명 A에서 B로 갈 때의 이동거리를 d1, B에서 C로 갈 때의 이동거리를 d2, ... 이렇게 다음 도시로

이동하는 거리를 차례로 d1; d2; d3; : : : 이라 하자. B에서 A보다 C가 더 먼 거리이므로 BA > BC, 즉d1 < d2 이다. C에서 다시 가장 먼 도시로 이동하므로 d3 ¸ CB = d2 이다. 이런 식으로 계속 가장 먼도시로 이동하므로

d1 < d2 · d3 · d4 · ¢ ¢ ¢가 된다. 따라서, 만일 다시 수도 A로 돌아오게 된다면 dk > d1 인데, 이것은 A에서 가장 먼 도시 B보다 더 먼 도시가 있다는 것이므로 모순이다. 그러므로, 이 여행객은 수도로는 되돌아오지 않게 된다. ¤

122. 6 £ 10 크기의 직사각형꼴 초콜렛을 가지고 두 명이서 게임을 한다. 이 초콜렛은 금을 따라서만 잘린다.

먼저 하는 사람이 한 직선을 따라 자르고, 자른 한 쪽을 먹어버린다. 두 번째 사람이 남은 조각을 역시 직선으로 자르고 한 쪽을 먹어버린다. 이런 식으로 번갈아 계속하는데, 마지막 한 조각을 남기는 사람이 승리한다. 완벽한 게임이라면 둘 중에 누가 이길까? (Towns 1986가을 J3)

풀이 먼저 하는 사람이 계속 정사각형을 만들면 이김. 칭성의 게임. }

123. 산책은 동서남북 중 어느 한 방향일 수 있는 길이 1인 걸음들의 연속으로 이루어져 있다. 같은 지점을 두번 지나지 않는 산책을 `자기회피적'이라고 한다. 원점에서 출발하여 n걸음 동안 자기회피적으로 산책하는 방법의 수를 f(n)이라고 한다. f(1), f(2), f(3), f(4)를 구하고, 다음 부등식을 보여라.

(캐나다 1979-5)

2n < f(n) · 4 ¢ 3n¡1

Page 222: 실전수학올림피아드 1400제 해답

222 조합

증명 첫 번째 걸음은 4가지 방법이 있고, 두 번째 걸음부터는 방금 지나온 지점으로 다시 돌아갈 수 없

으므로 많아야 3가지 방법만 있다. 그러므로 f(n) · 4 ¢ 3n¡1 의 오른쪽 부등식이 확인된다. 그리고, 처음부터 북쪽과 동쪽만 선택해서 움직인다면 같은 지점을 다시 지나지 않는 2n가지의 산택을 얻을 수 있다. 이외에 서쪽으로 계속 가는 산책도 있으므로 f(n) > 2n 의 왼쪽 부등식도 확인된다. ¤

124. 서로 다른 12n(n+ 1) 개의 수들이 아래와 같은 삼각형꼴로 임의로 배열되었다:

££ £

£ £ £...

......

...

£ £ £ £¢ ¢ ¢

위로부터 k번째 행에서 가장 큰 수를 Mk라 하자.

M1 < M2 < M3 < ¢ ¢ ¢ < Mn

일 확률을 구하여라. (캐나다 1990-2)

풀이 구하는 확률을 Pn이라 하자. 마지막 행에 가장 큰 수가 들어갈 확률은 nn(n+1)2

= 2n+1 이고, 마

지막 행에 가장 큰 수를 넣었으면 그 다음은 M1 < M2 < ¢ ¢ ¢ < Mn¡1 만 만족하면 되므로,

Pn =2

n+ 1Pn¡1 =

2

n+ 1¢ 2nPn¡2 = ¢ ¢ ¢ = 2n¡1

(n+ 1)n ¢ ¢ ¢ 3P1 =2n

(n+ 1)!¢ ¢ ¢ 답

위와 같은 점화식 풀이가 가능하다. }

125. 아홉 개의 정수 1; 2; : : : ; 9중에서 균일한 확률로 임의의 하나를 고를 수 있는 선택기가 있다.수를 n(> 1)번고른 후, 이 n개의 수의 곱이 10으로 나누어떨어질 확률을 구하여라. (미국 1972-3)

풀이 포함배제. 1¡ ( 89 )n ¡ ( 59 )n + ( 49 )n }

126. A, B와 다른 2001명의 사람이 원형으로 둘러서있다. A와 B는 서로 이웃해있지 않다. A로부터 시작하여둘이 번갈아가며 옆사람 중 한 명을 터치한다. 터치당한 사람은 그 즉시 형에서 떠나야한다. 상 방을터치한 사람이 승자가 된다. 둘 중 한 명에게 필승의 전략이 있음을 보이고 그 전략을 구하여라.

(중미 2001-1)

풀이 A의 전략: A와 B 사이(좌우 각각)에 홀수명씩 있도록 만든다. 칭성 or 불변량. }

127. 8£ 8 체스판의 남서쪽 구석에 9개의 졸이 3£ 3꼴로 놓여있다. 가로, 세로, 각선의 방향으로 졸이 다른졸 하나를 뛰어넘으며 이동하는 것이 가능하다. 이 졸들을 (a) 북서쪽 구석으로 3£ 3꼴로 옮겨놓을 수 있는가? (b) 북동쪽 구석으로는 가능한가? (Towns 1987봄 JA3)

풀이 각 졸의 위치는 x-좌표와 y-좌표 모두 홀짝이 항상 처음 그 로 유지된다. 좌상귀든 우상귀든 개

수가 맞지 않아서 불가능. }

128. 똑같이 생긴 n개의 양초가 촛 에 끼워져있다. i = 1; 2; : : : ; n 에 해, i번째 날에는 i개의 양초를 딱 한시간 동안 불을 밝힌다. n번째 날이 지났을 때 n개의 양초가 모두 다 소모되었다고 한다. 가능한 n을 모두 구하여라. 그리고, 각각의 n에 해, i번째 날에 불을 밝히는 규칙을 제시하여라. (스웨덴 1961-3)

풀이 양초가 탄 총시간은 1+ 2+ ¢ ¢ ¢+ n =n(n+1)2 이므로, 한 초당 n+1

2 시간씩 탔다. 따라서 n은 홀

수. 실제로 n이 홀수일 때, 1 + k번째 날과 n¡ k번째 날을 짝지어 한 날처럼 불을 밝히면 됨. }

129. 임의로 주어진 자연수 n에 해,1

a+1

b+1

c=1

n을 만족하는 자연수해 (a; b; c)는 유한개뿐임을 보여라.

(스웨덴 1967-3 변형)

Page 223: 실전수학올림피아드 1400제 해답

4.1 조합 중급문제 223

증명 WLOG a · b · c 라 할 때, 1n =1a+ 1

b+ 1

c· 1

a+ 1

a+ 1

a= 3

a, 즉 a · 3n 이므로 우선 a는 유

한가지. 각각의 a에 해서 b도 유한가지씩. ¤

130. 2003개의 돌이 한 더미를 이루고 있다. 두 사람이 번갈아서 이 더미에 있는 돌의 개수의 어떤 약수만큼 돌을 가져간다. 마지막 돌을 가져가는 사람이 지는 것으로 할 때, 둘 중 한 사람의 필승의 전략을 구하여라.

(중미 2003-1)

풀이 을이 항상 홀수를 만들어서 넘기면 이긴다. }

131. 3 £ n 격자판이 다음과 같이 채워져있다. 첫 번째 행에는 1에서 n까지의 수가 오름차순으로 있고, 두 번째 행은 첫행을 적당한 i에 해 i번 회전시킨 i + 1; : : : ; n; 1; 2; : : : ; i ¡ 1; i 꼴로 되어있다. 세 번째 행에도 1부터 n까지의 수가 적당한 순서로 들어있는데, 각각의 열의 세 수를 합한 값은 모두 같다고 한다. 이런 식으로 수를 배치하는 것이 가능한 n은 어떤 값들인가? 그런 모든 n에 해, 가능한 서로 다른 배치방법의 수를 각각 구하여라. (아일랜드 2002-6)

풀이 첫열부터 둘째줄에 n이 쓰인 열까지의 처음 n¡ i개열을 앞부분, 그 이후의 i개열을 뒷부분이라

부르자. 앞부분의 제3행에 쓰인 수들끼리는 모두 서로 홀짝이 같다. 뒷부분의 제3행에 쓰인 수들끼리도모두 서로 홀짝이 같다. 따라서, 제3행에서 앞부분, 뒷부분이 홀수 전체와 짝수 전체를 나눠가져야 하며,

그럼 n이 짝수이면 i = n2, n이 홀수이면 i = n§1

2 일 수밖에 없다. 각각 조사해보면 n일 홀수일 때의

2가지만 가능하다. }

132. 남북으로 m개의 길과 동서로 n개의 길이 난 직사각형꼴의 마을이 있다. 어떤 교차로에서 출발하여 이마을의 길을 따라 다른 모든 교차로를 딱 한 번씩 지나 처음의 위치로 돌아올 수 있었다. 이것이 가능한m;n > 1 을 모두 구하여라. (스웨덴 1972-2)

풀이 m 또는 n이 짝수이면 쉽게 가능. (회로는 하나만 잡으면 그 회로가 어떤 점을 출발점으로 하든

다 커버함) m, n이 모두 홀수이면 체스판처럼 흑백을 번갈아 칠하면 항상 흑백을 번갈아 지나야 하는데,

회로라면 흑백의 칸수가 같아야 해서 모순. }

133. 한 정육면체의 여덟 꼭지점을 자신의 꼭지점으로 하는, 스스로와 만나지 않는 공간 팔각형이 있다. 이 공간 팔각형이 정육면체의 모서리를 적어도 하나 포함함을 증명하여라. (레닌그라드 1987-16)

증명 스스로와 만나지 않으므로 주 각선은 많아야 하나, 각 면의 면 각선도 많아야 하나씩만 있다.

그럼 선분은 최 7개뿐. ¤

134. 흑백이 번갈아 칠해진 8£ 8 체스판이 있다. 이 체스판의 두 열을 맞바꾸거나 두 행을 맞바꿀 수 있다. 유한번의 이런 조작들만으로, 체스판의 왼쪽 절반은 모두 검은칸, 오른쪽 절반은 모두 흰칸이 되도록 할 수있는가? (레닌그라드 1987-25)

풀이 각 줄의 검은 칸의 개수는 항상 4개로 불변. }

135. 두 사람 A와 B가 숫자 1, 2, 3, 4, 5를 사용하여 2005 자리의 수 N을 만드는 게임을 한다. A가 먼저 첫 번째 자릿수를 택하고, 다음 B가 두 번째 자릿수를 택하고, 그 다음은 A가, 그 다음은 B가, 이런 식으로 번갈아 한다. N이 9의 배수가 되면 A가 이기는 것으로 한다. 두 사람 모두 최선의 경기를 한다면 누가 이기겠는가? (아일랜드 2005-7)

풀이 B가 계속 둘의 두 턴의 합이 6이 되도록 하면 이김. 2004자리까지 딱 9의 배수가 되기 때문에,

한 자리 더 하면 9의 배수가 안 됨. }

136. 10 £ 10 크기의 판에서 빨강과 파랑, 두 사람이 번갈아 다음과 같은 게임을 한다. 파랑이 먼저 시작하고,

각자의 차례에서는 (아직 선택된 적이 없는) 한 행이나 열을 택해 그 줄의 모든 칸을 자신의 색으로 칠한다. 그 줄에서 이미 색칠된 칸들도 새로운 색으로 덧칠된다. 20번의 차례가 지나 모든 행과 열이 한 번씩선택되어 색칠된 후에는 게임이 끝난다. 빨간 칸의 개수가 파란 칸의 개수보다 10개 이상 많으면 빨강이이기는 것으로 하고, 그렇지 않으면 파랑이 이긴다. 두 사람 중 누구에게 필승의 전략이 있는지 결정하고,

그 전략을 서술하여라. (중미 2005-4)

Page 224: 실전수학올림피아드 1400제 해답

224 조합

풀이 한 칸은 딱 두 번씩 색칠됨. 한 각선 위의 칸을 빨강이 모두 먹는다는 전략(i번째 행or열을 상

방이 먹으면 i번째 열or행을 먹는다는 전략)으로 하면 빨강 승리. 우상삼각형과 좌하삼각형 영역은 서로 칭적으로 반전된 형태가 됨. }

137. 3£ 3 표에서 2£ 2 부분영역을 택해 그 영역에 속하는 네 수를 1씩 증가시킬 수 있다. 처음에 9개의 수가모두 0이었다면, 이런 조작을 유한번 하여 다음을 만들 수 있는가? (레닌그라드 1988-1)

4 9 5

10 18 12

6 13 7

풀이 중앙은 네 코너 칸의 합과 같아야함. 불변량. }

별해 체스판처럼 흑백을 번갈아 칠하면 흑칸의 합과 백칸의 합이 항상 같아야 함. }

138. 8 £ 8 크기의 판에 각 행과 각 열에 정확히 4개씩의 검은칸이 있도록 흰색과 검은색으로 모든 칸을 색칠하였다. 서로 붙어있는 흰칸 쌍의 개수와 서로 붙어있는 검은칸 쌍의 개수가 같음을 보여라(단, 서로 붙어있다는 것은 한 변을 공유한다는 것을 뜻한다). (이탈리아 1991-4)

증명 흰흰 쌍 x개,검검 쌍 y개,흰검 쌍 z개라 하자.그럼 흰칸과 변이 만나는 횟수는 2x+y = 4¢32¡4¢4이고 검은칸과 변이 만나는 횟수는 y + 2z = 4 ¢ 32¡ 4 ¢ 4 로 같다. 즉, x = z. (서로 다른 두 가지 관점에서 세기) ¤

139. A와 B는 jAj + jBj > n ¡ 1 을 만족하는 f1; 2; : : : ; n ¡ 1g의 두 부분집합이다. a + b = n 을 만족하는a 2 A 와 b 2 B 를 찾을 수 있음을 증명하여라. (헝가리 1953-1)

증명 귀류법: 한 집합에 수 하나 넣으면 반 쪽 집합에서는 다른 수 하나가 못들어감(혹은 k, n¡ k는

A, B 통틀어 2개밖에 못들어감). 그럼 jAj+ jBj · n¡ 1. ¤

140. 단위 정육면체의 각 면에 다른 단위 정육면체를 하나씩 붙여 만든, 7개의 정육면체들로 만들어진 블록을`크로스'라고 부르자. 무한히 많은 크로스블록으로 공간 전체를 빈틈없이 가득 채울 수 있음을 보여라.

(체코슬로바키아 1986-3)

증명 그냥 착착 맞물려 쌓으면 다음의 타일을 차곡차곡 쌓은 것으로 볼 수 있음.OO

OOO

OO

이 타일은 평면을 규칙적으로 잘 깔 수 있으니까 끝 ¤

141. 어떤 정수 N이 존재하여, n ¸ N 인 모든 정수 n에 해 한 정사각형을 더 작은 정사각형 n개로 항상 쪼갤 수 있음을 보여라.

증명 (이탈리아 1994-1) n이 되면 n+ 3도 되므로, n = 6; 7; 8 일 때 하면 끝. ¤

142. fa; b; cg로 구성된 길이 n의 낱말 중에서 a를 짝수개 포함하는 것의 개수를 구하여라. (이탈리아 1996-4)

풀이 xn+1 = 2xn + (3n ¡ xn) = xn + 3n, x1 = 2 의 점화식. }

143. 공간에 있는 n개의 점을 모두 빨간색 혹은 파란색 선분으로 연결하였다고 할 때, 자신으로부터 그어져 나간 빨간색 선분의 개수가 같은 두 점이 존재함을 보여라. 단, n ¸ 2 이고 어떤 세 점도 같은 직선 위에 있지 않다고 한다. (한국 1992-4)

증명 유명한 악수 문제 그 자체네. 악수한 횟수가 다 다르다면 0부터 n¡ 1까지 다 한 명씩 가져야 하

는데, 0인 녀석과 n¡ 1 녀석이 서로 모순. ¤

144. n개의 서로 다른 양의 실수의 집합 fa1; a2; : : : ; ang이 있다. 공집합이 아닌 모든 가능한 부분집합에 해그 원소의 합을 계산한다. 적어도 n(n+ 1)=2개의 서로 다른 값(합)이 있음을 증명하여라. (소련 1963-8)

Page 225: 실전수학올림피아드 1400제 해답

4.2 조합 고급문제 225

증명 a1 < a2 < ¢ ¢ ¢ < an 라 할 때,

a1 < a2 < ¢ ¢ ¢ < an (n sums),a1 + an < a2 + an < ¢ ¢ ¢ < an¡1 + an (n¡ 1 sums),a1 + an¡1 + an < a2 + an¡1 + an < ¢ ¢ ¢ < an¡2 + an¡1 + an (n¡ 2 sums),: : : ;a1 + a2 + ¢ ¢ ¢+ an (1 sum).

¤

145. 1부터 100까지의 자연수들이 한 원의 둘레에 나열되어 있다. 이웃한 두 수의 합들 중에서 차이가 2보다큰 것이 반드시 존재하는지 증명하거나 반증하여라. (몰도바 1999 최종-y10-3)

증명 반례가 있음. 100 2 98 4 96 ... 46 54 48 52 50; 51 49 53 47 ... 97 3 99 1 로 하면 이웃한 두 수

의 합은 항상 100, 101, 102 중 하나. ¤

146. 반지름 r인 7개의 원판이 반지름이 1인 원판을 덮고 있다. r ¸ 12 임을 증명하여라. (Towns 1987봄 SA2)

증명 정육각형의 여섯 꼭지점과 중심까지 7개의 점을 생각하면, 반지름이 12보다 작은 원은 두 점을 동

시에 덮을 수 없음. ¤

147. 정육면체의 각 꼭지점에 1 또는 ¡1이 부여되어 있다. 또한 각 면에는, 그 면의 네 꼭지점에 부여되어 있는 수들의 곱이 부여된다. 이렇게 부여된 14개의 수 전부의 가능한 합을 모두 구하여라. (남미 1991-1)

풀이 꼭지점의 부호 하나를 바꾸면 네 수의 부호가 바뀌므로 mod 4로 일정함. 모두 1일 때 최 의 경

우로 14이고, 14, 10, 6, 2, ¡2, ¡6, ¡10은 모두 가능함. ¡14는 모두 ¡1이어야 하는데 이것은 불가능. }

148. a1; a2; a3; a4; a5; a6; a7과 b1; b2; b3; b4; b5; b6; b7은 각각 1, 2, 3, 4, 5, 6, 7을 재배열한 것이다. ja1 ¡ b1j,ja2 ¡ b2j, ja3 ¡ b3j, ja4 ¡ b4j, ja5 ¡ b5j, ja6 ¡ b6j, ja7 ¡ b7j 들이 모두 서로 다를 수는 없음을 보여라.

(스웨덴 1980-2)

증명 ja1 ¡ b1j+ ¢ ¢ ¢+ ja7 ¡ b7j ´ (a1 + ¢ ¢ ¢+ a7) + (b1 + ¢ ¢ ¢+ b7) ´ 0 (mod 2). 그런데 이들이 모

두 다르다면 0부터 6까지 딱 한 번씩 나와야 하므로 0 + 1 + ¢ ¢ ¢+ 6 = 21 은 홀수가 되어 모순. ¤

149. 어떤 농구 회에서 모든 두 참가팀 사이에 두 번씩의 경기가 있다. 경기에서 이기면 2점을 얻고, 지면 0점을 얻으며, 비기는 일은 없다. 우승팀이 26점을 얻었고, 꼴찌팀은 딱 두 팀이 공동으로 20점을 얻었다고할 때, 참가팀의 수를 구하여라. (이탈리아 2001-2)

풀이 나머지 팀의 점수는 22 또는 24. 26 + 20 ¢ 2 + 22(n¡ 3) · 2n(n¡ 1) · 26 + 20 ¢ 2 + 24(n¡ 3).정리하면 12n · n2 · 13n¡ 3. ¢ ¢ ¢ 답 12 }

150. kn명의 사람이 있다(k, n은 자연수). 각각의 사람은 자기 이외에 (k ¡ 1)n명 넘게 안다. 서로 모두 아는k + 1명이 존재함을 증명하여라. (단, A가 B를 알면 B도 A를 안다.) (폴란드 1995/1996 1차-3)

증명 각자는 모르는 사람이 많아야 n ¡ 2명. A1이 모르는 n ¡ 2명을 제외시키고 남은 집합을 X1.

X1에서 A2를 골라, A2가 모르는 n ¡ 2명을 제외시키고 남은 집합을 X2. 이런 식으로 계속하여 서로 아는 A1; A2; : : : ; Ak를 찾을 수 있고, 그때까지 k(n ¡ 2)명이 제외되었고 k명이 선택되었으므로k(n¡ 2) + k < kn 이라서 Xk에 남은 인원이 있으므로 그중 아무나 Ak+1로 하면 된다. ¤

4.2 조합 고급문제

1. (a) 4£ 7 체스판의 각각의 칸을 검은색이나 흰색으로 칠하자.

■ ■ ■ ■■ ■ ■

■ ■ ■ ■■ ■ ■

어떻게 칠해도 네 코너의 칸이 모두 같은 색인 직사각형(이 체스판의 가로선과 세로선으로 이루어진)이 항상 생기게 됨을 증명하여라.

(b) 4£ 6 체스판에서는 그런 직사각형이 하나도 없도록 칠할 수 있음을 보여라. (미국 1976-1)

Page 226: 실전수학올림피아드 1400제 해답

226 조합

증명 (a) 귀류법으로 풀자. WLOG 첫행에 검은칸이 넷 이상이라 하자. 그 검은칸을 갖는 네 열만 생

각할 때, 제2행, 제3행 각각에 흰칸이 셋 이상 있다(검은칸이 둘 이상 있으면 검은 직사각형 생김). 그럼이 제2행의 세 흰칸과 제3행의 세 흰칸 중에 서로 겹치는(같은 열에 있는) 짝이 둘 이상 있다. 그게 흰 직사각형을 이루므로 끝(사실상 7£ 3 만으로도 충분함).

■ ■ ■■ ■ ■

■ ■ ■■ ■ ■

(b) 위와 같은 예가 있다. ¤

2. A가 임의의 n자리 자연수이면 적당한 연속된 몇 자릿수의 곱이 완전제곱수가 되게 할 수 있다고 한다(예:7323658에서라면 2 ¢ 3 ¢ 6 = 62). 최소의 n을 구하여라. (1994 겨울학교 모의고사)

풀이 최소의 n은 16이다. 우선 15일 때의 반례는 232523272325232 등이 있다. 이 수의 경우 만일 7을

포함하면 72의 배수는 아니므로 곤란하고, 7을 포함하지 않으면 양쪽 중 어느 한 쪽 2325232의 부분이므로 비슷한 논리로 5를 포함하면 안 되고, 그럼 232의 부분이므로 3을 포함하면 안 되고, 그럼 2라서 결

국 모두 안 된다. n ¸ 16 일 때는, pk =kY

i=1

ai = 2xk3yk5zk7wk 로 정의할 때, (xk; yk; zk; wk)는 mod

2로 16종류이므로 (0; 0; 0; 0)인 것이 있거나 아니면 같은 종류인 둘이 있다. 전자이면 그 자체의 pk가 완

전제곱수이고, 후자이면 (xk; yk; zk; wk) ´ (xj ; yj ; zj ; wj) (mod 2) 라 할 때(k < j)pj

pk=

jYi=k+1

ai 가

완전제곱수이다. }

3. A와 B가 자연수가 하나씩 적힌 카드 2n장을 가지고 게임을 한다. 이 카드를 섞어 숫자가 위로 보이도록한 행으로 늘어놓는다. A가 먼저 시작하여 두 사람이 번갈아 차례를 갖는데, 자기 차례에서는 행의 양끝중 어느 하나의 카드를 빼내어 갖는다. B가 마지막 카드를 집으면 게임이 끝나고, 두 사람은 자기가 가진카드의 숫자를 합해 점수를 계산한다. A가 최선을 다하면 A의 점수가 언제나 B의 점수 이상이 됨을 증명하여라. (영국 2003/2004 1차-3)

증명 ( 원외국어고 3학년 최일규) 줄에 카드들을 왼쪽부터 1, 2, 1, 2, : : : 이런 식으로 번호를 매긴

다. 1끼리 더하고 2끼리 더하여 큰 것의 카드들을 잡는 방법이 있으면 언제나 이긴다. 1들의 합이 2들의합보다 크다고 가정하자.1, 2, 1, 2, : : : ; 1, 2 의 상태에서 1을 취한다. 그러면 2, 1, 2, : : : ; 1, 2 의 상태가 되는데 상 방은 2을 취할 수 밖에 없다. 그러면 1, 2, : : : ; 1, 2 혹은 그 거꾸로의 상태가 되는데 나는 다시 1을 취한다. 이런 식으로 하면 나는 1들만 취하게 되고, 상 는 2들만 취하게 된다. 1들의 합이 2들의 합보다 크므로 내가 이겼다. ¤

4. n ¸ 2 명의 선수가 참가하고 k일 동안 계속되는 시합이 열렸다. 매일 선수들은 1; 2; 3; : : : ; n 중 하나의점수를 받는데 어느 두 선수도 같은 점수를 받지 않는다. k일이 모두 지나서 보니 모든 선수가 똑같이 총26점씩을 받았다. 이것이 가능한 모든 순서쌍 (n; k)를 구하여라. (캐나다 1990-1)

풀이 매일n(n+1)2 의 점수가 나간다. k일이 지나서 n명의 모든 선수가 26점씩 받았다고 하였으므로

n(n+ 1)

2k = 26n

즉, k(n+ 1) = 52 이다. k와 n+ 1은 양의 정수이고, 문제의 조건에서 n+ 1 ¸ 3, 또 시합이 하루뿐이었다면 모두 같은 점수를 받을 수 없으므로, 가능한 약수의 쌍은 (k; n+ 1) = (2; 26); (4; 13); (13; 4)뿐이다. 즉, (n; k) = (25; 2); (12; 4); (3; 13) 이고,

(n; k) = (25; 2)

1일째 : 1 2 ¢ ¢ ¢ 24 25

2일째 : 25 24 ¢ ¢ ¢ 2 1

(n; k) = (12; 4)

1; 2일째 : 1 2 ¢ ¢ ¢ 11 12

3; 4일째 : 12 11 ¢ ¢ ¢ 2 1

(n; k) = (3; 13)

1일째 : 1 2 3

2일째 : 2 3 1

3일째 : 3 1 2

4일째 : 1 2 3

5일째 : 3 2 1

.

..

Page 227: 실전수학올림피아드 1400제 해답

4.2 조합 고급문제 227

이들 각각은 실제로 조건을 만족하는 점수 배정이 위와 같이 가능하다.¢ ¢ ¢ 답 (25; 2); (12; 4); (3; 13) }

5. 6명의 학생이 있고 각각은 서로 알거나 서로 모른다(A가 B를 아는데 B는 A를 모르는 경우는 없다). 그럼서로서로 다 알거나 서로서로 다 모르는 3명이 항상 있게 된다. 이 사실이 맞다는 가정 하에서, 6명 중에서로 다 알거나 서로 다 모르는 3명을 찾을 수 있는 방법이 사실은 항상 적어도 2가지씩 있음을 보여라.

그리고, 3가지를 찾는 것은 불가능할 수 있음을 보여라. (셈본중등고급 도전문제 1.1.2)

증명 (1) 앞의 문제에 의해, 일반성을 잃지 않고, A, B, C, D, E, F의 6명 중에 세 명 A, B, C가 서

로 안다고 하자. 귀류법으로, 서로 다 알거나 서로 다 모르는 3명의 관계가 더 존재하지 않는다고 해보자. D, E, F가 서로 다 알지는 못하므로 그 중에, 일반성을 잃지 않고, 두 명 D, E는 서로 모른다. 만일A가 D와 E를 모두 모르면 서로 모르는 3명이 생기므로, A는 D와 E 중 적어도 한 명을 안다. 마찬가지로 B와 C도 각각 D와 E 중 적어도 한 명을 안다. A, B, C의 3명이 각각 D, E 둘 중 적어도 한 명을 알고있으므로 비둘기집의 원리에 의해 D와 E 둘 중 어느 한 명은 A, B, C 중 적어도 2명을 안다. 그럼 그3명이 서로 아는 관계가 되어 모순. 따라서, 서로 다 알거나 서로 다 모르는 3명의 관계는 항상 적어도2가지씩은 있다.(2) A, B, C가 서로 다 알고, D, E, F가 서로 다 알고, 두 그룹 fA;B;Cg와 fD;E; Fg 사이에는 서로다 모른다고 하자. 그럼 서로 다 알거나 서로 다 모르는 3명의 관계가 정확히 2개뿐임을 확인할 수 있다.

따라서, 그런 관계를 3가지 찾는 것은 불가능할 수 있다. ¤

6. 임의의 정수 11개가 주어지면, 그 중 6개를 뽑아 그 합이 6의 배수가 되게 할 수 있음을 보여라. 단, 주어진 11개의 정수가 모두 서로 다를 필요는 없다. (한국 2003-J4)

증명 ( 전 유성중 3년 김도연)

일단 5개의 원소 중 3개를 뽑으면 그 합이 3의 배수임을 증명하자.

집합을 3으로 나눈 나머지가 같은 것끼리 3개로 나누면

A = f3; 6; 9; 12; 15; 18; 21; :::gB = f1; 4; 7; 10; 13; 16; 19; :::gC = f2; 5; 8; 11; 14; 17; 20; :::g

이다. 비둘기집의 원리로 적어도 한 집합에는 두 개의 원소가 존재

1) 두 원소가 한 집합에 속해 있고 각각 집합에서 1개씩 3개의 원소를 뽑으면 합이 3의 배수가 된다.

2) 세 집합 중 한 집합에 세 개의 원소가 속해 있어도 합은 3의 배수가 된다. : 증명끝!

그렇다면 5개의 원소 중 3개를 뽑으면 그 합이 3의 배수가 된다.

정수 11개가 주어지는데, 이 정수를 a; b; c; d; e; f; g; h; i; j; k라 하자. 이 중 5개인 a; b; c; d; e를 뽑아 그 합이 3의 배수가 되는 수를 a; b; c라 하자. (그래도 일반성을 잃지는 않는다.)

a+ b+ c = 3p (p는 정수) (1)

그리고, 나머지 중에 5수를 뽑아 d; e; f; g; h라 하고, 그들 중에서도 합이 3의 배수가 되는 것을, d; e; f라하면,

d+ e+ f = 3q (q는 정수) (2)

또, 다섯 수를 뽑아 그들을 g; h; i; j; k라 하면, 그들 중에서도 합이 3의 배수가 되는 것이 존재. 그것들을g; h; i라 하면,

g + h+ i = 3w (w는 정수) (3)

그런데, p; q; w 중 비둘기집의 원리로 합이 2의 배수인 것이 적어도 한 쌍 존재! 그것을 p; q라 하면p+ q = 2t. (1)과 (2)를 더하면,

a+ b+ c+ d+ e+ f = 3p+ 3q = 3(p+ q) = 3(2t) = 6t

결국 임의의 정수 11개가 주어지면 그 중 6개를 뽑아 합이 6의 배수가 될 수 있게 할 수 있다. ¤

7. 가로, 세로 격자선이 일정하게 그려진 무한한 격자판의 모든 칸을 검은색 또는 흰색으로 칠하는데, 6개의칸으로 구성된 직사각형(3£ 2 또는 2£ 3)에는 항상 2개의 검은칸이 들어있도록 하였다. 9£ 11 직사각형에는 몇 개의 검은칸이 들어있을 수 있는가? (오클랜드 1998-7)

Page 228: 실전수학올림피아드 1400제 해답

228 조합

풀이 연속한 3칸(일반성을 잃지 않고 가로방향이라 하자)에 2개 이상 검은칸이면(윗변에 붙어있는 경

우에는 그 밑 3칸은 비어있고 다시 그 밑 3칸은 다시 2개 검은칸이므로 일반성을 잃지 않고 변에 붙어있지 않은 경우로 생각할 수 있다) 그 위아래의 6칸은 모두 비어있고, 그럼 세로 방향의 3£ 2 직사각형들을 생각하면 원래의 3칸이 모두 검은칸이어야 해서 모순이 발생.

연속한 3칸이 모두 비어있을 때도 그 아래 3칸은 위와 같으므로 마찬가지.

따라서, 연속한 3칸에는 늘 검은칸이 1개씩. 그럼 모두 33개. }

8. 한 원 위의 서로 다른 9점을 연결하는 36개의 선분 각각을 빨강이나 파랑으로 색칠하였다. 이 9점 중 임의의 3점으로 이루어지는 삼각형 각각이 적어도 하나의 빨간 변을 가진다고 하자. 그럼 어떤 네 점이 있어서 그들 사이를 잇는 6개의 선분이 모두 빨강이 됨을 증명하여라. (캐나다 1976-8)

증명 귀류법으로, 파란 K3도 없고 빨간 K4도 없다고 하고 모순을 찾아보자. 9점 중 한 점을 A라 하자.

A는 파란 변 3개와 빨간 변 5개를 갖는다는 것을 보이자. 만일 파란 변을 4개 이상 가지면, 파란 K3이 없기 위해 이 4점들은 모두 빨간 변으로 연결되어야 하는데, 그럼 빨간 K4가 생겨서 모순이다. 한편, A에파란 변이 2개 이하이면 빨간 변이 6개 이상이고, 잘 알려진 로 이 6개의 점 중에 빨간 K3이 있거나(그래서 A와 함께 빨간 K4를 이루거나) 파란 K3이 있고, 어느 경우도 곤란하므로 역시 모순이다. 그럼 A는정확히 5개의 점과 빨간 변으로 연결되고, 칭적으로 모든 점의 빨간 변의 차수가 5가 된다. 그럼 홀수개의 점이 홀수 차수를 가지므로 빨간 변과 점이 만나는 것은 모두 홀수 번 세어지고, 이것은 빨간 변의개수의 2배여야 하는데 짝수가 아니므로 모순. 따라서, 문제의 명제가 참이다. ¤

9. 지름이 5cm인 원 안에 10개의 점이 있다면 어떤 두 점 간의 거리는 2cm보다 작음을 보여라.(통신강좌 1991-2-41)

풀이 원의 중심부에 지름 2 cm의 작은 원을 하나 그리고, 남은 도넛꼴 테두리 부분을 8등분하자. 그럼

모두 9개의 영역이 생기고, 10개의 점이 있으니 비둘기집의 원리에 의해 어느 한 영역에 두 점이 포함된다. 그 두 점 사이의 거리는 2 cm 이하이다. }

10. 2m개보다 적은 개수의 원소를 갖고 있는 집합인데, m개의 원소를 아무렇게나 택하면 그 합이 나머지 수들의 합보다 항상 클 때, 이 집합을 m-큰 집합이라고 부르자.

(1) 원소의 개수가 2m¡ 1개인 m-큰 집합의 원소는 모두 양수임을 보여라.

(2) 일반적으로,원소의 개수가 n개인 m-큰 집합에 최 로 속할 수 있는 양이 아닌 수의 개수를 km(n)이라고 하자. km(n)을 구하여라. (단, m · n < 2m) (ML프로포절 144-1, 레닌그라드 1991-11 변형)

(1)의 풀이 (성재중 졸업 예창완) 원소의 개수가 2m¡ 1개인 임의의 m-큰 집합의 원소들을

a1 · a2 · a3 · ¢ ¢ ¢ · a2m¡1

이라고 하자. 2m ¡ 1개의 원소를 가지므로 m개의 원소를 택하면 나머지 원소의 개수는 m ¡ 1개이다.

문제의 조건은 가장 작은 m개의 원소들을 택했을 때 그 합이 나머지 큰 원소들의 합보다 크다는 얘기로바꿔도 무방함을 알 수 있다. 따라서,

a1 + a2 + ¢ ¢ ¢+ am > am+1 + am+2 + ¢ ¢ ¢+ a2m¡1

좌변에서 a1을 제외한 모든 원소들을 우변으로 이항시키면

a1 > (am+1 ¡ a2) + (am+2 ¡ a3) + ¢ ¢ ¢+ (a2m¡1 ¡ am)

에서 우변의 각 항들이 모두 0 이상이므로 a1은 반드시 양수이다. 비슷한 방법으로, 혹은 a1 이상의 수들이므로, a2; : : : ; am도 모두 양수임을 알 수 있다. }

(2)의 풀이 (동국 한의 05학번 김규현) 앞에서 km(2m¡ 1) = 0 임을 보였고, km(m) = m¡ 1 인

것도 쉽게 알 수 있다. 여기서 km(n) = 2m¡ 1¡ n 임을 추측할 수 있다. 이제 이것을 증명해보자. 원소가 2m¡ 2 개인 m-큰 집합에서 다음이 성립한다.

a1 + a2 > (am+1 ¡ a3) + (am+2 ¡ a4) + ¢ ¢ ¢+ (a2m¡2 ¡ am) ¸ 0

Page 229: 실전수학올림피아드 1400제 해답

4.2 조합 고급문제 229

만일 a1 · 0 이면 a2 > 0 이므로 양이 아닌 수의 개수는 최 1개이다. 일반화시켜 생각해보면 2m¡k개의 원소를 갖는 m-큰 집합에서(1 · k · m)

a1 + a2 + ¢ ¢ ¢+ ak > (am+1 ¡ ak+1) + ¢ ¢ ¢+ (a2m¡k ¡ am) ¸ 0

이므로 a1; a2; : : : ; ak¡1 · 0 이면 ak > 0 이어서 양이 아닌 수는 기껏해야 k ¡ 1개가 된다. 또한 실제

로, k¡ 1 개의 0과 2m¡ 2k+ 1 개의 1로 이루어진 집합은, m개의 수의 합이 최소 m¡ k+ 1 이고 나머지 수의 합이 최 m¡ k 이므로 m-큰 집합임을 알 수 있다. 따라서, k ¡ 1개가 정확한 최 값이다. 즉,km(n) = 2m¡ 1¡ n 이 성립한다. }

별해 (미국 Cornell 학교 수학과 김현규)

(1) 양이 아닌 수가 있다고 하고, 최소의 원소를 ¡m이라 하자(m ¸ 0). 나머지 2m¡ 2 개의 원소를 둘로 나눠서 한 쪽의 합을 A, 다른 쪽의 합을 B라 하자. A+ (¡m) > B, B + (¡m) > A 가 성립하므로, 두 식을 연립하면 m < 0 으로 모순이 된다.

(2) n = 2m¡ p 라 하자(1 · p · m). 가장 큰 m¡ p개의 원소를 택해 그 합을 S, 그 다음 큰 m¡ p개의 원소를 택해 그 합을 T , 나머지 p개의 원소의 합을 U라 하자. m-큰 집합이므로 T + U > S, 즉U > S ¡ T ¸ 0 이므로, U의 모든 원소가 0 이하일 수는 없고 양수가 적어도 하나는 있다. 따라서,km(n) · p¡ 1. 그런데 0이 p¡ 1개 있고 나머지 2m¡ 2p+1개가 모두 1인 집합에 해 계산을 해보면 m-큰 집합이 됨을 확인할 수 있으므로, 등호가 성립함을 알 수 있다. 즉, km(n) = 2m¡n¡1.

}

11. 어떤 금속 막 기를 1001등분하는 1000개의 각 점에 빨간 구슬을 박고, 1002등분하는 1001개의 각 점에파란 구슬을 박은 다음, 이 막 기를 2003등분하여 잘라내었다. 이렇게 만들어진 2003개의 토막 중 구슬이 박혀있는 토막의 개수를 구하여라. 단, 구슬은 부피가 없는 점으로 간주한다. (한국 2002-J8)

풀이 막 기의 길이를 2003이라 하자. 2003개의 구간

In = [n¡ 1; n] (n = 1; 2; : : : ; 2003)

이 잘린 막 기에 해당한다. 빨간 구슬, 파란 구슬은 각각2003

1001i,2003

1002j 의 위치에 해당하는데,

2003i

1001= 2i+

i

10012 I2i+1;

2003j

1002= 2j ¡ j

10022 I2j

이다. 즉, 빨간 구슬은 I3; I5; ¢ ¢ ¢ ; I2001의 홀수 구간에 하나씩, 파란 구슬은 I2; I4; ¢ ¢ ¢ ; I2002의 짝수 구간에 하나씩 박혀 있다. 따라서, 구슬이 박혀 있는 토막은 I1과 I2003을 제외한 2001개이다. }

12. N이 양의 정수들의 집합일 때, 다음 세 조건을 만족하는 함수 f : N! N 가 존재할까? 있다면 그 중 하나를 찾고, 없다면 그것을 증명하여라.

(1) f(2000) = 1999 이고,

(2) 임의의 소수 p에 해, 수열 f(p); f(2p); f(3p); : : : ; f(kp); : : : 에는 모든 소수가 각각 무한히 많이나타나며,

(3) 임의의 양의 정수 n에 해 f(f(n)) = 1 이 항상 성립한다.(1998 대전.충남 영재수학교실 중급반/고급반 평가시험)

풀이 합성수는 소수에 응시키고, 소수는 1에 응시키는 함수라면 조건 (3)은 해결된다. 이런 함수

중에서 (2)를 만족시키는 것을 찾을 수 있으면, (1)의 조건을 추가하는 것은 간단하다. 서로 다른 소수p; q와 2 이상의 정수 t에 해 f(pqt) = q 로 정하고, 나머지 수들에 해서는 f(2000) = 1999 를 제외하고는 모두 f(n) = 1로 하자. 그럼 세 조건이 모두 만족함을 쉽게 알 수 있다. }

13. 다섯 점을 둘씩 모두 이은 완전그래프 K5는 평면에는 그릴 수 없지만 도너츠 모양의 면 위에서는 그릴 수있다. 그렇다면 평면에는 그릴 수 없지만 구면에는 그릴 수 있는 그래프가 존재하는가?

(셈본중등중급 도전문제 5.3.2)

풀이 존재하지 않는다. 즉, 구면에 그릴 수 있는 그래프는 모두 평면 위에 그릴 수 있다. 구면에 어떤

그래프를 그렸다 하면 그 그래프는 최소 한 개의 면을 가지고 있다. 그러면 그 면 내부의 점을 하나 잡고그 점에 구멍을 뚫은 후 그 구멍을 벌려서 평면을 만들수 있다. 과정에서 그래프의 연결 상태에는 전혀변화를 주지 않았으므로 구면에 그릴 수 있는 모든 그래프는 평면에도 그릴 수 있다. }

Page 230: 실전수학올림피아드 1400제 해답

230 조합

14. 3n £ 3n 체스판을 다음 두 모양의 타일로 덮으려고 한다(n ¸ 1).

(a) 딱 한 칸만 제외하고 완전히 덮는 것이 가능함을 보여라.

(b) 위와 같이 깔 때, B형 타일은 최소 몇 개를 써야 하는가? (통신강좌 1995-10-46)

풀이 (a)

일단 n = 1일 경우 성립함을 보자.

위의 그림과 같이 n = 1일 때는 성립한다.

그리고, n = 2일 때는 그림 1, 2, 3을 참고하면 아래처럼 가능하다.

빈칸이 들어있는 칸은 n = 1일 때처럼 채우면 된다. n ¸ 3일 때는 다음을 생각한다. 빈칸이 들어있는 9 £ 9부분을 제외하고는 각 9 £ 9를 그림처럼 채운다. 그리고 빈칸이 들어있는 부분은 n ¡ 1일때 성립하였으므로 채울 수 있다. 즉 모든 n ¸ 1에 해 완전히 덮는 것이 가능하다.

Α 형 Β 형

그림③그림②그림①

②①

9

99

Page 231: 실전수학올림피아드 1400제 해답

4.2 조합 고급문제 231

(b) 채울 칸수는 (3n)2 ¡ 1개이다. A형으로만 채워지려면 채울 칸수가 3의 배수여야 하는데 그렇지 않다. 따라서 B형은 적어도 1개는 사용되어야 한다. 한편 (a)에서 나타낸 방법을 쓰면 B형은 1개만사용된다. 따라서 B형을 사용하는 갯수의 최소값은 1이다.

}

15. 1에서 9까지의 자연수의 카드가 있다. 두 사람이 번갈아 가면서 카드를 하나씩 가져간다. 카드 세 장으로15를 먼저 만드는 사람이 이긴다. 첫 사람이 최선의 방법을 이용할 때 두 번째 사람은 결코 이길 수 없음을 보여라. (ML프로포절 136-4)

증명 1에서 9까지 중 세 개의 수의 합으로 15를 만드는 경우는 다음의 3£ 3 마방진에 모두 나온다.

8 1 6

3 5 7

4 9 2

따라서, 위의 문제는 이 3 £ 3 마방진에서 두 사람이 O, X 를 번갈아 그리며 Tic-Tac-Toe 게임을 하는것과 같다. 그리고, 잘 알려져있다시피, 이 Tic-Tac-Toe 게임은 두 사람이 최선의 방법을 사용하면 서로비기게 되어있다. ¤

16. 정육면체의 각 꼭지점에 1부터 8까지의 숫자를 하나씩 쓰고, 각각의 모서리에는 양끝 꼭지점의 숫자를 더한 값을 적는다. 모서리의 수들이 모두 다르도록 할 수 있는가? (플란더즈 2002-J3)

풀이 1; : : : ; 8에서 둘을 합하여 만들 수 있는 수는 3; : : : ; 15의 13가지가 있다. 정육면체에는 모두 12개

의 모서리가 있으므로, 모서리의 수들이 모두 다르다면 3; : : : ; 15 중 딱 하나를 제외한 모든 합이 다 나타나야 한다(¤).3, 4, 5, 6이 모두 나타난다고 하자. 그럼 3 = 1 + 2, 4 = 1 + 3 이므로 1점은 2점, 3점과 모두 이웃한다.

그럼 2점과 3점은 서로 이웃하지 않으므로 5 = 1 + 4 로 나타나야 하고, 1점은 4점과도 이웃한다. 그럼2점과 4점도 서로 이웃하지 않으므로 6 = 1 + 5 로 나타나야 하는데, 1점은 이미 2, 3, 4점과 이웃하여더 이상 다른 점과 이웃할 수 없으므로 모순. 따라서, 3, 4, 5, 6 중 모서리에 나타나지 않는 수가 적어도하나 있다. 칭적으로, 12, 13, 14, 15 중에도 모서리에 나타나지 않는 수가 적어도 하나 있고, 그럼 적어도 2개의 수가 모서리에 나타나지 않아야 하므로 (¤)에 모순이다. 따라서, 모서리의 수들이 모두 다르도록 할 수 없다. }

주 한 꼭지점은 3개의 모서리와 이웃하므로, 모든 모서리의 수를 다 합하면

(3 + ¢ ¢ ¢+ 15)¡ a = 3(1 + ¢ ¢ ¢+ 8)

즉, 모서리에 나타나지 않아야 하는 수는 a = 9 라는 것도 알아낼 수는 있다.

17. 4개 이상의 짝수 개의 구슬로 이루어진 목걸이가 있다. 각 구슬은 빨강, 파랑, 혹은 녹색이고 파랑 구슬과녹색 구슬의 수는 같다. 이 목걸이에서 두 곳을 잘라 두 줄로 나누는데, 두 줄의 구슬이 각각 짝수 개이고각각 같은 수의 파랑 구슬과 녹색 구슬을 갖도록 할 수는 없다고 한다. 이 목걸이의 빨강 구슬의 개수로가능한 값을 모두 찾아라. (호주 2004-7)

풀이 ( 전과학고 2학년 황윤하) 빨강, 파랑, 녹색의 구슬을 각각 R, B, G로 표시하고, 그 개수가 각

각 2m, n, n개라고 하자. B와 G가 연속해 있는 경우가 있으면 이 BG를 잘라내면 두 줄 각각 같은 수의 B와 G를 갖게 된다. 또, RR인 경우가 있어도 이것을 잘라내면 된다. 따라서, BG도 RR도 없어야 한다(¤). 두 구슬 X와 Y 사이에 R들뿐이면 X와 Y가 이웃하다고 말하기로 하자. n = 0 이면 RR이 반드시 있으므로 n ¸ 1 이다. 어느 B에서 출발하여 G를 만날 때까지 계속 간다고 생각하면 B와 G가 이웃한경우가 반드시 있다. 그럼 이 이웃한 B와 G 사이에는 (¤)에 의해 R이 꼭 하나 있어야 한다. 즉, BRG가반드시 있다. 이 양옆 어딘가에 R이 있으면 그것까지 잘라내면 또 두 줄 각각 같은 수의 B와 G를 갖게된다. 따라서, 이 양옆에 R이 올 수 없고, 또 BG가 생겨서도 안 되므로 BBRGG꼴이어야 한다. 이렇게계속하면 가능한 경우는

BB¢ ¢ ¢ ¢ ¢ ¢BR RGG¢ ¢ ¢ ¢ ¢ ¢G

과 같은 경우만 가능하다. 따라서, 빨강 구슬의 개수는 2개이다. }

Page 232: 실전수학올림피아드 1400제 해답

232 조합

18. 원탁에 10명의 손님자리가 명찰과 함께 놓여있다. 10명의 손님의 명찰을 확인하지 않고 아무렇게나 않았더니 제자리에 앉은 손님이 한 사람도 없었다. 이 때, 명찰이 놓인 원탁을 적당히 회전시키면 적어도 2명의 손님이 제자리에 앉게 됨을 증명하여라. (한국 1989-1)

증명 처음 앉은 자리에서 원탁을 한자리씩 돌리면 9번 돌리는 동안에 꼭 한번만 자기 자리의 명찰이

돌아오고 10번째 돌리면 처음 자리에 되돌아 온다.

처음에는 아무도 제자리에 앉은 사람이 없고, 9번 돌리는 동안 제자리에 앉게 되는 경우는 10번 있으므로 적어도 한 번은 두사람이 제자리에 앉게 되는 경우가 생긴다. ¤

19. 양의 실수로 이루어진 주어진 순서쌍 (a; b; c; d)를 (ab; bc; cd; da)로 변환한다. 이를 임의로 여러번 반복한다. a = b = c = d = 1 이 아닐 경우에 절 로 원래의 순서쌍과 같아지지 않음을 증명하여라.

(소련 1961-5a)

증명 (a; b; c; d)에서 (ab; bc; cd; da)로 변환할 때, 네 수의 곱은 abcd에서 (abcd)2으로 변화한다. 따라

서, 이 곱은 (abcd)2n

꼴이고, 원래의 순서쌍과 같아지기 위해서는 abcd = 1이어야 한다. d = 1=abc로 두자.

그러면, (a; b; c; 1=abc)! (ab; bc; 1=ab; 1=bc)가 된다. (x; y; 1=x; 1=y)꼴은 변환하면 (xy; y=x; 1=xy; x=y)로다시 (x0; y0; 1=x0; 1=y0)꼴이 되므로, 원래의 순서쌍으로 돌아오기 위해서는 ac = 1이어야 한다.

즉, 순서쌍은 (a; b; 1=a; 1=b)꼴이어야 한다.

이제 네 수의 합을 살펴보자. (a; b; 1=a; 1=b)는 (ab; b=a; 1=ab; a=b)가 되므로 합은 a+ b+ 1=a+ 1=b에서ab+b=a+1=ab+a=b꼴로 변화한다.이 식을 정리하면 (a+1=a)(b+1=b)꼴이 된다.그런데 a+1=a ¸ 2이고 b+ 1=b ¸ 2이므로

(a+1

a) + (b+

1

b) · (a+ 1

a)(b+

1

b)

가 성립한다(등호는 a = b = 1일때만 성립). 따라서 a = b = 1일때만 같고 그 외에는 항상 합이 증가한다. 즉, 원래의 순서쌍과 같아지기 위해서는 a = b = c = d = 1이어야만 한다.(부등식 증명) x; y ¸ 2이면

xy ¡ x¡ y + 1 = (x¡ 1)(y ¡ 1) ¸ 1이므로 xy ¸ x+ y이다(등호는 x = y = 2일때만 성립한다). ¤

20. n개의 서로 다른 소수가 주어져있다. 그 소수들로만 소인수분해되는 n+ 1개의 정수가 있을 때, 이 중 적당히 몇 개를 곱하여 완전제곱수를 만들 수 있음을 보여라. (통신강좌 1996-13-27)

개요 적당히 몇 개를 곱해 만들 수 있는 수는 2n+1 ¡ 1개. 각 소인수의 지수가 홀이냐 짝이냐로 분류

하면 2n가지. 비둘기집으로 지수의 홀짝이 같은 것이 있고, 둘을 곱한 후 중복된(그래서 제곱이 된) 항을 제거하면 됨. ¤

21. 한 정육면체의 모든 꼭지점에 1부터 8까지의 자연수를 아무렇게나 매겼다. 주 각선으로 마주보는 두 꼭지점을 정육면체의 세 변을 이용해 연결하는 경로 중에서, 이 경로 위에 있는 네 꼭지점의 수의 합이 21

이상이 되는 경우가 항상 있음을 증명하여라. (루마니아 2005 지역예선 y8-4)

증명 ( 전 덕중 3학년 이선재)

7과 8의 위치에 따라 경우를 나눠보자.

위 그림에서 `²'이 7과 8이 놓인 위치이다.

(1) 7과 8이 이웃해 있을 때: a와 A 중 큰 것을 택한다. 칭적이므로, 일반성을 잃지 않고 A가 크다고하자. 다음 b와 B 중 큰 것을 택한다. 일반성을 잃지 않고 B가 크다고 하자. 그럼 8 + 7+A+B ¸8 + 7 + 4 + 2 = 21 이 된다. (A와 B 중 작은 것은 그보다 더 작은 것이 있으므로 2 이상이고, A와B 중 큰 것은 네 수 a, A, b, B 중 가장 큰 수이므로 4 이상이다. 따라서 A+B ¸ 4 + 2 이다.)

Page 233: 실전수학올림피아드 1400제 해답

4.2 조합 고급문제 233

(2) 7과 8이 면 각선의 두 끝점일 때: 역시 a와 A 중 큰 것을 택하고, b와 B 중 큰 것을 택하면 (1)과같은 결론.

(3) 7과 8이 주 각선의 두 끝점일 때: 나머지 6개의 점이 모두 칭적이므로 일반성을 잃지 않고 A가가장 크다고 하면 A = 6 이고 이 점을 택해 연결하면 8 + 7 + 6 + x > 21 이 된다.

따라서, 어느 경우에나 주 각선의 양 끝점을 잇는 경로의 네 꼭지점의 수의 합이 21 이상이 되는 경우가 항상 있다. ¤

22. (a) 어느 해 연말에, 어떤 학급의 전체 20명의 학생이 각각 급우 10명에게 연하장을 보냈다. 그럼 서로 연하장을 주고받은 2명의 학생이 반드시 있음을 증명하여라.

(b) 위의 문제를 일반화하여, 학급의 학생수를 n명이라 하고, 각 학생은 각각 급우 m명에게 연하장을 보낸다고 하자. 서로 연하장을 주고받은 2명의 학생이 반드시 있음을 확신할 수 있으려면 n과 m은 어떤 값이라야 하는가? (플란더즈 1993-1)

풀이 m ¸ n=2 이면 비둘기집 원리로 됨. m < n=2 일 때의 반례는, n명의 학생을 원형으로 배열해두

고, 각 학생마다 자기 왼쪽으로 m명에게 연하장을 보내는 것으로 하면 됨. }

23. 어느 시험에서 채점을 한 결과, 어떠한 두 개의 문제를 선택하여도 이 두 문제 중 한 문제만을 푼 학생은항상 6명 이상이었다. 시험을 본 학생이 11명이었다면 출제된 문제의 개수가 12 이하임을 보여라. (단, 등호조건을 보일 필요는 없음.) (한국 2005-J4)

증명 (성남 수내중 3학년 임동혁)

문제가 n개이고, 1; 2; : : : ; n번 문제를 푼 학생들의 집합을 각각 A1; A2; : : : ; An이라 하자. 문제의 조건에서 임의의 i 6= j 에 해 jAi [Aj j ¡ jAi \ Aj j ¸ 6, 즉

jAij+ jAj j ¡ 2jAi \Aj j ¸ 6

이다. 이것을 모든 i; j 에 해 다 더해보자(1 · i < j · n).

S = (n¡ 1)(jA1j+ ¢ ¢ ¢+ jAnj)¡ 2(jA1 \A2j+ ¢ ¢ ¢+ jAn¡1 \Anj) ¸ 6³n2

´= 3n(n¡ 1)

이 된다. 각 학생이 푼 문제의 개수를 k1; k2; : : : ; k11이라 하면

S = (n¡ 1)(k1 + ¢ ¢ ¢+ k11)¡ 2µ³k1

2

´+ ¢ ¢ ¢+

³k112

´¶¸ 3n(n¡ 1)

S = n(k1 + ¢ ¢ ¢+ k11)¡ (k21 + ¢ ¢ ¢+ k211) ¸ 3n(n¡ 1)

이 된다. S를 완전제곱식으로 고치면

S = ¡11Xi=1

³ki ¡ n

2

´2+11n2

4· 11n2

4

이므로 11n2

4¸ 3n(n¡ 1) 이다. 즉, n · 12 이다. ¤

24. knight's tour란, 체스판에서 기사(장기의 `마'의 움직임과 같다)가 어느 한 칸도 두 번 들르지 않고 꼭 한번씩 지나가는 방법을 말한다. 4£ 4 체스판에서는 knight's tour가 존재하지 않음을 보여라.

(통신강좌 1997-14-12)

증명

체스판을 다음과 같은 세 종류로 나누어 보자.(각 칸은 꼭지점, `마의 움직임 가능'은 선분과 같은 역할을 한다. 그냥 참고삼아¢ ¢ ¢ )

Α

Β1

Β2

Α

Β2

Χ

Χ

Β1

Β1

Χ

Χ

Β2

Α

Β2

Β1

Α

Page 234: 실전수학올림피아드 1400제 해답

234 조합

이제 살펴보면, A에 knight가 있으면 다음 움직일 수 있는 곳은 C뿐이다.Bi에서는 또 다른 Bi 또는 C로 이동이 가능하다. (i = 1; 2)C에서는 B1; B2 또는 A로 이동이 가능하다.

이제, knight's tour가 존재한다면, knight가 이동한 곳의 기호를 한 줄로 나열하면, 위에서 구한 성질에의해

C ! AorBi들(Bi 여러개도 가능)! C ! AorBi들 ! ¢ ¢ ¢(물론 (A or Bi들)! C로 시작될 수도 있다.)

그런데, C는 4개인데, C사이에 있어야 할 (앞이나 뒤도 가능) A;Bi가 6가지 이상이므로 (A4개, B1; B2로B가 2가지)

이러한 배열은 불가능하다.) 모순) knight's tour가 존재하지 않는다. ¤

25. 연결상태가 구와 같은 다면체에서는 오일러의 공식 v ¡ e+ f = 2 가 성립함을 안다. (단, v, e, f는 각각꼭지점, 모서리, 면의 개수이다.) 연결상태가 도너츠와 같은, 즉 구멍이 하나 뚫린 다면체에서는 언제나

v ¡ e+ f = 0

이 성립함을 보여라. 단, 이 다면체의 각 면은 구멍 없는 다각형이어야 한다. 예를 들어 다음과 같이 구멍이 있는 면이 그려진 경우에는 그 면을 몇 개의 면으로 더 쪼개야 한다.

(KAIST 대전.충남 영재수학교실 중급반 2000)

풀이 도넛 형태의 다면체는 반으로 잘라서 구와 연결상태가 같은 두개의 다면체로 만들 수 있다. 이때

단면은 2개가 나오는데 그 단면 2개는 각각 다각형이 된다. 두 면의 v; e를 v0; e0이라 하면 v0 = e0이다.

한편, 나누어진 다면체 각각의 v; e; f를 vi; ei; fi(i = 1; 2)라 하면 vi ¡ ei + fi = 2이고 원래 도넛 형태의 다면체에서 v = v1 + v2 ¡ v0; e = e1 + e2 ¡ e0; f = (f1 ¡ 2) + (f2 ¡ 2)이므로

v ¡ e+ f = (v1 + v2 ¡ v0)¡ (e1 + e2 ¡ e0) + (f1 + f2 ¡ 4)= (v1 ¡ e1 + f1) + (v2 ¡ e2 + f2) + (e

0 ¡ v0)¡ 4= 2 + 2 + 0¡ 4 = 0

따라서 v ¡ e+ f = 0이다. }

26. 주어진 n개의 정수의 합이 94이다. 한 원의 원주 위에 시계뱡향으로 이 정수들을 늘어놓는데, 각각의 정수는 그 다음 두 정수의 차와 같도록 하였다. 가능한 n을 모두 구하여라. (뉴질랜드 2000-7)

풀이 늘어놓은 수가 모두 똑같을 수는 없다. 극값 원리로 가장 큰 것을 택해 논하면 a a 0 ¢ ¢ ¢ 의 반복

꼴뿐임이 금방 됨. }

27. 양의 정수들을 3 £ 3 으로 배열하는데 각 행의 수의 합, 각 열의 수의 합, 두 주 각선의 수의 합이 모두m으로 같도록 만든 것을 합방진이라 하자. m이 3의 배수임을 보이고, 이 배열의 수들 중 가장 큰 수는 최

2m3¡ 1 임을 보여라. (아일랜드 1989-2)

증명 합방진의 정가운데에 오는 수를 x라고 하자. 가운데를 지나는 모든 줄(행, 열, 각선)을 합하

면 모든 수를 더하고 가운데 원소를 3번 더 더한 것이므로 3m + 3x가 되고, 각 줄은 합이 m이므로3m+ 3x = 4m이다. 따라서 m = 3x로 3의 배수이다.

가운데가 x이고 각 줄의 합이 m = 3x이므로 가운데를 지나는 임의의 줄에서 나머지 원소는 x+ y; x¡y(y ¸ 0)로 표현할 수 있다. x¡ y > 0이므로 x > y이고 x+ y < 2x = 2m

3 이다. 그러므로 배열의 수들

은 기껏해야 2m3¡ 1이하이다. ¤

Page 235: 실전수학올림피아드 1400제 해답

4.2 조합 고급문제 235

28. n은 짝수이다. 자연수 1; 2; : : : ; n 중에서 a+ c = b+ d 를 만족하는 서로 다른 네 수의 집합 fa; b; c; dg를고르려고 한다. 이렇게 수를 고르는 경우의 수가

n(n¡ 2)(2n¡ 5)24

가지임을 보여라. (호주 1989-4)

풀이 귀납법. or a < b < c 인 세 수를 고르면 d는 저절로 결정. 2b = a+ c 인 경우(b = d 인 경우) 배

제하고 a, b, c 등의 순서가 섞이는 것을 고려해주면... }

29. 체스판에서 나이트(knight)는 날일자(日)의 각선 방향으로 이동한다. 즉, 체스판이 충분히 넓다면 나이트는 한 점에서 8군데로 이동할 수 있는 것이다. 7£ 7 체스판을 나이트가 각 칸을 정확히 한 번씩 들러 출발지로 되돌아 올 수 있는가? (통신강좌 1997-14-20)

풀이 체스판을 흑백을 교 로 칠하면, 나이트는 흑인 칸에서 백인 칸, 혹은 백인 칸에서 흑인 칸으로

만 이동할 수 있다. 체스판에 흑은 25개, 백은 24개이다. 한편 모든 칸을 돌고 오는 방법(해밀턴 회로)이있다면, 흑의 수와 백의 수가 같아야 하므로 모순, 따라서 불가능. }

30. k개의 변을 가지는 연결된 그래프 G의 각 변에 1; 2; 3; : : : ; k의 번호를 모두 붙이는데, 2개 이상의 변에 공통으로 속하는 G의 정점에서는 번호들의 최 공약수가 1이 되게 붙일 수 있음을 보여라.

(통신강좌 1997-15-22)

증명 (과기원 수학과 96학번 신희성) 주어진 점에 이웃한 변의 총수를 그 정점에서의 차수라 하자. 모

든 점에서의 차수의 총합은 변의 개수의 2배이므로 짝수이다. 따라서 차수가 홀수인 점은 짝수개이다.

그래프 G가 연결되어 있으므로, 이 그래프의 임의의 두 홀수점을 잇는 통로를 만들 수 있다. 따라서 우리는 그래프 G를 홀수점을 양 끝점으로 하는 몇 개의 통로와 몇 개의 회로로 분해할 수 있다. 그리고 회로는 다른 회로 및 통로와 합칠 수 있으므로 연결된 그래프 G가 2m개의 홀수점을 가지면 m개의 통로로, 홀수점이 없으면 오일러 회로로 분해된다.

각 통로나 회로에 순서 로 1, 2, 3, : : : , k까지 번호를 매기면 차수가 2이상인 점에 연결된 두 선분은 숫자가 연속이므로 그 점에 이웃한 변에 붙여진 숫자들의 최 공약수는 1이다. ¤

31. m£n 크기의 판의 각 칸에 한 자리 숫자를 하나씩 써넣는데, 어느 위치에서 가로, 세로 적당한 크기의 직사각형꼴의 부분영역을 잡아도 그 영역 안의 숫자 전체의 합이 5의 배수가 되지 않게 하고 싶다. mn의 최

값을 구하여라. (IT꿈나무 올림피아드 2006 1차)

풀이 한 줄에 5개의 수 a, b, c, d, e가 연속으로 있다고 하고 직사각형 영역의 다섯 가지 합 a, a + b,

a+ b+ c, a+ b+ c+ d, a+ b+ c+ d+ e 를 생각하자. 이 중에 5로 나눈 나머지가 같은 것이 있다면 그둘의 차 또한 직사각형 영역의 합이 되고 5의 배수가 되므로 곤란하다. 따라서, 이 다섯 합이 모두 나머지가 서로 다른데, 그럼 이중에 5의 배수(나머지가 0인 수)가 있으므로 역시 곤란하다. 그러므로, 한 줄에 5개의 수가 있으면 안 되고, m, n 모두 4 이하이다. 그리고, m = n = 4 일 때는 모든 칸에 1을 넣으면 조건을 만족한다. 따라서, mn의 최 값은 4 ¢ 4 = 16. }

32. 집합 f1; 2; 3; 4; 5; 6; 7; 8g에서 3개의 원소를 갖는 부분집합을 몇 개 만드는데, 어느 두 부분집합도 공통원소가 많아야 하나뿐이도록 하고자 한다. 이런 성질을 만족하는 부분집합들을 최 몇 개까지 만들 수 있는가? (뉴질랜드 2003-6)

풀이 (광주 동성고 3학년 방재혁)

우선 부분집합의 개수가 9개 이상이면 불가능함을 보이자. 부분집합의 개수가 9개 이상이면 원소의 총개수는 27개 이상이다. 그러면 비둘기집의 원리에 의해 1, ..., 8 중 적어도 하나는 4개 이상의 부분집합에 속한다. 그 수를 a라 하면 a를 포함하는 네 집합

fa; x1; x2g; fa; x3; x4g; fa; x5; x6g; fa; x7; x8g

에서 x1; : : : ; x8은 모두 달라야 한다. 그런데 x1; : : : ; x8은 a를 제외한 7개의 수 중에서 골라야 하므로 이

Page 236: 실전수학올림피아드 1400제 해답

236 조합

것은 모순이다. 8개의 부분집합은 다음과 같이 만들 수 있다.

A = f1; 4; 6gB = f1; 2; 3gC = f3; 6; 8gD = f3; 4; 5gE = f2; 5; 8gF = f5; 6; 7gG = f2; 4; 7gH = f1; 7; 8g

따라서, 최 8개까지 만들 수 있다. }

33. X는 집합 f1; 2; 3; : : : ; 1989g의 부분집합이다. X의 어떤 두 원소의 차도 4나 7이 되지 않는다고 할 때,X는 최 몇 개의 원소를 가질 수 있는가? (AIME 1989-13)

풀이 %EEE

(Thanks to Bugz Podder for this.)

We show first that we can choose at most 5 numbers from {1, 2, ... ,

11} such that no two have difference 4 or 7. Translating if necessary,

we make take the smallest number to be 1. That rules out 5,8. Now we

can take at most one from each of the pairs: 2,9; 3,7; 4,11; 6,10.

1989 = 180·11+9, so that means we can pick at most 5·181 = 905.

If we pick 1, 3, 4, 6, 9, then that works and moreover it allows us

also to pick 11+1, 11+3, 11+4, 11+6, 11+9. So 905 is also possible.

¢ ¢ ¢ 답 905 }

34. 평면 위에 n개의 다각형을 그렸는데, 임의의 두 다각형은 항상 서로 만나고, 접하는 점이나 세 다각형이모두 지나는 점은 없다고 한다. 이렇게 해서 생긴 교점의 수를 v라 하고, 다각형들에 의해 나뉜 평면의 영역(제일 바깥의 무한 영역은 제외)의 수를 f라 할 때,

f = v + 1

이 성립함을 증명하여라. (KAIST 대전.충남 영재수학교실 주말교육 2000)

증명 귀류법으로, 성립하지 않는 경우가 있다고 하자. n = 1 일 때 1 = 0 + 1 로 성립하므로 n ¸ 2 인

경우이다. 그런 경우들 중에 가장 작은 n을 택하자. 그려진 다각형 중 어느 하나를 택해, 그 다각형이 다른 다각형과 만나는 교점을 2k개라 하자. 그럼 그 다각형은 2k개의 둘레 조각으로 분할됨. 각각의 둘레조각은 원래 하나였던 영역을 둘로 나누고 있음. 따라서, 이 다각형을 들어내면 f 0 = f ¡2k, v0 = v¡2k가 됨. n의 최소성에 해 더 작은 n¡ 1일 때는 성립하므로 f 0 = v0 + 1 이고, 그럼 입하면 f = v + 1

도 성립하게 되어 모순. ¤

별증 오일러의 평면그래프 공식 v ¡ e + f = 1 에 4v = 2e (각 교점마다 4개의 변이 연결되므로) 를

입하면 바로 나옴. ¤

35. 어떤 왕국에 N명의 기사가 살고 있다. 각 기사들은 서로 간에 친구이거나 적이다. 그리고 각 기사들은 정확히 3명의 적을 가지고 있다. 만약 이 왕국이 \친구의 적은 나의 적이다."라는 법에 의해 지배된다면 가능한 N의 값은? (통신강좌 1998-16-28)

풀이 기사 A와 B가 서로 적이라고 하자. 조건에 의해 다른 기사들은 A; B중의 하나 이상과는 적이

다. 이에 따라 N ¸ 7이면 A; B 중 적어도 하나는 넷 이상의 적을 가진다.

그러므로 4 · N · 6이다. 각 N명의 기사에게 적이 세명이므로3N

2쌍의 적이 존재한다. 즉, N은 짝수

이다. 그러므로 N=4, 6만이 가능하다. 또한, 서로 모두 적인 경우 N = 4 가 가능하고, 3 : 3의 두 팀이서로 적 하는 상황이면 N = 6 도 가능하다. }

Page 237: 실전수학올림피아드 1400제 해답

4.2 조합 고급문제 237

36. 1 · m < n · 40 인 780종류의 도미노 [m;n]을 모두 모은 집합을 D라 하자. 이 도미노들 중에서 몇 개를일렬로 늘어놓는데, 인접한 두 도미노 [a; b] [c; d]는 b = c 를 만족해야 한다. 각각의 도미노 [m;n]은 [m;n]

또는 [n;m]과 같이 늘어놓을 수 있다. 최 몇 개의 도미노를 늘어놓을 수 있을까? (AIME 1998-15)

풀이 %EEE

There are 39 elements [m, 40]. Unless 39 appears as the first or

last number in the line, it must occur an even number of times.

Similarly for the other numbers. So 38 numbers and hence 19 dominos

[m, n] cannot appear. So the sequence cannot be longer than 780 - 19 = 761.

We claim that a sequence of 1 + (4 + 8 + ... + 4n-4) with first number 1

and last number 2 is possible for the set of dominos with largest number 2n.

Induction. n = 1. [1,2] gives 1.

[1 2n-1][2n-1 2][2 2n][2n 3][3 2n-1][2n-1 4] ... [2n-2 2n][2n 1] prefixed

to the line for 2n-2 gives 4n-4 more dominos than the line for 2n-2.

(The extra dominos are the pairs [2n-1 a] and [2n a] for a = 1, 2, ... , 2n-2.)

Putting n = 40 shows that 761 is realised.

¢ ¢ ¢ 답 761 }

37. A와 B 두 사람이 32개의 공기돌을 두고 돌 가져가기 게임을 한다. 먼저 A가 시작한 후 차례를 번갈아 돌을 가져가는데, 자기 차례에 1개 혹은 적당한 소수 개의 돌을 가져갈 수 있다. 마지막 돌을 가져가는 사람이 이기는 것으로 할 때, 두 사람 모두 최선의 전략을 따른다면 누가 이기겠는가? (IMTS R3-5)

풀이 B가 이긴다. A는 4의 배수를 만들 수 없고, B는 항상 4의 배수를 만들면 된다. }

38. 1부터 16까지의 숫자를 4£ 4 숫자판에 적당히 배치하였다. 각 행과 각 열의 숫자의 합을 구했더니 이 8개의 합이 모두 서로 다르고 또 모두 n의 배수가 되었다. n의 최 값은 얼마인가?

(IT꿈나무 올림피아드 2006 1차)

풀이 1 + 2 + ¢ ¢ ¢ + 16 = 8 ¢ 17 이므로 n은 8 ¢ 17의 약수이다. 만일 n이 17의 배수이면, 각 행과 열의

합 8개가 모두 다르다고 했으므로 그 중 가장 큰 값은 8 ¢ 17 이상이 된다. 이는 가장 큰 4개의 수의 합13 + 14+ 15+ 16 보다 훨씬 크므로 불가능하다. 따라서, n은 8의 약수이다. n = 8 일 때도 앞과 마찬가지로 8개의 합 중 가장 큰 것은 8 ¢ 8 이상이고 이것은 13 + 14 + 15 + 16 보다 크므로 불가능하다. n = 4

일 때는 다음과 같이 가능한 경우가 있다.

1 8 3 4 16

5 6 7 2 20

9 10 11 14 44

13 16 15 12 56

28 40 36 32 합

¢ ¢ ¢ 답 4 }

39. 가로 5, 세로 2인 칠판이 벽에 고정되어 있다. 칠판을 여러 개의 사각형들만으로 분할한 것을 `그림'이라하자. 각각의 분할된 사각형이 한 변의 길이가 1인 정사각형이거나 가로 1, 세로 2인 직사각형이거나 또는 가로 2, 세로 1인 직사각형일 때 그 그림을 `멋진 그림'이라 하자. 멋진 그림은 모두 몇 개인가?

(한국 2006 1차-J20)

풀이 (인천 광성중 2학년 이원열)

n£ 2 크기의 칠판에서 만들 수 있는 `멋진 그림'의 수를 f(n)이라 하고, 아래 그림과 같이 (n¡ 1)£ 2에한 개의 정사각형을 덧붙인 칠판에서 만들 수 있는 `멋진 그림'의 수를 g(n)이라 하자.

Page 238: 실전수학올림피아드 1400제 해답

238 조합

f(n)은 (i) a를 정사각형으로 분할할 경우: 총 g(n)가지, (ii) ac를 묶어 분할할 경우: 총 f(n ¡ 1)가지,(iii) ab로 묶어 분할할 경우는 c를 정사각형으로 분할할 경우: g(n ¡ 1)가지와 cd를 묶어 분할할 경우:f(n¡ 2)가지가 있다. 따라서,

f(n) = g(n) + f(n¡ 1) + g(n¡ 1) + f(n¡ 2) (n ¸ 3)

가 된다. 비슷하게 g(n)에 해서도 e를 기준으로 생각하면

g(n) = f(n¡ 1) + g(n¡ 1) (n ¸ 2)

이것을 이용해 f(n)을 간단히 하면

f(n) = 2g(n) + f(n¡ 2)

f(0) = 1, g(1) = 1, f(1) = 2 로 볼 수 있으므로 이 점화식으로 작은 n에 해 g, f의 값을 구해 나가면

g(2) = 1 + 2 = 3

g(3) = 3 + 7 = 10

g(4) = 10 + 22 = 32

g(5) = 32 + 71 = 103

f(2) = 2 ¢ 3 + 1 = 7f(3) = 2 ¢ 10 + 2 = 22f(4) = 2 ¢ 32 + 7 = 71f(5) = 2 ¢ 103 + 22 = 228

문제에서 원하는 것이 f(5)이므로 ¢ ¢ ¢ 답 228 }

40. 어떤 그룹에 29명의 학생이 있다. 각 학생은 항상 진실만을 말하거나 항상 거짓말을 한다. 어느날 모든 학생들이 원형으로 모여 앉았다. 그리고 각 학생들은 \내 옆의 두 학생은 모두 거짓말쟁이다."라고 얘기했다고 한다. 이 그룹에는 진실을 말하는 학생이 10명 이상임을 보여라. 또, 정확히 10명일 수 있겠는가?

(통신강좌 1998-16-30)

풀이

만약 10명 이하의 학생이 진실을 얘기한다면 최소한 세 거짓말쟁이는 이웃하여 앉게 된다. 이 경우 세거짓말쟁이 중 가운데 사람은 진실을 얘기하고 있는 것이므로 모순이 발생한다. 정확히 10명의 학생이진실을 얘기하는 예는 다음과 같다. (T는 진실을 얘기하는 학생, L은 거짓말쟁이 학생이다.) }

41. 3종류의 색을 사용해서 평면 위의 모든 점에 색을 칠한다. 이 때 반드시 그 거리가 1이 되는 2개의 똑같은색의 점이 있음을 증명하여라. (셈본중등초급 도전문제 1.2.2)

증명 귀류법을 이용하여, 거리가 1이 되는 두 점은 항상 서로 다른 색이라고 가정하고 모순을 찾아보

자. 한 변의 길이가 1인 정삼각형을 이루는 세 꼭지점 A, B, C는 항상 서로 다른 세 가지 색의 점이 된다. 선분 BC에 한 A의 칭점 A0도 B, C와 정삼각형을 이루므로, A와 A0은 같은 색의 점이 된다.A와 A0 사이의 거리는

p3이고, 거리가

p3이 되는 임의의 두 점은 항상 같은 색의 점이 됨을 알 수 있

다(그런 두 점 사이에는 항상 두 정삼각형을 이루는 B, C와 같은 점을 잡을 수 있으므로). 따라서, A를중심으로 반지름

p3인 원을 그리면 이 원 위의 점은 모두 A와 같은 색이고, 이 원 위에 거리 1인 두 점

이 있으므로 모순이다. 따라서, 거리가 1이 되는 2개의 같은 색의 점이 반드시 있다. ¤

T

T

L

L

LTT

T

T

LLL

L LL

T

L

L

L

LLT

LLTLLTLL

Page 239: 실전수학올림피아드 1400제 해답

4.2 조합 고급문제 239

42. 혼자서 하는 다음과 같은 퍼즐이 있다. 퍼즐판에는 말을 끼워넣을 수 있는 구멍 여러 개가 둥글게 원형으로 배열되어 있다. 한 번의 움직임이란, 어떤 구멍에 끼워져 있는 말 하나를 없애고 그 양옆의 구멍의 상태를 바꿔놓는 것이다(구멍의 상태를 바꿔놓는다는 것은, 거기에 말이 있으면 그 말을 없애는 것을, 말이없으면 새로운 말을 끼워넣는 것을 뜻한다). 처음에 퍼즐판에 딱 하나의 말이 놓여있는 것으로 시작하고,

퍼즐판에 말이 하나도 남지 않도록 하는 것을 목표로 한다. 퍼즐판의 구멍이 12개라면 이것이 가능한가?

구멍이 10개이면 어떤가? 일반적으로, 구멍의 개수가 어떤 경우에 이것이 가능한가?(오스트리아-폴란드 1981-4)

풀이 (부산 장전중 3학년 안현태, 수정됨)

퍼즐판의 구멍의 개수를 n이라 하자.

(i) n = 3k + 1꼴일 때: 말 하나에서 시작하여 다음과 같은 과정을 차례로 거치자.

: : O : : : : : : :: O : O : : : : : :: O O : O : : : : :

: O O O : O : : : :...

: O O O O O O O : OO O O O O O O O O :

그럼 3k개의 연속한 말을 얻을 수 있다. 이것을 3개씩 묶어 없애면 모든 말을 다 없앨 수 있으므로가능하다.

(ii) n = 3k + 2꼴일 때: (i)의 방법을 응용하여 먼저 다음의 꼴을 만들자.

: O O O O O O O O : OO O O O O O O O O O :

여기서 제일 앞의 말을 택해 움직이면 다음의 꼴이 된다.

: : O O O O O O O O O

그럼 역시 연속한 3k개의 말이 남으므로 모두 없앨 수 있다.

(iii) n = 3k꼴일 때: 이 때는 불가능함을 보이자. 퍼즐판의 각 위치에 차례로 1부터 3k까지 번호를붙이자. 1; 4; 7; : : : ; 3k ¡ 2 위치의 말의 개수를 p개, 2; 5; 8; : : : ; 3k ¡ 1 위치의 말의 개수를 q개,3; 6; 9; : : : ; 3k 위치의 말의 개수를 r개라 하자. 어느 위치의 말을 택해 움직이든, 항상 p, q, r의 말을 하나씩 상태를 바꾸게 되므로 p, q, r의 홀짝이 모두 한꺼번에 변한다. 즉, 맨처음에 (p; q; r) =(홀,짝,짝) 으로 시작하므로, (홀,짝,짝)과 (짝,홀,홀)만을 번갈아 갖게 된다. 말을 모두 없앴을 때는(짝,짝,짝)이므로 이것은 불가능하다.

따라서, n = 3k + 1 혹은 3k + 2꼴일 때만 가능하고 3k꼴일 때는 불가능하다. }

43. 한 자리 수, 즉 0; 1; 2; : : : ; 9로 이루어진 수열 a1; a2; : : : ; a99가 다음 성질을 만족한다.

an = 1 이면 an+1 6= 2; an = 3 이면 an+1 6= 4

이 때, ak = al, ak+1 = al+1 인 서로 다른 두 수 k; l 2 f1; 2; ¢ ¢ ¢ ; 98g 이 존재함을 보여라.

(폴란드 1996 2차-4)

증명 순서쌍 (ak; ak+1) 들을 생각하자. 문제는 이런 98개(k = 1; 2; :::; 98)의 순서쌍 중에 같은 것이

있음을 보이라는 것이다. 10개의 숫자로 순서쌍을 구성하므로 모두 100가지가 가능한데, 그중 (1; 2)와(3; 4)를 제외해야 하므로 가능한 순서쌍은 98가지. 따라서, 두 번 사용된 순서쌍이 없으려면 모든 가능한 순서쌍이 딱 한 번씩 사용되어야 한다. 그런데, 숫자 1은 앞자리로 9번, 뒷자리로 10번 사용되었으므로 맨마지막에 사용되어야 한다. 이것은 숫자 3도 마찬가지이므로 모순. ¤

44. n(> 1)개의 점이 주어져 있다. 어떤 두 점들은 선분으로 이어져 있다(같은 점에서 같은 점을 연결하는 선분은 없다). 어떤 두 점이 주어져도 한 점에서 다른 점까지 선분을 따라 도달하는 길이 꼭 하나씩 있다고하자. 이 때 n¡ 1 개의 선분이 있음을 증명하여라. (소련 1961-8)

Page 240: 실전수학올림피아드 1400제 해답

240 조합

증명 점의 숫자에 해 수학적 귀납법을 사용하자.

먼저 n = 1일때, 선분의 개수는 0이므로 n¡ 1개의 선분이 있다.

이제 n · k개의 점이 있을때 선분의 개수가 항상 n ¡ 1개라고 하자. 이때 k + 1개의 점이 있는 조건을만족시키는 그래프를 생각하자. 이 그래프에서 한 선분을 없앤다면 어떻게 될까?

제거하는 선분의 양 끝점을 각각 a; b라고 하자. 만일 임의의 점 c에 해 c에서 a로 가는 경로가 이 선분을 포함하지 않는다면, c에서 b로 가는 경로는 반드시 이 선분을 포함해야만 한다(b에서 c로 가는 경로가 유일하기 때문이다). 마찬가지로, c에서 a로 가는 경로가 이 선분을 포함한다면 c에서 b로 가는 경로는 이 선분을 포함하지 않아야 한다. 즉, 이 선분을 제거하면 그래프가 a를 포함하는 쪽과 b를 포함하는쪽 2개로 나눠지며 경로의 유일성이 보존된다.

이제 양쪽의 그래프의 점의 개수를 n1; n2라 하면 n1; n2 · k이고 k+ 1 = n1 + n2이다. 따라서 총 선분의 개수는

(n1 ¡ 1) + (n2 ¡ 1) + 1 = (n1 + n2)¡ 1= n¡ 1

이다. ¤

45. 한 변의 길이가 정수 cm인 각 크기의 정육면체 블록들이 무제한으로 주어져있다. 정확히 2001개의 블록을 이용해(빈틈없이 쌓아) 정육면체를 만들 때 필요한 서로 다른 크기의 블록은 최소 몇 종류인가?

(뉴질랜드 2001-6b)

풀이 2001a3이 세제곱수가 되는 경우는 없으므로 한 종류로는 안 된다. 23과 13 두 종류만 이용하여

만들 수 있음을 보이자. 23 블록을 n개 사용한다면 8n+ (2001¡ n) = m3 을 만족하는 경우를 먼저 찾아야 한다. 즉, m3 = 2001 + 7n ´ ¡1 (mod 7) 이고, 이런 m은 7k + 2; 3; 6꼴이면 항상 가능하다. n이작을수록 실제 쌓는 것이 유리하므로 2001보다 큰 m3 중에서 가장 작은 것을 찾으면 m = 133 = 2197.

이 때, 23 블록 28개와 13 블록 1973개를 사용하면 실제로 쉽게 만들 수 있다. }

46. 종이 위에 p ¸ 2 개의 점을 찍고 게임을 시작한 후, 두 명의 경기자가 번갈아 다음과 같이 변을 그린다: 한번의 차례에서 단 하나의 변을 그리는데, 이미 그려진 그림에서 두 점을 연결하는 곡선을 그리거나, 한 점을 자기 자신에게 연결하는 곡선을 그릴 수 있다. 단, 새로 그리는 변은 이미 그려진 다른 변이나 점과 교차할 수 없고, 각 점의 차수는 최 3까지만을 허락한다. 그리고 새로운 변을 그린 후에는 그 변의 중앙에새로운 한 점을 추가한다.

이런 시행을 반복하다가 더 이상 변을 추가할 수 없을 때, 마지막 변을 그린 사람이 이기는 것으로 한다.

이 게임은 항상 유한 번의 시행을 한 후 끝나게 됨을 보이고, 이 시행 횟수의 최 값을 p에 해 구하여라.(새싹 놀이, 통신강좌 1999-18-5)

시작 1번 2번 3번 4번

풀이 모든 점의 가능한 차수의 합을 생각하자. 처음에 p개의 점이 있으니 가능한 차수의 합은 3p에서

시작한다. 만일 어떤 두 점이 연결되면 그 두 점이 차수를 하나씩 잃고 가능한 차수가 1인 새로운 점이하나 생겨나므로 전체 가능한 차수의 합은 1이 줄어든다. 처음에 3p에서 시작하며 가능한 차수의 합이1보다 작아질 수 없으므로(가능한 차수의 합이 1이면 한 점이 남았다는 것인데 스스로를 연결하는 곡선은 차수를 2 줄이므로 불가능하다) 이 게임은 항상 유한 번의 시행 후에 끝난다.

최 한 게임이 오래가는 방법은 차수가 3p에서 시작하여 1씩 줄어들어 최후에 1이 남는 경우이다. 즉 시행횟수는 3p¡ 1이 최 라고 예상할 수 있다. p에 한 귀납법으로 3p¡ 1번의 시행이 가능함을 보이자.p = 1일때는 처음 점에 자기자신으로의 곡선을 긋고(1번) 그 곡선 위에 생긴 점과 처음 점을 연결하는곡선을 그으면(2번) 된다.p = k일때 3k¡1번의 시행이 가능하다고 하자. p = k+1개의 점이 있다면 먼저 k개의 점에 해 3k¡1번의 시행을 한 후, 나머지 1개의 점에 2번의 시행을 하고, 마지막으로 양쪽에서 가능한 차수가 1인 최후의 점들을 연결해주면 된다(최후의 점들이 바깥쪽에 드러나도록 항상 할 수 있으므로 가능하다).

따라서 3p¡ 1번의 시행이 최 이다. }

47. 음이 아닌 정수들의 수열 fakgk¸1 이 모든 k ¸ 1 에 해 ak ¸ a2k + a2k+1 을 만족한다.

Page 241: 실전수학올림피아드 1400제 해답

4.2 조합 고급문제 241

(가) 모든 양의 정수 n ¸ 1 에 해, 수열 fakg에는 n개의 연속한 0이 항상 있음을 보여라.

(나) 문제의 조건을 만족하면서 0이 아닌 항을 무한히 많이 갖는 수열의 예를 하나만 찾아 제시하여라.(루마니아 2005 지역예선 y9-4)

풀이 (가) ak가 0이 아니면 a2k와 a2k+1 중에 적어도 하나는 ak보다 작다. 즉, 0이 아닌 항에 해서

그보다 작은 항을 항상 찾을 수 있으므로, 음 아닌 정수들에서는 더 작은 항을 계속 쫓아가다보면 유한번 안에 0인 항을 만날 수 밖에 없다. 그리고, ak = 0 이면 a2k = a2k+1 = 0 으로 2개의 연속한 0이 나

타나게 되고, 그럼 다시 a4k = a4k+1 = a4k+2 = a4k+3 = 0 으로 4개의 연속한 0이 나타나게 되고, 귀

납적으로 임의의 자연수 m에 해 2m개의 연속한 0이 항상 나타나게 된다. 따라서, 임의의 자연수 n에해 n개의 연속한 0이 항상 있다.

(나) 다음의 수열이 한 예가 된다.

1; 1; 0; 1; 0; 0; 0; 1; 0; 0; 0; 0; 0; 0; 0; 1; 0; 0; 0; 0; 0; 0; 0; 0; : : :

즉, k = 2n 꼴일 때 ak = 1 이고 그외에는 모두 0인 수열. (이외에도 예는 얼마든지 많다.) }

48. A는 16개의 자연수를 모은 집합이다. A에서 서로 다른 임의의 두 원소를 곱하면 항상 1994를 넘지 않는다고 한다. A에서 서로소가 아닌 두 원소를 찾을 수 있음을 증명하여라. (인도지역예선 1994-5)

증명1 귀류법으로 풀자. A의 모든 수가 둘씩 서로소라 하자. 그럼 A에 속하는 1이 아닌 수 x는 반드시

소인수 p를 하나는 갖는데, p를 A의 다른 수와 공유할 수 없으므로 x마다 서로 다른 p를 하나씩 갖는다.A에 속하는 1이 아닌 수를 x1; x2; : : : ; x15라 하고, p1; p2; : : : ; p15들을 각각의 한 소인수라 하자. pi들은모두 서로 다르므로 일반성을 잃지 않고 p1 < p2 < ¢ ¢ ¢ < p15 라 할 수 있다. 소수를 작은 것부터 차례로 15개를 나열하면 2, 3, 5, 7, 11, 13, 17, 19, 23, 29, 31, 37, 41, 43, 47이므로 p14 ¸ 43, p15 ¸ 47 이다. 그럼

x14x15 ¸ p14p15 ¸ 43 ¢ 47 = 2021 > 1994가 되어 모순이다. 따라서, A의 모든 수가 다 둘씩 서로소일 수는 없다. ¤

증명2 역시 귀류법으로 풀자. a 2 A 가 합성수이고 p가 a의 한 소인수라면 A에서 a를 제거하고 신

p를 넣자. 그래도 A의 수들은 여전히 둘씩 서로소이고 또 두 수를 곱하면 1994 이하이다. 이런 과정을반복하여 A에 합성수가 하나도 없도록 할 수 있고, 그럼 1과 소수들만 남는다. 이 때, A의 가장 큰 두 수a14, a15는 각각 14번째와 15번째로 작은 소수인 43, 47 이상이므로,

a14a15 ¸ 43 ¢ 47 > 1994

가 되어 모순이다. 따라서, 귀류법의 가정이 틀렸고, A에서 서로소가 아닌 두 원소를 찾을 수 있다. ¤

49. 수직선 위의 점 1,1

2,1

3,1

4,1

5, : : : 들을 생각하자. 길이가 k인 5개의 닫힌구간으로 이 점들을 모두 덮을

수 있을 때, k의 최소값을 구하여라. (플란더즈 1996-3)

풀이 답은 0.1이다. k = 0:1 일 때는 다섯 개의 구간으로 [0; 110], [ 19; 15], [ 14; 13], 12, 1을 각각 덮으면 된

다. k < 0:1 이면 0:1¡ k 보다 작은 1n을 하나 택할 수 있고, 그럼 1

n, 110, 15, 13, 12, 1을 각각 따로 덮어

야 하므로 불가능. ¢ ¢ ¢ 답 0.1 }

50. 유한한 넓이를 가진 정사각형 모양의 평면이 있다. 갑과 을이 이 평면 위를 임의의 크기의 볼록다각형 타일로 채우는데, 도형이 서로 겹치게 되거나, 평면의 테두리를 넘게 되는 사람이 진다. 단, 도형의 크기는 바로 전에 놓인 도형의 크기 이상이어야 하고, 맨 처음에 놓이는 도형의 크기는 최 평면의 넓이의 1/100이다. 갑이 처음 시작할 때 갑의 필승전략을 구하여라. (ML프로포절 165-1)

풀이 갑은 처음에 평면의 중심 O를 점 칭이 되도록 덮는다. 그 후에는 을의 플레이를 O에 해 점

칭이 되도록 따라 하면 된다. 그럼 을이 타일을 둘 수 있다면 갑도 항상 타일을 둘 수 있고(점 칭이 유지되므로 을이 둔 타일을 제외하면 을이 둔 곳의 점 칭인 곳은 둘 수 있고, 타일은 볼록다각형이므로 바로 전에 을이 둔 타일과 새로 갑이 둔 타일이 어떤 점에서 만난다면 그와 점 칭인 점에서도 만나고 그럼 그 두 교점을 연결한 선분 위에 있는 O도 지나므로 모순), 넓이의 유한성에 의해 게임은 항상 유한 번안에 끝나므로 갑이 이긴다. }

Page 242: 실전수학올림피아드 1400제 해답

242 조합

51. 2, 3, : : : ; 102를 적당한 순서로 늘어놓은 것을

a1; a2; : : : ; a101

이라 하자. 이런 수열 중에서 모든 k에 해 ak가 k의 배수가 되는 것들을 모두 찾아라. (Towns 1980 J3)

풀이 ai 6= i 인 i에 해 생각해보자. ai = j 라 하면 j는 i가 아닌, 즉 i보다 큰 i의 배수이다. j는 이미

ai가 차지했으므로, aj = k 라 하면 k도 j보다 큰 j의 배수가 되고, 이런 식으로(an를 함수 a(n)으로 생

각하면)

i! j ! k ! ¢ ¢ ¢ (! y ! z) (¤)계속 자신보다 큰 배수로 응되는 fang의 부분수열이 생긴다. 102 이하의 수만 사용하므로 이것이 무한히 계속될 수 없고, 그럼 ay = z 에서 끝난다면 az는 더이상 정의되지 않는 항이어야 하므로 z = 102

여야 한다. a1 > 1 이므로 부분수열 (¤)은 1부터 시작하여 102로 끝나는 것 하나뿐이고, fang의 나머지항들은 모두 an = n 의 자기자신으로 응되는 항이어야 한다. 즉, 102 = 2 ¢ 3 ¢ 17 = p1 ¢ p2 ¢ p3 의 약수들

1; p1; p2; p3; p2p3; p3p1; p1p2; p1p2p3(= 102)

들중에서 (¤)을 만족하는 부분수열을 하나 정할 때마다 수열 fang이 하나씩 결정된다.

² 1! 102 : 1개

² 1! pq ! 102꼴 : 3개

² 1! p! 102꼴 : 3개

² 1! p! pq ! 102꼴 : 3£ 2 = 6개의 13개가 구하는 모든 수열이 된다. }

52. ABCD는 사면체이다. 네 점 A, B, C, D로부터 모두 같은 거리에 있는 평면은 몇 개인가?(플란더즈 예선 1989/1990)

풀이 평면은 항상 공간을 둘로 나누므로 네 점으로부터 같은 거리에 있는 평면은 점 A;B;C;D를 두

그룹으로 나눈다. 만일 A;B;C;D가 모두 한쪽에 있다면 A;B;C;D는 한 평면에 있고 4면체를 이루지못한다. 따라서 한쪽 그룹이 원소를 항상 1개 이상 가진다. A;B;C;D를 한쪽이 1개 이상이 되도록 분할하는 개수는 7개(= 24=2¡ 1)이므로, 평면의 개수는 총 7개이다. }

53. X는 집합 f1; 2; 3; : : : ; 15g의 부분집합이고, X의 서로 다른 어떤 두 부분집합도 원소의 합이 같지 않다고한다. X의 원소의 합의 최 값을 구하여라. (AIME 1986-12)

풀이 %EEE

The greedy algorithm gives X = {15, 14, 13, 11, 8}, sum 61,

but can we do better? If there are 6 elements, then there are 26 - 1 = 63

sums (excluding the empty subset). One of these has 6 elements and six

have 5 elements. The other 56 have 4 or less elements and hence sum

at most 15 + 14 + 13 + 12 = 54, contradiction (there are only 54 possible

distinct sums). It is now messy. If we do not include 15, 14, 13,

then ... If we do, then we cannot include 12. If we do not include 11 either,

then ... etc.

¢ ¢ ¢ 답 61 }

54. 삼각형의 세 변의 길이가 될 수 있는 서로 다른 세 수를 골라낼 수 있는 집합을 삼각성을 갖는다고 말하기로 하자. 집합 f4; 5; 6; : : : ; ng의 10개의 원소를 갖는 부분집합이 모두 삼각성을 가질 때, n ¸ 13 의 최값을 구하여라. (AIME 2001 2차-5)

풀이 %EEE

Clearly (4, 5, 9, 14, 23, 37, 60, 97, 157, 254) does not have the

triangle property (each element from 9 on is the sum of the previous two).

But if a_1 < a_2 < ... < a_10 does not have the property, then

a_10 ≥ a_9 + a_8 ≥ 2a_8 + a_7 ≥ 3a_7 + 2a_6 ≥ ... ≥ 34a_2 + 21a_1

≥ 34·5 + 21·4 = 254.

¢ ¢ ¢ 답 253 }

Page 243: 실전수학올림피아드 1400제 해답

4.2 조합 고급문제 243

55. 다음 그림과 같이 도너츠 모양의 별이 있다. 이 별에서 완전그래프 K7을 그릴 수 있는가?

완전그래프 Kn이란 n개의 도시가 각각 둘씩 모두 직행도로(도중에 다른 도로와 만나지 않는 도로)로 연결되어 있는 구조를 말한다. (통신강좌 1992-4-21)

풀이

}

듀얼 그릴 수 있다. 직사각형 위에 그림과 같은 그래프를 그린 후에 윗변과 아랫변을 E와 E, F와 F가

만나도록 붙이면 원통형의 모양이 된다.

그 원통을 다시 B와 B, C와 C, D와 D가 만나도록 맞붙이면 도너츠 모양이 되며 육각형 꼴의 각각의 영역은 모두 다른 색으로 색칠되어야 한다. }

56. 한 변의 길이가 1인 정사각형 안에 그 길이의 합이 18인 유한 개의 선분이 있다(정사각형은 경계를 포함하고 선분도 양끝점을 포함하는 것으로 하자). 각 선분은 정사각형의 어느 변에 평행해야 하고, 선분끼리서로 교차할 수 있다. 이 선분들로 나뉘어진 정사각형의 영역들 중에 적어도 하나는 0.01보다 적지 않은넓이를 가짐을 증명하여라. (Towns 1980 J5/S5)

증명 (광주고 1학년 서준영, 수정됨)

주어진 유한 개의 선분들이 정사각형을 몇 개의 영역 A1; A2; : : : ; An으로 나눈다고 하고, 임의의 영역A에 해 A의 넓이를 S(A), 그 영역의 내부 및 둘레의 선분의 길이의 합을 d(A)로 나타내기로 하자. 각

A

E

D

CG

A

F

J K I H A

G

C

BB

D

E

AJ K I H

6

7

2

1

3

4

5

F

Page 244: 실전수학올림피아드 1400제 해답

244 조합

d(A)를 모두 합하면 정사각형 둘레에 있는 것들은 한 영역의 경계로 쓰이고, 정사각형 내부에 있는 것들많아야 두 영역의 경계로 쓰이므로, X

d(Ai) · 2£ 18 + 4 = 40

이 된다. 임의의 영역 A에 해, A를 포함하는 가장 작은 (정사각형의 변에 평행한 변으로 된) 직사각형을 A0으로 나타내자. 그럼

S(A) · S(A0); d(A) ¸ d(A0)

이 성립한다. 또, A0의 두 변의 길이를 x, y라 하면 산술-기하 평균부등식으로

d(A0) = 2x+ 2y ¸ 4pxy = 4pS(A0)

이 성립한다. 따라서, 위의 식들을 종합하면

XpS(Ai) ·

XqS(A0i) ·

1

4

Xd(A0i) ·

1

4

Xd(Ai) · 1

4¢ 40 = 10

그런데, 모든 영역의 넓이의 합은 정사각형과 같아XS(Ai) = 1

이다. 즉, bi = 10pS(Ai) 으로 치환하면X

b2i = 100;X

bi · 100

의 상황이 된다. 귀류법을 쓰자. 모든 영역의 넓이가 0.01보다 작다면 bi < 1 이 된다. 그럼 b2i < bi 이다.

이것은100 =

Xb2i <

Xbi · 100

으로 100 < 100 이 되므로 모순. 따라서, 넓이가 0.01 이상인 영역이 적어도 하나 있다. ¤

57. n명의 사람이 둥근 탁자 주위에 앉아있다. 바로 오른쪽에 앉은 사람이 자신과 성별이 같은 사람의 수와바로 오른쪽에 앉은 사람이 자신과 성별이 다른 사람의 수가 같다고 한다. n이 4의 배수임을 증명하여라.

(독일BW 1973 1차-4)

증명 이웃한 두 사람의 짝을 모두 고려하자. 한 사람을 지정해 그 사람으로부터 출발하여 한 바퀴를

돌아 자기 자신으로 돌아올 때까지 성별이 짝수번 바뀌어야 원래의 성별로 돌아올 수 있으므로, 성별이다른 짝의 수는 짝수개(2a개라 하자). 또한 문제에서 성별이 같은 짝의 수와 성별이 다른 짝의 수가 같다고 했으므로, 짝은 모두 2a+ 2a = 4a개. 짝의 수는 사람의 수와 같으므로 n = 4a. ¤

별증 성별을 §1로 나타내면 aiai+1 = 1인 경우와 ¡1인 경우의 i의 개수가 같다.즉, S =P

aiai+1 =

0. 여기서 한 ak의 부호를 바꾸면 S의 항 중 둘의 부호가 바뀌는데, f¡1;¡1g ! f1; 1g이거나 f¡1; 1g !f1;¡1g 이거나 f1; 1g ! f¡1;¡1g 과 같이 바뀌고, 그럼 S는 각각 4 증가, 불변, 4 감소이므로, S가mod 4로 불변임을 알 수 있다. 이렇게 한 항씩 부호를 바꾸는 일을 반복하여 모든 항을 +1로 바꾸면 그때의 S의 값은 n이고 이것이 처음의 S의 값 0과 mod 4로 같으므로 n은 4의 배수. ¤

58. 갑과 을은 각각 n장의 카드를 갖고 있다. 각자 1부터 2n까지의 자연수를 자기 카드의 각 면에 하나씩 마음 로 적어넣었다. 두 사람의 카드 2n장을 모아 탁자에 늘어놓는데, 1부터 2n까지의 모든 수가 윗면에보이도록 할 수 있음을 보여라. (아벨콘테스트 1996 결선 3)

59. n과 m은 고정된 자연수들이다. 꼭지점이 A = (0; 0), B = (n; 0), C = (n;m), D = (n ¡ 1;m), E =(n¡ 1; 1), F = (0; 1) 로 주어진 육각형 ABCDEF를 n+m¡ 1개의 단위정사각형으로 분할하였다. 이그림의 격자선을 따라 각 점을 중복하여 들르는 일 없이 A에서 C로 가는 경로의 수를 구하여라.

(오스트리아-폴란드 1979-7)

60. p는 음이 아닌 정수이고 n = 2p 이다. 집합 f1; 2; : : : ; ng의 모든 부분집합들 중에서 x 2 A 인 모든 x에해 2x =2 A 인 부분집합 A는 최 몇 개의 원소를 갖는지 구하여라. (프랑스 1991-4)

Page 245: 실전수학올림피아드 1400제 해답

4.2 조합 고급문제 245

61. P와 Q는 n£ n 크기의 체스판에서 가로로 이웃한 두 칸으로, P가 Q의 왼쪽 칸이다. 말 하나를 P에서 출발시켜 체스판을 돌아다니게 하려고 한다. 허용된 움직임은 오른쪽이나 윗쪽, 혹은 좌하쪽으로 한 칸을 이동하는 것이다. (예를 들어, E5에 있었다면 E6이나 F5, 또는 D4로 이동할 수 있다.) 어떤 n에서도 이 말이 체스판의 모든 칸을 딱 한 번씩 들러 Q에 도착할 수는 없음을 증명하여라. (독일BW 1971 1차-4)

62. 2000개의 사과가 몇 개의 양동이에 담겨있다. 여기서 몇 개의 양동이를 없애고 다른 몇몇 양동이들의 사과 일부를 없앨 수 있다. 또는 몇 개의 양동이만 없애거나 몇몇 양동이에서 사과 일부만 없앨 수도 있다.

사과가 100개 이상 남아있으면서 남아있는 모든 양동이에 같은 수의 사과가 담겨있도록 할 수 있음을 증명하여라. (Towns 1987가을 J6)

증명 각 양동이에 담긴 사과의 개수를 a1 ¸ a2 ¸ ¢ ¢ ¢ ¸ am 이라 하자(m · 2000). a1 ¸ 100 이면

바로 끝. a2 ¸ 50 이어도 바로 끝. 이런 식으로 ai ¸ 100i 인 i = 1; 2; : : : ;m 이 있으면 바로 끝. 따

라서, ai < 100i 일 때만 보면 됨. 그럼 2000 = a1 + a2 + ¢ ¢ ¢ + am < 100(1 + 1

2+ 13+ ¢ ¢ ¢ + 1

m) ·

100(1 + ( 12+ 12) + ( 1

4+ 14+ 14+ 14) + ( 1

8+ ¢ ¢ ¢+ 1

8) + ¢ ¢ ¢+ ( 1

210+ ¢ ¢ ¢+ 1

210) = 100 ¢ 11 로 모순. ¤

63. 주어진 n 2 N 에 해, A를 f1; 2; : : : ; ng의 부분집합들을 일부 모은 집합이라고 하자. 모든 두 집합B;C 2 A 에 해서 집합 (B [C) n (B \C) 가 짝수개의 원소를 가진다고 할 때, A의 원소는 최 몇 개인가? 단, X n Y 는 차집합을 뜻한다. (체코슬로바키아 1986-1)

풀이 jBj+ jCj 가 항상 짝수라는 것과 같고, 이것은 다시 jBj들의 홀짝이 모두 같다는 것. 원소가 홀수

개(혹은 짝수개)인 부분집합의 개수는 항상 2n¡1개임이 잘 알려진...(일 일 응으로 증명가능) }

64. 직교좌표계의 제1사분면이 격자선들에 의해 무한개의 단위정사각형칸으로 분할되어있다. 각각의 칸에 자연수를 하나씩 써넣는데, 각 행과 각 열마다 모든 자연수가 꼭 하나씩 있도록 할 수 있는가?

(Towns 1988봄 JA6)

풀이 할 수 있다. 원점에 이웃한 칸에 먼저 1을 쓰고, 귀납적으로 다음과 같이 하면 된다: 원점에 이웃

한 2n £ 2n 정사각형의 꼴을 A라 하면, B = 2n +A (A의 각 항에 2n을 더해 얻은 꼴) 이라 할 때 원점

에 이웃한 2n+1 £ 2n+1 정사각형은B AA B

꼴이 되도록 하면 됨. }

65. A < B 인 모든 자연수 순서쌍 (A;B)들의 집합을 생각하자. 이 쌍들 중 일부를 `검은색'으로 표시하고나머지는 `흰색'으로 표시하였다. A, A + D, A + 2D꼴의 임의의 세 자연수에 해, 세 쌍 (A;A + D),(A;A+ 2D), (A+D;A+ 2D) 중에 항상 두 가지 색이 모두 있도록 색칠할 수 있는가?

(Towns 1987가을 S5)

풀이 할 수 있다. Y ¡X 의 소인수분해에서 2의 지수가 짝수이면 (X;Y )를 흰색, 홀수이면 검은색으

로 하면 됨. }

66. 체스판의 어떤 칸에 말이 하나 놓여있다. 두 명이 번갈아 이 말을 다른 칸으로 옮기는 게임을 한다. 규칙은 바로 전에 움직인 거리보다 항상 더 먼 거리로 움직여야 한다는 것이다(말은 항상 각 칸의 중심에만 놓을 수 있다). 더 움직이지 못하는 사람이 진다. 두 사람 모두 최선의 전략으로 게임을 한다고 할 때, 누가이기겠는가? (Towns 1989봄 JA2)

풀이 칭성의 게임 }

67. 연습장 한 페이지를 (충분히 많은) 작은 정사각형칸들로 분할하여 23가지의 색으로 칠하였다(23가지의색을 모두 사용해야 한다). 서로 다른 색의 두 칸이 이웃하면 그 두 색을 `좋은 쌍'이라 부르자. 좋은 쌍은최소 몇 개인가? (Towns 1988가을 SO4)

풀이 그래프가 연결되어야 하므로 변은 최소 22개. 지층처럼 나누어 23개의 색을 차례로 쓰거나, 하

나의 색을 바탕색으로 하고 나머지 색을 군데군데 한 칸씩 쓰면 됨. }

68. 3£ 3£ 3 루빅 큐브의 표면에 있는 각각의 작은 정사각형칸마다 각선을 하나씩 그려, 스스로와 만나지않는 한 꺾은선 경로를 얻을 수 있는가? (Towns 1989봄 SO4)

69. y = ax2 + bx+ c 꼴의 이차함수 100개의 그래프로 좌표평면을 분할할 때, 가능한 영역의 개수의 최 값은 얼마인가? (Towns 1990봄 JA1)

Page 246: 실전수학올림피아드 1400제 해답

246 조합

풀이 두 이차함수는 많아야 두 점에서 만난다. k개의 이차함수를 그렸을 때 분할된 영역의 수를 ak라

하면, 우선 a1 = 2 이고, k+ 1번째 이차함수를 그리면 이 이차함수의 그래프가 앞서 그려진 k개의 이차함수와 각각 두 점씩 최 2k개의 점에서 만나고, 그럼 그 교점들에 의해 k+1번째 곡선이 2k+1개의 조각으로 나뉘어지는데, 각 조각은 원래 하나였던 영역을 둘로 나누게 되므로 영역의 개수는 최 2k+1개늘어난다. 즉, ak+1 · ak+(2k+1). 이 점화식을 풀면 ak · k2+1 이 된다. 등호가 성립하는 경우인 모

든 이차함수가 서로 두 점씩 만나고 모든 교점이 서로 다를 때는 쉽게 구현할 수 있다. n = 1; 2; : : : ; 100

에 해 pn : y = 100nx2 ¡ n 정도로 하면 충분하다. ¢ ¢ ¢ 답 1002 + 1 }

70. 신발 30짝이 일렬로 놓여있다. 15짝은 오른발용이고 15짝은 왼발용이다. 왼쪽 5짝과 오른쪽 5짝으로 된연속한 10짝을 찾을 수 있음을 증명하여라. (Towns 1991봄 JO3)

증명 i번째 신발이 오른발용이면 ai = 0,왼발용이면 ai = 1로 나타내자. si = ai+ai+1+¢ ¢ ¢+ai+9 로

정의할 때, s1; s2; : : : ; s21 중에서 딱 5인 것이 있음을 증명하는 문제가 된다.귀류법으로,딱 5가 되는 것이없다고 하자. si는 i가 증가함에 따라 연속적으로 변화하므로(ai와 ai+10이 같으면 si = si+1 이고 다르면

jsi+1¡ sij = 1), 딱 5가 되는 것이 없다면 모두 다 4 이하이거나 모두 다 6 이상이다. 모두 6 이상인 것은

0과 1의 정의를 뒤바꾸면 마찬가지이므로 4 이하일 때만 풀면 충분하다. 그럼 X := s1+s2+ ¢ ¢ ¢+s21 ·21¢4 = 84. 그런데 X = 10(a1+a2+¢ ¢ ¢+a30)¡(9a1+8a2+¢ ¢ ¢+a9)¡(a22+2a23+¢ ¢ ¢+9a30)이다.이것을 X = 150¡L¡R 이라 놓으면,먼저 a1; : : : ; a9 중에는 1인 것이 4개 이하이므로 L · 9+8+7+6 = 30,마찬가지로 R · 30. 따라서, X = 150¡ L¡R ¸ 90 으로 앞에서 얻은 X · 84 와 모순이다. ¤

71. 주어진 자연수 N에 해, a+ b+ c = N 을 만족하는 서로 다른 자연수해 (a; b; c)를 생각하자. 이런 순서쌍 몇 개를 고르는데, 어떤 두 쌍에도 같은 수가 없도록 한다. 이렇게 고를 수 있는 최 의 개수를 K(N)이라 할 때, 다음을 증명하여라.

(a) K(N) >N

6¡ 1

(b) K(N) <2N

9(Towns 1989가을 JA4)

72. 평면의 각 점을 정확히 1988개의 원이 지나도록 하면서 평면을 원들로 가득 덮을 수 있을까?(Towns 1988봄 JA5)

풀이 1988 신 2로 바꿔놓았을 때 성공하면 충분하다(그 작업을 994번 반복하면 되므로). 그리고 이

것은, 무한개의 평행선을 그린 후 각각의 이웃한 두 평행선 사이에서 그 두 직선에 동시에 접하는 모든원을 다 모아놓으면 된다. }

73. 8£ 8 체스판을 다음과 같은 조건으로 만들 때 최소 몇 개의 칸을 칠해야 하는가?

(a) 색칠된 두 칸은 접할 수(변이나 점을 공유할 수) 없고,

(b) 색칠되지 않은 칸은 어떤 색칠된 칸과 접해야 한다. (Towns 1988가을 SA1)

74. 100개의 계단이 있다. 롱다리 K군이 이 계단을 내려가려 하는데, 6계단 혹은 7, 8계단 점프(k계단 점프란k¡ 1개의 계단을 넘어 k번째 계단에 도착하는 것을 말한다)로만 오르락과 내리락을 한 번씩 번갈아 하면서 같은 계단을 두 번 밟지 않고 내려갈 수 있는가? (Towns 1989봄 JA1)

75. A와 B가 N개의 성냥개비를 가지고 게임을 하였다. 먼저 A가 성냥개비들을 각 더미가 두 개 이상이 되도록 두 더미로 나눈다. 다음 B가 두 더미를 각각 둘로 나누는데 모두 1개 이상이 되도록 나눈다. 그리고B가 먼저 4개의 더미 중 정해진 규칙에 따라 2개를 갖고 A가 나머지를 갖는다. A, B는 모두 자기가 최한 많은 성냥개비를 갖도록 더미를 나누고 선택한다. 규칙은 다음의 3가지 중 어느 하나로 이루어져 있다.

(1) B는 가장 큰 더미(둘 이상일 경우엔 그 중 하나)와 가장 작은 더미(둘 이상일 경우엔 그 중 하나)를갖는다.

(2) B는 가운데의 두 더미(경우 1일 때 A가 갖게 되는 두 더미)를 갖는다.

(3) B는 가장 큰 더미와 가장 작은 더미를 갖거나, 두 개의 가운데 더미를 갖는 것에 한 선택권이 있다.

각각의 규칙에 해 A, B 모두 최선의 경기를 했을 때 A가 갖게 되는 성냥개비는 몇 개 일까?(소련 1961-10)

Page 247: 실전수학올림피아드 1400제 해답

4.2 조합 고급문제 247

풀이 v = bN2c, w = dN

2e 을 나타내는 것으로 하자. v + w = N 임에 유의하자.

(1) 답은 v이다. A가 나눈 두 더미를 x ¸ y 라 하면 B는 x를 x¡ 1과 1로 나눠 x를 다 가져갈 수 있다. 따라서, B는 항상 w 이상 가져갈 수 있다. 반 방향을 보자. A가 x¡ y = 0 또는 1이 되도록 나누면, B가한 쪽에서 두 더미를 다 가져가는 경우는 B가 w 이하만 가져가게 되므로, 양쪽에서 한 더미씩 가져가는경우만 살펴보면 된다. x = a+ b, y = c+ d 이고 최 더미와 최소 더미가 a, d라면, a+ d · a+ b = x.

최 더미와 최소 더미가 c, b일 때도 c+ b · c+ d = y. 어느 경우나 B는 w 이하만 가져갈 수 있다. 즉,

둘다 최선을 다한다면 B는 w만큼, 그리고 A는 v만큼 가져가게 된다.(2) 답은 w이다. A가 N ¡ 2와 2로 두 더미를 나누면 1인 더미가 무조건 둘 생기므로 중간의 두 더미는기껏해야 bN¡2

2c+ 1 = v개이다. 한편, A가 x ¸ y 로 나누면 B는 양쪽 더미를 각각 최 한 절반에 가깝

게 나누면 중간의 두 더미의 합은 v 이상이 됨을 확인할 수 있다. 따라서, B는 v개를 가져가고, A는 w개를 가져가게 된다.(3) 답은 v이다. B는 (1)의 전략을 취한다면 당연히 (1)에서의 결과인 w 이상을 가져가는 것이 보장된다. A가 x¡ y = 0 또는 1이 되도록 나누면, B가 한 쪽에서 두 더미를 다 가져가는 경우는 (1)과 마찬가지이므로, 역시 양쪽에서 한 더미씩 가져가는 경우만 살피면 된다. 또한 (1)에서 살펴보지 않은, B가 중간 두 더미를 선택하는 경우만 살피면 된다. 그 경우에도 (1)에서와 비슷하게 할 수 있는데, 최 더미와최소 더미가 a, d라면 b+ c · b+ a = x, c, b라면 a+ d · c+ d = y 여서 역시 중간 더미의 합도 w 이하이다. }

76. 총합이 같은 양수들의 집합이 두 개 있다. 첫 번째 집합은 m개의 수를 가지고 있고 두 번째 집합은 n개의수를 가지고 있다. 이 때 m£ n 크기의 행렬에 m+ n개보다 적은 수를 써넣어 각 행의 합이 첫 번째 집합과 같고 각 열의 합이 두 번째 집합과 같도록 할 수 있음을 증명하여라.

예 예를 들어 n(A) = 3, n(B) = 2, A = f1; 5; 3g, B = f2; 7g 일 때 다음과 같이 채울 수 있다.(소련 1962-14)24¡ 5

¡ 12 1

35증명 그냥 첫 행(최초의 수만 제외하고)의 수를 각 칸마다 그 칸이 속한 열의 합(두 번째 집합의 수

들)과 같은 수로 하고 마찬가지로 첫 열(최초의 수만 제외하고)의 수를 각 칸마다 그 칸이 속한 행의합(첫 번째 집합의 수들)과 같은 수로 하면 첫 행 첫 열에 해당하는 칸의 수는 행합이든 열합이든 어느한 쪽으로 정하면 나머지 한 쪽은 두 집합의 총합이 같다고 했으므로 저절로 성립함. ¤

77. 정육각형 A1A2A3A4A5A6의 모든 변과 각선을 파란색 또는 빨간색으로 칠했다. 단, 어떠한 삼각형AjAkAm (1 · j < k < m · 6) 도 세 변이 모두 파란색인 경우는 없다. 각각의 k = 1; 2; : : : ; 6 에 해

빨간 선분 AkAj (j 6= k) 의 개수를 Rk라 할 때, 다음을 보여라. (호주 1984-4)

6Xk=1

(2Rk ¡ 7)2 · 54

증명 m = min(Rk) 라 두면, m ¸ 2 일 때는 (2Rk¡7)2 · 32 으로 바로 끝. m = 0, m = 1 일 때만 별

도로 경우를 잘 확인해주면 됨. 각각 A · 49+ 1+1+1+1+1 = 54, A · 25+1+1+1+1+25 = 54의 꼴이 됨. ¤

78. Patera라는 나라에는 20개의 도시와 이들 도시간의 교통을 제공하는 두 항공사 Green Planes와 RedPlanes가 있다. 비행 노선은 다음과 같이 주어져있다.

(i) 임의의 두 도시 간의 비행 노선은 오직 한 항공사에 의해서만 운영되고, 노선이 없는 경우는 존재하지 않는다. 이 때 비행기는 두 방향 모두 운행하며 중간에 멈추는 일은 없다.

(ii) Patera에는 Red Planes만을 이용해서는 (중간에 다른 도시를 경유하더라도) 오갈 수 없는 두 도시S, B가 존재한다.

Patera의 어떠한 두 도시에 해서도, 중간에 다른 도시를 최 한 곳만 경유하면 Green Planes만을 이용하여 이 두 도시를 오고 갈 수 있음을 보여라. (호주 1987-3)

증명 X, Y 를 임의의 두 도시라 하자. XY 가 G면 끝이므로 XY 가 R일 때만 생각. XSY 나 XBY 가

GG이면 역시 끝이므로 R이 각각 하나 이상 있고 그럼 S, B 사이에 R만의 경로가 생기게 되어 모순. ¤

Page 248: 실전수학올림피아드 1400제 해답

248 조합

79. (a) 14개의 톱니를 가진 두 개의 똑같은 톱니바퀴가 있다. 하나가 다른 하나의 위에 톱니가 정확히 포개어지도록 얹어놓고, 응하는 네 쌍의 톱니를 임의로 골라서 잘랐다. 윗바퀴를 잘 돌려서 두 바퀴의 그림자가 완전한 하나의 톱니바퀴가 되도록 항상 할 수 있는가? (톱니바퀴 하나를 뒤집어서 포개는 것은 허락되지 않는다.)

(b) 13개의 톱니로 하고 (a)를 다시 풀어라. (Towns 1988가을 JA4)

풀이 (이윤한)

(a) 두 톱니바퀴가 포개어져 있을 때 위에 있는 잘린 4개의 톱니를 순서 로 a, b, c, d라 하고 아래 있는잘린 4개의 톱니는 위에 a, b, c, d와 응하는 로 a0, b0, c0, d0이라 하자. 그림자가 완벽한 톱니바퀴 모양을 하지 않을 때는 처음 a, b, c, d 모두 a0, b0, c0, d0와 겹치는 때 1가지, a가 b0, c0, d0과 포개지는 3가지, b가 a0, c0, d0과 포개지는 3가지, c가 a0, b0, d0과 포개지는 3가지, d가 a0, b0, c0과 포개지는 3가지, 총13가지 경우 이상이 될 수 없다. 따라서 톱니가 14개일 때는 가능하다.(b) 두 톱니 모두에서 1, 2, 4, 10을 자르면 된다. }

80. 모든 변의 길이가 1이고 마주보는 변끼리 모두 평행한 2n각형을 \rhombic"이라 한다. n = 4 일 때의 아래의 예시 그림처럼, 이런 rhombic 2n각형은 한 변의 길이가 1인 마름모들로 서로 다른 몇 가지 방법으로분할할 수 있다.

666개의 마름모로 분할할 수 있는 rhombic 2n-각형이 존재하는 n은 어떤 값들인가? (IMTS R11-3)

풀이 (송지용) 2n각형이 666개의 마름모들로 나누어진 그림을 생각하자. 이 그림에서 전체 선분들의

개수는(666£4+2n)

2= 36£ 37+ n개이다(¤). 2n각형 위에서 임의로 한 변(기준 꼭지점에서 시계방향으

로 k번째 변)을 잡아 이것과 평행한 선분들을 갖는 모든 마름모들의 집합을 Pk, k = 1; 2; : : : ; n이라 하자. 그러면 Pk는 자기 자신을 제외한 나머지 n¡ 1개의 Pm, m 2 f1; : : : ; ng ¡ fkg들과 꼭 하나의 마름모를 공통원소로 갖는다. 즉, 각각의 Pk에는 n¡1개의 마름모가 있고, k번째 변(임의의 변)과 평행한 선분은 각각 n개씩 있다. 따라서 여기서 알 수 있는 전체 선분들의 개수는 n2개다(¤¤). (¤)과 (¤¤)로부터다음과 같은 결론을 얻을 수 있다.

n2 = n+ 36£ 37

) n = 37. }

81. n£ n 체스판이 주어져있다(n ¸ 3). 凸모양을 이루는 어느 네 칸의 색을 동시에 바꾸는(흰색을 검은색으로, 검은색은 흰색으로) 조작이 허용된다. 이런 조작을 유한번 반복하여 체스판의 모든 칸의 색을 원래의색과 다르게 바꾸는 것이 가능한 n을 모두 구하여라. (러시아 1990 4차-y9-3)

풀이 홀수일 땐 불가. 짝수일 땐 항상 가능. 4의 배수일 땐 너무 쉽고, 이웃한 두 칸 뒤집기가 항상 다

음의 3 combo로 되기 때문에.XXA

OBCC

OOC

위와 같은 그림일 때 [XXAB] + [ACCC] + [BCCC] 콤보로 해결. 사실 [ACCC] + [BCCC] 만으로도꽤 의미가 있는 아이템( 각 두 칸 뒤집기)이지만 코너에서 좀 귀찮아져서; }

82. 한 변의 길이가 12인 정사각형의 내부에 1990개의 점이 주어져있다. 그 중 498개 이상의 점을 내부에 포함하는, 한 변의 길이가 11인 정삼각형을 그릴 수 있음을 증명하여라. (러시아 1990 4차-y9-4)

증명 밑변에서 1+p3

2 만큼 밖으로 미끄러뜨린 정삼각형을 깔면 중심을 포함하는 한 4등분된 사각형

을 덮게 됨. ¤

Page 249: 실전수학올림피아드 1400제 해답

4.2 조합 고급문제 249

83. n장의 카드가 한 곳에 쌓여있는데, 일부는 위를 향하고 있고 일부는 아래를 향하고 있다. 위에서부터 몇장의 카드 한 묶음을 집어 그 묶음을 뒤집은 후 다시 카드 더미의 맨 위에 놓는 작업을 할 수 있다. k번 작업하면 초기 상태에 상관없이 항상 모든 카드가 아래를 향하게 할 수 있다면, k의 최소값은 얼마인가?

(러시아 1993 4차-y9-4)

풀이 맨밑의, 아래로 이미 잘 되어있는 카드들은 고려에서 제외하고, 그 이외의 카드들을 같은 방향으

로 연속하여 놓여있는 것들끼리 한 덩어리라고 부르자. 한 번 작업할 때마다 덩어리의 수는 최 1개 감소한다. 따라서, 구하는 k는 이웃한 카드가 항상 서로 다른 방향이고 맨밑 카드가 위를 향했을 때의 n번.IT에서 출제했던 ABCBABCB...ABCBA 문제랑 비슷한듯. }

84. 8£ 8 체스판에서 16개의 칸을 고르는데, 각 행마다 2칸씩, 각 열마다 2칸씩 되도록 택했다. 이 16개의 칸에 8개의 흰 졸(말)과 8개의 검은 졸을 놓는데, 체스판의 각 행과 열마다 1개씩 놓이도록 할 수 있음을 증명하여라. (헝가리 1933-2)

증명 같은 행, 같은 열의 칸마다 edge로 이으면 몇 개의 짝수길이 cycle이 됨. ¤

85. 좌표평면에 무한개의 직사각형을 모은 집합을 갖고 있는데, 이 직사각형들은 모두 적당한 자연수 m, n에해 (0; 0), (0;m), (n; 0), (n;m)을 네 꼭지점으로 갖는 꼴들이다. 이 집합의 직사각형들 중에 하나가 다

른 것에 포함되는 경우가 있음을 증명하여라. (헝가리 1934-3)

증명 (n;m) 직사각형을 하나 잡으면 이것을 포함하는 녀석이 없으려면 나머지 직사각형들은 모두

x-좌표가 n보다 작거나 y-좌표가 m보다 작다. 그럼 x-좌표나 y-좌표가 같은 두 녀석이 있다. ¤

86. 삼각 프리즘(삼각기둥)의 각 꼭지점마다 실수가 하나씩 배정되었는데, 각 꼭지점의 수는 변으로 이웃한세 수의 평균이 된다고 한다. 여섯 개의 수가 모두 같음을 증명하여라. (헝가리 1935-3)

증명 여섯 개의 수 중 가장 작은 수 x가 있다. x가 그에 이웃한 세 수(모두 x 이상)의 평균이 되려면

이웃한 수도 모두 x일 수밖에 없다(아니면 평균이 x보다 커지므로). 같은 논리를 이웃한 수로 옮겨가서계속 하다보면 여섯 개의 수가 모두 같을 수밖에. ¤

87. 평면 위에 어느 세 점도 한 직선 위에 있지 않은 n개의 점이 있다. 이 점들에 임의로 A1; A2; : : : ; An로 어떻게 번호를 붙여도, 꺾인 선 A1A2 ¢ ¢ ¢An은 자기 자신과 만나지 않는다고 한다. 이것이 가능한 n의 최값을 구하여라. (주니어발칸 2003-2)

풀이 볼록사각형을 이루는 네 점이 존재하면 안 됨. 점 5개면 항상 볼록사각형을 이루는 네 점이 존재

하므로 n = 4 가 최 . }

88. 평면 위의 격자점(정수 좌표의 점)들에 해 생각하자. 유리수 기울기를 갖는 직선들에 해 다음을 보여라:

(i) 이런 직선은 격자점을 전혀 지나지 않거나 무한히 많은 격자점을 지난다.

(ii) 각각의 직선에 해 다음을 만족하는 양수 d가 존재한다: 이 직선 위의 점들을 제외하고는 어떤 격자점도 이 직선에 이르는 거리가 d보다 가깝지 않다. (Putnam 1951-A5)

증명 (박경태) (i)

(1) 이 직선이 격자점을 지나지 않을 때 ! 지나지 않는다.

(2) 이 직선이 한 번이라도 격자점을 지날 때귀류법으로 이 직선이 p번만 격자점을 만난다고 가정하자. 기울기

qp에 해서 이 직선이 점 (a; b)를

지난다고 할 때(이 때의 점은 가장 오른쪽 상단에 있거나 오른쪽 하단에 있는 점이라 하자) (a +p; b+ q)도 이 직선을 지난다. 따라서 우리의 가정이 틀렸고 이 직선이 한 번이라도 격자점을 지나면 무한히 많은 격자점을 지난다.

(ii)

(1) 이 직선이 격자점을 지나는 경우 : 기울기qp일 때

가로 p, 세로 q 직사각형을 그려 1단위로 다 자른 뒤에 각선을 하나 그린다. 그리고 직선 주위의격자점에서 직선에 수선의 발을 내려 길이가 가장 짧은 것을 택한다. 그것이 d의 최 값이다.

Page 250: 실전수학올림피아드 1400제 해답

250 조합

(2) 이 직선이 격자점을 지나지 않는 경우 : 기울기qp일 때

가로 p, 세로 q 직사각형을 그려 1단위로 자르되, 이 직선이 x축과 만나는 점에서 왼쪽 격자점까지의 거리를 직사각형 그림에 덧붙이고 오른쪽에는 그 길이만큼 감하여 각선을 그린 뒤 (1)과 같이한다.

¤

89. 어느 시골의 한 초등학교에 스무 명의 아이들이 다닌다. 그 중 어떤 두 아이도 할아버지(외할아버지와 친할아버지) 중에 같은 분이 있다. 이 학교에 적어도 14명의 손자손녀가 있는 할아버지가 적어도 한 명 있음을 보여라. (Towns 1994가을 JO4)

증명 2명의 학생의 할아버지쌍이 (A;B), (A;C)라 하면 (A;D)꼴이 또 있으면 그 다음부터 모순이므

로 할아버지는 A, B, C의 3명뿐. 비둘기집. ¤

90. 체스판의 각 행, 열, 각선마다 짝수개씩의 졸이 있게 하려면 졸은 최 몇 개까지 놓을 수 있는가?(러시아 1993 최종-y9-7)

풀이 필요조건: 홀수 길이의 각선마다 하나씩은 빼야 함. 충분조건: 두 주 각선만 싹 비운 그림이

48개의 한 예가 됨. }

91. 거짓말쟁이와 참말쟁이들의 국제 회의에 32명이 참가하여, 각 줄에 8명씩 네 줄로 앉아있다. 쉬는 시간 동안 각각의 참가자들은 모두 자신과 인접한(앞뒤 혹은 좌우로) 자리에 거짓말쟁이도 있고 참말쟁이도 있다고 주장을 했다. 거짓말쟁이는 최소 몇 명인가? (러시아 1994 4차-y9-4)

풀이 BbCEEeGG

BCCcEF-g

a-CDfFFH

AAdDDFhH

위와 같이 A, B, C, D, E, F, G, H 8개 영역으로 분할하면 각 영역마다 거짓말쟁이가 적어도 한 명은 있어야 하고, 8명을 실제 배치하는 경우는 위의 소문자 위치에 배치하는 경우가 있음. }

92. 집합 M = f1; 2; : : : ; 2ng (n ¸ 2) 을 서로소인 k개의 집합 M1;M2; : : : ;Mk로 분할하였다. 단, k3+1 · n

이다. k + 1개의 짝수 2j1; 2j2; : : : ; 2jk+1 이 한 집합 Mj에 속하고, 2j1 ¡ 1; : : : ; 2jk+1 ¡ 1 도 한 집합

Mr에 속하는(1 · j; r · k) 경우가 있음을 증명하여라. (불가리아 1979 4차-6)

증명 비둘기집. k2 + 1개 이상의 짝수를 갖는 집합이 있고, 이 짝수들에서 1을 뺀 홀수들 중 k + 1개

이상을 갖는 집합이 또 있다. ¤

93. 1보다 큰 정수들로 이루어진 무한수열 fang의 항들이 모두 서로 다르다고 한다. ak > k 를 만족하는 k를100개 이상 찾을 수 있음을 보여라. (Towns 1982봄 J3)

증명 무한히 많이 찾을 수 있겠지. 저런 k개 유한개라고 가정하고 모순을 이끌어내면 됨. 저런 k가 모

두 N 이하라 하고, a1; : : : ; aN 의 최 값을 M이라 하면(M > N 임은 자명), a1; : : : ; aM 은 모두 M 이하의 수들이 되어 모순. ¤

별해 ak > ak¡1; : : : ; a1 인 k가 무한히 많음을 이용할 수도. ¤

94. 모든 항이 0 또는 1인 m£ n 크기의 행렬들의 집합을 생각하자. 이런 행렬들 중에서 각각의 행과 열에 속한 1의 개수가 모두 짝수인 행렬의 개수를 구하여라. (아일랜드 1994-4)

풀이 m¡ 1개의 행을 결정하면 나머지 한 행도 자동적으로 결정됨. 2(m¡1)(n¡1)개 }

95. a와 b 두 문자로만 이루어진 어떤 이상한 언어가 있다. 문자 a는 하나의 낱말이라고 한다. 또한, 다른 모든낱말들은 다음과 같은 규칙을 따라 만들어진다.

1. 임의의 낱말로부터 맨 오른쪽에 b를 추가하여 새로운 낱말을 만들 수 있다.

Page 251: 실전수학올림피아드 1400제 해답

4.2 조합 고급문제 251

2. aaa라는 문자열을 포함하는 임의의 낱말로부터 그 하나의 aaa를 b로 바꾸어 새로운 낱말을 만들 수있다.

3. bbb라는 문자열을 포함하는 임의의 낱말로부터 그 하나의 bbb를 제거하여 새로운 낱말을 만들 수 있다.

4. 임의의 낱말로부터 그 낱말을 두 번 이어써서 새로운 낱말을 만들 수 있다.

예를 들어, 4에 의해 aa는 낱말이고, 다시 4에 의해 aaaa는 낱말이다. 그리고, 2에 의해 ba는 낱말이고, 다시 1에 의해 bab 또한 낱말이다. 다시, 1에 의해 babb는 낱말이고, 4에 의해 babbbabb는 낱말이다. 마지막으로 3에 의해 baabb는 낱말이다.

baabaabaa는 낱말이 아님을 증명하여라. (IMTS R44-1)

증명 a의 개수가 3의 배수인 것은 나오지 않는다. ¤

96. 아래의 상황에서 한 변의 길이가 7인 정사각형 K를 한 변의 길이가 3인 정사각형 여덟 개로 덮는 것이 가능한가?

(a) 작은 정사각형의 모든 변이 항상 큰 정사각형의 변과 평행해야 한다.

(b) 작은 정사각형의 변이 큰 정사각형의 변과 꼭 평행할 필요는 없다. (유고슬라비아 1980 고2-3)

풀이 (a) 3.5 간격으로 3£ 3개의 점을 찍으면 한 정사각형이 두 점을 덮을 수 없음. (b) 4개는 코너에

덮고, 남은 4개는 45도 기울여서 남은 십자영역을 1/4씩 덮으면 됨. }

97. 11£ 11 수판에 1; 2; 3; : : : ; 121의 수를 일 일로 써넣는다. 연속하는 두 수는 항상 서로 이웃한 칸에 들어가게 하면서 모든 완전제곱수가 한 열에 있도록 할 수 있는가? (러시아 1995 4차-y9-8)

풀이 1부터 121까지 차례로 여행한다고 생각하자. 홀수는 검은칸, 짝수는 흰칸. 제곱수열에는 제곱수

말고 다른 수는 들어갈 수 없으므로 3 4 5, 8 9 10 등은 항상 행으로 연속하게 놓여야 하고, 그럼 제곱수열은 홀수번째열(제곱수에는 홀수가 하나 더 많으므로)이면서 양끝열일 수 없으니 제3, 5, 7, 9열 중의하나이다. 제곱수에서는 제곱수열을 항상 가로질러 가야하므로 2-3, 10-15, 26-35, 50-63, 82-99 등 50개의 수가 제곱수열로 나뉜 한쪽 영역에 있어야 한다. 그런데 50개는 11개의 배수가 아니므로 모순. }

98. P를 볼록n각형이라고 하자. P의 각각의 꼭지점으로부터 그 점에 이웃하지 않은 P의 n¡ 2개의 변(혹은그 연장선)에 수선을 내렸다. 이 수선들 중 적어도 하나는 P 안에 완벽하게 들어가 있거나 혹은 P의 한변임을 증명하여라. (IMTS R24-5)

증명 (신한솔) AB를 가장 긴 변이라 하고 A, B를 지나며 AB와 수직인 직선을 L1, L2라 하자. L1과

L2사이에 점이 존재하지 않는다고 하자. 그럼 직선 L1바깥 쪽(L1에 해 B와 반 쪽)의 점 X, L2바깥쪽의 점 Y 가 있고 XY 가 P의 한 변이 되는 점X, Y 가 존재하고 그럼 XY 가 AB보다 크므로 모순. 따라서 L1와 L2사이에(직선포함) 점K가 존재한다. 그럼 각KAB, KBA가 예각 또는 직각이므로 K에서AB에 내린 수선은 P내부에 있다.P.S: 볼록다각형이므로 삼각형 KAB는 P내부에 있음 ¤

99. 유리수들을 뿌리나무(시작점이 있는 수형도)꼴로 나열하는데, 각각의 유리수a

b마다(a와 b는 서로소) 두

개의 자손a

a+ b와

b

a+ b가 있도록 하려고 한다. 0 < r < 1 인 모든 유리수 r이 딱 한 번씩 나열되게 하려

면 뿌리(시작점)에는 어떤 유리수를 놓아야 하는가? 그리고, 주어진 유리수p

q가 뿌리로부터 몇 번째 단

계(층)에 있는지 알아내는 절차를 설명하여라. (독일BW 1976 1차-3)

풀이 a=(a + b) < 1=2, b=(a + b) > 1=2 임에 주목. 시작점에는 1=2. p=q > 1=2 이면 그 위에는

(q ¡ p)=p가, p=q < 1=2 이면 그 위에는 p=(q ¡ p) 가 부모. 이렇게 거슬러 올라가면서 1=2이 될 때까지의 횟수를 세면 됨. }

100. 회에 참가한 사람들 중 어느 네 명을 골라도 그 중 어느 한 명은 나머지 세 명을 모두 안다고 하자. 이회에 참가한 다른 모두를 아는 참가자가 있음을 보여라. 단, A가 B를 알면 B도 A를 아는 것으로 한다.

(몰도바 1997 최종-y9-7)

Page 252: 실전수학올림피아드 1400제 해답

252 조합

증명 우선 참가자수는 4명 이상임이 자명하고, 4명일 때는 성립함이 자명. 성립하지 않는 경우가 있다

고 하고, 그런 가장 적은 인원을 n명이라 하자(n ¸ 5). 이 n명 중 가장 많은 사람들을 아는 사람을 A라하자. A를 제외한 나머지 n ¡ 1명만 생각하면 그 안에서도 문제의 조건이 잘 만족되고 n ¡ 1은 성립하는 수이므로, 이 n ¡ 1명중 다른 모두를 아는 사람 B가 존재한다. 그리고, n은 성립하지 않는 인원이므로 AB가 서로 모르고, A의 최 성에서 A는 B가 아는 모두를 안다. ABCD를 택하면 A와 B는 나머지세 명을 모두 아는 사람이 아니므로 CD는 서로 알고, C, D는 임의였으므로 A와 B를 제외한 나머지 모두가 서로 안다. 그럼 나머지 모두가 다른 모두를 아는 참가자가 되어 끝. 귀납법 + 유한극값 ¤

101. 스스로와 교차하지 않는 길이가 1000보다 큰 꺾인 선이 주어진 단위정사각형 안에 있다. 이 단위정사각형의 변들 중 하나와 평행하면서 이 꺾인 선과 적어도 501개의 점에서 만나는 직선이 항상 존재함을 보여라. (독일BW 1971 2차-4)

증명 다음과 같이 AB가 있을 때 이것을 지우고 신 CD와 EF로 체한다고 생각하자. 그래도 변에

평행한 직선이 주어진 선분들과 만나는 점의 수는 변함이 없다.

또한 AB · CD + EF . 꺾인 선들은P

AB의 합이므로

1000 ·X

AB ·X

CD +X

EF

일반성을 잃지 않고, 500 <P

CD라 하면 CD가 속해 있는 변 중 501개의 선분이 자신 위에 있는 점이

존재한다. ¤

102. K는 자연수 M의 자릿수들을 재배열하여 얻은 자연수이다. 다음을 증명하여라.

(a) 2M의 자릿수의 합과 2K의 자릿수의 합은 같다.

(b) M과 K가 둘다 짝수이면, M=2의 자릿수의 합과 K=2의 자릿수의 합은 같다.

(c) 5M의 자릿수의 합과 5K의 자릿수의 합은 같다. (Towns 1983봄 JO4)

증명 (a) 각 자릿수를 2배 했을 때 십의 자릿수는 기껏해야 1, 일의 자릿수는 최 8이므로, 서로 다른

자리에 있던 수의 2배가 중첩되어 carry가 발생하지 않으니까, 그냥 각각의 자릿수를 그 자리에서 2배한것의 자릿수의 합을 각각 구한 후 더한 것이나 마찬가지라서. (b) 홀수인 자릿수는 그 아랫자리에 1을꿔준 다음 생각하면 (a)에서 생각한 것과 마찬가지. (c) (a)랑 비슷하게, 각각의 자릿수 별로 분해해서더해도 그만. 자릿수의 5배는 십의 자리가 최 4, 일의 자리가 최 5이므로 중첩되어 carry가 발생하는 일이 없음. ¤

103. 어떤 증가하는 정수 수열이 홀수항으로 시작하고 짝수항, 홀수항, 짝수항, 이렇게 짝수와 홀수가 교 되며 나타날 때, 이 수열을 교 적이라고 한다. 항을 하나도 갖지 않는 공수열도 교 적인 것으로 인정하기로 하자. 집합 f1; 2; : : : ; ng에 있는 수들만이 허용된 교 적 수열들의 개수를 A(n)이라 하자. A(1) = 2 이고 A(2) = 3 임을 보여라. A(20)의 값을 구하고, 그것이 옳음을 증명하여라. (영국 1994 1차-5)

증명 (이종민+김규완) 짝수부터 시작하는 연속된 n개의 자연수중에서 숫자들을 뽑아 수열을 만들 때

증가하는 수열이고 짝수항으로 시작하고 홀수항, 짝수항 이렇게 짝수와 홀수가 교 되게 만드는 수열을B(n)이라 하자(n은 0 이상).

집합f1; 2; 3; 4; 5g에서 `교 적'수열을 만들때 수열의 초항이 1일때, 3일때, 5일때로 나눌 수 있다. 수열의 초항이 1일때 뽑을수있는 나머지 남는 수는 f2; 3; 4; 5g인데 이들을 뽑는 경우의 수는 B(4)이다. 마찬가지 방법으로 수열의 초항이 3일때 뽑을수있는 수는 f4; 5g이고 이들을 뽑는 경우의수는 B(2). 초항이5일때는 B(0). 따라서 A(5) = B(4) +B(2) + B(0)으로 나타낼수 있다. 이를 일반화하면,

A(n) = B(n¡ 1) +B(n¡ 3) +B(n¡ 5) + ¢ ¢ ¢

Page 253: 실전수학올림피아드 1400제 해답

4.2 조합 고급문제 253

(단, B(0) = 1). 그런데, 집합f1; 2; : : : ; ng의 교 적 수열에 +1한 수열은 집합f2; 3; : : : ; n + 1g에서짝수, 홀수 순서 로 뽑은 것과 일 일 응이므로 A(n) = B(n)이 성립한다. 따라서 A(n) = A(n ¡1) + A(n ¡ 3) + ¢ ¢ ¢ (단, A(0) = 1이고 괄호안의 숫자가 음수일때는 생각하지 않는다). A(n ¡ 1) =A(n¡2)+A(n¡4)+¢ ¢ ¢이므로 입하면 A(n) = A(n¡2)+A(n¡3)+A(n¡4)+¢ ¢ ¢+A(2)+A(1)+A(0).이로부터 A(n) = A(n¡ 1) + A(n¡ 2)를 얻는다(피보나치 수열). 2 3 5 8 13 21 34 55 89 144 233 377610 987 1597 2584 4181 6765 10946 17711 = A(20). ¤

104. 평면 위의 3개 이상의 점으로 이루어진, 다음을 만족하는 집합이 존재하는가?

(i) 이 집합의 어떤 세 점도 한 직선 위에 있지 않다;

(ii) 이 집합의 임의의 세 점에 해, 그 세 점을 지나는 원의 중심이 다시 이 집합에 속한다.(브라질 1996-2)

풀이1 (강성경) 이 점들의 볼록포 K중의 한 변 AB를 택하자. 그러면 직선 AB를 기준으로 한 쪽 평

면에만 나머지 점들이 존재하게 된다.i) 만약 A, B외의 점C가 있어서 \ACB가 둔각이라면 문제는 자명해진다. (증명 생략) [외심이 직선AB의 바깥쪽에 있어야 하므로, 한쪽 평면에만 점들이 있다는 것에 모순] 만약 \ACB가 직각이면 그 외심 O와 A, B는 한 직선 위에 있데 되어 모순. 만약 모든 A, B가 아닌 C에 해 \ACB가 예각이라고하면,

ii) 만약 4ABC가 둔각 삼각형이면, WLOG \CAB > 90. 이때 4ABC의 외심을 O라 하면, 삼각형4AOB에서 \ABO, \BAO < 90인 것은 당연하다. 따라서 \AOB가 둔각이면 i)의 경우, 예각이면iii)의 경우로 넘어가게 되어, 이 경우는 해결할 필요가 없어지게 된다.iii) 4ABC가 예각 삼각형이면, 4ABC의 외심 O1 역시 4ABC 안에 있다. 그리고 \ACB = µ면\AO1B = 2µ다. 이런 작업을 n번 하여 \AOnB가 최초로 둔각이 되게 하자. 이 경우는 i)번의 경우와같으므로 해결할 필요가 없다. }

풀이2 (정명준) 이런 집합 S가 존재한다고 하자. 처음, Sfa1; a2; a3g에는 세 점이 속하고, 이 세 점은

삼각형을 이룬다. 이제, 네 번째 원소 `a4'는 삼각형의 외심이므로, a1a2의 수직 이등분선 위에 있다. 또,

세 점 a1; a2; a4이 이루는 삼각형의 외심`a5'은 역시 a1a2의 수직 이등분선 위에 있다.

이제 세 점 a1; a2; a5이 이루는 삼각형의 외심`a6'도 a1a2의 수직 이등분선 위에 있으므로, S의 세 원소a4; a5; a6은 한 직선 위에 있다. 그럼 (i)에 의해 이 세 점이 모두 다른 점일 수 없다. 외심은 원래 삼각형의 꼭지점일 수 없으므로, a4 = a6만이 유일하게 가능하다.

각 a1a4a2 = x라 할 때, 약간의 중심각-원주각 계산을 통해 360도¡ 2(360도¡ 2x) = x의 식을 얻을 수있고, 따라서 x = 120도. a2a3, a3a1도 마찬가지로 a1a2처럼 한 점을 더 선택한 후 외심이 수직이등분선 위에 있음을 이용할 수 있으므로, 우선 S는 \a1a4a2 = \a2a4a3 = \a3a4a1 = 120±인 경우만 빼고는 존재할 수 없다(즉, a4와 a6들이 일치해서 일직선 위의 세 점 a4; a5; a6이 나오지 못하는 경우임).

그러면, 4a1a2a3이 정삼각형인 경우만 따로 보자. 우선 위와 같이 a5까지를 정의하자. 그럼 a3; a4; a5가한 직선 위의 서로 다른 세 점이 되어 모순. 즉 처음 세 점이 정삼각형을 이룰 때도 S가 존재하지 않는다. 즉 이런 집합 S는 존재할 수 없다. }

105. 길을 따라 (n¡ 1)2개의 단위 블록으로 구획된, 한 변의 길이가 n¡ 1인 정사각형꼴의 도시가 있다. 왕복버스 노선을 정하려고 하는데, 각 노선은 최 한 번 방향을 바꿀 수 있고(우회전 또는 좌회전), 임의의 두교차로 사이를 많아도 한 번의 환승이면 갈 수 있도록 하려고 한다. 최소 몇 개의 노선이 필요한가?

(러시아 1992 4차-y9-4)

풀이 우하 꼭지점 위치를 A라 하자. A와 i번째 열을 포함하는 노선을 ri로 정하자. 그럼 이 n개의 노

선으로 우선 충분함(항상 A에서 갈아타면 됨). 만일 노선이 n개보다 적다면, 어떤 노선도 세로방향으로는 지나지 않는 열이 적어도 하나 있고, 그럼 그 열 안의 n개의 교차점은 각 노선이 가로방향으로 한 번씩 지나가야 하므로 적어도 n개의 노선이 필요함. }

Page 254: 실전수학올림피아드 1400제 해답

254 조합

106. 정12각형의 모든 변과 각선을, 주어진 12가지 색깔 중에서 하나씩 골라 칠한다. 모든 세 가지 색깔에해서, 적당한 세 꼭지점이 존재해서, 그 세 꼭지점으로 이루어지는 삼각형이 그 세 가지 색깔의 변을 모두갖도록 할 수 있는가? (러시아 1995 4차-y9-4)

풀이 12개의 꼭지점에서 세 점으로 된 삼각형의 개수¡123

¢과 12가지 색깔에서 세 변을 이룰 세 가지 색

을 고르는 방법의 수¡123

¢이 똑같으므로, 각각의 서로 다른 3색 구성의 삼각형이 정확히 하나씩 존재함.

변(및 각선)의 수는¡122

¢= 66 > 5 ¢ 12 이므로 같은 색의 6개의 변이 존재하고, 같은 색의 두 변을 갖

는 삼각형이 없으므로 이 6개의 변은 어떤 둘도 한 점에서 만나는 법이 없다. 즉, 이 변을 한 변으로 하는삼각형은 각각 10개씩 총 60개 이상이 되는데, 3색 구성에서 그 색의 변을 갖는 삼각형은

¡112

¢= 55개이

므로 모순. 비둘기집. }

107. 어떤 무용 수업에서 15명의 남학생과 15명의 여학생이 각각 일렬로 줄을 서서 15쌍이 짝지어지도록 하였다. 각각의 짝에서 남학생과 여학생의 키는 기껏해야 10 cm밖에 차이가 나지 않았다고 한다. 남학생과 여학생을 각각 다시 큰 키 순서로 다시 줄을 세웠을 때, 새로운 짝들도 남학생과 여학생의 키 차이가 항상10 cm 이하가 됨을 증명하여라. (Towns 1984봄 JA2)

증명 a1 < a2 < ¢ ¢ ¢ < an, b1 < b2 < ¢ ¢ ¢ < bn 으로 줄을 세웠다고 하자. WLOG, bi ¡ ai > 10 이라

면 a1; : : : ; ai; bi; : : : ; bn 의 n + 1명을 생각하면 이 중에 짝이었던 둘 aj , bk가 반드시 있고 bk ¡ aj ¸bi ¡ ai > 10 이므로 모순. [주] 2열 횡 로 섰는데 항상 앞사람이 키가 컸을 때 각 행을 키순으로 배열하고 나서도 항상 앞사람이 키가 크다는 유명한 문제의 응용변형임. ¤

108. 1부터 9까지의 숫자를 꼭 한 번씩 써서 만들어지는 모든 아홉 자리의 수들을 생각하자. 이런 두 수의 합이987654321이 될 때 이 두 수를 `조건적인 쌍'이라고 부른다.

(a) 조건적인 쌍이 적어도 둘 있음을 보여라.

(b) 조건적인 쌍은 홀수개임을 보여라. (Towns 1983가을 S2)

증명 일의 자리의 합과 십의 자리의 합이 각각 11이므로, 일의 자리와 십의 자리를 둘다 교환해도 역

시 조건적인 쌍. ¤

109. (a) 20£ 20 크기의 종이의 각각의 격자칸마다 말이 하나씩 놓여있다. 바냐가 수 d를 고르고, 페탸는 각각의 말을 동시에 거리(칸과 칸 사이의 중심거리) d만큼의 위치로 이동시킨다. 바냐가 수 d를 고르고, 페탸는 각각의 말을 동시에 거리(칸과 칸 사이의 중심거리)가 d 이상 떨어진 위치로 이동시킨다. (최 의 d를 구하고, 그 d 이하에서는 모두 가능함을 증명하고, 그 d를 넘는 값에서는 모두 불가능함을 보여라.)

(b) 21£ 21 크기에서 위의 문제를 다시 풀어라. (Towns 1984봄 SO5)

증명 (a)중앙의 네 칸 중 한 칸을 생각하면 그 녀석은 최 거리 10p2까지만 움직일 수 있고, (제1,제3),

(제2,제4) 사분면끼리 평행이동하는 것을 생각하면 10p2가 실제로 가능하다. (b) 역시 중앙의 칸을 생

각하면 10p2까지가 한계. 이 거리의 이동을 실제로 구현할 수 있다. 20£ 20 은 (a)처럼 옮기고 나머지

L자 영역만 잘 옮기면 됨. ¤

110. 주어진 2 이상의 자연수 n에 해, 집합 A = f1; 2; : : : ; ng 를 생각하자. M은 A의 부분집합이고, M의 어떤 두 원소의 합도 42의 배수가 아니다. 이런 M이 가질 수 있는 원소의 최 개수를 S(n)으로 나타내다.S(n) = 1997 이면 n은 얼마인가? (몰도바 1997 최종-y11-4)

풀이 42로 나눈 나머지에 따라 분류한 그룹을 둘씩 짝짓기. 0과 21의 집합은 스스로와 짝이 되므로 별

도. 1997 = 20m+ 1 + 1 + a 에서 m = 99, a = 15. ¢ ¢ ¢ 답 4173 }

111. (a) 원주를 등간격의 호로 나누는 10개의 점이 있다. 이들을 둘씩 연결하여 5개의 현을 얻었다. 이 현의길이가 모두 다를 수 있을까?

(b) 원주를 등간격의 호로 나누는 20개의 점이 있다. 이들을 둘씩 연결하여 얻은 10개의 현 중에는 같은 길이의 두 현이 반드시 존재함을 증명하여라. (Towns 1982가을 J4)

증명 (a) 모두 다를 수 있다. (b) 두 가지 색으로 색칠. ¤

Page 255: 실전수학올림피아드 1400제 해답

4.2 조합 고급문제 255

112. 5£ 9(5행 9열) 크기의 직사각 체스판에서 다음과 같은 게임을 한다. 처음에 몇 개의 돌이 이 체스판의 임의의 칸들에 놓여있었다. 단, 한 칸에는 돌이 하나만 놓일 수 있다. 완전한 움직임이란 이 체스판의 모든돌을 다음과 같은 규칙에 따라 동시에 움직이는 것을 말한다.

(i) 각각의 돌은 위, 아래, 왼쪽, 오른쪽 중의 한 방향으로 한 칸을 이동할 수 있다.

(ii) 위나 아래로 움직였던 돌은 다음 번 완전한 움직임 때는 왼쪽이나 오른쪽으로 움직여야 한다.

(iii) 왼쪽이나 오른쪽으로 움직였던 돌은 다음 번 완전한 움직임 때는 위나 아래로 움직여야 한다.

(iv) 완전히 움직인 후에도 각 칸에는 두 개 이상의 돌이 있을 수 없다.

더 이상 완전한 움직임을 행할 수 없으면 이 게임은 끝난다. 처음에 이 체스판 위에 33개의 돌이 있었다면이 게임은 유한번 안에 끝남을 증명하여라. 또한, 이 게임이 영원히 계속 되도록 처음에 32개의 돌을 잘배치하고 게임을 진행하는 것이 가능함을 증명하여라. (아일랜드 1996-10)

증명 32개가 가능하다는 것은 4개씩 cycle로 묶어 4 £ 8 에 배치하면 끝. 33개 이상일 때 불가능하다

는 것: 체스판을 다음과 같이 색칠하면 모든 말은 ABCDABCD..나 ADCBADCB..순으로 움직이게 됨.ABABABABA A가 15개, B가 12개, C가 8개, D가 10개.

DCDCDCDCD A의 위치에 말이 9개 이상 있으면 두 번 진행했을 때 모두 C의 위치에 가게 되므로 곤란.

ABABABABA B와 D를 합해 17개 이상 있으면 다음에 A와 C를 합해 17개 이상 있게 되므로 역시 곤란.

DCDCDCDCD 따라서, $A + C + (B+D) \leq 8 + 8 + 16 = 32$.

ABABABABA

¤

113. N은 1415개의 점들로 이루어지고 둘레가 2001인 볼록다각형이다. N의 꼭지점들 중에서 넓이가 1보다 작은 삼각형을 이루는 세 점을 찾을 수 있음을 증명하여라. (주니어발칸 2001-4)

증명 귀류법으로 모든 삼각형의 넓이가 1 이상이라 하자. 임의의 한 변을 a라 하고, 거기서부터 각

변을 차례로 a1; b1; a2; b2; : : : ; a707; b707 이라 하자. ai, bi를 두 변으로 갖는 삼각형의 넓이는p2 ·p

2Si ·paibi · ai+bi

2. 이 식에 2를 곱한 후 모든 i에 해 합하고 또 a도 합하면 우변이 둘레의

길이가 되므로 a + 1414p2 · 2001, 즉 a · 2001 ¡ 1414p2. a는 임의였으므로 다시 둘레의 길이는

2001 · 1415(2001¡ 1414p2 ), 즉 1415p2 · 2001 인데 이것은 성립하지 않음. [주] 처음부터 a를 최

인 것으로 택하고 시작하면 귀류법 아닌 비둘기집 원리 등으로 쓸 수도 있음. [주] 1415는p2의 소수전

개와 관련이 있음. ¤

114. 정수 n > 2 이 주어져 있을 때, 다음 성질을 만족하는 서로 다른 n개의 수 a1; : : : ; an의 예를 찾아라: 임의의 두 원소의 합 ai + aj (i 6= j) 를 모두 모은 집합의 (서로 다른) 원소의 개수가 최소이다. 한편, 이 집

합의 원소의 개수가 최 인 경우도 예를 찾아라. (폴란드 1968 3차-3)

풀이 (1) a1 < a2 < ¢ ¢ ¢ < an 이라 할 때, a1 + a2 < a1 + a3 < ¢ ¢ ¢ < a1 + an < a2 + an < ¢ ¢ ¢ <an¡1 + an 의 2n¡ 3쌍이 모두 서로 다르고 실제로 등차수열일 때 이렇게 딱 2n¡ 3개의 합이 생김. (2)ak = 2

k 등으로 하면 모든 합이 다 서로 다름. }

115. 평면 위에 n > 3 개의 점이 주어져 있고, 어느 세 점도 한 직선 위에 있지 않다. n보다 작은 자연수 k에해 다음을 증명하여라.

(a) k · n2 일 때, 각각의 점은 어느 세 선분도 삼각형을 이루지 않도록 적어도 k개의 다른 점들과 선분

으로 연결될 수 있다.

(b) k > n2 이고 각각의 점은 적어도 k개의 다른 점들과 선분으로 연결되어 있을 때, 삼각형을 이루는

세 선분이 반드시 존재한다. (폴란드 1968 3차-6)

증명 (a) 완전이분그래프 (b) 귀류법으로 삼각형이 없다고 하자. A가 A1; A2; : : : ; Ak와 연결되어 있

을 때, 삼각형이 없으므로 A1은 A2; : : : ; Ak와 모두 연결되지 않는다. 그럼 A1과 연결될 수 있는 것은 최(n¡ 1)¡ (k ¡ 1) = n¡ k < k개이므로 모순. ¤

116. S는 집합 f1; 2; 3; : : : ; 1000g의 부분집합으로, S의 임의의 서로 다른 두 원소 m, n에 해 m+ n 은 항상S에 없다고 한다. S는 최 몇 개의 원소를 갖는가? (중미 1999-6)

Page 256: 실전수학올림피아드 1400제 해답

256 조합

풀이 일단 최 의 경우는 S = f500; 501; : : : ; 1000g 일 것으로 쉽게 추측됨. 즉 501개. 증명이 관

건. 귀류법으로 502개 이상의 S가 존재한다고 가정. 그럼 S에는 500 미만의 수들이 존재. 그 수들을a1 < a2 < ¢ ¢ ¢ < ak 라 하자. S에는 500이상의 수도 틀림없이 존재하고, 그중 가장 큰 수를 M이라 하자.

우선 M+1; : : : ; 1000의 1000¡M개의 수는 S에 없음(1). M¡a1 > M¡a2 > ¢ ¢ ¢ > M¡ak¡(1000¡M)은 모두 S에 있을 수 없는 500 이상 M 미만의 서로 다른 수들. 이게 k ¡ (1000¡M)개이므로 (1)과 함께 500 이상의 수가 적어도 k개는 S에 있을 수 없음. 그럼 jSj · 501 이 되어 모순. }

117. 다음을 만족하는 최 의 정수 A를 구하여라: 100 이하의 모든 자연수들의 임의의 재배열에 하여, 합이A 이상이 되는 연속한 10개의 수가 항상 존재한다. (폴란드 1971 3차-5)

풀이 한 재배열 ¼ 안에 나타나는 연속한 10항의 합의 최 값을 M(¼)라 쓸 때, A는 M(¼)의 최소

값을 말한다. 수열 ¼ = 100; 1; 99; 2; 98; 3; : : : ; 51; 50 에서는 연속한 10항의 합은 505 또는 500이므로M(¼) = 505. 즉 A · 505. 100항을 10항씩 10부분으로 분할할 때 100항의 총합은 5050 · 10M(¼) 이

므로 항상 M(¼) ¸ 505. ¢ ¢ ¢ 답 505 }

118. 두 기사가 바둑을 두는데 제한시간 시계를 이용하고 있다. 각자 40수씩 끝냈을 때 각자 소요한 시간이 2시간 30분씩으로 똑같았다. 두 사람이 사용한 시간의 차이가 정확히 1분 51초였던 순간이 있었음을 보여라.

그리고, 정확히 2분의 차이였던 순간은 반드시 있었을까? (Towns 1985가을 J4)

풀이 그런 순간이 없었다고 하면, a1 < 111, b1 < a1+111 < 2 ¢111, a1+a2 < b1+b2+111 < 3 ¢111,... b1 + ¢ ¢ ¢+ b40 < a1 + ¢ ¢ ¢+ a40 + 111 < 80 ¢ 111 = 8880 < 9000 = 2시간30분 으로 모순. }

119. 어떤 마을의 도로는 세 방향으로 이루어져 있는데, 이 마을을 같은 크기의 정삼각형 블록들로 분할하고있다. 교차로에 진입한 차량은 직진하거나 120±만큼 좌로 혹은 우로 회전할 수 있다.

교차로가 아닌 곳에서 방향을 바꿀 수는 없다. 한 교차로를 막 지난 지점에서 차 한 가 출발하였고, 이차가 다음 교차로에 들어서는 순간 두 번째 차가 같은 출발점에서 그를 향해 출발하였다. 두 모두 같은속력으로 계속 등속운동을 한다. 이 두 가 언젠가 서로 만날 수 있는가? (Towns 1986봄 J3)

풀이 A B C A B C

C A B C A B

B C A B C A

위와 같은 육방격자 색칠을 생각하면 항상 A ! B ! C 로 옮겨가게 되므로. 변도 AB ! BC ! CA

위를 움직이고. }

120. (\시지푸스 노동자") 언덕을 오르는 1001개의 계단이 있다. 그 중 몇몇 계단에는 바위가 있고, 한 계단에는 바위가 최 1개 있을 수 있다. 시지푸스는 바위 하나를 골라 그 윗쪽의 가장 가까운 빈 계단으로 옮길수 있다(즉, 바위가 연이어 있으면 한꺼번에 여러 단을 올릴 수도 있다). 그럼 훼방꾼은 바로 아랫단이 비어있는 바위 하나를 골라 한 계단 밀어내린다. 시지푸스와 훼방꾼은 이렇게 번갈아 움직인다. 처음에 가장 낮은 500개의 계단에 500개의 바위가 있었다. 시지푸스의 목표는 정상에 바위 하나를 올리는 것이다.

훼방꾼이 시지푸스가 목표를 이루지 못하도록 할 수 있을까? (Towns 1986봄 J6)

풀이 훼방할 수 있다. 연속한 바위들의 묶음과 묶음 사이에 빈 계단이 둘 이상 연속하여 있을 수 없도

록(또한, 맨밑 계단은 항상 비어있지 않도록) 만들 수 있음을 보이면 충분. }

121. n £ n 체스판의 한 각선의 칸을 1로 채우고 나머지 칸을 모두 0으로 채웠다. 스스로 교차하지 않는 임의의 닫힌 `룩-회로'(체스판의 모서리에 평행한 선분들로 이루어진 닫힌 경로)의 모든 칸의 수들을 1씩 증가시킬 수 있다. 이런 시행으로 모든 칸의 수가 똑같아지도록 할 수 있는가? (Towns 1992가을 JA1)

Page 257: 실전수학올림피아드 1400제 해답

4.2 조합 고급문제 257

풀이 칸에 흑백을 번갈아 색칠하면 흑칸과 백칸의 수가 똑같이 늘어나는데, 처음에 1은 모두 흑칸에

만 있어서 흑칸의 수의 합이 항상 n만큼 큼. 색칠. }

122. a1; a2; : : : ; a100은 1901부터 2000까지의 정수들을 재배열한 것이다. 이로부터 다음과 같이 부분합 수열을 만든다.

s1 = a1; s2 = a1 + a2; s3 = a1 + a2 + a3; : : : ; s100 = a1 + a2 + ¢ ¢ ¢+ a100

s1; : : : ; s100 중에 3으로 나누어지는 것이 없도록 하는 수열 a1; : : : ; a100은 모두 몇 개인가?(캐나다 2000-2)

풀이 정수 수열 a1; a2; : : : ; ak 의 부분합 중에 3의 배수인 것이 없으면, 이 수열의 맨앞을 제외한 사이

사이에 3의 배수인 항을 추가하거나 혹은 3의 배수인 항을 제거해서 만든 수열 a01; a02; : : : ; a

0k 도 부분합

중에 3의 배수인 것이 없다. 따라서, 1901; 1902; : : : ; 2000 중에서 3의 배수인 것은 제외하고 만든 후, 나중에 3의 배수들을 끼워넣기로 하자. 이 100개의 수 중에서 mod 3으로 0, 1, 2인 것은 각각 33, 33, 34개있다. 수열에 3의 배수가 없다면, 부분합 si가 mod 3으로 1, 2일 때 각각 다음 항은 mod 3으로 1, 2인것만 올 수 있다. 즉, mod 3으로 가능한 수열은

1; 1; 2; 1; 2; 1; 2; 1; : : : 과 2; 2; 1; 2; 1; 2; 1; 2; : : :

두 가지뿐인데, 2가 하나 더 많으므로 2; 2; 1; 2; 1; 2; : : : ; 1꼴이 유일하다. 이제 mod 3으로 1인 것과 2인것의 위치는 결정되었으므로 여기서 mod 3으로 같은 것끼리 순서를 바꾸는 경우의 수 33! 34!만큼 나오고, 다음 이 67개의 수에서 맨앞을 제외한 사이사이에 33개의 3의 배수들을 작은 것부터 차례로 끼워넣는 경우의 수는 67 ¢ 68 ¢ 69 ¢ ¢ ¢ 99가 된다. 따라서, 구하는 순열의 개수는 33! 34! 99!

66!¢ ¢ ¢ 답 }

123. n ¸ 3 인 양의 정수 n에 하여 Sn = f1; 2; : : : ; ng 이라 하자. 원소의 개수가 모두 짝수인 Sn의 부분집합 A1; A2; : : : ; An 이 주어져 있다. 그러면

Ai1 4Ai2 4 ¢ ¢ ¢ 4 Ait = ?

를 만족시키는 공집합이 아닌 Sn의 부분집합 fi1; i2; : : : ; itg가 존재함을 보여라. (단, 집합 A, B에 하여 A4B = (A [B)¡ (A \B) 이다.) (한국 2004-J2)

증명 임의의 I = fi1; i2; : : : ; itg 에 해

¤I =

t4k=1

Aik = Ai1 4Ai2 4 ¢ ¢ ¢ 4 Ait

와 같이 쓰기로 하자. 우선 집합 연산 4에 한 다음의 성질을 확인해두자(증명은 뒤에).

(1) 4은 교환법칙과 결합법칙이 성립한다(따라서 위와 같이 쓸 수 있다).

(2) A4B = ? () A = B.

(3) ¤I는 fAigi2I 들중 홀수 개에 포함되는 원소들만을 모은 집합이다.

(4) ¤J 4¤K = ¤J4K .

I( 6= ?) ½ Sn 인 모든 첨자집합 I들의 모임을 G라 하자. 이 때 문제는

¤I = ? 인 I 2 G 가 존재함을 보여라.

라는 것이다. 주어진 I 2 G 에 해, k 2 Ai 인 i 2 I의 개수를 ak(I)로 나타내기로 하자. Ai들은 모두짝수 개의 원소를 가지므로

a1(I) + a2(I) + ¢ ¢ ¢+ an(I) =Xi2I

jAij = 짝수

가 된다. 따라서, ak(I) 중 홀수인 것은 짝수 개이고, (3)에 의해 그런 k들만 ¤I의 원소이므로, ¤I는 항상짝수 개의 원소를 갖는다. Sn의 부분집합 중에서 짝수 개의 원소를 갖는 것은 2n¡1개 있다(1; 2; : : : ; n¡1의 포함여부를 알면 n의 포함여부가 저절로 결정되기 때문에). 따라서, ¤I의 종류는 많아야 2n¡1개있다. 그런데, G는 Sn의 부분집합 중 공집합을 제외한 것들이므로 2n ¡ 1개 있고, 이것은 n ¸ 3 일 때2n¡1보다 크므로, 비둘기집의 원리에 의해

¤J = ¤K

Page 258: 실전수학올림피아드 1400제 해답

258 조합

인 서로 다른 두 J;K 2 G 가 있다. 즉, I = J 4K 라 하면 (4)와 (2)에 의해

¤I = ¤J 4¤K = ?

이 된다. 그리고 J 6= K 이므로 (2)에 의해 I 6= ?, 즉 I 2 G 이므로 원하던 I를 찾았다.

위의 4의 성질들은 아래와 같이 증명된다.

(1) 교환법칙:A4B = (A [B)¡ (A \B) = (B [ A)¡ (B \ A) = B 4A

결합법칙: 교집합 기호는 생략하고 여집합 Xc는 ¹X로 신 쓰겠다. 벤 다이어그램을 그려보면(A4B)4 C는 다음과 같음을 확인할 수 있다.

(A4B)4 C = A ¹B ¹C [ ¹AB ¹C [ ¹A ¹BC [ABC

이 결과는 칭적인 식이므로, A, B, C의 순서를 적당히 뒤바꾼 (B 4 C)4 A 도 똑같은 집합이되고, 교환법칙이 성립함을 이용하면

(A4B)4 C = (B 4 C)4A = A4 (B 4 C)

임을 알 수 있다.

(2) 벤 다이어그램 등을 이용하면A4B = (A¡B) [ (B ¡A)

이 됨을 금방 알 수 있다. 이것을 이용하자.

A4B = ? () (A¡ B) [ (B ¡ A) = ?() A¡ B = B ¡A = ?() A ½ B ½ A

() A = B

(3) jIj에 한 수학적 귀납법으로 증명하자:jIj = 1 일 때는 ¤I = Ai 이므로 자명.jIj = m 일 때 성립한다고 가정하자.jIj = m+ 1 일 때, I에서 한 원소 x를 골라 I 0 = I ¡ fxg 라 하자.fAigi2I 중 홀수 개에 포함되는 원소는

(i) Ax에 포함되고 fAigi2I0 중 짝수 개에 포함되거나

(ii) Ax에 포함되지 않고 fAigi2I0 중 홀수 개에 포함된다.

즉, 귀납법의 가정에 의해,

(i) Ax에 포함되고 ¤I0에 포함되지 않거나

(ii) Ax에 포함되지 않고 ¤I0에 포함된다.

따라서, 이런 원소는 ¤I = Ax 4¤I0 에 포함된다.

비슷하게, fAigi2I 중 짝수 개에 포함되는 원소는 Ax와 ¤I0에 동시에 포함되거나 동시에 포함되지 않아서, ¤I에 포함되지 않는다는 것도 확인할 수 있다. 즉, jIj = m+ 1 일 때도 성립.

따라서, 수학적 귀납법에 의해 증명이 되었다.

(4) J 4K 에서는 J와 K에 동시에 나타나는 첨자들만 (2)에 의해 소거되고 J와 K의 어느 한 쪽에 포함되는 것들은 남는다. 즉,

am(J 4K) = am(J) + am(K)¡ 2am(J \K)

´ am(J) + am(K) (mod 2)

이다. 즉 임의의 m 2 Sn 에 해, Aj (j 2 J) 들과 Ak (k 2 K) 들중 m을 포함하는 것들을 모두 센

개수가 Ai (i 2 J 4K) 들중 m을 포함하는 것들의 개수와 홀짝이 같다. 따라서, (3)에 의해 성립.

¤

Page 259: 실전수학올림피아드 1400제 해답

4.2 조합 고급문제 259

124. 한 원 위의 10000개의 점에 시계방향으로 1부터 10000까지의 번호를 붙였다. 이 점들을 둘씩 짝지어5000개의 선분으로 이었는데, 각각의 선분은 다른 딱 하나의 선분과 만난다. 5000개의 선분 각각에 그 양끝점의 번호의 곱을 적었다. 선분에 적힌 수를 모두 합하면 4의 배수가 됨을 보여라. (중미 2001-6)

증명 연결될만한 현 PQ를 잡고, P에서 Q까지 시계방향으로 도는 호를 ®, 반 방향으로 도는 호를

¯라 하자. PQ를 가로지르는 선분은 딱 하나이므로 ® 안에서(¯ 안에서도) 딱 한 점을 제외하면 나머지점들은 PQ® 영역 안에서 네 점씩 묶인다. 따라서, ®에 있는 점의 개수는 4k + 1꼴. 즉, P의 수를 n이라 하면 Q의 수는 n+ 4k + 2에 해당하고, n의 홀짝에 따라 n(n+ 4k+ 2) ´ ¡1 또는 0 (mod 4). 그럼P

n(n+ 4k + 2) ´ ¡1 ¢ 2500 ´ 0 (mod 4). ¤

125. 수 1; 2; : : : ; n 을 한 원의 둘레에 배열하는데, 각각의 수는 (시계방향으로) 그 다음 두 수의 합을 나누도록한다. 가능한 n > 2 은 어떤 수들인가? (중미 2002-1)

풀이 홀짝성. 짝수 다음 또 짝수이면 모두 짝수가 되어 모순. 그럼 짝홀홀짝홀홀... 이라서 역시 n ¸ 5이면 모순. }

126. 집합 f1; 2; : : : ; 1985g의 부분집합 중에서 다음을 만족하는 가장 큰 집합을 구하여라: 이 부분집합의 어떤두 수의 차도 소수가 아니다. (1은 소수가 아님에 주의할 것!) (Towns 1985봄 SO2)

풀이 연속한 두 수를 뽑는 경우가 있으면, 그 두 수의 앞뒤로 적어도 7개씩의 수는 부분집합에 속할 수

없음. 혹은 연속한 8개의 수 중에서 많아야 2개만 뽑을 수 있음. 따라서, 4k + 1꼴들만 모두 뽑은 경우가 가능하므로 이것이 최선. }

127. 64개의 단위정육면체 블록들이 8£ 8 정사각형을 이루도록 쌓여있다. 이들을 4£ 4 £ 4 크기의 정육면체꼴로 다시 쌓는데, 면으로 이웃했던 두 블록은 항상 다시 면으로 이웃해있도록 할 수 있는가?

(레닌그라드 1987-24)

풀이 4 £ 4 £ 4 정육면체의 꼭지점에 놓이는 8개의 블록 중 적어도 하나는 8£ 8 정사각형의 변에 있

었음. 그 주변을 차근차근 탐문해가면 4£ 4£ 4 의 한 모서리의 길이가 너무 짧음을 알게 됨. }

128. 자연수들로 이루어진 집합 fr1; r2; : : : ; rkg의 원소들은 자연수 m으로 나누었을 때 나머지가 모두 서로 다르다. k > m

2 이라면, ri + rj 가 m의 배수가 되는 경우가 존재함을 증명하여라. 단, i, j가 서로 다를 필

요는 없다. (폴란드 1979 3차-1)

증명 r1; : : : ; rk들 중에는 m으로 나눈 나머지가 0인 것이 있을 수 없고, f1;¡1g, f2;¡2g, ..., fbm2 c;¡bm2 cg의 각 집합에서 하나씩만 뽑을 수 있음. 그럼 k · bm

2· m

2 인데, 이것은 모순. 간단한 비둘기집. ¤

129. 한 천문학자가 어떤 50개의 별을 관찰하여 각 별들 사이의 거리를 모두 합하였더니 S가 되었다. 갑자기구름이 나타나 25개의 별을 가렸다. 남은 25개의 별들 사이의 거리를 모두 합한 값은 S=2보다 작음을 증명하여라. (레닌그라드 1987-30)

증명 가려진 별들을 A1; A2; : : : , 남은 별들을 B1; B2; : : : 라 하면, BiBj · BiAk +AkBj 의 삼각부

등식을 모두 합했을 때 25P

BiBj · 24P

BiAk < 24(P

BiAk +P

AiAj). 즉, 25T < 24(S ¡ T ). 정

리하면 T < 2449

S. ¤

130. 올림피아라는 나라에는 n개의 섬이 있다. 그 중 주민이 가장 많은 섬은 파나센테르라는 곳이고, 각 섬에는 서로 다른 수의 주민이 산다. 이들 섬 사이에 양방향으로 오갈 수 있는 다리를 몇 개 건설하려고 하는데, 다음과 같은 조건을 만족해야 한다.

(a) 각 두 섬을 잇는 다리는 많아야 하나씩이다.

(b) 파나센테르로부터 이들 다리를 통해 모든 섬에 갈 수 있다.

(c) 파나센테르로부터 이들 다리를 각각 최 한 번씩만 이용해 다른 섬에 간다면, 도중에 거치는 섬의주민의 수는 항상 점점 감소한다.

이렇게 다리를 건설하는 방법의 수를 구하여라. (중미 2006-5)

Page 260: 실전수학올림피아드 1400제 해답

260 조합

풀이 (b)는 연결된 그래프임을, (c)는 회로가 없음을 의미. 따라서, 나무. 주민이 많은 섬부터 차례로

연결시키는 과정을 생각해보면, ¢ ¢ ¢ 답 (n¡ 1)!. }

131. 갑 항공사와 을 항공사는 6개 도시를 연결하는 비행 직항로를 운항하고 있다. 각각의 두 도시 쌍마다 갑과 을 중 한 회사가 (양방향의) 직항로를 운항한다. 한 회사만을 이용하여 순환여행할 수 있는 네 도시가있음을 증명하여라. (단, 네 도시 P , Q, R, S의 순환여행이라 함은 P ! Q! R! S ! P 와 같이 돌아오는 경로를 뜻한다.) (아일랜드 2007-4)

증명1 A에서 B, C, D, E로 갑항로가 있다고 하자. 그럼 B, C, D, E 사이에는 한 도시에서 두 도시를

연결한 갑항로가 있어선 안 되고, 그럼 BC가 갑항로라면 B나 C에선 더이상 갑항로가 없으므로 기껏해야 갑항로는 DE만 추가될뿐. 그럼 B, C, D, E 사이를 훑는 을항로 사각형 존재. 따라서, 한 도시에서 네도시 이상으로 같은 회사의 항로가 있어선 안된다. 이번엔 A에서 B, C, D로 갑항로가, E, F로는 을항로가 있다고 하자. B에서 E, F에 모두 을항로가 있어선 안 되므로 B에서 E나 F로 갑항로 존재. C, D에서도 마찬가지로 E나 F로 갑항로 존재. 그럼 B, C, D 중에 어느 둘은 같은 E 또는 F로 갑항로가 있고 그럼 갑사각형 존재. ¤

증명2 삼각형 2개가 있다는 것은 잘 알려진 바, 갑항로로 삼각형 ABC가 있다고 하자. D에서 A, B, C

중 두 곳으로 갑항로가 있으면 안 되므로 D에서 A, B, C 중 적어도 두 곳으로 을항로 존재. E, F에서도마찬가지로 A, B, C 중 각각 적어도 두 곳으로 을항로 존재. 두 을항로 집합이 D, E, F에서 모두 달라야 하므로(같으면 을사각형 존재) ABC와 DEF를 왔다갔다 하는 을육각형 존재. 여기서 뚝딱뚝딱 그림마무리하면 됨. ¤

132. 어떤 행성의 주민들이 두 문자 A와 O로만 이루어진 언어를 사용한다. 실수를 피하기 위해서 같은 길이의

두 단어에서는 항상 철자가 적어도 세 자리는 다르다. 길이 n짜리 단어는2n

n+ 1개 이하임을 보여라.

(독일BW 1971 2차-2)

증명 각 단어와 많아야 한 자리만 다른 문자열들을 모두 모으면 모두 서로 다른 문자열임. ¤

133. 1£ 1 칸들로 구획된 M £N 직사각형판이 있다. 1£ 2 의 도미노도 많이 있다. 각 도미노가 두 칸을 차지하도록 판에 몇 개의 도미노를 올려놓았다. 판이 가득 차지는 않았지만, 어떤 도미노도 움직일 수 없다(판의 선을 따라서 옆칸으로 미끄러져갈 수 없다). 판의 가장자리에는 틀이 있어서 도미노는 판 밖으로 삐져나올 수도 없다. 빈칸의 수는 1

5MN보다 작음을 증명하여라. (Towns 1989가을 JA5)

증명 도미노를 하나 이상 사용했으므로 일단 MN ¸ 2. 빈 칸이 있으면 그 빈 칸의 주면은 항상 다음

과 같이 구획되어야 함을 증명할 수 있음.114

2X4

233

(증명: 빈칸이 있으면 그 빈칸을 포함하는 빈칸들의 연결된 영역 중에서 가장 바깥부분(즉, 도미노와 만나는 부분)의 칸이 있을 것이고, 그 빈칸을 X, 도미노가 있는 칸(WLOG, X의 윗칸)을 1이라 하자. 1을까는 도미노는 X를 향하지 않는 방향으로 깔려 있을 것이고, 그럼 WLOG 위와 같이 11 밑에 X. X 왼쪽의 2도 빈칸일 순 없으므로 이곳은 도미노가 있고 역시 X를 향하는 방향이 아닐테니 그림처럼 11, 22,X. 계속 비슷하게 33, 44도 마찬가지.) 만일, X 옆칸(상하좌우의 칸)이 다른 빈 칸과 이웃하면 위의 그림이 잘 그려지지 않으므로 모순. 즉, X의 옆칸 4곳은 X만의 것. 즉, 빈칸마다 그 옆 네 칸을 포함한 다섯 칸이 고유의 영역으로 확보되어야 하므로 MN ¸ 5k. ¤

134. 30쌍의 부부가 둥근 탁자에 둘러앉아 있다. 앉은 자리들이 정다각형을 이룬다고 할 때, 부부간의 거리가같은 두 부부가 존재함을 보여라. (이탈리아 1989-2)

증명 자리를 흑백으로 번갈아 칠하면 흑색 30석, 백색 30석. 부부간 거리는 (짧은 쪽으로) 1부터 30까

지 30종류뿐이므로, 거리가 같은 두 부부가 없다면 30종류의 거리가 모두 나타나야 함. 그럼 15가지의홀수 거리에 앉은 부부는 서로 다른 색의 자리에 앉으므로 이제 흑색 15석과 백색 15석이 남고, 남은 부부들은 서로 같은 색 자리에 앉아야 하는데 한 가지 색이 홀수개씩 남아있으므로 모순. ¤

135. 집합 f1; 2; : : : ; 3ng에서 n + 2개의 수를 뽑으면, 그 중 차이가 n보다 크고 2n보다 작은 두 수를 찾을 수있음을 증명하여라. (헝가리 1952-2)

Page 261: 실전수학올림피아드 1400제 해답

4.2 조합 고급문제 261

증명 0 · a < b < n 인 a, b에 해 fa; n+ bg, fn+ a; 2n+ bg, fb; 2n+ ag 의 각 집합에서 하나의 수

만 뽑을 수 있으므로 mod n으로 두 종류 a와 b인 수들에서 많아야 3개만 뽑을 수 있음. n+ 2개 이상의수를 뽑으면 두 종류에서 4개 이상을 뽑게 되는 경우가 생김. 비둘기집. ¤

136. 공간의 모든 점을 빨간색 혹은 파란색으로 칠했다. 세 꼭지점이 빨간색이거나 네 꼭지점이 모두 파란색인단위정사각형이 존재함을 보여라. (독일BW 1976 2차-4)

증명 귀류법. 단위 거리의 두 빨간 점 P , Q가 있다고 하자. 정사각형 PQRS를 생각하면 R, S는 항

상 파란색. 그럼 PQ를 축으로 RS를 60도 회전하여 생긴 TU도 파란색이고 RSTU가 파란색 단위정사각형이 됨. 단위 거리의 두 빨간 점이 없다면, 빨간 한 점으로부터 단위 거리의 점은 모두 파란점이므로그 구면 위의 네 점으로 된 단위정사각형을 찾으면 끝. 빨간 점이 전혀 없으면 그것도 그냥 끝. ¤

별증 역시 귀류법. 정사각형 PQRS를 잡는데, 파란 점은 2개 or 3개. 파란 점이 2개이면, P와 R이면

정팔면체 A-PQRS-B를 생각하면 끝. 이제 이런 두 빨간점과 두 파란점이 각선으로 마주보는 관계도없다고 가정. 파란 점이 2개이고 P와 Q가 파란 점이면 정육면체 PQRS-TUVW를 생각하면 끝. 이제빨간점 둘과 파란점 둘도 없다고 가정. 파란 점이 3개이면 또 정팔면체 A-PQRS-B를 생각하면 끝. ¤

137. 모든 꼭지점이 변으로 연결되어 있는 다면체가 있다. 즉, 각선을 갖지 않는다. 이 다면체는 사면체임을증명하여라. (헝가리 1948-2)

증명 K5는 평면에 그릴 수 없음. ¤

별증 점이 5개 이상이면, 임의로 세 점을 잡았을 때, 다른 점들은 이 세 점을 지나는 평면에 해 항상

반 쪽 영역에 있어야 함. 그럼 일단 점은 5개뿐이고, 이 때 처음에 잡았던 세 점은 임의이므로 어떤 세점도 면을 구성할 수 없고 내부를 가르는 평면이 되어야 함. 이것은 다면체에 면이 없다는 것이므로 모순. ¤

138. 어떤 고 법정에서는 재판관들을 위한 23개의 좌석이 한 줄로 배열되어 있었다. 재판이 길어질 경우, 몇몇 재판관은 법정을 떠나기도 하고 다른 재판관들이 들어오기도 한다. 법정으로 들어오는 경우 판사는 혼자 들어올 수도 있고 두 명이 짝을 지어 들어올 수도 있는데, 만약 두 명이 짝을 지어 들어오는 경우 법정을 떠날 때도 함께 나가야 하며 법정을 나간 뒤에는 짝의 구분이 없어진다. 또한 법정에는 관리인이 존재하여 만약 두 재판관이 짝을 지어 들어오는 경우 연속된 두 좌석에 앉도록 안내한다. 만약 재판이 진행되는 동안의 어느 시점에서도 법정에 앉아있는 재판관이 항상 16명 이하라면, 관리인이 이 작업을 항상 해낼 수 있음을 보여라. (호주 1988-5)

증명 23개의 자리를 3인실 7개와 2인실 1개로 나눈다. 즉, [ABC] [DEF] [GHI] [JKL] [MNO] [PQR]

[STU] [VW] 로 분할한다. 규칙은 `각 3인실의 중앙석에는 솔로부 는 앉을 수 없다' 뿐으로 하면 된다.

그럼 솔로부 가 앉을 수 있는 자리는 (7개의 중앙석을 제외하고) 총 16석이므로 솔로부 는 언제든지앉을 수 있다. 또한, 커플이 들어올 때 3인실에 들어갈 수 없다면 각 3인실은 이미 두 명 이상이 앉아 있는 상태이고, 그럼 3인실에 이미 총 14명이 앉아있으므로, 그 커플은 비어있는 2인실에 앉으면 된다. ¤

139. 각 행과 열에 자연수 번호를 붙인 무한한 크기의 체스판을 생각하자. 각 칸에는 최 1개의 동전을 놓을수 있다. 임의의 두 자연수열 (an), (bn)에 해 n번째 행에는 an개의 동전이 있고 m번째 열에는 bm개의동전이 있도록 동전을 놓을 수 있음을 보여라. 단, 동전은 무한히 제공된다. (이탈리아 1993-3)

증명 먼저 각 열에는 1개씩의 동전만 있도록 하고(예를 들어, 수열 an이 처음 몇 항이 2, 4, 3이라면

아래와 같이 함)

제1행: OO

제2행: ..OOOO

제3행: ......OOO

그 다음 각각의 돌은 같은 행 위에서만 움직이는데, 첫 번째 열부터 bi들의 개수에 맞도록 오른쪽에서 돌들을 그 열로 끌어오면 됨. ¤

140. 한 변의 길이가 n인 정사각형을 아래 타일로 덮을 수 있으려면 n은 어떤 값이어야 하는가?(이탈리아 1995-1)

Page 262: 실전수학올림피아드 1400제 해답

262 조합

풀이 우선 넓이를 고려할 때 9 j n2 이므로 3 j n. 체스판의 흑백을 칠하듯 하면 위의 타일은 항상 두

가지 색을 6:3으로 깔게 되므로 깔린 두 색의 차이는 항상 3의 배수. 그런데 n이 홀수라면 두 색의 칸의개수 차이는 1이므로 곤란. 즉 2 j n. 따라서 6 j n 이고, 6£ 6 이 되므로 이것은 항상 잘 됨. }

141. 마을에 주점 A, B, C, D가 있고, A와 D 사이를 제외하고는 어떤 둘을 택해도 서로 연결되어 있다. 한 취객이 A에서 시작해서, 술을 한 잔 마시고 연결된 다른 주점 중 아무 곳으로나 같은 확률로 이동하는 행동을 반복한다.

(a) 다섯 번째 잔을 마실 때 C에 있을 확률은 얼마인가?

(b) n > 5 일 때, n번째 잔을 마실 때 이 취객이 있을 확률이 가장 높은 주점은 어디인가?(이탈리아 1995-3)

풀이 n번째 잔을 마실 때 A나 D에 있을 확률을 pn이라 하면, B나 C에 있을 확률은 qn = 1 ¡ pn.

p1 = 1 이고, pn+1 =23qn =

23¡ 23pn. pn+1 ¡ 2

5= ¡ 2

3(pn ¡ 2

5). 즉, pn =

25+ 35(¡ 23)n¡1. }

142. 20명의 학생이 필기와 구두로 수학 시험을 쳤다. 이 학생들 중 어떤 두 명을 택해도 필기와 구두 점수가모두 같지는 않다. A의 필기, 구두 점수가 둘다 B의 점수 이상이라면 A가 B보다 낫다고 말한다. 각 시험점수가 1 이상 10 이하의 정수라고 할 때 다음 물음에 답하여라.

(a) A가 B보다 낫고, B가 C보다 나은 학생 A, B, C가 있음을 증명하여라.

(b) 학생이 19명이어도 (a)가 성립할까? (이탈리아 1995-2)

증명 한 학생이 받을 수 있는 모든 점수쌍들을 20£ 20 의 표로 나타내어보자. 그럼 각 학생은 어느 칸

엔가 속하고, 한 칸에 두 학생이 동시에 속할 수 없다. (a) 귀류법으로 풀면, 한 행 혹은 한 열에 세 학생이 있을 수 없다. 그럼 모든 행과 모든 열마다 둘씩 있다. 그럼 제일 왼쪽(필기 점수가 제일 작은) 열의두 학생을 택하면 그 중 필기 점수가 높은 학생과 같은 행에 또 한 명이 있으므로 그래도 세 학생을 찾게됨. (b) 위의 표에서 / 방향의 주 각선과 그 바로 위의 각선을 택하면 19명이 그런 세 학생 없이 잘구성됨. ¤

143. 한 원의 내부에 100개의 점이 있다. 어떤 점도 원의 중심에 있지 않고, 어떤 두 점도 같은 반지름 위에 놓여있지 않다.

(a) 정확히 10개의 점을 갖는, 중심각이2¼

11인 부채꼴이 존재함을 보여라.

(b) 정확히 11개의 점을 갖는 중심각2¼

11인 부채꼴도 항상 존재하는가? (러시아 1990 4차-y10-2)

증명 (a) 원판을 11등분했을 때 비둘기집의 원리에 의해 9 < 10011

< 10 이므로 9개 이하의 점을 갖

는 부채꼴도 있고 10개 이상의 점을 갖는 부채꼴도 있음. 그 두 부채꼴 사이를 연속적으로 움직이는 동안 그 움직이는 부채꼴에 포함되는 점의 개수도 연속적으로 변하므로 딱 10개의 점을 갖는 부채꼴 존재.(b) 원판을 10등분하고 각각의 부채꼴의 경계마다 10개씩의 점을 거의 같은 반지름 위에 위치시키면 불가능한 경우가 됨. ¤

144. 8£ 8 체스판의 각 칸에 수 1; 2; 3; : : : ; 64를 하나씩 넣었다(모든 수가 한 번씩 사용됨). 수의 합이 100보다큰 2£ 2 영역이 적어도 네 곳 있음을 보여라. (스웨덴 1977-5)

증명 1 + 2+ 3+ ¢ ¢ ¢+64 = 64¢652

= 2080. 2£ 2 영역 16개로 분할. 귀류법으로, 이 중에서 13개 이상

의 영역에서 수의 합이 100 이하이면, 전체수의 합은 S · 13 ¢100+(53+54+ ¢ ¢ ¢+64) = 1300+702 =2002 < 2080 으로 모순. ¤

145. Antonio와 Bernado가 게임을 한다. 각각 m개와 n개의 칩이 있는 두 칩 더미가 있다. Antonio가 먼저 시작하고 다음 (i){(iii) 중 하나를 골라 행동한다.

(i) 칩을 한 더미에서 하나 가져오거나,

(ii) 칩을 양 더미에서 하나씩 가져오거나,

(iii) 칩을 한 더미에서 다른 더미로 하나 옮긴다.

더 이상 이런 작업을 할 수 없는 사람이 진다. 둘 중 어느 쪽이든 한 명의 필승 전략이 존재하는지를 m,n에 관한 함수로 표현하고, 존재할 때의 필승 전략을 서술하여라. (이탈리아 2004-4)

Page 263: 실전수학올림피아드 1400제 해답

4.2 조합 고급문제 263

풀이 (짝;짝) 2 LS. 불변량 게임. }

146. (1) 한 변의 길이가 1인 정사각형의 내부에 288개의 점이 있다. AB와 평행하고 변 AD와 BC를 잇는길이 1의 선분들의 집합 S를 그리는데, 288개의 점 각각에서 AD와 평행하게 S의 선에 연결되는 선분을 그렸을 때, 이런 선분 전체의 길이의 합이 24보다 작도록 할 수 있음을 보여라.

(2) (1)에서, 선분 전체의 길이의 합이 18 이하가 되도록 할 수도 있음을 보여라. (스웨덴 1961-4 변형)

증명 (1) n개의 열로 등분할한 후 각 열의 중앙선을 그리면 가장 가까운 중앙선까지는 기껏해야 12n

이므로, 길이합은 최 n+ 288 ¢ 12n= n+ 144

n 이고, 이것은 n = 12 일 때 최소 24. (2) AB부터 CD까

지 등간격으로 2n개의 세로선을 그리고, 짝수번째 선들과 홀수번째 선들의 집합을 분리. 각 점에서 짝수번째 선들에 이르는 최단거리의 합과 홀수번째 선들에 이르는 최단거리의 합은 각 점에 해 세로선간격만큼 기여되므로 288 ¢ 1

2n¡1 . 따라서, 두 집합 중 총합이 더 적은 쪽을 택하면 선분의 길이의 합은

최 gn =12(2n + 288

2n¡1 ). gn의 최소값을 구하면 gn =12+ 12((2n ¡ 1) + 288

2n¡1 ) ¸ 12+p288 인데

n은 자연수라야 하므로 등호조건 2n ¡ 1 = p288 ; 17 에 가장 가까운 n = 9 를 택하면 gn ¸ g9 =

12(18 + 288

17) = 17 + 8

17. ¤

147. mn명의 학생이 있는데, 이들의 키는 모두 다르다. 이 학생들을 m개의 행과 n개의 열을 이루도록 세운다(m;n > 1). 각각의 행에서 가장 작은 학생을 뽑고, 그 중 가장 큰 학생의 키를 A라 하자. 그리고 각각의 열에서 가장 큰 학생을 뽑고, 그 중 가장 작은 학생의 키를 B라 하자. 다음 중 어떤 것이 가능할까?

(헝가리 1963-1)

A < B; A = B; A > B

풀이 그냥 15퍼즐처럼 순서 로 세우면 A = B. 거기서 9와 13의 위치를 바꾸면 B는 불변인데 A는

줄어들어 A < B. A > B 는 항상 불가능함. 왜냐하면 A > B 라 하면 A = ai, B = bj 일 때 i행의 학생

들은 모두 j열의 학생들보다 크니까 i행 j열의 학생은 오도가도 못함. }

148. 8개의 단위정육면체가 주어져있다. 이 정육면체들에서 24개의 면은 흰색으로, 나머지 24개의 면은 검은색으로 칠해져있다. 이 정육면체를 잘 붙여서, 겉면에서 나타난 흰색과 검은색의 넓이가 똑같은 한 변의길이 2인 정육면체를 만들 수 있음을 보여라. (러시아 1990 4차-y11-3)

증명 한 블록의 서로 마주보는 면을 짝으로 생각할 때, 임의의 블록의 임의의 짝의 두 면 중에서 어느

면이 드러나고 어느 면이 숨도록 할지를 독립적으로 선택할 수 있음. 한 짝의 두 면이 같은 색이면 상관할 거 없고, 두 면이 서로 다른 짝(이런 짝은 짝수개)들만 적당히 설정해주면 되므로 그냥 끝. ¤

149. 파티에 있는 사람 중 임의로 네 명을 뽑으면, 그 중에는 나머지 세 명을 알고 있는 한 명이 반드시 포함되어 있다고 한다. 파티에 있는 사람 중 파티장에 있는 모든 사람을 알고 있는 사람이 있음을 증명하여라.

(헝가리 1960-1)

증명 가장 많이 아는 사람 A가 그 사람. A가 모르는 사람 B가 있다고 하자. A는 적어도 3명을 아니

까 A가 아는 사람의 그룹 X의 크기는 3 이상. X에 서로 모르는 쌍 C, D가 있다면, fA;B;C;Dg가 문제에 모순. 따라서, X의 사람들은 모두 서로 안다. 그럼 A의 최 성에서 X의 사람들은 더이상 다른 누구, 즉 B를 모른다. 그럼 그 경우에도 X에서 C, D를 아무나 골라 fA;B;C;Dg가 문제에 모순. ¤

150. 평면 위에 어느 세 점도 한 직선 위에 있지 않은 3n개의 점이 주어져있다. 이 점들을 꼭지점으로 하는 n개의 속이 꽉찬 삼각형을 구성하는데, 어떤 두 삼각형도 서로 만나지 않도록 항상 구성할 수 있는가?

(스웨덴 1969-6)

풀이 모든 점의 x-좌표가 다르도록 하는 x-축을 하나 잡고, x-좌표값이 가장 작은 것부터 3개씩 묶어

구성하면 됨. }

Page 264: 실전수학올림피아드 1400제 해답

264 조합

Page 265: 실전수학올림피아드 1400제 해답

제 5 장

종합문제

5.1 종합 중급문제

1. 어떤 원의 지름을 AB, 그리고 이 지름 위의 A와 B 사이의 어떤 고정점을 C라 하고, Q를 이 원의 둘레

위를 움직이는 점이라고 하자. Q와 C를 잇는 직선 위에AC

CB=

QC

CP를 만족하는 점 P를 잡자. 점 P의

자취를 서술하고 그를 증명하여라. (캐나다 1976-4)

풀이 P는 Q에 따라 결정되는 점이므로 P (Q)의 함수꼴로 표현하자. 그럼 함수 P는 C를 닮음의 중심

으로 하여 ACCB의 비를 갖도록 점을 옮기는 축소/확 변환이다. 문제에서 제시된 비례식 AC

CB= QC

CP 에

나타난 선분에 방향성이 있다고 하면(즉, P가 C에 해 Q와 반 쪽에 있는 점이라 하면), 직선 AB 위의 점 D = P (B) 에 해, P의 자취는 BD를 지름으로 하는 원이 된다.

만일 앞의 비례식에 나타난 선분에 방향성을 고려하지 않는다면, P가 C에 해 Q와 같은 방향에 나타나는 자취가 하나 더 있고, 이것은 앞에서 구한 자취를 점 C에 해 점 칭이동한 원이다. }

2. 눈금이 없는 세 개의 물통이 있다. 각각 m리터와 n리터의 용량을 가진 두 물통은 비어있고, m+ n리터의용량을 가진 나머지 한 물통은 물로 가득차 있다. m과 n은 서로소인 자연수이다.임의의 k = 1; 2; : : : ;m+

n¡ 1 에 하여, 한 물통의 물을 다른 관으로 적당히 옮기는 작업을 반복하여 용량 m+ n리터의 통에 정확히 k리터의 물이 남도록 할 수 있음을 보여라. (폴란드 1994 3차-4)

증명 m + n ! m ! n ! m + n 으로만 이동하는 것으로 한다면 n에는 늘 km mod n 이 들어가게

됨. m, n이 서로소니까 km은 n에 한 완전잉여계를 구성하게 되어서... ¤

3. 화폐수집가 프레드는 동전을 몇 개 가지고 있다. 각 동전의 지름은 10 cm 이하이고, 모든 동전은 30 cm£70 cm 크기의 단층(동전들이 서로 겹쳐놓이지 않는) 상자에 들어있다. 그는 지름이 25 cm인 새 동전을 선물받았다. 그의 모든 동전을 55 cm£ 55 cm 크기의 단층 상자에 넣을 수 있음을 증명하여라.

(Towns 1992봄 JO3)

증명 30£ 70 에서 오른쪽 R = 30£ 25 에 완전히 포함되는 동전들만 따로 떼어냄. 그럼 남은 동전들

은 폭 10을 범퍼로 하여 모두 L = 30£ 55 안에 있음. 이제 L 좌하에 R을 90도 회전시켜서 놓고, 우하에 25 크기의 동전을 놓으면 55£ 55 에 잘 들어감. ¤

Page 266: 실전수학올림피아드 1400제 해답

266 종합문제

4. Randy: \안녕 Rachel, 재미있는 이차방정식을 썼구나? 근이 어떻게 되지?"Rachel: \근은 두 개의 양의 정수야. 한 근은 내 나이와 같고 다른 근은 내 동생 Jimmy의 나이야."Randy: \와, 멋진데! 그럼 내가 너와 Jimmy의 나이를 알아낼 수 있나 두고봐. 네 방정식의 계수들이 모두 정수니까 그다지 어렵진 않을 것 같아. 그런데, 이 세 계수들을 모두 합하니까 소수가 되었어."Rachel: \재미있군. 이제 내가 몇 살인지 알아맞혀봐."Randy: \잠깐, 내가 네 나이를 추측해서 그걸 이 이차방정식의 x에 입해볼게... 이런, 0이 아니라¡55가 나왔네."Rachel: \맙소사, 저리 가!"

(1) Jimmy가 두 살임을 증명하여라.

(2) Rachel의 나이를 알아내어라. (캐나다 2001-1)

풀이 Rachel과 Jimmy의 나이를 a > b 라 하자. 그럼 이차방정식은 m(x ¡ a)(x ¡ b) = 0 의 전개인

mx2¡m(a+b)x+mab = 0이 된다.세 계수의 합은 x = 1을 입한 값과 같으므로 m(a¡1)(b¡1) = p(소수) 이다.

(m;a¡ 1; b¡ 1) = (1; p; 1); 즉 (m; a; b) = (1; p+ 1; 2)

일 수밖에 없다. 다음, Randy가 추측한 Rachel의 나이를 c라 하면 m(c¡ a)(c¡ b) = ¡55, 즉 (c¡ p¡1)(c ¡ 2) = ¡55 이다. 55 = 5 ¢ 11 이고 c ¡ p ¡ 1 < c ¡ 2 이므로, (c ¡ p ¡ 1; c ¡ 2) = (¡5; 11) 또는(¡11; 5) 이다. 어느 경우에나 p¡ 1 = (c¡ 2)¡ (c¡ p¡ 1) = 16 이므로, p = 17. 따라서, Jimmy의 나이는 두 살이 틀림없고, Rachel의 나이는 18살이다. }

5. 자연수 n을 최소공배수로 갖는 두 개의 자연수의 순서쌍 (a; b)의 개수는 n2의 양의 약수의 개수와 같음을 보여라. (헝가리 1962-1)

증명 각 소인수에 해 그 지수의 종류를 세어 비교하면 됨. pe k n 이면 (a; b)의 p의 지수쌍은

(0; e); (1; e); : : : ; (e; e); : : : ; (e; 1); (e; 0)의 2e+ 1가지. ¤

6. 세 변의 길이가 a, b, c인 삼각형의 넓이를 S라 하자. 다음 부등식을 증명하여라.

S ·p3

12(a2 + b2 + c2)

증명 양변 제곱하고 헤론의 공식 입하여 정리하면 (a2 ¡ b2)2 + (b2 ¡ c2)2 + (c2 ¡ a2)2 ¸ 0 과 동

치. ¤

7. 한 원을 이웃한 두 호의 중심각의 합이 항상 103± 이하가 되도록 7개의 호로 분할하였다. 이 7개의 호의중심각이 모두 A± 이상임을 말할 수 있는 A의 최 값을 구하여라. 그리고, 이 값이 정확한 최 값임을 증명하여라. (Towns 1992봄 JO4)

풀이 51, 52, 51, 52, 51, 52, 51 로 하면 조건을 만족하며 모든 각은 51도 이상. 만약 51도 미만의 각

이 존재한다면 그 각과 103도 이하의 각 셋(두 중심각의 합들)을 더하면 360도보다 작으므로 모순. ¢ ¢ ¢답 51도 }

8. 다항식 x4 + 3x3 + 6x2 + 9x+ 12 를 정수계수 이차식 두 개의 곱으로 나타낼 수 없음을 보여라.(플란더즈 1988-1)

증명 (x2 + ax+ b)(x2 + cx+ d) 를 전개하여 비교. WLOG 3 j b 라 하면 ! 3 j a, 3 j c ! 3 j d 에서

모순. ¤

9. 어떤 IMO에서, 34개국의 단장과 부단장들이 심판관 회의에 참석하였다. 회의 전에, 몇몇 사람들은 서로악수를 하였지만 같은 나라의 단장과 부단장끼리는 악수를 하지 않았다. 나중에 한국팀 단장이 나머지67명에게 각자가 악수한 사람들의 수를 물어보았더니 모두 다른 답을 하였다. 한국팀의 단장과 부단장이 악수한 사람들은 각각 모두 몇 명인가? (통신강좌 1989-D8)

Page 267: 실전수학올림피아드 1400제 해답

5.1 종합 중급문제 267

풀이 각각의 참석자들은 자기 자신과 자기 나라 사람과는 악수하지 않으므로, 그들은 0»66명의 사람

들과 악수할 수 있다. 한국팀 단장의 질문에 한 답이 모두 달랐으므로, i명과 악수한 사람을 Pi라나타내면 우리는 한국팀 단장을 제외한 참가자들을 P0; : : : ; P66로 표시할 수 있다. P66는 자기 자신과 자기 나라 사람을 제외한 모든 참석자들과 악수해야 한다. 그는 P0와는 악수하지 않았으므로, 그가악수한 사람은 P1; P2; : : : ; P65 그리고 한국팀 단장이다. 그러므로 P0와 P66은 같은 나라 사람이다.

마찬가지로 1 이상 65 이하인 i에 해, Pi는 P1; P2; : : : ; P65중 i ¡ 1명과 한국팀 단장과 악수하였다. 같은 방법으로, 32이하의 j에 해 Pj와 P66¡j는 같은 나라 사람이고 j < i < n ¡ j라면 Pj는

Pj+1; : : : ; Pn¡j¡1중 i¡ j ¡ 1명과 한국팀 단장과 악수하였음이 귀납적으로 증명된다. 그러므로 한국

팀 부단장은 P33이고 한국팀 단장과 부단장은 각각 P34; P35; : : : ; P66과 악수하였음을 알 수 있다. }

10. 1보다 작은 양의 유리수들 중에서 십진법으로 전개했을 때 10개의 서로 다른 숫자로 이루어진 길이 10의순순환마디를 갖는 것들을 모두 모은 집합을 M이라 하자.

(가) M의 모든 원소들의 산술 평균을 구하여라.

(나) 다음을 만족하는 양의 정수 n (1 < n < 1010)이 존재함을 증명하여라: 모든 a 2M 에 해 n ¢a¡a

이 음이 아닌 정수이다. (루마니아 2005 지역예선 y8-1)

풀이 (가) M의 임의의 원소 a = 0: _a1a2 ¢ ¢ ¢ _a10 는, bi = 9 ¡ ai 로 정해지는 또다른 M의 원소

b = 0:_b1b2 ¢ ¢ ¢ _b10 와 항상 짝을 지을 수 있다. 그리고 이 두 수 a와 b는 a+ b = 0: _999999999_9 = 1 이므로 평균이 1

2 이다. 따라서, M의 모든 원소의 평균도 역시 12 이다.

(나) M의 임의의 원소 a는

a = 0: _a1a2 ¢ ¢ ¢ _a10 = a1a2 ¢ ¢ ¢ a109999999999

=19¢ a1a2 ¢ ¢ ¢ a101111111111

의 꼴이 된다. 여기서 a1 + ¢ ¢ ¢+ a10 = 0+ 1+ ¢ ¢ ¢+ 9 = 45 로 9의 배수이므로 a1a2 ¢ ¢ ¢ a10도 9의 배수,

따라서 분자가 정수이다. 즉, 1111111111a는 언제나 음이 아닌 정수이므로 n = 1111111112 로 택하면된다. }

11. n을 2보다 큰 정수라고 하자. 직각삼각형의 빗변의 길이를 n번 거듭제곱한 것은 다른 두 변의 n제곱을 합한 것보다 더 큼을 증명하여라. (헝가리 1908-2)

증명 빗변의 길이를 c, 나머지 두 변의 길이를 a, b라 할 때, 다음의 부등식이면 충분하다.

cn = cn¡2c2 = cn¡2(a2 + b2) = cn¡2a2 + cn¡2b2 > an¡2a2 + bn¡2b2 = an + bn

¤

12. 세 자연수 a, b, c가 다음 방정식을 만족한다.

2abc+ 2ab+ 2ac+ 3bc+ 2a+ 3b+ 3c = 998

c의 최 값을 구하여라. (IT꿈나무 올림피아드 2006 1차)

풀이 양변에 3을 더하면 (2a+3)(b+1)(c+1) = 1001 = 7 ¢ 11 ¢ 13 으로 인수분해된다. 좌변의 인수들

은 모두 1보다 크므로 각각 7, 11, 13과 하나씩 응하게 된다. c가 최 일 때를 구하는 것이니 c+1 = 13

일 때 c = 12. }

13. T목 게임은 오목 게임과 비슷한 것인데, 오목을 만드는 신, 돌 네 개가 T자 모양(혹은 이것을 90±씩 회전한 모양)으로 늘어서도록 만드는 것을 목표로 한다. A와 B가 게임을 하고, A가 흑을 쥐고 먼저 두는 것으로 할 때, 두 사람이 최선의 전략으로 둔다면 A와 B 중에 누가 이기고, 게임은 몇 수 이내에 끝날까?

풀이 A가 7수만에 이긴다. 먼저 A가 중앙에 두고 그곳은 원점으로 잡자. 일반성을 잃지 않고 B가 2번

째 수를 제3사분면에 둔다고 하고, 그럼 A는 3번째 수를 (1; 1)에 둔다. 다음, 역시 일반성을 잃지 않고B가 x ¸ y 인 점 (x; y)에 둔다고 하고, 그럼 A는 5번째 수를 (0; 1)에 둔다.그 다음에는 (0; 2)와 (¡1; 1)이맞보기이므로 7번째 수에서 A가 이긴다. }

14. 어떤 사다리꼴을 중단 평행선(윗변과 밑변에 평행하고 같은 거리에 있는 직선)으로 잘랐더니 두 영역의넓이의 비가 1 : k 가 되었다. k가 가질 수 있는 값의 범위를 구하여라. (플란더즈 예선 1993-1994)

Page 268: 실전수학올림피아드 1400제 해답

268 종합문제

풀이 윗변의 길이를 a, 아랫변의 길이를 b라고 할 때 중단평행선의 길이는 a+b2 이다. 이 때 중단 평행

선에 잘려 생긴 두 사다리꼴의 높이는 같으므로 그 넓이 비는 a+ a+b2: a+b2+ b = 3a+ b : a+ 3b이다.

넓이 비 a+3b3a+b는 a또는 b 값이 매우 작게 변함에 따라서 3 또는 1

3에 가까워지므로 13< k < 3을 증명하

자.a+3b3a+b

= k ) a+ 3b = 3ak + bk ) (3¡ k)b = (3k ¡ 1)a이 때 1

3< k < 3라면, 임의의 양수 a에 해 위 식을 만족하는 b를 찾을 수 있고, k · 1

3 또는 k ¸ 3일때에는 a, b 중 하나가 0 이하가 되어 불가능하다.

즉, k의 범위는 13< k < 3: }

15. 최 공약수가 1인 서로 다른 3개의 수가 있는데 이중 어느 두 수도 서로소가 아니라고 한다. 이 3개의 수의 최소공배수 L의 최소값을 구하여라. (IT꿈나무 올림피아드 2006 1차)

풀이 3개의 수를 a, b, c라 하자. 어떤 두 수도 서로소가 아니므로 공통소인수를 갖는다. b와 c의 한 공

통소인수를 p라 하고, c와 a의 한 공통소인수를 q, a와 b의 한 공통소인수를 r이라 하자. a, b, c의 최공약수는 1이므로, 그럼 p는 a의 약수일 수 없고, q는 b의 약수일 수 없고, r은 c의 약수일 수 없다. 따라서, p, q, r은 모두 서로 다르다. 그럼 L ¸ pqr ¸ 2 ¢ 3 ¢ 5 = 30. 실제로, 세 수가 6, 10, 15이면 L = 30 인경우가 있으므로 30이 최소값이다. }

16. ABCD는 볼록사각형으로

\CBD = 2\ADB; \ABD = 2\CDB; AB = CB

를 만족한다. AD = CD 임을 증명하여라. (캐나다 2000-4)

증명 BD의 B쪽 연장선 위에 BA = BC = BE 인 점 E를 잡자.

그럼 B는 4CAK의 외심이므로 원주각의 성질에 의해

\BKC =1

2\CBD = \ADB

\BKA =1

2\ABD = \CDB

따라서, ¤AECD는 평행사변형이다. AC와 BD의 교점을 O라 하면 평행사변형은 O에 해 칭적이고,

그럼 O에 한 B의 칭점 F를 생각하면 ¤ABCF는 마름모가 된다. 즉, AC ? BF 이고 ¤AECD도마름모이다. 그러므로, AD = CD. ¤

17. 어떤 양의 실수를 입력하면 그 수의 소수부분을 버리고 정수부분만 취해서 얻어진 정수와 원래 수를 곱해서 다시 그 수의 정수부분만 취해서 돌려주는 프로그램이 있다. (즉, [x[x]]를 돌려준다.) 이 프로그램에a를 입력했더니 10이 나왔고 2a를 입력했더니 51이 나왔다. 3a를 입력하면 얼마가 나오겠는가?

(IT꿈나무 올림피아드 2006 1차)

풀이 우선 [a[a]] = 10 을 살펴보자. [a] · 2 이면 a < 3 이므로 a[a] < 6 이 되어 곤란하고, [a] ¸ 4 이

면 a ¸ 4 이므로 a[a] ¸ 16 이 되어 역시 곤란하다. 따라서, [a] = 3 일 수밖에 없다. [a[a]] = 10 은 그럼[3a] = 10 이 되고, 이것은 10 · 3a < 11, 즉 10

3· a < 11

3 을 얻는다(1).

다음 [2a[2a]] = 51 을 살펴보자. [2a] · 6 이면 2a[2a] < 42 이므로 곤란, [2a] ¸ 8 이면 2a[2a] ¸ 64 이므로 곤란하여, [2a] = 7 이다. 따라서 [14a] = 51, 즉 51 · 14a < 52, 51

14· a < 52

14 를 얻는다(2).

(1)과 (2)의 공통부분은 5114· a < 11

3 이다. 그럼 15314(= 10:9 ¢ ¢ ¢ ) · 3a < 11 이므로 [3a] = 10 이고,

[3a[3a]] = [30a] = 109 가 된다. }

Page 269: 실전수학올림피아드 1400제 해답

5.1 종합 중급문제 269

18. x, y, z는 다음을 만족시키는 자연수이다.

7x2 ¡ 3y2 + 4z2 = 8

16x2 ¡ 7y2 + 9z2 = ¡3

이 때, x2 + y2 + z2 의 값은 얼마인가? (IMTS R10-1)

풀이 두 식을 차례로 A, B라 하자. 7A ¡ 3B 하면 x2 + z2 = 65. 여기서 x2 = 1; 16; 49; 64 만 가능

함을 알 수 있다. 9A ¡ 4B 하면 ¡x2 + y2 = 84 이고, 앞선 x2 값들을 입해 y2이 가능한지 살펴보면x2 = 16 만 가능함을 확인할 수 있다. 그럼 y2 = 100, z2 = 49. ¢ ¢ ¢ 답 165 }

19. 검은 말과 흰 말로 A와 B가 3£ 3 체스판에서 게임을 한다. 규칙은 다음과 같다:

가. 두 사람은 번갈아 게임을 한다.

나. 자기 차례에서는 말 하나를 체스판의 빈칸 중 한 곳에 놓는다.

다. 말을 놓을 때는 검은 색이나 흰 색 말 중 어느 것을 선택해도 되고, 항상 같은 색의 말을 놓지 않아도 된다.

라. 판이 가득 찼을 때, A는 각 행, 각 열, 각 각선마다 검은 말이 짝수 개 있으면 1점씩 얻는다. B는각 행, 각 열, 각 각선마다 검은 말이 홀수 개 있으면 1점씩 얻는다.

마. 전체 8점 중 5점 이상을 얻은 사람이 이긴다.

(1) 4 : 4 무승부가 가능한가? 설명하여라.

(2) 처음하는 사람이 반드시 이기는 방법을 서술하여라. (캐나다 1978-5)

풀이 (1) 가능하다. 예를 들어 다음과 같이 게임이 끝난다면.

±

±

±

±

²² ²²±

(2) 먼저 하는 사람이 A라면 처음에 검은 말을 중앙에 놓는다. 그 후에는 항상 B가 놓는 곳의 칭에 되는 위치에 B가 놓은 말과 다른 색의 말을 놓는다. 그럼 게임이 끝났을 때 A는 중앙을 지나는 4개의 선에는 모두 짝수 개의 검은 말이 있어서 우선 4점을 얻고, 좌변과 우변이 서로 색이 칭으로 뒤집혀 있으므로 둘 중 한 곳에서 1점, 또 상변과 하변 중 한 곳에서 1점을 얻어 총 6점을 얻게 된다.

먼저 하는 사람이 B라면 처음에 흰 말을 중앙에 놓고 같은 방식으로 하면 된다. }

20. 한 공무원이 인구조사를 위해 어느 집을 방문하였다. 그 집에는 몇 명의 여자들이 함께 살고 있었는데, 이들은 자신의 나이를 밝히기를 꺼려하였고, 신에 \당신이 우리들 중 두 사람을 지목하면 그 두 명의 나이의 합은 기꺼이 답할게요." 라고 말하였다. 그러자 공무원은 \그렇다면 가능한 모든 쌍의 나이의 합을 제게 다 알려주세요." 라고 하였고, 30, 33, 41, 58, 66, 69이라는 답을 받았다. 이로써 공무원은 계산을끝내 즐겁게 돌아갈 수 있었다. 그는 각 사람의 나이를 어떻게 계산했을까? (인도지역예선 1990-7)

풀이¡n2

¢= 6 이므로 n = 4명이다. 네 사람의 나이를 a · b · c · d 라 하자. 그럼

a+ b · a+ c · b+ c; a+ d · b+ d · c+ d

이므로(b+ c와 a+ d 사이의 크기 관계만 불확실하다),

a+ b = 30; a+ c = 33; b+ d = 66; c+ d = 69

이다. 여기서 a와 b의 홀짝이 같음을 알 수 있고, 따라서 a + c 와 b+ c 도 홀짝이 같아서 b+ c 도 홀수여야 한다. 그럼

b+ c = 41 이고, a+ d = 58

이다. 이제 a =(a+b)+(a+c)¡(b+c)

2= 30+33¡41

2= 11살 이고, 나머지도 비슷하게 구하면 각각 19, 22,

47살이다. ¢ ¢ ¢ 답 11, 19, 22, 47살 }

Page 270: 실전수학올림피아드 1400제 해답

270 종합문제

21. 원 O의 지름 AB와 현 CD가 점 H에서 직교한다. AB의 길이는 두 자리의 자연수이고, AB의 자릿수의순서를 바꾸어놓으면 CD의 길이가 된다고 한다. OH의 길이가 양의 유리수일 때, AB의 길이를 구하여라. (AIME 1983-12)

풀이 %EEE

Put AB = 10m+n, then OH = 3/2 √(11(m+n)(m-n)), so 11 divides

m+n and m-n is square. Hence m=6, n=5.

¢ ¢ ¢ 답 65 }

22. n은 7의 배수가 아니지만 자리수의 합은 7의 배수가 되는 자연수이다. n의 자리수들 사이에 0을 몇 개 적당히 끼워넣어 7의 배수로 만들 수 있음을 보여라. (KAIST Cyber영재교육 2005 가을)

증명 n에는 0과 7이 아닌 자릿수가 2개 이상 있다. (그런 자릿수가 하나도 없으면 n은 7의 배수이고,

딱 하나 있을 때는 자릿수의 합이 7의 배수가 될 수 없다.) n의 자릿수 전체를 앞쪽(A)과 중간(B), 그리고 뒷쪽(C)의 세 부분으로 나누는데(즉, n = ABC = A ¢ 10x + B ¢ 10y + C꼴), A와 C에는 각각 0과7이 아닌 자릿수가 딱 하나 포함되도록 하자. 그럼 A와 C는 7의 배수가 아니다. BC가 7의 배수가 아니면 그 로 두고, 7의 배수이면 사이에 0을 끼워넣어 B0C로 만들자(그럼 7의 배수가 아니다). 이제 A,10A, 100A, 1000A, 10000A, 100000A는 7로 나눈 나머지가 모두 다르므로(그리고 7의 배수가 아니므로), BC(혹은 B0C)의 앞에 적절한 것을 붙이면 7의 배수가 된다. ¤

별증 n = am ¢ ¢ ¢ a1a0 이라 하면 각 자릿수 사이마다 0을 5개씩 끼워넣은 수 am00000am¡100 ¢ ¢ ¢ 00a100000a0이 7의 배수이다.왜냐하면, 106 ´ 1 (mod 7)이므로, 106mam+¢ ¢ ¢+106a1+a0 ´ am+¢ ¢ ¢+a1+a0 ´ 0(mod 7) 이기 때문이다. ¤

23. 현이 아버지는 어린 아들 현이에게 숫자 교육을 시키기 위해,세 자리짜리 숫자카드 세트를 샀다(000{999).

그러나 집에 와서 카드의 개수를 세어보았더니 1000장이 안되었다. 그래도 똑똑한 현이는 이 숫자카드 세트만 가지고도 000부터 999까지 모두 만들어내서 잘 갖고 놀았다. 그러면 이 카드 세트에는 최소 몇 장의카드가 있었을까? 주 0, 1, 6, 8, 9는 뒤집어서 보면 각각 0, 1, 9, 8, 6이다. (통신강좌 1993-7-30)

풀이 서로 다른 숫자 카드 A, B가 각각 뒤집어서, B, A가 된다면 둘 중에 하나는 없어도 된다. 그런

쌍의 개수를 세어 보자. 일단 2, 3, 4, 5, 7 중 어느 하나가 있는 카드는 뒤집어서 숫자가 되지 않으므로생각할 필요가 없다. 이제 0, 1, 6, 8, 9로만 이루어진 125개의 카드에 해서 생각하자. 뒤집어서 자기자신이 되는 카드의 개수 = 5 (첫째 자리와 그에 응하는 셋째 자리) £ 3 (가운데 자리 숫자로 올 수있는 것의 개수 ) = 15. 따라서 125¡ 15 = 110 은 55개의 쌍으로 나눌 수 있고 둘 중의 하나는 없어도

된다. ¢ ¢ ¢ 답 945 }

24. (이 문제가 치러진 회는 2005년이다.) 전설에 따르면, 때가 되면 깨어나 이 문제를 풀고 있는 모든 사람을 잡아먹고 다시 그 년도의 자릿수의 합에 해당하는 해 동안 잠이 들게 되는 몬스터가 있다. 이 몬스터는서기 234년에 처음으로 이 회를 습격하였다. 그러나 여러분, 동요할 필요는 없다. 여러분의 희망처럼 이번 해의 여러분들은, 그리고 앞으로도 10년 동안은, 안전함을 증명하여라. (플란더즈 2005-J1)

증명 (제주 기고 1학년 오재성)

처음 왔던 해인 234는 9의 배수이고 9의 배수의 자릿수의 합은 9의 배수이다. 9의 배수 해만큼 잠들었다다시 나타나면 역시 그 해도 9의 배수이고, 따라서, 몬스터가 나타나는 해는 항상 9의 배수이다. 그런데,2020년까지 각 자릿수의 합은 28이 최 이다. 그럼 몬스터가 잠드는 기간은 9년, 18년, 혹은 27년이고,

따라서 연속된 세 9의 배수인 해 중 적어도 한 번은 몬스터가 찾아온다. 즉, 1980년, 1989년, 1998년 중적어도 한 번은 몬스터가 찾아온다.

1980! 1998; 1989! 2016; 1998! 2025

따라서, 2007년에는 찾아올리 없고, 2015년까지 앞으로 10년은 안전하다. ¤

25. \B = \C = 90± 이고 AB = 12 인 사다리꼴 ABCD가 있다. BC의 중점을 M이라 할 때, AM ? DM

이라고 한다. 이 사다리꼴의 높이가 정수가 되도록 하는 CD의 길이를 모두 구하여라.(플란더즈 예선 1995/1996 1차)

풀이 CD = x 라 하면 AD = x+ 12. (x+ 12)2 ¡ (x¡ 12)2 = h2, 즉 48x = h2... }

Page 271: 실전수학올림피아드 1400제 해답

5.1 종합 중급문제 271

26. 12개의 정사각형, 8개의 정육각형, 6개의 정팔각형으로 구성된 26개의 면을 갖는 볼록다면체가 있다. 각각의 꼭지점에서는 세 면이 만나는데, 정사각형, 정육각형, 정팔각형인 면이 꼭 하나씩 만난다. 두 꼭지점을 잇는 다면체 내부의(모서리나 표면 위의 선이 아닌) 각선은 모두 몇 개인가? (AIME 1988-10)

풀이 %EEE

There are (12·4 + 8·6 + 6·8)/3 = 48 vertices. From each vertex

there are 3 edges and 1 + 3 + 5 = 9 diagonals. Hence 47 - 3 - 9 = 35

internal segments. Hence 48·35/2 in total.

¢ ¢ ¢ 답 840 }

27. n£ n 숫자배열이 주어져 있다. n은 홀수이고 이 배열의 각 수는 1이거나 ¡1이다. 이 때 ¡1을 홀수개 포함하고 있는 행의 개수와 열의 개수의 합이 n이 될 수 없음을 증명하여라. (소련 1962-5)

증명 한 행 혹은 한 열의 수들을 모두 곱한 것을 생각하자. 그럼 그 결과는 그 줄에 ¡1이 짝수개이면

1, 홀수개이면 ¡1이 된다. 각 줄에 해 이것을 계산한 것을 다시 모두 곱한 것은 각각의 칸에 배정된 수에 해서는 그 수를 포함하는 행과 열에서 2번 포함되므로 제곱한 것이 되어 1이 된다. 따라서, 각 줄의결과가 ¡1인 것이 홀수개일 수 없다. ¤

28. x2 ¡ y2 = 20002 을 만족하는 정수해 (x; y)는 몇 개인가? (AIME 2000 2차-2)

풀이 %EEE

(x+y)(x-y)=2^8 5^6. There are two solutions with y = 0. None

with x = 0. Suppose x, y are positive. Then x+y and x-y have same

parity, so must both be even. The no. of factors of 2000/4 is

(6+1)(6+1) = 49. There is one factor 2^4 5^3 and 24 pairs (2a,2b)

with a, b unequal and 4ab = 2000. Each of these gives a lattice point

(x,y). But each of these gives 4 lattice points (±x,±y). So

2 + 4·24=98 in all.

¢ ¢ ¢ 답 98개 }

29. 다음의 계산을 관찰하고 그 일반적인 규칙을 증명하여라. (셈본중등초급 도전문제 2.4.2)

22 + 32 + 42 + 142 = 152

42 + 52 + 62 + 382 = 392

62 + 72 + 82 + 742 = 752

82 + 92 + 102 + 1222 = 1232

풀이 일반적인 규칙은 다음과 같다.

(2n)2 + (2n+ 1)2 + (2n+ 2)2 + (6n2 + 6n+ 2)2 = (6n2 + 6n+ 3)2

이 등식의 확인은 다음과 같이 할 수 있다.

(6n2 + 6n+ 3)2 ¡ (6n2 + 6n+ 2)2

= (6n2 + 6n+ 3 + 6n2 + 6n+ 2)(6n2 + 6n+ 3¡ 6n2 ¡ 6n¡ 2)

= 12n2 + 12n+ 5

= (4n2) + (4n2 + 4n+ 1) + (4n2 + 8n+ 4)

이 식이 (2n)2 + (2n+ 1)2 + (2n+ 2)2 과 같으므로 확인되었다. }

30. 볼록사각형 ABCD의 6개의 거리 AB, AC, AD, BC, BD, CD 중에서 최 의 것을 g, 최소의 것을 h라하자. g ¸ h

p2 임을 증명하여라. (호주 1991-1)

Page 272: 실전수학올림피아드 1400제 해답

272 종합문제

증명 90± 이상의 내각에 한 각선(둔각삼각형의 빗변)을 g0이라 하면 끝. ¤

31. 4개의 문자 A, B, C, D에서 중복을 허용하여 고른 길이 n의 문자열을 생각하자. 이렇게 만든 임의의 문자열에서 각 문자를 0; 1; 2; 3의 숫자로 하나씩 응하여 적당히 바꾸어주면 최고 자리가 0이 아니고 3의배수인 4진법의 n자리의 수를 항상 얻을 수 있음을 보여라. (호주 2003-8 변형)

증명 일반성을 잃지 않고, (왼쪽에서부터) 첫째 자리를 A라고 하자. A, B, C, D의 개수를 각각

a; b; c; d라고 하면 문자열의 값은

Aa+Bb+ Cc+Dd ´ ( mod 3)

일때 3의 배수가 된다. a; b; c; d 중 적어도 2개는 mod 3에 해 같은 값을 가진다. 같은 값을 가지는 두문자에 1, 2를 입하고 나머지에 3, 0을 입하면 충분하다.

예를 들어, 두 문자를 A;B라고 하자. 그러면 A = 1; B = 2; C = 3; D = 0이라 하면 a + 2b + 3c ´ 0(mod 3)으로 3의 배수가 된다. 만일 두 문자가 B;C라면 A = 3; B = 1; C = 2; D = 0이라 두면 3의 배수가 된다. ¤

32. x가 소수이고, y, z는 x2 + y2 = z2 을 만족하는 음이 아닌 정수들이다. y, z를 x에 관한 식으로 구하여라.(플란더즈 예선 2000/2001 1차)

풀이 x2 = (z + y)(z ¡ y) 이고 z + y ¸ 0; z ¡ y 이므로 x가 소수일 때 x2을 나누어 갖는 방법은

(z + y; z ¡ y) = (x2; 1)뿐이다. ¢ ¢ ¢ 답 y = x2¡12, z = x2+1

2}

33. 1; 2; : : : ; 99; 100 에서 55개의 수를 어떻게 택해도 그중에 항상 차이가 9가 되는 두 수가 있음을 증명하여라. (플란더즈 1989-1)

풀이 f1; 10; 19; :::; 100g, f2; 11; 20; :::; 92g, ..., f9; 18; 27; :::; 99g 등으로 비둘기집 구분. 한 집합의 원

소의 개수가 11{12개이므로 각각 최 6개까지만 뽑을 수 있음. 그럼 54개가 최 . }

34. 서로 다른 2000개의 자연수가 있는데, 홀수와 짝수의 개수가 같고, 총합은 3000000보다 작다. 여기에 3의배수가 있음을 보여라. (독일BW 1977 1차-1)

증명 우증명. 3의 배수가 없으면 가장 작은 2000개의 수로 우선 만들더라도 총합이 3000¢30012

¡ 3 ¢1000¢1001

2 혹은 그냥 1000 ¢ 3000 이 되어 3000000을 넘는다. ¤

35. 볼록사각형 ABCD의 각선 AC와 BD가 점 O에서 만난다. 삼각형 AOB와 COD의 넓이를 각각 S1과S2라 하고 사각형 ABCD의 넓이를 S라 할 때, 다음 부등식을 증명하여라.p

S1 +pS2 ·

pS

또, 등호가 성립할 조건은 AB와 CD가 평행할 때임을 보여라. (스웨덴 1986-2)

증명 OA, OB, OC, OD를 각각 a, b, c, d라 하면 j4AOBj : j4BOCj : j4CODj : j4DOAj = ab :

bc : cd : da. 따라서, 준식은pab+

pcd ·p(a+ c)(b+ d) 와 동치이고, 이것은 양변을 제곱해서 정리

하면 (pad ¡pbc )2 ¸ 0. 이것은 틀림없이 성립하고, 등호는 ad = bc 일 때, 즉 a : b = c : d 일 때이므

로 AB k CD 일 때. ¤

36. 사장은 1; 2; : : : ; 9의 번호가 붙은 9통의 편지를 차례 로 쓰고 있다. 그는 편지 한 통을 다 쓰면 서류받침의 편지들 맨 위에 올려놓는다. 비서는 서류받침에 놓인 편지들 중에서 제일 위에 있는 것을 뽑아들고와서 타이프를 치고 있다. 편지 한 통을 다 타이프하면 서류받침의 제일 위에 있는 편지를 다시 들고와서 타이프한다. 비서는 점심식사를 하려고 잠시 자리를 비우면서 `8번 편지를 막 끝냈음'이라고 메모를 하고 나갔다. 아직 타이프하지 않은 편지들의 가능한 순서는 몇 가지나 되는가? (예를 들어, 1, 7, 8번 편지를 이미 입력하였고 6, 5, 9, 4, 3, 2의 편지가 이 순서 로 남아있다면, 순서 6, 5, 9, 4, 3, 2가 한 가지 가능성이 된다.) (AIME 1988-15)

Page 273: 실전수학올림피아드 1400제 해답

5.1 종합 중급문제 273

풀이 %EEE

8 has been typed, so 1, 2, ... , 7 have already been put into

the tray. Any left must be typed in decreasing order. 9 can come

anywhere in the order or not at all (if already typed). So for a subset

of {1, 2, ... , 7} with k elements, there are k+2 possibilities for 9.

Hence 1·2 + 7·3 + 21·4 + 35·5 + 35·6 + 21·7 + 7·8 + 1·9 =

2 + 21 + 84 + 175 + 210 + 147 + 56 + 9 = 704.

¢ ¢ ¢ 답 704 }

37. 자연수들로 이루어진 어떤 무한 등차수열이 제곱수를 포함한다. 이 수열이 무한히 많은 제곱수를 포함함을 증명하여라. 단, 등차수열이란, 바로 앞의 항에 일정한 수를 더하여 다음 항을 계속 만들어가는 수열,

즉 a; a+ d; a+ 2d; a+ 3d; : : : 꼴의 수열을 뜻한다. (소련 1963-10)

증명 제곱수인 항을 n2, 공차를 d라 하면, n2 + dk꼴의 수들은 모두 이 수열의 항이 된다. 이 때,

(n+md)2 = n2 + d(2mn+m2d) 이므로 이것은 항상 이 수열의 항이 된다. ¤

38. 세 변의 길이가 연속하는 세 정수인 삼각형이 있다. 두 번째로 긴 변에 내린 수선이 그 변을 길이의 차가4인 두 선분으로 분할함을 증명하여라. (Towns 1989가을 JO2)

증명 AB = n + 1, BC = n, CA = n ¡ 1 이고 A에서 BC에 내린 수선의 발을 D, 그리고 BD = b,

CD = c 로 두자(b+ c = n). 피타고라스의 정리로 (n+ 1)2 ¡ b2 = AD2 = (n¡ 1)2 ¡ c2. 넘겨서 정리하면 4n = b2 ¡ c2 = (b+ c)(b¡ c) = n(b¡ c). 따라서, b¡ c = 4. ¤

39. f가 상수함수가 아닌 실수함수라 하자. 모든 실수 x에 해, f(x+ 1) + f(x¡ 1) = p3 f(x) 를 만족한다면 f는 주기함수임을 증명하여라. 또한, 이런 모든 f에 해, 모든 x에 해 f(x + p) = f(x) 가 되도록하는 가장 작은 양수 p는 무엇인가? (IMTS R5-4)

풀이 f(x+ 1) =p3 f(x)¡ f(x¡ 1) 의 점화식으로 보자. f(r) = a, f(r + 1) = b 로 두고 이 점화식

에 의해 차례로 구하면 다음과 같다.

x r r + 1 r + 2 r + 3 r + 4 r + 5 r + 6

f(x) a bp3 b¡ a 2b¡p3 a p

3 b¡ 2a b¡p3 a ¡a

따라서, f(x+ 6) = ¡f(x), 그럼

f(x+ 12) = f(x)

가 된다. a와 b는 임의로 정할 수 있으므로, p가 12보다 작은 자연수일 수는 없다.x와 y의 차가 정수일 때 x와 y가 같은 클래스에 있다고 말하기로 하자. 문제의 조건식은 같은 클래스에속하는 수들끼리만 관련되도록 함을 알 수 있으므로, 서로 다른 클래스의 수들마다 독립적으로 f를 정의할 수 있다. 따라서, 주기 p는 정수일 때만 의미가 있고, 최소값은 위에서 본 것처럼 12이다. }

40. 8원짜리와 15원짜리 우표들만 이용할 수 있고 또 무제한으로 공급된다. 그럼 어떤 우편 요금|예를 들어18원과 29원|은 정확히 만들어질 수 없다. 얻어낼 수 없는 가장 큰 우편요금|즉 이를 n이라 할 때, n보다 더 큰 요금들은 모두 얻어낼 수 있고 이 요금은 얻어낼 수 없는 그런 요금|은 얼마인가? 그리고 그 이유를 설명하여라. (캐나다 1974-6)

풀이 8과 15는 서로소이므로 15k (k = 0; 1; : : : ; 7) 들은 8로 나눈 나머지가 모두 다르다. 그리고, 8로

나눈 나머지가 같은 각각의 우표값들 중에서 가장 작은 값들이 15k들이다. 따라서, 8로 나눈 나머지가같은 각각의 요금 중 정확히 만들어질 수 없는 가장 큰 요금은 15k ¡ 8꼴이고, 이 중에서 가장 큰 값은15 ¢ 7¡ 8 = 97원 ¢ ¢ ¢ 답 }

41. 다음 중 음이 아닌 정수해를 갖는 방정식은 몇 개인가? 또, 자연수해를 갖는 방정식은 몇 개인가?

Page 274: 실전수학올림피아드 1400제 해답

274 종합문제

(플란더즈 예선 1997/1998 1차)

px =

px

px ¢ px =

px+

px

px ¢ px ¢ px =

px+

px+

px

px ¢ px ¢ px ¢ px =

px+

px+

px+

px

px ¢ px ¢ px ¢ px ¢ px =

px+

px+

px+

px+

px

풀이 모든 식이 x = 0을 근으로 가지므로 모든 식이 정수해를 갖는다. 한편, 자연수해를 갖지 않는 식

은 네 번째와 다섯 번째 식뿐이다. ¢ ¢ ¢ 답 5개, 3개 }

42. 벽에 똑같은 시계가 2개 있다. 하나는 현재의 모스크바의 시각을, 다른 하나는 현재의 그 지역의 시각을나타낸다. 두 시침끝 사이의 가장 가까울 때의 거리는 m, 가장 멀 때의 거리는 M이다. 두 시계의 중심 사이의 거리는 얼마인가? (Towns 1990가을 SO3)

43. 15£ 15 크기의 정사각형 표의 각 칸을 빨강, 파랑, 혹은 녹색으로 칠했다. 그럼 적어도 어느 한 가지 색에해서는, 그 색이 칠해진 칸의 수가 서로 같은 두 행이 있음을 증명하여라. (뉴질랜드 2000-9)

증명 귀류법. 각 행마다 각각의 색에 해 쓰인 횟수가 0; : : : ; 15 중 하나만 빼고 모두 나와야 함. 빠진

수를 각각 x, y, z라 하면, 다 합하면 3(1 + ¢ ¢ ¢+ 15)¡ (x+ y + z) = 15 ¢ 15. 여기서 모순. ¤

44. z > y > x ¸ 3 일 때,1

x+1

y=1

2+1

z를 만족하는 세 정수 x, y, z의 값을 구하여라. (1995 교육청경시)

풀이 우선 주어진 조건에 의해 1z< 1

y< 1

x· 13이다.

² x = 3

왜냐하면 x ¸ 4라면, 1y +1x< 14¢ 2 = 1

2< 12+ 1

z로 모순이다.

² y < 6

만일 y ¸ 6이라면, 1x +1y· 13+ 16= 12< 12+ 1

z:

즉, y = 4 또는 5이므로 (x; y; z) = (3; 4; 12) 또는 (3; 5; 30)

}

45. a가 자연수일 때, 부정방정식

x1 + 2x2 + 3x3 + ¢ ¢ ¢+ nxn = a

의 양의 정수해의 수는

y1 + 2y2 + 3y3 + ¢ ¢ ¢+ nyn = a¡ n(n+ 1)

2

의 음이 아닌 정수해의 수와 같음을 보여라. 단, 여러 변수가 있는 방정식의 해란, 그 식을 만족시키는 수들의 순서쌍 (x1; x2; : : : ; xn)을 의미한다. (헝가리 1904-2)

증명 두 방정식의 정수해의 집합 사이의 응 xi = yi +1이 양방향으로 다 되는 응, 즉 역함수가 있

는 응이므로 일 일 응이다. 두 집합의 해가 유한 개임은 자명하고, 이 응이 두 해집합의 조건 "양의 정수"와 "음 아닌 정수"에도 잘 부합하므로 이 응은 정의도 잘 된다. ¤

46. n은 십진법으로 전개했을 때 7가지 숫자가 나타나고, 그 숫자들은 모두 n의 약수이다. n의 자릿수로 나타날 수 없는 숫자를 모두 찾아라. (플란더즈 2000-1)

Page 275: 실전수학올림피아드 1400제 해답

5.1 종합 중급문제 275

풀이 0과 5는 나타날 수 없다. 나머지는 모두 가능하다. 7의 배수, 8의 배수, 9의 배수가 되도록 하면

충분한데, aaa:::aa (같은 숫자가 18개) 이면 항상 7의 배수와 9의 배수를 만족하고, 그 중에서 888..88

을 제일 나중에 쓰면 8의 배수도 된다. 짧은 수의 예를 찾으라면, 1369872같은 것은 4를 제외하고 모두잘 나타나는 수이다... }

47. 8£ 8 체스판에서 흑색칸 1개, 백색칸 1개를 지운것은 31개의 도미노( ; )로 겹치지 않고 채울 수 있음을 보여라. (통신강좌 1997-14-7)

증명 우선,가로나 세로 둘 중 하나가 짝수인 직사각형꼴의 Board는 도미노로 채울 수 있다.(

이렇게¢ ¢ ¢ )(단, 0도 짝수에 포함)

이제, 지워진 두개의 칸이 하나는 흑색, 하나는 백색이므로, 이 두칸을 각의 양끝으로 하는 직사각형은 가로, 세로중 하나는 홀수, 하나는 짝수이다.

일반성을 잃지 않고 가로가 짝수라고 하면, Chess Board를 다음과 같이 분할한다.

1°; 3°은 세로의 칸수가 짝수 ) 채움이 가능하다.2°; 4°는 가로의 칸수가 짝수 ) 채움이 가능하다.5°는

5°¡ 2와 5°¡ 3은 세로가 짝수 ) 가능5°¡ 1은 가로가 짝수 ) 가능) 모든 각 영역들이 독립적으로 도미노로 겹치지 않고 채움이 가능이때 Chess Board의 칸수는 62개이므로 도미노는 31개가 필요하였다.) 증명끝 ¤

48. (a) 12개의 합동인 삼각형으로 분할되는 삼각형의 예를 하나 찾아라. (b) 5개의 합동인 삼각형으로 분할되는 삼각형의 예를 하나 찾아라. (러시아 1989 4차-y8-6)

풀이 (a) 정삼각형을 4등분하고 각각을 중심3등분함. (b) 1:2 직각삼각형을 4+1개 쌓으면 됨. (b)의

일반화로 m2 + n2개가 항상 됨... APMO 2005 참조. }

49. 세 개의 주머니에 들어있는 구슬은 모두 a개이며, 첫째 주머니에 187개, 둘째 주머니에 a=5개, 셋째 주머니에 a=13의 배수 개가 들어있다. 이 때, a의 값은 얼마인가? (KMC예선 2002전기 고1)

풀이 셋째 주머니에 든 구슬 수를 ak=13이라고 하자. (k는 자연수)187 + a

5+ ak13= a

) 11 ¢ 17 = 52¡5k65

a:

처음 식에서 a는 65의 배수여야한다. 즉 52¡5k는 1; 11; 17 중에 하나여야 하고 이 때 52¡5k = 17이다.11 ¢ 17 = 17 ¢ 65=a) a = 65 ¢ 11 = 715 ¢ ¢ ¢ 답 }

...

...

③②

⑤−1

⑤.3

⑤.2

Page 276: 실전수학올림피아드 1400제 해답

276 종합문제

50. 어떤 삼각형의 세 변의 길이 a, b, c가 두 등식

a¡ 2b+ c = 0; a2 + b2 + 3c2 + 2ab¡ 4bc¡ 4ca = 0

을 만족한다. a, b, c는 모두 자연수이고, 이 중에 어느 하나는 5이다. a+ b+ c 는 얼마인가?(1999 KAIST 대전.충남 영재수학교실 2차평가)

풀이 a2 + b2 + 3c2 + 2ab¡ 4bc¡ 4ca = 0) (a+ b¡ 2c)2 = c2

이 때 a+ b¡ 2c = ¡c라면 a+ b = c로 삼각형의 세 변의 길이라는데 모순되므로 a+ b¡ 2c = c이다.(a¡ 2b+ c)¡ (a+ b¡ 3c) = 0 = 4c¡ 3b) 4c = 3b) a = 2b¡ c = 5

3c

세 수가 자연수이므로 a = 5; b = 4; c = 3이 유일한 값이다. ¢ ¢ ¢ 답 12 }

51. 주연이는 파란 구슬, 빨간 구슬, 노란 구슬을 가지고 있다. 주연이와 승오의 화를 읽고 주연이가 가진 구슬의 개수를 색깔별로 각각 구하면? 단, 주연이와 승오는 매우 똑똑하다. (ML프로포절 165-2)

주연 : 내가 가진 각 색깔별 구슬의 개수를 모두 곱하면 240이 돼.

승오 : 그것으로는 모르겠어.

주연 : 내가 가진 구슬의 개수는 네가 가진 구슬의 개수와 같아.

승오 : 아직도 모르겠어.

주연 : 나는 노란 구슬보다 파란 구슬을 11개 더 많이 갖고 있어.

승오 : 아! 알겠어.

풀이 (서울 경희고 1학년 이승후)

240을 세 인수의 곱으로 나타내는 방법은 여러 가지이므로 첫 번째 질문에서 모르는 것은 당연하다. 세번째 질문에서 240을 세 인수로 분해했을 때 그 중 어느 두 수의 차가 11이어야 하는데, 240의 약수

1 2 3 4 5 6 8 10 12 15 16 20 24 30 40 48 60 80 120 240

중에서 차가 11인 쌍은 (1; 12), (4; 15), (5; 16)뿐이다. 그리고, 각각 240 = 1 ¢12 ¢20 = 4 ¢4 ¢15 = 3 ¢5 ¢16으로 240을 나타낼 수 있다. 그런데, (1; 12; 20)과 (4; 4; 15)은 인수의 합이 각각 33, 23이 되는 것으로240을 나타나는 방법이 유일하므로(막노동) 이것 중에 답이 있었다면 두 번째 질문에서 승오가 알아내었을 것이다. (3; 5; 16)는 (2; 10; 12)와 인수의 합이 같으므로 이것만이 두 번째 질문에서 `모른다'고 한

것에 적합하다. ¢ ¢ ¢ 답 16, 3, 5개 }

52. a; b > 0 의 조화평균은2ab

a+ b를 말한다. 조화평균이 620이 되는 두 수 m < n 의 쌍은 모두 몇 개인가?

(AIME 1996-8)

풀이 %EEE

We have mn = 2^19 3^20 m + 2^19 3^20 n, or

(m - 2^19 3^20)(n - 2^19 3^20) = 2^38 3^40. Now 2^38 3^40 has 39·41

factors. Leaving aside the square root 2^19 3^20 which does not give

a solution, the factors form 799 pairs (one < square root and one >).

Each pair gives a solution m, n.

¢ ¢ ¢ 답 799 }

53. 네 개의 접시 A, B, C, D가 있고, 1000개의 콩이 있다. 2a3b5c꼴의 수(a; b; c = 0; 1; : : : ; 9)가 각각의 콩에 하나씩 적혀있다. 어떤 콩을 옮긴다는 것은, 그 콩이 A에 있었으면 B로, B에 있었으면 C로, C에 있었으면 D로, D에 있었으면 A로 옮기는 것을 말한다. 처음에 모든 콩은 접시 A에 있었고, 이제 콩 하나를택해 그 콩에 적힌 수의 약수를 갖는 콩(물론 그럼 자기 자신도 포함된다)들을 모두 옮긴다. 또다른 콩 하나를 택해 그 약수에 해당하는 콩들을 모두 옮기고, 이런 식으로 모든 콩에 해 한 번씩 작업한다(작업한콩에는 표시를 해놓고 표시가 안 된 콩 중에 하나를 계속 택한다고 생각하면 되겠다). 이 작업이 모두 끝난 후, 접시 A에 있는 콩은 모두 몇 개인가? (AIME 1999-7)

Page 277: 실전수학올림피아드 1400제 해답

5.1 종합 중급문제 277

풀이 %EEE

2^a 3^b 5^c divides 2^A 3^B 5^C iff a ≤ A, b ≤ B and c ≤ C. So

there are (10-a)(10-b)(10-c) multiples of 2^a 3^b 5^c. Thus it ends up in

position A iff 4 divides (10-a)(10-b)(10-c). So we have: a = 2,6, any

b, c, 200 poss; a = 0,4,8 b,c not both odd 3(100-25) = 225 poss; a odd,

b = 2,5, any c, 100 poss; a odd, b odd, c 2 or 6, 50 poss; or a odd,

b 0,4,8, c even 75 poss. Total 650.

¢ ¢ ¢ 답 650 }

54. 4ABC의 내부에 한 점 P를 잡았을 때 j4PABj ¸ j4PBCj ¸ 2j4PCAj 일 확률을 구하여라. 단, jXj는도형 X의 넓이를 나타낸다. (1995 학교경시)

55. 1보다 큰 자연수 n이 주어져있다. p, q는n¡ 1n

<p

q<

n

n+ 1을 만족하는 자연수일 때, q의 최소값을 구

하여라. (플란더즈 예선 2003 1차 변형)

풀이 1에서 각 항을 뺀 것으로 부등식을 다시 쓴다.1n> q¡p

q> 1

n+1:

이 때 q ¡ p는 양수이고 q ¡ p = 1인 경우 n + 1 > q > n이 되어 모순이므로 q ¡ p ¸ 2여야 한다. 또q > n ¢ (q ¡ p)이므로 q ¡ p가 커질수록 q값이 커진다. 따라서 q의 최소값은 q ¡ p = 2일 때일 것이라고짐작할 수 있다.q ¡ p = 2) 2(n+ 1) > q > 2n) q = 2n+ 1; p = 2n¡ 1:이 때 q < n ¢ 3이므로 q ¡ p ¸ 3일 때의 q 값은 항상 2n+ 1보다 크다.¢ ¢ ¢ 답 2n+ 1 }

56. 다음 방정식의 자연수해는 모두 몇 개인가?h x10

i=h x11

i+ 1

단, [t]는 t를 넘지 않는 가장 큰 정수를 나타낸다. (Towns 1989가을 JA1)

57. I0 = f¡1; 1g 로 주어져있고, y가 In¡1의 모든 원소를 취할 수 있을 때 방정식

x2 ¡ 2xy + y2 ¡ 4n = 0

의 모든 해 x를 모은 집합을 In으로 정의하자. 모든 음 아닌 정수 n에 해 In들의 합집합을 구하여라.(이탈리아 1987-4)

풀이 x¡y = §2n. In = (In¡1+2n)[(In¡1¡2n) = f¡(2n+1¡1); : : : ;¡3;¡1; 1; 3; : : : ; 2n+1¡1g.즉 합집합은 모든 홀수들의 집합. }

58. 2001을 1979개의 양의 완전제곱수의 합으로 표현하는 방법을 모두 구하여라. 단, 더하는 순서는 무시한다. (유고슬라비아 1979 고2-3)

풀이 22 ¡ 1, 32 ¡ 1, 42 ¡ 1, 즉 3, 8, 15의 합으로 22를 나타내는 방법. mod 3으로 생각하면 경우가

3 + 3 + 8 + 8뿐임이 금방. }

59. 이등변삼각형 ABC에서 AD는 밑변 BC에 내린 수선이고, M은 변 AC 위의 한 점이다. DB ¡ DM <AB ¡AM 임을 증명하여라. (몰도바 1996 최종-y7-3)

증명 AD에 한 M의 칭점을 N이라 하면 준식은 DB ¡DN < BN 이고 이것은 삼각부등식. ¤

60. 쌍곡선 2xy ¡ 5x+ y = 55 위에 있는 x, y가 정수인 점 (x; y)의 개수를 구하여라. (IMTS R20-1)

61. 모양의 타일은 이 한 종류만으로 다음과 같이 무한히 기다란 띠를 만들 수 있다.

Page 278: 실전수학올림피아드 1400제 해답

278 종합문제

이처럼 주어진 한 종류의 타일들만으로 띠를 만들 수 있는 타일을 모두 골라라.(IT꿈나무 올림피아드 2006 1차)

(1) (2) (3) (4) (5)

풀이 (1)은 두 장을 맞붙여 직사각형을 만들 수 있고 그것을 계속 이어붙여 띠를 만들 수 있다. (3),

(4)는 두 장을 맞붙여 꼴을 만들 수 있고, 이 꼴을 일렬로 계속 이어붙이면 띠를 만들 수 있

다. (2)는 오목한 부분을 어떻게 채울지 따져보면 와 같이 계단형의 띠가 무한히 이

어지게 되는데 이 계단형 띠는 어떤 가로선도 존재할 수 없게 하므로 일자형 띠는 생길 수 없다. (5)는오목한 부분을 채우는 방법을 생각해보면 깔 수 있는 방법은 유일하고 그 경우 띠를 이룰 수 있는 직선이 생길 수 없음을 관찰할 수 있다. ¢ ¢ ¢ 답 ○×○○× }

62. 볼록사각형의 두 각선이 이 사각형을 네 영역으로 나누는데, 각 영역의 넓이가 정수이다. 이 네 영역의넓이의 곱이 완전제곱수가 됨을 보여라. (유고슬라비아 1981 고2-2)

증명 각선이 교점에 의해 각각 a : b, c : d 로 내분되었다면 네 영역의 넓이가 ac; bc; ad; bd꼴이 되겠

군. ¤

63. m과 n은 m2 +mn+ n2 이 9의 배수가 되게 하는 정수이다. m, n은 3의 배수임을 증명하여라.(헝가리 1958-2)

증명 9 j (m¡ n)2 + 3mn 이므로 3 j (m¡ n)2. ¤

64. 자연수 n을 주면 1부터 n까지의 역수의 합(1 + 12+ 13+ ¢ ¢ ¢+ 1

n)을 계산해주는 버튼이 들어있는 계산기

가 있다. 이 계산기에 허용된 연산은 이것과 사칙연산뿐이다. 다음 중 이 계산기에서 n이 얼마나 크든 상관없이 20번 이내의 연산으로 항상 결과를 얻을 수 있는 식들을 모두 골라라.

(IT꿈나무 올림피아드 2006 1차)

(1) 2에서 2n 까지의 짝수들의 역수의 합을 구하는 함수

(2) 1에서 2n¡ 1 까지의 홀수들의 역수의 합을 구하는 함수

(3) 1¡ 12+1

3¡ 14+ ¢ ¢ ¢+ 1

2n¡ 1 ¡1

2n을 구하는 함수

(4) 1 +1p2+

1p3+ ¢ ¢ ¢+ 1p

n을 구하는 함수

(5) 1¡ 1p2+

1p3¡ 1p

4+ ¢ ¢ ¢+ 1p

2n¡ 1 ¡1p2n

을 구하는 함수

풀이 f(n) = 1+ 12+ 13+ ¢ ¢ ¢+ 1

n 라 하자. (1)은 f(n)=2에 해당한다. (2)는 f(2n)¡(1). (3)은 (2)¡(1)이므로 모두 가능하다. 이 계산기의 연산으로는 항상 유리수만 나오므로 (4), (5)는 일반적으로는 얻을수 없다. }

65. 어떠한 세 점을 뽑아도 그 중에 단위 거리만큼 떨어진 두 점이 항상 존재하도록, 평면 위에 일곱 개의 점을 배치할 수 있음을 보여라. (IMTS R29-4)

증명 (이주호) 다음과 같이 하면된다.. 아래 그림은 단위길이인 선분만 연결한 것이다.

Page 279: 실전수학올림피아드 1400제 해답

5.1 종합 중급문제 279

두 정삼각형 ABC와 A0B0C0을 생각하자. 이 여섯 점 중에서 세 점을 택하면 그 중 두 점은 한 쪽 정삼각형에 속하므로 단위 거리에 있다. 이제 일곱 번째 꼭지점 O를 택하는데, OABC와 OA0B0C0이 마름모가 되도록 하자. 이 두 마름모를 O를 중심으로 회전시키다가 BB0 = 1이 되도록 하여 정지시킨다. 이제 O를 택할 때만 보면 되는데, A, C, A0, C0 중에 한 점이라도 택하면 O와 단위 거리에 있게 되므로,

나머지 두 점으로 B, B0을 택할 때만 보면 되고, 이 때에는 BB0 = 1이므로 역시 성립한다. ¤

66. 연속하는 10개의 정수들의 집합에는 다른 모든 원소와 서로소인 원소가 항상 존재함을 보여라. (예를 들어, 집합 f114; 115; : : : ; 123g에서는 119와 121이 그러한 원소이다.) (아일랜드 2000-9)

증명 2, 3, 5, 7의 배수가 아닌 수가 존재한다는 것과 동치(?). 연속한 5개의 홀수 중에는 3의 배수는

최 2개, 5의 배수는 최 1개, 7의 배수도 최 1개이므로, 2, 3, 5, 7의 배수가 아닌 수가 적어도 하나는 항상 있다. 고중조에 아마 이것을 직접 질문하는 문제가 있었던 듯. 출전 비교하여 적절히 조치할 것.¤

67. a, b, c가 서로 다른 유리수일 때,1

(b¡ c)2+

1

(c¡ a)2+

1

(a¡ b)2이 어떤 유리수의 제곱임을 보여라.

(스웨덴 1976-3)

증명 (x; y; z) = (b¡ c; c¡ a; a¡ b) 로 치환. x+ y+ z = 0 이므로 x2 + y2 + z2 = ¡2(xy+ yz+ zx).

1x2+ 1

y2+ 1

z2¡³1x+ 1

y+ 1

z

´2= ¡2

³1xy+ 1

yz+ 1

zx

´= ¡2x+y+z

xyz= 0 ¤

68. a, b, c가 삼각형이 세 변의 길이일 때, a2 + b2 + c2 < 2(bc+ ca+ ab) 임을 보여라.(플란더즈 예선 1998/1999 1차)

풀이 a2 + b2 + c2 < a(b+ c) + b(c+ a) + c(a+ b) 로 증명끝. }

69. 연속한 세 자연수를 세 변의 길이로 하는 한 삼각형이 있다. 한 중선이 한 각의 이등분선과 직교할 때, 각변의 길이를 구하여라. (몰도바 1997 최종-y7-2)

풀이 AM이 그 중선, BE가 각의 이등분선이라 하면, AB = BM =MC 이므로 AB = x, BC = 2x.

2x¡ x = 1 or 2. x = 2 일 때 2, 3, 4만 답. }

70. k는 양의 짝수이다. 1부터 k ¡ 1까지의 정수들을 늘어놓는데, 어떤 연속한 몇 개의 수의 합도 k의 배수가되지 않도록 할 수 있음을 보여라. (Towns 1989봄 JA4)

71. 각각 임의의 개수의 공을 넣은 두 개의 항아리가 있다. 우리에게는 다음과 같은 두 종류의 시행만이 허용되어 있다: (a) 두 항아리에서 같은 개수의 공을 동시에 덜어낼 수 있고, (b) 어느 한 항아리의 공의 개수를 두 배로 만들 수 있다. 이 때, 이런 시행을 유한 번 하여 두 항아리를 싹 비워낼 수 있음을 증명하여라.

(인도지역예선 1991-4)

증명1 공의 개수가 같으면 바로 비워내면 되므로, 공의 개수가 다를 때만 생각하자. 공의 개수가 적은

쪽을 계속 2배 하여 많은 쪽을 넘기 직전까지 만들자(그럼 많은 쪽의 절반보다는 많다). 그런 다음 양쪽에서 적당히 같은 개수를 덜어내어 적은 쪽이 큰 쪽의 정확히 절반이 되도록 만들자. 이제 적은 쪽을 2배한 후 양쪽을 동시에 덜어내면 된다. ¤

증명2 공의 개수가 적은 쪽이 1개가 되도록 양쪽에서 같은 개수만큼 덜어내자. 그런 다음 1개를 2개로

만들고, 다시 양쪽에서 1개씩 덜어내는 것을 반복하자. 그럼 언젠가는 양쪽다 1개가 되고, 그것을 동시에 덜어내면 된다. ¤

72. 어느 달의 첫날에 한 가게가 10가지 상품을 모두 같은 가격에 내놓았다. 각 상품의 가격은 매일 두 배 또는 세 배로 뛴다. 다음 달의 첫날엔 모든 상품의 가격이 다 달랐다. 최 가격과 최소 가격의 비가 27보다큼을 증명하여라. (Towns 1992봄 JO1)

증명 해당하는 한 달을 m일이라 하자. 처음의 가격을 a라 하면 m일 동안 매일 2배 혹은 3배가 되므

로 한 달 후에 가능한 가격은 2m¡k3ka, 즉 2m( 32)ka꼴들이 된다(0 · k · m). 한 달 후 10가지 상품의

가격이 모두 다르게 되었으므로 k로 서로 다른 10가지를 고른 것이다. 그런 k 중 가장 작은 것과 큰 것은 적어도 9 차이가 나므로, k가 가장 작은 상품과 가장 큰 상품은 가격의 비가 ( 3

2)9 이상 차이가 나게

된다. 즉, ( 32 )9 > 27 을 보이면 되고, 이것은 36 > 29, 즉 729 > 512 와 동치이므로 성립한다. ¤

Page 280: 실전수학올림피아드 1400제 해답

280 종합문제

73. a2 + b2 = c2 + 3 이 무한히 많은 정수해 (a; b; c)를 가짐을 증명하여라. (이탈리아 1996-2)

증명 c¡ a = 1 일 때만 생각해보기로 한다면... (a; b; c) = (2k2 ¡ 2; 2k; 2k2 ¡ 1) 로 이것만 해도 무한

히 많음. ¤

74. 첫항은 2000으로 시작하고, 각각의 n에 해 n번째 항의 각 자릿수의 10승의 합을 n + 1번째 항으로 하는 수열이 있다. 이 무한수열에서 어떤 두 항은 같음을 증명하여라. (몰도바 2000 최종-y9-8)

증명 N ¢ 9N < 10N 을 만족하는 자연수 N이 존재. 그럼 첫항이 N자리 이하의 수이면 계속 N자리

이하의 수이므로 무한비둘기집. ¤

75. 어떤 삼각형의 세 변의 길이가 등차수열을 이루고, 세 높이도 또다른 등차수열을 이룬다. 이 삼각형은 정삼각형임을 보여라. (남미 1988-1)

증명 세 변을 a, b, c라 하면 세 높이는 ka, kb, kc... ¤

76. 파란 눈의 사람들 중 직모(곱슬머리가 아닌 머리카락)인 사람의 비율은 모든 사람들 중 직모인 사람의 비율보다 큼이 알려져 있다. 직모인 사람들 중 파란 눈의 비율과 모든 사람들 중 파란 눈의 비율은 어느 쪽이 큰가? (Towns 1988가을 JO1)

풀이 집합적{벤다이어그램으로 생각해보면 전자가 항상 큼. }

77. 모든 자연수 n에 해 다음의 수가 항상 1998로 나누어떨어짐을 보여라.

E = 666n + 648n + (¡1)n+1684n (몰도바 1998 최종-y10-5)

증명 1998 = 2 ¢ 33 ¢ 37. 2의 배수임은 자명. mod 27로 E ´ (¡9)n + 0 ¡ (¡1)n9n = 0 이므로 27의

배수. mod 37로 E ´ 0 + (¡18)n ¡ (¡1)n18n = 0 이므로 37의 배수. ¤

78. 어느 자동차의 가격은 네 자리의 자연수이고, 그 가격표의 숫자들은 (전자시계에 보여지는 것처럼) 디지털 방식으로 표시되어 있었다. 판매원이 다른 곳을 보는 동안 어느 구매자가 그 가격표를 뒤집어 놓았고,

그래서 1626만큼 싸게 그 자동차를 샀다. 원래 자동차의 가격은 얼마였을까? (플란더즈 2004-J3/S3)

풀이 거꾸로 뒤집었을 때도 숫자로 보이는 것은 다음의 것들뿐이다.

1 2 5 6 8 9 0

여기서 6과 9는 뒤집으면 서로 바뀌고, 나머지는 뒤집어도 자기 자신과 같다. 처음 수에서 나중 수를 뺐을 때 1의 자리가 6이 되려면

(8; 2); (6; 0); (5; 9); (2; 6); (1; 5)

처음 수에서 나중 수를 뺐을 때 1000의 자리는 1 또는 2가 줄어들어야 하므로, 위의 경우들은 뒤집은

(2; 8); (0; 9); (6; 5); (9; 2); (5; 1)

들중에서 해당되는 것은 (6; 5)뿐이다. 따라서, 구하는 수는 6ab5꼴이다. a와 b에 해서도 비슷한 과정으로 경우를 찾아보면 6815만이 유일하게 만족하는 해임을 발견할 수 있다. }

79. 아래에 있는 § 기호를 각각 ¡ 혹은 +로 바꾸어서 아래 식이 성립되도록 할 수 있는가?

§1§ 2§ 3§ 4 ¢ ¢ ¢ § 96 = 1996

또한, 최 몇 개의 § 기호가 +로 바뀔 수 있는가? (IMTS R19-1)

풀이 다 더하면 4656이고, 가급적 큰 수의 부호를 -로 하는 것이 가장 유리한 방법. }

80. 자연수 a, b, n이 주어져 있는데 k 6= b 인 모든 자연수 k에 해서 b¡ k j a¡ kn 이라고 한다. a = bn 임을 증명하여라. (소련 1964-11)

Page 281: 실전수학올림피아드 1400제 해답

5.1 종합 중급문제 281

증명 k = b+ q 라 두면 q j a ¡ bn 이 됨. a ¡ bn 은 상수인데 0 아닌 임의의 정수 q의 배수가 되려면

0일 수 밖에 없음. ¤

81. 8명이 토너먼트 경기를 한다. 모든 사람은 다른 모든 사람들과 한번씩 경기를 한다. 경기의 승자는 1점,

패자는 0점, 비긴 경우 각각 0.5점씩 얻는다. 경기가 끝난 후 결과는 모든 사람들이 서로 다른 점수를 얻었고 높은 순서 로 했을 때 2등인 사람은 점수가 가장 낮은 네 명의 점수를 더한 합과 같았다. 3등과 7등의 경기 결과는 각각 몇 점인가? (소련 1963-2)

82. 정사각형을 그림처럼 5개의 직사각형으로 분할하였다.

영역 1, 2, 3, 4의 넓이가 모두 같다면 5는 정사각형임을 보여라. (Towns 1985봄 JA4)

증명 a(1 ¡ b) = b(1 ¡ c) = c(1 ¡ d) = d(1 ¡ a) 로 두고, WLOG a · b 라 하면, 1 ¡ b ¸ 1 ¡ c, 즉

b · c. 비슷하게 하여 a · b · c · d · a. 다 등호성립해야 함. ¤

83. 원에 내접하는 홀수각형의 모든 내각의 크기가 서로 같을 때, 이 다각형은 정다각형임을 증명하여라.(폴란드 1967 3차-6)

증명 꼭지점들로 분할된 원주의 호를 생각할 때, 이웃한 두 호의 길이의 합이 항상 일정. 간단한 연립

방정식으로 모든 호의 길이가 일정. ¤

84. m3 = n3 + n 의 모든 정수해 m, n을 구하여라. (스웨덴 1969-1)

풀이1 n과 n2 + 1 이 서로소이므로 각각 세제곱수. 그럼 n2도 세제곱수인데, 그럼 두 세제곱수의 차

이가 1인 경우이므로 n2 = 0 일 때뿐. }

풀이2 m과 n의 부호가 같아야 함을 알 수 있으므로, n3 < m3 < (n+ 1)3 식의 부등식법도 가능. }

85. 체스판에서 임의로 한 칸을 택하였다. 이 칸에서 모든 검은 칸까지의 중심거리의 제곱의 합을 a라 하자.

마찬가지로, 모든 흰 칸까지의 중심거리의 제곱의 합을 b라 하자. a = b 임을 증명하여라.(Towns 1987봄 SO4)

증명 2£ 2 의 두 흰 칸과 두 검은 칸에 이르는 거리제곱합이 서로 같음. ¤

86. M은 A = f1; 2; : : : ; 50g 의 부분집합으로, M의 서로 다른 어떤 두 원소를 합해도 7로 나누어지지 않는다. M은 최 몇 개의 원소를 갖는가? (몰도바 1997 최종-y7-4)

풀이 mod 7로 0인 것은 1개. mod 7로 (1,6), (2,5), (3,4)인 것은 동시에 가질 수 없으므로 어느 한

쪽(1, 2, 3인 것들)만 다 가질 때 최 8 + 7 + 7 + 1 = 23개. }

87. 4ABC에서 A를 지나는 직선과 B를 지나는 직선을 하나씩 그려 삼각형을 네 영역으로 분할하였다. 이중 세 영역의 넓이가 같으면 그 중 하나는 사각형임을 보여라. (Towns 1986봄 J1)

증명 세 영역이 모두 삼각형이면 두 체바선은 서로를 이등분. 그럼 평행사변형이 발견되어 AC k BC

가 되므로 모순. ¤

88. p는 소수이다.m

n+1

p2=

m+ p

n+ p를 만족하는 서로소인 두 자연수 m, n을 모두 찾아라. (스웨덴 1973-4)

풀이 분모 넘겨서 정리하면 (n¡m)p3 = n(n+p). n은 n¡m과 서로소이므로 n j p3. n = 1; p; p2; p3일 때 각각 풀면 됨. ¢ ¢ ¢ 답 p = 2일 때는 (m;n) = (1; 2), (1; 4). p > 2일 때는 (m;n) = (p2¡p¡1; p2).}

Page 282: 실전수학올림피아드 1400제 해답

282 종합문제

89. 좌표평면의 원점에서 벼룩이 뛰기 시작하는데, 모두 40번을 뛰고, 각각의 n에 해 n번째 뜀에서 n2=2의거리를 뛰고, 처음부터 차례 로 동, 북, 서, 남, 동, 북, 서, 남, ... 의 방향으로 뛴다. 뜀을 모두 마쳤을 때,

벼룩은 원점에서 얼마나 떨어져있는가? (AIME 1993-2)

풀이 x = (12¡32+52¡72+¢ ¢ ¢¡392)=2 = ¡4(1+3+5+¢ ¢ ¢+19) = ¡400. y = (22¡42+62¡¢ ¢ ¢+402)=2 = ¡2(3+7+11+ ¢ ¢ ¢+39) = ¡420. 그럼 원점으로부터의 거리는 20

p202 + 212 = 20 ¢29 = 580

¢ ¢ ¢ 답 }

90. A와 B가 철물점에 있다. 그 가게는 열쇠를 구별하기 위한 컬러 열쇠덮개를 팔고 있었다. 다음은 A, B의화이다.

A 네 열쇠들에 덮개를 달아주려는 거야?

B 그러고 싶지만, 내 열쇠는 8개인데 여기는 색깔이 7가지뿐이야.

A 으응, 하지만 노란 열쇠 다음의 빨간 열쇠와 파란 열쇠 다음의 빨간 열쇠가 다르다는 것은 구분할 수있잖아.

B `다음에' 또는 `세 열쇠 건너' 같은 말은 조심해야 할걸? 열쇠들은 둥글게 배열되어 있고 열쇠고리는 돌릴 수도 있잖아.

A 그래도 8개의 색깔은 필요 없어.

문제: 모든 열쇠에 덮개를 달아줄 때, n개의 열쇠를 구별하기 위한 색깔은 최소 몇 가지가 필요한가?(캐나다 1984-2)

풀이 필요한 색깔의 수를 c(n)이라 하자. c(1) = 1 임은 자명하고, n ¸ 2 이면 한 가지 색깔로는 불충

분하므로 c(n) ¸ 2 임도 자명하다. c(2) = 2 면 충분하다는 것도 자명하다. n = 3; 4; 5 일 때는 두 가지색으로는 어떻게 해도 항상 거울 칭이 생긴다는 것을 모든 경우들을 검토해보면 금방 알 수 있다. 그리고, 이웃한 어느 두 열쇠에 각각 빨강, 파랑 덮개를 달고 나머지 모든 열쇠에 노랑 덮개를 달면, `빨간 열쇠로부터 파란 열쇠의 방향으로 몇 번째' 라고 모든 열쇠를 구분할 수 있다. 즉, c(3) = c(4) = c(5) = 3.

n ¸ 6 일 때에는 어느 이웃한 네 열쇠를 순서 로 빨강, 빨강, 파랑, 빨강으로 하고 나머지 모든 열쇠를파랑으로 하면, `빨간 열쇠 2개짜리로부터 빨간 열쇠 1개짜리의 방향으로 몇 번째' 라고 모든 열쇠를 구분할 수 있다. 따라서, c(6) = c(7) = ¢ ¢ ¢ = 2. }

91. 삼각형 ABC에서 각 A의 이등분선과 체바선(꼭지점과 변의 한 점을 잇는 선분) BD, CE가 삼각형 내부의 한 점 P에서 만난다. AB = AC 일 때, 또 그 때만 사각형 ADPE가 내접원을 가짐을 보여라.

(중미 2007-2)

증명 사각형이 원에 외접할 필요충분조건은 변의 합이 같음(듀란드의 정리던가). 그럼 바로 가는 방

향은 당연하고. 역은 AP에 한 칭성을 생각하면 역시 당연함. ¤

92. 한 화물열차가 모스크바에서 x시 y분에 출발하여 사라토바에 y시 z분에 도착했다(당일이 아닐 수도 있다). 출발에서 도착까지 걸린 시간은 z시간 x분이었다. 이것이 가능한 x의 값을 모두 찾아라.

(러시아 1995 최종-y9-1)

풀이 k일 지난 후도 가능한 문제로 생각한다면 (60x + y) + (60z + x) + 1440k = 60y + z, 61x =

59(y ¡ z) + 1440k. 여기서 2x ´ 24k (mod 59), 즉 x ´ 12k (mod 59). `z분'의 표현이 가능하려면z < 60, 즉 z시간 x분은 3일 미만이므로 k · 3. `x시'의 표현이 가능하려면 x < 24 이므로 x = 0; 12 만가능. x = 0 일 때: k = 0, y = z 이면 항상 가능. x = 12 일 때: k = 1, z = y + 12 이면 항상 가능. }

93. a, b, c는 한 자리의 자연수이다. 함수 f(x) = ax2 ¡ bx+ 2c 가 f(1); f(5) > 0, f(2); f(4) < 0 을 만족할때, a, b, c의 값을 구하여라. (2000 교육청경시)

Page 283: 실전수학올림피아드 1400제 해답

5.2 종합 고급문제 283

풀이 주어진 조건을 식으로 표현하면 a¡b+2c > 0; 25a¡5b+2c > 0; 4a¡2b+2c < 0; 16a¡4b+2c < 0이다. 이들을 연립하면 b ¡ 5a > 0을 얻고 a,b가 모두 한자리 수이므로 a는 1일수밖에 없고 b는 5보다커야한다. a에 1을 입해보면 2c > 5b¡ 25도 얻고 이를 이용하면 b는 6,7,8중에 하나이다. b에 6,7,8을

입해서 조사해보면 (a,b,c) = (1,6,3)이 유일한 해이다. }

94. 어떤 회사에 좀 이상하게 작동하는 전화 시스템이 있다. 이 회사의 내선은 3자리로 이루어져있고, 누군가내선으로 전화를 걸면 종종 이 시스템은 이웃한 자릿수 둘을 서로 바꿔서 인식한다. 즉, 누군가 x318로 전화하면 x318, x138 혹은 x381로 연결하고, x044에서 받은 전화는 원래 x404나 x044에 걸었던 전화일 수있다는 말이다. 회사에서는 이 시스템을 교체하지 않고, 신에 컴퓨터를 한 더 두어서 잘못 걸린 번호를 찾아내어 그 두 자리를 다시 원래 로 바꾸는 기능을 하도록 하기로 했다. 그런데 추가된 컴퓨터와 함께 이 시스템이 정상적으로 작동하려면 일부 3자리 전화번호는 사용하지 않도록 조치해야했다. 이 계획에따를 때 회사가 가질 수 있는 3자리 전화번호는 최 몇 개인가? (IMTS R44-4)

풀이 2¡103

¢+ 10P2 +

¡101

¢= 240 + 90 + 10 = 340 ¢ ¢ ¢ 답 }

95. 평면에 직선 r과 이 직선 밖의 두 점 A, B가 있다. A와 B에서 r에 내린 수선의 발을 각각 R과 S라 하고,AB의 중점을 M이라 하자. 삼각형 ARM과 BSM의 외접원이 합동임을 보여라. (이탈리아 2002-3)

증명 AM = BM 이고 \ARM과 \BSM은 (A, B의 r에 한 위치관계에 따라) 같거나 혹은 서로 보

각. ¤

96. 다음을 만족하는 정수 a, b, c를 모두 구하여라. (헝가리 1965-1)

a2 + b2 + c2 + 3 < ab+ 3b+ 2c

풀이 양변에 4를 곱해 정리하면 (2a ¡ b)2 + 3(b¡ 2)2 + 4(c¡ 1)2 < 4. 우선 c = 1 이고, b = 1; 3 일

땐 안 되므로 b = 2뿐. a = 0; 1; 2. }

97. 세르게이는 방정식 f(19x ¡ 96x) = 0 를 만족하는 11개의 서로 다른 해를 찾았다. 좀더 열심히 풀었다면

해를 적어도 하나 더 찾을 수 있었음을 증명하여라. (상트페테르부르크 1996-15)

증명 19x ¡ 96x= t 라 두면 19x2 ¡ tx ¡ 96 = 0 의 이차방정식에서 D = t2 + 4 ¢ 19 ¢ 96 > 0 이므로

x는 항상 둘씩 짝을 이뤄 같은 t 값을 가짐. ¤

98. 십진법에서 뒤로부터 읽어도 자신과 똑같은 자연수를 대칭수라 부르자. 모든 칭수들을 증가하는 순서로 나열한 수열을 (xn)이라 하자. xn+1 ¡ xn 의 약수가 될 수 있는 소수를 모두 찾아라.

(폴란드 1995/1996 1차-2)

풀이 xn+1 ¡ xn 은 11 ¢ 10k꼴 또는 10k꼴 또는 2. 따라서, ¢ ¢ ¢ 답 2, 5, 11 }

99. 정십이면체의 몇몇 꼭지점에 표시를 하여 각 면이 표시된 꼭지점을 적어도 하나씩은 갖도록 하려고 한다.

최소 몇 개의 표시를 하면 되는가? (Towns 1990봄 SO3a)

100. 방정식 x3 ¡ 3x+ n = 0 의 해가 모두 정수인 n의 값을 모두 구하여라. (이탈리아 2002-4)

풀이 세 근을 a, b, c라 하면 a + b + c = 0, ab + bc + ca = ¡3. 여기서 a2 + b2 + c2 = 6 이고 이

것을 만족하는 fa; b; cg = f§1;§1;§2g. a + b + c = 0 이려면 f¡1;¡1; 2g 또는 f1; 1;¡2g이다. 그럼

n = ¡abc = §2 ¢ ¢ ¢ 답 }

5.2 종합 고급문제

1. 주어진 삼각형과 같은 넓이를 갖는 정삼각형을 자와 컴퍼스만으로 작도하는 방법을 서술하여라.(영국 1969-10)

Page 284: 실전수학올림피아드 1400제 해답

284 종합문제

풀이 (한석원)

먼저 x, y를 작도할 수 있으면pxy를 작도할 수 있음은 잘 알려져있다(지름 x + y인 원에서의 방멱을

이용하면 됨). 임의의 삼각형을 PQR이라 하자. 점 P를 지나고 QR에 평행한 직선을 작도한 뒤 그 직선위에 점 S를 \SQR = 60± 를 만족하도록 잡는다. 그러면 SP와 QR은 평행하므로 [SQR] = [PQR]이

다([XY Z]는 삼각형 XY Z의 넓이). 이 때 [SQR] = 12¢ SQ ¢QR ¢ sin 60± =

p34¢ SQ ¢QR 이다. 이 때,

한 변이pSQ ¢QR 인 정삼각형을 작도하면 그 넓이는

p34¢ SQ ¢QR = [SQR] = [PQR] 이 된다. }

2. (x1; y1; z1)과 (x2; y2; z2)는 모든 정수쌍 (m;n)에 해, x1m + y1n + z1 과 x2m + y2n + z2 중 적어도하나는 짝수가 되는 실수쌍이다. 두 순서쌍 중 적어도 하나는 정수로 이루어져 있음을 증명하여라.

(헝가리 1950-3)

증명 각각의 m = 0; 1; 2 에 해, n 2 f0; 1; 2g 중 둘 이상의 n에 해 짝수가 되는 식이 하나씩 있다.

그럼 어떤 식은 이런 m 2 f0; 1; 2g 중 둘 이상에 해 선택된다. 그 식을 WLOG x1m+ y1n+ z1 라 하고, m = a; b 에 해 각각 n = c; d; c; e 일 때 짝수가 된다고 하자. 그럼 (m;n) = (a; c); (a; d) 인 두 식을 빼면 (c¡ d)y1 = 짝수 인데 jc¡ dj = 1 또는 2 이므로 y1 = 정수. 마찬가지로 (m;n) = (a; c); (b; c)

인 두 식을 뺐을 때 x1 = 정수. 그럼 z1도 자동적으로 정수. ¤

3. 5개의 원이 주어져 있다. 어떤 네 원을 골라도 공통점이 있다면, 5개의 원의 공통점이 있음을 증명하여라.(소련 1963-1)

증명 귀류법. 각각의 원에 해 그 원만 안 지나고 나머지 네 원은 모두 지나는 점이 존재. 다섯 원을

a, b, c, d, e라 하고, 자기를 제외한 나머지 네 원이 지나는 점을 각각 A, B, C, D, E라 하자. 이 다섯 점이 각각 서로 다름은 자명. a, b는 둘다 세 점 C, D, E를 지남. 즉, CDE의 외접원을 유일하므로 a = b

동일원. 그럼 a도 b가 지나는 A를 지나므로 모순. ¤

4. 다음을 만족하는 함수 f : N! N 이 존재하는가? (호주 2004-6)

f(f(1)) = 5; f(f(2)) = 6; f(f(3)) = 4; f(f(4)) = 3; f(f(n)) = n+ 2 (n ¸ 5)

풀이 f(3) = a, f(4) = b 라 하면 f(f(3)) = 4, f(f(4)) = 3 임에서 다음과 같은 f의 응 관계가 성

립한다.3! a! 4! b! 3 (¤)

그럼 f(f(a)) = b, f(f(b)) = a 이고, 문제에서 제시된 응 관계를 살피면 이런 경우는 fa; bg = f3; 4g뿐임을 알 수 있다. a와 b 중에 어느 것이 3이든 (¤)에서 3! 3의 응이 나타나게 되고, 이것은 f(3) = 3

을 뜻하므로 f(f(3)) = 3 이 되어야 해서 모순. 따라서, 이런 함수는 존재하지 않는다. }

5. 0 < a · b · c 인 정수일 때, 다음 부등식을 증명하여라. (셈본중등고급 도전문제 3.3.1 변형)

abbcca ¸ bacbac

증명 우선 b = c 일 때 좌변 = 우변이 되어 성립한다. 이제 a와 b를 고정시켜놓고 c를 c = b; b+ 1; b+

2; : : : 이렇게 증가시키는 것을 생각해보자. 일반적으로

b ¢ (c+ 1)a

ca¸ a ¢ (c+ 1)

b

cb(2)

이 성립한다면 (1)과 (2)를 변변 곱하여 c가 하나 더 증가된 식도 계속 성립함을 알 수 있다. 따라서, 이(2)만 증명하면 되겠다. (2)을 변형하면

b

a¸µc+ 1

c

¶b¡a이고 이것은 다음과 같이 증명된다:

b

a=

b

b¡ 1 ¢b¡ 1b¡ 2 ¢ ¢ ¢

a+ 1

a¸ b+ 1

b¢ b+ 1

b¢ ¢ ¢ b+ 1

b=

µb+ 1

b

¶b¡a¸µc+ 1

c

¶b¡a¤

Page 285: 실전수학올림피아드 1400제 해답

5.2 종합 고급문제 285

6. 세 각이 120±인 볼록칠각형이 한 원에 내접하고 있다. 이 7각형의 변 중에 길이가 같은 두 변이 있음을 보여라. (유고슬라비아 1979 고1-2)

풀이 인접하지 않은 세 각이 120±이면 중심각이 360±를 넘어서 모순. 인접한 각이 120±이면 거기서

바로 삼각형 합동에 의해 건너뛴 두 변의 길이가 같음. }

7. 자릿수가 1, 3, 7, 9들로 이루어진(각각 적어도 한 번씩 나타나는) 자연수가 있다. 이 수의 각 자릿수를 적당히 재배열하여 7의 배수를 얻을 수 있음을 보여라. (폴란드 1975 3차-4)

풀이 1379, 1793, 3719, 1739, 1397, 1937, 1973은 각각 mod 7로 0, 1, 2, 3, 4, 5, 6인 수. 이 수를 A.

나머지 숫자들을 아무렇게나 재배열한 수를 B라 할 때, 104B +A 로 A를 적당히 선택해주면 됨. }

8. 총합이 1인 음 아닌 8개의 실수가 한 정육면체의 꼭지점들에 분배되어있다. 각 모서리에는 양끝점의 수의곱을 적었다. 이 12 모서리의 수를 모두 합한 값 S는 1/4을 넘을 수 없음을 증명하여라.

(레닌그라드 1987-47)

증명 정육면체의 각 꼭지점을 흑백으로,이웃한 점이 서로 다른 색이 되도록 색칠하면 모서리는 항상 흑

점 하나와 백점 하나를 잇는다. 그럼, 흑점의 합을 X, 백점의 합을 Y 라 하면 S · XY ·³X+Y2

´2= 14.

¤

9. 정삼각형 ABC에서 점 D와 E는 각각 변 AB와 AC 위의 점이다. DF , EG가 삼각형 ADE의 각의 이등분선들일 때, 삼각형 DEF와 DEG의 넓이의 합은 삼각형 ABC의 넓이 이하임을 보여라. 등호는 언제성립하는가? (주니어발칸 2001-3)

풀이 AD ¢EF+AE ¢GD · a2 임을 보이라는 것이고, (DE;EA;AD) = a(x; y; z)라 두면 z ¢ xyx+z

+y ¢xzx+y

· 1,즉 xyz(2x+y+z) · (x+y)(x+z)와 동치가 됨.이것은 AM-GM으로 (x+y)(x+z) ¸ 4xpyz

임에서pyz(2x+ y + z) · 4 를 보이면 충분하고 이건 그냥 성립. }

10. 숫자 1, 2, 3, 4들만으로 이루어진 모든 n자리의 자연수들을 생각하자. 그 중 숫자 4의 오른쪽에 3이 나타나는 경우가 없는 것들의 개수를 an이라 하자. 예를 들어 n이 6일 때, 123314와 113424는 조건을 만족하는 것들이지만 114234는 그렇지 않다. 적당한 자연수 p, q에 해 an+1 = pan + qn 이 항상 성립한다는것을 보이고, p와 q를 구하여라. (플란더즈 예선 1996/1997 1차)

¢ ¢ ¢ 답 p = q = 3

11. 임의의 정수 계수 다항식이 유한개의 정수 계수 다항식의 세제곱의 합으로 항상 나타내어질 수 있는가?(폴란드 1996 2차-1)

풀이 (¢ ¢ ¢+ ax+ b)3 = ¢ ¢ ¢+ 3ab2x+ b3 으로 일차항의 계수는 항상 3의 배수이다. 따라서, 일차항의

계수가 3의 배수가 아닌 다항식은 만들 수 없다. }

12. 평면 위에 6개의 점이 있다. 가장 멀리 떨어진 두 점 사이의 거리와, 가장 가까운 두 점 사이의 거리의 비가 적어도

p3 이상임을 증명하여라. (폴란드 1966 3차-6)

증명 120도 이상의 각이 있다는 것만 보이면. 볼록육각형일 때 vs 삼각형과 그 내부의 점의 관계가 있

을 때. ¤

13. 주어진 양의 정수 n과 p에 하여, 연립방정식

x+ py = n; x+ y = pz

이 양의 정수해 (x; y; z)를 가질 필요충분조건을 구하여라. 또 그 해는 많아야 하나뿐임을 증명하여라.(헝가리 1905-1)

힌트 pz < n < pz+1의 부등식을 얻으면 z의 값이 결정된다. x 혹은 y를 소거시키고 양변에서 1을 빼

주면, p¡ 1과 n¡ 1의 관계가 얻어진다. 그 관계만으로 x, y가 결정된다는 것을 확인하면 된다. }

Page 286: 실전수학올림피아드 1400제 해답

286 종합문제

14. 세 형제와 그 부인들이 병원을 방문했다. 각각의 사람은 병원에 정확히 한 번 방문했다. 방문자 중 몇몇은서로 방문한 시간이 겹친다고 한다. 그래서 세 명의 형제는 각각 다른 두 형제의 부인들을 병원에서 만날수 있었다고 한다. 세 명의 형제 중 한 명은 병원에서 자신의 부인을 만났음을 증명하여라.

(헝가리 1959-3)

증명 부인 X, Y , Z; 남편 x, y, z. 한 부인 X의 방문 구간이 다른 부인 Y 의 구간에 완전히 포함되는

경우가 있으면 y를 생각하면 자명. 이제 그런 경우는 없다고 하고, 그럼 가장 먼저 병원에 들어간 부인을 X, 가장 나중에 나온 부인을 Y 라 하면, z는 Z를 만나게 됨. 왜냐하면, z는 X, Y 와 모두 겹치는 부분이 있어야 하고, Z가 z와 서로소라면 X 또는 Y 에 완전히 포함될 수밖에 없어서. ¤

15. 수열 1 = a1; a2; : : : 은 임의의 k > 1 에 해 다음을 만족한다.

ak < 1 + a1 + a2 + ¢ ¢ ¢+ ak¡1

모든 자연수는 ai들 일부의 합으로 표현 가능함을 보여라. (헝가리 1960-2)

증명 a1; : : : ; ak¡1 로 1부터 a1 + ¢ ¢ ¢+ ak¡1 까지 표현 가능함을 귀납적으로 증명. ¤

16. ABC는 a, b, c를 세 변의 길이로 하는 삼각형이다. 삼각형의 각 변은 n등분되어있다. 꼭지점에서 변의각 분점까지의 거리의 제곱의 합을 S라고 할 때 다음 식의 값이 유리수임을 증명하여라. (남미 1988-4)

S

a2 + b2 + c2

증명 AH를 수선이라 할 때 AH, BH 등의 길이에 의해 피타고라스 정리로 그냥 열심히 계산하면 됨.S

a2 + b2 + c2=(n¡1)(5n¡1)

6n¤

17. n ¸ 3, k ¸ 2 인 정수라 하자. 1; n; n2; n3; : : : ; nk¡1을 맨윗줄에 적고, 이웃한 두 수의 차를 두 수의 사이한 줄 밑에 적어나간다고 하자. 다음은 (n; k) = (3; 5) 일 때의 예이다.

1 3 9 27 812 6 18 54

4 12 368 2416

어떤 n, k에 해서도, 이 역삼각형에 나타나는 수는 모두 서로 다름을 증명하여라. (IMTS R5-2)

힌트 왼쪽부터 작은 수가 차례로 나타날 걸... }

18. 삼각형 ABC에 하여 \BPA + \AQC = 90±를 만족하는 점 P , Q가 있다. 단, 두 삼각형 BAP와ACQ의 세 꼭지점은 반시계방향으로 (또는 시계방향으로) 모두 B, A, P와 A, C, Q의 순으로 놓여 있다 하자. 이 두 삼각형의 두 외접원의 교점을 N (A 6= N , 단, 두 원의 교점이 A뿐일 때는 A = N), 선분BC의 중점을 M이라 할 때, 선분 MN의 길이는 P , Q에 상관없이 항상 일정함을 보여라.

(한국 2005-J2/S2)

증명 (일산, 이동영)

Page 287: 실전수학올림피아드 1400제 해답

5.2 종합 고급문제 287

그림에서\BNC = \BNA+ \CNA = \BPA+ \CQA = 90±

따라서, N은 BC를 지름으로 하는 원 위의 점이다. 그럼

MN = BM = CM =1

2BC

두 원이 안으로 혹은 밖으로 접할 때도 비슷하게 \BNC가 직각이 됨을 보일 수 있다. 혹은 N이 점점A에 가까워질 때의 극한으로 생각해도 된다. ¤

19. (a) 3k ¡ 1 = x3 인 자연수쌍 (x; k)를 모두 구하여라.

(b) 1보다 크고 3이 아닌 정수 n에 해, 3k ¡ 1 = xn 인 자연수쌍 (x; k)는 존재하지 않음을 보여라.(이탈리아 1999-6)

풀이 일반적인 자연수 n에 해 (b)를 풀자. mod 3으로 생각하면 n은 짝수일 수 없으므로 홀수.

3k = x2m+1+1 = (x+1)(x2m¡x2m¡1+ ¢ ¢ ¢ ¡x+1) = 3a3b꼴이고 3a < 3b 이므로 x2m¡x2m¡1+¢ ¢ ¢ ¡ x + 1 ´ 0 (mod x + 1) 이어야 함. 그런데 x ´ ¡1 (mod x + 1) 이므로 이것은 2m + 1 ´ 0(mod x+1), 즉 (3a =)x+1 j n(= 2m+1) 임을 의미함. 즉, 3 j n. 그럼 xn = y3 으로 치환할 수 있고,3k = (y+1)(y2¡ y+1) 임에서 앞에서처럼 y+1 j 3 이 되므로 y = 2. 그럼 x = 2, n = 2 만 가능함. }

20. 반지름 1의 속이 꽉 찬 구 2개가 들어있는 정육면체의 한 변의 길이는 최소 얼마인가? 당신의 답이 틀림없음을 증명하여라. (플란더즈 1986-4)

증명 원래 정육면체의 표면으로부터 1만큼 안쪽에 있는 작은 정육면체 A를 생각. 그럼 두 구의 중심

은 A 안에 있고, 두 중심 사이의 거리는 ¸ 2. (a¡ 2)p3 ¸ 2 ¤

21. 호수 위를 m £ n 크기의 격자 직사각형으로 구획하여 각 격자점의 위치마다 수상 주거구역을 만들었다.

각 주거구역마다 이웃한 구역으로 연결되는 다리가 꼭 p개씩 있다. 단, `이웃하다'는 것에 각선 방향은해당하지 않으며, 이웃한 두 구역이 여러 개의 다리로 연결되어 있을 수도 있다. 임의의 구역에서 다른 어떤 구역에도 갈 수 있도록 다리를 배치할 수 있는 순서쌍 (m;n; p)를 모두 구하여라. (이탈리아 1999-5)

풀이 mn이 짝수이면 p = 2 일 때 우선 간단한 오일러회로를 만들어서 가능하고 p > 2 일 때도 도미

노 타일링처럼 둘씩 짝지어 짝마다 다리를 추가하는 방법으로 가능함. mn이 홀수이면 체스판처럼 칠했을 때 모든 다리는 검은구역과 흰구역과 같은 횟수로 만나는데, 검은구역과 흰구역의 수가 다르므로 불가능함. }

22. 다음과 같이 정의된 함수 f : (0; 1)! R 가 있다.

f(x) =

(x (x가 무리수일 때)p+1q

(x = pq, (p; q) = 1 일 때)

구간 (7=8; 8=9)에서 f의 최 값을 구하여라. (체코슬로바키아 1992-5)

풀이 무리수일 땐 89를 넘지 못하고, 유리수일 때 우선 f( 15

17) = 16

17. 이것보다 더 클 수는 없음을 보이

자. x =q¡aq 라 둘 때 7

8q < q ¡ a < 8

9q 이므로 8a < q < 9a. a = 1 일 땐 해 없고, a = 2 일 땐 앞에서

구한 것이고, a ¸ 3 일 땐 f( q¡aq) = q¡a+1

q= 1¡ a¡1

q< 1¡ a¡1

9a= 8a+1

9a=8+ 1

a9

· 8+ 139= 2527

< 1617.

}

23. a, b가 서로소이고 정a각형, 정b각형이 작도 가능하면 정ab각형도 작도 가능함을 보여라.(셈본중등고급 도전문제 4.2.2)

증명 a, b가 서로소이면 ax¡ by = §1 0 < x < b, 0 < y < a인 정수해가 있음을 보이자. a, b는 서로

소이므로 ax¡ by = §1인 자연수해 x, y는 반드시 존재한다.x = bs1 + x0, y = as2 + y0(0 < x0 < b; 0 < y0 < a)로 놓으면

ax¡ by = (abs1 + ax0)¡ (abs2 + by0) = §1

Page 288: 실전수학올림피아드 1400제 해답

288 종합문제

ab(s1 ¡ s2) + (ax0 ¡ by0) = §1

¡ab+ 1 < a¡ b(a¡ 1) · ax0 ¡ by0 · a(b¡ 1)¡ b < ab¡ 1이므로

ab(s1 ¡ s2 ¡ 1) + 1 < ab(s1 ¡ s2) + (ax0by0) < ab(s1 ¡ s2 + 1)¡ 1

ab(s1 ¡ s2 ¡ 1) + 1 < 1 < ab(s1 + s2 + 1)¡ 1

) s1 ¡ s2 = 0

이로부터 ax¡ by = §1이면 ax0 ¡ by0 = §1임을 알 수 있다.

이때, 0 < x0 < b, 0 < y0 < a이므로 ax¡ by = §1, 0 < x < b, 0 < y < a인 정수해가 존재함을 알 수있다.x0, y0가 ax¡ by = §1, 0 < x < b, 0 < y < a의 정수해라 하자. 그리고 단위원 위에 정a각형, 정b각형을 작도하여 각각의 꼭지점을 A0; A1; : : : ; Aa¡1, B0; B1; : : : ; Bb¡1이라 하자.

원의 중심을 O라 하면

\A0OAy ¡ \B0OBx =2¼

ay ¡ 2¼

bx =

ab(¡ax+ by) = §2¼

ab

이므로, 단위원에서 원주각이2¼

ab인 호를 구할 수 있다. ab개의 호를 연결하면 정ab각형이 작도 된다. ¤

24. 실수 계수 2차 다항식 p(x) = ax2 + bx+ c 가 임의의 정수 x에 해 항상 정수값을 가질 동치조건을 구하여라. (헝가리 1902-1 변형)

풀이 p(0) = c, p(1) = a + b + c, p(¡1) = a ¡ b + c 들이 모두 정수이므로, c는 정수이고, a =p(1)+p(¡1)

2¡p(0), b =

p(1)¡p(¡1)2

. p(1)+p(¡1) 과 p(1)¡p(¡1) 은 홀짝이 같으므로, a, b는 둘다 정수

이거나 아니면 둘다 홀수2 꼴이다.그리고,이것이 동치조건임을 보이기 위해 이 경우 항상 p(x)가 정수가 됨

을 확인해보자. 2a, 2b가 홀짝이 같은 정수이고 c도 정수이면, 2p(x) = 2ax2+2bx+2c ´ 2ax(x+1) ´ 0(mod 2) 이므로 p(x)는 정수이다. ¢ ¢ ¢ 답 2a, 2b가 홀짝이 같은 정수이고 c도 정수 }

별해 p(x) = Ax(x¡1)2

+Bx+C 로 변형해보자. p(0) = C, p(1) = B+C, p(2) = A+2B+C 가 모두

정수이면 A, B, C가 모두 정수이고, A, B, C가 모두 정수이면 p(x)가 항상 정수가 되므로, p(x)가 항상정수가 될 동치조건은 A, B, C가 정수라는 것이 된다. 원래의 식과 계수 비교하면 a = A

2, b = B ¡ A

2,

c = C 이므로 ¢ ¢ ¢ 답 2a, 2b가 홀짝이 같은 정수이고 c도 정수 }

25. 십진수로 나타냈을 때의 자리의 수와 서로 다른 소인수의 개수가 같은 자연수를 균형잡힌 수라고 부르자.

균형잡힌 수는 유한개뿐임을 증명하여라. (이탈리아 1999-2)

증명 2 ¢ 3 ¢ 5 ¢ 7 ¢ 11 ¢ 13 ¢ 17 ¢ 19 ¢ 23 ¢ 29 ¢ 31 > 1011 이므로 11자리 이상의 수로는 존재하지 않는다. ¤

26. 원에 내접하는 육각형 ABCDEF의 세 각선 AD, BE, CF가 원 안의 한 점에서 만나고 있다.

AB

BC¢ CD

DE¢ EF

FA= 1

임을 증명하여라. (통신강좌 1990-1-3)

풀이 4ABP » 4EDP 이므로,AB

DE=

PB

PE=

PA

PD(1). 같은 방법으로,

EF

BC=

PF

PB=

PE

PC(2) 와

CD

FA=

PD

PF=

PC

PA(3) 을 얻는다. (1), (2), (3)을 모두 곱하여,

AB

BC¢ CD

DE¢ EF

FA=

AB

DE¢ EF

BC¢ CD

FA=

PA

PE¢ PE

PC¢ PC

PA= 1:

}

Page 289: 실전수학올림피아드 1400제 해답

5.2 종합 고급문제 289

27. 볼록팔각형 ABCDEFGH의 모든 내각이 서로 같다. 그리고 변의 길이도 번갈아 건너뛰며 같다: 즉AB = CD = EF = GH 이고 BC = DE = FG = HA 이다. 이런 도형을 준정팔각형이라 한다. 각선AD, BE, CF , DG, EH, FA, GB, HC들이 이 팔각형을 몇 개의 영역으로 나눈다. 중심을 포함하는 영역을 생각하자. 이 영역이 8각형이면 명백하게 준정팔각형이다. 이 경우 이 중앙팔각형에서 같은 방식으로 각선들을 그리고, 이렇게 계속 반복하다가 중앙의 도형이 8각형이 아니면 중단한다. 이 과정이 중단되지 않고 무한히 계속될 때는 정팔각형일 때뿐임을 증명하여라. (Towns 1991가을 JA6)

증명 단계 n의 팔각형 두 변을 an, bn이라 하면 an+1 =p2 an ¡ bn, bn+1 =

p2 bn ¡ an 의 점화식

이 성립. an+1 + bn+1 = (p2¡ 1)(an + bn) 이고 0 <

p2¡ 1 < 1 이므로 두 변의 길이의 합은 점점 감

소함(0으로 수렴). 그런데 an+1 ¡ bn+1 = (p2 + 1)(an ¡ bn) 이고

p2 + 1 > 1 이므로 두 변의 길이의

차는 처음에 0이 아니었다면 점점 커짐(무한 로 발산). 따라서, 언젠가는 차가 합보다 커지게 되어 모순. 즉, 두 변의 차가 처음부터 0일 때인 정팔각형일 때만 이런 과정이 무한히 계속될 수 있다. ¤

28. 반지름이 n인 원 안에 길이가 1인 선분 4n개가 있다. 임의의 주어진 직선 L에 해, 4n개의 선분 중 적어도 2개와 만나는, L과 평행하거나 수직한 현이 존재함을 보여라. (헝가리 1968-2)

증명 각각의 선분 li를 L과 평행한 방향과 수직한 방향으로 각각 사영시킨 길이를 ai, bi라 하자.

1 = li =qa2i + b2i · ai + bi 이므로

P(ai + bi) ¸ 4n. WLOG

Pai ¸ 2n 이라 하자. 그럼 이 길이의

합이 지름 이상이므로 사영 ai들중 겹치는 부분(한 점이더라도)이 있고, 그 점을 지나는 L에 평행한 직선이 두 선분을 지남. 이와 비슷한 유형이 KMO 초기에도 나왔었고... 종종 나오는 유형임. ¤

29. n £ n 크기의 수판의 i행 j열의 칸에1

i+ j ¡ 1 을 배정하였다(1 · i; j · n). 같은 행 혹은 같은 열에서

뽑은 두 칸이 없도록 n개의 칸을 뽑으면, 그 n개의 수의 합은 항상 1보다 크거나 같음을 증명하여라.(Towns 1992봄 SA3)

증명1 n개의 수를 선택하는 방법은 유한하므로 그 n개의 수의 합이 최소가 되는 경우가 존재하고, 그

경우가 i + j ¡ 1 = n 인 각선의 칸들을 뽑은 경우임을 보이겠다. i행 j열을 (i; j)의 좌표로 나타내기로 하자. 이 최소의 경우에서 증가하는(ia < ib 이고 ja < jb 인) 두 좌표 (ia; ja), (ib; jb) 가 있다고 하자. ja와 jb를 뒤바꾼 경우와 비교해보면 다음과 같이 동치변형된다.

1

ia + ja ¡ 1+

1

ib + jb ¡ 1>

1

ia + jb ¡ 1+

1

ib + ja ¡ 1ia + ja + ib + jb ¡ 2

(ia + ja ¡ 1)(ib + jb ¡ 1)>

ia + jb + ib + ja ¡ 2(ia + jb ¡ 1)(ib + ja ¡ 1)

(ia + jb ¡ 1)(ib + ja ¡ 1) > (ia + ja ¡ 1)(ib + jb ¡ 1)iaja + ibjb > iajb + ibja

(ia ¡ ib)(ja ¡ jb) > 0

이것은 참이므로 처음의 부등식도 참이고, 그럼 두 좌표에서 열 ja와 jb를 서로 바꾸어 선택하면 합이더 작은 선택이 된다는 의미가 된다. 이것은 합이 최소가 되는 경우를 택했다는 것에 모순이고, 따라서합이 최소가 되는 경우에는 증가하는 두 좌표가 없다. 즉, 모든 두 좌표가 감소하는 관계에 있으므로,i+ j ¡ 1 = n 인 각선의 칸들을 모두 뽑은 경우일 수밖에 없다. 그 때, 1n +

1n+ ¢ ¢ ¢+ 1

n= n ¢ 1

n= 1

이므로 n개의 수의 합은 항상 1 이상이다. ¤

증명2 fi1; : : : ; ing = fj1; : : : ; jng = f1; : : : ; ng 일 때, 산술-조화평균 부등식(혹은 코시-슈바르츠 부

등식)에 의해

1

i1 + j1 ¡ 1+ ¢ ¢ ¢+ 1

in + jn ¡ 1¸ n2

(i1 + j1 ¡ 1) + ¢ ¢ ¢+ (in + jn ¡ 1)

=n2

(i1 + ¢ ¢ ¢+ in) + (j1 + ¢ ¢ ¢+ jn)¡ n

=n2

n(n+1)2

+n(n+1)2

¡ n=

n2

n2= 1

임이 쉽게 확인된다. ¤

Page 290: 실전수학올림피아드 1400제 해답

290 종합문제

30. (1) 삼각형 ABC에서, 각 변 BC, CA, AB 위의 점 X, Y , Z를

BX · XC; CY · Y A; AZ · ZB

가 만족되도록 잡는다. 이 때, 삼각형 XY Z의 넓이의 4배가 삼각형 ABC의 넓이보다 작지 않음을증명하여라.

(2) 삼각형 ABC에서, 점 X, Y , Z는 각각 변 BC, CA, AB 위의 점이고, (1)에서와 같은 부등식에 의한 제한은 없다고 하자. 삼각형 AZY , BXZ, CY X 중 어느 하나는 삼각형 XY Z보다 넓이가 크지않음을 보여라. (Putnam 1973)

증명 (1) 변 AB, BC, AC의 중점을 각각 D, E, F라 하자. j4DEF j = 14j4ABCj. DE==AC 이므로

ZX는 DE와 만난다. 이는 곧 F에서 ZX에 이르는 거리가 Y 에서 이르는 거리보다 작음을 의미하므로j4XY Zj ¸ j4ZXF j. EF==AB 이므로 FX는 AB와 만난다. 이는 곧 D에서 EX에 이르는 거리가 Z에서 이르는 거리보다 작음을 의미하므로 j4ZXF j ¸ j4DFXj. EF==BC 이므로 j4DFXj = j4DEF j.따라서,

j4XY Zj ¸ j4DEF j = 1

4j4ABCj

(2) 다음과 같이 두 가지 경우에 해서 생각해보자.

i) 4ADF , 4BDE, 4EFC가 각각 X, Y , Z를 하나씩 포함하는 경우:(1)에서 j4XY Zj ¸ 1

4j4ABCj 이다. 모든 삼각형의 넓이가 4XY Z의 넓이보다 크다면

j4AZY j+ j4BXZj+ j4CY Xj+ j4XY Zj > 4j4XY Zj = j4ABCj

이것은 모순. 그러므로 어느 하나는 삼각형 XYZ보다 넓이가 크지 않다.

ii) 4ADF , 4BDE, 4EFC 중 하나가 X, Y , Z를 두 개 포함하는 경우:

일반성을 잃지 않고 AZ · AD, AY · AF 라 하고 AX와 ZF가 H에서 만난다고 하면, AH ·12AX. H가 AX와 DF의 교점보다 A에 가깝기 때문에 AH · XH 이다.

j4AZY jj4XY Zj =

AH

XH· 1

따라서, j4AZY j · j4XY Zj.그러므로, 문제가 언제나 성립한다. ¤

31. a, b, n, m은 2 이상의 자연수들이고, an ¡ 1 과 bm + 1 은 소수이다. a, b, n, m에 한 정보를 알아낼 수있는 만큼 알아내어라. (플란더즈 1999-4)

풀이 a ¡ 1 j an ¡ 1 이므로 a ¸ 3, n ¸ 2 이면 an ¡ 1 은 합성수. 따라서, (a; n) = (2; p) or

(p+ 1; 1)꼴. (p는 소수) m이 홀수이면 b+ 1 j bm + 1 이고, m이 홀수인 소인수를 가질 때도 비슷. 따라서, (b;m) = (?; 2k) or (p¡ 1; 1)꼴. }

32. 다섯 개의 양의 실수가 주어져 있는데 여기서 세 수를 어떻게 골라도 항상 삼각형의 세 변의 길이를 이룰수 있다고 한다. 그럼 세 수를 잘 고르면 예각삼각형의 세 변의 길이를 이룸을 증명하여라.

(독일BW 1971 1차-3)

33. 코끼리 15마리가 한 줄로 섰다. 각각의 몸무게는 정수 kg이다. 각 코끼리(오른쪽 끝의 코끼리는 제외)의몸무게에 그 오른쪽에 이웃한 코끼리의 몸무게의 2배를 합하면 항상 정확히 15톤이 된다고 한다. 각 코끼리의 몸무게를 구하여라. (Towns 1990봄 JA3)

풀이 제일 오른쪽에서 n번째 코끼리의 무게를 an킬로그램이라 하자. 문제의 조건에 의해 an+1 =

15000¡2an 의 점화식이 성립. an+1¡5000 = ¡2(an¡5000)으로 정리하면 an¡5000 = (¡2)n¡1(a1¡5000). 따라서, a14 = 5000¡ 213(a1 ¡ 5000) 이고 a15 = 5000+ 214(a1 ¡ 5000). 213; 214 > 5000 이므로 a1 > 5000 이면 a14 < 0 이 되어 곤란하고, a1 < 5000 이면 a15 < 0이 되어 곤란. 따라서, a1 = 5000

일 수밖에 없고, 그럼 a2; a3; : : : 도 차례로 모두 5000이 된다. ¢ ¢ ¢ 답 모두 5000 kg }

Page 291: 실전수학올림피아드 1400제 해답

5.2 종합 고급문제 291

34. 다음 방정식의 음이 아닌 정수해를 모두 구하여라. (미국 1987-1)

(a2 + b)(a+ b2) = (a¡ b)3

풀이 양변을 전개하여 b에 한 이차방정식으로 고치면,

2b2 + (a2 ¡ 3a)b+ (3a2 + a) = 0

가 된다. 근의 공식에 의해

b =¡(a2 ¡ 3a)§p(a2 ¡ 3a)2 ¡ 8(3a2 + a)

4

이다. a가 정수일때 b도 정수가 될 조건을 찾자.

근호 안의 내용을 풀어서 전개하면 a4¡6a3¡16a2¡8a가 되고 이것은 a(a¡8)(a+1)2으로 인수분해 된다. 즉, a(a¡8)이 완전제곱수가 되면 b도 정수가 된다. (a; a¡8) = (a; 8)이고 여기서 (a; 8) = 1; 2; 4; 8의네가지 경우만이 존재한다.

만일 (a; 8)이 1이면, a와 a¡8이 서로소이므로 둘다 제곱수여야만 하고 a = 9인 경우 밖에 없다. (a; 8)이2이면 a(a¡ 8) = 22(a=2)(a=2¡ 4)이고 a=2와 a=2¡ 4가 서로소이므로 둘다 제곱수여야 한다. 이런 a는존재하지 않는다. 4; 8인 경우에도 같은 논의를 적용하면, a = 8과 a = 9만이 가능하다.

것을 위의 b에 관한 식에 적용하면 가능한 경우는 (9;¡6); (9;¡21); (8;¡10) 밖에 없다. }

35. 삼각형 ABC의 변 BC, CA, AB의 바깥쪽에 점 P , Q, R을 잡자. \BPC +\CQA+\ARB = 180± 를만족하면 4BPC, 4CQA, 4ARB의 외접원이 한 점에서 만남을 증명하여라.

증명 BPC의 외접원과 CQA의 외접원의 교점을 F라 하자.

B;P;C; F는 한 원 위에 있으므로 \BFC = 180¡ \P .C;Q;A; F도 한 원 위에 있으므로 \CFA = 180¡ \Q. 따라서,

\AFB = 360¡ (180¡ \P + 180¡ \Q) = \P + \Q = 180¡ \R:

따라서, A;R;B; F도 한 원 위에 있다. ¤

36. 어떤 나라에 n개의 도시와 m개의 도로가 있다. 각 도로는 서로 다른 두 도시를 직접 연결하고, 두 도시 사이에는 많아야 하나의 도로가 있다.

(1) m ¸ (n¡ 1)(n¡ 2)2

+ 1 이면 이 나라의 모든 도시는 도로로 연결되어 있음(몇 개의 도시를 거쳐

서로 오갈 수 있음)을 보여라(n ¸ 2).

(2) m =(n¡ 1)(n¡ 2)

2이면서 모든 도시가 연결되지는 않은 가능한 도로망을 모두 구하여라.

(통신강좌 1997-14-10)

풀이 (1) 연결되어 있지 않다면 n개의 점들이 두 집합 A;B로 나뉘어져 각 집합은 어느 두 점도 서로

변을 공유하지 않는다. (A;B 6= Á) 이때, n(A) = l이라면, n(B) = n¡ l이고, 변이 최 가 될 수 있는것은 각 A;B집합의 점들이 완전그래프를 형성할 경우, 즉, 변의 수의 최 값은,µ

l2

¶+

µn¡ l2

¶=

l(l ¡ 1)2

+(n¡ l)(n¡ l ¡ 1)

2

=n2 ¡ n¡ 2nl + 2l2

2¢ ¢ ¢ ¢ ¢ ¢ 1°

(n¡ 1)(n¡ 2)2

¢ ¢ ¢ ¢ ¢ ¢ 2°

라면, 2°¡ 1° = (l ¡ 1)(n¡ 1¡ l)1 · l · n¡ 1이므로 2°¡ 1° ¸ 0) 변의 수는

(n¡ 1)(n¡ 2)2

이하 즉, m ¸ (n¡ 1)(n¡ 2)2

+ 1에 모순.

) 연결되어 있다.

(2) (n ¡ 1)개의 점들은 완전 그래프를 이루고, (변의 개수는(n¡ 1)(n¡ 2)

2가 된다.) 나머지 한 점은

아무점과도 연결되어 있지 않는 경우, (1)의 증명에서 2°¡ 1° ¸ 0의 등호조건에 의해, 이 경우 뿐임을 알 수 있다.

}

Page 292: 실전수학올림피아드 1400제 해답

292 종합문제

37. (a) x1; : : : ; xn은 ¡1; 0; 1 만으로 이루어진 수열이다. 모든 xixj (1 · i < j · n) 를 더한 값의 가능한

최소값은 얼마인가?

(b) 만일 xi의 값이 0 · jxij · 1 (1 · i · n) 인 실수라면 (a)에서 구한 값이 달라지겠는가?(소련 1965-1)

풀이 (b) 한 변수에만 주목하면 그 변수에 한 일차식이므로 최 /최소의 경우는 구간의 양끝점 중에

서만 고려하면 충분. 따라서, 모든 변수가 ¡1, 1일 때만 고려하면 충분하고, 그럼 (a)가 됨. (a) 1이 a개,¡1이 b개 라고 하면 구하는 값은 X = 1

2(S2 ¡ (x21 + ¢ ¢ ¢+ x2n)) =

12((a¡ b)2 ¡ n) 이므로 a와 b가 가장

비슷할 때 최소. }

38. a, b는 자연수이다. a = b 일 때, 또 그 때만 a2 + b2 이 ab로 나누어짐을 보여라. (몰도바 1997 최종-y7-7)

증명 a2 + b2 = kab 라 두고 판별식이 제곱수가 됨을 이용. ¤

39. 직선 l에 의해 결정된 반평면 위에 세 개의 직각삼각형이 각각 한 옆변(빗변이 아닌 변)을 l 위에 두고 놓여 있다. 이 세 삼각형과 만나는 현의 길이가 모두 같은, l과 평행한 직선이 존재한다고 한다. 각 삼각형을돌려서 다른 옆변이 l 위에 놓이도록 해도, 이 세 삼각형과 만나는 현의 길이가 모두 같은 l과 평행한 직선이 존재함을 보여라. (러시아 1993 4차-y9-6)

증명 각각의 직각삼각형에서 현과 그 현에 이르는 높이로 이루어진 내접직사각형이 모두 합동임. ¤

40. 12시 정오에, Anne, Beth, Carmen 세 사람이 300미터 길이의 원형 트랙을 같은 지점에서 출발하여 뛰기시작했다. 각 주자는 정해지지 않은 시간 동안 가능한 두 방향 중 어느 한 방향으로 일정한 속력을 유지하며 달린다. 만일 Anne의 속력이 다른 두 주자의 속력과 다르다면, 언젠가는 Anne이 다른 두 주자와 각각100미터 이상 떨어져 달리고 있는 순간이 옴을 보여라. 단, 두 주자 사이의 거리는 트랙을 따라 생기는 두호 중 작은 쪽을 측정하는 것으로 한다. (캐나다 2000-1)

증명 Anne의 위치에서의 상 적인 시점으로 생각하여, Anne이 트랙의 한 지점 O에 고정되어 있다고

보자. O로부터 100미터 떨어진 두 지점을 각각 A, B라 하고, 그림과 같이 구간 X, Y 라 하자.

이 문제는 Beth와 Carmen이 동시에 구간 X에 있을 때를 보이라는 것이다. 두 사람은 Anne과 속력이다르므로, 그 상 속도는 0이 아니다. 일반성을 잃지 않고 Beth의 속력보다 Carmen의 속력이 빠르거나같다고 하고, Beth는 A지점을 향하는 방향으로 출발한다고 하자. Carmen의 속력이 Beth의 속력의 2배이하일 때는, Beth가 출발하여 처음 A지점에 이르렀을 때 Carmen의 위치는 구간 X에 있다. 이것을 두사람이 달리는 방향이 서로 달라도 마찬가지이다. Carmen의 속력이 Beth의 속력의 2배를 넘을 때는,Beth가 X 구간을 달리는 동안 Carmen은 Y 구간의 길이보다 더 달리므로 X도 지나게 된다. ¤

별증 트랙을 적당히 회전하며 관찰하여, Anne이 움직이지 않는 것으로 보자. Anne의 위치로부터

100미터 이상 떨어진 영역을 X라 하자. 출발 후 t초가 지났을 때까지 Beth가 처음 X와 만난다면 Beth가X를 지나는 시점은 [t; 2t], [4t; 5t], [7t; 8t], : : : 등이고, 이 시점에서 Carmen이 뛴 거리가 [100; 200],[400; 500], [700; 800], : : : 등의 구간을 지나는 경우가 있으면 둘이 함께 X를 지나는 경우가 된다. 즉, 이들 구간을 좌표로 표현하면 그림과 같이 균일하게 사각형 영역이 배치되는 형태이고, 여기서 Carmen의경로인 원점에서 출발하는 기울기(= 속력)가 0이 아닌 직선이 반드시 사각형들과 만나느냐 하는 문제가

Page 293: 실전수학올림피아드 1400제 해답

5.2 종합 고급문제 293

된다.

그림에서 이 직선은 특히 진하게 표시된 사각형들과 반드시 만날 수밖에 없음이 분명하다. ¤

41. 3x2 + 32y2꼴(x; y는 정수)로 나타내어지는 수를 운수 좋은 수라 부르기로 하자. n이 운수 좋은 수이면97n도 운수 좋은 수임을 증명하여라. (IMTS R23-2)

증명 (박경태)

97n = 291x2 + 3104y2

= 288x2 + 192xy + 32y2 + 3x2 ¡ 192xy + 3072y2

= 32(9x2 + 6xy + y2) + 3(x2 ¡ 64xy + 1024y2)= 32(3x+ y)2 + 3(x¡ 32y)2

이므로 n이 운수 좋은 수이면 97n도 운수 좋은 수이다. ¤

42. 적당한 k ¸ 2와 적당한 자연수들 a1; a2; : : : ; ak 이 유일하게 존재하여 n = a1+a2+¢ ¢ ¢+ak = a1a2 ¢ ¢ ¢ ak을 만족할 때, n을 좋은 자연수라고 부르기로 하자. [예를 들어 10 = 5 + 2 + 1 + 1 + 1 = 5 ¢ 2 ¢ 1 ¢ 1 ¢ 1 이고 이게 유일한 표현이므로 10은 좋은 자연수이다.] 어떤 자연수가 좋은 자연수인지, 소수에 관련한 문장으로, 모두 찾아 말하여라. (아일랜드 1993-2)

풀이 일반성을 잃지 않고, 항상 ai ¸ ai+1이라 할 수 있다.

만일 n이 소수라면 곱셈 표현방법이 n£ 1k¡1로 항상 나타나게 된다. 따라서 a1 = n; ai¸2 = 1이 된다.

그런데 k ¸ 2이므로 ai들을 더하면 항상 n+ 1 이상이 된다. 따라서 n은 소수가 아니다.n = ab(a; b ¸ 2)라 하자. 그런데 2보다 크거나 같은 두 수의 곱은 항상 합보다 크거나 같고 등호는 두수가 모두 2일때만 성립한다. 따라서 a1 = a; a2 = b로 두고 ai = 1을 n ¡ a ¡ b개 채우면 ai들이 주어진 성질을 만족한다. 즉, 조건을 만족하는 ai들이 존재한다. 한편 n = ab = cd이고 a 6= c, a 6= d라면a + b 6= c + d이고 따라서 표현방식이 2가지 이상 존재한다. 따라서 n이 좋은 자연수이려면 적어도 두개의 소인수를 가지면서 두 수의 곱으로 나타내는 방식이 유일해야하며 이는 n이 두 소수의 곱임을 의미한다.¢ ¢ ¢ 답 n = pq꼴의 자연수(p = q도 가능). }

43. 세 변의 길이를 a, b, c라 하고 외접원의 반지름을 R이라 할 때 R(b+ c) = apbc 를 만족하는 삼각형을 구

하여라. (주니어발칸 1997-4)

풀이 R(b+ c) ¸ 2Rpbc ¸ apbc 이므로 b = c 이고 a = 2R. 직각이등변삼각형 }

44. (a) 단위 정사각형들로 이루어진 l £ m 크기의 격자직사각형에 각선을 그리면, 이 각선은 모두l +m¡ d 개의 단위 정사각형을 통과함을 보여라. 단, d는 l과 m의 최 공약수이다.

(b) 단위 정육면체들로 이루어진 l£m£n 크기의 격자직육면체에 주 각선을 그렸을 때, 이 각선이통과하는 단위 정육면체는 모두 몇 개인가? (아일랜드 1993-10, 격자대각선)

Page 294: 실전수학올림피아드 1400제 해답

294 종합문제

풀이 (a) 격자점을 하나도 통과하지 않는다면 각선이 한 꼭지점에서 출발하여 새로운 칸을 지날 때

마다 x-값이나 y-값이 둘 중 하나만 증가하므로, 처음 0 + 0에서 마지막 l+m까지 l+m번 값이 증가함에 의해 l +m개의 칸을 지나게 된다. 그런데, gcd(l;m) = d 이면 ( l

d; md)만큼 진행할 때마다 격자점을

한 번씩 지나게 되고, 그게 모두 d번이고, 그 때마다 x-값과 y-값이 동시에 증가하여 칸을 하나씩 덜 지나게 되므로, 총 l +m¡ d개 ¢ ¢ ¢ 답(b) 역시 모서리나 격자점을 하나도 안 지난다면 l +m + n칸을 지나게 되는데, 격자점이 아닌 모서리를 지날 때마다 1칸씩 덜 지니게 되고, 격자점을 지날 때는 2칸씩 덜 지나게 된다. 그런데, 격자점은 세방향의 모서리에 모두 속하므로 그냥 모서리를 지날 경우를 다 빼준다면 오히려 3칸씩 뺀 것이 되므로한 번은 도로 보충해줘야 한다. 즉, gcd(l;m) = d1, gcd(m;n) = d2, gcd(n; l) = d3, gcd(m;n; l) = d

라 하면, 세 방향의 모서리에서 각각 d1, d2, d3번씩 덜 지나고, 다시 격자점에서 d번 더 지나게 되므로,l +m+ n¡ d1 ¡ d2 ¡ d3 + d개 ¢ ¢ ¢ 답 }

주 이런 식으로 포함-배제의 방식으로 차원이 올라갈 때마다 계속된다.

45. 다음과 같은 무한 과정의 계산을 했을 때 어떤 양수가 나온다고 가정하자. 이 값을 구하여라.(IMTS R21-4)

풀이 준식을 x = 1+ xa 라 두면, a = 3+ 4+x

a¡2 . 두 식을 다항식꼴로 만든 후 가감법으로 a를 소거해주

면 x3 = 2. }

46. 아래 그림과 같이, 세 원 C0; C1; C2의 중심은 한 직선 위에 있다.

원 C1과 C2의 교점을 A와 B라고 하고, 직선 AB와 원 C0의 한 교점을 C라 하자. 이 때, 그림에서처럼C0, C1, BC에 접하는, 또 C0, C2, BC에 접하는 두 작은 원은 합동임을 증명하여라. (IMTS R20-5)

증명 C0, C1, C2의 중심을 각각 O, O1, O2라 하고 반지름을 각각 1, R, r로 두자(R ¸ r). AB \O1O2 = X 라 할 때, O1B2 ¡ O2B2 = O1X2 ¡ O2X2 임에서 OX =

jR¡rj(1¡R)+(1¡r) 을 얻음. 왼쪽 위

작은 원의 중심을 P , 반지름을 w라 할 때, 각각 PO1, PO를 빗변으로 하는 직각삼각형에서 피타고라스

정리로 얻은 식을 연립하면 w =(1¡R)(1¡r)(1¡R)+(1¡r) 이 됨. 이것은 R, r에 한 칭식이므로 오른쪽 작은 원

도 마찬가지. ¤

47. 6x3 = y2 + 45 를 만족하는 유리수해를 모두 구하여라. (KMC예선 2002전기 고1)

개요 해는 없다. x = a=c, y = b=c 를 입하여 a, b, c가 서로소가 될 수 없음을 증명... }

Page 295: 실전수학올림피아드 1400제 해답

5.2 종합 고급문제 295

48. Q1을 1보다 큰 모든 유리수들의 집합이라 하자.

(1) Q1을 두 집합 A와 B로 분할하는데, 두 집합 모두 덧셈에 해 닫혀있도록 할 수 있는가?

(2) 덧셈 신 각각 곱셈에 해 닫혀있도록 할 수 있는가? (벨로루시 1995-B7)

49. 자연수들의 수열 (an)이 모든 자연수 n에 해서 an =

sa2n¡1 + a2n+1

2을 만족한다. 이 수열은 상수 수

열임을 보여라. (오스트리아 1985-5)

증명 제곱하여 점화적으로 정리하면 a2n = n(a21 ¡ a20) + a0. 만약 a1 < a0 이면 언젠가는 0보다

작아지니깐 모순. a1 > a0 이면 이 수열은 강증가 수열(ai+1 > ai). a2n = n(a21 ¡ a20) + a0 인데,

a2n+1 ¡ a2n = a21 ¡ a20 으로써 상수. 연속한 두 제곱수 사이의 차는 계속 증가하므로, 적당히 n을 크게

잡으면, a2n+1 > a2n > (an+1 ¡ 1)2 으로 만들 수 있어서 모순. 따라서 a0 = a1 이어야 하고, 이 때 모든

항이 an = a0. ¤

50. n ¸ 3 은 정수이다. 모든 k에 해 다음 조건을 만족하는 함수 f : f1; 2; : : : ; ng ! f1; 2; : : : ; ng 의 개수를 구하여라: (루마니아 2005 지역예선 y10-4)

f(f(k)) = (f(k))3 ¡ 6(f(k))2 + 12f(k)¡ 6 (k ¸ 1)

풀이 (부산 건국고 2학년 신승현)

f의 치역을 A라 하자. 그럼 준식을 잘 관찰하면 임의의 a 2 A 에 해

f(a) = (a¡ 1)(a¡ 2)(a¡ 3) + a

이 된다. f(a) 2 A 임은 자명하고, 만일 a 62 f1; 2; 3g 이라면 f(a) > a 가 된다. 즉,

a < f(a) < f(f(a)) < f(f(f(a))) < ¢ ¢ ¢

가 되고 이것이 모두 A의 원소이므로 A가 유한집합임에 모순이다. 따라서, a 2 f1; 2; 3g 이어야 한다.

A ½ f1; 2; 3g

A 6= ? 이므로 다음과 같이 경우를 나눌 수 있다.

(1) n(A) = 3 일 때:¡33

¢£ 3n¡3가지.

(2) n(A) = 2 일 때:¡32

¢£ 2n¡2가지.

(3) n(A) = 1 일 때:¡31

¢£ 1n¡1가지.

따라서, 구하고자 하는 답은 3n¡3 + 3 ¢ 2n¡2 + 3. }

51. 볼록사각형 ABCD에서 마주보는 변의 중점끼리 서로 선분으로 연결하여 작은 네 개의 사각형으로 분할하였다. 각각의 작은 사각형의 둘레의 길이가 모두 같다면, ABCD는 평행사변형임을 증명하여라.

(레닌그라드 1987-13)

증명 AB, BC, CD, DA의 중점을 각각 E, F , G, H라 하고 EG \ FH = O 라 하자. EFGH는 평행

사변형이므로 그 각선 EG와 FH는 서로를 이등분. 그럼 변으로 이웃한 두 작은 사각형의 둘레를 비교하면 AE = GD, 즉 AB = CD, 마찬가지로 BC = AD. ¤

52. h는 양의 정수이고, 수열 an은 다음과 같이 정의된다: a0 = 1 이고

an+1 =

(an=2 (an이 짝수일 때)

an + h (그렇지 않을 때)

예를 들어, h = 27 인 경우 a1 = 28, a2 = 14, a3 = 7, a4 = 34 등이 된다. h가 어떤 수일 때 an = 1 인n > 0 이 생기게 되는가? (이탈리아 2005-5)

Page 296: 실전수학올림피아드 1400제 해답

296 종합문제

풀이 (서울 경희고 1학년 이승후)

수열에 나타나는 모든 항들이 양의 정수임에는 틀림없다(1). 또 수열의 한 항이 주어지면 다음 항은 유일하게 결정된다. h가 짝수이면 홀수인 항만 나와서 계속 an+1 = an + h 만 적용되므로 증가수열이 되어 1이 더 이상 나오지 않는다. 따라서, h는 홀수. h = 1 일 때는 1! 2! 1 로 성립한다. 이제 h가 1보다 큰 홀수일 때만 살펴보면 되겠다. 다음을 보이자.

(2) 이 수열의 어떤 항도 2h 이상일 수 없다.

만일 2h를 넘는 항이 존재한다면, 그런 최초의 항을 ak라 하자.

a1 = 1 + h; a2 =1 + h

2; a3 =

1 + h

4or1 + 3h

2

가 모두 2h보다 작으므로 k ¸ 4. ak가 최초이므로 ak¡1 < ak 이고, 그럼 ak = ak¡1 + h 여야만 한다.

이 때, ak¡1은 홀수이고 ¸ h 이다. ak¡1 = ak¡2=2 라면 ak¡2 ¸ 2h 로 ak의 최초성에 모순이 되므로,

ak¡1 = ak¡2 + h 여야 한다. 그럼 ak¡2는 짝수이므로 이것 또한 모순이다. 따라서, (2)가 증명되었다.

이제 다음을 알 수 있다.

ak¡1 =

(2ak (1 < ak < h 일 때)

ak ¡ h (h < ak < 2h 일 때)

1 < ak < h 인 경우에는 (1)에 의해 ak¡1 > 0 이어야 해서 그렇고, h < ak < 2h 인 경우에는 (2) 때문

에 그렇다. 또, ak = h 인 경우는 ak¡1이 0 또는 2h가 되어 곤란하므로 그런 경우는 없다. 이상으로 이

수열은 역으로 가는 점화식에 의해

(3) 뒷항이 주어지면 그 앞항도 유일하게 결정됨을 알 수 있다.

(1), (2)에 의해 이 수열에는 유한 개의 수들만 나타나므로, 비둘기집 원리에 의해 같은 값을 갖는 두 항aj = ak (j < k) 가 존재한다. 그럼 (3)에 의해 aj로부터 a0 = 1 이 얻어지듯이 ak로부터 ak¡j = 1 이

얻어진다. 따라서, 1이 되는 항이 존재한다.

위를 종합한 결과로, an = 1 인 n > 0 이 존재하는 h들은 모든 홀수들이다. }

53. A와 B 두 문자로 이루어진 낱말들의 열이 있다. 맨처음 낱말은 A이고, 다음 낱말은 B이며, k번째 낱말은 k ¡ 2번째 낱말과 k ¡ 1번째 낱말을 연이어 쓴 것이다. 즉, A, B, AB, BAB, ABBAB, ... 와 같이 계속 된다. 여기에 주기적인 낱말, 즉 PP¢ ¢ ¢P꼴로 P가 두 번 이상 반복되는 낱말이 존재하는가? 예를 들어,BABBBABB는 BABB가 두 번 반복된 주기적인 낱말이다. (Towns 1988봄 JA7)

54. 삼각형 ABC에서 각 A의 이등분선과 외접원이 다시 만나는 점을 P라고 하자. 점 Q와 R도 비슷하게 정의하자. 이 때 다음을 증명하여라. (호주 1982-3)

AP +BQ+ CR > AB +BC + CA

증명 2AP = 2(AI + IP ) = (AI +BP ) + (AI + CP ). 이런 거 셋 더하면 삼각부등식. ¤

55. 몇 개의 단위정육면체들을 6개의 색으로 칠하는데, 각 정육면체마다 6면의 색이 모두 다르도록 하였다(정육면체마다 색의 위치구성은 다를 수 있다). 이 정육면체들을 탁자 위에 직사각형꼴로 배열하였다. 한 열을 택해 그 열의 정육면체들을 장축(열방향의 축)을 축으로 하여 회전시킬 수 있다. 행에 해서도 마찬가지 조작이 가능하다. 이 직사각형의 윗면이 모두 같은 색이 되도록 하는 것이 항상 가능한가?

(헝가리 1969-3, Towns 1987봄 JA5)

풀이 X+i Y +j X¡i Y ¡j 로 하면 다른 것은 다 그 로 둔 채 i행 j열의 블록만 돌려놓을 수 있음. 이 요령

만 잘 쓰면 됨. }

별해 한 행씩 완성하면 됨. 한 행을 완성한 후 다른 행을 작업하기 전에 열작업할 때 영향을 받지 않도

록 잠깐 그 행을 돌려놓으면 됨. }

56. 0 · j < i · 99 인 정수일 때, 10i ¡ 10j 중에서 1001의 배수인 것은 몇 개인가? (AIME 2001 2차-10)

Page 297: 실전수학올림피아드 1400제 해답

5.2 종합 고급문제 297

풀이 %EEE

(10^6 - 1) = (10^3 + 1)(10^3 - 1), so 10^6 - 1 and hence also 10^{6n} - 1

is certainly divisible by 1001. But 7 divides 1001 and 10^m - 1 is not

divisible by 7 for m not a multiple of 6. Obviously 10^m is never divisible

by 7. Thus 10^m(10^n - 1) is divisible by 1001 iff n is a multiple of 6.

So for j = 0, there are 16 values of i (6, 12, ... , 96). Similarly for

j = 1, 2, 3. For the next 6 values of j there are 15 values of i, total 6·15.

For the next 6, there are 14 each, total 6·14, and so on up to the

6 values 88 to 93 for which there is one each, total 6·1. For j > 93,

there are no values of i. Hence total = 64 + 6(15 + 14 + ... + 1) = 64 + 720 = 784.

¢ ¢ ¢ 답 784 }

57. 폴과 피터는 내기를 한다. 자연수를 두고 그 자연수의 가장 큰 홀수인 약수를 구한 후 그 홀수를 4로 나눠서 나머지가 1이면 피터가 폴에게 1원을 주고 아니면 폴이 피터에게 1원을 준다. 그 이후는 그 자연수 바로 다음 수로 같은 게임을 하고 그 이후에는 그 다음 자연수로 같은 게임을 한다. 애초에 자연수를 1부터시작했고 어느 정도 게임을 한 후 서로의 돈을 정산했다. 폴이 더 많은 돈을 땄음을 보여라.

(독일BW 1974 2차-4)

증명 1에서 n까지 게임을 하고 끝냈다고 하자. 먼저 1에서 n까지의 수들 중 홀수만 조사하면 (홀수의

가장 큰 홀수인 약수는 자기 자신이므로) 4로 나눈 나머지가 1, 3, 1, 3, ... 이렇게 1부터 시작하여 번갈아 나오므로, 이 수들만 따졌을 때 폴이 지지 않았음을 알 수 있다. 다음으로 1에서 n까지의 수들 중 짝수들 중에 2£ (홀수)꼴들만 조사해보자. 이 경우에도 (홀수)가 1, 3, 1, 3, ... 이렇게 1부터 시작하여 번갈아 나오므로, 이런 수들에서도 폴이 지지 않았음을 알 수 있다. 일반적으로 각각의 k = 0; 1; 2; : : :에

해 2k £ (홀수)꼴의 수들만 생각했을 때 항상 폴이 지지 않음을 알 수 있다. 특히, 2m · n < 2m+1 을만족하는 m이 유일하게 존재하고, k를 m으로 택했을 때는 2m £ 1만 게임을 했으므로 폴이 1원 땄다.

따라서, k = 0; 1; 2; : : : ;m 에 해 각각의 경우를 모두 합해도 폴이 적어도 1원 이상 따게 됨을 알 수 있다. ¤

58. n£ n 크기의 판에서 한 코너의 1£ 1 칸을 잘라냈다. 이 도형을 몇 개의 삼각형으로 자른다면 최소 몇 조각으로 자를 수 있는가? (러시아 1990 4차-y9-6)

풀이 이 판은 3개씩의 평행선 2 세트로 구성됨. 4조각으로는 항상 가능. 3조각으로 된다고 가정해보

자. 한 세트의 3개 평행변에 해 각 변의 전부 또는 일부를 한 변으로 갖는 삼각형이 하나씩 응되어야함. 그 삼각형이 조각의 전부. 즉, 각 삼각형은 이 판의 평행하지 않은 두 변을 자신의 두 변으로 가져야하고 그럼 경우는 뻔함. 중간에 영역이 하나 생기는데 그것이 퇴화되는 n = 2 일 때만 3조각으로 가능.

나머지는 4조각이 꼭 필요. }

59. p2 ¡ p+ 1 이 세제곱수가 되는 소수 p를 모두 구하여라. (발칸 2005-2)

풀이 (한국과학영재학교 1학년 권혁준)

a3 = p2 ¡ p+ 1 이라 하면 (p ¸ 2 임에서 a > 1 이고) a3 ¡ 1 = p2 ¡ p, 즉

(a¡ 1)(a2 + a+ 1) = p(p¡ 1)

만일 a ¸ p 라면 (a¡ 1)(a2 + a + 1) > (a ¡ 1)a ¸ (p¡ 1)p 로 등식이 성립하지 않으므로 a < p 이다.

따라서, a보다 큰 소수 p는 a¡ 1 의 소인수일 수 없고, 그럼

p j a2 + a+ 1; 그러면 a¡ 1 j p¡ 1

이제 p¡ 1 = k(a¡ 1) 라 하자. 위의 식에 입하여 p를 소거하면

(a¡ 1)(a2 + a+ 1) = (k(a¡ 1) + 1)k(a¡ 1)a2 + a+ 1 = k2(a¡ 1) + k

a2 + (1¡ k2)a+ (k2 ¡ k + 1) = 0

여기서 a는 정수해를 가지므로 이 식의 판별식이 제곱수가 되어야 한다.

D = (1¡ k2)2 ¡ 4(k2 ¡ k + 1) = k4 ¡ 6k2 + 4k ¡ 3

Page 298: 실전수학올림피아드 1400제 해답

298 종합문제

k = 1; 2 일 때는 D < 0 이 되어 곤란하고, k ¸ 3 일 때는

(k2 ¡ 3)2 · D < (k2 ¡ 2)2

이 성립함을 금방 확인할 수 있다. 따라서, D = (k2 ¡ 3)2 이어야 하고, 이것을 풀면 k = 3. 이 때,

a2 ¡ 8a+ 7 = 0 이므로 a = 7, p = 19 이다. 192 ¡ 19 + 1 = 343 = 73 은 잘 성립한다. ¢ ¢ ¢ 답 19 }

60. 수열 1; 12; 13; 14; : : : 를 생각하자. 여기서 몇 개 항을 골라

(1) 길이 5인 등차수열이 되도록 할 수 있는가?

(2) 길이가 5보다 긴 등차수열이 되도록 할 수 있는가? 할 수 있다면, 가능한 길이는 얼마인가?(Towns 1982봄 J5)

풀이 얼마든지 길게 할 수 있다. 1n! ;2n!; : : : ; n

n! 로 하면 된다. }

61. 좌표평면 위에 있는 삼각형이 정수 좌표의 세 꼭지점을 가지고, 세 변의 길이도 모두 정수일 때, 이 삼각형을 `정수적'이라고 한다.

(1) 둘레의 길이가 42인 정수적인 삼각형을 하나 찾아라.

(2) 둘레의 길이가 43인 정수적인 삼각형이 존재하는가? (IMTS R35-4)

풀이 (김범수)

좌표평면에 놓을때, 일반성을 잃지 않고 한 점 A가 (0,0)이 되도록 놓을 수 있다.(1) A(0; 0), B(¡5; 12), C(9; 12)로 잡자. AB = 13, BC = 14, CA = 15. 둘레가 42.(2) A(0; 0), B(x1; y1), C(x2; y2)가 조건을 만족시킨다고 가정. 1 ´ 43 = AB + BC + CA ´ AB2 +BC2+CA2 = x21+ y21 +x22+ y22 +(x1¡ y1)2+(x2¡ y2)2 = 2(x21+x22+ y21 + y22 ¡ x1y1¡ x2y2) ´ 0(mod 2) 즉, 1 ´ 0 (mod 2) 로 모순. 따라서 만족시킬 수 없다. }

62. 두 수열 fang, fbng은 다음과 같이 정의된다.

a1 = 3; b1 = 100

an+1 = 3an ; bn+1 = 100

bn

bm > a100 을 만족시키는 최소의 자연수 m을 구하여라. (호주 1986-2)

63. 평면 위에 어느 세 점도 한 직선 위에 있지 않은 50개의 점이 있다. 각각의 점들을 주어진 네 가지 색 중하나로 각각 칠하였다. 이 중 다음을 만족하는 색이 있음을 증명하여라: 이 색의 점만을 꼭지점으로 하는부등변 삼각형(세 변의 길이가 서로 다른 삼각형)이 130개 이상 있다. (주니어발칸 2007-3)

증명 같은 색의 점 13개가 존재한다. 이 점들로 만들어지는 삼각형은 13C3 = 286개. 이 점들 중 두

점으로 만든 선분을 밑변으로 하는 이등변삼각형은 최 2개 이므로(3개 이상이면 한 직선 위의 세 점이 생겨서 모순), 이등변삼각형은 최 2 * 13C2 = 156개. 따라서, 부등변삼각형은 적어도 286-156 =130개 이상. ¤

64. 볼록오각형의 내부에 있는 5개의 각선 중에서 삼각형의 세 변의 길이를 이루는 세 각선을 항상 찾을수 있음을 증명하여라. 한편, 오목오각형에서는 위의 성질이 성립하지 않는 경우가 있다. 그런 오각형을찾아라. (KMC 2000전기)

풀이 볼록오각형 ABCDE를 그린 후 일반성을 잃지 않고 가장 긴 각선을 AD로 두자. 그리고 BD,

AC를 긋고 그 교점을 F로 두자. 그럼 AFD는 삼각형을 이루므로

AD < AF + FD < AC +BD

따라서, AD, AC, BD는 삼각형을 이룬다. 따라서 조건에 맞는 세 각선을 항상 찾을 수 있다.

오목오각형에서는 다음과 같은 예가 있다.

다섯 개의 각선의 길이를 짧은 것부터 a, b, c, d, e라 하면, a+ b < c, b+ c < d, c+ d < e 를 만족하고 있음을 확인할 수 있다. }

Page 299: 실전수학올림피아드 1400제 해답

5.2 종합 고급문제 299

65. 다음을 만족하는 가장 큰 자연수 n은 무엇인가?:p

n+pm+

pn¡pm 이 자연수가 되도록 하는 자연

수 m (< n2) 이 오직 한 개만 존재한다. (IMTS R22-2)

풀이 n = a2 + b, m = 4a2b꼴이면(0 · b < a2) 위 식이 자연수가 되므로, 이런 a, b가 둘 있다면 일단

배제됨. a = bpnc 와 a = bpnc ¡ 1 을 모두 택할 수 있는 n의 범위를 부등식으로 구해서 먼저 배제하고, 남은 작은 n에 해 가장 큰 것을 다시 처음부터 놓고 확인하면 되지 않을런지... 이런 방법으로 결과가 귀찮아진다면 xMO의 김 성쌤 풀이 참조. }

66. a, b, c, d는 a2 + b2 + (a ¡ b)2 = c2 + d2 + (c ¡ d)2 을 만족하는 자연수들이다. a4 + b4 + (a ¡ b)4 =c4 + d4 + (c¡ d)4 임을 증명하여라. (IMTS R14-1)

증명 a2 ¡ ab+ b2 = c2 ¡ cd+ d2 을 제곱하고 두 배하면 구하는 식. ¤

67. A와 B가 게임을 한다. 탁자에 1999개의 성냥개비가 놓여있다. 각 경기자는 한 번에 성냥개비를 1개 이상,

그리고 남은 개수의 절반 이하로 가져갈 수 있다. 성냥개비를 1개 남기는 사람이 진다. B가 먼저 시작한다고 할 때, 누가 필승 전략을 가지고 있는지, 그리고 그 필승 전략은 무엇인지 서술하여라.

(이탈리아 1999-4)

풀이 n 2 L 이면 2n+ 1 2 L. 2 2 L 로부터 점화적으로 1535 2 L 임을 얻을 수 있고, 그럼 B 승. }

68. R은 양의 실수이다. R, 1, R + 12 가 어떤 삼각형의 세 변일 때, R인 변과 R + 1

2 인 변 사이의 각을 µ라

하자(라디안으로). 2Rµ가 1과 ¼ 사이에 있음을 증명하여라. (플란더즈 1988-4)

증명 (1) 각 µ의 꼭지점을 B라 하고, R인 변을 AB, 1인 변을 AC라 하자. B를 중심으로 하고 반지름

R인 원을 그려 BC와 만나는 점을 D라 하자. 그럼 2Rµ = 2 ¢호AD > 2AD > 1 이 된다. 마지막 부등식은 삼각형 ADC에서 AD +DC > 1 임을 이용했다.(2a) µ ¸ ¼=2 일 때: µ < ¼ 이므로 2Rµ < 2R¼ · ¼. R < 1

2 임을 이용했는데, 이것은 C를 중심으로 하

고 A를 지나는 원이 BC의 B쪽 연장선과 만나는 점을 E라 할 때 R + 12< 1 이 되니까. (2b) µ < ¼=2

일 때: A에서 BC에 내린 수선의 발을 H, A의 BC에 한 칭점을 D라 할 때, B를 중심으로 하고 반지름 R인 원의 호 AD와 H를 중심으로 하고 반지름 AH인 반원을 비교하면 2Rµ < AH¼ < ¼. ¤

69. 각각의 자연수 n에 해, n의 양의 약수의 개수를 d(n)이라 하자.

d(n) =n

3

을 만족하는 모든 자연수 n을 구하여라. (호주 1990-7)

풀이 d(n) < 2pn 임을 이용. 홀수인 경우에는 d(n) <

pn 으로 더 빠름. }

70. 자연수 x의 모든 (십진) 자릿수를 곱한 것을 P (x)로 나타내자. 임의로 주어진 자연수 x1으로부터 시작하여 xn+1 = xn + P (xn) 의 점화식으로 정의된 수열 (xn)이 얼마든지 큰 값을 가질 수 있는가?

(오스트리아-폴란드 1982-4)

풀이 10m¡1부터 10m¡1 + 10m¡2까지 모든 수가 자릿수로 0을 포함함. xn은 이 구간을 점프해야함.

즉, 10m¡2 < xn+1 ¡ xn = P (xn) · 9m 인 n이 존재해야 하는데, m이 충분히 크면 9m < 10m¡2 이므로 이것은 모순. }

71. 다음의 규칙으로 정의된 수열 fxng이 있다.

x1 = 2; nxn = 2(2n¡ 1)xn¡1 (n = 2; 3; : : : )

모든 자연수 n에 해 xn은 정수임을 증명하여라. (아일랜드 1994-6)

증명 점화식 쭈욱 풀면 an =¡2nn

¢이 됨. 혹시 조합적 논의로도 가능할까? ¤

72. 폐구간 [0; 1]이 서로소인 두 집합 A, B로 분할되었다. B = A+ a 를 만족하는 실수 a는 존재하지 않음을보여라. (단, A+ a = fx+ a j x 2 Ag 이다.) (오스트리아-폴란드 1982-5)

Page 300: 실전수학올림피아드 1400제 해답

300 종합문제

증명 구간 [0; a)의 수는 B에 있을 수 없으므로 [0; a) ½ A 이고, 그럼 [a; 2a) ½ B 이고, 또 그럼

[2a; 3a)에 속하는 수는 A로부터 만들어질 수 없으므로 B에 있을 수 없어서 [2a; 3a) ½ A 이고, 이렇게반복하다보면... 1=a가 짝수라야 하고 마지막에 1이 곤란해짐. ¤

73. ABCD는 한 변의 길이가 1인 정사각형이다. P와 Q는 변 AB 위의 점이고, R은 변 CD 위의 점이다. 삼각형 PQR의 외접원의 반지름이 취할 수 있는 범위를 구하여라. (스웨덴 1962-2)

풀이 Q, R을 각선상의 두 꼭지점에 위치시키고 P를 Q에 접근시키면 외접원은 한 꼭지점을 중심으

로 한 사분원에 점점 근접하게 됨. ¢ ¢ ¢ 답 [1=2; 1) }

74. 어떤 자연수의 자릿수들을 재배열하여 새로운 자연수를 만들고, 그 수를 원래의 자연수에 더했더니 1010이되었다. 원래 자연수가 10의 배수였음을 증명하여라. (소련 1967-2b)

증명 (a)는 홀수 자리의 99 ¢ ¢ ¢ 9가 이렇게 나올 수 없다는 것이었고, 그 문제를 이용. 일의 자리에서

자리올림이 있으면 그 앞이 9개의 9가 되어야 하고 그것은 모순. 따라서, 일의 자리에서 자리올림이 없고 그럼 둘다 0이었으므로 10의 배수. ¤

75. 서로 다른 세 개의 숫자만을 이용하여 16개의 세 자리 수를 만드는데, 이 수들이 16으로 나눈 나머지가 모두 다르도록 할 수 있는가? (주니어발칸 1998-4)

풀이 일의 자리가 홀수인 것도 짝수인 것도 있어야 하므로 세 수는 홀홀짝 or 짝짝홀. 홀홀짝일 때 일

의 자리가 짝수인 9개의 수 중에 짝수 나머지 8가지가 모두 나와야 함. 짝짝홀일 때도 일의 자라가 홀수인 것으로 생각하여 비슷. xyz = 100x+10y+ z ´ 4x¡ 6y+ z (mod 16) 임에서 4x¡ 6y 가 mod 16으로 8종류를 가져야 함. 즉, 2x¡ 3y 가 mod 8로 모든 나머지를 가져야 함(x; y 2 fa; b; cg). 그런데, 홀수가 하나뿐이면 2x¡ 3y가 홀수이기 위해선 y가 홀수라야 하고 그럼 x의 선택은 3가지뿐이므로 mod 8의홀수 나머지 4가지가 다 나올 수 없음. 또, 짝수가 하나뿐이면 2x¡ 3y가 짝수이기 위해선 y가 짝수라야하고 그렇게 역시 마찬가지. }

76. M ½ N 에 속하는 모든 두 자연수 m, n에 해서 jm¡ nj ¸ mn=25 를 만족한다고 가정하자. 집합 M은원소를 9개 이하로 가짐을 보여라. (체코슬로바키아 1986-6)

증명 양변을 mn으로 나누면 j 1m¡ 1

nj ¸ 1

25. m < n 이라 할 때 m < 25 여야 하고, 그럼 M에 10개

이상의 원소가 있다면 그 중 1개를 제외하면 25 미만이므로 25 미만의 수가 9개 이상. 가장 작은 m부터차례로 더 작은 m으로 체할수록 유리해지므로, 1, 2, 3, 4, 5, 7, 10, 17, 54 이렇게 차례로 9개를 찾아보면 25를 넘음. ¤

77. 한 직선 위에 n개의 파랑점과 n개의 빨강점이 주어져있다. 같은 색의 두 점 사이의 거리를 모두 합한 것은 서로 다른 색의 두 점 사이의 거리를 모두 합한 것보다 크지 않음을 보여라. (러시아 1994 최종-y9-4)

증명 WLOG 임의의 Ri; Rj (Ri · Rj) 와 Bi; Bj (Bi · Bj) 에 해 (Rj ¡ Ri) + (Bj ¡ Bi) ·jRi ¡Bj j+ jRj ¡Bij 를 삼각부등식으로 확인하면 끝. ¤

78. x ¸ 12 이면, 부등식 jx¡ n2j ·

qx¡ 1

4 을 만족하는 정수 n이 항상 존재함을 보여라. (스웨덴 1965-3)

증명 양변 제곱하여 정리한 후 이차식을 풀면 n2 ¡ n + 12· x · n2 + n + 1

2. n2 + n + 1

2=

(n+ 1)2 ¡ (n+ 1) + 12 이므로 이 구간들은 1

2 이상의 실수 전체를 커버한다. ¤

79. 주어진 원에 내접하는 삼각형들의 수심의 자취를 구하여라. (Towns 1986가을 J5)

풀이 주어진 원을 3배 확 한 원의 내부의 모든 점 }

80. ad¡ bc = 1 과 ab> p

q> c

d 를 만족하는 자연수 a, b, c, d, p, q에 해 다음을 증명하여라.

(1) q ¸ b+ d;

(2) q = b+ d 이면, p = a+ c. (폴란드 1969 3차-3)

81. 평면이 세 방향의 평행선 묶음으로 인해 N개의 영역으로 나뉘어 있다. N > 1981 이기 위해 필요한 직선은 최소 몇 개인가? (오스트리아-폴란드 1981-8)

Page 301: 실전수학올림피아드 1400제 해답

5.2 종합 고급문제 301

풀이 각 방향으로 a, b, c개 있다고 하고 ab+ bc+ ca · 13(a+ b+ c)2 부등식 이용하여 좀 풀면. }

82. 직사각형 ABCD의 내부의 점 M이 있다. 이 직사각형의 넓이를 S라 할 때 다음을 증명하여라.(Towns 1988가을 SA6)

S · AM ¢ CM +BM ¢DM

83. n은 홀수이고, 칠판에 ¤x2 + ¤x + ¤ = 0 꼴의 식이 n개 적혀있다. 두 명이 번갈아서 이 식들 중의 ¤ 하나를 골라 0이 아닌 실수로 바꾼다. 3n번 작업하여 n개의 이차방정식을 얻었다. 먼저 하는 사람은 실근을갖지 않는 방정식의 개수를 최 화하는 것이 목표이고, 나중에 하는 사람은 이것을 방해하는 것이 목표이다. 나중에 하는 사람이 최선의 경기를 할 때, 먼저 하는 사람은 실근을 갖지 않는 방정식을 최 몇 개 만들 수 있는가? (러시아 1993 최종-y9-8)

풀이 각각의 식은 ax2 + bx+ c = 0 의 꼴로 읽도록 하자.

[갑(먼저하는 사람)의 전략] (0) 우선 맨처음엔 한 식을 택해 내 식으로 선언한다(b에 충 아무거나 써넣는다). (1) 을이 내 식을 건드리면(a나 c를 썼을테니) 그 식을 완성하여 내가 먹는다(4ac > b2 이 되도록 할 수 있으니 무조건 내가 먹음). (2) 을이 자신의 식을 두번째로 건드리면 걍 그 식을 완성하여 을이 먹도록 써비스해준다. (3) 을이 새로운 식을 자신의 식으로 선언하면 나도 새로운 식을 하나 택해 내식으로 선언한다. 식이 홀수개이므로 을의 차례에는 새로운 식은 항상 짝수개가 남아있어서, 또 을의 차례에 계수가 하나만 남은 식은 하나도 없어서, 갑은 이런 전략 로 항상 행할 수 있고, 그럼 갑은 무조건을보다 (최소) 1개 더 많은 식을 먹을 수 있다.[을의 전략] (1) 갑이 새로운 식을 자신의 식으로 선언했을 때: (1-1) 나도 새로운 식을 하나 택해 내 식으로 선언한다(a에 충 아무거나 써넣는다). (1-2) 만일 새로운 식이 더이상 남아있지 않으면 내 식을하나 택해 먹는다(ac < 0 이 되도록 하면 b가 뒤늦게 어떻게 결정되든 상관없이 무조건 내 먹이). (1-3)

새로운 식도 없고 내 식도 다 먹었으면 걍 아무렇게나 플레이한다. (2) 갑이 내 식을 건드리면 그 식을완성하여 내가 먹는다(갑이 b를 건드려도 c를 건드려도 항상 내가 b2 > 4ac 이 되도록 완성할 수 있음).(3) 갑이 갑의 식을 건드린다면(아마 그럴 일은 없겠지만) 그 식을 내가 완성하여 내가 먹어버린다. 새로운 식이 다 동나기 전에는 갑이 선언하는만큼 나도 선언하게 되므로 을은 갑보다 1개 적게 선언하게 되고, 일단 내 식으로 선언된 것은 항상 내가 먹으므로 을은 최소 (n¡ 1)=2 개의 식을 먹을 수 있다.

위의 두 사람의 전략에 의하면 상 방의 전략에 상관없이 갑은 항상 최소 (n+ 1)=2 개의 식을, 을은 항상 최소 (n ¡ 1)=2 개의 식을 먹을 수 있고, 두 값이 서로의 경계점에 해당하므로 답은 갑의 (n + 1)=2

개. }

84. 주어진 미로 A, B, 또는 C 위에서 고양이가 쥐를 쫓는 `고양이와 쥐'라는 게임이 있다.

고양이는 K 위치에서 출발하고, 쥐는 M에서 출발한다. 고양이가 먼저 한 칸을 움직이고, 그 다음 쥐가한 칸을 움직이고, 이렇게 번갈아서 한 칸씩 움직인다. 둘이 같은 위치에 있으면 고양이가 쥐를 잡게 된다.A, B, C 각 경우에 해, 고양이가 쥐를 반드시 잡을 수 있는 전략이 존재하는가? (Towns 1985가을 J2)

풀이 A에서는 쥐가 고양이에게 달려들지 않는 이상 어떻게 움직여도 잡히지 않음. 혹은 칭 플레이.

C에서도 점 칭 플레이. B에서는 고양이가 빗변을 한 번 돌고 M의 위치로 왔을 때, 쥐는 빗변을 돌려고 하면 먼저 잡히므로 빗변을 돌지 못하고 좌상귀(혹은 우하귀) 아니면 좌변중하(혹은 하변중좌)의 위치에 있게 되고, 그럼 적당히 구석으로 몰면 3번 내에 잡게 됨. }

85. 2m · n 인 임의의 자연수 m, n에 해(n¡m)!

m!·µn+ 1

2

¶n¡2m임을 보여라. (스웨덴 1970-6)

증명 n = 2m+ r 로 놓고 입하면 (m+ r)(m+ r¡ 1) ¢ ¢ ¢ (m+ n) · (m+ r+12)r 을 증명하는 것이

됨. 이건 AM-GM. ¤

Page 302: 실전수학올림피아드 1400제 해답

302 종합문제

86. N은 양의 정수들의 집합이다. 임의의 n > 1 에 해 f(n) = f(f(n¡ 1)) + f(f(n+ 1)) 을 만족하는 함수f : N! N 이 존재하는가? (아일랜드 1990-3)

풀이 f(2); f(3); f(4); : : : 중에서 가장 작은 것 하나를 f(m) 이라 하자. n ¸ 2 이면 f(n) = f(f(n ¡1)) + f(f(n+ 1)) ¸ 1 + 1 = 2 이므로, f(f(n)) ¸ f(m) 이다. 따라서,

f(m) = f(f(m¡ 1)) + f(f(m+ 1)) ¸ 1 + f(m)

이 되고, 이것은 모순. }

87. 사면체 ABCD의 각각의 면의 외접원의 반지름의 길이가 모두 같으면, AB = CD, AC = BD, AD = BC

임을 증명하여라. (루마니아 2005 지역예선 y8-3)

증명 이 사면체에서 임의의 어느 한 변에 해 마주보는 두 각은(예를 들어, AB에 해서는 \ACB와

\ADB) 같은 크기의 원에서 같은 길이의 현에 한 원주각이므로 서로 크기가 같거나 아니면 서로 보각이 된다(\ACB = \ADB 또는 \ACB = 180± ¡\ADB). 사면체 ABCD의 어느 한 꼭지점 주변의각을 P , Q, R이라 하고, 그에 응되는 밑면의 각을 그림과 같이 각각 p, q, r이라 하자(넓은 각은 편의상 꼭지점의 바깥쪽에 표시하겠다).

(P; p), (Q; q), (R; r)이 모두 서로 보각이라면

P +Q+R = ¹p+ ¹q + ¹r = 3 ¢ 180± ¡ (p+ q + r) = 360±

가 되는데(x의 보각을 ¹x와 같이 나타내기로 한다), 한 꼭지점 주위에 모인 각의 합은 360±보다 작아야입체를 구성할 수 있으므로 이것은 모순이다. 또, (P; p), (Q; q), (R; r) 중에 서로 보각인 것이 딱 1쌍이라면, 일반성을 잃지 않고 (P; p)라면,

Q+R = q + r = 180± ¡ p = ¹p = P

가 되는데, 그럼 Q와 R의 각에 해당하는 면이 P의 각에 해당하는 면에 완전히 접혀져 들어가게 되므로입체가 구성되지 않아 역시 모순이다. 따라서, 이 사면체의 한 꼭지점에 모인 세 각은 그에 응되는 각들과 보각을 이루는 것이 0쌍이거나 2쌍이어야 한다(¤).서로 마주보는 6쌍의 각 중에서 서로 보각인 것이 있다고 하자(¤¤).

그럼 (¤)에 의해, 일반성을 잃지 않고, 2쌍의 보각 (¹p; p), (¹q; q)과 1쌍의 같은 각 (r; r)이 한 꼭지점 A에모여 있을 것이다. 꼭지점 B와 D 주변에는 이미 보각인 것이 하나씩 있으므로, (¤)에 의해 보각인 것이꼭 하나씩 더 있다. 그럼 AC에 해 마주보는 두 각이 (s; s)꼴이냐 (¹s; s)꼴이냐에 따라 위의 그림과 같이 두 가지 경우로 나눌 수 있다. 즉, 왼쪽 그림에서는 서로 보각인 것이 3쌍 있고, 오른쪽 그림에서는4쌍이 있다. 왼쪽 그림에서 3개 삼각형면 ABC, ACD, BCD의 각을 합하면

¹p+ p+ ¹q + q + ¹s+ s+ r + t+ u = 3 ¢ 180±

Page 303: 실전수학올림피아드 1400제 해답

5.2 종합 고급문제 303

이 되는데, 여기서 r + t+ u = 0± 가 되어 모순이다. 오른쪽 그림에서는 4개면의 각을 모두 합하면

¹p+ p+ ¹q + q + r + r + s+ s+ ¹t+ t+ ¹u+ u = 4 ¢ 180±

이 되어야 하는데, r + s = 0± 가 되어 역시 모순이다. 따라서, (¤¤)의 가정이 성립하는 경우는 없고, 아래의 왼쪽 그림(전개도)처럼 마주보는 모든 각이 서로 같음을 알 수 있다.

그럼 각 면의 내각의 합에서

p+ s+ u = 180± (1)

q + s+ t = 180± (2)

r + t+ u = 180± (3)

p+ q + r = 180± (4)

(1) + (2) = (3) + (4) 로부터 r = s 임을 알 수 있다. 그리고 p = t, q = u 도 알 수 있다. 그럼 위의 오른쪽 그림에서 모든 면이 합동이고, 마주보는 변끼리 서로 길이가 같다는 것도 바로 확인된다. ¤

88. 다음 식이 1998개의 근호로 이루어져 있다고 할 때 가능한 모든 정수해를 구하여라.r(x+

q(x:::(x+

p(x)):::)) = y (소련 1964-6)

풀이 근호 안은 완전제곱수. 그 안의 근호도 완전제곱수. 계속하면 결국 x는 완전제곱수. x = k2.

x+px = k2 + k 도 완전제곱수. k2 + k = h2, k = (h+ k)(h¡ k) 에서 k > 0 이라면 h > k 이기도 하

고 우변이 더 커서 모순. }

89. 충분히 긴 고속도로에서 버스를 타고있는 한 승객이 차들을 관찰하고 있다. 한 시간 동안 이 버스는 일정한 속도로 달리며 a 의 차를 추월했고 b 의 차로부터 추월당했으며, 맞은편 차선에서는 c 의 차가 지나쳐갔다. 맞은편도 이쪽과 똑같은 정도로 혼잡하다고 가정할 때, 한 시간 동안 이 고속도로를 지나는 차가 몇 인지 알 수 있을까? (이 승객이 탄 버스를 제외한 다른 모든 차의 속도는 두 가지뿐이라고 가정해도 좋다.) (이탈리아 1986-7)

¢ ¢ ¢ 답 ¡a+b+c2

(+1)

90. 4ABC에서 변 BC, AB 위에 각각 점 D, E를 \CAD = \ADE = \B 가 되도록 잡는다. 4ABC,4EBD, 4ADC의 둘레의 길이를 각각 p, q, r이라 할 때, 4(q+ r) · 5p 임을 증명하여라. (1995 도대회)

풀이 \DAC = \ABC이고 \ACB가 공통이므로 4ABC와 4DAC는 닮음이다. 또 AC와 DE이 평

행이므로 4ABC와 4EBD도 닮음이다. 따라서

q =BD

BCp

r =AC

BCp

이다. 4(q + r)¡ 5p를 계산하면

4(q + r)¡ 5p =

·4BD +AC

BC¡ 5¸p

=

·4BC ¡ CD +AC

BC¡ 5¸p

=

·4AC ¡ CD

BC¡ 1¸p

Page 304: 실전수학올림피아드 1400제 해답

304 종합문제

이고 AC=BC = CD=AC이므로 CD = AC2=BC이다. 이를 입하면

4(q + r)¡ 5p =

·¡4AC

2

BC2+ 4

AC

BC¡ 1¸p

= ¡µ2AC

BC¡ 1¶2

p · 0

그러므로 4(q + r) · 5p이다. }

91. 피보나치 수열은 F0 = 0, F1 = 1, 그리고 n ¸ 0 에 해 Fn+2 = Fn +Fn+1 로 정의된다. 다음을 증명하여라.

(1) \Fn+k ¡ Fn 이 모든 자연수 n에 해 10으로 나누어떨어진다"는 문장은 k = 60 일 때는 참이지만

k < 60 이면 항상 거짓이다.

(2) \Fn+t ¡ Fn 이 모든 자연수 n에 해 100으로 나누어떨어진다"는 문장은 t = 300 일 때는 참이지만 t < 300 이면 항상 거짓이다. (아일랜드 1996-6)

증명 (1) mod 2, 5에 한 순환잉여수열. 좀 막노동. 15 j k 일 때를 특히 주목하면 조금 효율이 좋아

지긴 하지만. (2)는 아마 더 노동일 듯. ¤

92. 함수 f : N! N 이 다음을 만족한다.

(i) a와 b가 서로소이면 f(ab) = f(a)f(b).

(ii) p와 q가 소수이면 f(p+ q) = f(p) + f(q).

f(2) = 2, f(3) = 3, f(1999) = 1999 임을 증명하여라. (아일랜드 1999-7)

증명 f(n) = n 을 만족하는 n을 다음과 같이 차근차근 늘려가면 됨: 2, 3, 5, 7, 14, 15, 13, 11, 2002(2002 = 2 ¢ 7 ¢ 11 ¢ 13), 1999 ¤

93. 선분 AD 위의 점 B, C가 AB = CD 를 만족한다. 평면 위의 임의의 점 P에 해 PA+PD ¸ PB+PC

임을 증명하여라. (소련 1966-2a)

94. 임의의 실수 A에 해 A의 정수부(A를 넘지 않는 가장 큰 정수)를 I(A)라 하고 A의 소수부를 F (A), 즉F (A) = A¡ I(A) 라 하자.

(a) 다음을 만족하는 양수 A의 한 예를 찾아라:

F (A) + F (1=A) = 1

(b) 위에 해당하는 A가 유리수일 수 있는가? (Towns 1985봄 JA2)

풀이 (a) 곱이 1이 되는 켤레무리수 하나 잘 골라주면. (b) 유리수라 하면 A+ 1A 가 정수. pq j p2+ q2

임에서 뭐 충해보면 p = q = 1 로 모순. }

95. 정십각형이 주어져 있고 모든 각선(두 꼭지점을 연결하는 선분)이 그려져 있다. 각각의 꼭지점과 각각의 각선들의 교점(내부의 교점만을 고려한다)마다 \+1"을 붙였다. 이제 우리는 변 혹은 각선 중에서하나를 골라 그 위에 있는 수들의 부호를 모두 한꺼번에 바꿀 수 있다(+인 것은 ¡로, ¡인 것은 +로). 유한번의 작업 후에 모든 수의 부호가 ¡가 되는 것이 가능한가? (Towns 1984가을 SA3)

풀이 서로 만나는 세 각선을 생각할 때, 이 세 각선 중에 뒤집힌 횟수의 홀짝이 같은 두 각선이

있고, 그럼 그 두 각선의 교점은 부호가 +. 단, 이 세 각선의 교점은 서로 다르며 또한 다른 각선위에는 놓이지 않는 것으로 택해야 함. }

Page 305: 실전수학올림피아드 1400제 해답

5.2 종합 고급문제 305

96. a, b, c는 어떤 삼각형의 세 변의 길이이다. a+ b+ c = 1 일 때, 다음을 증명하여라.

a2 + b2 + c2 + 4abc · 1

2(이탈리아 1990-4)

증명 동차식 (a+b+c)(a2+b2+c2)+4abc · 12(a+b+c)3 으로 만들어 전개-정리하면 a3+b3+c3+

2abc · ab2+ac2+ba2+bc2+ca2+cb2. 이것은 (a+b¡c)(a¡b+c)(¡a+b+c) > 0 를 전개하면 바로나옴. 혹은 a; b; c < 1

2 이므로 (1¡ 2a)(1¡ 2b)(1¡ 2c) > 0 을 전개하여 ¡2(ab+ bc+ ca) + 4abc < ¡ 12

로부터 얻을 수도 있음. a2 + b2 + c2 = (a+ b+ c)2 ¡ 2(ab+ bc+ ca) = 1¡ 2(ab+ bc+ ca) 니까. ¤

97. 같은 크기의 n ¸ 3 개의 둥근 원판들 B1; B2; : : : ; Bn이 평평한 탁자 위에 놓여있는데, 각각의 i =1; 2; : : : ; n 에 해 Bi는 Bi+1과 맞닿아있다(Bn+1 = B1). 또 하나의 같은 크기의 원판 B를 앞서

놓여진 원판 무리의 바깥쪽 경계선을 따라서 굴리자. B가 제자리로 돌아오기까지 몇 바퀴를 회전하겠는가? (독일BW 1972 1차-2)

풀이 n개의 원판들 중심을 이은 n각형을 생각하면 한 꼭지점 X를 중심으로 하는 원판 위를 도는 각

은 (360± ¡\B¡ 2 ¢ 60±). 따라서, B는 다른 원판 주위로 총 (360±n¡ (n각형의 내각의 합)¡ 120±n) =(240±n¡ 180±(n¡ 2)) = 360± + 60±n 를 돌게 됨. 즉, 1 + n

6 바퀴를 돌게 됨.

근데 여기까지만 생각하면 절반밖에 못 푼거고. 왜냐하면 이건 상 편 원의 둘레 위를 얼마만큼의 거리를 진행했느냐 하는 것만 계산한 거니까. 이 로는 상 편 원의 중심에서 관찰했을 때 몇 바퀴를 돌았느냐에 한 답일 뿐, 지면을 기준으로 몇 바퀴를 돌았는지를 계산한 것은 아님.B가 자기와 같은 크기의 원판 주위로 k바퀴를 돌면 B가 그만큼의 원의 둘레 위를 진행해갔고 또한 두 원이 만나는 접점도 같은 거리만큼 옮겨갔으니까 B는 두 배의 속도로 회전하게 됨. 그래서 정답은 2+ n

3바

퀴. }

98. 경계를 포함하지 않는 6개의 원판이 한 평면 위에 있다. 어느 원판의 중심도 다른 원판의 내부에 놓여있지 않다. 6개의 원판 모두의 내부에 포함되는 점이 존재하지 않음을 증명하여라. (스웨덴 1970-2)

증명 귀류법으로 그런 점 P가 존재한다고 가정. \OiPOi+1 · 60± 인 두 중심 Oi, Oi+1이 존재.

OiOi+1 · max(POi; POi+1) 임을 확인할 수 있으므로 끝. ¤

99. 각 칸을 흑백으로 번갈아 칠한 1998£ 1998 크기의 체스판이 있다.

(a) 몇 개의 칸을 골라 동전을 하나씩 놓아서, 모든 행과 열마다 정확히 42개씩의 동전이 있도록 할 수있음을 보여라.

(b) 위와 같이 동전을 놓았다면, 검은칸에 놓은 동전은 짝수개임을 보여라. (몰도바 1998-y11-4)

증명 (a) 그냥 42개의 확장 각선들에 놓으면 됨. [(a)별해] 조건에 맞는 42£ 42 와 43£ 43 체스판을

만들고, 이것을 1998 £ 1998 의 각선에 적당히 나열하면 됨(1998 = 23 ¢ 42 + 24 ¢ 43). (b) 다음과 같이 4색으로 색칠하면 각 색의 칸에 놓인 동전의 수를 모아 생각했을 때 A+B = 999 ¢ 42 = A+C 이므로 B = C 이고 A = D. 각 줄마다 '짝수'개씩의 동전이 있는 것으로 해도 마찬가지. ¤

100. 넓이가 1인 볼록사각형이 있다. 이 사각형의 내부나 둘레 위에서 4개의 점을 택하는데, 어떤 세 점을 택해도 그 세 점이 이루는 삼각형의 넓이가 1=4보다 크거나 같도록 할 수 있음을 증명하여라.

(이탈리아 1992-2)

증명 원래의 네 점이 그런 네 점이면 그 네 점을 택하면 됨. 그렇지 않다면, WLOG j4ABCj < 14 이

라면, j4ACDj > 34 이므로 ACD의 넓이를 3등분하는 점(무게중심)과 함께 A, C, D를 택하면 됨. ¤